455
[Fact Pattern #1] The management and employees of a large household goods moving company decided to adopt total quality management (TQM) and continuous improvement (CI). They believed that, if their company became nationally known as adhering to TQM and CI, one result would be an increase in the company’s profits and market share. [1] Gleim #: 1.1 -- Source: CIA 1195 III-12 (Refers to Fact Pattern #1) The primary reason for adopting TQM was to achieve Greater customer satisfaction. A. Reduced delivery time. B. Reduced delivery charges. C. Greater employee participation. D. Answer (A) is correct. TQM is an integrated system that anticipates, meets, and exceeds customers’ needs, wants, and expectations. Answer (B) is incorrect because reduced delivery time is one of many potential activities that need improvement. Answer (C) is incorrect because reduced delivery charges is one of many potential activities that need improvement. Answer (D) is incorrect because increased employee participation is necessary to achieve TQM, but it is not the primary purpose for establishing the program. [2] Gleim #: 1.2 -- Source: CIA 1195 III-13 (Refers to Fact Pattern #1) Quality is achieved more economically if the company focuses on Appraisal costs. A. Prevention costs. B. Internal failure costs. C. External failure costs. D. Answer (A) is incorrect because prevention is ordinarily less costly than the combined costs of appraisal, internal failure, and external failure. Answer (B) is correct. Prevention attempts to avoid defective output. Prevention costs include preventive maintenance, employee training, review of equipment design, and evaluation of suppliers. Prevention is less costly than detection and correction of defective output. Answer (C) is incorrect because prevention is ordinarily less costly than the combined costs of appraisal, internal failure, and external failure. Answer (D) is incorrect because prevention is ordinarily less costly than the combined costs of appraisal, internal failure, and external failure. [3] Gleim #: 1.3 -- Source: CIA 1195 III-28 A traditional quality control process in manufacturing consists of mass inspection of goods only at the end of a production process. A major deficiency of the traditional control process is that It is expensive to do the inspections at the end of the process. A. It is not possible to rework defective items. B. It is not 100% effective. C. It does not focus on improving the entire production process. D. Answer (A) is incorrect because other quality control processes can also be expensive. Answer (B) is incorrect because reworking defective items may be possible although costly. Answer (C) is incorrect because no quality control system will be 100% effective. Answer (D) is correct. The process used to produce the goods is not thoroughly reviewed and evaluated for efficiency and effectiveness. Preventing defects and increasing efficiency by improving the production process raises quality standards and decreases costs. [4] Gleim #: 1.4 -- Source: CIA 1196 III-24 Under a total quality management (TQM) approach Gleim's CIA Test Prep: Part III: Business Analysis and Information Technology (1165 questions) Copyright 2004 Gleim Publications, Inc. Page 1 Printed for g j

Part three cia_with_ answers

  • Upload
    deloitte

  • View
    4.897

  • Download
    21

Embed Size (px)

DESCRIPTION

 

Citation preview

Page 1: Part three cia_with_ answers

[Fact Pattern #1]The management and employees of a large household goods moving company decided to adopt total quality management (TQM)and continuous improvement (CI). They believed that, if their company became nationally known as adhering to TQM and CI,one result would be an increase in the company’s profits and market share.

[1] Gleim #: 1.1 -- Source: CIA 1195 III-12

(Refers to Fact Pattern #1)The primary reason for adopting TQM was to achieve

Greater customer satisfaction.A.Reduced delivery time.B.Reduced delivery charges.C.Greater employee participation.D.

Answer (A) is correct. TQM is an integrated system that anticipates, meets, and exceeds customers’ needs, wants,and expectations.Answer (B) is incorrect because reduced delivery time is one of many potential activities that need improvement.Answer (C) is incorrect because reduced delivery charges is one of many potential activities that need improvement.Answer (D) is incorrect because increased employee participation is necessary to achieve TQM, but it is not theprimary purpose for establishing the program.

[2] Gleim #: 1.2 -- Source: CIA 1195 III-13

(Refers to Fact Pattern #1)Quality is achieved more economically if the company focuses on

Appraisal costs.A.Prevention costs.B.Internal failure costs.C.External failure costs.D.

Answer (A) is incorrect because prevention is ordinarily less costly than the combined costs of appraisal, internalfailure, and external failure.Answer (B) is correct. Prevention attempts to avoid defective output. Prevention costs include preventivemaintenance, employee training, review of equipment design, and evaluation of suppliers. Prevention is less costlythan detection and correction of defective output.Answer (C) is incorrect because prevention is ordinarily less costly than the combined costs of appraisal, internalfailure, and external failure.Answer (D) is incorrect because prevention is ordinarily less costly than the combined costs of appraisal, internalfailure, and external failure.

[3] Gleim #: 1.3 -- Source: CIA 1195 III-28

A traditional quality control process in manufacturing consists of mass inspection of goods only at the end of a productionprocess. A major deficiency of the traditional control process is that

It is expensive to do the inspections at the end of the process.A.It is not possible to rework defective items.B.It is not 100% effective.C.It does not focus on improving the entire production process.D.

Answer (A) is incorrect because other quality control processes can also be expensive.Answer (B) is incorrect because reworking defective items may be possible although costly.Answer (C) is incorrect because no quality control system will be 100% effective.Answer (D) is correct. The process used to produce the goods is not thoroughly reviewed and evaluated forefficiency and effectiveness. Preventing defects and increasing efficiency by improving the production process raisesquality standards and decreases costs.

[4] Gleim #: 1.4 -- Source: CIA 1196 III-24

Under a total quality management (TQM) approach

Gleim's CIA Test Prep: Part III: Business Analysis and Information Technology(1165 questions)

Copyright 2004 Gleim Publications, Inc. Page 1Printed for g j

Page 2: Part three cia_with_ answers

Measurement occurs throughout the process, and errors are caught and corrected at the source.A.Quality control is performed by highly trained inspectors at the end of the production process.B.Upper management assumes the primary responsibility for the quality of the products and services.C.A large number of suppliers are used in order to obtain the lowest possible prices.D.

Answer (A) is correct. Total quality management emphasizes quality as a basic organizational function. TQM is thecontinuous pursuit of quality in every aspect of organizational activities. One of the basic tenets of TQM is doing it right thefirst time. Thus, errors should be caught and corrected at the source.Answer (B) is incorrect because total quality management emphasizes discovering errors throughout the process, notinspection of finished goods.Answer (C) is incorrect because all members of the organization assume responsibility for quality of the products andservices.Answer (D) is incorrect because the total quality management philosophy recommends limiting the number of suppliers tocreate a strong relationship.

[5] Gleim #: 1.5 -- Source: CIA 596 III-29

Which of the following is a characteristic of total quality management (TQM)?

Management by objectives.A.On-the-job training by other workers.B.Quality by final inspection.C.Education and self-improvement.D.

Answer (A) is incorrect because one of the 14 points recommends elimination of numerical quotas. MBO causesaggressive pursuit of numerical quotas.Answer (B) is incorrect because informal learning from coworkers serves to entrench bad work habits. One of the 14points stresses proper training of everyone.Answer (C) is incorrect because another of the 14 points states that quality by final inspection is unnecessary ifquality is built in from the start.Answer (D) is correct. According to management theorist W. Edwards Deming’s well-known 14 points, educationand self-improvement are essential. Knowledge is opportunity. Hence, continuous improvement should be everyone’sprimary career objective.

[6] Gleim #: 1.6 -- Source: CIA 596 III-30

In which of the following organizational structures does total quality management (TQM) work best?

Hierarchal.A.Teams of people from the same specialty.B.Teams of people from different specialties.C.Specialists working individually.D.

Answer (A) is incorrect because hierarchal organization stifles TQM.Answer (B) is incorrect because TQM works best with teams of people from different specialties.Answer (C) is correct. TQM advocates replacement of the traditional hierarchal structure with teams of people fromdifferent specialities. This change follows from TQM’s emphasis on empowering employees and teamwork.Employees should have proper training, necessary information, and the best tools; be fully engaged in the decisionprocess; and receive fair compensation. If such empowered employees are assembled in teams of individuals with therequired skills, TQM theorists believe they will be more effective than people performing their tasks separately in arigid structure.Answer (D) is incorrect because teamwork is essential for TQM.

[7] Gleim #: 1.7 -- Source: CIA 1196 III-23

The most important component of quality control is

Ensuring goods and services conform to the design specifications.A.Satisfying upper management.B.Conforming with ISO-9000 specifications.C.Determining the appropriate timing of inspections.D.

Gleim's CIA Test Prep: Part III: Business Analysis and Information Technology(1165 questions)

Copyright 2004 Gleim Publications, Inc. Page 2Printed for g j

Page 3: Part three cia_with_ answers

Answer (A) is correct. The intent of quality control is to ensure that goods and services conform to the designspecifications. Whether the focus is on feedforward, feedback, or concurrent control, the emphasis is on ensuring product orservice conformity.Answer (B) is incorrect because quality control is geared towards satisfying the customer, not upper management.Answer (C) is incorrect because ensuring the conformance with ISO-9000 specifications is a component of a complianceaudit, not quality control.Answer (D) is incorrect because determining the appropriate timing of inspections is only one step towards approachingquality control. Consequently, it is not the primary component of the quality control function.

[8] Gleim #: 1.8 -- Source: CMA 1296 3-22

The cost of scrap, rework, and tooling changes in a product quality cost system is categorized as a(n)

Training cost.A.External failure cost.B.Internal failure cost.C.Prevention cost.D.

Answer (A) is incorrect because training costs are prevention costs.Answer (B) is incorrect because the costs of external failure, such as warranty expense, product liability, andcustomer ill will, arise when problems are discovered after products have been shipped.Answer (C) is correct. According to SMA 4-R, internal failure costs are incurred when detection of defectiveproducts occurs before shipment. Examples of internal failure costs are scrap, rework, tooling changes, anddowntime.Answer (D) is incorrect because prevention costs are incurred to avoid defective output. Examples include preventivemaintenance, employee training, review of equipment design, and evaluation of suppliers.

[9] Gleim #: 1.9 -- Source: CMA 1295 3-12

The four categories of costs associated with product quality costs are

External failure, internal failure, prevention, and carrying.A.External failure, internal failure, prevention, and appraisal.B.External failure, internal failure, training, and appraisal.C.Warranty, product liability, training, and appraisal.D.

Answer (A) is incorrect because carrying cost is not one of the elements of quality costs.Answer (B) is correct. SMA 4R lists four categories of quality costs: prevention, appraisal, internal failure, andexternal failure (lost opportunity). Costs of prevention include attempts to avoid defective output, including employeetraining, review of equipment design, preventive maintenance, and evaluation of suppliers. Appraisal costs includequality control programs, inspection, and testing. Internal failure costs are incurred when detection of defectiveproducts occurs before shipment, including scrap, rework, tooling changes, and downtime. External failure costs areincurred after the product has been shipped, including the costs associated with warranties, product liability, andcustomer ill will.Answer (C) is incorrect because training costs are not a category of quality costs.Answer (D) is incorrect because warranty, product liability, and training are not cost categories identified by SMA4R.

[10] Gleim #: 1.10 -- Source: CMA 1295 3-14

The cost of statistical quality control in a product quality cost system is categorized as a(n)

Internal failure cost.A.Training cost.B.External failure cost.C.Appraisal cost.D.

Gleim's CIA Test Prep: Part III: Business Analysis and Information Technology(1165 questions)

Copyright 2004 Gleim Publications, Inc. Page 3Printed for g j

Page 4: Part three cia_with_ answers

Answer (A) is incorrect because internal failure costs arise after poor quality has been found; statistical quality control isdesigned to detect quality problems.Answer (B) is incorrect because statistical quality control is not a training cost.Answer (C) is incorrect because external failure costs are incurred after the product has been shipped, including the costsassociated with warranties, product liability, and customer ill will.Answer (D) is correct. SMA 4R lists four categories of quality costs: prevention, appraisal, internal failure, and externalfailure (lost opportunity). Appraisal costs include quality control programs, inspection, and testing. However, someauthorities regard statistical quality and process control as preventive activities because they not only detect faulty work butalso allow for adjustment of processes to avoid future defects.

[11] Gleim #: 1.11 -- Source: CMA 697 3-28

Listed below are selected line items from the cost-of-quality report for Watson Products for last month.

Category AmountRework $    725Equipment maintenance 1,154Product testing 786Product repair 695

What is Watson’s total prevention and appraisal cost for last month?

$786A.$1,154B.$1,940C.$2,665D.

Answer (A) is incorrect because $786 is the appraisal cost.Answer (B) is incorrect because $1,154 is the prevention cost.Answer (C) is correct. According to SMA 4R, the costs of prevention and appraisal are conformance costs that serveas financial measures of internal performance. Prevention costs are incurred to prevent defective output. These costsinclude preventive maintenance, employee training, review of equipment design, and evaluation of suppliers.Appraisal costs are incurred to detect nonconforming output. They embrace such activities as statistical qualitycontrol programs, inspection, and testing. The equipment maintenance cost of $1,154 is a prevention cost. Theproduct testing cost of $786 is an appraisal cost. Their sum is $1,940.Answer (D) is incorrect because $2,665 includes rework, an internal failure cost.

[12] Gleim #: 1.12 -- Source: CMA 697 3-27

All of the following would generally be included in a cost-of-quality report except

Warranty claims.A.Design engineering.B.Supplier evaluations.C.Lost contribution margin.D.

Answer (A) is incorrect because the costs of warranty claims are readily measurable external failure costs captured bythe accounting system.Answer (B) is incorrect because the cost of design engineering is a prevention cost that is usually included in cost-of-quality reports.Answer (C) is incorrect because the cost of supplier evaluations is a prevention cost that is usually included in cost-of-quality reports.Answer (D) is correct. A cost-of-quality report includes most costs related to quality, including the costs of externalfailure, internal failure, prevention, and appraisal. Lost contribution margins from poor product quality are externalfailure costs that normally do not appear on a cost-of-quality report because they are opportunity costs. Opportunitycosts are not usually recorded by the accounting system, thereby understating the costs of poor quality. Lostcontribution margins from reduced sales, market share, and sales prices are external failure costs that are also notusually included in a cost-of-quality report.

[13] Gleim #: 1.13 -- Source: Publisher

Conformance is how well a product and its components meet applicable standards. According to the robust quality concept,

Gleim's CIA Test Prep: Part III: Business Analysis and Information Technology(1165 questions)

Copyright 2004 Gleim Publications, Inc. Page 4Printed for g j

Page 5: Part three cia_with_ answers

A certain percentage of defective units is acceptable.A.Units are acceptable if their characteristics lie within an acceptable range of values.B.The goal is for all units to be within specifications.C.Every unit should reach a target value.D.

Answer (A) is incorrect because the traditional view of quality treats a certain number of defective units as acceptable.Answer (B) is incorrect because the traditional view of quality deems that a unit is acceptable if it is within a range ofspecified values.Answer (C) is incorrect because the robust quality concept is an extension of the zero-defects approach. The goal of robustquality is in every case to reach a target value, not merely a range of acceptable values.Answer (D) is correct. Conformance is how well a product and its components meet applicable standards. The traditionalview is that conforming products are those with characteristics that lie within an acceptable specified range of values thatincludes a target value. This view also regarded a certain percentage of defective (nonconforming) units as acceptable. Thetraditional view was superseded by the zero-defects approach that sought to eliminate all nonconforming output. Anextension of this approach is the robust quality concept. Its goal is to reach the target value in every case. The reason is thathidden quality costs occur when output varies from the target even though the units are within specifications.

[14] Gleim #: 1.14 -- Source: Publisher

The Plan-Do-Check-Act (PDCA) Cycle is a quality tool devised by W.E. Deming. It is best described as

A “management by fact” approach to continuous improvement.A.An ongoing evaluation of the practices of best-in-class organizations.B.The translation of customer requirements into design requirements.C.The responsibility of every employee, work group, department, or supplier to inspect the work.D.

Answer (A) is correct. PDCA is a “management by fact” or scientific method approach to continuous improvement.PDCA creates a process-centered environment because it involves studying the current process, collecting andanalyzing data to identify causes of problems, planning for improvement, and deciding how to measure improvement(Plan). The plan is then implemented on a small scale if possible (Do). The next step is to determine what happened(Check). If the experiment was successful, the plan is fully implemented (Act). The cycle is then repeated using whatwas learned from the preceding cycle.Answer (B) is incorrect because competitive benchmarking is an ongoing evaluation of the practices of best-in-classorganizations.Answer (C) is incorrect because quality deployment is the translation of customer requirements into designrequirements.Answer (D) is incorrect because the “quality at the source” concept emphasizes the responsibility of every employee,work group, department, or supplier to inspect the work.

[15] Gleim #: 1.15 -- Source: Publisher

Which of the following quality costs are nonconformance costs?

Systems development costs.A.Costs of inspecting in-process items.B.Environmental costs.C.Costs of quality circles.D.

Answer (A) is incorrect because systems development costs are prevention (conformance) costs.Answer (B) is incorrect because costs of inspecting in-process items are appraisal (conformance) costs.Answer (C) is correct. Nonconformance costs include internal and external failure costs. External failure costsinclude environmental costs, e.g., fines for violations of environmental laws and loss of customer goodwill.Answer (D) is incorrect because costs of quality circles are prevention (conformance) costs.

[16] Gleim #: 1.16 -- Source: Publisher

Quality costing is similar in service and manufacturing organizations. Nevertheless, the differences between theseorganizations have certain implications for quality management. Thus,

Gleim's CIA Test Prep: Part III: Business Analysis and Information Technology(1165 questions)

Copyright 2004 Gleim Publications, Inc. Page 5Printed for g j

Page 6: Part three cia_with_ answers

Direct labor costs are usually a higher percentage of total costs in manufacturing organizations.A.External failure costs are relatively greater in service organizations.B.Quality improvements resulting in more efficient use of labor time are more likely to be accepted by employees in serviceorganizations.

C.

Poor service is less likely to result in loss of customers than a faulty product.D.

Answer (A) is incorrect because direct labor costs are usually a higher percentage of total costs in service organizations.Answer (B) is correct. External failure costs arise when problems occur after delivery. They occur because products orservices are nonconforming or otherwise do not satisfy customers. External failure costs in service enterprises are even moreimportant than in manufacturing environments. Faulty goods sometimes may be reworked or replaced to a customer’ssatisfaction, but poor service tends to result in a loss of customers.Answer (C) is incorrect because service activities are usually more labor intensive than in modern manufacturingenvironments. Thus, more efficient labor usage is more likely to be viewed as a threat to employee job security in serviceorganizations.Answer (D) is incorrect because the badwill resulting from poor service may be even more likely than a defective product toresult in loss of customers.

[17] Gleim #: 1.17 -- Source: Publisher

According to the robust quality concept,

The minimum point on the total quality cost curve occurs when conformance cost per unit equals nonconformancecost per unit.

A.

Improving quality requires tradeoffs among categories of quality costs.B.Beyond some point, incurrence of prevention and appraisal costs is not cost beneficial.C.Costs in all categories of quality costs may be reduced while improving quality.D.

Answer (A) is incorrect because, according to the robust quality concept, conformance costs do not necessarily haveto continue to increase to obtain additional reductions in nonconformance costs.Answer (B) is incorrect because, according to the robust quality concept, conformance costs do not necessarily haveto continue to increase to obtain additional reductions in nonconformance costs.Answer (C) is incorrect because, according to the robust quality concept, conformance costs do not necessarily haveto continue to increase to obtain additional reductions in nonconformance costs.Answer (D) is correct. The optimal level of quality costs traditionally has been deemed to occur where theconformance cost curve intercepts the nonconformance cost curve, which corresponds to the minimum point on thetotal cost curve. Thus, beyond some point, incurrence of prevention and appraisal costs is not cost beneficial.However, the modern robust quality view is that this relationship does not always hold. Improving quality andreducing costs in each category may be possible if the most efficient prevention methods are applied. For example,selection of a supplier meeting high quality standards regarding defect rates and delivery times may drive down notonly failure costs but also the prevention and appraisal costs incurred when supplier performance was less reliable.

[18] Gleim #: 1.18 -- Source: CMA 693 4-28

When evaluating projects, breakeven time is best described as

Annual fixed costs ÷ monthly contribution margin.A.Project investment ÷ annual net cash inflows.B.The point at which cumulative cash inflows on a project equal total cash outflows.C.The point at which discounted cumulative cash inflows on a project equal discounted total cash outflows.D.

Answer (A) is incorrect because it is related to breakeven point, not breakeven time.Answer (B) is incorrect because the payback period equals investment divided by annual undiscounted net cashinflows.Answer (C) is incorrect because the payback period is the period required for total undiscounted cash inflows toequal total undiscounted cash outflows.Answer (D) is correct. Breakeven time evaluates the rapidity of new product development. The usual calculationdetermines the period beginning with project approval that is required for the discounted cumulative cash inflows toequal the discounted cumulative cash outflows. However, it may also be calculated as the point at which discountedcumulative cash inflows on a project equal discounted total cash outflows. The concept is similar to the paybackperiod, but it is more sophisticated because it incorporates the time value of money. It also differs from the paybackmethod because the period covered begins at the outset of a project, not when the initial cash outflow occurs.

Gleim's CIA Test Prep: Part III: Business Analysis and Information Technology(1165 questions)

Copyright 2004 Gleim Publications, Inc. Page 6Printed for g j

Page 7: Part three cia_with_ answers

[19] Gleim #: 1.19 -- Source: CIA 594 III-56Management of a company is attempting to build a reputation as a world-class manufacturer of quality products. Which ofthe following measures would not be used by the firm to measure quality?

The percentage of shipments returned by customers because of poor quality.A.The number of parts shipped per day.B.The number of defective parts per million.C.The percentage of products passing quality tests the first time.D.

Answer (A) is incorrect because the percentage of shipments returned measures quality by the number of defectiveunits.Answer (B) is correct. The number of parts shipped per day would most likely be used as a measure of theeffectiveness and efficiency of shipping procedures, not the quality of the product. This measure does not considerhow many of the parts are defective.Answer (C) is incorrect because the number of defective parts per million measures quality by the number ofdefective units.Answer (D) is incorrect because the percentage of products passing quality tests the first time measures quality by thenumber of nondefective products.

[20] Gleim #: 1.20 -- Source: CMA 694 3-16

In year 2, a manufacturing company instituted a total quality management (TQM) program producing the following report:

Summary Cost of Quality Report (000s)

  Year 1 Year 2 % Change Prevention costs $   200 $   300 +50Appraisal costs 210 315 +50Internal failure costs 190 114 –40External failure costs 1,200 621 –48 

Total quality costs $1,800 $1,350 –25 

On the basis of this report, which one of the following statements is most likely true?

An increase in conformance costs resulted in a higher quality product and therefore resulted in a decrease innonconformance costs.

A.

An increase in inspection costs was solely responsible for the decrease in quality costs.B.Quality costs such as scrap and rework decreased by 48%.C.Quality costs such as returns and repairs under warranty decreased by 40%.D.

Answer (A) is correct. TQM emphasizes the supplier’s relationship with the customer, and recognizes that everyonein a process is at some time a customer or supplier of someone else, either within or outside the organization. Thecosts of quality include costs of conformance and costs of nonconformance. Costs of conformance include preventioncosts and appraisal (inspection) costs. Nonconformance costs are composed of internal failure costs and externalfailure costs, such as lost opportunity. Conformance costs (prevention and appraisal) increased substantially, whereasthe nonconformance costs (internal and external failure) decreased. Hence, the increase in conformance costs resultedin a higher quality product.Answer (B) is incorrect because prevention costs also increased substantially, which could also have led to higherquality products.Answer (C) is incorrect because scrap and rework are internal failure costs, which decreased by 40%.Answer (D) is incorrect because returns and repairs are external failure costs, which decreased by 48%.

Gleim's CIA Test Prep: Part III: Business Analysis and Information Technology(1165 questions)

Copyright 2004 Gleim Publications, Inc. Page 7Printed for g j

Page 8: Part three cia_with_ answers

[21] Gleim #: 1.21 -- Source: CIA 1195 III-98Quality cost indices are often used to measure and analyze the cost of maintaining a given level of quality. One example ofa quality cost index, which uses a direct labor base, is computed as

The following quality cost data were collected for May and June:

  May June Prevention costs $ 4,000 $    5,000Appraisal costs 6,000 5,000Internal failure costs 12,000 15,000External failure costs 14,000 11,000Direct labor costs 90,000 100,000

Based upon these cost data, the quality cost index

Decreased 4 points from May to June.A.Was unchanged from May to June.B.Increased 10 points from May to June.C.Decreased 10 points from May to June.D.

Answer (A) is correct. The index for May was 40% [($4,000 + $6,000 + $12,000 + $14,000) ÷ $90,000], and theindex for June was 36% [($5,000 + $5,000 + $15,000 + $11,000) ÷ $100,000].Answer (B) is incorrect because the index decreased.Answer (C) is incorrect because the increase in prevention costs was 10% of the increase in labor costs.Answer (D) is incorrect because the decrease in appraisal costs was 10% of the increase in labor costs.

[22] Gleim #: 1.22 -- Source: Publisher

Tocon Company produces two components:  A-1 and A-2. The unit throughput contribution margins for A-1 and A-2 are$150 and $300, respectively. Each component must proceed through two processes: Operation 1 and Operation 2. Thecapacity of Operation 1 is 180 machine hours, with A-1 and A-2 requiring 1 hour and 3 hours, respectively. Furthermore,Tocon can sell only 45 units of A-1 and 100 units of A-2. However, Tocon is considering expanding Operation 1’s capacityby 90 machine hours at a cost of $80 per hour. Assuming that Operation 2 has sufficient capacity to handle any additionaloutput from Operation 1, Tocon should produce

  Units of A-1 Units of A-2 A. 180 0B. 45 100C. 45 75D. 0 60

Answer (A) is incorrect because Tocon can sell only 45 units of A-1.Answer (B) is incorrect because Tocon can produce only 75 units of A-2 if it produces 45 units of the more profitableA-1.Answer (C) is correct. A-1’s throughput contribution margin per unit of the scarce resource (the internal bindingconstraint) is $150 ($150 UCM ÷ 1 machining hour). A-2’s throughput contribution margin per unit of the scarceresource is $100 ($300 UCM ÷ 3 machine hours). Consequently, Tocon should produce as much A-1 as it can sell(45 units). If Tocon adds 90 machine hours to increase the capacity of Operation 1 to 270 hours (180 + 90), it cannotproduce additional units of A-1 because the external binding constraint has not been relaxed. However, it canproduce additional units of A-2. Given that the UCM per machine hour of A-2 is $100 and that the cost is $80 perhour, adding capacity to Operation 1 is profitable. Thus, Tocon should use 45 machine hours to produce 45 units ofA-1. The remaining 225 machine hours (270 - 45) should be used to produce 75 units (225 ÷ 3 hours) of A-2. Thelatter amount is within the external binding constraint.Answer (D) is incorrect because Tocon should produce as much of A-1 as it can sell.

Gleim's CIA Test Prep: Part III: Business Analysis and Information Technology(1165 questions)

Copyright 2004 Gleim Publications, Inc. Page 8Printed for g j

Page 9: Part three cia_with_ answers

[Fact Pattern #2]

Wolk Corporation is a highly automated manufacturingfirm. The vice president of finance has decided thattraditional standards are inappropriate for performancemeasures in an automated environment. Labor isinsignificant in terms of the total cost of production andtends to be fixed, material quality is considered moreimportant than minimizing material cost, and customersatisfaction is the number one priority. As a result, deliveryperformance measures have been chosen to evaluateperformance.

The following information is considered typical of the timeinvolved to complete orders:

• Wait time:- From order being placed to

start of production 10.0 days- From start of production to

completion 5.0 days• Inspection time 1.5 days• Process time 3.0 days• Move time 2.5 days

[23] Gleim #: 1.23 -- Source: CMA 693 3-17

(Refers to Fact Pattern #2)What is the manufacturing cycle efficiency for this order?

25.0%.A.13.6%.B.37.5%.C.69.2%.D.

Answer (A) is correct. Manufacturing cycle efficiency is defined as the quotient of the time required for value-addedproduction divided by total manufacturing lead time. For this order, the total lead time is 12 days (5.0 + 1.5 + 3.0 +2.5), and the manufacturing cycle efficiency is 25% (3 days of processing ÷ 12).Answer (B) is incorrect because 13.6% includes the 10 days prior to production in the denominator, a period notincluded in the calculation of manufacturing cycle efficiency.Answer (C) is incorrect because inspection time and move time should be included in the denominator.Answer (D) is incorrect because the calculation involves dividing the 3 days of processing time by the total of 12days to complete production.

[24] Gleim #: 1.24 -- Source: CMA 693 3-18

(Refers to Fact Pattern #2)What is the delivery cycle time for this order?

7 days.A.12 days.B.15 days.C.22 days.D.

Answer (A) is incorrect because 7 days excludes the wait time.Answer (B) is incorrect because 12 days ignores the 10 days of the waiting period prior to the start of production.Answer (C) is incorrect because 15 days incorporates the wait time but not the production periods.Answer (D) is correct. The delivery cycle time is defined as the entire time from receipt of the order until delivery ofthe order. This period equals 22 days (10.0 + 5.0 + 1.5 + 3.0 + 2.5)

Gleim's CIA Test Prep: Part III: Business Analysis and Information Technology(1165 questions)

Copyright 2004 Gleim Publications, Inc. Page 9Printed for g j

Page 10: Part three cia_with_ answers

[Fact Pattern #3]This information is relevant to a theory of constraints (TOC) analysis. A manufacturer that can sell all of its output produces itssole product using three operations. Each unit sells for $120, and direct materials costing $48 per unit are added at the start of thefirst operation. Other variable costs are immaterial. The following annual cost and capacity information is available concerningthose operations:

  Operation 1 Operation 2 Operation 3 Total capacity per year 200,000 units 150,000 units 180,000 unitsTotal output per year 150,000 units 150,000 units 150,000 unitsFixed cost of operations (not including

direct materials) $1,200,000 $1,800,000 $2,250,000

[25] Gleim #: 1.25 -- Source: Publisher

(Refers to Fact Pattern #3)Assume that additional workers are hired for the bottleneck operation to expedite setups and materials handling. The costof the additional workers is $50,000 per year. As a result, the annual output of the bottleneck operation will increase by500 units. The change in operating income attributable to the increase in workers is

$50,000A.$36,000B.$(14,000)C.$(20,000)D.

Answer (A) is incorrect because $50,000 is the incremental cost.Answer (B) is incorrect because $36,000 is the incremental throughput contribution.Answer (C) is correct. Operation 2 is the bottleneck because it is functioning at its capacity. The incremental annualthroughput contribution (revenues – direct materials costs) from adding workers to Operation 2 is $36,000 [500 units× ($120 unit price – $48 DM per unit)]. Because the cost of the additional workers is $50,000, the change inoperating income is $(14,000).Answer (D) is incorrect because $(20,000) is based on the assumption that an additional $12 per unit of fixed costswill be applied.

[26] Gleim #: 1.26 -- Source: Publisher

(Refers to Fact Pattern #3)Assume that X Company offers to perform the Operation 2 function on 1,000 units at a unit price of $40, excluding directmaterials cost. Which of these mutually exclusive offers is acceptable?

X but not Y.A.Y but not X.B.X or Y.C.Neither offer should be accepted.D.

Answer (A) is correct. X’s offer should be accepted because its cost is $40,000 (1,000 units × $40), but the increasein throughput contribution is $72,000 [1,000 units × ($120 unit price – $48 DM per unit)]. X’s offer effectivelyincreases the capacity of the bottleneck operation. Y’s offer should be rejected because it will result in the incurrenceof additional costs with no increase in throughput contribution, given that Operation 2 is already producing at its150,000-unit capacity.Answer (B) is incorrect because X's offer but not Y's offer is acceptable.Answer (C) is incorrect because X's offer but not Y's offer is acceptable.Answer (D) is incorrect because X's offer but not Y's offer is acceptable.

[27] Gleim #: 1.27 -- Source: Publisher

Which of the following statements regarding benchmarking is false?

Gleim's CIA Test Prep: Part III: Business Analysis and Information Technology(1165 questions)

Copyright 2004 Gleim Publications, Inc. Page 10Printed for g j

Page 11: Part three cia_with_ answers

Benchmarking involves continuously evaluating the practices of best-in-class organization and adapting company processesto incorporate the best of these practices.

A.

Benchmarking, in practice, usually involves a company’s formation of benchmarking teams.B.Benchmarking is an ongoing process that entails quantitative and qualitative measurement of the difference between thecompany’s performance of an activity and the performance by the best in the world or the best in the industry.

C.

The benchmarking organization against which a firm is comparing itself must be a direct competitor.D.

Answer (A) is incorrect because they are true statements about benchmarking.Answer (B) is incorrect because they are true statements about benchmarking.Answer (C) is incorrect because they are true statements about benchmarking.Answer (D) is correct. Benchmarking is an ongoing process that entails quantitative and qualitative measurement of thedifference between the company’s performance of an activity and the performance by a best-in-class organization. Thebenchmarking organization against which a firm is comparing itself need not be a direct competitor. The importantconsideration is that the benchmarking organization be an outstanding performer in its industry.

[28] Gleim #: 1.28 -- Source: CIA 595 III-22

An example of an internal nonfinancial benchmark is

The labor rate of comparably skilled employees at a major competitor’s plant.A.The average actual cost per pound of a specific product at the company’s most efficient plant.B.A $50,000 limit on the cost of employee training programs at each of the company’s plants.C.The percentage of customer orders delivered on time at the company’s most efficient plant.D.

Answer (A) is incorrect because the labor rate of a competitor is a financial benchmark.Answer (B) is incorrect because the cost per pound of a product at the company’s most efficient plant is a financialbenchmark.Answer (C) is incorrect because the cost of a training program is a financial benchmark.Answer (D) is correct. Benchmarking is a continuous evaluation of the practices of the best organizations in theirclass and the adaptation of processes to reflect the best of these practices. It entails analysis and measurement of keyoutputs against those of the best organizations. This procedure also involves identifying the underlying key actionsand causes that contribute to the performance difference. The percentage of orders delivered on time at the company’smost efficient plant is an example of an internal nonfinancial benchmark.

[29] Gleim #: 1.29 -- Source: Publisher

According to ISO 9000 standards, which of the following is false in regard to registrars?

Most employees are subject to being audited.A.Companies may have preliminary audits by registrars.B.Employees must be able to competently describe their jobs and demonstrate that they are performing properly.C.Upon satisfactory completion of an on-site visit, a registrar may issue a certificate describing the scope of theregistration, which is valid for 3 years.

D.

Answer (A) is correct. During an on-site visit, the registrar has the right to audit all employees if he or she decides todo so. Each employee must have the ability to explain their jobs and show that they are capable of performingproperly.Answer (B) is incorrect because some companies have preliminary audits by registrars to prepare for the officialaudit.Answer (C) is incorrect because employees must be able to “say what they do” and demonstrate that they “do whatthey say.”Answer (D) is incorrect because a registrar who is convinced that a quality system conforms to the selected standardissues a certificate describing the scope of the registration. The registration is usually valid for a 3-year period.

[30] Gleim #: 1.30 -- Source: Publisher

Which of the following is not true about the advantages of adopting ISO 9000 standards?

Adoption of ISO 9000 standards may allow the company to sell products in foreign markets.A.ISO registration makes customers more comfortable with the supplier’s products and services.B.ISO 9000 allows companies to understand who internal customers and users are without sharing private information.C.ISO registration may help companies discover internal process and quality improvements.D.

Gleim's CIA Test Prep: Part III: Business Analysis and Information Technology(1165 questions)

Copyright 2004 Gleim Publications, Inc. Page 11Printed for g j

Page 12: Part three cia_with_ answers

Answer (A) is incorrect because many foreign countries are beginning to require adoption of ISO 9000 standards as aprerequisite for a company to sell products or services in that country.Answer (B) is incorrect because many companies view ISO registration as a key to remaining competitive. ISO registrationallows customers to be more comfortable with suppliers’ products and services.Answer (C) is correct. Market pressure is usually the main driving force for companies that adopt ISO 9000 standards.However, many of the companies that register uncover internal process and quality improvement as a result. ISO 9000forces companies to share information, which leads to a better understanding of who internal customers and users are.Answer (D) is incorrect because many companies that implement ISO 9000 standards uncover internal process and qualityimprovements.

[31] Gleim #: 1.31 -- Source: Publisher

Why have many European Union countries not adopted ISO 14000 standards?

Adhering to ISO 14000 standards will not reduce monitoring and inspection by regulatory agencies.A.Individual European Union countries’ standards are typically more strict than ISO 14000 standards.B.Regulators are permitted to use voluntary audits as a basis for punitive action.C.ISO 14000 standards will not make it easier to do business across borders.D.

Answer (A) is incorrect because adhering to ISO 14000 standards does not guarantee less monitoring or inspection byregulatory agencies.Answer (B) is correct. Many European countries already have environmental systems in place, and many individualcountries’ standards are typically more strict than the ISO 14000 standards.Answer (C) is incorrect because many countries in the European Union have adopted measures similar to the ones inthe US to prevent self-incrimination during voluntary ISO audits.Answer (D) is incorrect because ISO 14000 establishes internationally recognized standards that are intended todiminish trade barriers and make it easier to do business across borders.

[32] Gleim #: 1.32 -- Source: Publisher

Which of the following is not required when ISO 9000 standards are adopted?

Organization of a quality management system.A.Creation of an internal audit system.B.Consistent high quality products.C.On-site inspections by a registrar.D.

Answer (A) is incorrect because one of the most important steps to adhering to ISO 9000 standards is to organize aQMS. A QMS explains the company’s quality control and management’s commitment to quality.Answer (B) is incorrect because internal audits assure that the company is complying with the documented QMSprocedures and ISO 9000 standards.Answer (C) is correct. ISO 9000 is a set of generic standards for establishing and maintaining a quality system withina company. The standards provide no basis for judging the quality of the end product. The marketplace will make thisdetermination on its own. The objective of ISO 9000 standards is to ensure consistent quality, even if the quality ispoor.Answer (D) is incorrect because a registrar must ensure that the company’s quality control system conforms to theselected standard.

[33] Gleim #: 1.33 -- Source: CIA 593 III-64

What coefficient of correlation results from the following data?

X Y1 102 83 64 45 2

Gleim's CIA Test Prep: Part III: Business Analysis and Information Technology(1165 questions)

Copyright 2004 Gleim Publications, Inc. Page 12Printed for g j

Page 13: Part three cia_with_ answers

0A.–1B.+1C.Cannot be determined from the data given.D.

Answer (A) is incorrect because a perfect negative correlation exists.Answer (B) is correct. The coefficient of correlation (in standard notation, r) measures the strength of the linearrelationship. The magnitude of r is independent of the scales of measurement of X and Y. Its range is –1.0 to 1.0. A value of–1.0 indicates a perfectly inverse linear relationship between X and Y. A value of zero indicates no linear relationshipbetween X and Y. A value of +1.0 indicates a perfectly direct relationship between X and Y. As X increases by 1, Yconsistently decreases by 2. Hence, a perfectly inverse relationship exists, and r must be equal to –1.0.Answer (C) is incorrect because an inverse, not a direct, relationship exists.Answer (D) is incorrect because a linear relationship exists between X and Y.

[34] Gleim #: 1.34 -- Source: CIA 1194 II-46

In regression analysis, which of the following correlation coefficients represents the strongest relationship between theindependent and dependent variables?

1.03A.–.02B.–.89C..75D.

Answer (A) is incorrect because 1.03 is an impossible value.Answer (B) is incorrect because –.02 is a very weak correlation coefficient.Answer (C) is correct. Because the range of values is between –1.0 and 1.0, –.89 suggests a very strong inverserelationship between the independent and dependent variables. A value of –1.0 signifies a perfect inverserelationship, and a value of 1.0 signifies a perfect direct relationship.Answer (D) is incorrect because .75 is .25 from the maximum value, whereas –.89 is .11 from the minimum value.

[35] Gleim #: 1.35 -- Source: CIA 595 II-46

The internal auditor of a bank has developed a multiple regression model which has been used for a number of years toestimate the amount of interest income from commercial loans. During the current year, the auditor applies the model anddiscovers that the r2 value has decreased dramatically, but the model otherwise seems to be working okay. Which of thefollowing conclusions are justified by the change?

Changing to a cross-sectional regression analysis should cause r2 to increase.A.Regression analysis is no longer an appropriate technique to estimate interest income.B.Some new factors, not included in the model, are causing interest income to change.C.A linear regression analysis would increase the model’s reliability.D.

Answer (A) is incorrect because cross-sectional regression analysis is inappropriate. The auditor is trying to estimatechanges in a single account balance over time.Answer (B) is incorrect because regression analysis may still be the most appropriate methodology to estimateinterest income, but the auditor should first understand the factors that may be causing r2 to decrease. The reason maybe a systematic error in the account balance.Answer (C) is correct. The coefficient of determination (r2) is the amount of variation in the dependent variable(interest income) that is explained by the independent variables. In this case, less of the change in interest income isexplained by the model. Thus, some other factor must be causing interest income to change. This change merits auditinvestigation.Answer (D) is incorrect because linear regression models are simpler models, but the auditor should be searching fora systematic error in the account balance or applying a more complex model.

Gleim's CIA Test Prep: Part III: Business Analysis and Information Technology(1165 questions)

Copyright 2004 Gleim Publications, Inc. Page 13Printed for g j

Page 14: Part three cia_with_ answers

[Fact Pattern #4]

In preparing the annual profit plan for the coming year,Wilkens Company wants to determine the cost behaviorpattern of the maintenance costs. Wilkens has decided to uselinear regression by employing the equation y = a + bx formaintenance costs. The prior year’s data regardingmaintenance hours and costs, and the results of theregression analysis, are given below and in the oppositecolumn.

Average cost per hour $9.00a 684.65b 7.2884Standard error of a 49.515Standard error of b .12126Standard error of the estimate 34.469r2 .99724

Hours of MaintenanceActivity Costs

January 480 $  4,200February 320 3,000March 400 3,600April 300 2,820May 500 4,350June 310 2,960July 320 3,030August 520 4,470September 490 4,260October 470 4,050November 350 3,300December 340 3,160

Sum 4,800 $43,200

Average 400 $  3,600

[36] Gleim #: 1.36 -- Source: CMA 1290 4-27

(Refers to Fact Pattern #4)In the standard regression equation y a + bx, the letter b is best described as a(n)

Independent variable.A.Dependent variable.B.Constant coefficient.C.Variable coefficient.D.

Answer (A) is incorrect because x is the independent variable.Answer (B) is incorrect because the dependent variable is y.Answer (C) is incorrect because the constant coefficient is a.Answer (D) is correct. In the standard regression equation, b represents the variable coefficient. For example, in acost determination regression, y equals total costs, b is the variable cost per unit, x is the number of units produced,and a is fixed cost.

[37] Gleim #: 1.37 -- Source: CMA 1290 4-28

(Refers to Fact Pattern #4)The letter x in the standard regression equation is best described as a(n)

Independent variable.A.Dependent variable.B.Constant coefficient.C.Coefficient of determination.D.

Answer (A) is correct. The letter x in the standard regression equation is the independent variable. For example, in aregression to determine the total cost of production, x equals units produced.Answer (B) is incorrect because the dependent variable is y.Answer (C) is incorrect because the constant coefficient is a.Answer (D) is incorrect because r 2 is the coefficient of determination.

[38] Gleim #: 1.38 -- Source: CMA 1290 4-29

(Refers to Fact Pattern #4)Based upon the data derived from the regression analysis, 420 maintenance hours in a month would mean the maintenancecosts (rounded to the nearest dollar) would be budgeted at

Gleim's CIA Test Prep: Part III: Business Analysis and Information Technology(1165 questions)

Copyright 2004 Gleim Publications, Inc. Page 14Printed for g j

Page 15: Part three cia_with_ answers

$3,780A.$3,600B.$3,790C.$3,746D.

Answer (A) is incorrect because the budgeted maintenance costs are $3,746.Answer (B) is incorrect because the budgeted maintenance costs are $3,746.Answer (C) is incorrect because the budgeted maintenance costs are $3,746.Answer (D) is correct. Substituting the given data into the regression equation results in a budgeted cost of $3,746(rounded to the nearest dollar).

y = a + bxy = 684.65 + 7.2884(420)y = $3,746

[39] Gleim #: 1.39 -- Source: CMA 1290 4-30

(Refers to Fact Pattern #4)The percentage of the total variance that can be explained by the regression equation is

99.724%A.69.613%B.80.982%C.99.862%D.

Answer (A) is correct. The coefficient of determination (r²) measures the percentage of the total variance in cost thatcan be explained by the regression equation. If the coefficient of determination is .99724, 99.724% of the variance isexplained by the regression equation. Thus, the values in the regression equation explain virtually the entire amountof total cost.Answer (B) is incorrect because the percentage of the total variance explained by the regression equation is 99.724%,which corresponds to the coefficient of determination (r²), or .99724.Answer (C) is incorrect because the percentage of the total variance explained by the regression equation is 99.724%,which corresponds to the coefficient of determination (r²), or .99724.Answer (D) is incorrect because the percentage of the total variance explained by the regression equation is 99.724%,which corresponds to the coefficient of determination (r²), or .99724.

[40] Gleim #: 1.40 -- Source: CIA 1195 II-30

A division uses a regression in which monthly advertising expenditures are used to predict monthly product sales (both inmillions of dollars). The results show a regression coefficient for the independent variable equal to 0.8. This coefficientvalue indicates that

The average monthly advertising expenditure in the sample is $800,000.A.When monthly advertising is at its average level, product sales will be $800,000.B.On average, every additional dollar of advertising results in $.80 of additional sales.C.Advertising is not a good predictor of sales because the coefficient is so small.D.

Answer (A) is incorrect because a regression coefficient is unrelated to the means of the variables.Answer (B) is incorrect because to predict a specific value of sales, the value of the independent variable is multipliedby the coefficient. The product is then added to the y-intercept value.Answer (C) is correct. The regression coefficient represents the change in the dependent variable corresponding to aunit change in the independent variable. Thus, it is the slope of the regression line.Answer (D) is incorrect because the absolute size of the coefficient bears no necessary relationship to the importanceof the variable.

[41] Gleim #: 1.41 -- Source: CIA 595 II-47

An internal auditor for a large automotive parts retailer wishes to perform a risk analysis and wants to use an appropriatestatistical tool to help identify stores that would be out of line compared to the majority of stores. The most appropriatestatistical tool to use is

Gleim's CIA Test Prep: Part III: Business Analysis and Information Technology(1165 questions)

Copyright 2004 Gleim Publications, Inc. Page 15Printed for g j

Page 16: Part three cia_with_ answers

Linear time series analysis.A.Cross-sectional regression analysis.B.Cross tabulations with chi-square analysis of significance.C.Time series multiple regression analysis to identify changes in individual stores over time.D.

Answer (A) is incorrect because linear time series analysis is inapplicable. It is a simple model that compares data for anindividual store over time.Answer (B) is correct. Time series data pertain to a given entity over a number of prior time periods. Cross-sectional data,however, pertain to different entities for a given time period or at a given time. Thus, cross-sectional regression analysis isthe most appropriate statistical tool because it compares attributes of all stores’ operating statistics at one moment in time.Answer (C) is incorrect because cross tabulations have to be built on a model of expectations. Unless the model is built, theanalysis is not useful.Answer (D) is incorrect because the objective is to compare stores at one moment in time. Multiple regression time seriesanalysis compares the performance of an individual store over a period of time.

[42] Gleim #: 1.42 -- Source: Publisher

Violation of which assumption underlying regression analysis is prevalent in time series analysis?

Variance of error term is constant.A.Error terms are independent.B.Distribution of error terms is normal.C.Expected value of error term equals zero.D.

Answer (A) is incorrect because constant variance of the error term is usually met.Answer (B) is correct. Time series analysis is a regression model in which the independent variable is time. In timeseries analysis, the value of the next time period is frequently dependent on the value of the time period before that.Hence, the error terms are usually correlated or dependent on the prior period; i.e., they are characterized byautocorrelation (serial correlation).Answer (C) is incorrect because normal distribution of the error term is usually met.Answer (D) is incorrect because an expected value of the error term equal to zero is usually met.

[43] Gleim #: 1.43 -- Source: CIA 589 III-50

The moving-average method of forecasting

Is a cross-sectional forecasting method.A.Regresses the variable of interest on a related variable to develop a forecast.B.Derives final forecasts by adjusting the initial forecast based on the smoothing constant.C.Includes each new observation in the average as it becomes available and discards the oldest observation.D.

Answer (A) is incorrect because cross-sectional regression analysis examines relationships among large amounts ofdata (e.g., many or different production methods or locations) at a particular moment in time.Answer (B) is incorrect because regression analysis relates the forecast to changes in particular variables.Answer (C) is incorrect because, under exponential smoothing, each forecast equals the sum of the last observationtimes the smoothing constant, plus the last forecast times one minus the constant.Answer (D) is correct. The simple moving-average method is a smoothing technique that uses the experience of thepast N periods (through time period t) to forecast a value for the next period. Thus, the average includes each newobservation and discards the oldest observation. The forecast formula for the next period (for time period t+1) is thesum of the last N observations divided by N.

[44] Gleim #: 1.44 -- Source: CIA 594 II-38

As part of a risk analysis, an auditor wishes to forecast the percentage growth in next month’s sales for a particular plantusing the past 30 months’ sales results. Significant changes in the organization affecting sales volumes were made withinthe last 9 months. The most effective analysis technique to use would be

Unweighted moving average.A.Exponential smoothing.B.Queuing theory.C.Linear regression analysis.D.

Gleim's CIA Test Prep: Part III: Business Analysis and Information Technology(1165 questions)

Copyright 2004 Gleim Publications, Inc. Page 16Printed for g j

Page 17: Part three cia_with_ answers

Answer (A) is incorrect because an unweighted average will not give more importance to more recent data.Answer (B) is correct. Under exponential smoothing, each forecast equals the sum of the last observation times thesmoothing constant, plus the last forecast times one minus the constant. Thus, exponential means that greater weight isplaced on the most recent data, with the weights of all data falling off exponentially as the data age. This feature isimportant because of the organizational changes that affected sales volume.Answer (C) is incorrect because queuing theory is used to minimize the cost of waiting lines.Answer (D) is incorrect because linear regression analysis determines the equation for the relationship among variables. Itdoes not give more importance to more recent data.

[Fact Pattern #5]

Moss Point Manufacturing recently completed and sold anorder of 50 units that had costs as shown in the nextcolumn.

The company has now been requested to prepare a bid for150 units of the same product.

Direct materials $  1,500Direct labor (1,000 hours × $8.50) 8,500Variable overhead (1,000 hours × $4.00)* 4,000Fixed overhead ** 1,400

$15,400

* Applied on the basis of direct labor hours** Applied at the rate of 10% of variable cost

[45] Gleim #: 1.45 -- Source: CMA 1288 5-19

(Refers to Fact Pattern #5)If an 80% learning curve is applicable, Moss Point’s total cost on this order would be estimated at

$26,400A.$32,000B.$38,000C.$41,800D.

Answer (A) is correct. Assuming that the cumulative average-time model applies, an 80% learning curve means thatthe cumulative average time per unit (and labor cost, given a constant labor rate) declines by 20% when unit outputdoubles in the early stages of production. The first lot size was 50 units, which was produced at a total cost of$15,400 ($1,500 for materials and $13,900 for labor and overhead). Materials costs are strictly variable and shouldremain proportional to production. The labor ($8,500) and variable overhead ($4,000) costs (labor-related), however,will be affected by the learning curve. The average cost per lot for labor and variable overhead after 100 units havebeen produced should be 80% of the costs of the first lot of 50 units. Thus, the average labor and variable overheadcost per 50-unit lot will be $10,000 (80% × $12,500). If production doubles again (to a total production of 200 unitsor four lots of 50 each), the cumulative average cost for labor and variable overhead will be $8,000 per lot (80% ×$10,000). Given four lots of 50 each, at an average cost of $8,000 per lot, the total cost for labor and variableoverhead must be $32,000. Adding $6,000 for raw materials ($1,500 per 50-unit lot) gives a total variable cost of$38,000 for 200 units. Fixed overhead is 10% of total variable cost, so total cost is $41,800. The total cost for the last150 units is $26,400 ($41,800 – $15,400).Answer (B) is incorrect because $32,000 is the total cost for labor and variable overhead for 200 units.Answer (C) is incorrect because $38,000 is the total variable cost for 200 units.Answer (D) is incorrect because $41,800 is the total cost for 200 units.

[46] Gleim #: 1.46 -- Source: CMA 1288 5-20

(Refers to Fact Pattern #5)If Moss Point had experienced a 70% learning curve, the bid for the 150 units would

Show a 30% reduction in the total direct labor hours required with no learning curve.A.Include increased fixed overhead costs.B.Be 10% lower than the total bid at an 80% learning curve.C.Include 6.40 direct labor hours per unit at $8.50 per hour.D.

Gleim's CIA Test Prep: Part III: Business Analysis and Information Technology(1165 questions)

Copyright 2004 Gleim Publications, Inc. Page 17Printed for g j

Page 18: Part three cia_with_ answers

Answer (A) is incorrect because, with no learning curve effect, estimated total hours would be 4,000 instead of 1,960, achange of more than 50%.Answer (B) is incorrect because fixed costs applied per lot would decline because they are based on labor hours, which aredeclining.Answer (C) is incorrect because, with no learning curve effect, estimated total hours would be 4,000 instead of 1,960, achange of more than 50%.Answer (D) is correct. The sum of the direct labor hours for the initial lot of 50 units was 1,000. A second lot of 50 wouldreduce the cumulative hours per lot to 700 (70% × 1,000). A doubling to four lots would reduce the cumulative hours perlot to 490 (70% × 700). Thus, for an output of 200 units, the total hours worked would be 1,960 (4 × 490). Subtracting the1,000 hours required for the first 50 units from the 1,960-hour total gives 960 hours for the last 150 units. Dividing 960hours by 150 units produces a per-unit time of 6.4 hours.

[47] Gleim #: 1.47 -- Source: CMA 1293 4-24

The average labor cost per unit for the first batch produced by a new process is $120. The cumulative average labor costafter the second batch is $72 per product. Using a batch size of 100 and assuming the learning curve continues, the totallabor cost of four batches will be

$4,320A.$10,368B.$2,592C.$17,280D.

Answer (A) is incorrect because $4,320 equals the cost of the items in the fourth batch.Answer (B) is incorrect because $10,368 is based on the assumption that the cumulative average unit labor cost isreduced by the learning curve percentage with each batch, not each doubling of output.Answer (C) is incorrect because $2,592 represents the labor cost of 100 units at the unit rate expected after anotherdoubling of production to eight batches.Answer (D) is correct. The learning curve reflects the increased rate at which people perform tasks as they gainexperience. The time required to perform a given task becomes progressively shorter. Ordinarily, the curve isexpressed in a percentage of reduced time to complete a task for each doubling of cumulative production. Onecommon assumption in a learning curve model is that the cumulative average time (and labor cost) per unit is reducedby a certain percentage each time production doubles. Given a $120 cost per unit for the first 100 units and a $72 costper unit when cumulative production doubled to 200 units, the learning curve percentage must be 60% ($72 ÷ $120).If production is again doubled to 400 units (four batches), the average unit labor cost should be $43.20 (60% × $72).Hence, total labor cost for 400 units is estimated to be $17,280 (400 × $43.20).

[48] Gleim #: 1.48 -- Source: CMA 683 5-8

A company is simulating the actions of a government agency in which 50% of the time a recall of a product is required,40% of the time only notification of the buyer about a potential defect is required, and 10% of the time no action on its partis required. Random numbers of 1 to 100 are being used. An appropriate assignment of random numbers for the recallcategory would be

1-40A.40-90B.61-100C.11-60D.

Answer (A) is incorrect because 1-40 is an appropriate assignment of random numbers for the notification category.Answer (B) is incorrect because 40-90 includes 51 numbers.Answer (C) is incorrect because 61-100 is an appropriate assignment of random numbers for the notificationcategory.Answer (D) is correct. Given a 50% chance of a recall, 50 different numbers should be assigned to that alternative.This answer is the only alternative with 50 numbers (11-60).

Gleim's CIA Test Prep: Part III: Business Analysis and Information Technology(1165 questions)

Copyright 2004 Gleim Publications, Inc. Page 18Printed for g j

Page 19: Part three cia_with_ answers

[49] Gleim #: 1.49 -- Source: CMA 1294 4-29Quick Response Plumbing (QRP), a wholesale distributor, supplies plumbing contractors and retailers throughout theNortheast on a next-day delivery basis. QRP has a centrally located warehouse to accept receipts of plumbing supplies. Thewarehouse has a single dock to accept and unload railroad freight cars during the night. It takes 5 hours to unload eachfreight car. QRP’s prior records indicate that the number of freight cars that arrive in the course of a night range from zeroto five or more, with no indicated pattern of arrivals. If more than two freight cars arrive on the same night, some freightmust be held until the next day for unloading. QRP wants to estimate the wait time when more than two freight cars arrivein the same night. The appropriate technique to analyze the arrival of freight cars is

Integer programming.A.Linear programming.B.Monte Carlo simulation.C.Regression analysis.D.

Answer (A) is incorrect because integer programming is a variation of linear programming that concerns problems inwhich some variables are not continuous. Integer programming problems are also known as discrete models becausethe variables take on discrete, noncontinuous values.Answer (B) is incorrect because linear programming is a technique used to maximize a revenue or profit function, orminimize a cost function, subject to constraints such as limited resources or minimum (or maximum) levels ofproduction time.Answer (C) is correct. The Monte Carlo simulation method is often used to generate the individual values for arandom variable. The performance of a quantitative model under uncertainty may be investigated by randomlyselecting values for each variable in the model (based on the probability distribution of each variable) and thencalculating the value of the solution. If this process is performed many times, the distribution of results from themodel will be obtained.Answer (D) is incorrect because regression analysis is used to find an equation for the linear relationships amongvariables.

[50] Gleim #: 1.50 -- Source: CMA 690 5-21

Through the use of decision models, managers thoroughly analyze many alternatives and decide on the best alternative forthe company. Often the actual results achieved from a particular decision are not what was expected when the decision wasmade. In addition, an alternative that was not selected would have actually been the best decision for the company. Theappropriate technique to analyze the alternatives by using expected inputs and altering them before a decision is made is

Expected value analysis.A.Linear programming.B.Program Evaluation Review Technique (PERT).C.Sensitivity analysis.D.

Answer (A) is incorrect because expected value analysis is used to determine an anticipated return or cost based uponprobabilities of events and their related outcomes.Answer (B) is incorrect because linear programming optimizes a function given certain constraints.Answer (C) is incorrect because PERT is a network technique used to plan and control large projects.Answer (D) is correct. Sensitivity modeling can be used to determine the outcome of a variety of decisions. A trial-and-error method may be adopted, usually in a computer model, to calculate the sensitivity of the solution (variabilityof outcomes) to changes in a variable.

[51] Gleim #: 1.51 -- Source: CMA 1291 3-7

Which one of the following items would most likely not be incorporated into the calculation of a division’s investment basewhen using the residual income approach for performance measurement and evaluation?

Fixed assets employed in division operations.A.Land being held by the division as a site for a new plant.B.Division inventories when division management exercises control over the inventory levels.C.Division accounts payable when division management exercises control over the amount of short-term credit used.D.

Gleim's CIA Test Prep: Part III: Business Analysis and Information Technology(1165 questions)

Copyright 2004 Gleim Publications, Inc. Page 19Printed for g j

Page 20: Part three cia_with_ answers

Answer (A) is incorrect because fixed operating assets are controlled by the division manager and contribute to profits.Answer (B) is correct. An evaluation of an investment center is based upon the return on the investment base. These assetsinclude plant and equipment, inventories, and receivables. Most likely, however, an asset, such as land, that is being held bythe division as a site for a new plant would not be included in the investment base because it is not currently being used inoperations. Total assets in use rather than total assets available is preferable when the investment center has been forced tocarry idle assets.Answer (C) is incorrect because inventories are operating assets that contribute to profits and are controlled by the divisionmanager.Answer (D) is incorrect because the level of accounts payable is an operating decision that should be considered in theevaluation of the division manager.

[52] Gleim #: 1.52 -- Source: CMA 1296 3-27

Which denominator used in the return on investment (ROI) formula is criticized because it combines the effects ofoperating decisions made at one organizational level with financing decisions made at another organizational level?

Total assets employed.A.Equity.B.Working capital plus other assets.C.Total assets available.D.

Answer (A) is incorrect because it reflects an assumption that the subunit manager does not influence the resourcebase (denominator of the ROI calculation).Answer (B) is correct. ROI equals income divided by invested capital. The denominator may be defined in variousways, e.g., total assets available, assets employed, working capital plus other assets, and equity. If equity (total assets– total liabilities) is chosen, a portion of long-term liabilities must be allocated to the investment center to determinethe manager’s resource base. One problem with this definition of the resource base is that, although it has theadvantage of emphasizing return to owners, it reflects decisions at different levels of the entity: short-term liabilitiesincurred by the responsibility center (operating decisions) and long-term liabilities controlled at the corporate level(long-term financing decisions).Answer (C) is incorrect because working capital plus other assets reflects the assumption that the manager controlsshort-term credit. However, no corporate-level decision to allocate long-term liabilities to subunits is necessary.Answer (D) is incorrect because it reflects an assumption that the subunit manager does not influence the resourcebase (denominator of the ROI calculation).

[53] Gleim #: 1.53 -- Source: CMA 697 3-29

Listed below is selected financial information for the Western Division of the Hinzel Company for last year.

  AmountAccount (thousands)

Average working capital $   625General and administrative expenses 75Net sales 4,000Average plant and equipment 1,775Cost of goods sold 3,525

If Hinzel treats the Western Division as an investment center for performance measurement purposes, what is the before-taxreturn on investment (ROI) for last year?

34.78%A.22.54%B.19.79%C.16.67%D.

Gleim's CIA Test Prep: Part III: Business Analysis and Information Technology(1165 questions)

Copyright 2004 Gleim Publications, Inc. Page 20Printed for g j

Page 21: Part three cia_with_ answers

Answer (A) is incorrect because 34.78% results from subtracting working capital from plant and equipment in calculatingthe net investment.Answer (B) is incorrect because 22.54% fails to include average working capital in the total for the net investment.Answer (C) is incorrect because 19.79% results from not subtracting general and administrative expenses in the calculationof before-tax profit.Answer (D) is correct. An investment center is responsible for revenues, expenses, and invested capital. Given averageplant and equipment of $1,775 and average working capital of $625, the net investment is $2,400. Before-tax profit is $400($4,000 sales – $3,525 cost of goods sold – $75 general expenses). If before-tax ROI equals before-tax profit divided by netinvestment, the answer is 16.67% ($400 ÷ $2,400).

[54] Gleim #: 1.54 -- Source: CMA 697 3-24

Residual income is a better measure for performance evaluation of an investment center manager than return on investmentbecause

The problems associated with measuring the asset base are eliminated.A.Desirable investment decisions will not be neglected by high-return divisions.B.Only the gross book value of assets needs to be calculated.C.The arguments about the implicit cost of interest are eliminated.D.

Answer (A) is incorrect because this method uses the same asset base.Answer (B) is correct. Residual income is the excess of the amount of the ROI over a targeted amount equal to animputed interest charge on invested capital. The advantage of using residual income rather than percentage ROI isthat the former emphasizes maximizing a dollar amount instead of a percentage. Managers of divisions with a highROI are encouraged to accept projects with returns exceeding the cost of capital even if those projects reduce thedepartment’s ROI.Answer (C) is incorrect because this method uses the same asset base.Answer (D) is incorrect because use of the residual income method requires a knowledge of the cost of capital; thus,arguments about the implicit cost of interest may escalate with use of the residual income method.

[55] Gleim #: 1.55 -- Source: CMA 697 3-30

James Webb is the general manager of the Industrial Product Division, and his performance is measured using the residualincome method. Webb is reviewing the following forecasted information for his division for next year:

  AmountCategory (thousands)

Working capital $ 1,800Revenue 30,000Plant and equipment 17,200

If the imputed interest charge is 15% and Webb wants to achieve a residual income target of $2,000,000, what will costshave to be in order to achieve the target?

$9,000,000A.$10,800,000B.$25,150,000C.$25,690,000D.

Answer (A) is incorrect because this level of cost would result in a residual income greater than $2,000,000.Answer (B) is incorrect because this level of cost would result in a residual income greater than $2,000,000.Answer (C) is correct. Residual income is the excess of the amount of the ROI over a targeted amount equal to animputed interest charge on invested capital. If a manager has $19,000,000 of invested capital ($17,200,000 of plantand equipment + $1,800,000 of working capital), a 15% imputed interest charge equals $2,850,000. Adding$2,000,000 of residual income to the imputed interest results in a target profit of $4,850,000. This profit can beachieved if costs are $25,150,000 ($30,000,000 revenue – $4,850,000 profit).Answer (D) is incorrect because $25,690,000 results from subtracting working capital from plant and equipment indetermining invested capital.

Gleim's CIA Test Prep: Part III: Business Analysis and Information Technology(1165 questions)

Copyright 2004 Gleim Publications, Inc. Page 21Printed for g j

Page 22: Part three cia_with_ answers

[56] Gleim #: 1.56 -- Source: CMA 695 3-20REB Service Co. is a computer service center. For the month of May, REB had the following operating statistics:

Sales $450,000Operating income 25,000Net profit after taxes 8,000Total assets 500,000Shareholders’ equity 200,000Cost of capital 6%

Based on the above information, which one of the following statements is true?  REB has a

Return on investment of 4%.A.Residual income of $(5,000).B.Return on investment of 1.6%.C.Residual income of $(22,000).D.

Answer (A) is incorrect because, although the firm’s return on equity investment was 4%, its return on all fundsinvested was 5% ($25,000 pretax operating income ÷ $500,000).Answer (B) is correct. Return on investment is commonly calculated by dividing pretax income by total assetsavailable. Residual income is the excess of the return on investment over a targeted amount equal to an imputedinterest charge on invested capital. The rate used is ordinarily the weighted-average cost of capital. Some companiesmeasure managerial performance in terms of the amount of residual income rather than the percentage return oninvestment. Because REB has assets of $500,000 and a cost of capital of 6%, it must earn $30,000 on those assets tocover the cost of capital. Given that operating income was only $25,000, it had a negative residual income of $5,000.Answer (C) is incorrect because ROI is commonly based on before-tax income.Answer (D) is incorrect because $(22,000) equals the difference between net profit after taxes and targeted income.

[57] Gleim #: 1.57 -- Source: Publisher

Managerial performance may be measured in many ways. For example, an internal nonfinancial measure is

Market share.A.Delivery performance.B.Customer satisfaction.C.Manufacturing lead time.D.

Answer (A) is incorrect because market share is an external nonfinancial measure.Answer (B) is incorrect because delivery performance is an external nonfinancial measure.Answer (C) is incorrect because customer satisfaction is an external nonfinancial measure.Answer (D) is correct. Feedback regarding managerial performance may take the form of financial and nonfinancialmeasures that may be internally or externally generated. Moreover, different measures have a long-term or short-termemphasis. Examples of internal nonfinancial measures are product quality, new product development time, andmanufacturing lead time (cycle time).

[58] Gleim #: 1.58 -- Source: Publisher

Many forms of performance feedback are based on accounting information. For example, a divisional manager may beevaluated based on return on investment (income ÷ investment). One step in the process of developing a performancemeasure based on accounting information is to determine the basis for stating the measure in terms of dollars. Thus, if ROIis the chosen measure, and investment is defined as total assets, comparability issues are most likely to arise when theattribute used to calculate total assets is

Current cost.A.Current disposal price.B.Historical cost.C.Present value.D.

Gleim's CIA Test Prep: Part III: Business Analysis and Information Technology(1165 questions)

Copyright 2004 Gleim Publications, Inc. Page 22Printed for g j

Page 23: Part three cia_with_ answers

Answer (A) is incorrect because current cost is an attempt to remedy the theoretical deficiencies of historical cost bypresenting more accurate balance sheet values.Answer (B) is incorrect because current disposal price is an attempt to remedy the theoretical deficiencies of historical costby presenting more accurate balance sheet values.Answer (C) is correct. Historical cost creates comparability issues because returns on significantly depreciated assets maybe higher than those on newer assets that have been acquired using inflated dollars. Thus, otherwise similarly situatedmanagers may report different operating results. Moreover, managers may be reluctant to replace aging assets.Answer (D) is incorrect because present value is an attempt to remedy the theoretical deficiencies of historical cost bypresenting more accurate balance sheet values.

[59] Gleim #: 1.59 -- Source: Publisher

An organization’s managerial decision-making model for capital budgeting is based on the net present value of discountedcash flows. The same organization’s managerial performance evaluation model is based on annual divisional return oninvestment. Which of the following is true?

Divisional managers are likely to maximize the measures in the decision-making model.A.Divisional managers are likely to maximize the measures in the performance evaluation model.B.The manager has an incentive to accept a project with a positive net present value that initially has a negative effecton net income.

C.

The use of models with different criteria promotes goal congruence.D.

Answer (A) is incorrect because self-interest provides an incentive to maximize the measures used in performanceevaluation.Answer (B) is correct. Effective management control requires performance measurement and feedback. This processaffects allocation of resources to organizational subunits. It also affects decisions about managers’ compensation,advancement, and future assignments. Furthermore, evaluating their performance serves to motivate managers tooptimize the measures in the performance evaluation model. However, that model may be inconsistent with theorganization’s model for managerial decision making.Answer (C) is incorrect because a manager evaluated on the basis of annual ROI has an interest in maximizing short-term net income, not long-term NPV.Answer (D) is incorrect because the models should be synchronized so that the goals of the organization and themanager are congruent.

[60] Gleim #: 1.60 -- Source: CIA 597 III-14

Which of the following criteria would be most useful to a sales department manager in evaluating the performance of themanager’s customer-service group?

The customer is always right.A.Customer complaints should be processed promptly.B.Employees should maintain a positive attitude when dealing with customers.C.All customer inquiries should be answered within 7 days of receipt.D.

Answer (A) is incorrect because customer orientation is difficult to quantify.Answer (B) is incorrect because the standard specified is vague.Answer (C) is incorrect because no measure of a positive attitude has been specified for the employee.Answer (D) is correct. A criterion that requires all customer inquiries to be answered within 7 days of receipt permitsaccurate measurement of performance. The quantitative and specific nature of the appraisal using this standard avoidsthe vagueness, subjectivity, and personal bias that may afflict other forms of personnel evaluations.

[61] Gleim #: 1.61 -- Source: Publisher

Using the balanced scorecard approach, an organization evaluates managerial performance based on

A single ultimate measure of operating results, such as residual income.A.Multiple financial and nonfinancial measures.B.Multiple nonfinancial measures only.C.Multiple financial measures only.D.

Gleim's CIA Test Prep: Part III: Business Analysis and Information Technology(1165 questions)

Copyright 2004 Gleim Publications, Inc. Page 23Printed for g j

Page 24: Part three cia_with_ answers

Answer (A) is incorrect because the balanced scorecard approach uses multiple measures.Answer (B) is correct. The trend in managerial performance evaluation is the balanced scorecard approach. Multiplemeasures of performance permit a determination as to whether a manager is achieving certain objectives at the expense ofothers that may be equally or more important. These measures may be financial or nonfinancial and usually include items infour categories:  profitability; customer satisfaction; innovation; and efficiency, quality, and time.Answer (C) is incorrect because the balanced scorecard approach includes financial measures.Answer (D) is incorrect because the balanced scorecard approach includes nonfinancial measures.

[62] Gleim #: 1.62 -- Source: CIA 597 III-3

Focusing on customers, promoting innovation, learning new philosophies, driving out fear, and providing extensivetraining are all elements of a major change in organizations. These elements are aimed primarily at

Copying leading organizations to better compete with them.A.Focusing on the total quality of products and services.B.Being efficient and effective at the same time, in order to indirectly affect profits.C.Managing costs of products and services better, in order to become the low-cost provider.D.

Answer (A) is incorrect because competitive benchmarking is just one tool for implementing TQM.Answer (B) is correct. TQM is a comprehensive approach to quality. It treats the pursuit of quality as a basicorganizational function that is as important as production or marketing. TQM is the continuous pursuit of quality inevery aspect of organizational activities through a philosophy of doing it right the first time, employee training andempowerment, promotion of teamwork, improvement of processes, and attention to satisfaction of customers, bothinternal and external. TQM emphasizes the supplier’s relationship with the customer, identifies customer needs, andrecognizes that everyone in a process is at some time a customer or supplier of someone else, either within or withoutthe organization.Answer (C) is incorrect because TQM’s primary focus is not profitability.Answer (D) is incorrect because TQM’s primary focus is not cost reduction.

[Fact Pattern #6]The management and employees of a large household goods moving company decided to adopt total quality management (TQM)and continuous improvement (CI). They believed that, if their company became nationally known as adhering to TQM and CI,one result would be an increase in the company’s profits and market share.

[63] Gleim #: 1.63 -- Source: CIA 1195 III-12

(Refers to Fact Pattern #6)The primary reason for adopting TQM was to achieve

Greater customer satisfaction.A.Reduced delivery time.B.Reduced delivery charges.C.Greater employee participation.D.

Answer (A) is correct. TQM is an integrated system that anticipates, meets, and exceeds customers’ needs, wants,and expectations.Answer (B) is incorrect because reduced delivery time is one of many potential activities that need improvement.Answer (C) is incorrect because reduced delivery charges is one of many potential activities that need improvement.Answer (D) is incorrect because increased employee participation is necessary to achieve TQM, but it is not theprimary purpose for establishing the program.

[64] Gleim #: 1.64 -- Source: CIA 1195 III-13

(Refers to Fact Pattern #6)Quality is achieved more economically if the company focuses on

Appraisal costs.A.Prevention costs.B.Internal failure costs.C.External failure costs.D.

Gleim's CIA Test Prep: Part III: Business Analysis and Information Technology(1165 questions)

Copyright 2004 Gleim Publications, Inc. Page 24Printed for g j

Page 25: Part three cia_with_ answers

Answer (A) is incorrect because prevention is ordinarily less costly than the combined costs of appraisal, internal failure,and external failure.Answer (B) is correct. Prevention attempts to avoid defective output. Prevention costs include preventive maintenance,employee training, review of equipment design, and evaluation of suppliers. Prevention is less costly than detection andcorrection of defective output.Answer (C) is incorrect because prevention is ordinarily less costly than the combined costs of appraisal, internal failure,and external failure.Answer (D) is incorrect because prevention is ordinarily less costly than the combined costs of appraisal, internal failure,and external failure.

[65] Gleim #: 1.65 -- Source: CIA 597 III-4

Total quality management in a manufacturing environment is best exemplified by

Identifying and reworking production defects before sale.A.Designing the product to minimize defects.B.Performing inspections to isolate defects as early as possible.C.Making machine adjustments periodically to reduce defects.D.

Answer (A) is incorrect because TQM emphasizes prevention, not rework. The approach of TQM is to build in anddesign in quality, not to “fix it in” or “inspect it in.”Answer (B) is correct. Total quality management emphasizes quality as a basic organizational function. TQM is thecontinuous pursuit of quality in every aspect of organizational activities. One of the basic tenets of TQM is doing itright the first time. Thus, errors should be caught and corrected at the source, and quality should be built in (designedin) from the start.Answer (C) is incorrect because TQM emphasizes prevention, not inspection. The approach of TQM is to build inand design in quality, not to “fix it in” or “inspect it in.”Answer (D) is incorrect because TQM emphasizes prevention, not adjustment. The approach of TQM is to build inand design in quality, not to “fix it in” or “inspect it in.”

[66] Gleim #: 1.66 -- Source: CIA 597 III-24

One of the main reasons that implementation of a total quality management program works better through the use of teamsis

Teams are more efficient and help an organization reduce its staffing.A.Employee motivation is always higher for team members than for individual contributors.B.Teams are a natural vehicle for sharing ideas, which leads to process improvement.C.The use of teams eliminates the need for supervision, thereby allowing a company to reduce staffing.D.

Answer (A) is incorrect because teams are often inefficient and costly.Answer (B) is incorrect because high motivation does not directly affect the process improvement that is the key toquality improvement.Answer (C) is correct. TQM promotes teamwork by modifying or eliminating traditional (and rigid) verticalhierarchies and instead forming flexible groups of specialists. Quality circles, cross-functional teams, and self-managed teams are typical formats. Teams are an excellent vehicle for encouraging the sharing of ideas and removingprocess improvement obstacles.Answer (D) is incorrect because the use of teams with less supervision and reduced staffing may be by-products ofTQM, but they are not ultimate objectives.

[67] Gleim #: 1.67 -- Source: CIA 1195 III-28

A traditional quality control process in manufacturing consists of mass inspection of goods only at the end of a productionprocess. A major deficiency of the traditional control process is that

It is expensive to do the inspections at the end of the process.A.It is not possible to rework defective items.B.It is not 100% effective.C.It does not focus on improving the entire production process.D.

Gleim's CIA Test Prep: Part III: Business Analysis and Information Technology(1165 questions)

Copyright 2004 Gleim Publications, Inc. Page 25Printed for g j

Page 26: Part three cia_with_ answers

Answer (A) is incorrect because other quality control processes can also be expensive.Answer (B) is incorrect because reworking defective items may be possible although costly.Answer (C) is incorrect because no quality control system will be 100% effective.Answer (D) is correct. The process used to produce the goods is not thoroughly reviewed and evaluated for efficiency andeffectiveness. Preventing defects and increasing efficiency by improving the production process raises quality standards anddecreases costs.

[68] Gleim #: 1.68 -- Source: CIA 1196 III-24

Under a total quality management (TQM) approach

Measurement occurs throughout the process, and errors are caught and corrected at the source.A.Quality control is performed by highly trained inspectors at the end of the production process.B.Upper management assumes the primary responsibility for the quality of the products and services.C.A large number of suppliers are used in order to obtain the lowest possible prices.D.

Answer (A) is correct. Total quality management emphasizes quality as a basic organizational function. TQM is thecontinuous pursuit of quality in every aspect of organizational activities. One of the basic tenets of TQM is doing itright the first time. Thus, errors should be caught and corrected at the source.Answer (B) is incorrect because total quality management emphasizes discovering errors throughout the process, notinspection of finished goods.Answer (C) is incorrect because all members of the organization assume responsibility for quality of the products andservices.Answer (D) is incorrect because the total quality management philosophy recommends limiting the number ofsuppliers to create a strong relationship.

[69] Gleim #: 1.69 -- Source: CIA 597 III-23

Which of the following is a key to successful total quality management?

Training quality inspectors.A.Focusing intensely on the customer.B.Creating appropriate hierarchies to increase efficiency.C.Establishing a well-defined quality standard, then focusing on meeting it.D.

Answer (A) is incorrect because total quality management (TQM) de-emphasizes specialized quality inspectors.Answer (B) is correct. TQM emphasizes satisfaction of customers, both internal and external. TQM considers thesupplier’s relationship with the customer, identifies customer needs, and recognizes that everyone in a process is atsome time a customer or supplier of someone else, either within or without the organization. Thus, TQM begins withexternal customer requirements, identifies internal customer-supplier relationships and requirements, and establishesrequirements for external suppliers.Answer (C) is incorrect because centralization often needs to be reduced to implement a TQM process.Answer (D) is incorrect because TQM involves continuous improvement; once a standard is reached, continuousimprovement requires its constant reevaluation.

[70] Gleim #: 1.70 -- Source: CIA 596 III-29

Which of the following is a characteristic of total quality management (TQM)?

Management by objectives.A.On-the-job training by other workers.B.Quality by final inspection.C.Education and self-improvement.D.

Gleim's CIA Test Prep: Part III: Business Analysis and Information Technology(1165 questions)

Copyright 2004 Gleim Publications, Inc. Page 26Printed for g j

Page 27: Part three cia_with_ answers

Answer (A) is incorrect because one of the 14 points recommends elimination of numerical quotas. MBO causes aggressivepursuit of numerical quotas.Answer (B) is incorrect because informal learning from coworkers serves to entrench bad work habits. One of the 14 pointsstresses proper training of everyone.Answer (C) is incorrect because another of the 14 points states that quality by final inspection is unnecessary if quality isbuilt in from the start.Answer (D) is correct. According to management theorist W. Edwards Deming’s well-known 14 points, education and self-improvement are essential. Knowledge is opportunity. Hence, continuous improvement should be everyone’s primary careerobjective.

[71] Gleim #: 1.71 -- Source: CIA 596 III-30

In which of the following organizational structures does total quality management (TQM) work best?

Hierarchal.A.Teams of people from the same specialty.B.Teams of people from different specialties.C.Specialists working individually.D.

Answer (A) is incorrect because hierarchal organization stifles TQM.Answer (B) is incorrect because TQM works best with teams of people from different specialties.Answer (C) is correct. TQM advocates replacement of the traditional hierarchal structure with teams of people fromdifferent specialities. This change follows from TQM’s emphasis on empowering employees and teamwork.Employees should have proper training, necessary information, and the best tools; be fully engaged in the decisionprocess; and receive fair compensation. If such empowered employees are assembled in teams of individuals with therequired skills, TQM theorists believe they will be more effective than people performing their tasks separately in arigid structure.Answer (D) is incorrect because teamwork is essential for TQM.

[72] Gleim #: 1.72 -- Source: CIA 597 III-20

One of the main reasons total quality management (TQM) can be used as a strategic weapon is that

The cumulative improvement from a company’s TQM efforts cannot readily be copied by competitors.A.Introducing new products can lure customers away from competitors.B.Reduced costs associated with better quality can support higher shareholder dividends.C.TQM provides a comprehensive planning process for a business.D.

Answer (A) is correct. TQM is a comprehensive approach to quality. It treats the pursuit of quality as a basicorganizational function that is as important as production or marketing. Because TQM affects every aspect of theorganization’s activities, it permeates the organizational culture. Thus, the cumulative effect of TQM’s continuousimprovement process can attract and hold customers and cannot be duplicated by competitors.Answer (B) is incorrect because new products can be quickly copied by competitors and therefore do not provide asustained competitive advantage.Answer (C) is incorrect because TQM does not focus solely on cost reduction.Answer (D) is incorrect because TQM is only one tool of strategic management.

[73] Gleim #: 1.73 -- Source: CIA 597 III-20

Faced with 3 years of steadily decreasing profits despite increased sales and a growing economy, which of the following isthe healthiest course of action for a chief executive officer to take?

Set a turnaround goal of significantly increasing profits within 2 months. Set clear short-term objectives for eachoperating unit that, together, should produce the turnaround.

A.

Reduce staff by 10% in every unit.B.Classify all job functions as either (a) adding value in the eyes of the customer (such as production and sales) or (b)not adding value in the eyes of the customer (such as accounting and human resources). Reduce staff in the non-value-adding functions by 20%.

C.

Implement a plan to encourage innovation at all levels. Use early retirement and reemployment programs to trim staffsize.

D.

Gleim's CIA Test Prep: Part III: Business Analysis and Information Technology(1165 questions)

Copyright 2004 Gleim Publications, Inc. Page 27Printed for g j

Page 28: Part three cia_with_ answers

Answer (A) is incorrect because this response illustrates two of the characteristics of organizational decline:  increasedcentralization of decision making and lack of long-term planning. The exclusive emphasis on short-term results is likely tobe counterproductive.Answer (B) is incorrect because another characteristic of organizational decline is nonprioritized downsizing. By itself,downsizing rarely turns a company around.Answer (C) is incorrect because reducing staff disproportionately in control functions could have disastrous consequences.Answer (D) is correct. Organizational decline has been found to have the following characteristics:  greater centralization,lack of long-term planning, reduced innovation, scapegoating, resistance to change, high turnover of competent leaders, lowmorale, nonprioritized downsizing, and conflict. Reversing these characteristics is the key to reversing organizationaldecline, for example, by encouraging innovation in all aspects of the organization’s activities and by redeploying personnel.

[74] Gleim #: 1.74 -- Source: CIA 1196 III-60

The International Organization for Standardization has developed standards for ring networks that include faultmanagement, configuration management, accounting management, security management, and performance monitoring.Which of the following controls is included in the performance-monitoring standards?

Reporting the failure of network fiber-optic lines.A.Recording unauthorized access violations.B.Compiling statistics on the number of times that application software is used.C.Allocating network costs to system users of the network.D.

Answer (A) is incorrect because the failure of network fiber-optic lines is a fault management control.Answer (B) is incorrect because recording unauthorized access violations is a security management control.Answer (C) is correct. The International Organization for Standards consists of standards organizations from morethan 75 countries. Its performance-monitoring standards pertain to management’s ongoing assessment of the qualityof performance over time. Recording software usage is a performance-monitoring control concerned with the extentand efficiency of network software use.Answer (D) is incorrect because allocating network costs to system users of the network is an accountingmanagement control.

[Fact Pattern #7]Listed below are costs of quality that a manufacturing company has incurred throughout its operations. The company plans toprepare a report that classifies these costs into the following four categories:  preventive costs, appraisal costs, internal failurecosts, and external failure costs.

Cost Items AmountDesign reviews $275,000Finished goods returned due to failure 55,000Freight on replacement finished goods 27,000Labor inspection during manufacturing 75,000Labor inspection of raw materials 32,000Manufacturing product-testing labor 63,000Manufacturing rework labor and overhead 150,000Materials used in warranty repairs 68,000Process engineering 180,000Product-liability claims 145,000Product-testing equipment 35,000Repairs to equipment due to breakdowns 22,000Scheduled equipment maintenance 90,000Scrap material 125,000Training of manufacturing workers 156,000

[75] Gleim #: 1.75 -- Source: CIA 597 III-96

(Refers to Fact Pattern #7)The costs of quality that are incurred in detecting units of product that do not conform to product specifications are referredto as

Gleim's CIA Test Prep: Part III: Business Analysis and Information Technology(1165 questions)

Copyright 2004 Gleim Publications, Inc. Page 28Printed for g j

Page 29: Part three cia_with_ answers

Preventive costs.A.Appraisal costs.B.Internal failure costs.C.External failure costs.D.

Answer (A) is incorrect because prevention attempts to avoid defective output, e.g., by employee training, review ofequipment design, preventive maintenance, and evaluation of suppliers.Answer (B) is correct. The categories of quality costs include conformance costs (prevention and appraisal) andnonconformance costs (internal failure and external failure). Appraisal costs embrace such activities as statistical qualitycontrol programs, inspection, and testing. Thus, the cost of detecting nonconforming products is an appraisal cost.Answer (C) is incorrect because internal failure costs are incurred when detection of defective products occurs beforeshipment, including scrap, rework, tooling changes, and downtime.Answer (D) is incorrect because external failure costs are incurred after the product has been shipped, including the costsassociated with warranties, product liability, and loss of customer goodwill.

[76] Gleim #: 1.76 -- Source: CIA 597 III-97

(Refers to Fact Pattern #7)The dollar amount of the costs of quality classified as preventive costs for the manufacturing firm would be

$643,000A.$701,000B.$736,000C.$768,000D.

Answer (A) is incorrect because $643,000 omits scheduled equipment maintenance and includes labor inspection ofraw materials (an appraisal cost).Answer (B) is correct. Prevention attempts to avoid defective output, e.g., by employee training, review of equipmentdesign, preventive maintenance, and evaluation of suppliers. Accordingly, the preventive costs equal $701,000($275,000 design reviews + $180,000 process engineering + $90,000 scheduled maintenance + $156,000 training).Answer (C) is incorrect because $736,000 includes the cost of product testing equipment (an appraisal cost).Answer (D) is incorrect because $768,000 includes the cost of product testing equipment and labor inspection of rawmaterials. Both costs are appraisal costs.

[77] Gleim #: 1.77 -- Source: CMA 694 4-1

When using PERT (Program Evaluation Review Technique), the expected time for an activity when given an optimistictime (A), a pessimistic time (B), and a most likely time (m) is calculated by which one of the following formulas?

(b – a) ÷ 2A.(a + b) ÷ 2B.(a + 4m + b) ÷ 6C.(4abm) ÷ 6D.

Answer (A) is incorrect because the most likely time estimate should be included in the formula.Answer (B) is incorrect because the most likely time estimate should be included in the formula.Answer (C) is correct. PERT was developed to aid managers in controlling large, complex projects. PERT analysisincludes probabilistic estimates of activity completion times. Three time estimates are made:  optimistic, most likely,and pessimistic. The time estimates for an activity are assumed to approximate a beta probability distribution. PERTapproximates the mean of the beta distribution by dividing the sum of the optimistic time, the pessimistic time, andfour times the most likely time by six.Answer (D) is incorrect because all time estimates are not weighted equally.

[78] Gleim #: 1.78 -- Source: CIA 597 III-98

The process of adding resources to shorten selected activity times on the critical path in project scheduling is called

Crashing.A.The Delphi technique.B.Material-requirements planning.C.A branch-and-bound solution.D.

Gleim's CIA Test Prep: Part III: Business Analysis and Information Technology(1165 questions)

Copyright 2004 Gleim Publications, Inc. Page 29Printed for g j

Page 30: Part three cia_with_ answers

Answer (A) is correct. When making a cost-time trade-off, the first activity to be crashed (have its completion timeaccelerated) is one on the critical path. To select an activity on another path would not reduce the total time of completion.The activity chosen should have a completion time that can be accelerated at the lowest possible cost per unit of time saved.Answer (B) is incorrect because the Delphi technique is a qualitative forecasting approach.Answer (C) is incorrect because material-requirements planning is an inventory model.Answer (D) is incorrect because a branch-and-bound solution is an integer programming solution.

[79] Gleim #: 1.79 -- Source: CMA 1288 5-24

California Building Corporation uses the critical path method to monitor construction jobs. The company is currently 2weeks behind schedule on Job #181, which is subject to a $10,500-per-week completion penalty. Path A-B-C-F-G-H-I hasa normal completion time of 20 weeks, and critical path A-D-E-F-G-H-I has a normal completion time of 22 weeks. Thefollowing activities can be crashed:

  Cost to Crash Cost to CrashActivities 1 Week 2 Weeks

BC $  8,000 $15,000DE 10,000 19,600EF 8,800 19,500

California Building desires to reduce the normal completion time of Job #181 and, at the same time, report the highestpossible income for the year. California Building should crash

Activity BC 1 week and activity EF 1 week.A.Activity BC 2 weeks.B.Activity EF 2 weeks.C.Activity DE 1 week and activity EF 1 week.D.

Answer (A) is incorrect because BC is not on the critical path.Answer (B) is incorrect because BC is not on the critical path.Answer (C) is incorrect because the cost of crashing EF 2 weeks is $19,500, which is greater than the total cost tocrash DE and EF for 1 week each.Answer (D) is correct. Activities that are to be crashed in a CPM problem should be ones that are on the critical(longest) path. Thus, activity BC should not be selected because it is not on the critical path. To finish activity BC 2weeks early would not reduce the total time to complete the project. Therefore, the only feasible choices are DE andEF on the critical path. The total cost to crash DE and EF for 1 week each is $18,800 ($10,000 + $8,800), which isless than the cost to crash either activity for 2 weeks. Thus, DE and EF should be crashed for 1 week each because thetotal cost is less than the $21,000 ($10,500 × 2) 2-week delay penalty.

[80] Gleim #: 1.80 -- Source: CMA 688 5-13

In a PERT network, the optimistic time for a particular activity is 9 weeks, and the pessimistic time is 21 weeks. Which oneof the following is the best estimate of the standard deviation for the activity?

2A.6B.9C.12D.

Gleim's CIA Test Prep: Part III: Business Analysis and Information Technology(1165 questions)

Copyright 2004 Gleim Publications, Inc. Page 30Printed for g j

Page 31: Part three cia_with_ answers

Answer (A) is correct. PERT analysis includes probabilistic estimates of activity completion times. Three time estimates aremade: optimistic, most likely, and pessimistic. The time estimates for an activity are assumed to approximate a betaprobability distribution. In contrast to the normal distribution, this distribution has finite endpoints (the optimistic andpessimistic estimates) and is unimodal; that is, it has only one mode (the most likely time). PERT approximates the mean ofthe beta distribution by dividing the sum of the optimistic time, the pessimistic time, and four times the most likely time (themode) by six. The standard deviation is approximated by dividing the difference between the pessimistic and optimistictimes by six. The basis for the latter approximation is that various probability distributions have tails that lie about plus orminus three standard deviations from the mean. For example, 99.9% of observations in the normal distribution are expectedto lie within this range. Accordingly, if the pessimistic and optimistic times are 21 and 9 weeks, respectively, the standarddeviation is 2 weeks (12 ÷ 6).Answer (B) is incorrect because 6 is the approximate number of standard deviations between the tails of the normaldistribution.Answer (C) is incorrect because 9 weeks is the pessimistic estimate.Answer (D) is incorrect because 12 weeks is the difference between the optimistic and the pessimistic estimates.

[81] Gleim #: 1.81 -- Source: CMA 688 5-17

A PERT network has only two activities on its critical path. These activities have standard deviations of 6 and 8,respectively. The standard deviation of the project completion time is

7A.10B.14C.48D.

Answer (A) is incorrect because 7 is the average standard deviation.Answer (B) is correct. The mean time for the critical path is simply the sum of the means of the activity times.However, the standard deviation equals the square root of the sum of the variances (squares of the standarddeviations) of the times for activities on the critical path. The standard deviation of the project completion time (timefor the critical path) is therefore the square root of 100 (62 + 82), or 10.Answer (C) is incorrect because 14 is the sum of the standard deviations.Answer (D) is incorrect because 48 is the product of the standard deviations.

[Fact Pattern #8]The PERT network diagram and the corresponding activity cost chart for a manufacturing project at Networks, Inc. are presentedbelow. The numbers in the diagram are the expected times (in days) to perform each activity in the project.

(Refer to Figure CIA3_01_08.)    Normal CrashActivity Cost Crash Time Cost

AB $3,000 3.50 days $4,000AC 5,000 4.50 5,250AD 4,000 4.00 4,750BE 6,000 5.00 7,000CE 8,000 5.00 9,200DE 6,000 6.50 6,750BC 2,500 .50 3,500BD 2,000 .25 2,500

[82] Gleim #: 1.82 -- Source: CMA 1290 4-7

(Refers to Fact Pattern #8)The expected time of the critical path is

12.0 days.A.13.0 days.B.11.5 days.C.11.0 days.D.

Gleim's CIA Test Prep: Part III: Business Analysis and Information Technology(1165 questions)

Copyright 2004 Gleim Publications, Inc. Page 31Printed for g j

Page 32: Part three cia_with_ answers

Answer (A) is incorrect because the expected time of the critical path is 13.0 days.Answer (B) is correct. The critical path is the longest path. The longest path in the diagram is A-D-E, which requires 13days (5.5 + 7.5) based on expected times.Answer (C) is incorrect because the expected time of the critical path is 13.0 days.Answer (D) is incorrect because the expected time of the critical path is 13.0 days.

[83] Gleim #: 1.83 -- Source: CMA 1290 4-8

(Refers to Fact Pattern #8)In order to keep costs at a minimum and decrease the completion time by 1 1/2 days, Networks, Inc. should crashactivity(ies)

AD and AB.A.DE.B.AD.C.AB and CE.D.

Answer (A) is correct. The critical (longest) path is A-D-E, which has an expected time of 13 days (see precedingquestion). However, to decrease the project’s completion time by 1.5 days, paths A-B-C-E (4.5 + 1.0 + 6.5 = 12 days)and A-B-D-E (4.5 + .5 + 7.5 = 12.5 days) as well as A-D-E must also be shortened. Hence, A-D-E must be reducedby 1.5 days, A-B-C-E by .5 day, and A-B-D-E by 1.0 day. The only way to decrease A-D-E by 1.5 days is to crashactivity AD (5.5 expective time – 4.0 crash time = 1.5 days). Crashing DE results in a 1.0-day saving (7.5 – 6.5) only.Crashing AB is the efficient way to reduce both A-B-C-E and A-B-D-E by the desired amount of time because it ispart of both paths. The incremental cost of crashing AB is $1,000 ($4,000 crash cost – $3,000 normal cost) to shortenthe completion time by 1.0 day (4.5 – 3.5). The alternatives for decreasing both A-B-C-E and A-B-D-E are morecostly.Answer (B) is incorrect because crashing activity DE saves only 1.0 day (7.5 – 6.5) on the critical path and does notreduce the time needed for A-B-C-E.Answer (C) is incorrect because crashing AD does not reduce the time necessary to complete A-B-C-E or A-B-D-E.Answer (D) is incorrect because AB and CE are not on the critical path.

[84] Gleim #: 1.84 -- Source: CMA 694 4-3

When making a cost-time trade-off in PERT analysis, the first activity that should be crashed is the activity

With the largest amount of slack.A.With the lowest unit crash cost.B.On the critical path with the maximum possible time reduction.C.On the critical path with the lowest unit crash cost.D.

Answer (A) is incorrect because eliminating an activity with slack will not reduce the total time of the project.Answer (B) is incorrect because the activity with the lowest unit crash cost may not be on the critical path.Answer (C) is incorrect because the time reduction should be related to its cost. The maximum time reduction maynot be cost effective.Answer (D) is correct. When making a cost-time trade-off, the first activity to be crashed (have its completion timeaccelerated) is one on the critical path. To select an activity on another path would not reduce the total time ofcompletion. The initial activity chosen should be the one with the completion time that can be accelerated at thelowest possible cost per unit of time saved.

[85] Gleim #: 1.85 -- Source: CMA 695 4-12

In PERT, slack is the

Uncertainty associated with time estimates.A.Difference between the latest starting time and earliest finishing time.B.Path that has the largest amount of time associated with it.C.Number of days an activity can be delayed without forcing a delay for the entire project.D.

Gleim's CIA Test Prep: Part III: Business Analysis and Information Technology(1165 questions)

Copyright 2004 Gleim Publications, Inc. Page 32Printed for g j

Page 33: Part three cia_with_ answers

Answer (A) is incorrect because uncertainty is reflected in the use of probabilistic estimates of completion times.Answer (B) is incorrect because the difference between the latest starting time and earliest finishing time is irrelevant.Answer (C) is incorrect because the path with the largest amount of time associated with it is the critical path.Answer (D) is correct. PERT diagrams are free-form networks showing each activity in a large project as a line betweenevents. The critical path is the longest path in time through the network. That path is critical in that, if any activity on thecritical path takes longer than expected, the entire project will be delayed. Paths that are not critical have slack time. Slack isthe number of days an activity can be delayed without forcing a delay for the entire project.

[86] Gleim #: 1.86 -- Source: CIA 1196 III-96

The network below describes the interrelationships of several activities necessary to complete a project. The arrowsrepresent the activities. The numbers between the arrows indicate the number of months to complete each activity.

(Refer to Figure CIA3_01_09.)

The shortest time to complete the project is

5 months.A.6 months.B.8 months.C.14 months.D.

Answer (A) is incorrect because the project cannot be completed in less than 8 months.Answer (B) is incorrect because the project cannot be completed in less than 8 months.Answer (C) is correct. The longest, or critical, path in the network from node (A) to node (F) is path A-C-D-F. Allother paths are shorter than path A-C-D-F, so the activities along those paths can be completed before the activitiesalong path A-C-D-F. Thus, the shortest time to complete the project is 8 months (3 + 3 + 2).Answer (D) is incorrect because no path through the network requires 14 months.

[87] Gleim #: 1.87 -- Source: CIA 596 III-100

A shortest-route algorithm is used in network models to

Identify bottlenecks in a network and hence identify the longest path.A.Minimize total travel time from one site to each of the other sites in a transportation system.B.Maximize throughput in networks with distinct entry (source node) and exit (sink node) points.C.Identify the set of connecting branches having the shortest combined length.D.

Answer (A) is incorrect because the critical path method (CPM) is intended to identify bottlenecks in a network andhence identify the longest path.Answer (B) is correct. Network models are used to solve managerial problems pertaining to project scheduling,information systems design, and transportation systems design. Networks consisting of nodes and arcs may be createdto represent in graphic form problems related to transportation, assignment, and transshipment. The shortest-route,minimal spanning tree, and maximal flow problems are other applications of network models. A shortest-routealgorithm minimizes total travel time from one site to each of the other sites in a transportation system.Answer (C) is incorrect because the maximal flow algorithm maximizes throughput in networks with distinct entry(source node) and exit (sink node) points. Examples of applications are highway transportation systems and oilpipelines. Flows are limited by capacities of the arcs (e.g., highways or pipes).Answer (D) is incorrect because the minimal spanning tree algorithm identifies the set of connecting branches havingthe shortest combined length. A spanning tree is a group of branches (arcs) that connects each node in the network toevery other node. An example problem is the determination of the shortest telecommunications linkage among usersat remote sites and a central computer.

[88] Gleim #: 1.88 -- Source: CIA 597 III-32

Reengineering is the thorough analysis, fundamental rethinking, and complete redesign of essential business processes. Theintended result is a dramatic improvement in service, quality, speed, and cost. An internal auditor’s involvement inreengineering should include all of the following except

Gleim's CIA Test Prep: Part III: Business Analysis and Information Technology(1165 questions)

Copyright 2004 Gleim Publications, Inc. Page 33Printed for g j

Page 34: Part three cia_with_ answers

Determining whether the process has senior management’s support.A.Recommending areas for consideration.B.Developing audit plans for the new system.C.Directing the implementation of the redesigned process.D.

Answer (A) is incorrect because internal auditors may perform the functions of determining whether the process has seniormanagement's support, recommending areas for consideration, and developing audit plans for the new system.Answer (B) is incorrect because internal auditors may perform the functions of determining whether the process has seniormanagement's support, recommending areas for consideration, and developing audit plans for the new system.Answer (C) is incorrect because internal auditors may perform the functions of determining whether the process has seniormanagement's support, recommending areas for consideration, and developing audit plans for the new system.Answer (D) is correct. Internal auditors should not become directly involved in the implementation of the redesign process.This involvement would impair their independence and objectivity. Staff assignments of internal auditors should be rotatedperiodically whenever it is practicable to do so.

[89] Gleim #: 1.89 -- Source: CIA 1195 I-66

Monitoring is an important component of internal control. Which of the following items would not be an example ofmonitoring?

Management regularly compares divisional performance with budgets for the division.A.Data processing management regularly generates exception reports for unusual transactions or volumes oftransactions and follows up with investigation as to causes.

B.

Data processing management regularly reconciles batch control totals for items processed with batch controls foritems submitted.

C.

Management has asked internal auditing to perform regular audits of the controls over cash processing.D.

Answer (A) is incorrect because budgetary comparison is a typical example of a monitoring control.Answer (B) is incorrect because investigation of exceptions is a monitoring control used by lower-level managementto determine when their operations may be out of control.Answer (C) is correct. Monitoring assesses the quality of internal control over time. Management considers whetherinternal control is properly designed and operating as intended and modifies it to reflect changing conditions.Monitoring may be in the form of separate, periodic evaluations or of ongoing monitoring. Ongoing monitoringoccurs as part of routine operations. It includes management and supervisory review, comparisons, reconciliations,and other actions by personnel as part of their regular activities. However, reconciling batch control totals is aprocessing control.Answer (D) is incorrect because internal auditing is a form of monitoring. It serves to evaluate management's othercontrols.

[90] Gleim #: 1.90 -- Source: Publisher

Which of the following is a short-term approach to managing bottlenecks or binding constraints in production anddistribution processes?

Theory of constraints.A.Reengineering.B.Rationalization.C.Micromotion study.D.

Answer (A) is correct. The theory of constraints (TOC) is a short-term approach to managing bottlenecks (bindingconstraints) in production and distribution processes. Its basic principle is that short-term profit maximizationrequires maximizing the contribution margin of the binding constraint (the throughput contribution).Answer (B) is incorrect because reengineering an approach to business process analysis that entails processinnovation and core process redesign. Instead of improving existing procedures, it finds new ways of doing things.Answer (C) is incorrect because rationalization is yet another approach to business process analysis. It is thestreamlining of procedures to make automation more efficient.Answer (D) is incorrect because a micromotion study is a method used for work management, and it requiresvideotaping the performance of a job.

[91] Gleim #: 1.91 -- Source: Publisher

Which of the following statements is not true regarding ISO 9000 standards?

Gleim's CIA Test Prep: Part III: Business Analysis and Information Technology(1165 questions)

Copyright 2004 Gleim Publications, Inc. Page 34Printed for g j

Page 35: Part three cia_with_ answers

Compliance with the standards is voluntary.A.The ISO 9000 standards are revised every 5 years to account for technological and market developments.B.The objective of ISO 9000 standards is to ensure high quality products and services.C.ISO 9000 is a set of generic standards for establishing and maintaining a quality system within a company.D.

Answer (A) is incorrect because compliance is voluntary but many companies are adopting the standards for competitivereasons or out of fear that the standards will become a requirement in foreign markets.Answer (B) is incorrect because the ISO rules specify that standards are periodically revised every 5 years in light oftechnological and market developments.Answer (C) is correct. The objective of ISO 9000 standards is to ensure consistent quality even if the quality is poor. Themarket will determine the quality of the end result.Answer (D) is incorrect because ISO 9000 standards are generic in nature and only ensure consistent quality in the productbeing produced.

[92] Gleim #: 1.92 -- Source: Publisher

What is the main benefit for most companies who adopt ISO 14000 standards?

Reduced monitoring and inspection by regulatory agencies.A.Companies are required to adopt ISO 14000 standards to operate in the European Union.B.The Environmental Protection Agency cannot use voluntary ISO audits as a basis for imposing penalties.C.Companies learn how well their environmental management system operates relative to those of other companies.D.

Answer (A) is incorrect because not all companies are regulated by government agencies. Also, there is no guaranteethat compliance with ISO 14000 standards will reduce monitoring and inspection by agencies.Answer (B) is incorrect because ISO 14000 standards are not currently required by the European Union.Answer (C) is incorrect because this is a benefit to some companies but not others. Not all companies are subject tothe EPA’s audits.Answer (D) is correct. The main benefit of instituting ISO 14000 standards is internal; companies learn how welltheir environmental management system operates relative to those of other companies. This is accomplished throughthe sharing of information required by ISO standards.

[93] Gleim #: 1.93 -- Source: Publisher

What ensures that a company is complying with the documented Quality Management System procedures and ISO 9000standards?

The internal audit.A.The registrar.B.On-site inspections by an external auditor.C.Management commitment.D.

Answer (A) is correct. An important requirement of the ISO 9000 standards is the internal audit system. Internalaudits assure that the company is complying with the documented QMS procedures and ISO 9000 standards.Answer (B) is incorrect because the registrar evaluates whether the quality control system conforms to the selectedstandard.Answer (C) is incorrect because the registrar is the external auditor who performs the on-site inspection. The registrarobserves the quality control system and evaluates the conformity to the selected standard.Answer (D) is incorrect because the internal audit is responsible for ensuring compliance with the QMS and ISO9000 standards. However, management commitment plays an important role in determining the amount of resourcesdedicated to quality.

[94] Gleim #: 1.94 -- Source: CIA 591 III-44

The following data on variables x and y was collected from June to October:

  June July August September October x 24 31 19 15 22y 104 76 124 140 112

The correlation coefficient between variables x and y is nearest to

Gleim's CIA Test Prep: Part III: Business Analysis and Information Technology(1165 questions)

Copyright 2004 Gleim Publications, Inc. Page 35Printed for g j

Page 36: Part three cia_with_ answers

1.00A.-1.00B.0.50C.0.00D.

Answer (A) is incorrect because a positive correlation coefficient implies that one variable increases (decreases) as the otherincreases (decreases). The data clearly do not support this conclusion.Answer (B) is correct. A correlation coefficient of -1.00 implies a perfect inverse correlation; that is, the observations fallexactly along a straight line and the value of one variable increases (decreases) as the other decreases (increases). In theexample to the left, the equation of the straight line is

 y  =  200 – 4xor  x  =  50 – 1y

   4

Answer (C) is incorrect because a positive correlation coefficient implies that one variable increases (decreases) as the otherincreases (decreases). The data clearly do not support this conclusion.Answer (D) is incorrect because a correlation coefficient of zero implies that the two variables are unrelated. The dataclearly indicate that the two variables move in opposite directions.

[95] Gleim #: 1.95 -- Source: CIA 595 III-97

A company is formulating its plans for the coming year, including the preparation of its cash budget. Historically, 30% ofthe company’s sales are cash sales. The remaining 70% are credit sales with the following collection pattern.

Collections on Account PercentageIn the month of sale 40%In the month following the sale 58%Uncollectible 2%

Sales for the first five months of the coming year are forecast as follows.

January $3,500,000February 3,800,000March 3,600,000April 4,000,000May 4,200,000

For the month of April, the total cash receipts from sales and collections on account would be

$3,729,968A.$3,781,600B.$4,025,200C.$4,408,000D.

Answer (A) is incorrect because $3,729,968 results from improperly calculating the collections of April and Marchcredit sales. The 2% uncollectible amount should not be removed from the credit sales before the collectionpercentage is applied.Answer (B) is correct. The cash receipts for April equal April’s cash sales (30% × $4,000,000 = $1,200,000), 40% ofApril’s credit sales, and 58% of March’s credit sales. Consequently, total cash receipts equal $3,781,600 [$1,200,000+ (40% × 70% × $4,000,000) + (58% × 70% × $3,600,000)].Answer (C) is incorrect because $4,025,200 includes the collection of May credit sales rather than March credit sales.Answer (D) is incorrect because $4,408,000 improperly calculates the collections of March credit sales. Thecalculation treats the entire sales figure for March as credit sales.

[96] Gleim #: 1.96 -- Source: CIA 597 I-32

Auditors must always be alert for the possibility of fraud. Assume the controls over each risk listed below are marginal.Which of the following possible frauds or misuses of organization assets should be considered the area of greatest risk?

Gleim's CIA Test Prep: Part III: Business Analysis and Information Technology(1165 questions)

Copyright 2004 Gleim Publications, Inc. Page 36Printed for g j

Page 37: Part three cia_with_ answers

The president is using company travel and entertainment funds for activities that might be considered questionable.A.Purchases of supplies are made from fictitious vendors.B.Grants are made to organizations that might be associated with the president or are not for purposes dictated in theorganization’s charter.

C.

The payroll clerk has added ghost employees.D.

Answer (A) is incorrect because administrative expense is 2% ($10 ÷ $500) of current revenue.Answer (B) is incorrect because purchases of supplies from fictitious vendors involve risk exposures that are far less thanthose arising from inappropriate grants.Answer (C) is correct. Grants represent 83.6% ($418 ÷ $500) of current revenue. Consequently, fraudulent grantsconstitute a much greater risk exposure than any of the other items listed.Answer (D) is incorrect because the payroll clerk’s addition of ghost employees involve risk exposures that are far less thanthose arising from inappropriate grants.

[97] Gleim #: 1.97 -- Source: CIA 597 III-100

The following information applies to a project:

    ImmediateActivity Time (days) Predecessor

A 5 NoneB 3 NoneC 4 AD 2 BE 6 C, D

The earliest completion time for the project is

11 days.A.14 days.B.15 days.C.20 days.D.

Answer (A) is incorrect because 11 days is the shortest, not the longest, time to completion.Answer (B) is incorrect because 14 days is not the completion time of a path to completion.Answer (C) is correct. The two paths through the network are ACE (5 + 4 + 6 = 15 days) and BDE (3 + 2 + 6 = 11days). The critical or longest path is A-C-E. Hence, the earliest completion time is 15 days.Answer (D) is incorrect because 20 days is the sum of all of the activity times.

[98] Gleim #: 1.98 -- Source: CIA 1191 III-33

The data below were gathered on two different machine centers and two products.

  Product Hours per Unit   A B Hours Available Machine Center 1 2.5 4 60Machine Center 2 6 3 70Contribution per unit $4 $5

Which item below would be part of a linear programming formulation of this problem?

Maximize: Contribution > 4A + 5B.A.Subject to: A < 0.B.Subject to: 2.5A + 4B < 60.C.Subject to: 4A + 5B < 130.D.

Gleim's CIA Test Prep: Part III: Business Analysis and Information Technology(1165 questions)

Copyright 2004 Gleim Publications, Inc. Page 37Printed for g j

Page 38: Part three cia_with_ answers

Answer (A) is incorrect because the objective function is to maximize the total contribution margin (4A + 5B). This amountis not expressed as an inequality.Answer (B) is incorrect because this equation (A < 0) is nonsensical.Answer (C) is correct. The linear programming solution is subject to constraints on the availability of machine hours inboth centers. For example, products A and B require 2.5 and 4 hours per unit, respectively, in Machine Center 1, but only60 hours are available. Hence, the optimal production of A and B to the following constraint:

2.5A + 4B < 60Answer (D) is incorrect because the hours available in the two machine centers cannot be added together. Also, the left sideof the equation states the objective function, not a constraint.

[99] Gleim #: 1.99 -- Source: CIA 594 II-40

Various tools are employed to control large scale projects. They include all of the following except:

PERT.A.CPM.B.Statistical process control.C.Gantt charts.D.

Answer (A) is incorrect because PERT breaks down a project into a set of events, arranges the events into a strictpriority network, and establishes a completion time for each event.Answer (B) is incorrect because CPM is used to control large scale projects. Essentially a subset of PERT, CPM usesa deterministic rather than a probabilistic estimate of the time required for task completion.Answer (C) is correct. Statistical process (quality) control is not used to control large scale projects. Statisticalquality control is a method of determining whether the shipment or production run of units lies within acceptablelimits. It is also used to determine whether production processes are out of control.Answer (D) is incorrect because Gantt charts compare scheduled production with actual production. Its controlfunction lies in its ability to identify variations and thus formulate corrective actions.

[100] Gleim #: 1.100 -- Source: CIA 594 III-61

Which of the following terms is not used in project management?

Dummy activity.A.Latest finish.B.Optimistic time.C.Lumpy demand.D.

Answer (A) is incorrect because a dummy activity is one that consumes no time but establishes precedence amongactivities. It is used specifically in project management.Answer (B) is incorrect because the latest finish is the latest that an activity can finish without causing delay in thecompletion of the project.Answer (C) is incorrect because optimistic time is the time for completing a project if all goes well.Answer (D) is correct. Project management concerns managing teams assigned to special projects. Lumpy demandrefers to periodic demand for a product or service that increases in large, lumpy increments.

[101] Gleim #: 1.101 -- Source: CIA 592 III-69

Activity scheduling information for the installation of a new computer system is given below.

Immediate DurationActivity Predecessor (Days)

A - 4B - 3C A 9D A 6E B, D 5

For this project, the critical path is

Gleim's CIA Test Prep: Part III: Business Analysis and Information Technology(1165 questions)

Copyright 2004 Gleim Publications, Inc. Page 38Printed for g j

Page 39: Part three cia_with_ answers

A-C.A.B-E.B.A-D-E.C.B-D-C.D.

Answer (A) is incorrect because the path length of A-C is 13.Answer (B) is incorrect because the path length of B-E is 8.Answer (C) is correct. The critical path is the longest path because it defines the minimum duration of the project. A-D-E(4 + 6 + 5 = 15) is the critical path.Answer (D) is incorrect because B-D-C is not a path. The predecessor of C is A.

[102] Gleim #: 1.102 -- Source: CIA 1191 III-37

In a critical path analysis, if slack time in an activity exists, the activity

Is not essential to the overall project.A.Is a backup activity to replace a main activity should it fail.B.Could be delayed without delaying the overall project.C.Involves essentially no time to complete.D.

Answer (A) is incorrect because an activity with slack may nevertheless be essential to the overall project.Answer (B) is incorrect because it is not a backup activity.Answer (C) is correct. Slack is the free time associated with each activity. In other words, paths that are not criticalhave slack time. Slack represents unused resources that can be diverted to the critical path.Answer (D) is incorrect because time is involved in a slack activity.

[103] Gleim #: 1.103 -- Source: CIA 1193 III-67

A company has several departments that conduct technical studies and prepare reports for clients. Recently, there have beenlong delays in having these reports copied at the company’s centralized copy center because of the dramatic increase inbusiness. Management is considering decentralizing copy services to reduce the turnaround and provide clients with timelyreports. An appropriate technique for minimizing turnaround time and the cost of providing copy services is

Queuing theory.A.Linear programming.B.Regression analysis.C.Game theory.D.

Answer (A) is correct. Two basic costs are involved in queuing (waiting-line) models: (1) the cost of providingservice (including facility costs and operating costs), and (2) the cost of idle resources waiting in line. The latter maybe a direct cost if paid employees are waiting, or an opportunity cost in the case of waiting customers. The objectiveof queuing theory is to minimize the total cost of the system, including both service and waiting costs, for a given rateof arrivals. This minimization occurs at the point at which cost of waiting is balanced by the cost of providingservice. This company wishes to reduce the total of waiting costs (turnaround time) and the cost of copy services.Answer (B) is incorrect because linear programming is a mathematical technique for optimizing a given objectivefunction subject to constraints.Answer (C) is incorrect because regression analysis is a statistical procedure for estimating the relation betweenvariables.Answer (D) is incorrect because game theory is a mathematical approach to decision-making in which each decision-maker takes into account the courses of action of competitors.

Gleim's CIA Test Prep: Part III: Business Analysis and Information Technology(1165 questions)

Copyright 2004 Gleim Publications, Inc. Page 39Printed for g j

Page 40: Part three cia_with_ answers

[Fact Pattern #9]A bank has two drive-in lanes to serve customers, one attached to the bank itself, the second on an island. One teller serves bothstations. The bank is interested in determining the average waiting times of customers and has developed a model based onrandom numbers. The two key factors are the time between successive car arrivals and the time customers wait in line.

Assume that the analysis with cars just arriving at both service windows, both requiring 3 minutes of service time. Car 1 is at theattached window and car 2 at the island window. A car will always go to the window attached to the bank unless that window hasmore cars waiting than the island window. The lone teller will always serve the car that arrived first. If two cars arrivesimultaneously, the one at the attached window will be served before the one at the island.

Based on a known probability distribution, the bank assigns random numbers to arrival and service times:

TimeRandom between Random

# arrivals: # Service time1 1 minute 1, 2 1 minute2, 3 2 minutes 3 2 minutes4, 5, 6 3 minutes 4, 5, 6 3 minutes7, 8 4 minutes 7, 8, 9 4 minutes

The bank then selects random numbers for the next two cars as Random Numbers Selected

Arrival ServiceCar 3 #3 #7Car 4 #7 #8

[104] Gleim #: 1.104 -- Source: CIA 1192 III-95

(Refers to Fact Pattern #9)The problem just described is best approached by use of which quantitative method?

Integrated autoregressive-moving average (ARIMA) modeling.A.Exponential smoothing, multiple parameters.B.Queuing theory.C.Linear programming.D.

Answer (A) is incorrect because ARIMA modeling is a times-series technique.Answer (B) is incorrect because exponential smoothing is a time-series model.Answer (C) is correct. Two basic costs are involved in queuing (waiting-line) models, (1) the cost of providingservice (including facility costs and operating costs), and (2) the cost of idle resources waiting in line. The latter maybe a direct cost if paid employees are waiting, or an opportunity cost in the case of waiting customers. The objectiveof the queuing theory is to minimize the total cost of the system, including both service and waiting costs, for a givenrate of arrivals. This minimization occurs at the point where the cost of waiting is balanced by the cost of providingservice.Answer (D) is incorrect because linear programming optimizes outputs given scarce resources.

[105] Gleim #: 1.105 -- Source: CIA 1192 III-96

(Refers to Fact Pattern #9)The technique used in analyzing the problem is best described as

Simulation theory.A.Integrated autoregressive-moving average (ARIMA) modeling.B.Linear programming.C.Differential calculus.D.

Gleim's CIA Test Prep: Part III: Business Analysis and Information Technology(1165 questions)

Copyright 2004 Gleim Publications, Inc. Page 40Printed for g j

Page 41: Part three cia_with_ answers

Answer (A) is correct. Simulation is a technique for experimenting with logical/mathematical models using a computer.Despite the power of mathematics, many problems cannot be solved by known analytical methods because of the behaviorof the variables and the complexity of their interactions. However, the performance of a quantitative model underuncertainty may be investigated by randomly selecting values for each of the variables in the model (based on theprobability distribution of each variable) and then calculating the value of the solution. If this process is performed a largenumber of times, the distribution of results from the model will be obtained.Answer (B) is incorrect because ARIMA modeling is a time-series technique.Answer (C) is incorrect because linear programming optimizes outputs given scarce resources.Answer (D) is incorrect because differential calculus is used to establish optimization points.

[106] Gleim #: 1.106 -- Source: CIA 1192 III-97

(Refers to Fact Pattern #9)The process of making sure that the model measures what it is supposed to measure is called

Statistical inference.A.Hypothesis testing.B.Confidence coefficient analysis.C.Validation.D.

Answer (A) is incorrect because statistical inference refers to estimation or hypothesis testing.Answer (B) is incorrect because hypothesis testing relates to the testing of specific null hypotheses.Answer (C) is incorrect because confidence coefficients are an integral part of statistical estimation and hypothesistesting.Answer (D) is correct. Validation is a step in the simulation procedure. Some assurance is needed that the results ofthe experiment will be realistic. This assurance requires validation of the model -- often using historical data. If themodel gives results equivalent to what actually happened, the model is historically valid. There is still some risk,however, that changes could make the model invalid for the future.

[107] Gleim #: 1.107 -- Source: CIA 1193 III-71

The marketing department of your company is deciding on the price to charge for a key product. In setting this price,marketing needs to consider the price that a major competitor will charge for a similar product because the competitor’sprice will affect the demand for your company’s product. Similarly, in setting its price, the competitor will consider whatyour company will charge. An appropriate mathematical technique for analyzing such a decision is

Game theory.A.Probability theory.B.Linear programming.C.Sensitivity analysis.D.

Answer (A) is correct. Game (or decision) theory is a mathematical approach to decision making when confrontedwith an enemy or competitor. Games are classified according to the number of players and the algebraic sum of thepayoffs. In a two-person game, if the payoff is given by the loser to the winner, the algebraic sum is zero and thegame is called a zero-sum game. If it is possible for both players to profit, however, the game is a positive-sum game.Mathematical models have been developed to select optimal strategies for certain simple games.Answer (B) is incorrect because probability theory is a mathematical technique used to express quantitatively thelikelihood of occurrence of an event.Answer (C) is incorrect because linear programming is a mathematical technique for optimizing a given objectivesubject to certain constraints.Answer (D) is incorrect because sensitivity analysis is a method for studying the effects of changes in one or morevariables on the results of a decision model.

[108] Gleim #: 1.108 -- Source: CIA 592 III-64

Because of the large number of factors that could affect the demand for its new product, interactions among these factors,and the probabilities associated with different values of these factors, the marketing department would like to develop acomputerized model for projecting demand for this product. By using a random-number procedure to generate values forthe different factors, it will be able to estimate the distribution of demand for this new product. This method of estimatingthe distribution of demand for the new product is called

Gleim's CIA Test Prep: Part III: Business Analysis and Information Technology(1165 questions)

Copyright 2004 Gleim Publications, Inc. Page 41Printed for g j

Page 42: Part three cia_with_ answers

Monte Carlo simulation.A.Linear programming.B.Correlation analysis.C.Differential analysis.D.

Answer (A) is correct. Simulation is a technique in which a probabilistic process is first modeled. The inputs to the modelare then varied a large number of times to estimate the distribution of possible outcomes from the model of the variable ofinterest. Simulations that use a random-number procedure to generate values for the inputs are referred to as Monte Carlosimulations.Answer (B) is incorrect because linear programming is a mathematical technique for maximizing or minimizing a givenobjective subject to certain constraints.Answer (C) is incorrect because correlation analysis is a statistical procedure for studying the relations among variables.Answer (D) is incorrect because differential analysis is a method used for decision-making that compares differences incosts (and revenues) of two or more possibilities.

[109] Gleim #: 1.109 -- Source: CIA 1190 III-41

An account executive has just designed a Monte Carlo model to estimate the costs of a particular type of project. Validatingthe model could include all except

Checking for errors in the computer programming.A.Checking that assumed probability distributions are reasonable.B.Comparing test results with previously validated models.C.Applying the model.D.

Answer (A) is incorrect because it is part of the validation process.Answer (B) is incorrect because it is part of the validation process.Answer (C) is incorrect because it is part of the validation process.Answer (D) is correct. The Monte Carlo technique is used in a simulation to generate the individual values for arandom value. An essential step in the simulation procedure is to validate the mathematical model used. This processinvolves not only searching for errors but also verifying the assumptions. It also should provide some assurance thatthe results of the experiment will be realistic. This assurance is often obtained using historical data. If the model givesresults equivalent to what actually happened, the model is historically valid. There is still some risk, however, thatchanges could make the model invalid for the future. The model should not be implemented until this validationprocess is complete.

[110] Gleim #: 1.110 -- Source: CIA 593 III-65

A large fishing operation has information on the interval, time, and probabilities of shrimp schools staying beneath theirfishing boats. In order to use this information to predict when and where to send their boats, which of the followingtechniques should be used?

Simulation.A.Least squares.B.Queuing theory.C.Exponential smoothing.D.

Answer (A) is correct. Simulation (Monte Carlo simulation) is a technique for experimenting withlogical/mathematical models using a computer. Despite the power of mathematics, many problems cannot be solvedby known analytical methods because of the behavior of the variables and the complexity of their interactions.However, the performance of a quantitative model under uncertainty may be investigated by randomly selectingvalues for each variable in the model (based on its probability distribution) and then calculating the value of thesolution. If this process is performed many times, the distribution of results from the model will be obtained.Answer (B) is incorrect because least squares is a prediction and estimation technique using a single dependent andsingle or multiple independent variables.Answer (C) is incorrect because queuing theory is a waiting-line technique used to balance desirable service levelsagainst the cost of providing more service.Answer (D) is incorrect because exponential smoothing is a forecasting technique using arbitrary weights.

[111] Gleim #: 1.111 -- Source: CIA 594 III-60

Which of the following is not true about simulation models?

Gleim's CIA Test Prep: Part III: Business Analysis and Information Technology(1165 questions)

Copyright 2004 Gleim Publications, Inc. Page 42Printed for g j

Page 43: Part three cia_with_ answers

They are deterministic in nature.A.The may involve sampling.B.They mathematically estimate what actual performance would be.C.They emulate stochastic systems.D.

Answer (A) is correct. Simulation is a technique for experimenting with logical/mathematical models using a computer.The simulation procedure has five steps: define the objectives, formulate the model, validate the model, design theexperiment, and conduct the simulation and evaluate the results. A simulation uses the laws of probability to generate valuesfor random variables. Thus, simulation models are probabilistic, not deterministic.Answer (B) is incorrect because simulation modeling samples the operation of a system.Answer (C) is incorrect because simulation models mathematically estimate what performance would be under variousconditions.Answer (D) is incorrect because simulation models are by definition stochastic or probabilistic models.

[112] Gleim #: 1.112 -- Source: CIA 593 III-66

A cost-volume-profit model developed in a dynamic environment determined that the estimated parameters used may varybetween limits. Subsequent testing of the model with respect to all possible values of the estimated parameters is termed

A sensitivity analysis.A.Statistical estimation.B.Statistical hypothesis testing.C.A time-series study.D.

Answer (A) is correct. After a problem has been formulated into any mathematical model, it may be subjected tosensitivity analysis. Sensitivity analysis is a method for studying the effects of changes in one or more variables onthe results of a decision model.Answer (B) is incorrect because statistical estimation involves the estimation of parameters.Answer (C) is incorrect because statistical hypothesis testing calculates the conditional probability that both thehypothesis is true and the sample results have occurred.Answer (D) is incorrect because a time-series study involves forecasting data over time.

[113] Gleim #: 1.113 -- Source: CIA 1194 III-58

An investment company is attempting to allocate its available funds between two investment alternatives, stocks and bonds,which differ in terms of expected return and risk. The company would like to minimize its risk while earning an expectedreturn of at least 10% and investing no more than 70% in either of the investment alternatives. An appropriate technique forallocating its funds between stocks and bonds is

Linear programming.A.Capital budgeting.B.Differential analysis.C.Queuing theory.D.

Answer (A) is correct. Linear programming is a mathematical technique for planning resource allocation thatoptimizes a given objective function that is subject to certain constraints. In this case, the maximum investment isconstrained by a 70% limit on either investment choice.Answer (B) is incorrect because capital budgeting is used to analyze and evaluate long-term capital investments.Answer (C) is incorrect because differential analysis is used for decision making when differences in costs (revenues)for two or more options are compared.Answer (D) is incorrect because queuing theory is used to minimize the sum of the costs of waiting lines andservicing waiting lines when items arrive randomly at a service point and are serviced sequentially.

[114] Gleim #: 1.114 -- Source: CIA 1194 III-61

A company is deciding whether to purchase an automated machine to manufacture one of its products. Expected net cashflows from this decision depend on several factors, interactions among those factors, and the probabilities associated withdifferent levels of those factors. The method that the company should use to evaluate the distribution of net cash flows fromthis decision and changes in net cash flows resulting from changes in levels of various factors is

Gleim's CIA Test Prep: Part III: Business Analysis and Information Technology(1165 questions)

Copyright 2004 Gleim Publications, Inc. Page 43Printed for g j

Page 44: Part three cia_with_ answers

Simulation and sensitivity analysis.A.Linear programming.B.Correlation analysis.C.Differential analysis.D.

Answer (A) is correct. Simulation is a technique used to describe the behavior of a real-world system over time. Thistechnique usually employs a computer program to perform the simulation computations. Sensitivity analysis examines howoutcomes change as the model parameters change.Answer (B) is incorrect because linear programming is a mathematical technique for optimizing a given objective functionsubject to certain constraints.Answer (C) is incorrect because correlation analysis is a statistical procedure for studying the relation between variables.Answer (D) is incorrect because differential analysis is used for decision making that compares differences in costs(revenues) of two or more options.

[115] Gleim #: 1.115 -- Source: CIA 1196 III-8

In forecasting purchases of inventory for a firm, all of the following are useful except

Knowledge of the behavior of business cycles.A.Internal allocations of costs to different segments of the firm.B.Information on the seasonal variations in demand.C.Econometric modeling.D.

Answer (A) is incorrect because, in time series analysis, the cyclical fluctuation in business activity is usuallyincorporated as an index number in the forecasting model.Answer (B) is correct. Internal allocations of costs relate to costs already incurred, that is, to sunk costs. Sunk costsare not relevant to decision making, for example, to forecasting future purchases.Answer (C) is incorrect because, in time series analysis, the seasonal variation is usually incorporated as an indexnumber in the forecasting model.Answer (D) is incorrect because an econometric model is an application of statistical methods to economic problems.Such a model is used as a forecasting tool.

[116] Gleim #: 1.116 -- Source: CMA 689 5-24

The primary difference between PERT and CPM is that

CPM uses probabilities on the activity times and PERT does not.A.PERT considers activity costs and CPM does not.B.PERT can assign probabilities to activity times and CPM does not.C.CPM considers activity costs and PERT does not.D.

Answer (A) is incorrect because CPM specifies the activity times (uses deterministic estimates).Answer (B) is incorrect because CPM but not PERT uses activity costs and considers crash times.Answer (C) is incorrect because a less significant difference between PERT and CPM is that PERT uses probabilisticestimates of completion times. CPM times are deterministic. The 1:4:1 method is typically used in PERT. Under thismethod, the most optimistic and pessimistic estimates are weighted equally, but the most likely estimate is weightedfour times more heavily than the others.Answer (D) is correct. Both PERT and CPM are network analysis techniques. But CPM was developedindependently of PERT and is widely used in the construction industry. CPM may be thought of as a subset of PERT.Like PERT, it is a network technique, but, unlike PERT, it uses deterministic time and cost estimates. Its advantagesinclude cost estimates plus the concept of "crash" efforts and costs. Activity times are estimated for normal effort andcrash effort. Crash time is the time to complete an activity assuming that all available resources were devoted to thetask (overtime, extra crew, etc.). Activity costs are also estimated for normal and crash efforts. These estimates allowthe project manager to estimate the costs of completing the project if some of the activities are completed on a crashbasis. The network diagram is constructed in the same manner as PERT diagrams. Once the diagram is constructed,the critical paths are found for normal and crash times. More than one critical path may exist for each diagram.

[117] Gleim #: 1.117 -- Source: CMA 689 5-25

A Gantt chart

Gleim's CIA Test Prep: Part III: Business Analysis and Information Technology(1165 questions)

Copyright 2004 Gleim Publications, Inc. Page 44Printed for g j

Page 45: Part three cia_with_ answers

Shows the critical path for a project.A.Is used for determining an optimal product mix.B.Shows only the activities along the critical path of a network.C.Does not necessarily show the critical path through a network.D.

Answer (A) is incorrect because the critical path is not shown.Answer (B) is incorrect because linear programming is used to determine an optimal product mix.Answer (C) is incorrect because a Gantt chart shows the activities to be completed but not their relationships (sequencing).Answer (D) is correct. A Gantt or bar chart is sometimes used in conjunction with PERT or CPM to show the progress of aspecial project. Time is shown on the horizontal axis, the length of a bar equals the length of an activity, and shadingindicates the degree of completion. However, the Gantt chart is not as sophisticated as PERT or CPM in that it does notreflect the relationships among the activities or define a critical path.

[118] Gleim #: 1.118 -- Source: CMA 697 4-26

A regression equation

Estimates the dependent variables.A.Encompasses factors outside the relevant range.B.Is based on objective and constraint functions.C.Estimates the independent variable.D.

Answer (A) is correct. Regression analysis is used to find an equation for the linear relationship among variables.The behavior of the dependent variable is explained in terms of one or more independent variables. Regressionanalysis is often used to estimate a dependent variable (such as cost) given a known independent variable (such asproduction).Answer (B) is incorrect because regression results are limited to observations within the relevant range.Answer (C) is incorrect because regression analysis does not use constraint functions.Answer (D) is incorrect because the dependent variable is estimated using regression analysis.

[119] Gleim #: 1.119 -- Source: CMA 1285 5-27

The correlation coefficient that indicates the weakest linear association between two variables is

-0.73A.-0.11B.0.12C.0.35D.

Answer (A) is incorrect because -0.73 signifies a strong negative correlation.Answer (B) is correct. The correlation coefficient can vary from -1 to +1. A -1 relationship indicates a perfectnegative correlation, and a +1 relationship indicates a perfect positive correlation. A zero correlation coefficientwould indicate no association between the variables. Thus, the correlation coefficient that is nearest to zero wouldindicate the weakest linear association. Of the options given in the question, the correlation coefficient that is nearestto zero is -0.11.Answer (C) is incorrect because 0.12 is a slightly stronger correlation.Answer (D) is incorrect because 0.35 is a considerably stronger correlation.

[120] Gleim #: 1.120 -- Source: CMA 1289 5-14

Correlation is a term frequently used in conjunction with regression analysis and is measured by the value of the coefficientof correlation, r. The best explanation of the value r is that it

Is always positive.A.Interprets variances in terms of the independent variable.B.Ranges in size from negative infinity to positive infinity.C.Is a measure of the relative relationship between two variables.D.

Gleim's CIA Test Prep: Part III: Business Analysis and Information Technology(1165 questions)

Copyright 2004 Gleim Publications, Inc. Page 45Printed for g j

Page 46: Part three cia_with_ answers

Answer (A) is incorrect because the coefficient is negative if the relationship between the variables is inverse.Answer (B) is incorrect because the coefficient relates the two variables to each other.Answer (C) is incorrect because the size of the coefficient varies between -1.0 and +1.0.Answer (D) is correct. The coefficient of correlation (r) measures the strength of the linear relationship between thedependent and independent variables. The magnitude of r is independent of the scales of measurement of x and y. Thecoefficient lies between -1.0 and +1.0. A value of zero indicates no linear relationship between the x and y variables. Avalue of +1.0 indicates a perfectly direct relationship, and a value of -1.0 indicates a perfectly inverse relationship.

[121] Gleim #: 1.121 -- Source: CMA 1293 4-25

The four components of time series data are secular trend, cyclical variation, seasonality, and random variation. Theseasonality in the data can be removed by

Multiplying the data by a seasonality factor.A.Ignoring it.B.Taking the weighted average over four time periods.C.Subtracting a seasonality factor from the data.D.

Answer (A) is incorrect because adding a seasonality factor to, or subtracting it from, a forecast based on trendanalysis is a means of adjusting for seasonality.Answer (B) is incorrect because seasonality factors cannot be ignored; they are reflected in the data and must beconsidered for a model to be accurate.Answer (C) is correct. Time series analysis relies on past experience. Changes in the value of a variable may haveseveral possible components including secular trends, cyclical variation, seasonality, and random variation. Seasonalvariations are common in many businesses. A variety of methods exist for including seasonal variations in aforecasting model, but most methods use a seasonal index. Alternatively, seasonal variations can be removed fromdata by using a weighted average of several time periods instead of data from individual periods.Answer (D) is incorrect because the seasonality adjustment for a single season’s data may be an increase or adecrease.

[122] Gleim #: 1.122 -- Source: CMA 696 4-20

A widely used approach that managers use to recognize uncertainty about individual items and to obtain an immediatefinancial estimate of the consequences of possible prediction errors is

Expected value analysis.A.Learning curve analysis.B.Sensitivity analysis.C.Regression analysis.D.

Answer (A) is incorrect because expected value is the probabilistically weighted average of the outcomes of anaction.Answer (B) is incorrect because learning curve analysis quantifies how labor costs decline as employees learn theirjobs through repetition.Answer (C) is correct. Sensitivity analysis determines how a result varies with changes in a given variable orparameter in a mathematical decision model. For example, in a present value analysis, a manager might first calculatethe net present value or internal rate of return assuming that a new asset has a 10-year life. The NPV or IRR can thenbe recalculated using a 5-year life to determine how sensitive the result is to the change in the assumption.Answer (D) is incorrect because regression, or least squares, analysis determines the average change in the dependentvariable given a unit change in one or more independent variables.

Gleim's CIA Test Prep: Part III: Business Analysis and Information Technology(1165 questions)

Copyright 2004 Gleim Publications, Inc. Page 46Printed for g j

Page 47: Part three cia_with_ answers

[Fact Pattern #10]An international nonprofit organization finances medical research. The majority of its revenue and support comes from fund-raising activities, investments, and specific grants from an initial sponsoring corporation. The organization has been in operationover 15 years and has a small internal auditing department. The organization has just finished a major fund-raising drive thatraised $500 million for the current fiscal period.

The following are selected data from recent financial statements (dollar figures in millions):

  Current Past  Year Year Revenue $500 $425Investments (average balances) 210 185Medical research grants made 418 325Investment income 16 20Administrative expense 10 8

[123] Gleim #: 1.123 -- Source: CIA 597 I-31

(Refers to Fact Pattern #10)The auditor wishes to determine if the change in investment income during the current year was due to (a) changes ininvestment strategy, (b) changes in portfolio mix, or (c) other factors. Which of the following analytical review proceduresshould the auditor use?

Simple linear regression that compares investment income changes over the past 5 years to determine the nature ofthe changes.

A.

Ratio analysis that compares changes in the investment portfolio on a monthly basis.B.Trend analysis that compares the changes in investment income as a percentage of total assets and of investmentassets over the past 5 years.

C.

Multiple regression analysis that includes independent variables related to the nature of the investment portfolio andmarket conditions.

D.

Answer (A) is incorrect because simple linear regression is based on just one independent variable.Answer (B) is incorrect because ratio analysis measure changes but do not explain them.Answer (C) is incorrect because trend analysis measure changes but do not explain them.Answer (D) is correct. Regression analysis develops an equation to explain the behavior of a dependent variable (forexample, investment income) in terms of one or more independent variables (for example, market risk and the risks ofparticular investments). Multiple regression analysis is the best approach because it allows the auditor to regress thechange in investment income on more than one independent variable.

[124] Gleim #: 1.124 -- Source: CIA R98 III-35

(Refers to Fact Pattern #10)Which of the following will allow a manufacturer with limited resources to maximize profits?

The Delphi technique.A.Exponential smoothing.B.Regression analysis.C.Linear programming.D.

Gleim's CIA Test Prep: Part III: Business Analysis and Information Technology(1165 questions)

Copyright 2004 Gleim Publications, Inc. Page 47Printed for g j

Page 48: Part three cia_with_ answers

Answer (A) is incorrect because the Delphi technique is an approach in which the manager solicits opinions on a problemfrom experts, summarizes the opinions, and feeds the summaries back to the experts (without revealing any of theparticipants to each other). The process is reiterated until the opinions converge on an optimal solution.Answer (B) is incorrect because under exponential smoothing, each forecast equals the sum of the last observation times thesmoothing constant, plus the last forecast times one minus the constant. Thus, exponential means that greater weight isplaced on the most recent data, with the weights of all data falling off exponentially as the data age.Answer (C) is incorrect because regression analysis is used to fit a linear trend line to a dependent variable based on one ormore independent variables.Answer (D) is correct. Linear programming is a technique used to optimize an objective function, that is, to maximize arevenue or profit function or to minimize a cost function, subject to constraints, e.g., limited (scarce) resources orminimum/maximum levels of production, performance, etc. In business, linear programming is used for planning resourceallocations. Managers are often faced with problems of selecting the most profitable or least costly way to use availableresources.

[125] Gleim #: 1.125 -- Source: CIA R98 III-36

Which of the following is not an appropriate time series forecasting technique?

Least squares.A.Exponential smoothing.B.The Delphi technique.C.Moving averages.D.

Answer (A) is incorrect because regression (least squares) analysis extends correlation to find an equation for thelinear relationship among variables. The behavior of the dependent variable is explained in terms of one or moreindependent variables. Thus, regression analysis determines functional relationships among quantitative variables.Answer (B) is incorrect because, under exponential smoothing, each forecast equals the sum of the last observationtimes the smoothing constant, plus the last forecast times one minus the constant. Thus, exponential means thatgreater weight is placed on the most recent data, with the weights of all data falling off exponentially as the data age.Answer (C) is correct. The Delphi technique is an approach in which the manager solicits opinions on a problemfrom experts, summarizes the opinions, and feeds the summaries back to the experts (without revealing any of theparticipants to each other). The process is reiterated until the opinions converge on an optimal solution. Thus, theDelphi technique is a qualitative, not a quantitative, method.Answer (D) is incorrect because a moving average is the simplest form of smoothing. Each forecast is based on afixed number of prior observations.

[126] Gleim #: 1.126 -- Source: CIA 1184 III-12

Which of the following observations concerning quality control is more accurate?

Process quality control is used when examining the quality of goods or services which already exist.A.Process quality control is used to prevent defects, whereas product quality control is used primarily to identify defectsafter the fact.

B.

Product quality control is designed to balance the marketability of higher quality against the cost of attaining higherquality.

C.

Product quality control is more important than process quality control.D.

Answer (A) is incorrect because it describes product (not process) quality control.Answer (B) is correct. Process analysis studies the means of producing a product with a view to lowering costs andincreasing efficiency while producing items of appropriate quality. The effect of process analysis is to act as a pre-control to prevent defects. Product quality control involves inspection during production to eliminate defective items.Answer (C) is incorrect because it is applicable to process (not product) quality control.Answer (D) is incorrect because process quality control and product quality control are both important elements in aquality control program.

[127] Gleim #: 1.127 -- Source: CIA 594 III-53

Which statement best describes Total Quality Management (TQM)?

TQM emphasizes reducing the cost of inspection.A.TQM emphasizes participation by all employees in the decision-making process.B.TQM emphasizes encouraging cross-functional teamwork.C.TQM emphasizes doing each job right the first time.D.

Gleim's CIA Test Prep: Part III: Business Analysis and Information Technology(1165 questions)

Copyright 2004 Gleim Publications, Inc. Page 48Printed for g j

Page 49: Part three cia_with_ answers

Answer (A) is incorrect because TQM has a broader emphasis. It focuses on improving quality, reducing cycle time,providing increased customer satisfaction, and achieving the lowest overall business cost. Reducing the cost of inspectionhelps achieve the lowest overall business cost.Answer (B) is incorrect because this statement describes participative management.Answer (C) is incorrect because TQM has a broader emphasis. It focuses on improving quality, reducing cycle time,providing increased customer satisfaction, and achieving the lowest overall business cost. Encouraging cross-functionalteamwork helps achieve the lowest overall business cost.Answer (D) is correct. TQM establishes quality as an organizational objective and views it as a major component of theorganization’s service to its customers. It emphasizes employee training and commitment, product/service design andproduction, and customer service. Ordinarily, the quality of a product or service is as important to customers as cost andtimeliness. Superior product quality is not attained merely through more inspection, better statistical quality control, andcross-functional teamwork. Manufacturers must make fundamental changes in the way they produce products and do eachjob right the first time.

[128] Gleim #: 1.128 -- Source: CIA 1191 III-11

The operations manager of a company notified the treasurer of that organization 60 days in advance that a new, expensivepiece of machinery was going to be purchased. This notification allowed the treasurer to make an orderly liquidation ofsome of the company’s investment portfolio on favorable terms. What type of control was involved?

Feedback.A.Strategic.B.Budgetary.C.Feedforward.D.

Answer (A) is incorrect because feedback controls apply to decision making based on evaluations of pastperformance.Answer (B) is incorrect because strategic controls are broad-based and affect an organization over a long period.They apply to such long-term variables as quality and R&D.Answer (C) is incorrect because control of budgeted expenditures is not mentioned in the example.Answer (D) is correct. Feedforward control provides for the active anticipation of problems so that they can beavoided or resolved in a timely manner. Another example is the quality control inspection of raw materials and work-in-process to avoid defective finished goods.

[129] Gleim #: 1.129 -- Source: CIA 592 III-17

As part of a Total Quality Control program, a firm not only inspects finished goods but also monitors product returns andcustomer complaints. Which type of control best describes these efforts?

Feedback control.A.Feedforward control.B.Production control.C.Inventory control.D.

Answer (A) is correct. A feedback control measures actual performance, something that has already occurred, toensure that a desired future state is attained. It is used to evaluate the past to improve future performance. Inspectingfinished goods, monitoring product returns, and evaluating complaints are post-action controls intended to eliminatedeviations in future cycles of the process under control.Answer (B) is incorrect because feedforward controls anticipate problems before they occur.Answer (C) is incorrect because customer complaints are not part of production control.Answer (D) is incorrect because TQC, which is often associated with JIT systems but is applicable to otherenvironments, is a philosophy emphasizing that each member of the organization is a quality control inspector. It de-emphasizes the role of separate quality control departments by making each employee responsible for his/her work.Hence, TQC is not limited to inventory control.

[130] Gleim #: 1.130 -- Source: CIA 1191 III-10

To be successful, large companies must develop means to keep the organization focused in the proper direction.Organizational control systems help keep companies focused. These control systems consist of which three basiccomponents?

Gleim's CIA Test Prep: Part III: Business Analysis and Information Technology(1165 questions)

Copyright 2004 Gleim Publications, Inc. Page 49Printed for g j

Page 50: Part three cia_with_ answers

Budgeting, financial ratio analysis, and cash management.A.Objectives, standards, and an evaluation-reward system.B.Role analysis, team building, and survey feedback.C.Setting goals, empowering employees, and job enrichment.D.

Answer (A) is incorrect because budgeting, financial ratio analysis, and cash management are means of financial control.Answer (B) is correct. An organizational control system must be based on a statement of what is to be achieved and when(objectives). The accomplishment of objectives depends on the establishment of standards that serve as feedforwardcontrols. Adherence to performance standards signifies that an organization is on course to attain its objectives. A well-developed evaluation-reward system that is clearly communicated to employees should motivate them to improvedperformance.Answer (C) is incorrect because role analysis, team building, and survey feedback are used in developmental efforts tocorrect organizational deficiencies.Answer (D) is incorrect because goal setting, empowerment, and job enrichment are all motivational strategies.

[131] Gleim #: 1.131 -- Source: CIA 588 III-12

Performance appraisal systems might use any of three different approaches: (1) who did the job, (2) how the job was done,or (3) what was accomplished. Which approach is used by a system that places the focus on how the job was done?

Behavior-oriented.A.Goal-oriented.B.Trait-oriented.C.Employee-oriented.D.

Answer (A) is correct. Behavior-oriented performance evaluation rewards the behavior that is desired bymanagement. Behavior control involves examining work processes rather than work output.Answer (B) is incorrect because the goal-oriented approach measures how well the employee attained the objectivesor goals set by management.Answer (C) is incorrect because a trait-oriented approach tends to reward what the supervisor thinks of the employeerather than the job the employee did.Answer (D) is incorrect because an employee-oriented approach would focus on who did the job.

[132] Gleim #: 1.132 -- Source: CIA 592 III-7

Identify the management technique in which employees assist in setting goals, making decisions, solving problems, anddesigning and implementing organizational changes.

Total quality control.A.Job enlargement.B.Kanban.C.Participative management.D.

Answer (A) is incorrect because TQC is a quality control program in which everyone sees quality control as his/herjob.Answer (B) is incorrect because job enlargement refers to increasing the number of tasks one must perform.Answer (C) is incorrect because kanban is a manual inventory technique used in just-in-time inventory controlsystems.Answer (D) is correct. According to Marshall Soshkin ["Participative Management is an Ethical Imperative,"Organizational Dynamics 12 (Spring 1984): 4-22] employees may participate in goal setting, decision making,problem solving, and designing and implementing change. Subject to various individual (values and attitudes),organizational (e.g., job design and company culture), and environmental (such as technological change andcompetition) contingencies, workers who participate in the aforementioned ways should experience greater autonomyand find their tasks more meaningful. In turn, this experience should lead to satisfaction, challenge, security, andacceptance and commitment. Ultimate results are innovation and improved performance and productivity.

[133] Gleim #: 1.133 -- Source: Publisher

Is PERT analysis a control tool or a planning tool?

Gleim's CIA Test Prep: Part III: Business Analysis and Information Technology(1165 questions)

Copyright 2004 Gleim Publications, Inc. Page 50Printed for g j

Page 51: Part three cia_with_ answers

  Planning Control A. Yes NoB. Yes YesC. No YesD. No No

Answer (A) is incorrect because PERT is a control tool as well as a planning tool.Answer (B) is correct. PERT (Program Evaluation Review Technique) is applied in the management of complex projects. Itanalyzes the project in terms of its component activities and determines their sequencing and timing. Thus, it systematicallycombines planning with control.Answer (C) is incorrect because PERT is a planning tool.Answer (D) is incorrect because PERT is a planning tool.

[134] Gleim #: 1.134 -- Source: CIA 1196 III-60

ISO 9000 standards for ring networks include fault management, configuration management, accounting management,security management, and performance monitoring. Which of the following controls is included in the performance-monitoring standards?

Reporting the failure of network fiber-optic lines.A.Recording unauthorized access violations.B.Compiling statistics on the number of times that application software is used.C.Allocating network costs to system users of the network.D.

Answer (A) is incorrect because the failure of network fiber-optic lines is a fault management control.Answer (B) is incorrect because recording unauthorized access violations is a security management control.Answer (C) is correct. The ISO 9000 performance-monitoring standards pertain to management’s ongoingassessment of the quality of performance over time. Recording software usage is a performance-monitoring controlconcerned with the extent and efficiency of network software use.Answer (D) is incorrect because allocating network costs to system users of the network is an accountingmanagement control.

[135] Gleim #: 1.135 -- Source: CIA 596 III-32

If a company is customer-centered, its customers are defined as

Only people external to the company who have purchased something from the company.A.Only people internal to the company who directly use its product.B.Anyone external to the company and those internal who rely on its product to get their job done.C.Everybody external to the company who is currently doing, or may in the future do, business with the company.D.

Answer (A) is incorrect because customers are internal as well as external.Answer (B) is incorrect because customers are internal as well as external.Answer (C) is correct. One of the tenets of TQM is customer orientation, whether the customer is internal orexternal. An internal customer is a member of the organization who relies on another member’s work to accomplishhis/her task.Answer (D) is incorrect because customers are internal as well as external.

[136] Gleim #: 1.136 -- Source: CIA 596 III-20

Which one of the following is not a characteristic of an innovative manufacturing company?

Emphasis on continuous improvement.A.Responsiveness to the changing manufacturing environment.B.Emphasis on existing products.C.Improved customer satisfaction through product quality.D.

Gleim's CIA Test Prep: Part III: Business Analysis and Information Technology(1165 questions)

Copyright 2004 Gleim Publications, Inc. Page 51Printed for g j

Page 52: Part three cia_with_ answers

Answer (A) is incorrect because continuous improvement is important for achieving and maintaining high levels ofperformance.Answer (B) is incorrect because more and more manufacturers are automating to achieve high quality, deliver customizedproducts on time, minimize inventory, and increase flexibility.Answer (C) is correct. Innovative companies are customer driven. Because customers demand ever better quality andcompetitors are attempting to provide that quality, continuous improvement (called kaizen by the Japanese) is essential forsuch companies. Thus, the flow of innovative products and services must be continuous. Simply emphasizing existingproducts is not an effective strategy for most organizations.Answer (D) is incorrect because customer satisfaction is the highest priority according to modern management practice.

[137] Gleim #: 1.137 -- Source: CIA 593 III-11

A leader who is able to gain compliance from a group based solely on personal attraction is said to have

Reward power.A.Coercive power.B.Referent power.C.Legitimate power.D.

Answer (A) is incorrect because reward power is based on a person’s ability to grant benefits.Answer (B) is incorrect because coercive power is rooted in the fear or threat of punishment.Answer (C) is correct. Referent power is based on identification of subordinates with a superior. Thus, personalmagnetism (charisma) may be a basis for influencing others to comply with a manager’s directives.Answer (D) is incorrect because legitimate power is based on a person’s superior position.

[138] Gleim #: 1.138 -- Source: CIA 594 III-57

Which of the following is an example of an efficiency measure?

The rate of absenteeism.A.The goal of becoming a leading manufacturer.B.The number of insurance claims processed per day.C.The goal of increasing market share.D.

Answer (A) is incorrect because the rate of absenteeism does not compare input and output.Answer (B) is incorrect because the goal of becoming a leading manufacturer concerns effectiveness, not efficiency.Answer (C) is correct. An organizational structure is efficient if it facilitates the accomplishment of organizationalobjectives with minimum resources and fewest unsought consequences. An efficient organizational structuremaximizes output for a given amount of input. Thus, an efficiency measure compares input to output. Insuranceclaims processed per day relates output (claims processed) to input (a day’s work).Answer (D) is incorrect because the goal of increasing market share concerns effectiveness, not efficiency.

[139] Gleim #: 1.139 -- Source: Publisher

Goal congruence is

The desire and the commitment to achieve a specific goal.A.The sharing of goals by supervisors and subordinates.B.The extent to which individuals have the authority to make decisions.C.The extent of the attempt to accomplish a specific goal.D.

Answer (A) is incorrect because motivation is the desire and the commitment to achieve a specific goal.Answer (B) is correct. Goal congruence is agreement on the goals of the organization and/or the segment by bothsupervisors and subordinates. Performance is assumed to be optimized when there is an understanding that personaland segmental goals be consistent with those of the organization.Answer (C) is incorrect because autonomy is the extent to which individuals have the authority to make decisions.Answer (D) is incorrect because managerial effort is the extent of the attempt to accomplish a specific goal.

[140] Gleim #: 1.140 -- Source: Publisher

A budget is often the result of a management-by-objectives (MBO) program. A characteristic of MBO is

Gleim's CIA Test Prep: Part III: Business Analysis and Information Technology(1165 questions)

Copyright 2004 Gleim Publications, Inc. Page 52Printed for g j

Page 53: Part three cia_with_ answers

Development of a single measure of employee performance.A.Statement of objectives in general terms.B.Establishment of objectives through both top-down and bottom-up processes.C.A flexible time frame for achievement of objectives.D.

Answer (A) is incorrect because multiple measures are preferable.Answer (B) is incorrect because objectives should be specific.Answer (C) is correct. Budgets and standards are often the result of a formal management-by-objectives program. MBO is atop-down process because the organization’s objectives are successively restated into objectives for each lower level.However, it is also a bottom-up process because of the participation of subordinates.Answer (D) is incorrect because a specific time period is established.

[141] Gleim #: 1.141 -- Source: Publisher

The primary variable affecting active participation in and commitment to the budgeting and control system is

Management efforts to achieve the budget rather than optimize results.A.Rigid adherence to the budget without recognizing changing conditions.B.Top management involvement in support of the budget.C.The opportunity budgeting gives to managers for departmental growth.D.

Answer (A) is incorrect because “managing to a budget” can result in suboptimal achievement.Answer (B) is incorrect because rigid adherence to the budget could prevent a manager from taking an appropriateaction and thus miss a profitable opportunity.Answer (C) is correct. Top management involvement in support of the budget/control system is absolutely vital forthe continued success of the operation. The attitude of top management will affect the implementation of the budgetand control system because lower level management will recognize and reflect top management’s attitude.Answer (D) is incorrect because budgeting is not directly related to departmental growth. A budget should promoteoverall company strategies, missions, objectives, and policies.

[142] Gleim #: 1.142 -- Source: Publisher

The major disadvantage of a budget produced by means of a top-down process is

Impairment of goal congruence.A.Lack of involvement by upper-level management.B.Inconsistency with strategic plans.C.Absence of a significant motivational effect.D.

Answer (A) is correct. Budgets provide a means for coordinating the plans of all organizational subunits. Thus,budgets are a way to promote goal congruence. Although budgets should be consistent with the strategic plans of topmanagement, they should also be based on input from lower-level managers since the latter have detailed knowledgeof operating activities. Successful budgets are therefore a compromise. In a top-down process, however, budgets areimposed on subordinates without their participation. This lack of participation may impair the coordination of thegoals of subunits with those of the organization (goal congruence) since lower-level managers will tend not to haveunderstanding of and support for the top-down budget.Answer (B) is incorrect because, in a top-down process, upper-level management imposes a budget on those below,so consistency with strategic plans is unlikely to be impaired.Answer (C) is incorrect because, in a top-down process, upper-level management imposes a budget on those below,so consistency with strategic plans is unlikely to be impaired.Answer (D) is incorrect because the motivational effect is likely to be very negative.

[143] Gleim #: 1.143 -- Source: CIA 1196 III-26

An effective management by objectives (MBO) program can increase organizational effectiveness. Which of the followingcontributes to an effective MBO program?

Emphasis on “should do” rather than “must do” objectives.A.Objectives that are quantified, clearly measurable, and state target dates for completion.B.Managers who hold their subordinates strictly accountable for achieving their objectives precisely as they have beenwritten.

C.

All of the answers are correct.D.

Gleim's CIA Test Prep: Part III: Business Analysis and Information Technology(1165 questions)

Copyright 2004 Gleim Publications, Inc. Page 53Printed for g j

Page 54: Part three cia_with_ answers

Answer (A) is incorrect because “must do” objectives should be given priority over “should do” objectives.Answer (B) is correct. Management by objectives is a behavioral, communications-oriented, responsibility approach toemployee self-direction. Objectives should be set mutually by management and employees. They should be measurable andcapable of being accomplished by the end of the period. The organization must also remain flexible to accommodateunforeseen changes.Answer (C) is incorrect because the organization must be able to remain flexible to accommodate unforeseen changesduring the period.Answer (D) is incorrect because “must do” objectives should be emphasized, and the organization should remain flexible toaccommodate unforeseen changes during the period.

[144] Gleim #: 1.144 -- Source: Publisher

An advantage of participatory budgeting is that it

Minimizes the cost of developing budgets.A.Reduces the effect on the budgetary process of employee biases.B.Yields information known to management but not to employees.C.Encourages acceptance of the budget by employees.D.

Answer (A) is incorrect because disadvantages of participatory budgeting and standard setting include the cost interms of time and money.Answer (B) is incorrect because the quality of participation is affected by the goals, values, beliefs, and expectationsof those involved.Answer (C) is incorrect because an advantage of participatory budgeting is that it yields information known toemployees but not to management.Answer (D) is correct. Participatory budgeting (grass-roots budgeting) and standard setting use input from lower-level and middle-level employees. Participation encourages employees to have a sense of ownership of the output ofthe process. The result is an acceptance of, and commitment to, the goals expressed in the budget.

[145] Gleim #: 1.145 -- Source: CMA 1293 3-17

The budgeting process that uses management by objectives and input from the individual manager is an example of theapplication of

Flexible budgeting.A.Human resource management.B.Responsibility accounting.C.Capital budgeting.D.

Answer (A) is incorrect because flexible budgeting is the process of preparing a series of multiple budgets for varyinglevels of production or sales.Answer (B) is incorrect because human resource management is the process of managing personnel.Answer (C) is correct. Managerial performance should ideally be evaluated only on the basis of those factorscontrollable by the manager. Managers may control revenues, costs, or investments in resources. A well-designedresponsibility accounting system establishes responsibility centers within the organization. However, controllability isnot an absolute basis for establishment of responsibility. More than one manager may be able to influence a cost, andresponsibility may be assigned on the basis of knowledge about the incurrence of a cost rather than the ability tocontrol it. Management by objectives (MBO) is a related concept. It is a behavioral, communications-oriented,responsibility approach to employee self-direction. Under MBO, a manager and his/her subordinates agree uponobjectives and the means of attaining them. The plans that result are reflected in responsibility accounting and in thebudgeting process.Answer (D) is incorrect because capital budgeting is a means of evaluating long-term investments.

[146] Gleim #: 1.146 -- Source: CIA 1190 IV-21

When comparing the residual income of several investment centers, the validity of comparisons may be destroyed by

Peculiarities of each investment center.A.Differences in the relative amount of income.B.Consistent use of an imputed interest rate.C.Common amounts of invested capital for each investment center.D.

Gleim's CIA Test Prep: Part III: Business Analysis and Information Technology(1165 questions)

Copyright 2004 Gleim Publications, Inc. Page 54Printed for g j

Page 55: Part three cia_with_ answers

Answer (A) is correct. Residual income is income of an investment center, minus an imputed interest charge for investedcapital. The theory is that earning an income greater than residual income indicates that expansion is desirable. However,comparisons of investment centers based on residual income may be misleading because of differences in products, markets,costs, and local conditions.Answer (B) is incorrect because comparisons are valid if adjustments are made for relative investment bases.Answer (C) is incorrect because use of the same imputed interest rate provides a consistent objective against which eachinvestment can be measured.Answer (D) is incorrect because common amounts of invested capital would eliminate a major factor causing differences inresidual income.

[147] Gleim #: 1.147 -- Source: CIA 1191 IV-17

The receipt of raw materials used in the manufacture of products and the shipping of finished goods to customers is underthe control of the warehouse supervisor. The warehouse supervisor’s time is spent approximately 60% on receivingactivities and 40% on shipping activities. Separate staffs for the receiving and shipping operations are employed. Thelabor-related costs for the warehousing function are as follows:

Warehouse supervisor’s salary $   40,000Receiving clerks’ wages 75,000Shipping clerks’ wages 55,000Employee benefit costs (30% of wage and salary costs) 51,000   $221,000 

The company employs a responsibility accounting system for performance reporting purposes. The costs are classified onthe report as period or product costs. The total labor-related costs that would be listed on the responsibility accountingperformance report as product costs under the control of the warehouse supervisor for the warehousing function would be

$97,500.A.$128,700.B.$130,000.C.$221,000.D.

Answer (A) is correct. The responsibility accounting report should list only the costs over which the warehousingsupervisor exercises control. The supervisor’s salary should therefore be excluded because it is controlled by thewarehouse supervisor’s superior. Moreover, only the product costs are to be considered. These exclude the shippingclerks’ wages and fringe benefits because they are period costs (shipping is a selling expense). Thus, the only productcost under the control of the warehouse supervisor is the receiving clerks’ wages ($75,000) and the related fringebenefits (.3 × $75,000 = $22,500), or a total of $97,500.Answer (B) is incorrect because the product costs controlled by the supervisor total $97,500.Answer (C) is incorrect because the product costs controlled by the supervisor total $97,500.Answer (D) is incorrect because the product costs controlled by the supervisor total $97,500.

[148] Gleim #: 1.148 -- Source: CIA 595 III-95

Making segment disclosures is an advantage to a company because it

Facilitates evaluation of company management by providing data on particular segments.A.Eliminates the interdependence of segments.B.Masks the effect of intersegment transfers.C.Provides competitors with comparative information on the company’s performance.D.

Answer (A) is correct. Segment reporting is an aspect of responsibility accounting. It facilitates evaluation ofcompany management and of the quality of the economic investment in particular segments.Answer (B) is incorrect because interdependence of segments is not affected by reporting methods.Answer (C) is incorrect because masking the effects of intersegment transfers is a disadvantage of segment reporting.Answer (D) is incorrect because providing information to competitors is a disadvantage of segment reporting.

Gleim's CIA Test Prep: Part III: Business Analysis and Information Technology(1165 questions)

Copyright 2004 Gleim Publications, Inc. Page 55Printed for g j

Page 56: Part three cia_with_ answers

[149] Gleim #: 1.149 -- Source: CIA 1195 III-67Return on investment (ROI) is a very popular measure employed to evaluate the performance of corporate segmentsbecause it incorporates all of the major ingredients of profitability (revenue, cost, investment) into a single measure. Underwhich one of the following combination of actions regarding a segment’s revenues, costs, and investment would asegment’s ROI always increase?

  Revenues Costs Investment A. Increase Decrease IncreaseB. Decrease Decrease DecreaseC. Increase Increase IncreaseD. Increase Decrease Decrease

Answer (A) is incorrect because ROI is not certain to increase if investment increases.Answer (B) is incorrect because ROI is not certain to increase if revenue, costs, and investment decrease.Answer (C) is incorrect because ROI is not certain to increase if costs and investment increase.Answer (D) is correct. An increase in revenue and a decrease in costs will increase the ROI numerator. A decrease ininvestment will decrease the denominator. The ROI must increase in this situation.

[150] Gleim #: 1.150 -- Source: CIA 1195 III-79

Residual income is a performance evaluation that is used in conjunction with return on investment (ROI) or instead of ROI.In many cases, residual income is preferred over ROI because

Residual income is a measure over time while ROI represents the results for a single time period.A.Residual income concentrates on maximizing absolute dollars of income rather than a percentage return as with ROI.B.The imputed interest rate used in calculating residual income is more easily derived than the target rate that iscompared to the calculated ROI.

C.

Average investment is employed with residual income while year-end investment is employed with ROI.D.

Answer (A) is incorrect because both measures represent the results for a single period.Answer (B) is correct. Residual income equals earnings in excess of a minimum desired return. Thus, it is measuredin dollars. If performance is evaluated using ROI, a manager may reject a project that exceeds the minimum return ifthe project will decrease overall ROI. For example, given a target rate of 20%, a project with an ROI of 22% might berejected if the current ROI is 25%.Answer (C) is incorrect because the target rate for ROI is the same as the imputed interest rate used in the residualincome calculation.Answer (D) is incorrect because the same investment base should be employed in both methods.

[151] Gleim #: 1.151 -- Source: CMA 1294 3-20

Fairmount Inc. uses an accounting system that charges costs to the manager who has been delegated the authority to makethe decisions incurring the costs. For example, if the sales manager accepts a rush order that will result in higher-than-normal manufacturing costs, these additional costs are charged to the sales manager because the authority to accept ordecline the rush order was given to the sales manager. This type of accounting system is known as

Responsibility accounting.A.Functional accounting.B.Reciprocal allocation.C.Transfer price accounting.D.

Answer (A) is correct. Responsibility accounting holds managers responsible only for factors under their control. Forthis purpose, operations are organized into responsibility centers. Costs are classified as controllable andnoncontrollable, which implies that some revenues and costs can be changed through effective management. If amanager has authority to incur costs, a responsibility accounting system will charge them to the manager’sresponsibility center.Answer (B) is incorrect because functional accounting allocates costs to functions regardless of responsibility.Answer (C) is incorrect because reciprocal allocation is a means of allocating service department costs.Answer (D) is incorrect because transfer price accounting is a means of charging one department for productsacquired from another department in the same organization.

Gleim's CIA Test Prep: Part III: Business Analysis and Information Technology(1165 questions)

Copyright 2004 Gleim Publications, Inc. Page 56Printed for g j

Page 57: Part three cia_with_ answers

[152] Gleim #: 1.152 -- Source: CIA 596 II-1A company, which has many branch stores, has decided to benchmark one of its stores for the purpose of analyzing theaccuracy and reliability of branch store financial reporting. Which one of the following is the most likely measure to beincluded in a financial benchmark?

High turnover of employees.A.High level of employee participation in setting budgets.B.High amount of bad debt write-offs.C.High number of suppliers.D.

Answer (A) is incorrect because turnover of employees is not a financial benchmark.Answer (B) is incorrect because employee participation in setting budgets is not a financial benchmark.Answer (C) is correct. A high level of bad debt write-offs could indicate fraud, which would compromise theaccuracy and reliability of financial reports. Bad debt write-offs may result from recording fictitious sales.Answer (D) is incorrect because the number of suppliers is not a financial benchmark.

[153] Gleim #: 1.153 -- Source: CIA 1196 III-23

The most important component of quality control is

Ensuring goods and services conform to the design specifications.A.Satisfying upper management.B.Conforming with ISO-9000 specifications.C.Determining the appropriate timing of inspections.D.

Answer (A) is correct. The intent of quality control is to ensure that goods and services conform to the designspecifications. Whether the focus is on feedforward, feedback, or concurrent control, the emphasis is on ensuringproduct or service conformity.Answer (B) is incorrect because quality control is geared towards satisfying the customer, not upper management.Answer (C) is incorrect because ensuring the conformance with ISO-9000 specifications is a component of acompliance audit, not quality control.Answer (D) is incorrect because determining the appropriate timing of inspections is only one step towardsapproaching quality control. Consequently, it is not the primary component of the quality control function.

[154] Gleim #: 1.154 -- Source: CIA 596 III-23

Which statement best describes the emphasis of total quality management (TQM)?

Reducing the cost of inspection.A.Implementing better statistical quality control techniques.B.Doing each job right the first time.C.Encouraging cross-functional teamwork.D.

Answer (A) is incorrect because reducing the cost of inspection is a detail of the TQM emphasis.Answer (B) is incorrect because implementing better statistical quality control techniques is a detail of the TQMemphasis.Answer (C) is correct. The basic principles of TQM include doing each job right the first time, being customeroriented, committing the company culture to continuous improvement, and building teamwork and employeeempowerment.Answer (D) is incorrect because encouraging cross-functional teamwork is a detail of the TQM emphasis.

Gleim's CIA Test Prep: Part III: Business Analysis and Information Technology(1165 questions)

Copyright 2004 Gleim Publications, Inc. Page 57Printed for g j

Page 58: Part three cia_with_ answers

[155] Gleim #: 1.155 -- Source: CIA 1196 III-92Quality cost indices are often used to measure and analyze the cost of maintaining or improving the level of quality. Suchindices are computed by dividing the total cost of quality over a given period by some measure of activity during thatperiod (for example, sales dollars). The following cost data are available for a company for the month of March. Thecompany's quality cost index is calculated using total cost of quality divided by sales dollars.

Sales $400,000Direct materials cost 100,000Direct labor cost 80,000Testing and inspection cost 6,400Scrap and rework cost 16,800Quality planning cost 2,800Cost of customer complaints and returns 4,000

The quality cost index for March is

7.5%A.6.5%B.22.0%C.5.9%D.

Answer (A) is correct. he total cost of quality equals the sum of prevention costs (quality planning), appraisal costs(testing and inspection), internal failure costs (scrap and rework), and external failure costs (customer complaints andreturns), or $30,000 ($2,800 + $6,400 + $16,800 + $4,000). The quality cost index equals the total costs of quality,divided by sales, times 100%. Thus, the quality cost index for March is 7.5% [($30,000 ÷ $400,000) × 100%].Answer (B) is incorrect because 6.5% does not include the $4,000 cost of customer complaints.Answer (C) is incorrect because 22.0% equals the sum of direct labor and testing and inspection costs, divided bysales.Answer (D) is incorrect because 5.9% does not include testing and inspection cost in quality costs.

[156] Gleim #: 1.156 -- Source: CIA 1196 III-23

The most important component of quality control is

Ensuring goods and services conform to the design specifications.A.Satisfying upper management.B.Conforming with ISO-9000 specifications.C.Determining the appropriate timing of inspections.D.

Answer (A) is correct. The intent of quality control is to ensure that goods and services conform to the designspecifications. Whether the focus is on feedforward, feedback, or concurrent control, the emphasis is on ensuringproduct or service conformity.Answer (B) is incorrect because quality control is geared towards satisfying the customer, not upper management.Answer (C) is incorrect because ensuring the conformance with ISO-9000 specifications is a component of acompliance audit, not quality control.Answer (D) is incorrect because determining the appropriate timing of inspections is only one step towardsapproaching quality control. Consequently, it is not the primary component of the quality control function.

[157] Gleim #: 1.157 -- Source: CMA 693 3-13

Product-quality-related costs are part of a total quality control program. A product-quality-related cost incurred in detectingindividual products that do not conform to specifications is an example of a(n)

Prevention cost.A.Appraisal cost.B.External failure cost.C.Opportunity cost.D.

Gleim's CIA Test Prep: Part III: Business Analysis and Information Technology(1165 questions)

Copyright 2004 Gleim Publications, Inc. Page 58Printed for g j

Page 59: Part three cia_with_ answers

Answer (A) is incorrect because prevention costs are incurred in an attempt to avoid defective output.Answer (B) is correct. Quality-related costs can be subdivided into four categories: external failure costs, internal failurecosts, prevention costs, and appraisal costs. Appraisal costs embrace such activities as statistical quality control programs,inspection, and testing. Thus, the cost of detecting nonconforming individual products is an appraisal cost.Answer (C) is incorrect because external failure costs arise when quality problems occur after shipment.Answer (D) is incorrect because opportunity costs are not specifically associated with product quality. Opportunity cost isthe maximum benefit forgone by using a scarce resource for a given purpose; it is the benefit provided by the next best useof that resource.

[158] Gleim #: 1.158 -- Source: CMA 694 3-16

In 2004, a manufacturer instituted a total quality management (TQM) program producing the following report (in currencyunits):

  Summary Cost of Quality Report (000s)   2003 2004 % Change Prevention costs 200 300 +50Appraisal costs 210 315 +50Internal failure costs 190 114 -40External failure costs 1,200 621 -48 Total quality costs 1,800 1,350 -25 

On the basis of this report, which one of the following statements is most likely true?

An increase in conformance costs resulted in a higher quality product and therefore resulted in a decrease innonconformance costs.

A.

An increase in inspection costs was solely responsible for the decrease in quality costs.B.Quality costs such as scrap and rework decreased by 48%.C.Quality costs such as returns and repairs under warranty decreased by 40%.D.

Answer (A) is correct. TQM emphasizes the supplier’s relationship with the customer and recognizes that everyonein a process is at some time a customer or supplier of someone else, either within or outside the organization. Thecosts of quality include costs of conformance and costs of nonconformance. Costs of conformance include preventioncosts and appraisal (inspection) costs. Nonconformance costs are composed of internal failure costs and externalfailure costs, such as lost opportunity. Conformance costs (prevention and appraisal) increased substantially, whereasthe nonconformance costs (internal and external failure) decreased. Hence, the increase in conformance costs resultedin a higher quality product.Answer (B) is incorrect because prevention costs also increased substantially, which could also have led to higherquality products.Answer (C) is incorrect because scrap and rework are internal failure costs, which decreased by 40%.Answer (D) is incorrect because returns and repairs are external failure costs, which decreased by 48%.

[159] Gleim #: 1.159 -- Source: CIA 1184 III-11

Which of the following statements best describes the relationship between planning and controlling?

Planning looks to the future; controlling is concerned with the past.A.Planning and controlling are completely independent of each other.B.Planning prevent problems; controlling is initiated by problems which have occurred.C.Controlling cannot operate effectively without the tools provided by planning.D.

Answer (A) is incorrect because a control system looks to the future when it provides for corrective action and reviewand revision of standards.Answer (B) is incorrect because planning and controlling overlap.Answer (C) is incorrect because comprehensive planning includes creation of controls.Answer (D) is correct. Control is the process of making certain that plans are achieving the desired objectives. Acontrol system operates through establishing standards of performance, measuring actual performance, analysis andcomparison of performance with standards, taking corrective action, and reviewing and revising standards. Planningprovides needed tools for the control process by establishing standards, e.g., budgets.

Gleim's CIA Test Prep: Part III: Business Analysis and Information Technology(1165 questions)

Copyright 2004 Gleim Publications, Inc. Page 59Printed for g j

Page 60: Part three cia_with_ answers

[160] Gleim #: 1.160 -- Source: CIA 593 III-63

A firm must decide the mix of production of Product X and Product Y. There are only two resources used in the twoproducts, resources A and B. Data related to the two products is given in the following table:

  Products   X Y Resource A 3 7Resource B 2 1Unit Profit $8 $6

What is the appropriate objective function to maximize profit?

3X + 7YA.2X + YB.8X + 6YC.5X + 8YD.

Answer (A) is incorrect because 3X + 7Y is a constraint.Answer (B) is incorrect because 2X + Y is a constraint.Answer (C) is correct. The objective function is the function to be optimized. This firm wishes to maximize profitson the sales of two products (X and Y). Based on profits per unit ($8 and $6, respectively), the objective function is8X + 6Y.Answer (D) is incorrect because 5X + 8Y is the sum of the constraints.

[161] Gleim #: 1.161 -- Source: CIA R98 III-32

An appropriate technique for planning and controlling manufacturing inventories, such as raw materials, components, andsubassemblies whose demand depends on the level of production is

Materials requirements planning.A.Regression analysis.B.Capital budgeting.C.Linear programming.D.

Answer (A) is correct. Materials requirements planning (MRP) is a system that translates a production schedule intorequirements for each component needed to meet the schedule. It is usually implemented in the form of a computer-based information system designed to plan and control raw materials used in production. It assumes that forecasteddemand is reasonably accurate and that suppliers can deliver based upon this accurate schedule. MRP is a centralizedpush-through system; output based on forecasted demand is pushed through to the next department or to inventory.Answer (B) is incorrect because regression analysis is used to fit a linear trend line to a dependent variable based onone or more independent variables.Answer (C) is incorrect because capital budgeting is the process of planning expenditures for long-lived assets. Itinvolves choosing among investment proposals using a ranking procedure.Answer (D) is incorrect because linear programming is a decision model concerned with allocating scarce resourcesto maximize profit or minimize costs.

[162] Gleim #: 2.1 -- Source: CIA 597 III-91

When the economic order quantity (EOQ) decision model is employed, the <List A> are being offset or balanced by the<List B>.

  List A List B A. Ordering costs Carrying costsB. Purchase costs Carrying costsC. Purchase costs Quality costsD. Ordering costs Stockout costs

Gleim's CIA Test Prep: Part III: Business Analysis and Information Technology(1165 questions)

Copyright 2004 Gleim Publications, Inc. Page 60Printed for g j

Page 61: Part three cia_with_ answers

Answer (A) is correct. The objective of the EOQ model is to find an optimal order quantity that balances carrying andordering costs. Only variable costs should be considered. The EOQ is the point where the ordering cost and carrying costcurves intersect. It corresponds to the minimum point on the total inventory cost curve.Answer (B) is incorrect because purchase costs are not directly incorporated into the EOQ model.Answer (C) is incorrect because neither purchase costs nor quality costs are incorporated into the EOQ model.Answer (D) is incorrect because stockout costs are not directly incorporated into the EOQ model.

[Fact Pattern #11]

Candman Company is a wholesale distributor ofcandy. The company leases space in a publicwarehouse and is charged according to the squarefeet occupied. Candman has decided to employ theeconomic order quantity (EOQ) method to determinethe optimum number of cases of candy to order.

The company placed 2,400 orders the last year. Datafor the high-activity month, the low-activity month,and the year for the purchasing and warehouseoperations appear in the next column.

The annual charges for the warehouse totaled$12,750 last year. In addition, the annual insuranceand property taxes on the candy stored in thewarehouse amounted to $1,500 and $2,250,respectively. The average monthly inventory lastyear was $75,000.

Long-term capital investments are expected to earn 12% after incometaxes. Candman is subject to an effective income tax rate of 40%.

High-Activity Low-ActivityMonth Month Annual

(160 Orders) (100 Orders) CostsPurchasing Dept.

Manager $ 1,600 $1,600 $  23,400Clerks 1,750 1,250 18,000Supplies 400 260 3,500

WarehouseSupervisor 1,550 1,550 18,600Rcvg. clerks 2,200 1,700 20,500Ship. clerks 2,800 2,500 31,200

Totals $10,300 $8,860 $115,200

[163] Gleim #: 2.2 -- Source: CMA 686 5-29

(Refers to Fact Pattern #11)The incremental cost of placing an order that would be used in the EOQ model is

$48A.$35B.$24C.$19D.

Answer (A) is incorrect because $48 equals total annual cost divided by total orders from the previous year.Answer (B) is incorrect because $35 equals total annual cost minus the costs associated with the shipping clerks,divided by total orders from the previous year.Answer (C) is incorrect because $24 includes the costs associated with shipping.Answer (D) is correct. The incremental cost of additional orders equals the variable costs incurred in purchasing andreceiving inventory (but not costs associated with shipping). Using the high-low method of analysis, these variablecosts can be computed by determining the apparent variable costs for the two given levels of activity. For 160 orders,the relevant costs with a variable element include purchasing clerks, $1,750; supplies, $400; and receiving clerks,$2,200. The total is $4,350. For 100 orders, these costs are $1,250, $260, and $1,700, respectively, for a total of$3,210. As the number of orders increased by 60, the costs increased by $1,140. Consequently, the variable orincremental costs per order must have been $19 ($1,140 ÷ 60 orders).

[164] Gleim #: 2.3 -- Source: CMA 686 5-30

(Refers to Fact Pattern #11)The annual carrying cost, stated as a percentage, that would be used in the EOQ model is

42%A.37%B.34%C.22%D.

Gleim's CIA Test Prep: Part III: Business Analysis and Information Technology(1165 questions)

Copyright 2004 Gleim Publications, Inc. Page 61Printed for g j

Page 62: Part three cia_with_ answers

Answer (A) is correct. The annual carrying costs are $12,750 for the warehouse, $1,500 for insurance, and $2,250 forproperty taxes. These costs total $16,500. In addition, the company desires a 12% after-tax return on investments. Becausethe tax rate is 40%, the 12% after-tax return equals a 20% before-tax return (12% ÷ 60%). A 20% return on the $75,000average investment in inventory is $15,000. Hence, the total carrying cost is $31,500 ($16,500 + $15,000). This amount is42% of the $75,000 investment in inventory.Answer (B) is incorrect because 37% does not consider insurance and property taxes.Answer (C) is incorrect because 34% is based on a 12% before-tax return.Answer (D) is incorrect because 22% equals carrying costs other than return on investment divided by average inventory.

[165] Gleim #: 2.4 -- Source: CMA 1290 4-9

The calculation of an economic order quantity (EOQ) considers

The purchasing manager’s salary.A.A corporate charge for advertising expenses.B.The shipping costs to deliver the product to the customer.C.Capital costs.D.

Answer (A) is incorrect because the purchasing manager’s salary is a fixed cost. The EOQ model includes variablecosts only.Answer (B) is incorrect because advertising is not an ordering or carrying cost.Answer (C) is incorrect because the cost of shipping to customers is a selling expense.Answer (D) is correct. The determination of the economic order quantity balances the variable costs of ordering andcarrying inventory. Factors in the equation include the cost of placing an order, unit carrying cost, and annualdemand in units. Carrying costs include storage costs, handling costs, insurance, property taxes, obsolescence, andthe opportunity cost of investing capital in inventory. Thus, the return on capital that is forgone when it is invested ininventory should be considered.

[166] Gleim #: 2.5 -- Source: CMA 1294 4-8

A characteristic of the basic economic order quantity (EOQ) model is that it

Is relatively insensitive to error.A.Should not be used when carrying costs are large in relation to procurement costs.B.Is used when product demand, lead-time, and ordering costs are uncertain.C.Should not be used in conjunction with computerized perpetual inventory systems.D.

Answer (A) is correct. The basic EOQ model equals the square root of the quotient of (1) the product of twice thedemand times the cost per order, (2) divided by the periodic carrying cost. Hence, the model is relatively insensitiveto error. A given percentage error in a value results in a lower percentage change in the EOQ.Answer (B) is incorrect because the EOQ model can be used regardless of the relationship between carrying andholding costs.Answer (C) is incorrect because product demand and ordering costs must be known with some certainty.Answer (D) is incorrect because an EOQ model can be used with any type of system.

[167] Gleim #: 2.6 -- Source: CMA 1294 4-6

Companies that adopt just-in-time purchasing systems often experience

An increase in carrying costs.A.A reduction in the number of suppliers.B.A greater need for inspection of goods as the goods arrive.C.Less need for linkage with a vendor’s computerized order entry system.D.

Gleim's CIA Test Prep: Part III: Business Analysis and Information Technology(1165 questions)

Copyright 2004 Gleim Publications, Inc. Page 62Printed for g j

Page 63: Part three cia_with_ answers

Answer (A) is incorrect because carrying costs typically decline in JIT companies. Less inventory is on hand.Answer (B) is correct. The objective of JIT is to reduce carrying costs by eliminating inventories and increasing thedeliveries made by suppliers. Ideally, shipments of raw materials are received just in time to be incorporated into themanufacturing process. The focus of quality control under JIT is the prevention of quality problems. Quality control isshifted to the supplier. JIT companies typically do not inspect incoming goods; the assumption is that receipts are of perfectquality. Suppliers are limited to those who guarantee perfect quality and prompt delivery.Answer (C) is incorrect because in a JIT system, materials are delivered directly to the production line ready for insertion inthe finished product.Answer (D) is incorrect because the need for communication with the vendor is greater. Orders and deliveries must be madeon short notice, sometimes several times a day.

[168] Gleim #: 2.7 -- Source: CIA 597 III-94

A manufacturing company is attempting to implement a just-in-time (JIT) purchase policy system by negotiating with itsprimary suppliers to accept long-term purchase orders which result in more frequent deliveries of smaller quantities of rawmaterials. If the JIT purchase policy is successful in reducing the total inventory costs of the manufacturing company,which of the following combinations of cost changes would be most likely to occur?

  Cost Category Cost Category  to Increase to Decrease A. Purchasing costs Stockout costsB. Purchasing costs Quality costsC. Quality costs Ordering costsD. Stockout costs Carrying costs

Answer (A) is incorrect because the supplier may seek a concession on the selling price that will raise purchasingcosts, but the manufacturing company’s stockout costs will increase.Answer (B) is incorrect because the cost of quality is not necessarily affected by a JIT system.Answer (C) is incorrect because fewer purchase orders are processed by the manufacturer, so the ordering costs arelikely to decrease. However, the cost of quality is not necessarily affected by a JIT system.Answer (D) is correct. The objective of a JIT system is to reduce carrying costs by eliminating inventories andincreasing the deliveries made by suppliers. Ideally, shipments are received just in time to be incorporated into themanufacturing process. This system increases the risk of stockout costs because the inventory buffer is reduced oreliminated.

[169] Gleim #: 2.8 -- Source: CMA 1288 5-22

Arnold Enterprises uses the EOQ model for inventory control. The company has an annual demand of 50,000 units for partnumber 191 and has computed an optimal lot size of 6,250 units. Per-unit carrying costs and stockout costs are $13 and $3,respectively. The following data have been gathered in an attempt to determine an appropriate safety stock level:

Units Short Because of Number of Times ShortExcess Demand During in the Last 40the Lead Time Period Reorder Cycles

200 6300 12400 6

The annual cost of establishing a 200-unit safety stock is expected to be

$2,600A.$4,040B.$4,260C.$5,200D.

Gleim's CIA Test Prep: Part III: Business Analysis and Information Technology(1165 questions)

Copyright 2004 Gleim Publications, Inc. Page 63Printed for g j

Page 64: Part three cia_with_ answers

Answer (A) is incorrect because $2,600 is the annual carrying cost of 200 units of safety stock.Answer (B) is correct. The annual cost consists of the carrying cost of the 200 units of safety stock at $13 each, or $2,600,plus the stockout costs incurred when 200 units are insufficient. The stockout cost per unit is $3. The excess demand hasbeen 100 units (300 – 200) greater than the proposed safety stock 30% of the time (12 ÷ 40). The cost per stockout was$300 ($3 × 100). Demand has exceeded the safety stock by 200 units (400 – 200) 15% of the time (6 ÷ 40). The cost perstockout was $600 ($3 × 200). Given 30% and 15% probabilities of $300 and $600 stockout costs, respectively, theexpected stockout cost for a 200-unit safety stock is $180 per inventory cycle [(30% × $300) + (15% × $600)]. Given 8cycles (50,000 units ÷ 6,250 EOQ), the annual cost of a 200-unit safety stock is therefore $4,040 [$2,600 + (8 × $180)].Answer (C) is incorrect because $4,260 is the annual cost of establishing a 300-unit safety stock.Answer (D) is incorrect because $5,200 is the annual cost of establishing a 400-unit safety stock.

[Fact Pattern #12]The diagram presented represents the economic order quantity (EOQ) model.

(Refer to Figure CIA3_02_14.)

[170] Gleim #: 2.9 -- Source: CMA 1289 5-16

(Refers to Fact Pattern #12)Which line segment represents the reorder lead time?

AB.A.AE.B.AF.C.BC.D.

Answer (A) is incorrect because AB is the time between receipt of the last order and the placing of the next order.Answer (B) is incorrect because AE is the safety stock.Answer (C) is incorrect because AF represents the quantity of inventory that will be used during the reorder leadtime.Answer (D) is correct. The quantity of inventory on hand is represented by the y axis and time by the x axis. Thereorder lead time is represented by the line segment BC.

[171] Gleim #: 2.10 -- Source: CMA 1289 5-17

(Refers to Fact Pattern #12)Which line segment identifies the quantity of safety stock maintained?

AB.A.AE.B.AC.C.BC.D.

Answer (A) is incorrect because AB is the time between the receipt of the last order and the placing of the next order.Answer (B) is correct. Quantities of inventory are shown along the y axis. Safety stock is represented by the line AE.Answer (C) is incorrect because AC is the time to consume the EOQ.Answer (D) is incorrect because BC is the reorder lead time.

[172] Gleim #: 2.11 -- Source: CMA 1289 5-18

(Refers to Fact Pattern #12)Which line segment represents the length of time to consume the total quantity of materials ordered?

DE.A.BC.B.AC.C.AE.D.

Gleim's CIA Test Prep: Part III: Business Analysis and Information Technology(1165 questions)

Copyright 2004 Gleim Publications, Inc. Page 64Printed for g j

Page 65: Part three cia_with_ answers

Answer (A) is incorrect because DE represents the total inventory on hand just after an order has been received.Answer (B) is incorrect because BC is the reorder lead time.Answer (C) is correct. Time is shown along the x axis. The line segment AC depicts the time to consume an entire order (toreduce the inventory to the safety stock).Answer (D) is incorrect because AE is the safety stock.

[173] Gleim #: 2.12 -- Source: CMA 692 4-23

Each stockout of a product sold by A.W. Inn Co. costs $1,750 per occurrence. The carrying cost per unit of inventory is $5per year, and the company orders 1,500 units of product 24 times a year at a cost of $100 per order. The probability of astockout at various levels of safety stock is

Units of ProbabilitySafety Stock of a Stockout

0 .50100 .30200 .14300 .05400 .01

The optimal safety stock level for the company is

0 units.A.100 units.B.300 units.C.400 units.D.

Answer (A) is incorrect because a safety stock of 0 units has a total expected cost of $21,000.Answer (B) is incorrect because a safety stock of 100 units has a total expected cost of $13,100.Answer (C) is incorrect because a safety stock of 300 units has a total expected cost of $3,600.Answer (D) is correct. The total expected cost of safety stock equals the sum of the expected annual stockout costand the expected annual carrying cost. Annual expected stockout cost equals the cost per occurrence ($1,750), timesthe probability of a stockout per cycle, times the number of cycles (24). Annual expected carrying cost of a safetystock equals the unit carrying cost ($5) times the number of units. Hence, a safety stock of 400 units has the lowesttotal expected cost.

    Expected Expected TotalUnits Carrying Stockout Cost Stockout Cost ExpectedHeld Cost Per Cycle for 24 Cycles Cost

0 $       0 $875.00 $21,000 $21,000100 500 525.00 12,600 13,100200 1,000 245.00 5,880 6,880300 1,500 87.50 2,100 3,600400 2,000 17.50 420 2,420

[Fact Pattern #13]The Huron Corporation purchases 60,000 headbands per year. The average purchase lead time is 20 working days. Maximum leadtime is 27 working days. The corporation works 240 days per year.

[174] Gleim #: 2.13 -- Source: CMA 1293 4-8

(Refers to Fact Pattern #13)Huron Corporation should carry safety stock of

5,000 units.A.6,750 units.B.1,750 units.C.250 units.D.

Gleim's CIA Test Prep: Part III: Business Analysis and Information Technology(1165 questions)

Copyright 2004 Gleim Publications, Inc. Page 65Printed for g j

Page 66: Part three cia_with_ answers

Answer (A) is incorrect because 5,000 units represents the quantity expected to be sold during the normal lead time.Answer (B) is incorrect because 6,750 units includes not only the safety stock but also the units expected to be sold duringthe normal lead time.Answer (C) is correct. Safety stock is defined as the amount of extra stock that is kept to guard against stockouts. It is theinventory level at the reorder point minus the expected usage during the lead time. Daily usage is 250 units (60,000 per year÷ 240 days). Given a maximum lead time of 27 days and a normal lead time of 20 days, a safety stock for 7 days (27 – 20)should be maintained. Hence, safety stock is 1,750 units (7 × 250 units).Answer (D) is incorrect because 250 units is 1 day’s sales.

[175] Gleim #: 2.14 -- Source: CMA 1293 4-9

(Refers to Fact Pattern #13)Huron Corporation should reorder headbands when the quantity in inventory reaches

5,000 units.A.6,750 units.B.1,750 units.C.5,250 units.D.

Answer (A) is incorrect because 5,000 units does not allow for safety stock.Answer (B) is correct. The reorder point is the quantity on hand when an order is placed. With a 20-day normal leadtime, a 7-day safety stock, and usage of 250 units per day, an order should be placed when 27 days of inventory areon hand, a total of 6,750 units (27 × 250).Answer (C) is incorrect because 1,750 units covers only safety stock.Answer (D) is incorrect because 5,250 units includes only 1 day of safety stock.

[Fact Pattern #14]Ryerson Computer Furniture Inc. (RCF) manufactures a line of office computer chairs. The annual demand for the chairs isestimated to be 5,000 units. The annual cost to hold one unit in inventory is $10 per year, and the cost to initiate a production runis $1,000. There are no computer chairs on hand, and RCF has scheduled four equal production runs of computer chairs for thecoming year, the first of which is to be run immediately. RCF has 250 business days per year, sales occur uniformly throughoutthe year, and production start-up is within one day. RCF is considering using the following formula for determining the economicorder quantity (EOQ):

If: A = cost to initiate a production run per purchase order  D = annual unit demand  K = cost of carrying one unit per year

[176] Gleim #: 2.15 -- Source: CMA 694 4-23

(Refers to Fact Pattern #14)The number of production runs per year of computer chairs that would minimize the sum of carrying and setup costs for thecoming year is

1A.2B.4C.5D.

Gleim's CIA Test Prep: Part III: Business Analysis and Information Technology(1165 questions)

Copyright 2004 Gleim Publications, Inc. Page 66Printed for g j

Page 67: Part three cia_with_ answers

Answer (A) is incorrect because a single production run indicates an EOQ of 5,000 units. The carrying costs of $25,000[$10 × ($5,000 ÷ 2)] would exceed the $1,000 of setup costs.Answer (B) is incorrect because two production runs correspond to an EOQ of 2,500 units and an average inventory of1,250 units. The resulting $12,500 of carrying costs would exceed the $2,000 of setup costs.Answer (C) is incorrect because four production runs correspond to an EOQ of 1,250 units and an average inventory of 625units. The resulting $6,250 of carrying costs would exceed the $4,000 of setup costs.Answer (D) is correct. The EOQ minimizes the sum of carrying and setup costs. The EOQ is the amount at which carryingcosts are equal to setup costs. Thus, plugging the data into the EOQ formula results in the following:

Thus, if each lot consists of 1,000 units, five production runs per year are needed to meet the 5,000-unit demand. At thislevel, setup costs will total $5,000 (5 × $1,000). Carrying costs will also equal $5,000 ($10 per unit carrying cost × averageinventory of 500 units). Accordingly, total costs are minimized at $10,000.

[177] Gleim #: 2.16 -- Source: CMA 694 4-22

(Refers to Fact Pattern #14)If RCF does not maintain a safety stock, the estimated total carrying costs for the computer chairs for the coming yearbased on their current schedule is

$4,000A.$5,000B.$6,250C.$12,500D.

Answer (A) is incorrect because the cost of maintaining an average inventory of 625 units is $6,250.Answer (B) is incorrect because $5,000 is based upon an EOQ of 1,000 units and an average inventory of 500 units.Answer (C) is correct. Given four production runs and an annual demand of 5,000 units, each production run mustgenerate 1,250 units. Inventory will total 1,250 units at the completion of each run but will decline to zero just priorto the next run. Thus, the average inventory is 625 units (1,250 ÷ 2), and the total carrying cost is $6,250 ($10 × 625units).Answer (D) is incorrect because $12,500 is based on the maximum inventory level.

[178] Gleim #: 2.17 -- Source: CIA 596 IV-78

Which one of the graphs depicts the demand curve for prestige goods?

(Refer to Figure CIA3_02_17.)A.

(Refer to Figure CIA3_02_18.)B.

(Refer to Figure CIA3_02_19.)C.

(Refer to Figure CIA3_02_20.)D.

Answer (A) is incorrect because this graph describes the familiar, negatively sloped relation between price chargedand the resulting demand level for normal goods.Answer (B) is incorrect because the demand curve can be linear or curvilinear.Answer (C) is correct. Over some intermediate range of prices, the reaction to a price increase for prestige goods isan increase, not a decrease, in the quantity demanded. Within this range, the demand curve is upward sloping. Thereason is that consumers interpret the higher price to indicate a better or more desirable product. Above some pricelevel, the relation between price and quantity demanded will again become negatively sloped.Answer (D) is incorrect because this demand curve has the same basic shape as the demand curve for prestige goods,but it bends the wrong way. As prices increase, quantity demanded first falls and then rises in this graph.

Gleim's CIA Test Prep: Part III: Business Analysis and Information Technology(1165 questions)

Copyright 2004 Gleim Publications, Inc. Page 67Printed for g j

Page 68: Part three cia_with_ answers

[179] Gleim #: 2.18 -- Source: CIA 1194 IV-78

Buyer-based pricing involves

Adding a standard markup to the cost of the product.A.Determining the price at which the product will earn a target profit.B.Basing prices on the product’s perceived value.C.Basing prices on competitors’ prices.D.

Answer (A) is incorrect because adding a standard markup to the cost of the product is cost-plus pricing.Answer (B) is incorrect because determining the price at which the product will earn a target profit is target profitpricing.Answer (C) is correct. Buyer-based pricing involves basing prices on the product’s perceived value rather than onthe seller’s cost. Nonprice variables in the marketing mix augment the perceived value. For example, a cup of coffeemay have a higher price at an expensive restaurant than at a fast-food outlet.Answer (D) is incorrect because basing prices on competitors’ prices is going-rate pricing.

[180] Gleim #: 2.19 -- Source: CIA 1194 IV-79

Market-skimming pricing strategies could be appropriate when

No buyers want the product at a high price.A.The costs of producing a small volume are low.B.Competitors can easily enter the market.C.The product is of poor quality.D.

Answer (A) is incorrect because, if no buyers want the product at a high price, this marketing strategy isinappropriate.Answer (B) is correct. Market-skimming pricing is used when a new product is introduced at the highest pricepossible given the benefits of the product. For market skimming to work, the product must appear to be worth itsprice, the costs of producing a small volume cannot be so high that they eliminate the advantage of charging more,and competitors cannot enter the market and undercut the price.Answer (C) is incorrect because, if competitors can easily enter the market, they can undercut the price.Answer (D) is incorrect because the product quality and image must support a high price.

[181] Gleim #: 2.20 -- Source: CIA 595 IV-74

Which of the following pricing policies involves the selling company setting freight charges to customers at the actualaverage freight cost?

Freight absorption pricing.A.Uniform delivered pricing.B.Zone pricing.C.FOB-origin pricing.D.

Answer (A) is incorrect because, in freight absorption pricing, the selling company absorbs all or part of the actualfreight charges. Customers are not charged actual delivery costs.Answer (B) is correct. In uniform delivered pricing, the company charges the same price, inclusive of shipping costs,to all customers regardless of their location. This price is the company’s average actual freight cost. Thus, bothnearby and distant customers are charged the same amount. This policy is easy to administer, permits the company toadvertise one price nationwide, and facilitates marketing to faraway customers.Answer (C) is incorrect because, in zone pricing, differential freight charges are set for customers on the basis of theirlocation. Customers are not charged actual average freight costs.Answer (D) is incorrect because, in FOB-origin pricing, each customer pays its actual freight costs.

[182] Gleim #: 2.21 -- Source: CIA 1195 IV-75

Which of the following price adjustment strategies is designed to stabilize production for the selling firm?

Cash discounts.A.Quantity discounts.B.Functional discounts.C.Seasonal discounts.D.

Gleim's CIA Test Prep: Part III: Business Analysis and Information Technology(1165 questions)

Copyright 2004 Gleim Publications, Inc. Page 68Printed for g j

Page 69: Part three cia_with_ answers

Answer (A) is incorrect because cash discounts encourage prompt payment.Answer (B) is incorrect because quantity discounts encourage large volume purchases.Answer (C) is incorrect because functional or trade discounts are provided to channel members in return for theperformance of certain functions, such as selling, storing, and record keeping.Answer (D) is correct. Seasonal discounts are designed to smooth production by the selling firm. For example, a skimanufacturer offers seasonal discounts to retailers in the spring and summer to encourage early ordering.

[183] Gleim #: 2.22 -- Source: CIA 1195 IV-76

In which product-mix pricing strategy is it appropriate for the seller to accept any price that exceeds the storage anddelivery costs for the product?

By-product pricing.A.Optional-product pricing.B.Captive-product pricing.C.Product-bundle pricing.D.

Answer (A) is correct. A by-product is a product of relatively minor importance generated during the production ofone or more other products. Its production entails no additional costs. Any amount received above the storage anddelivery costs for a by-product allows the seller to reduce the main product’s price to make it more competitive.Answer (B) is incorrect because optional products are offered for sale along with the main product. They are unlikelyto have a zero production cost, so the seller must receive a price above their storage and delivery costs.Answer (C) is incorrect because captive products must be used along with the main product, such as film for use witha camera. Sellers often make their profits on the captive products rather than on the main product, which is sold at alow price. The captive products therefore will be priced well above the storage and delivery costs.Answer (D) is incorrect because product bundles are combinations of products sold together at a reduced price, suchas season tickets for a theater. Products are bundled to promote the sale of certain items that consumers might nototherwise purchase. The combined price of the bundle must be low enough to encourage consumers to buy the bundlebut must recover production costs and provide some profit for the seller, so the price must exceed storage anddelivery costs.

[184] Gleim #: 2.23 -- Source: CMA 1296 4-6

Several surveys point out that most managers use full product costs, including unit fixed costs and unit variable costs, indeveloping cost-based pricing. Which one of the following is least associated with cost-based pricing?

Price stability.A.Price justification.B.Target pricing.C.Fixed-cost recovery.D.

Answer (A) is incorrect because full-cost pricing promotes price stability. It limits the ability to cut prices.Answer (B) is incorrect because full-cost pricing provides evidence that the company is not violating antitrust lawsagainst predatory pricing.Answer (C) is correct. A target price is the expected market price of a product, given the company’s knowledge of itscustomers and competitors. Hence, under target pricing, the sales price is known before the product is developed.Subtracting the unit target profit margin determines the long-term unit target cost. If cost-cutting measures do notpermit the product to be made at or below the target cost, it will be abandoned.Answer (D) is incorrect because full-cost pricing has the advantage of recovering the full long-term costs of theproduct. In the long term, all costs are relevant.

[185] Gleim #: 2.24 -- Source: Publisher

If a U.S. manufacturer’s price in the U.S. market is below an appropriate measure of costs and the seller has a reasonableprospect of recovering the resulting loss in the future through higher prices or a greater market share, the seller has engagedin

Collusive pricing.A.Dumping.B.Predatory pricing.C.Price discrimination.D.

Gleim's CIA Test Prep: Part III: Business Analysis and Information Technology(1165 questions)

Copyright 2004 Gleim Publications, Inc. Page 69Printed for g j

Page 70: Part three cia_with_ answers

Answer (A) is incorrect because collusive pricing involves a conspiracy to set higher prices.Answer (B) is incorrect because dumping is defined under U.S. law as sale by a non-U.S. company in the U.S. market of aproduct below its market value in the country where it was produced. Such sale is illegal if it threatens material injury to aU.S. industry.Answer (C) is correct. Predatory pricing is intentionally pricing below cost to eliminate competition and reduce supply.Federal statutes and many state laws prohibit the practice. The U.S. Supreme Court has held that pricing is predatory whentwo conditions are met:  (1) the seller’s price is below “an appropriate measure of its costs,” and (2) it has a reasonableprospect of recovering the resulting loss through higher prices or greater market share.Answer (D) is incorrect because price discrimination entails charging different prices to different customers for essentiallythe same product if the effect is to lessen competition substantially; to tend to create a monopoly; or to injure, destroy, orprevent competition.

[186] Gleim #: 2.25 -- Source: Publisher

Fulford Company applies the target pricing and costing approach. The following information about costs and revenues ofFulford’s product are available for the year just ended:

Unit sales 60,000Unit selling price $            400Cost of goods sold 13,200,000Value-chain operating

costs excluding production 7,920,000

Fulford plans to increase unit sales to 80,000 by reducing the product’s unit price to $320. If Fulford desires a unit targetoperating income of 12%, by what amount must it reduce the full cost per unit?

$32.00A.$38.40B.$70.40C.$80.00D.

Answer (A) is incorrect because $32.00 equals the current full cost per unit minus the new unit target price.Answer (B) is incorrect because $38.40 is the unit target operating income.Answer (C) is correct. Unit target operating income is $38.40 (12% × $320 unit target price). Hence, the unit targetfull cost is $281.60 ($320 – $38.40). The current full cost per unit is $352.00 [($13,200,000 CGS + $7,920,000 othervalue chain operating costs) ÷ 60,000 units sold], so the necessary reduction in the full cost per unit is $70.40($352.00 – $281.60).Answer (D) is incorrect because $80.00 equals the change in the unit price.

[187] Gleim #: 2.26 -- Source: Publisher

A company’s product has an expected 4-year life cycle from research, development, and design through its withdrawalfrom the market. Budgeted costs are

Upstream costs (R&D, design) $2,000,000Manufacturing costs 3,000,000Downstream costs (marketing,

distribution, customer service) 1,200,000After-purchase costs 1,000,000

The company plans to produce 200,000 units and price the product at 125% of the whole-life unit cost. Thus, the budgetedunit selling price is

$15A.$31B.$36C.$45D.

Gleim's CIA Test Prep: Part III: Business Analysis and Information Technology(1165 questions)

Copyright 2004 Gleim Publications, Inc. Page 70Printed for g j

Page 71: Part three cia_with_ answers

Answer (A) is incorrect because $15 is the budgeted unit manufacturing cost.Answer (B) is incorrect because $31 is the budgeted unit life-cycle cost.Answer (C) is incorrect because $36 is the budgeted unit whole-life cost.Answer (D) is correct. Whole-life costs include after-purchase costs (operating, support, repair, and disposal) incurred bycustomers as well as life-cycle costs (R&D, design, manufacturing, marketing, distribution, and research). Hence, thebudgeted unit whole-life cost is $36 [($2,000,000 + $3,000,000 + $1,200,000 + $1,000,000) ÷ 200,000 units], and thebudgeted unit selling price is $45 (125% × $36).

[188] Gleim #: 2.27 -- Source: CMA 696 1-20

The protected group under the Age Discrimination in Employment Act of 1967, as amended, is defined as anyone in whichage group?

35 through 60.A.40 through 75.B.40 and older.C.45 and older.D.

Answer (A) is incorrect because the act provides protection for individuals who are aged 40 and older.Answer (B) is incorrect because the act provides protection for individuals who are aged 40 and older.Answer (C) is correct. The Age Discrimination in Employment Act is designed to protect individuals aged 40 andolder from employment discrimination. The act has been amended to eliminate the mandatory retirement age.However, certain managerial employees are not protected by this amendment.Answer (D) is incorrect because the act provides protection for individuals who are aged 40 and older.

[189] Gleim #: 2.28 -- Source: Publisher

Evaluating performance is not done to

Determine the amount of nondiscriminatory benefits that each employee deserves.A.Assess the available human resources of the firm.B.Motivate the employees.C.Determine which employees deserve salary increases.D.

Answer (A) is correct. There are many reasons for evaluating performance. Evaluations reinforce accomplishments,help in assessing employee strengths and weaknesses, provide motivation, assist in employee development, permit theorganization to assess its human resource needs, and serve as a basis for wage increases. Nondiscriminatory benefitsare given to everyone in the organization in equal amounts regardless of title, pay, or achievement of objectives.Answer (B) is incorrect because it is a purpose of a performance evaluation.Answer (C) is incorrect because it is a purpose of a performance evaluation.Answer (D) is incorrect because it is a purpose of a performance evaluation.

[190] Gleim #: 2.29 -- Source: Publisher

When a manager generalizes from the evaluation of one or a few traits to the employee’s total performance, (s)he has made

A judgmental evaluation.A.An evaluation subject to the halo effect.B.A projection.C.An objective evaluation.D.

Answer (A) is incorrect because a judgmental evaluation is based on nonverifiable, subjective criteria.Answer (B) is correct. The halo effect occurs when the appraiser judges one or a few employee traits and carries overthis judgment to the evaluation of the employee’s other traits. The halo effect can be positive or negative.Answer (C) is incorrect because projection is the process of attributing one’s own traits to another person.Answer (D) is incorrect because an objective evaluation uses verifiable, often quantitative criteria.

[191] Gleim #: 2.30 -- Source: CIA 596 II-33

A manager discovers by chance that a newly hired employee has strong beliefs that are very different from the manager’sand from those of most of the other employees. The manager’s best course of action would be to

Gleim's CIA Test Prep: Part III: Business Analysis and Information Technology(1165 questions)

Copyright 2004 Gleim Publications, Inc. Page 71Printed for g j

Page 72: Part three cia_with_ answers

Facilitate the reassignment of the new hire as quickly as possible before this situation becomes disruptive.A.Ask the rest of the team for their reaction and act according to the group consensus.B.Take no action unless the new hire’s behavior is likely to cause harm to the organization.C.Try to counsel the new hire into more reasonable beliefs.D.

Answer (A) is incorrect because personal beliefs alone are not an appropriate basis for managerial action.Answer (B) is incorrect because personal beliefs alone are not an appropriate basis for managerial action.Answer (C) is correct. The only legitimate grounds on which the supervisor may take action is the employee’s behavior.Personal beliefs, such as those on religious and political matters, cannot be the basis of personnel actions. Discrimination onthe basis of personal beliefs could expose the organization to legal action.Answer (D) is incorrect because personal beliefs alone are not an appropriate basis for managerial action.

[192] Gleim #: 2.31 -- Source: Publisher

A company allows each of its departments to develop its own system for evaluating performance. Linda Ward, thepersonnel director, should communicate appraisal information to a new employee by

Presenting anything that is pertinent to the listener’s situation.A.Providing an overview of all systems within the company.B.Describing how her own performance is evaluated.C.Discussing each department’s evaluation system in detail.D.

Answer (A) is correct. The personnel director should tailor the discussion to the listener by describing how theemployee’s department evaluates performance and what is expected of him/her. The director should also obtainfeedback from the employee to determine if everything is clearly understood.Answer (B) is incorrect because discussing information that is irrelevant to the new employee’s appraisal informationwould confuse the new employee.Answer (C) is incorrect because discussing information that is irrelevant to the new employee’s appraisal informationwould confuse the new employee.Answer (D) is incorrect because discussing information that is irrelevant to the new employee’s appraisal informationwould confuse the new employee.

[193] Gleim #: 2.32 -- Source: Publisher

A disadvantage of separating performance evaluations from wage-increase decisions is that

Not enough emphasis is placed on short-run performance.A.Financial rewards may lose their motivational effect.B.Employees may not be motivated by good appraisals.C.The employee’s performance evaluation does not consider the financial status of the company overall.D.

Answer (A) is incorrect because an advantage of separating appraisals from wage increases is that more emphasis isplaced on long-term objectives and goals.Answer (B) is incorrect because this separation does not deprive money of its motivational power, but it doesemphasize other rewards, such as feelings of achievement and the recognition of superiors.Answer (C) is correct. The employee may not be motivated immediately by a good appraisal because of the delay inreceipt of any monetary reward. The evaluation may also not be taken as seriously by the employee if compensation isnot correlated with performance.Answer (D) is incorrect because an advantage of separating performance evaluations from wage-increase decisions isthat the employee’s good performance can be separated from the overall company’s bad financial performance.

[194] Gleim #: 2.33 -- Source: Publisher

A company that wishes to improve its rate of retention of its experienced employees might

Abolish its hire-from-within policy.A.Improve its fringe benefit package.B.Initiate job simplification programs.C.Set a mandatory retirement age.D.

Gleim's CIA Test Prep: Part III: Business Analysis and Information Technology(1165 questions)

Copyright 2004 Gleim Publications, Inc. Page 72Printed for g j

Page 73: Part three cia_with_ answers

Answer (A) is incorrect because reduced opportunity for advancement may motivate employees to find jobs elsewhere.Answer (B) is correct. Fringe benefits (e.g., pensions and profit-sharing plans) may be contingent on duration ofemployment. Accordingly, they motivate employees to remain with the company.Answer (C) is incorrect because job simplification may result in boredom, lessened job satisfaction, and a higher turnoverrate.Answer (D) is incorrect because abolition of a mandatory retirement age would be more appropriate.

[195] Gleim #: 2.34 -- Source: Publisher

A company that formerly paid certain management employees on a salary plus commission basis decided to compensate atest group solely with commissions. Performance of these employees declined. The most likely explanation for this result isthat

The employee received special attention.A.Compensation was a motivational factor.B.Compensation was not a hygiene factor.C.Increased concerns about security made the employees risk averse.D.

Answer (A) is incorrect because the famous Hawthorne studies suggest that the employees’ knowledge of theirspecial status would improve their output.Answer (B) is incorrect because Herzberg defines a motivational factor as one whose absence will not diminishperformance but whose presence will be a motivator.Answer (C) is incorrect because a performance decline followed a reduction in the security of compensation; hence,money was probably a hygiene (maintenance) factor according to Herzberg’s two-factor theory of motivation.Answer (D) is correct. A straight salary with commissions rewarded increased efforts while providing greater securityand a reduction in anxiety. Satisfaction of the need for security may have permitted the employees to adopt promisingand profitable but more risky strategies. Since people concerned about security needs tend to be risk averse, thechange in compensation method probably caused the performance decline.

[196] Gleim #: 2.35 -- Source: CIA 1195 III-22

The human resource department of an organization observed that accounting staff turnover was unusually high. Exitinterviews indicated that the accounting department work schedule was highly restrictive for accountants who had youngchildren. To improve the retention of skilled employees in the accounting department, the best solution would be to

Implement a program of job rotation within the accounting department.A.Promote job enlargement for the positions experiencing the greatest turnover.B.Provide job sharing and flextime opportunities for accounting department employees.C.Enrich the jobs of accounting department employees.D.

Answer (A) is incorrect because job rotation would not adequately address the scheduling issue.Answer (B) is incorrect because job enlargement would not adequately address the scheduling issue.Answer (C) is correct. Job sharing and flextime allow employees to adjust their work schedules and hours to betterachieve personal objectives. These programs can increase worker loyalty and motivation.Answer (D) is incorrect because job enrichment would not adequately address the scheduling issue.

[197] Gleim #: 2.36 -- Source: Publisher

The factor that would not contribute to retention of experienced employees would be

Initiation of job enrichment programs.A.Adherence to federal regulations regarding sex discrimination in paying and promoting workers.B.Paying new employees more than older ones.C.Adoption of flextime rules.D.

Gleim's CIA Test Prep: Part III: Business Analysis and Information Technology(1165 questions)

Copyright 2004 Gleim Publications, Inc. Page 73Printed for g j

Page 74: Part three cia_with_ answers

Answer (A) is incorrect because a more intrinsically satisfying job will improve the retention rate.Answer (B) is incorrect because employees subject to discrimination may be strongly motivated to find other jobs.Answer (C) is correct. When old employees discover that newcomers are being hired at higher salaries, the group hired firstwill be unhappy, feeling that their greater experience should warrant greater reward. In the short run, the newcomers will besatisfied. But it is possible that next year’s newcomers will be hired at still higher salaries, contributing to the overalldissatisfaction. Thus, such a policy will provide incentives for experienced workers to leave the company.Answer (D) is incorrect because greater freedom in determining his/her hours may enable a worker to remain with thecompany when other responsibilities (e.g., child care) might have compelled him/her to leave.

[198] Gleim #: 2.37 -- Source: Publisher

The value of retaining employees is determined by

Human asset accounting.A.Financial accounting.B.Cost accounting.C.Human resource planning.D.

Answer (A) is correct. Human resource or human asset accounting attempts to measure the value, and the changes invalue, of the organization’s investment in human assets. Although this “asset” is enormously valuable (sometimesestimated at two or three times the annual payroll), it is not shown in balance sheets or accounted for in earningsstatements. One experimental measurement approach is a sort of “present value” of human resources. Another is a“cost” approach, with dollar investments (training, customer goodwill, etc.) offset by reductions (e.g., retirement).Answer (B) is incorrect because this traditional accounting method does not value the organization’s human assets.Answer (C) is incorrect because this traditional accounting method does not value the organization’s human assets.Answer (D) is incorrect because it merely audits the skills of the current employees and forecasts needs.

[199] Gleim #: 2.38 -- Source: Publisher

An employee with a good background and years of experience earns a salary at the top of his/her range. Under thecompany’s compensation program, the employee must earn a promotion in order to increase his/her salary above the usualannual increase. Which of the following is most likely to be an effect on his/her behavior?

The employee may refuse new duties or tasks.A.The employee may become less productive.B.The employee may not be motivated to improve performance.C.The employee may seek a position with another company.D.

Answer (A) is incorrect because this behavior would undermine the employee’s chance for a promotion.Answer (B) is incorrect because this behavior would undermine the employee’s chance for a promotion.Answer (C) is incorrect because this behavior would undermine the employee’s chance for a promotion.Answer (D) is correct. When an employee can earn a desired salary increase only through a promotion, (s)he is likelyto be motivated to perform better. If this does not result in a promotion, (s)he will probably look for another job.

[200] Gleim #: 2.39 -- Source: Publisher

A company’s compensation program does not allow for salary increases based on above-average performance after anemployee reaches the top of a position’s salary range. It also pays some employees higher salaries because of theireducational qualifications. This company could improve the program by

Allowing each manager to set up his/her own system of salary increases.A.Providing only monetary compensation.B.Developing a system that equates years of experience with education.C.Decreasing the annual percentage increase in each salary range.D.

Gleim's CIA Test Prep: Part III: Business Analysis and Information Technology(1165 questions)

Copyright 2004 Gleim Publications, Inc. Page 74Printed for g j

Page 75: Part three cia_with_ answers

Answer (A) is incorrect because the company should administer a uniform system of salary increases.Answer (B) is incorrect because the company could improve its system if it provided other compensation, such as assistingwith an employee’s tuition or recognizing an employee who has given many years of service.Answer (C) is correct. A company with a compensation program that stops rewarding above-average performance at the topof each salary range and rewards education with higher compensation could improve the system in several ways. Theseinclude starting a bonus program for above-average performance, refusing to hire a person at the top of a salary range unless(s)he will be promoted quickly, and equating experience and education so that employees are treated fairly.Answer (D) is incorrect because the company would please all employees by increasing this percentage.

[201] Gleim #: 2.40 -- Source: Publisher

Which of the following methods of employee evaluation is best used in a situation where employees are heavily involved inteamwork?

360-degree performance appraisal.A.Attribution method.B.Critical incident technique.C.Behaviorally anchored rating scales (BARS).D.

Answer (A) is correct. The 360-degree performance appraisal is a model for employee assessment in the age ofteamwork. It is based on giving workers feedback from peers, customers, supervisors, and those who work for theemployee. Feedback is typically provided anonymously and is usually subjective.Answer (B) is incorrect because this is a nonsense term.Answer (C) is incorrect because the critical incident technique is based on having a list of critical aspects of a jobagainst which to compare the employee’s performance; it would usually not be effective in a team situation.Answer (D) is incorrect because BARS is based on specific job-related behaviors.

[202] Gleim #: 2.41 -- Source: Publisher

If a supervisor fails to give an employee a negative evaluation because of fear of damaging a good working relationship,this is known as the

Leniency error.A.Recency effect.B.Halo effect.C.Contrast error.D.

Answer (A) is correct. A leniency error is when a manager fails to give a negative evaluation because of fear ofdamaging a good working relationship with an employee.Answer (B) is incorrect because the recency effect means that the employee’s most recent behavior overshadowsoverall performance.Answer (C) is incorrect because the halo effect means the manager’s judgment on one positive trait affects the ratingon other traits.Answer (D) is incorrect because a contrast error is the tendency to rate people relative to other people, withoutconsideration of performance standards.

[203] Gleim #: 2.42 -- Source: CIA 591 III-11

During an audit of the personnel function, it was noted that numerous questions were asked of potential hirees. Which ofthe following questions would normally be appropriate when interviewing an inexperienced person applying for an entry-level internal auditing position?

How have you kept up your internal auditing education?A.What kind of reports have you written in previous jobs?B.What are your career goals?C.What is your marital status?D.

Gleim's CIA Test Prep: Part III: Business Analysis and Information Technology(1165 questions)

Copyright 2004 Gleim Publications, Inc. Page 75Printed for g j

Page 76: Part three cia_with_ answers

Answer (A) is incorrect because the inexperienced applicant may have just finished his/her education. Thus, continuingeducation may not be relevant.Answer (B) is incorrect because the applicant is inexperienced and thus may not have written reports in previous jobs.Answer (C) is correct. Sawyer’s Internal Auditing (IIA, 1988. p. 791) suggests the following questions for inexperiencedcandidates:

What is your concept of internal auditing?How did you hear about it?Why do you think you’d like it?What kinds of assignments would you like best?What are your outside interests?What are your personal goals?

Answer (D) is incorrect because federal law prohibits asking about marital status.

[204] Gleim #: 2.43 -- Source: CIA 1192 III-10

When faced with the problem of filling a newly created or recently vacated executive position, organizations must decidewhether to promote from within or to hire an outsider. One of the disadvantages of promoting from within is that

Internal promotions can have negative motivational effect on the employees of the firm.A.Internal promotions are more expensive to the organization than hiring an outsider.B.It is difficult to identify proven performers among internal candidates.C.Hiring an insider leads to the possibility of social inbreeding within the firm.D.

Answer (A) is incorrect because internal promotions usually lead to increased motivation among employees.Answer (B) is incorrect because internal promotions are less expensive. The firm can avoid the expenses associatedwith an executive search and certain training costs.Answer (C) is incorrect because it is more difficult to identify proven performers from among outside candidates thaninternal candidates.Answer (D) is correct. Hiring an internal candidate can lead to social inbreeding. Many firms look to externalcandidates for certain jobs because they bring a fresh perspective to the organization’s problems and may have moreup-to-date training or education.

[205] Gleim #: 2.44 -- Source: CIA 597 IV-80

The airlines have been leaders in the use of technology. Customers can make reservations either with an airline or through atravel agency. In this situation, a travel agency is classified as which type of distribution channel?

An intermediary.A.A jobber.B.A distributor.C.A facilitating agent.D.

Answer (A) is correct. Marketing intermediaries assist companies in promoting, selling, and distributing their goodsand services to ultimate consumers. For example, travel agents access an airline’s computerized reservation systemand make reservations for their customers without ever taking title to the ticket.Answer (B) is incorrect because jobbers buy from manufacturers, then resell the products.Answer (C) is incorrect because distributors, or wholesalers, usually have selective or exclusive distribution rights.Answer (D) is incorrect because facilitating agents assist in functions other than buying, selling, or transferring title.

[206] Gleim #: 2.45 -- Source: CIA 595 III-32

Just-in-time (JIT) inventory systems have been adopted by large manufacturers to minimize the carrying costs ofinventories. Identify the primary vulnerability of JIT systems.

Computer resources.A.Materials supply contracts.B.Work stoppages.C.Implementation time.D.

Gleim's CIA Test Prep: Part III: Business Analysis and Information Technology(1165 questions)

Copyright 2004 Gleim Publications, Inc. Page 76Printed for g j

Page 77: Part three cia_with_ answers

Answer (A) is incorrect because JIT systems can require significant computer resources, but they can also be maintainedmanually.Answer (B) is incorrect because contracts may have to be renegotiated with strict delivery and quality specifications, butthese changes usually occur over extended periods.Answer (C) is correct. JIT minimizes inventory by relying on coordination with suppliers to provide deliveries when theyare needed for production. Consequently, work stoppages at suppliers or transportation disruptions can cause almostimmediate work stoppages at the manufacturer plant.Answer (D) is incorrect because JIT can be implemented over an extended period or a shorter time frame depending on themanufacturer’s immediate needs.

[207] Gleim #: 2.46 -- Source: CIA 596 III-98

A company manufactures banana hooks for retail sale. The bill of material for this item and the parts inventory for eachmaterial required are as follows:

Bill of MaterialsRaw Material Quantity Required On HandWooden neck 1 0Wooden base 1 0Swag hook 1 300Wood screws 2 400Foot pads 4 1,000

An incoming order calls for delivery of 2,000 banana hooks in 2 weeks. The company has 200 finished banana hooks incurrent inventory. If no safety stocks are required for inventory, what are the company’s net requirements for swag hooksand screws needed to fill this order?

  Swag Hooks Wood Screws A. 1,500 1,400B. 1,500 3,200C. 1,700 3,600D. 1,800 3,600

Answer (A) is incorrect because 1,400 wood screws assumes that one wood screw is used per banana hook.Answer (B) is correct. The company needs 1,800 banana hooks (2,000 – 200) and therefore 1,800 swag hooks (1 ×1,800) and 3,600 wood screws (2 × 1,800). Given that 300 swag hooks and 400 wood screws are on hand, thecompany must obtain 1,500 swag hooks (1,800 – 300) and 3,200 wood screws (3,600 – 400).Answer (C) is incorrect because 1,700 swag hooks and 3,600 wood screws would be needed if no banana hooks werein current inventory.Answer (D) is incorrect because 1,800 swag hooks would be needed if no swag hooks were in current inventory.Also, 3,600 wood screws would be needed if no banana hooks were in current inventory.

[208] Gleim #: 2.47 -- Source: CIA 594 III-66

The purpose of the economic order quantity model is to:

Minimize the safety stock.A.Minimize the sum of the order costs and the holding costs.B.Minimize the inventory quantities.C.Minimize the sum of the demand costs and the backlog costs.D.

Answer (A) is incorrect because the basic EOQ model does not include safety stock.Answer (B) is correct. The purpose of the EOQ model is to minimize the sum of inventory order costs and holdingcosts. The EOQ equals the square root of: twice the annual demand multiplied by the variable cost per order, dividedby the unit periodic holding cost.Answer (C) is incorrect because, in the EOQ model, costs, not quantities, are to be minimized.Answer (D) is incorrect because quantity demanded is a variable in the model, but order costs, not demand costs, arerelevant. Backlogs are customer orders that cannot be filled immediately because of stockouts. Backlog costs are notquantified in the model.

Gleim's CIA Test Prep: Part III: Business Analysis and Information Technology(1165 questions)

Copyright 2004 Gleim Publications, Inc. Page 77Printed for g j

Page 78: Part three cia_with_ answers

[209] Gleim #: 2.48 -- Source: CIA 593 III-70Which of the following is used in determining the economic order quantity (EOQ)?

Regression analysis.A.Calculus.B.Markov process.C.Queuing theory.D.

Answer (A) is incorrect because regression analysis is used to fit a linear trend line to a dependent variable based onone or more independent variables.Answer (B) is correct. The primary business application of differential calculus is to identify the maxima or minimaof curvilinear functions. In business and economics, these are the points of revenue or profit maximization (maxima)or cost minimization (minima). The EOQ results from differentiating the total cost with regard to order quantity.Answer (C) is incorrect because Markov process models are used to study the evolution of certain systems overrepeated trials.Answer (D) is incorrect because queuing theory is a waiting-line technique used to balance desirable service levelsagainst the cost of providing more service.

[210] Gleim #: 2.49 -- Source: CIA 590 III-45

The EOQ (economic order quantity) model calculates the cost-minimizing quantity of a product to order, based on aconstant annual demand, carrying costs per unit per annum, and cost per order. For example, the EOQ is approximately 447units if the annual demand is 10,000 units, carrying costs are 1 currency unit per item per annum, and the cost of placing anorder is $10. What will the EOQ be if the demand falls to 5,000 units per annum and the carrying and ordering costsremain at $1 and $10, respectively?

316A.447B.483C.500D.

Answer (A) is correct. The EOQ formula is

If: D is the demand, or number of units used per year. A is the cost of placing one order. K is the cost of carrying one item for 1 year.

Answer (B) is incorrect because 447 is the EOQ when demand is 10,000 units per year.Answer (C) is incorrect because 483 results when annual demand is 11,664 units.Answer (D) is incorrect because 500 results when annual demand is 12,500 units.

[211] Gleim #: 2.50 -- Source: CIA 1190 III-42

A company stocks, maintains, and distributes inventory. The company decides to add to the safety stock and expeditedelivery for several product lines on a trial basis. For the selected product lines the company will experience

An increase in some costs but no change in the service level.A.A change in the service level.B.An increase in ordering, carrying, and delivery costs.C.A decrease in ordering, carrying, and delivery costs.D.

Gleim's CIA Test Prep: Part III: Business Analysis and Information Technology(1165 questions)

Copyright 2004 Gleim Publications, Inc. Page 78Printed for g j

Page 79: Part three cia_with_ answers

Answer (A) is incorrect because service will improve.Answer (B) is correct. Safety stocks are amounts held in excess of forecasted demand to avoid the losses associated withstockouts. Holding safety stocks improves the level of service to customers at the expense of increased holding costs.Answer (C) is incorrect because ordering costs will not increase. The fixed costs of the ordering department will beunaffected. Also, the department’s variable costs should not change because the EOQ will be the same. However, in the firstyear, an additional order may be necessary to increase the safety stock.Answer (D) is incorrect because delivery will increase under the new expedited delivery policy. Moreover, increasing thesafety stock increases carrying costs.

[212] Gleim #: 2.51 -- Source: CIA 1193 IV-26

Which of the following inventory items would be the most frequently reviewed in an ABC inventory control system?

Expensive, frequently used, high stock-out cost items with short lead times.A.Expensive, frequently used, low stock-out cost items with long lead times.B.Inexpensive, frequently used, high stock-out cost items with long lead times.C.Expensive, frequently used, high stock-out cost items with long lead times.D.

Answer (A) is incorrect because long, not short, lead times prompt a more frequent review.Answer (B) is incorrect because high, not low, stockout costs prompt a more frequent review.Answer (C) is incorrect because expensive, not inexpensive, items prompt a more frequent review.Answer (D) is correct. The ABC system is a method for controlling inventories that divides inventory items intothree groups:

Group A -- high-dollar value items, which account for a small portion(perhaps 10%) of the total inventory usage

 Group B -- medium-dollar value items, which may account for about 20%

of the total inventory items Group C -- low-dollar value items, which account for the remaining

70% of sales or usage

The ABC system permits the proper degree of managerial control to be identified and exercised over each group.Group A items are reviewed on a regular basis. Group B items may not have to be reviewed as often as group Aitems, but more often than group C items. For group C, extensive use of models and records is not cost effective. It ischeaper to order large quantities infrequently. The ABC method therefore reduces the safety-stock investmentbecause high-value items are frequently monitored and medium-value items are monitored more often thaninexpensive items. Frequent review can prevent stockouts and decrease inventory levels, and the cost of such reviewis minimized if it is limited to high- or medium-value items.

[213] Gleim #: 2.52 -- Source: CIA 1193 IV-25

The company uses a planning system that focuses first on the amount and timing of finished goods demanded and thendetermines the derived demand for raw material, components, and subassemblies at each of the prior stages of production.This system is referred to as

Economic order quantity.A.Materials requirements planning.B.Linear programming.C.Just-in-time purchasing.D.

Gleim's CIA Test Prep: Part III: Business Analysis and Information Technology(1165 questions)

Copyright 2004 Gleim Publications, Inc. Page 79Printed for g j

Page 80: Part three cia_with_ answers

Answer (A) is incorrect because the economic order quantity is a decision model that focuses on the trade-off betweencarrying and ordering costs.Answer (B) is correct. Materials requirements planning (MRP) is usually a computer-based information system designed toplan and control raw materials used in a production setting. It assumes that estimated demand for materials is reasonablyaccurate and that suppliers can deliver based upon this accurate schedule. It is crucial that delivery delays be avoidedbecause, under MRP, production delays are almost unavoidable if the materials are not on hand. An MRP system uses aparts list, often called a bill of materials, and lead times for each type of material to obtain materials just as they are neededfor planned production.Answer (C) is incorrect because linear programming is a decision model concerned with allocating scarce resources tomaximize profit or minimize costs.Answer (D) is incorrect because just-in-time purchasing involves the purchase of goods such that delivery immediatelyprecedes demand or use.

[214] Gleim #: 2.53 -- Source: CIA 592 III-62

An inventory planning method that minimizes inventories by arranging to have raw materials and subcomponents arriveimmediately preceding their use is called

A safety stock planning system.A.An economic order quantity model.B.A just-in-time inventory system.C.A master budgeting system.D.

Answer (A) is incorrect because safety stock is the inventory maintained in order to reduce the number of stockoutsresulting from higher-than-expected demand during lead time.Answer (B) is incorrect because the economic order quantity is the order quantity that minimizes total inventorycosts.Answer (C) is correct. JIT is a manufacturing philosophy popularized by the Japanese that combines purchasing,production, and inventory control. As with MRP, minimization of inventory is a goal; however, JIT also encompasseschanges in the production process itself. An emphasis on quality and a “pull” of materials related to demand are keydifferences between JIT and MRP. The factory is organized so as to bring materials and tools close to the point of userather than keeping them in storage areas. A key element of the JIT system is reduction or elimination of waste ofmaterials, labor, factory space, and machine usage. Minimizing inventory is the key to reducing waste. When a part isneeded on the production line, it arrives just in time, not before. Daily deliveries from suppliers are the ultimateobjective, and some Japanese users have been able to get twice-daily deliveries.Answer (D) is incorrect because the master budget is the detailed financial plan for the next period.

[215] Gleim #: 2.54 -- Source: CIA 595 III-98

One of the elements included in the economic order quantity (EOQ) formula is

Safety stock.A.Yearly demand.B.Selling price of item.C.Lead time for delivery.D.

Answer (A) is incorrect because the safety stock is not included in the basic EOQ formula.Answer (B) is correct. The basic EOQ formula is used to minimize the total of inventory carrying and ordering costs.The basic EOQ equals the square root of a fraction consisting of a numerator equal to the product of twice the unitperiodic demand and the variable cost per order and a denominator equal to the unit periodic carrying cost.Answer (C) is incorrect because the selling price of the item is not included in the basic EOQ formula.Answer (D) is incorrect because the lead time for delivery is not included in the basic EOQ formula.

[Fact Pattern #15]Using an EOQ analysis (assuming a constant demand), it is determined that the optimal order quantity is 2,500. The companydesires a safety stock of 500 units. A five-day lead time is needed for delivery. Annual inventory holding costs equal 25% of theaverage inventory level. It costs the company $4 per unit to buy the product, which it sells for $8. It costs the company $150 toplace a detailed order, and the monthly demand for the product is 4,000 units.

[216] Gleim #: 2.55 -- Source: CIA 595 III-99

(Refers to Fact Pattern #15)Annual inventory holding costs equal

Gleim's CIA Test Prep: Part III: Business Analysis and Information Technology(1165 questions)

Copyright 2004 Gleim Publications, Inc. Page 80Printed for g j

Page 81: Part three cia_with_ answers

$750A.$1,250B.$1,750C.$2,250D.

Answer (A) is incorrect because $750 results from subtracting instead of adding the cost of holding safety stock.Answer (B) is incorrect because $1,250 ignores safety stock.Answer (C) is correct. Given that demand is constant and the EOQ is 2,500 units, the average inventory level withoutregard to safety stock is 1,250 units (2,500 ÷ 2). Adding safety stock results in an average level of 1,750 units (1,250 +500). Given also that annual holding costs are 25% of average inventory and that unit cost is $4, total annual holding cost is$1,750 [(1,750 units × $4) × 25%].Answer (D) is incorrect because $2,250 results from double counting the cost of holding safety stock.

[217] Gleim #: 2.56 -- Source: CIA 595 III-100

(Refers to Fact Pattern #15)Total inventory ordering costs per year equal

$1,250A.$2,400B.$2,880C.$3,600D.

Answer (A) is incorrect because $1,250 equals the annual holding cost of the average inventory excluding safetystock.Answer (B) is incorrect because $2,400 assumes an EOQ of 3,000 units.Answer (C) is correct. Total annual demand is 48,000 units (4,000 per month × 12). Hence, total annual orderingcosts equal $2,880 [$150 cost per order × (48,000 units ÷ 2,500 EOQ)].Answer (D) is incorrect because $3,600 assumes an EOQ of 2,000 units.

[218] Gleim #: 2.57 -- Source: CIA 1195 III-66

An organization sells a product for which demand is uncertain. Management would like to ensure that there is sufficientinventory on hand during periods of high demand so that it does not lose sales (and customers). To do so, the organizationshould

Keep a safety stock.A.Use a just-in-time inventory system.B.Employ a materials requirements planning system.C.Keep a master production schedule.D.

Answer (A) is correct. Safety stock is inventory maintained to reduce the number of stockouts resulting from higher-than-expected demand during lead time. Maintaining a safety stock avoids the costs of stockouts, e.g., lost sales andcustomer dissatisfaction.Answer (B) is incorrect because a just-in-time inventory system involves the purchase of materials and production ofcomponents immediately preceding their use.Answer (C) is incorrect because materials requirements planning is a system for scheduling production andcontrolling the level of inventory for components with dependent demand.Answer (D) is incorrect because a master production schedule is a statement of the timing and amounts of individualitems to be produced.

[219] Gleim #: 2.58 -- Source: CIA 1195 III-97

The economic order quantity is the size of the order that minimizes total inventory costs which include ordering andholding costs. It can be calculated using the formula

If: Q = order size in units, D = annual demand in units, p = cost per purchase order, s = carrying cost per year for one unitof inventory. If the annual demand decreases by 36% the optimal order size will

Gleim's CIA Test Prep: Part III: Business Analysis and Information Technology(1165 questions)

Copyright 2004 Gleim Publications, Inc. Page 81Printed for g j

Page 82: Part three cia_with_ answers

Decrease by 20%.A.Increase by 20%.B.Increase by 6%.C.Decrease by 6%.D.

Answer (A) is correct. If D decreases by 36%, that is, from 100% to 64%, the EOQ will decrease by 20%.

Answer (B) is incorrect because the new EOQ decreases to 80% of its former value.Answer (C) is incorrect because 6% is the square root of 36%.Answer (D) is incorrect because 6% is the square root of 36%.

[220] Gleim #: 2.59 -- Source: CIA 1196 III-95

The economic order quantity, Q, is the size of the order that minimizes total inventory costs. These costs, which arecomposed of ordering and holding costs, can be computed using the following expression:

If:   TC  = total inventory costsQ = size of each orderD = annual demand in unitsF = fixed costs of orderingp = variable cost of placing one orders = holding cost per year for one unit of inventory

The following inventory information is available for an organization (costs stated in currency units):

Annual demand (D) 20,000 unitsVariable cost of placing one order (p) CU100Holding cost per unit (s) CU1Economic order quantity (Q) 2,000 unitsFixed cost of ordering CU0

If the organization decides to order 4,000 units at a time rather than 2,000 units, by how much will its total inventory costschange?

CU500 increase.A.CU1,000 increase.B.CU1,000 decrease.C.CU900 increase.D.

Answer (A) is correct. When the order size is 2,000 units, the variable inventory costs are CU2,000 {[(2,000 units ×CU1) ÷ 2] + [(20,000 units ÷ 2,000 units) × CU100]}. When the order size is 4,000 units, the variable inventorycosts are CU2,500 {[(4,000 units × CU1) ÷ 2] + [(20,000 units ÷ 4,000 units) × CU100]}. The increase in inventorycosts is CU500 (CU2,500 – CU2,000).Answer (B) is incorrect because a CU1,000 increase omits the change in the number of orders.Answer (C) is incorrect because increasing the order size increases total costs. The EOQ is 2,000 units.Answer (D) is incorrect because a CU900 increase results from an order size of 5,000 units.

[221] Gleim #: 2.60 -- Source: Publisher

Which of the following is the method of adding a standard markup to the cost of the product?

Gleim's CIA Test Prep: Part III: Business Analysis and Information Technology(1165 questions)

Copyright 2004 Gleim Publications, Inc. Page 82Printed for g j

Page 83: Part three cia_with_ answers

Penetration pricing.A.Going-rate pricing.B.Cost-based pricing.C.Price skimming.D.

Answer (A) is incorrect because penetration pricing is a new product pricing method in which an introductory price is setrelatively low to attract as many customers as possible.Answer (B) is incorrect because going-rate pricing is a method of pricing based largely on competitors’ prices.Answer (C) is correct. Cost-based pricing is a pricing method in which a standard markup or target profit is added to thecost of the product in determining the product’s sales price.Answer (D) is incorrect because price skimming sets the introductory price relatively high to attract buyers who are notconcerned with price.

[222] Gleim #: 2.61 -- Source: Publisher

Which of the following is not a component of physical distribution?

Transportation.A.Pricing.B.Location of retail outlets.C.Warehousing.D.

Answer (A) is incorrect because a physical distribution system includes transportation.Answer (B) is correct. Physical distribution (market logistics) involves planning, implementing, and controlling themovement of materials and final goods to meet customer needs while earning a profit. Physical distribution systemscoordinate suppliers, purchasing agents, marketers, channels, and customers. They include warehousing,transportation, and retail outlets.Answer (C) is incorrect because a physical distribution system includes the location of retail outlets.Answer (D) is incorrect because a physical distribution system includes warehousing.

[223] Gleim #: 2.62 -- Source: Publisher

Physical distribution is the moving of finished products to

Retail outlets.A.Shipping point.B.Consumers.C.Warehouses.D.

Answer (A) is incorrect because physical distribution is the efficient movement of finished products to consumers.Answer (B) is incorrect because physical distribution is the efficient movement of finished products to consumers.Answer (C) is correct. Physical distribution is the efficient management of supply chains. It controls value-addedflows from suppliers to consumers.Answer (D) is incorrect because physical distribution is the efficient movement of finished products to consumers.

[224] Gleim #: 2.63 -- Source: Publisher

Intensive distribution is most likely to be used for

Shopping goods.A.Specialty goods.B.Convenience goods.C.Unsought goods.D.

Gleim's CIA Test Prep: Part III: Business Analysis and Information Technology(1165 questions)

Copyright 2004 Gleim Publications, Inc. Page 83Printed for g j

Page 84: Part three cia_with_ answers

Answer (A) is incorrect because shopping goods are usually sold through selective distribution.Answer (B) is incorrect because specialty goods are usually sold through selective or exclusive distribution.Answer (C) is correct. Convenience goods are consumer goods and services that are usually low-priced and widelyavailable. Consumers buy them often and with a minimum of comparison and effort. Examples are soap and newspapers.Producers of convenience goods ordinarily use intensive distribution to sell their products through a large number of retailor wholesale units.Answer (D) is incorrect because unsought goods, e.g., life insurance, are often marketed through personal selling.

[225] Gleim #: 2.64 -- Source: CMA 691 4-4

A decrease in inventory order costs will

Decrease the economic order quantity.A.Increase the reorder point.B.Increase the economic order quantity.C.Decrease the holding cost percentage.D.

Answer (A) is correct. A decrease in inventory ordering costs should decrease the economic order quantity. Theeffect is that more orders can be made (of smaller quantities) without increasing costs. Accordingly, in the EOQmodel, ordering cost is a numerator value.Answer (B) is incorrect because the reorder point is based on lead time, not the EOQ model.Answer (C) is incorrect because the EOQ does not increase when inventory order costs decrease.Answer (D) is incorrect because the holding cost percentage will always be identical to the ordering cost percentagein accordance with the fundamental calculus underlying the EOQ model.

[226] Gleim #: 2.65 -- Source: CMA 1294 4-7

Which one of the following items is not directly reflected in the basic economic order quantity (EOQ) model?

Interest on invested capital.A.Inventory obsolescence.B.Public warehouse rental charges.C.Quantity discounts lost on inventory purchases.D.

Answer (A) is incorrect because interest on invested capital is considered in the basic EOQ model.Answer (B) is incorrect because inventory obsolescence is considered in the basic EOQ model.Answer (C) is incorrect because public warehouse rental charges is considered in the basic EOQ model.Answer (D) is correct. The basic EOQ model minimizes the sum of ordering (or setup) and carrying costs. Includedin the formula are annual demand, ordering (or setup) costs, and carrying costs. Carrying costs include warehousingcosts, insurance, spoilage, obsolescence, and interest on invested capital. The cost of the inventory itself and anyquantity discounts lost on inventory purchases are not components of the EOQ model.

[227] Gleim #: 2.66 -- Source: CMA 1286 5-10

In production management, product breakdown into component parts and lead times for procuring these parts is necessaryfor

A critical path method (CPM) system.A.A materials requirements planning (MRP) system.B.An economic order quantity (EOQ) system.C.An ABC system.D.

Gleim's CIA Test Prep: Part III: Business Analysis and Information Technology(1165 questions)

Copyright 2004 Gleim Publications, Inc. Page 84Printed for g j

Page 85: Part three cia_with_ answers

Answer (A) is incorrect because a CPM system is a project management and scheduling technique that determines thelongest time path from the first to the last event for a project.Answer (B) is correct. Materials requirements planning (MRP) is usually considered a computer-based information systemdesigned to plan and control raw materials used in a production setting. It assumes that estimated demand for materials isreasonably accurate and that suppliers can deliver based upon this accurate schedule. It is crucial that delivery delays beavoided because, under MRP, production delays are almost unavoidable if the materials are not on hand. An MRP systemuses a parts list, often called a bill of materials, and lead times for each type of material to obtain materials just as they areneeded for planned production.Answer (C) is incorrect because the EOQ model does not adjust for the dependent demand for the components of aninventory item.Answer (D) is incorrect because the main purpose of the ABC system is to increase control over items accounting for themost profit.

[228] Gleim #: 2.67 -- Source: CMA 1286 5-11

The inventory model that follows the concept that 80% of the value of an inventory is in 20% of the inventory items is the

ABC system.A.Economic order quantity (EOQ) model.B.Just-in-time inventory system.C.Materials requirements planning (MRP) system.D.

Answer (A) is correct. The ABC method of inventory control requires management to exert greatest control over theA classification items, which usually include a relatively small percentage of total items but a high percentage of thedollar volume. This method is analogous to the 80/20 rule, which says, for instance, that 20% of the customersaccount for 80% of the profit.Answer (B) is incorrect because the EOQ model is intended to minimize the sum of holding and ordering costs.Answer (C) is incorrect because a just-in-time system attempts to reduce holding costs by scheduling deliveries ofmaterials as closely as possible to when they are needed in production.Answer (D) is incorrect because an MRP system recognizes that the demand for one inventory item creates adependent demand for the components of that item.

[229] Gleim #: 2.68 -- Source: CMA 688 5-22

When a specified level of safety stock is carried for an item in inventory, the average inventory level for that item

Decreases by the amount of the safety stock.A.Is one-half the level of the safety stock.B.Increases by one-half the amount of the safety stock.C.Increases by the amount of the safety stock.D.

Answer (A) is incorrect because the average inventory level increases by the amount of the safety stock.Answer (B) is incorrect because the average inventory level increases by the amount of the safety stock.Answer (C) is incorrect because the average inventory level increases by the amount of the safety stock.Answer (D) is correct. If there is no safety stock, the average level of inventory will be one-half of the economicorder quantity. For example, if the EOQ is 500, the basic model predicts that the average inventory level will be 250.The company will have 500 units immediately after a purchase and zero immediately before the receipt of the nextpurchase (replenishment is assumed to be instantaneous). However, safety stock increases the average inventory levelby the amount of the safety stock. The modified EOQ model assumes that safety stock will never be used. Thus, if asafety stock of 100 is carried by the company in the example, the average inventory level will increase to 350.Inventory will be 600 units immediately upon receipt of a purchase and 100 units immediately before the receipt ofthe next purchase.

[230] Gleim #: 2.69 -- Source: CIA 597 IV-78

A distribution channel moves goods from producers to customers. Suppose a channel has four producers, each serving thesame four customers, and no middlemen. If a distributor is introduced, the number of contacts (among producers,customers, and the distributor) in the channel will

Be unaffected.A.Decrease from 8 to 4.B.Increase from 8 to 16.C.Decrease from 16 to 8.D.

Gleim's CIA Test Prep: Part III: Business Analysis and Information Technology(1165 questions)

Copyright 2004 Gleim Publications, Inc. Page 85Printed for g j

Page 86: Part three cia_with_ answers

Answer (A) is incorrect because the introduction of a distributor will affect the number of contacts in the marketing channel.Answer (B) is incorrect because the number of contacts declined from 16 to 8.Answer (C) is incorrect because the number of contacts decreased.Answer (D) is correct. The number of contacts without a distributor is 16 (four producers × four customers). The numberwith a distributor is 8 (four producers + four customers). Thus, the effort required of producers and consumers is reduced bythe distributor, thereby increasing marketing efficiency.

[231] Gleim #: 2.70 -- Source: CIA R98 III-32

An appropriate technique for planning and controlling manufacturing inventories, such as raw materials, components, andsubassemblies whose demand depends on the level of production is

Materials requirements planning.A.Regression analysis.B.Capital budgeting.C.Linear programming.D.

Answer (A) is correct. Materials requirements planning (MRP) is a system that translates a production schedule intorequirements for each component needed to meet the schedule. It is usually implemented in the form of a computer-based information system designed to plan and control raw materials used in production. It assumes that forecasteddemand is reasonably accurate and that suppliers can deliver based upon this accurate schedule. MRP is a centralizedpush-through system; output based on forecasted demand is pushed through to the next department or to inventory.Answer (B) is incorrect because regression analysis is used to fit a linear trend line to a dependent variable based onone or more independent variables.Answer (C) is incorrect because capital budgeting is the process of planning expenditures for long-lived assets. Itinvolves choosing among investment proposals using a ranking procedure.Answer (D) is incorrect because linear programming is a decision model concerned with allocating scarce resourcesto maximize profit or minimize costs.

[232] Gleim #: 2.71 -- Source: CIA 1194 IV-79

Price skimming involves

Setting a low introductory price.A.Setting a high introductory price.B.Setting different freight charges for customers in different zones.C.Using markups tied closely to the price paid for a product.D.

Answer (A) is incorrect because it refers to penetration pricing.Answer (B) is correct. Price skimming is used when a new product is introduced at the highest price possible giventhe benefits of the product. For market skimming to work, the product must appear to be worth its price, the costs ofproducing a small volume cannot be so high that they eliminate the advantage of charging more, and competitorscannot enter the market and undercut the price.Answer (C) is incorrect because it refers to zone pricing.Answer (D) is incorrect because it relates to pricing by intermediaries.

[233] Gleim #: 2.72 -- Source: CIA 1195 IV-75

Which of the following price adjustment strategies is designed to smooth production for the selling firm?

Cash discounts.A.Quantity discounts.B.Functional discounts.C.Seasonal discounts.D.

Answer (A) is incorrect because cash discounts encourage prompt payment.Answer (B) is incorrect because quantity discounts encourage large volume purchases.Answer (C) is incorrect because functional or trade discounts are provided to channel members in return for theperformance of certain functions, such as selling, storing, and record keeping.Answer (D) is correct. Seasonal discounts are designed to smooth production by the selling firm. For example, a skimanufacturer offers seasonal discounts to retailers in the spring and summer to encourage early ordering.

Gleim's CIA Test Prep: Part III: Business Analysis and Information Technology(1165 questions)

Copyright 2004 Gleim Publications, Inc. Page 86Printed for g j

Page 87: Part three cia_with_ answers

[234] Gleim #: 2.73 -- Source: Publisher

A seller that does which of the following is engaged in uniform delivered pricing?

Charges each customer its actual freight costs.A.Charges the same price, inclusive of shipping, to all customers regardless of location.B.Absorbs all of the actual freight charges.C.Sets differential freight charges for customers on the basis of their location.D.

Answer (A) is incorrect because it refers to FOB-origin pricing.Answer (B) is correct. A seller that uses uniform delivered pricing charges the same price, inclusive of shipping, toall customers regardless of their location.Answer (C) is incorrect because it refers to freight-absorption pricing.Answer (D) is incorrect because it refers to zone pricing.

[235] Gleim #: 2.74 -- Source: Publisher

Jabberwock Corporation’s corporate headquarters is in Carroll City. Jabberwock also has six distribution centers, one inCarroll City and the others located throughout North America. Jabberwock sets freight costs based on the distance of thedestination city from Carroll City. What is this pricing method?

Basing-point pricing.A.Zone pricing.B.FOB-origin pricing.C.Cost-plus pricing.D.

Answer (A) is correct. Basing-point pricing charges each customer the freight costs incurred from a specified city tothe destination, regardless of the actual point of origin of the shipment.Answer (B) is incorrect because zone pricing sets differential freight charges for customers on the basis of theirlocation.Answer (C) is incorrect because FOB-origin pricing charges each customer its actual freight costs.Answer (D) is incorrect because cost-plus pricing is not a geographical pricing method.

[236] Gleim #: 2.75 -- Source: Publisher

Manx Co. produces and sells two products. Product A costs $10 per unit and Product B costs $5 per unit. The As areselling very well, but sales of B are low. In order to increase sales of B, Manx has begun setting a price of $13 for one ofeach. This pricing method is

Product-bundle pricing.A.By-product pricing.B.Value pricing.C.Collusive pricing.D.

Answer (A) is correct. Product-bundle pricing entails selling combinations of products at a price lower than thecombined prices of the individual items. This strategy promotes sales of items consumers might not otherwise buy.Answer (B) is incorrect because it refers to the practice of accepting prices at any amount in excess of storing anddelivering the by-products.Answer (C) is incorrect because it refers to the practice of redesigning products to improve quality without raisingprices, or offering the same quality at lower prices.Answer (D) is incorrect because collusive pricing occurs when companies conspire to restrict output and setartificially high prices.

[237] Gleim #: 2.76 -- Source: Publisher

Selling below cost in other countries is called

Predatory pricing.A.Price discrimination.B.Dumping.C.Collusive pricing.D.

Gleim's CIA Test Prep: Part III: Business Analysis and Information Technology(1165 questions)

Copyright 2004 Gleim Publications, Inc. Page 87Printed for g j

Page 88: Part three cia_with_ answers

Answer (A) is incorrect because it is pricing products below cost. It may be done both domestically and in other countries.Answer (B) is incorrect because price discrimination is charging different prices to different customers.Answer (C) is correct. Dumping, which refers to selling below cost in other countries, is an inappropriate pricing tactic thatmay trigger retaliatory tariffs and other sanctions.Answer (D) is incorrect because collusive pricing occurs when companies conspire to restrict output and set artificially highprices.

[238] Gleim #: 2.77 -- Source: Publisher

All of the following are objectives of pricing except

Image oriented objectives.A.Profit maximization.B.Stabilization.C.Production maximization.D.

Answer (A) is incorrect because it is a pricing objective.Answer (B) is incorrect because it is a pricing objective.Answer (C) is incorrect because it is a pricing objective.Answer (D) is correct. Pricing objectives include

profit maximization, which assumes that all firms select the price that results in the highest profit1.target margin maximization, which is stated as a percentage ratio of profits to sales2.volume-oriented objectives, which refers to setting prices to meet target sales volumes or market shares3.image oriented objectives, which refers to setting prices to enhance the consumer’s perception of the firm’smerchandise mix

4.

stabilization objectives, which refers to setting prices to maintain a stable relationship between the firm’s pricesand the industry leader’s prices

5.

Price setting is not a factor in maximizing production.

[239] Gleim #: 2.78 -- Source: Publisher

Monolith Company, which produces Hal computers, uses a target pricing and costing approach. The following isMonolith’s costs and revenues for the year just ended:

Number of computers sold 10,000Sales price per computer $1,500Cost of goods sold $8,000,000Operating costs excluding production $5,500,000

Monolith plans to increase sales of computers to 15,000 in the next year by reducing the unit price to $1,250. If Monolithwishes to achieve a unit target operating income of 10%, by what amount must it reduce the full cost per unit?

$100A.$125B.$225C.$250D.

Answer (A) is incorrect because it equals the current full cost per unit minus the new unit target price.Answer (B) is incorrect because it is the unit target operating income.Answer (C) is correct. Unit target operating income is $125 (10% × $1,250 target price). Thus, the unit target fullcost is $1,125 ($1,250 – $125). The current full cost per unit is $1,350 [($8,000,000 cost of goods sold + $5,500,000operating costs excluding production) ÷ 10,000 units sold], so the necessary reduction in full cost per unit is $225($1,350 – $1,125).Answer (D) is incorrect because it is the change in unit price.

[240] Gleim #: 2.79 -- Source: CIA 1194 IV-78

Market-based pricing involves

Gleim's CIA Test Prep: Part III: Business Analysis and Information Technology(1165 questions)

Copyright 2004 Gleim Publications, Inc. Page 88Printed for g j

Page 89: Part three cia_with_ answers

Adding a standard markup to the cost of the product.A.Determining the price at which the product will earn a target profit.B.Basing prices on the product’s perceived value.C.Basing prices on competitors’ prices.D.

Answer (A) is incorrect because adding a standard markup to the cost of the product is cost-plus pricing.Answer (B) is incorrect because determining the price at which the product will earn a target profit is target-profit pricing.Answer (C) is correct. Market-based pricing involves basing prices on the product’s perceived value rather than on theseller’s cost. Nonprice variables in the marketing mix augment the perceived value. For example, a cup of coffee may have ahigher price at an expensive restaurant than at a fast food outlet.Answer (D) is incorrect because basing prices on competitors’ prices is going-rate pricing.

[241] Gleim #: 2.80 -- Source: CMA 1296 4-6

Several surveys point out that most managers use full product costs, including unit fixed costs and unit variable costs, indeveloping cost-based pricing. Which one of the following is least associated with cost-based pricing?

Price stability.A.Price justification.B.Target pricing.C.Fixed-cost recovery.D.

Answer (A) is incorrect because full-cost pricing promotes price stability. It limits the ability to cut prices.Answer (B) is incorrect because full-cost pricing provides evidence that the company is not violating antitrust lawsagainst predatory pricing.Answer (C) is correct. A target price is the expected market price of a product, given the company’s knowledge of itscustomers and competitors. Hence, under target pricing, the sales price is known before the product is developed.Subtracting the unit target profit margin determines the long-term unit target cost. If cost-cutting measures do notpermit the product to be made at or below the target cost, the product will be abandoned.Answer (D) is incorrect because full-cost pricing has the advantage of recovering the full long-term costs of theproduct. In the long term, all costs are relevant.

[242] Gleim #: 2.81 -- Source: Publisher

Value engineering is

A policy of seeking continuous improvement in all phases of company activities.A.A method of determining prices based on a cost determination.B.An element of a marketing mix strategy.C.A systematic approach to assessing all aspects of cost buildup for a product.D.

Answer (A) is incorrect because it refers to Kaizen.Answer (B) is incorrect because it refers to cost-plus pricing.Answer (C) is incorrect because value added engineering is not directly related to marketing mix strategies.Answer (D) is correct. Value engineering is a means of reaching targeted cost levels. It is a systematic approach toassessing all aspects of the value chain cost buildup for a product: R&D, design of products, design of processes,production, marketing, distribution, and customer service. The purpose is to minimize costs without sacrificingcustomer satisfaction.

[243] Gleim #: 2.82 -- Source: Publisher

Sunshine Co. is a producer of citrus flavored dessert products. Its most popular product is its key lime pies, which are madewith many fine ingredients, including egg yolks. Sunshine also sells the egg whites that are left over from its productionprocess. These egg whites are sold at any price that is greater than the cost of storing and delivering them. The pricing ofthe egg whites is

By-product pricing.A.Captive-product pricing.B.Product-bundle pricing.C.Value pricing.D.

Gleim's CIA Test Prep: Part III: Business Analysis and Information Technology(1165 questions)

Copyright 2004 Gleim Publications, Inc. Page 89Printed for g j

Page 90: Part three cia_with_ answers

Answer (A) is correct. By-product pricing usually sets prices at any amount in excess of storing and delivering by-products.Such prices allow the seller to reduce the costs and therefore the prices of the main products.Answer (B) is incorrect because captive-product pricing involves products that must be used with a main product.Answer (C) is incorrect because product-bundle pricing entails selling combinations of products at a price lower than thecombined prices of individual items.Answer (D) is incorrect because value pricing entails redesigning products to improve quality without raising prices oroffering the same quality at lower prices.

[244] Gleim #: 2.83 -- Source: Publisher

Which of the following is a definition of predatory pricing?

Pricing products below cost to destroy competitors.A.Pricing products at different prices for different customers.B.Conspiring to restrict output and set artificially high prices.C.Selling below cost in other countries.D.

Answer (A) is correct. Predatory pricing is the practice of pricing products below cost to destroy competitors. TheU.S. Supreme Court has held that a price is predatory if it is below an appropriate measure of costs and the seller hasa reasonable prospect of recovering its losses in the future through higher prices or greater market share.Answer (B) is incorrect because it refers to price discrimination.Answer (C) is incorrect because it refers to collusive pricing.Answer (D) is incorrect because it refers to dumping.

[245] Gleim #: 2.84 -- Source: Publisher

Prices vary directly with capacity usage under

Collusive pricing.A.Peak-load pricing.B.Market-based pricing.C.Cost-based pricing.D.

Answer (A) is incorrect because it refers to the illegal practice of conspiring to restrict output and set artificially highprices.Answer (B) is correct. Under peak-load pricing, prices vary directly with capacity usage. Thus, when idle capacity isavailable, that is, when demand falls, the price of a product or service tends to be higher given a peak-load pricingapproach.Answer (C) is incorrect because it involves basing prices on the product’s perceived value.Answer (D) is incorrect because it involves making a cost determination and setting a price that will recover costs andprovide a return on investment.

[246] Gleim #: 2.85 -- Source: Publisher

Which of the following is a false statement about cost-based pricing?

A cost-based price should cover costs and provide the desired return on investment.A.A commonly used cost-plus pricing formula is to add a markup percentage to the total cost.B.Variable costs may not be used as the basis for costs.C.A cost-plus price equals the cost plus a markup.D.

Answer (A) is incorrect because it is a true statement about cost-based pricing.Answer (B) is incorrect because it is a true statement about cost-based pricing.Answer (C) is correct. Cost-based pricing begins with a cost determination followed by setting a price that willrecover the value chain costs and provide the desired return on investment. A cost-plus price equals the cost plus amarkup. Cost may be defined in many ways. Most companies use either absorption manufacturing cost or total costwhen calculating the price. Variable costs may be used as the basis for cost, but then fixed costs must be covered bythe markup.Answer (D) is incorrect because it is a true statement about cost-based pricing.

Gleim's CIA Test Prep: Part III: Business Analysis and Information Technology(1165 questions)

Copyright 2004 Gleim Publications, Inc. Page 90Printed for g j

Page 91: Part three cia_with_ answers

[247] Gleim #: 2.86 -- Source: PublisherA company uses a target pricing and costing approach. The following is its costs and revenues for the current year:

Units sold 100,000Sales price per unit $           10Cost of goods sold 515,000Operating costs excluding production 335,000

The company intends to increase unit sales to 120,000 in the next year by reducing the unit price to $8. If the company is toachieve a unit target operating income of 15%, by what amount must it reduce the full cost per unit?

$0.50A.$1.20B.$1.50C.$1.70D.

Answer (A) is incorrect because it equals the current full cost per unit minus the new unit target price.Answer (B) is incorrect because it is the unit target operating income.Answer (C) is incorrect because it is the change in unit price.Answer (D) is correct. Unit target operating income is $1.20 (15% × $8 target price). Thus, the unit target full cost is$6.80 ($8 – $1.20). The current full cost per unit is $8.50 [$515,000 cost of goods sold + $335,000 operating costsexcluding production) ÷ 100,00 units sold], so the necessary reduction in full cost per unit is $1.70 ($8.50 – $6.80).

[248] Gleim #: 2.87 -- Source: Publisher

A situation in which companies conspire to restrict output and set artificially high prices is known as:

Collusive pricing.A.Predatory pricing.B.Price discrimination.C.Market-based pricing.D.

Answer (A) is correct. Collusive pricing is illegal. It involves two or more competitors conspiring to restrict outputand charge artificially high prices.Answer (B) is incorrect because predatory pricing occurs when a company sells products below cost to destroycompetitors.Answer (C) is incorrect because price discrimination occurs when different prices are charged to different customers.Answer (D) is incorrect because market-based pricing bases prices on a product’s perceived value rather than on cost.

[249] Gleim #: 2.88 -- Source: CIA 595 IV-74

Which of the following pricing policies involves the selling company setting freight charges to customers at the actualaverage freight cost?

Freight absorption pricing.A.Uniform delivered pricing.B.Zone pricing.C.FOB-origin pricing.D.

Answer (A) is incorrect because, in freight absorption pricing, the selling company absorbs all or part of the actualfreight charges. Customers are not charged actual delivery costs.Answer (B) is correct. In uniform delivered pricing, the company charges the same price, inclusive of shipping costs,to all customers regardless of their location. This price is the company’s average actual freight cost. Thus, bothnearby and distant customers are charged the same amount. This policy is easy to administer, permits the company toadvertise one price nationwide, and facilitates marketing to faraway customers.Answer (C) is incorrect because, in zone pricing, differential freight charges are set for customers on the basis of theirlocation. Customers are not charged actual average freight costs.Answer (D) is incorrect because, in FOB-origin pricing, each customer pays its actual freight costs.

Gleim's CIA Test Prep: Part III: Business Analysis and Information Technology(1165 questions)

Copyright 2004 Gleim Publications, Inc. Page 91Printed for g j

Page 92: Part three cia_with_ answers

[250] Gleim #: 2.89 -- Source: PublisherSetting the introductory price of a new product relatively high is

Market-based pricing.A.Price skimming.B.Penetration pricing.C.Pricing by intermediaries.D.

Answer (A) is incorrect because the price is based on buyer perceptions in market-based pricing, rather than simplybeing set at a high price.Answer (B) is correct. Price skimming is the practice of setting an introductory price relatively high to attract buyerswho are not concerned about price and to recover research and development costs rather quickly. The oppositeapproach is to use penetration pricing, which involves setting a relatively low price to gain deep market penetrationquickly.Answer (C) is incorrect because penetration pricing sets a relatively low price to gain deep market penetrationquickly.Answer (D) is incorrect because pricing by intermediaries does not describe the setting of a high introductory price.

[251] Gleim #: 2.90 -- Source: Publisher

Washboard Inc. charges customers the shipping costs incurred from Detroit to the destination regardless of the shippingpoint of origin. What is this pricing method commonly called?

Discriminatory pricing.A.Zone pricing.B.Basing-point pricing.C.FOB-origin pricing.D.

Answer (A) is incorrect because it is not discriminatory to charge everyone the same from the same point.Answer (B) is incorrect because zone pricing differs by region of the country to which a shipment is made.Answer (C) is correct. Basing-point pricing charges each customer the freight costs incurred from a specified city tothe destination regardless of the actual point of origin of the shipment.Answer (D) is incorrect because FOB-origin pricing charges each customer its actual freight costs.

[252] Gleim #: 2.91 -- Source: Publisher

A pricing strategy that sells combinations of products at a price lower than the combined prices of the individual items is

Predatory pricing.A.Going-rate pricing.B.Product-bundle pricing.C.Captive-product pricing.D.

Answer (A) is incorrect because predatory prices are prices set below cost to destroy competitors.Answer (B) is incorrect because this means charging the going rate in the market.Answer (C) is correct. Product-bundle pricing is a strategy to promote items consumers might not otherwise buy byselling combinations of products at a price lower than the prices of the individual items combined.Answer (D) is incorrect because captive-product pricing involves charging a low price for a main product (such as arazor), while razor blades are sold at a high markup. Once a person has a razor, he has to buy blades.

[253] Gleim #: 2.92 -- Source: Publisher

Which of the following statements is true with respect to value engineering?

Value engineering is a means of reaching targeted cost levels.I.Value engineering is the Japanese concept that emphasizes continuous improvement in all phases of companyactivities through numerous minor changes.

II.

Value engineering requires distinguishing between cost incurrence and locked-in costs, with an emphasis oncontrolling costs at the design stage.

III.

Gleim's CIA Test Prep: Part III: Business Analysis and Information Technology(1165 questions)

Copyright 2004 Gleim Publications, Inc. Page 92Printed for g j

Page 93: Part three cia_with_ answers

Items I and II only.A.Items I and III only.B.Items II and III only.C.Items I, II, and III.D.

Answer (A) is incorrect because the Japanese concept of continuous improvement is known as “Kaizen.”Answer (B) is correct. Value engineering is a means of reaching targeted cost levels. Value engineering requiresdistinguishing between cost incurrence and locked-in costs, with emphasis on controlling costs at the design stage. It is notJapanese.Answer (C) is incorrect because the Japanese concept of continuous improvement is known as “Kaizen.”Answer (D) is incorrect because the Japanese concept of continuous improvement is known as “Kaizen.”

[254] Gleim #: 2.93 -- Source: CIA 1195 IV-76

In which product-mix pricing strategy is it appropriate for the seller to accept any price that exceeds the storage anddelivery costs for the product?

By-product pricing.A.Optional-product pricing.B.Captive-product pricing.C.Product-bundle pricing.D.

Answer (A) is correct. A by-product is a product of relatively minor importance generated during the production ofone or more other products. Its production entails no additional costs. Any amount received above the storage anddelivery costs for a by-product allows the seller to reduce the main product’s price to make it more competitive.Answer (B) is incorrect because optional products are offered for sale along with the main product. They are unlikelyto have a zero production cost, so the seller must receive a price above their storage and delivery costs.Answer (C) is incorrect because captive products must be used along with the main product, such as film for use witha camera. Sellers often make their profits on the captive products rather than on the main product, which is sold at alow price. The captive products therefore will be priced well above the storage and delivery costs.Answer (D) is incorrect because product bundles are combinations of products sold together at a reduced price, suchas season tickets for a theater. Products are bundled to promote the sale of certain items that consumers might nototherwise purchase. The combined price of the bundle must be low enough to encourage consumers to buy the bundlebut must recover production costs and provide some profit for the seller, so the price must exceed storage anddelivery costs.

[255] Gleim #: 2.94 -- Source: Publisher

If a U.S. manufacturer’s price in the U.S. market is below an appropriate measure of costs and the seller has a reasonableprospect of recovering the resulting loss in the future through higher prices or a greater market share, the seller has engagedin

Collusive pricing.A.Dumping.B.Predatory pricing.C.Price discrimination.D.

Answer (A) is incorrect because collusive pricing involves a conspiracy to set higher prices.Answer (B) is incorrect because dumping is defined under U.S. law as sale by a non-U.S. company in the U.S. marketof a product below its market value in the country where it was produced. Such sale is illegal if it threatens materialinjury to a U.S. industry.Answer (C) is correct. Predatory pricing is intentionally pricing below cost to eliminate competition and reducesupply. Federal statutes and many state laws prohibit the practice. The U.S. Supreme Court has held that pricing ispredatory when two conditions are met: (1) the seller’s price is below “an appropriate measure of its costs,” and (2) ithas a reasonable prospect of recovering the resulting loss through higher prices or greater market share.Answer (D) is incorrect because price discrimination entails charging different prices to different customers foressentially the same product if the effect is to lessen competition substantially; to tend to create a monopoly; or toinjure, destroy, or prevent competition.

Gleim's CIA Test Prep: Part III: Business Analysis and Information Technology(1165 questions)

Copyright 2004 Gleim Publications, Inc. Page 93Printed for g j

Page 94: Part three cia_with_ answers

[256] Gleim #: 2.95 -- Source: PublisherFlesher Furniture Company applies a target pricing and costing approach. The following information about costs andrevenues of Flesher’s product are available for the year just ended:

Unit sales 60,000Unit selling price $              400Cost of goods sold 13,200,000

Value-chain operating costs excluding production 7,920,000

Flesher plans to increase unit sales to 80,000 by reducing the product’s unit price to $320. If Flesher desires a unit targetoperating income of 12%, by what amount must it reduce the full cost per unit?

$32.00A.$38.40B.$70.40C.$80.00D.

Answer (A) is incorrect because $32.00 equals the current full cost per unit minus the new unit target price.Answer (B) is incorrect because because $38.40 is the unit target operating income.Answer (C) is correct. Unit target operating income is $38.40 (12% × $320 unit target price). Hence, the unit targetfull cost is $281.60 ($320 – $38.40). The current full cost per unit is $352.00 [($13,200,000 CGS + $7,920,000 othervalue chain operating costs) &divide; 60,000 units sold], so the necessary reduction in the full cost per unit is $70.40($352.00 – $281.60).Answer (D) is incorrect because $80.00 equals the change in the unit price.

[257] Gleim #: 2.96 -- Source: Publisher

Labrador Hardware Co. has implemented the following discount policies:

Customers may trade in older models of tools when buying the latest models.Customers purchasing with cash receive a 10% discount. Credit customers who pay their balances in full within 30days receive a 5% discount.Customers who purchase supplies such as nails in bulk receive discounted prices.

Labrador uses all of the following discount methods except

Cash discounts.A.Quantity discounts.B.Allowances.C.Trade discounts.D.

Answer (A) is incorrect because it is a discount offered by Labrador.Answer (B) is incorrect because it is a discount offered by Labrador.Answer (C) is incorrect because it is a discount offered by Labrador.Answer (D) is correct. Cash discounts encourage prompt payment, improve cash flows, and avoid bad debts.Quantity discounts encourage large volume purchases. Trade (functional) discounts are offered to other members ofthe marketing channel for performing certain services, such as selling. Seasonal discounts are offered for sales out ofseason. They help smooth production. Allowances such as trade-ins reduce list prices.

[258] Gleim #: 2.97 -- Source: CMA 696 1-20

The protected group under the Age Discrimination in Employment Act of 1967, as amended, is defined as anyone in whichage group?

Between 35 and 60.A.Between 40 and 75.B.40 and older.C.45 and older.D.

Gleim's CIA Test Prep: Part III: Business Analysis and Information Technology(1165 questions)

Copyright 2004 Gleim Publications, Inc. Page 94Printed for g j

Page 95: Part three cia_with_ answers

Answer (A) is incorrect because the act provides protection for individuals who are aged 40 and older.Answer (B) is incorrect because the act provides protection for individuals who are aged 40 and older.Answer (C) is correct. The Age Discrimination in Employment Act is designed to protect individuals aged 40 and olderfrom employment discrimination. The act has been amended to eliminate the mandatory retirement age. However, certainmanagerial employees are not protected by this amendment.Answer (D) is incorrect because the act provides protection for individuals who are aged 40 and older.

[259] Gleim #: 2.98 -- Source: Publisher

Questions about which of the following can legally be asked in recruiting interviews?

Age.A.Number of children.B.Kind of military discharge.C.Criminal record (but only for security clearance purposes).D.

Answer (A) is incorrect because age may not be asked in a recruiting interview.Answer (B) is incorrect because number of children may not be asked in a recruiting interview.Answer (C) is incorrect because the kind of military discharge cannot be asked in a recruiting interview.Answer (D) is correct. In a recruiting interview, one may not ask about the applicant’s marital status, number ofchildren, height and weight, criminal record (except for the purpose of giving a security clearance), kind of militarydischarge, or age.

[260] Gleim #: 2.99 -- Source: Publisher

The halo effect may diminish the accuracy of the evaluation of employees. Which of the following is an example?

A manager rates some employees as very good and some as very poor.A.A manager’s judgment of one positive trait affects his/her rating of other traits.B.The most recent behavior overshadows overall performance.C.All personnel who work together are rated within the same narrow range.D.

Answer (A) is incorrect because this is not an example of the halo effect.Answer (B) is correct. The halo effect diminishes the accuracy of an evaluation when a manager’s judgment on onepositive trait affects the rating of other traits.Answer (C) is incorrect because the recency effect is when the most recent behavior overshadows overallperformance.Answer (D) is incorrect because the central tendency effect is when all personnel who work together are rated withinthe same narrow range.

[261] Gleim #: 2.100 -- Source: Publisher

Which of the following is not a characteristic of an effective evaluation system?

Relevant.A.Significant.B.Unbiased.C.Highly structured.D.

Answer (A) is incorrect because the evaluation system should be relevant.Answer (B) is incorrect because the evaluation system should be significant.Answer (C) is incorrect because the evaluation system should be unbiased.Answer (D) is correct. The system should be relevant, unbiased, significant, and practical. The evaluation systemshould be as objective, easy to use, clearly understood, and efficient as possible, but it is not necessary that it behighly structured.

[262] Gleim #: 3.1 -- Source: CIA 597 IV-1

The measurement basis most often used to report a long-term payable representing a commitment to pay money at adeterminable future date is

Gleim's CIA Test Prep: Part III: Business Analysis and Information Technology(1165 questions)

Copyright 2004 Gleim Publications, Inc. Page 95Printed for g j

Page 96: Part three cia_with_ answers

Historical cost.A.Current cost.B.Net realizable value.C.Present value of future cash flows.D.

Answer (A) is incorrect because historical cost is used to measure property, plant, and equipment and most inventories.Answer (B) is incorrect because some inventories are measured at current (replacement) cost.Answer (C) is incorrect because short-term receivables and some inventories are reported at net realizable value.Answer (D) is correct. The measurement basis most commonly adopted by enterprises in preparing their financialstatements is historical cost. However, it is usually combined with other measurement bases (attributes). The attribute usedto measure a long-term receivable or payable is the present or discounted value of its future cash flows.

[263] Gleim #: 3.2 -- Source: CIA 593 IV-25

In December year 1, catalogs were printed for use in a special promotion in January year 2. The catalogs were delivered bythe printer on December 13, year 1, with an invoice for 70,000 attached. Payment was made in January year 2. The 70,000should be reported as a deferred cost at the December 31, year 1 balance sheet date because of the

Matching principle.A.Revenue recognition principle.B.Reliability principle.C.Cost principle.D.

Answer (A) is correct. Matching is the simultaneous or combined recognition of revenues and expenses resultingdirectly and jointly from the same transactions or other events. Expenses should be associated with the revenues thatthey help to create. Because the catalogs are still on hand at the balance sheet date, they will not contribute to aninflow of economic benefits until the next period. Hence, the cost should be deferred and matched with the revenuesof the following period.Answer (B) is incorrect because the revenue recognition principle determines the period in which revenue isrecognized.Answer (C) is incorrect because reliable information is free of error and bias and is representationally faithful.Answer (D) is incorrect because the cost principle states that cost is the usual basis for recording most assets andliabilities.

[264] Gleim #: 3.3 -- Source: CIA 1192 IV-37

Because of inexact estimates of the service life and the residual value of a plant asset, a fully depreciated asset was sold in2004 at a material gain. This gain most likely should be reported

In the other revenues and gains section of the 2004 income statement.A.As part of sales revenue on the 2004 income statement.B.In the extraordinary item section of the 2004 income statement.C.As an adjustment to prior periods’ depreciation on the statement of changes in equity.D.

Answer (A) is correct. Income includes revenue and gains. Their essential nature is the same, and they are not treatedas separate financial statement elements. Revenues occur in the course of ordinary activities. Gains may or may notoccur in the course of ordinary activities. For example, gains may occur from the sale of noncurrent assets. Thus, thegain on the sale of a plant asset is not an operating item and should be classified in an income statement with separateoperating and nonoperating sections in the other revenues and gains section.Answer (B) is incorrect because the asset sold was not stock in trade.Answer (C) is incorrect because the transaction does not meet the criteria of an extraordinary item, which results froman event or transaction that is clearly different from ordinary activities and is not expected to recur frequently andregularly.Answer (D) is incorrect because the transaction is not the correction of a fundamental error in the financial statementsof a prior period.

[265] Gleim #: 3.4 -- Source: CIA 595 IV-29

Assume that employees confessed to a 500,000 inventory theft but are not able to make restitution. How should thismaterial fraud be shown in the company’s financial statements?

Gleim's CIA Test Prep: Part III: Business Analysis and Information Technology(1165 questions)

Copyright 2004 Gleim Publications, Inc. Page 96Printed for g j

Page 97: Part three cia_with_ answers

Classified as a loss and shown as a separate line item in the income statement.A.Initially classified as an accounts receivable because the employees are responsible for the goods. Because they cannot pay,the loss would be recognized as a write-off of accounts receivable.

B.

Included in cost of goods sold because the goods are not on hand, losses on inventory shrinkage are ordinary, and it wouldcause the least amount of attention.

C.

Recorded directly to retained earnings because it is not an income-producing item.D.

Answer (A) is correct. Expenses include losses. Their essential nature is the same, and they are not treated as separatefinancial statement elements. Losses may or may not occur in the course of ordinary activities. For example, they may resultfrom nonreciprocal transactions (e.g., theft), reciprocal transactions (e.g., a sale of plant assets), or from holding assets orliabilities. Losses are typically displayed separately.Answer (B) is incorrect because no restitution will be made. Thus, recording the item as a receivable, then writing it off, isnot consistent with the substance of the event.Answer (C) is incorrect because, although some inventory shrinkage is expected in the normal course of processing, fraud isabnormal. Hence, the item should be recorded as a loss.Answer (D) is incorrect because losses are included in the determination of net profit or loss.

[266] Gleim #: 3.5 -- Source: CIA 591 IV-44

The assets of a liquidating enterprise should be shown on the balance sheet at their

Undepreciated historical cost.A.Fair market value.B.Realizable value.C.Current cost.D.

Answer (A) is incorrect because a going concern should report assets at their undepreciated historical cost. Whenliquidation appears imminent, historical cost is inappropriate for balance sheet reporting.Answer (B) is incorrect because an enterprise facing liquidation is expected to dispose of its assets in a “forced” or“distressed” sale and is unlikely to realize the fair value amount. The net realizable value of the assets is theappropriate amount for reporting purposes.Answer (C) is correct. When liquidation is imminent, and the going concern assumption is no longer valid, the mostappropriate valuation method for assets is realizable value, which is the amount of cash currently obtainable by salein an orderly disposal.Answer (D) is incorrect because current cost is only appropriate when the going concern assumption is applicable andthe effects of changing prices are to be measured and reported in the financial statements.

[267] Gleim #: 3.6 -- Source: CIA 1190 IV-27

An objective of financial reporting is

Providing information useful to investors, creditors, donors, and other users for decision making.A.Assessing the adequacy of internal control.B.Evaluating management results compared with standards.C.Providing information on compliance with established procedures.D.

Answer (A) is correct. The objectives of financial reporting are concerned with the underlying goals and purposes ofaccounting. They are to provide information that (1) is useful to those making investment and credit decisions,assuming that those individuals have a reasonable understanding of business and economic activities; (2) is helpful tocurrent and potential investors and creditors and other users in assessing the amount, timing, and uncertainty of futurecash flows; and (3) discloses economic resources, claims to those resources, and the changes therein.Answer (B) is incorrect because assessing the adequacy of internal control is a function of internal auditing, notfinancial reporting.Answer (C) is incorrect because evaluating management results compared with standards is a function of internalauditing, not financial reporting.Answer (D) is incorrect because providing information on compliance with established procedures is a function ofinternal auditing, not financial reporting.

Gleim's CIA Test Prep: Part III: Business Analysis and Information Technology(1165 questions)

Copyright 2004 Gleim Publications, Inc. Page 97Printed for g j

Page 98: Part three cia_with_ answers

[268] Gleim #: 3.7 -- Source: CIA 592 IV-29An enterprise with total assets of 100,000,000 and net profit of 9,000,000 purchases staplers with an estimated life of 10years for 1,000. In connection with the purchase, the company debits miscellaneous expense. This scenario is most closelyassociated with which of the following concepts or principles?

Materiality and going concern.A.Relevance and neutrality.B.Reliability and comparability.C.Materiality and the balance between cost and benefit.D.

Answer (A) is incorrect because the going-concern principle relates to circumstances in which there is doubt as to theviability of the enterprise.Answer (B) is incorrect because relevance and reliability are two of the principal qualitative characteristics ofinformation in financial statements. Information is relevant if it permits users to predict the outcome of future eventsor confirm or correct their prior expectations. Reliability provides assurance that the information is reasonably freefrom error and bias and represents what it purports to represent. Thus, reliable information must be neutral, that is,free from error and bias.Answer (C) is incorrect because comparability is a principal qualitative characteristic. Financial statements must becomparable for the same enterprise over time and also among different enterprises. Information is relevant if itpermits users to predict the outcome of future events or confirm or correct their prior expectations.Answer (D) is correct. In principle, wasting assets should be capitalized and depreciated. However, the effect on thefinancial statements of expensing rather than capitalizing and depreciating the staplers is clearly not material giventhat they cost 1,000 and the enterprise has total assets of 100,000,000. The choice of treatment is not large enough toinfluence the decisions of financial statement users. The balance between benefit and cost is a pervasive constraint,not a qualitative characteristic. The benefits should exceed the cost of information. Specifically, the cost of producingthe information about depreciation expense over 10 years for the staplers probably is higher than the benefits of theinformation for decision making. Thus, the expedient procedure of expensing the 1,000 should be followed.

[269] Gleim #: 3.8 -- Source: CIA 1192 IV-38

In recording transactions, which of the following best describes the relation between expenses and losses?

Losses are extraordinary charges to income, whereas expenses are ordinary charges to income.A.Losses are material items, whereas expenses are immaterial items.B.Losses are expenses that may or may not arise in the course of ordinary activities.C.Expenses can always be prevented, whereas losses can never be prevented.D.

Answer (A) is incorrect because not all losses are extraordinary items.Answer (B) is incorrect because losses may be immaterial, and most expenses are material.Answer (C) is correct. Expenses are defined as “decreases in economic benefits during the accounting period in theform of outflows or depletions of assets or incurrences of liabilities that result in decreases in equity, other than thoserelating to distributions to equity participants.”  Expenses include losses. Thus, losses are not treated as separateelements. Expenses include items arising in the course of ordinary activities. Losses meet the definition of expensesbut may or may not occur in the course of ordinary activities. For example, losses may result from the sale ofnoncurrent assets or from natural disasters.Answer (D) is incorrect because some expenses cannot be prevented, or at least not in the short run. Some losses canbe prevented.

[270] Gleim #: 3.9 -- Source: CIA 1191 IV-29

To comply with the matching principle, the cost of labor services of an employee who participates in the manufacturing ofa product normally should be charged to the income statement in the period in which the

Work is performed.A.Employee is paid.B.Product is completed.C.Product is sold.D.

Gleim's CIA Test Prep: Part III: Business Analysis and Information Technology(1165 questions)

Copyright 2004 Gleim Publications, Inc. Page 98Printed for g j

Page 99: Part three cia_with_ answers

Answer (A) is incorrect because direct labor cost is a product (inventoriable) cost that is normally not recognized until sale.Answer (B) is incorrect because direct labor cost is a product (inventoriable) cost that is normally not recognized until sale.Answer (C) is incorrect because direct labor cost is a product (inventoriable) cost that is normally not recognized until sale.Answer (D) is correct. Recognition of expenses occurs concurrently with recognition of increases in liabilities or decreasesin assets. Expenses are recognized if the costs are directly associated with the earning of particular income items. Thisprocess is often called matching. Matching is simultaneous or combined recognition of the revenues and expenses thatresult directly and jointly from the same transactions or other events. This direct relationship is exemplified by the sale of aproduct. This transaction results in revenue (sales revenue) for receipt of cash or a receivable, the recognition of an expense(cost of sales) for the sacrifice of the product to a customer, and a decrease in inventory. The direct labor cost ofmanufacturing the product is absorbed by the finished goods inventory and is not recognized until sale.

[271] Gleim #: 3.10 -- Source: CIA 597 IV-12

A service enterprise keeps its accounting records on a cash basis. During the recent year, the enterprise collected 600,000from customers. The following information is also available:

  Beginning End of  of Year Year Accounts receivable 120,000 180,000Unearned revenue 0 15,000

What was the amount of service revenue for the year on an accrual basis?

525,000A.555,000B.645,000C.675,000D.

Answer (A) is incorrect because 525,000 deducts rather than adds the 60,000 increase in receivables.Answer (B) is incorrect because 555,000 deducts the increase in receivables and adds the increase in unearnedrevenue.Answer (C) is correct. The amount of service revenue for the year on an accrual basis equals 645,000 (600,000 cashcollected – 15,000 unearned revenue + 60,000 increase in accounts receivable).Answer (D) is incorrect because 675,000 adds the 15,000 increase in unearned revenue.

[Fact Pattern #16]

An enterprise is in the equipment rental business. Part of itsunadjusted trial balance at December 31, 2004, is as follows:

Cash 17,400Prepaid insurance 3,600Property, plant, and equipment 180,000Accumulated depreciation 32,000Accounts payable 12,000Share capital 60,000Retained earnings 38,000Rental revenue 171,000Salaries and wages expense 80,000Utilities expense 14,000

On October 1, 2004, the enterprise paid 18,000 to renew itsonly insurance policy for a 3-year period beginning on thatdate. This transaction has not been recorded.

Salaries and wages of 1,700 have been incurred but not paidas of December 31, 2004.

At December 31, 2004, the balance per bank statement was12,000. Outstanding checks amounted to 6,900. Interest of40 was credited to the enterprise’s account by the bankduring December, but has not yet been entered on the books.

[272] Gleim #: 3.11 -- Source: CIA 597 IV-4

(Refers to Fact Pattern #16)What amounts should be reported for prepaid insurance and insurance expense in the annual financial statements preparedat December 31, 2004?

Gleim's CIA Test Prep: Part III: Business Analysis and Information Technology(1165 questions)

Copyright 2004 Gleim Publications, Inc. Page 99Printed for g j

Page 100: Part three cia_with_ answers

  Prepaid Insurance  Insurance Expense A. 16,500 1,500B. 16,500 5,100C. 18,000 3,600D. 20,100 1,500

Answer (A) is incorrect because the existing balance in the prepaid insurance account should be expensed.Answer (B) is correct. The insurance policy in effect at the beginning of the year expired and was renewed during the year.Hence, the entire 3,600 beginning balance in the prepaid insurance account should be expensed. The amount of theprepayment on the new policy to be expensed is 1,500 [3 months elapsed × (18,000 ÷ 36-month duration of the policy)].The debit balance in prepaid insurance at year-end is therefore 16,500 (18,000 – 1,500), and total insurance expense is5,100 (3,600 + 1,500).Answer (C) is incorrect because insurance expense should be recognized for the last 3 months of the year.Answer (D) is incorrect because prepaid insurance should be credited and insurance expense debited for 3,600.

[273] Gleim #: 3.12 -- Source: CIA 597 IV-5

(Refers to Fact Pattern #16)The required adjusting entry at December 31, 2004 related to salaries and wages is

A. Salaries & wages expense 1,700Income summary 1,700

B. Salaries & wages payable 1,700Salaries & wages expense 1,700

C. Salaries & wages expense 1,700Salaries & wages payable 1,700

D. Income summary 1,700Salaries & wages payable 1,700

Answer (A) is incorrect because the income summary account is used for closing entries, not adjusting entries.Answer (B) is incorrect because the entry shown is a reversing entry that could be made at the beginning of thesubsequent period.Answer (C) is correct. An expense incurred but not yet paid is an accrued expense. The adjusting entry to record anaccrued expense of 1,700 for unpaid salaries and wages is to debit an expense account and credit a liability account.Answer (D) is incorrect because the debit should be to an expense account.

[274] Gleim #: 3.13 -- Source: CIA 597 IV-6

(Refers to Fact Pattern #16)The journal entry required to close the utilities expense account is

A. Utilities expense 14,000Income summary 14,000

B. Income summary 14,000Utilities expense 14,000

C. Rental revenue 14,000Utilities expense 14,000

D. Utilities expense 14,000Retained earnings 14,000

Answer (A) is incorrect because an expense account should be credited in a closing entry, and the income summaryaccount should be debited.Answer (B) is correct. All income statement account balances are closed either to a summary account (such asincome summary or revenue and expense summary) or to retained earnings. The normal balance of an expenseaccount is a debit; therefore, an expense account is credited in a closing entry.Answer (C) is incorrect because an expense account should not be closed to a revenue account. Such offsetting is notgood practice.Answer (D) is incorrect because an expense account should be credited in a closing entry.

Gleim's CIA Test Prep: Part III: Business Analysis and Information Technology(1165 questions)

Copyright 2004 Gleim Publications, Inc. Page 100Printed for g j

Page 101: Part three cia_with_ answers

[275] Gleim #: 3.14 -- Source: CIA 1195 IV-3The correct order of the following steps of the accounting cycle is

Posting, closing, adjusting, reversing.A.Posting, adjusting, closing, reversing.B.Posting, reversing, adjusting, closing.C.Adjusting, posting, closing, reversing.D.

Answer (A) is incorrect because adjusting entries are made prior to closing.Answer (B) is correct. The order of the steps in the accounting cycle is identification and measurement oftransactions and other events required to be recognized, journalization, posting from the journals to the ledgers, thedevelopment of a trial balance, adjustments to produce an adjusted trial balance, statement presentation, closing,taking a postclosing trial balance (optional), and making reversing entries (optional).Answer (C) is incorrect because reversing entries are made after adjustments and closing entries.Answer (D) is incorrect because posting is done prior to adjusting.

[276] Gleim #: 3.15 -- Source: CIA 596 IV-5

An enterprise has made all necessary adjusting entries and is now closing its accounts for the period. Dividends of 30,000were declared and distributed during the year. The entry to close the dividends account would be

A. Retained earnings 30,000Dividends 30,000

B. Dividends 30,000Retained earnings 30,000

C. Income summary 30,000Dividends 30,000

D. Dividends 30,000Income summary 30,000

Answer (A) is correct. Assuming an account entitled “dividends” or “dividends declared” is debited when dividendspayable is credited on the declaration date, it must be closed at the end of the period. The balance in this account isclosed directly to retained earnings. The effect of declaring and paying dividends is to reduce retained earnings, sothe entry is a debit to retained earnings and a credit to dividends.Answer (B) is incorrect because the closing entry should credit dividends and debit retained earnings.Answer (C) is incorrect because dividends is closed directly to retained earnings.Answer (D) is incorrect because dividends is closed directly to retained earnings by a credit.

[277] Gleim #: 3.16 -- Source: CIA 1195 IV-4

Which of the following statements is the best description of reversing entries?

The recording of reversing entries is a mandatory step in the accounting cycle.A.Reversing entries are made at the end of the next accounting period, after recording regular transactions of the period.B.Reversing entries are identical to the adjusting entries made in the previous period.C.Reversing entries are the exact opposite of the adjustments made in the previous period.D.

Answer (A) is incorrect because reversing entries are optional.Answer (B) is incorrect because reversing entries are made at the beginning of the next accounting period.Answer (C) is incorrect because reversing entries are the exact opposite of the adjustments made in the previousperiod.Answer (D) is correct. Reversing entries are made at the beginning of a period to reverse the effects of adjustingentries made at the end of the preceding period. They are optional entries made for the sake of convenience inrecording the transactions of the period. In order for reversing entries to reverse the prior adjustments, they must bethe exact opposite of the adjustments made in the previous period.

[278] Gleim #: 3.17 -- Source: CIA 1195 IV-27

The practice of recording advance payments from customers as a liability is an example of applying the

Gleim's CIA Test Prep: Part III: Business Analysis and Information Technology(1165 questions)

Copyright 2004 Gleim Publications, Inc. Page 101Printed for g j

Page 102: Part three cia_with_ answers

Going concern assumption.A.Monetary unit assumption.B.Historical cost principle.C.Revenue recognition principle.D.

Answer (A) is incorrect because the going concern assumption is that the business will have an indefinite life.Answer (B) is incorrect because the monetary unit assumption is that money is the common denominator by whicheconomic activity is conducted and that the monetary unit provides an appropriate basis for accounting measurement andanalysis.Answer (C) is incorrect because the historical cost principle reflects the practice that many assets and liabilities areaccounted for and reported on the basis of acquisition price.Answer (D) is correct. Recognition of revenue occurs when the flow of future economic benefits to the enterprise isprobable and such benefits are reliably measurable. Recording advance payments as a liability reflects a determination thatthe receipt of future economic benefits is not sufficiently certain to merit revenue recognition, given that the enterprise hasnot yet performed its obligations.

[279] Gleim #: 3.18 -- Source: CIA 1194 IV-24

An enterprise sells a durable good to a customer on January 1, 2003, and the customer is automatically given a 1-yearwarranty. The customer also buys an extended warranty package, extending the coverage for an additional 2 years to theend of 2005. At the time of the original sale, the company expects warranty costs to be incurred evenly over the life of thewarranty contracts. The customer has only one warranty claim during the 3-year period, and the claim occurs during 2004.The company will recognize income from the sale of the extended warranty

On January 1, 2003.A.In years 2004 and 2005.B.At the time of the claim in 2004.C.December 31, 2005, when the warranty expires.D.

Answer (A) is incorrect because the recognition of income from the sale of the extended warranty is deferred until theextended warranty period begins.Answer (B) is correct. Because warranty costs are expected to be incurred evenly over the life of the warrantycontracts, the income should be recognized on the straight-line basis over the life of the extended warranty contract.Answer (C) is incorrect because the income should be recognized evenly over the life of the contract. It is not relatedto the timing of the claims.Answer (D) is incorrect because income is recognized over the life of the warranty, not at expiration.

[280] Gleim #: 3.19 -- Source: CIA 596 IV-4

An enterprise had cash receipts from sales of 175,000 during 2004, of which 30,000 was unearned at the end of 2004. Atthe end of 2003, the company had 40,000 of unearned revenue, all of which was earned in 2004. The company’s salesrevenue for 2004 would be

145,000A.165,000B.175,000C.185,000D.

Answer (A) is incorrect because 145,000 equals 2004 revenue from 2004 sales.Answer (B) is incorrect because 165,000 adds, rather than subtracts, the 2004 unearned revenue and subtracts, ratherthan adds, the 2003 receipts for which revenue was earned in 2004.Answer (C) is incorrect because 175,000 equals the cash receipts for 2004.Answer (D) is correct. The sales revenue earned in 2004 equals 2004 cash receipts, minus any receipts in 2004 forwhich the revenue has not yet been earned, plus the revenue earned from cash receipts in 2003, or 185,000 (175,000– 30,000 + 40,000).

[281] Gleim #: 3.20 -- Source: CIA 590 IV-26

ABC operates a catering service that specializes in business luncheons for large corporations. ABC requires customers toplace their orders 2 weeks in advance of the scheduled events. ABC bills its customers on the tenth day of the monthfollowing the date of service and requires that payment be made within 30 days of the billing date. Conceptually, ABCshould recognize revenue from its catering services at the date when a

Gleim's CIA Test Prep: Part III: Business Analysis and Information Technology(1165 questions)

Copyright 2004 Gleim Publications, Inc. Page 102Printed for g j

Page 103: Part three cia_with_ answers

Customer places an order.A.Luncheon is served.B.Billing is mailed.C.Customer’s payment is received.D.

Answer (A) is incorrect because the certainty and measurability criteria are not met when the customer places an order.Answer (B) is correct. Income, which includes revenue and gains, should not be recognized until an increase in futureeconomic benefits related to an increase in an asset or a decrease in a liability is sufficiently certain and can be measuredreliably. The most common time at which these two conditions are met is when the product or merchandise is delivered orservices are rendered to customers. Thus, the enterprise has substantially accomplished what it must do to be entitled tofuture economic benefits when it serves the luncheon. It should then accrue a receivable and revenue.Answer (C) is incorrect because the date for billing is a matter of administrative procedure and convenience. The revenueshould be recognized at the date the service was performed.Answer (D) is incorrect because the revenue should be recognized at the point of performance of the service. To wait untilthe receivable is collected is to ignore the accrual basis of accounting, which is identified in the Framework for thePreparation and Presentation of Financial Statements as an underlying assumption of financial accounting.

[282] Gleim #: 3.21 -- Source: CIA 595 IV-11

If sales are accounted for using the installment method, which of the following is(are) only recognized in proportion to thecash collected on the sales during the period?

Sales.A.Sales and cost of sales.B.Sales and cost of sales and selling expenses.C.Sales and cost of sales and administrative expenses.D.

Answer (A) is incorrect because sales and cost of sales are recognized in proportion to cash collections.Answer (B) is correct. Under the installment method, the gross profit on sales (sales – cost of sales) is not recognizeduntil cash is collected. The proportion of cash collected on the sales during the accounting period determines theproportion of the gross profit on those sales that is recognized during the period. Hence, both sales and cost of salesare deferred.Answer (C) is incorrect because only the gross profit is deferred on sales for which cash has not yet been collected.Answer (D) is incorrect because only the gross profit is deferred on sales for which cash has not yet been collected.

[283] Gleim #: 3.22 -- Source: CIA 597 IV-10

Using the cost-recovery method of revenue recognition, profit on an installment sale is recognized

On the date of the installment sale.A.In proportion to the cash collections.B.After cash collections equal to the cost of goods sold have been received.C.On the date the final cash collection is received.D.

Answer (A) is incorrect because the accrual basis recognizes revenue on the date of the installment sale.Answer (B) is incorrect because the installment basis recognizes revenue in proportion to the cash collections.Answer (C) is correct. Under the cost-recovery method, no revenue is recognized until cash payments by the buyerexceed the seller’s cost of the merchandise sold. This method is appropriate when collection of the revenue is veryuncertain.Answer (D) is incorrect because, after the cash collections equal the cost of sales, revenue is to be recognized for anyfurther collections.

[284] Gleim #: 3.23 -- Source: CIA 1190 IV-28

On February 1, year 1 a computer software firm agrees to program a software package. Twelve payments of 10,000 on thefirst of each month are to be made, with the first payment March 1, year 1. The software is accepted by the client June 1,year 2. How much year 1 revenue should be recognized?

0A.100,000B.110,000C.120,000D.

Gleim's CIA Test Prep: Part III: Business Analysis and Information Technology(1165 questions)

Copyright 2004 Gleim Publications, Inc. Page 103Printed for g j

Page 104: Part three cia_with_ answers

Answer (A) is correct. Recognition of an element of financial statements (e.g., income, which includes revenue and gains)requires that two criteria be met. It must be probable that any future economic benefit associated with the item will flow toor from the enterprise, and the cost or value of the item must be measurable with reliability. The usual procedures forincome recognition, e.g., that income be earned, reflect these criteria. Thus, income is recognized when an increase in futureeconomic benefits is associated with an increase in an asset or a decrease in a liability. However, the enterprise has notsubstantially completed what it must do to be entitled to the benefits represented by the advance payment, and the receipt offuture economic benefits is not sufficiently certain to merit income recognition. Accordingly, a liability should berecognized because the entity has a current obligation arising from a past event that will require an outflow of economicbenefits, that is, to deliver the software or to refund the customer’s money. Thus, a liability for 100,000 and revenue of 0should be recognized for year 1.

NOTE:  This analysis assumes that the sale of the software is a sale either of goods or of services for which the appropriateconditions have not been met. Under IAS 18, Revenue Recognition, revenue is recognized for a sale of goods when theenterprise has transferred the significant risks and rewards of ownership, the enterprise has neither continuing managerialinvolvement to an extent associated with ownership nor effective control over the goods, the amount can be reliablymeasured, it is probable that the economic benefits will flow to the enterprise, and transaction costs can be reliablymeasured. For a sale of services, revenue is recognized when revenue can be reliably measured, it is probable that theeconomic benefits will flow to the enterprise, the stage of completion can be reliably measured, and the costs incurred andthe costs to complete can be reliably measured.Answer (B) is incorrect because income, which includes revenue and gains, should not be recognized until an increase infuture economic benefits related to an increase in an asset or a decrease in a liability is sufficiently certain and can bemeasured reliably.Answer (C) is incorrect because income, which includes revenue and gains, should not be recognized until an increase infuture economic benefits related to an increase in an asset or a decrease in a liability is sufficiently certain and can bemeasured reliably.Answer (D) is incorrect because income, which includes revenue and gains, should not be recognized until an increase infuture economic benefits related to an increase in an asset or a decrease in a liability is sufficiently certain and can bemeasured reliably.

[285] Gleim #: 3.24 -- Source: CIA 1193 IV-28

A building contractor has a fixed-price contract to construct a large building. It is estimated that the building will take 2years to complete. Progress billings will be sent to the customer at quarterly intervals. Which of the following describes thepreferable point for revenue recognition for this contract if the outcome of the contract can be estimated reliably?

After the contract is signed.A.As progress is made toward completion of the contract.B.As cash is received.C.When the contract is completed.D.

Answer (A) is incorrect because revenue is not recognized until progress has been made toward completion.Answer (B) is correct. Under the percentage-of-completion method, revenues and expenses are recognized based onthe stage of completion at the balance sheet date if the outcome of the contract can be estimated reliably. For a fixed-price contract, the outcome can be estimated reliably if (1) total revenue can be measured reliably, (2) it is probablethat the economic benefits of the contract will flow to the enterprise, (3) contract costs to complete and stage ofcompletion can be measured reliably, and (4) contract costs can be clearly identified and measured reliably so thatactual and estimated costs can be compared.Answer (C) is incorrect because the cash basis is inappropriate. An accrual method, that is, the percentage-of-completion method, should be used.Answer (D) is incorrect because the completed-contract method is not a permissible method.

[286] Gleim #: 3.25 -- Source: CIA 590 IV-32

The major distinction between the multiple-step and single-step income statement formats is the separation of

Operating and nonoperating data.A.Income tax expense and administrative expenses.B.Cost of goods sold expense and administrative expenses.C.The effect on income taxes of extraordinary items and the effect on income taxes of profit or loss from ordinaryactivities.

D.

Gleim's CIA Test Prep: Part III: Business Analysis and Information Technology(1165 questions)

Copyright 2004 Gleim Publications, Inc. Page 104Printed for g j

Page 105: Part three cia_with_ answers

Answer (A) is correct. The IASs do not require a particular income statement format, although, at a minimum, certain lineitems must be presented, including one for the results of operating activities. The single-step income statement provides onegrouping for income items and one for expense items. The “single step” is the one subtraction necessary to arrive at netprofit or loss. The multiple-step income statement matches operating income and expenses separately from nonoperatingitems.Answer (B) is incorrect because both formats separate income tax expense and administrative expenses.Answer (C) is incorrect because both formats separate cost of goods sold expense and administrative expenses.Answer (D) is incorrect because intraperiod income tax allocation procedures must be applied to both formats.

[287] Gleim #: 3.26 -- Source: CIA 1196 IV-5

At January 1, year 1, a sole proprietorship’s assets totaled 210,000, and its liabilities amounted to 120,000. During year 1,owner investments amounted to 72,000, and owner withdrawals totaled 75,000. At December 31, year 1, assets totaled270,000, and liabilities amounted to 171,000. The amount of net profit for year 1 was

0A.6,000B.9,000C.12,000D.

Answer (A) is incorrect because the enterprise did have a net profit.Answer (B) is incorrect because 6,000 mistakenly deducts the 72,000 and adds the 75,000.Answer (C) is incorrect because 9,000 is the difference between beginning and ending equity without taking intoconsideration the capital transactions with, and distributions to, the owner.Answer (D) is correct. Net profit or loss may be derived using the basic accounting equation (assets = liabilities +equity). Equity at 1/1/year 1 was 90,000 (210,000 of assets – 120,000 of liabilities). Equity at 12/31/year 1 was99,000 (270,000 – 171,000). Because owner transactions decreased net assets by 3,000 (72,000 investment – 75,000withdrawals), net profit must have been 12,000 [99,000 – (90,000 – 3,000)].

[288] Gleim #: 3.27 -- Source: CIA 596 IV-44

An enterprise has a 50% gross margin, general and administrative expenses of 50, interest expense of 20, and net profit of10 for the year just ended. If the corporate tax rate is 50%, the level of sales for the year just ended was

90A.135B.150C.180D.

Answer (A) is incorrect because 90 is the gross margin.Answer (B) is incorrect because 135 is 150% of the gross margin.Answer (C) is incorrect because 150 assumes profit before tax equals 50% of net profit.Answer (D) is correct. Net profit equals sales minus cost of sales, G&A expenses, interest, and tax. Given a 50% taxrate, profit before tax must have been 20 [10 net profit ÷ (1.0 – .5 tax rate)]. Accordingly, profit before interest andtax must have been 40 (20 profit before tax + 20 interest), and the gross margin (sales – cost of sales) must have been90 (40 profit before interest and tax + 50 G&A expenses). If the gross margin is 50% of sales, sales equals 180 (90gross margin ÷ .5).

[289] Gleim #: 3.28 -- Source: CIA 1193 IV-33

Which combination below explains the impact of credit card interest incurred and paid during the period on (1) equity onthe balance sheet and (2) the statement of cash flows?

Gleim's CIA Test Prep: Part III: Business Analysis and Information Technology(1165 questions)

Copyright 2004 Gleim Publications, Inc. Page 105Printed for g j

Page 106: Part three cia_with_ answers

  (2)  (1) Reflected on  Effect on Equity Statement of  on Balance Sheet Cash Flows as a(n) A. Decrease Investing outflowB. Decrease Operating or financing

outflowC. No effect Financing or investing

outflowD. No effect Operating outflow

Answer (A) is incorrect because interest payments are classified as an operating or financing outflow on the statement ofcash flows.Answer (B) is correct. Interest incurred is classified as interest expense on the income statement, which in turn reducesequity on the balance sheet by reducing retained earnings. According to IAS 7, cash payments for interest made by anenterprise that is not a financial institution may be classified on the statement of cash flows as an outflow of cash fromoperating or financing activities.Answer (C) is incorrect because credit card interest charges reduce equity.Answer (D) is incorrect because credit card interest charges reduce equity.

[Fact Pattern #17]Balance sheets on December 31, year 1 and December 31, year 2 are presented below:

Dec 31, year 1 Dec. 31, year 2Assets:

Cash 50,000 60,000Accounts receivable 95,000 89,000Allowance for

uncollectibleaccounts (4,000) (3,000)

Inventory 120,000 140,000Property, plant, and

equipment 295,000 340,000Accumulated

depreciation (102,000) (119,000)Total Assets 454,000 507,000

Liabilities and Equity:Trade accounts payable 62,000 49,000Interest payable 8,000 11,000Bonds payable 200,000 200,000Unamortized bond discount (15,000) (10,000)Equity 199,000 257,000

Total Liabilities andEquity 454,000 507,000

Additional information for the year 1:

Cash payments to suppliers of merchandise were180,000.

1.

Sales revenue was 338,000.2.3,000 of accounts receivable was written off.3.Equipment was acquired for 65,000.4.Depreciation expense was 30,000.5.Interest expense was 20,000.6.

[290] Gleim #: 3.29 -- Source: CIA 1196 IV-13

(Refers to Fact Pattern #17)Cost of goods sold in year 2 was

147,000A.160,000B.167,000C.180,000D.

Gleim's CIA Test Prep: Part III: Business Analysis and Information Technology(1165 questions)

Copyright 2004 Gleim Publications, Inc. Page 106Printed for g j

Page 107: Part three cia_with_ answers

Answer (A) is correct. Cost of goods sold equals beginning inventory, plus purchases, minus ending inventory. Todetermine cost of goods sold, purchases must be calculated. Purchases equal 167,000 (49,000 ending accounts payable +180,000 payments to suppliers – 62,000 beginning accounts payable). Thus, cost of goods sold equals 147,000 (120,000beginning inventory + 167,000 purchases – 140,000 ending inventory).Answer (B) is incorrect because 160,000 results from assuming that 180,000 of cash payments to suppliers equaledpurchases.Answer (C) is incorrect because 167,000 equals purchases.Answer (D) is incorrect because 180,000 is the amount of cash payments to suppliers.

[291] Gleim #: 3.30 -- Source: CIA 1196 IV-14

(Refers to Fact Pattern #17)Cash collections from customers in year 2 were

341,000A.338,000B.344,000C.335,000D.

Answer (A) is correct. Cash collections from customers equals beginning accounts receivable, plus sales revenue,minus accounts written off, minus ending accounts receivable. In year 2, cash collections from customers were341,000 (95,000 + 338,000 – 3,000 – 89,000).Answer (B) is incorrect because 338,000 is the sales revenue for the year.Answer (C) is incorrect because 344,000 includes the 3,000 of accounts written off.Answer (D) is incorrect because 335,000 is sales revenue minus accounts written off.

[292] Gleim #: 3.31 -- Source: CIA 1196 IV-15

(Refers to Fact Pattern #17)The carrying amount (cost minus accumulated depreciation) of property, plant, and equipment disposed of in year 2 was

7,000A.17,000B.20,000C.32,000D.

Answer (A) is correct. The cost of PPE disposed of is 20,000 (295,000 beginning PPE + 65,000 acquisitions –340,000 ending PPE). The accumulated depreciation is 13,000 (102,000 beginning accumulated depreciation +30,000 depreciation expense – 119,000 ending accumulated depreciation). Thus, the carrying amount of PPEdisposed of is 7,000 (20,000 cost of PPE – 13,000 accumulated depreciation).Answer (B) is incorrect because 17,000 is the difference between ending and beginning accumulated depreciation.Answer (C) is incorrect because 20,000 is the cost of the PPE disposed.Answer (D) is incorrect because 32,000 results from using the change in the PPE account without acquisitions minusthe accumulated depreciation.

[293] Gleim #: 3.32 -- Source: CIA 1196 IV-16

(Refers to Fact Pattern #17)Cash interest payments in year 2 were

8,000A.12,000B.20,000C.25,000D.

Answer (A) is incorrect because 8,000 is the beginning interest payable.Answer (B) is correct. The interest payable credited in year 1 was 15,000 (20,000 interest expense – 5,000 amortizedbond discount). Thus, the cash interest payment was 12,000 (8,000 beginning interest payable + 15,000 interestpayable credited in year 1 – 11,000 ending interest payable).Answer (C) is incorrect because 20,000 is the interest expense for year 1.Answer (D) is incorrect because 25,000 is the interest expense plus the amortized discount.

Gleim's CIA Test Prep: Part III: Business Analysis and Information Technology(1165 questions)

Copyright 2004 Gleim Publications, Inc. Page 107Printed for g j

Page 108: Part three cia_with_ answers

[294] Gleim #: 3.33 -- Source: CIA 1192 IV-32

A reader of a statement of cash flows wishes to analyze the major classes of gross cash receipts and gross cash paymentsfrom operating activities. Which methods of reporting cash flows from operating activities will supply that information?

Both the direct and indirect methods.A.Only the direct method.B.Only the indirect method.C.Neither method.D.

Answer (A) is incorrect because only the direct method supplies information about major classes of gross cashreceipts and payments related to operating activities.Answer (B) is correct. The statement of cash flows may report cash flows from operating activities in either anindirect or a direct format. The direct format reports the major classes of operating cash receipts and cash payments asgross amounts. The indirect presentation adjusts net profit or loss to the same amount of net cash from operatingactivities that would be determined in accordance with the direct method. To arrive at this amount, the indirectmethod adjusts net profit or loss for the effects of noncash transactions, deferrals or accruals of past or futureoperating cash flows, and income or expense related to financing or investing activities.Answer (C) is incorrect because the direct method, rather than the indirect method, supplies information about majorclasses of gross cash receipts and payments related to operating activities.Answer (D) is incorrect because the direct method reports major classes of gross cash receipts and payments fromoperating activities.

[295] Gleim #: 3.34 -- Source: CIA 1195 IV-34

In the statement of cash flows, the payment of cash dividends appears in the <List A> activities section as a <List B> ofcash.

  List A List B A. Operating or investing SourceB. Operating or financing UseC. Investing or financing UseD. Investing Source

Answer (A) is incorrect because payment of cash dividends is a use of cash for an operating or financing activity.Answer (B) is correct. According to IAS 7, dividends paid may be treated as a cash outflow from financing activitiesbecause they are a cost of obtaining resources from owners. However, they may also be treated as operating items tohelp determine the enterprise’s ability to pay dividends from operating cash flows.Answer (C) is incorrect because payment of cash dividends is a use of cash for an operating or financing activity.Answer (D) is incorrect because payment of cash dividends is a use of cash for an operating or financing activity.

Gleim's CIA Test Prep: Part III: Business Analysis and Information Technology(1165 questions)

Copyright 2004 Gleim Publications, Inc. Page 108Printed for g j

Page 109: Part three cia_with_ answers

[296] Gleim #: 3.35 -- Source: CIA 596 IV-10The comparative balance sheet for an enterprise that had net profit of 150,000 for the year ended December 31, 2004, andpaid 125,000 of dividends during 2004 is as follows:

  12/31/04 12/31/03 Cash 150,000 180,000Accounts receivable 200,000 220,000 Total assets 350,000 400,000 Payables 80,000 160,000Share capital 130,000 125,000Retained earnings 140,000 115,000 Total 350,000 400,000 

If dividends paid are treated as a cost of obtaining financial resources, the amount of net cash from operating activitiesduring 2004 was

70,000A.90,000B.150,000C.210,000D.

Answer (A) is incorrect because 70,000 fails to add to net profit the reduction in accounts receivable.Answer (B) is correct. Net profit is adjusted to determine the net cash from operations. The payment of cashdividends is regarded as a cash flow from a financing activity. Hence, it is not a reconciling item. However, thedecrease in accounts receivable (220,000 – 200,000 = 20,000) during the period represents a cash inflow (collectionsof pre-2004 receivables) not reflected in 2004 net profit. Moreover, the decrease in payables (160,000 – 80,000 =80,000) indicates a cash outflow (payment of pre-2004 liabilities) that also is not reflected in 2004 net income.Accordingly, net cash from operations was 90,000 (150,000 + 20,000 – 80,000).Answer (C) is incorrect because 150,000 is net profit.Answer (D) is incorrect because 210,000 subtracts the reduction in receivables and adds the reduction in payables.

[297] Gleim #: 3.36 -- Source: CIA 595 IV-2

In the determination of cost of goods sold, <List A> must be <List B> cash payments for goods along with otheradjustments.

  List A List B A. An increase in accounts

payableAdded to

B. A decrease in accountspayable

Added to

C. An increase in inventory Added toD. A decrease in inventory Subtracted from

Answer (A) is correct. To convert from the cash basis (cash payments) to the accrual basis (cost of goods sold), anincrease in accounts payable must be added to cash payments for goods to determine net purchases. Net purchases isthen adjusted for the change in inventory to determine cost of goods sold.Answer (B) is incorrect because a decrease in accounts payable must be subtracted from, not added to, cashpayments.Answer (C) is incorrect because an increase in inventory must be subtracted from, not added to, cash payments tocalculate cost of goods sold.Answer (D) is incorrect because a decrease in inventory must be added to, not subtracted from, cash payments tocalculate cost of goods sold.

Gleim's CIA Test Prep: Part III: Business Analysis and Information Technology(1165 questions)

Copyright 2004 Gleim Publications, Inc. Page 109Printed for g j

Page 110: Part three cia_with_ answers

[298] Gleim #: 3.37 -- Source: CIA 1196 IV-1A corporation reported salaries expense of 190,000 for December of the current year. The following data are from itsrecords:

  Dec. 31 Nov. 30 Prepaid salaries 46,000 40,000Salaries payable 170,000 140,000

The amount of cash payments for salaries during December of the current year was

154,000A.166,000B.214,000C.226,000D.

Answer (A) is incorrect because 154,000 subtracts the 6,000 increase in prepaid salaries instead of adding it.Answer (B) is correct. An increase in prepaid salaries indicates that salaries expense is less than the cash paid forsalaries. An increase in salaries payable indicates that salaries expense is more than the cash paid for salaries. Thus,the amount of cash payments for salaries was 166,000 (190,000 salaries expense + 6,000 increase in prepaid salaries– 30,000 increase in salaries payable).Answer (C) is incorrect because 214,000 subtracts the increase in prepaid salaries instead of adding it and adds the30,000 increase in salaries payable instead of subtracting it.Answer (D) is incorrect because 226,000 adds the 30,000 increase in salaries payable instead of subtracting it.

[299] Gleim #: 3.38 -- Source: CIA 593 IV-44

In reconciling net profit on an accrual basis to net cash from operating activities, what adjustment is needed to net profitbecause of (1) an increase during the period in prepaid expenses and (2) the periodic amortization of premium on bondspayable?

  (1) (2)  Increase in Amortization of Premium  Prepaid Expenses on Bonds Payable A. Add AddB. Add DeductC. Deduct AddD. Deduct Deduct

Answer (A) is incorrect because the increase in prepaid expenses requires a deduction from net profit in thereconciliation.Answer (B) is incorrect because the increase in prepaid expenses requires a deduction from net profit in thereconciliation.Answer (C) is incorrect because amortization of premium on bonds payable requires a deduction from net profit inthe reconciliation.Answer (D) is correct. An increase in prepaid expenses indicates that cash outlays for expenses exceeded the relatedexpense incurred; thus, net profit exceeded net cash from operating activities, and a deduction is needed in thereconciliation. Also, the amortization of premium on bonds payable causes a reduction of interest expense but doesnot increase cash; therefore, net profit exceeds net cash from operating activities, and a deduction is needed in thereconciliation.

[300] Gleim #: 3.39 -- Source: CIA 1193 IV-34

An enterprise has publicly announced a detailed, formal plan to dispose in its entirety of a component of the enterprise thatrepresents a separate major line of business that is distinct operationally and financially. Which of the following is theproper treatment of the disclosures that should be made after the announcement?

As part of continuing operations.A.As a discontinuing operation.B.As an extraordinary item.C.As a prior-period item.D.

Gleim's CIA Test Prep: Part III: Business Analysis and Information Technology(1165 questions)

Copyright 2004 Gleim Publications, Inc. Page 110Printed for g j

Page 111: Part three cia_with_ answers

Answer (A) is incorrect because the component is to be terminated (discontinued).Answer (B) is correct. A discontinuing operation (DO) is a separate major line of business or geographical operating areathat is distinct for operational and reporting purposes. Furthermore, it is a component that the enterprise is disposing ofunder a single plan. The initial disclosure event (IDE) is the earlier of entry into a binding agreement to sell substantially allof the DO’s assets or the approval and announcement by the enterprise’s governing body of a detailed, formal plan ofdiscontinuance. Disclosures about a DO should be included in the financial statements beginning with the period of theIDE.Answer (C) is incorrect because discontinuing operations are not extraordinary items. They are not clearly distinct fromordinary activities.Answer (D) is incorrect because disclosures begin with the period of the IDE.

[301] Gleim #: 3.40 -- Source: CIA 590 IV-35

Assuming all of the following involve material amounts, which is most likely to be classified as an extraordinary item in theincome statement?

A loss because of an expropriation of assets by a foreign government.A.A loss because of adjustments of accruals on long-term contracts.B.A gain because of the disposal of assets associated with a discontinuing operation of a business.C.A loss because of a lawsuit that resulted from charges of patent infringement. The company had unsuccessfullydefended a similar suit 5 years ago.

D.

Answer (A) is correct. Net profit or loss includes profit or loss from ordinary activities and extraordinary items. Bothare disclosed on the face of the income statement. Extraordinary items result from events or transactions that areclearly different from ordinary activities and are not expected to recur frequently and regularly. Examples are anexpropriation of assets or a natural disaster. Each extraordinary item is separately disclosed. However, a total may bedisclosed on the income statement if the nature and amount of each item are disclosed in the notes.Answer (B) is incorrect because an extraordinary item must be clearly different from ordinary activities.Answer (C) is incorrect because an extraordinary item must be clearly different from ordinary activities.Answer (D) is incorrect because an extraordinary item must be clearly different from ordinary activities.

[302] Gleim #: 3.41 -- Source: CIA 595 IV-21

An enterprise changes its method of accounting for depreciation during the current year because it believes that the resultwill be a more appropriate presentation in the financial statements. In its income statement for the year, how should theenterprise report the adjustment resulting from the change in accounting policy if the alternative treatment allowed by theIASs is used?

Not disclosed in the financial statements.A.Reported as an adjustment to beginning retained earnings.B.Disclosed as a separate type of depreciation expense, directly following depreciation expense for the current year.C.Included in the determination of net profit or loss for the current period.D.

Answer (A) is incorrect because changes in accounting policies must be disclosed in the financial statements for theperiod.Answer (B) is incorrect because the benchmark treatment reports an adjustment to beginning retained earnings.Answer (C) is incorrect because the cumulative effect on net profit or loss should be reported.Answer (D) is correct. Under IAS 8, Net Profit or Loss for the Period, Fundamental Errors, and Changes inAccounting Policies, the benchmark treatment provides that a change in accounting policy should be appliedretrospectively unless any resulting adjustment that relates to prior periods is not reasonably determinable. If theadjustment to the opening balance of retained earnings cannot be reasonably determined, the change in accountingpolicy should be applied prospectively. Given that the benchmark treatment is not indicated, the allowed alternativetreatment, that is, a cumulative-effect adjustment included in the determination of current net profit or loss, ispermissible unless the amount to be included in current net profit or loss is not reasonably determinable. In that case,the allowed alternative treatment provides for the change in accounting policy to be applied prospectively. In thecurrent period, a change in depreciation methods is reported by a cumulative-effect adjustment under the allowedalternative treatment. Pro forma comparative information also should be presented if practicable.

[303] Gleim #: 3.42 -- Source: CIA 591 IV-38

Which of the following is a unique reporting problem associated with the determination of the results of operations for aninterim period?

Gleim's CIA Test Prep: Part III: Business Analysis and Information Technology(1165 questions)

Copyright 2004 Gleim Publications, Inc. Page 111Printed for g j

Page 112: Part three cia_with_ answers

Advertising and similar costs expensed in one interim period may benefit other interim periods in the same annual period.A.Cost of goods sold for an interim period reflects only the amount of product cost applicable to sales revenue recognized inthe interim period.

B.

Depreciation for an interim period represents an estimate.C.An extraordinary loss occurring in the second quarter must be prorate d over the last three interim periods of the year.D.

Answer (A) is correct. Costs and expenses other than product costs should be either charged to income in interim periods asincurred or allocated among interim periods based upon the benefits received. Accordingly, costs such as advertising shouldbe deferred in an interim period if the benefits extend beyond that period; otherwise, they should be expensed as incurred.But such a determination is difficult, and deferral raises the additional issue of how the deferred costs should be allocatedamong quarters. Thus, many companies expense the costs as incurred even though they may benefit other interim periods inthe same annual period.Answer (B) is incorrect because the only product costs appropriate to expense in an interim period are the ones related tothe revenue transactions recognized in the same interim period.Answer (C) is incorrect because the annual depreciation amount is an estimate. The depreciation amount for an interimperiod is simply a pro rata amount of the annual estimate.Answer (D) is incorrect because an extraordinary item is to be reported in the interim period in which it occurs rather thanallocated to multiple interim periods. This approach is consistent with the way extraordinary items are handled on an annualbasis.

[304] Gleim #: 3.43 -- Source: CIA 1191 IV-42

An extraordinary gain occurs in the second fiscal quarter. How should the gain be accounted for?

Recognized in full in the second quarter.A.Recognized equally over the second, third, and fourth quarters.B.Recognized only in the annual financial statements.C.Recognized equally in each quarter, by restating the first quarter.D.

Answer (A) is correct. Extraordinary items result from events or transactions that are clearly different from ordinaryactivities and are not expected to recur frequently and regularly. Examples are an expropriation of assets or a naturaldisaster. Extraordinary items should be disclosed separately and included in the determination of net profit for theinterim period in which they occur. Gains and losses similar to those that would not be deferred at year-end shouldnot be deferred to later interim periods of the same year. Hence, the extraordinary gain should not be prorated.Answer (B) is incorrect because the gain should be recognized in full in the second quarter.Answer (C) is incorrect because the gain should be recognized in full in the second quarter.Answer (D) is incorrect because the gain should be recognized in full in the second quarter.

[305] Gleim #: 3.44 -- Source: CIA 1192 IV-26

A newly acquired plant asset is to be depreciated over its useful life. The rationale for this process is the

Economic entity assumption.A.Monetary unit assumption.B.Materiality assumption.C.Going concern assumption.D.

Answer (A) is incorrect because the economic entity assumption provides that economic activity can be identifiedwith a particular unit of accountability.Answer (B) is incorrect because the monetary unit assumption provides that all transactions and events can bemeasured in terms of a common denominator, for instance, the dollar.Answer (C) is incorrect because the materiality assumption simply implies that items of insignificant value may beexpensed rather than capitalized and depreciated or amortized. The difference in treatment is not large enough toinfluence users if the item is not material.Answer (D) is correct. A basic feature of financial accounting is that the enterprise is assumed to be a going concernin the absence of evidence to the contrary. The going concern concept is based on the empirical observation thatmany enterprises have an indefinite life. The reporting entity is assumed to have a life long enough to fulfill itsobjectives and commitments and therefore to depreciate wasting assets over their useful lives.

Gleim's CIA Test Prep: Part III: Business Analysis and Information Technology(1165 questions)

Copyright 2004 Gleim Publications, Inc. Page 112Printed for g j

Page 113: Part three cia_with_ answers

[306] Gleim #: 3.45 -- Source: CIA 1188 IV-33The following data were extracted from the financial statements of a company for the year ended December 31, 2004.

Net profit 70,000Depreciation expense 14,000Amortization of intangibles 1,000Decrease in accounts receivable 2,000Increase in inventories 9,000Increase in accounts payable 4,000Increase in plant assets 47,000Increase in share capital 31,000Decrease in short-term notes payable 55,000

There were no disposals of plant assets during the year. Based on the above, a statement of cash flows will report a netincrease in cash of

11,000A.17,000B.54,000C.69,000D.

Answer (A) is correct. Depreciation and amortization are noncash expenses and are added to net profit. A decrease inreceivables indicates that cash collections exceed sales on an accrual basis, so it is added to net profit. To account forthe difference between cost of goods sold (a deduction from net profit) and cash paid to suppliers, a two-stepadjustment of net profit is necessary. The difference between CGS and purchases is the change in inventory. Thedifference between purchases and the amount paid to suppliers is the change in accounts payable. Accordingly, theconversion of CGS to cash paid to suppliers requires deducting the inventory increase and adding the accountspayable increase. An increase in plant assets indicates an acquisition of plant assets, causing a decrease in cash, so itis deducted. An increase in share capital represents a cash inflow and is added to net profit. A decrease in short-termnotes payable is deducted from net profit because it reflects a cash outflow. Thus, cash increased by 11,000 (70,000net profit + 14,000 + 1,000 + 2,000 – 9,000 + 4,000 – 47,000 + 31,000 – 55,000).Answer (B) is incorrect because 17,000 results from subtracting the amortization and the decrease in receivables andadding the increase in inventories.Answer (C) is incorrect because 54,000 results from adjusting net profit for the increase in plant assets and theincrease in share capital only.Answer (D) is incorrect because 69,000 results from failing to make the adjustments for receivables, inventories,notes payable, and accounts payable.

[307] Gleim #: 3.46 -- Source: CIA 593 IV-27

An airline should recognize revenue from airline tickets in the period when

Passenger reservations are booked.A.Passenger reservations are confirmed.B.Tickets are issued.C.Related flights occur.D.

Answer (A) is incorrect because a sufficient degree of certainty that economic benefits will flow to the enterprise doesnot exist when the reservations are booked or confirmed.Answer (B) is incorrect because a sufficient degree of certainty that economic benefits will flow to the enterprise doesnot exist when the reservations are booked or confirmed.Answer (C) is incorrect because a sufficient degree of certainty that economic benefits will flow to the enterprise doesnot exist when the ticket is issued.Answer (D) is correct. Recognition of an element of financial statements (e.g., income, which includes revenue andgains) requires that two criteria be met. It must be probable that any future economic benefit associated with the itemwill flow to or from the enterprise, and the cost or value of the item must be measurable with reliability. The usualprocedures for income recognition, e.g., that income be earned, reflect these criteria. Thus, income is recognizedwhen an increase in future economic benefits is associated with an increase in an asset or a decrease in a liability. Therecognition criteria are not met with respect to the collections from sales of airline tickets until the receipt of futureeconomic benefits is sufficiently certain. Such certainty exists when the airline performs the contracted-for service,that is, when the related flights occur. The critical event in the earning process for the airline is the delivery of theservice to the customer, which occurs when the related flight takes place.

Gleim's CIA Test Prep: Part III: Business Analysis and Information Technology(1165 questions)

Copyright 2004 Gleim Publications, Inc. Page 113Printed for g j

Page 114: Part three cia_with_ answers

[308] Gleim #: 3.47 -- Source: CIA 1192 IV-27

A company provides fertilization, insect control, and disease control services for a variety of trees, plants, and shrubs on acontract basis. For 50 per month, the company will visit the subscriber’s premises and apply appropriate mixtures. If thesubscriber has any problems between the regularly scheduled application dates, the company’s personnel will promptlymake additional service calls to correct the situation. Some subscribers elect to pay for an entire year because the companyoffers an annual price of 540 if paid in advance. For a subscriber who pays the annual fee in advance, the company shouldrecognize the related revenue

When the cash is collected.A.Evenly over the year as the services are performed.B.At the end of the contract year after all of the services have been performed.C.At the end of the fiscal year.D.

Answer (A) is incorrect because a liability is recognized when the cash is collected prior to the rendition of theservice.Answer (B) is correct. Income, which includes revenue and gains, should not be recognized until an increase infuture economic benefits related to an increase in an asset or a decrease in a liability is sufficiently certain and can bemeasured reliably. The most common time at which these two conditions are met is when the product or merchandiseis delivered or services are rendered to customers. In the situation presented, the performance of the service (monthlyspraying) is so significant to creating a sufficient certainty of an increase in future economic benefits that revenueshould not be recognized until delivery occurs.Answer (C) is incorrect because revenue from services rendered is recognized when the services have beenperformed. A portion of the services is performed monthly. Thus, a portion of the related revenue should berecognized monthly rather than waiting until the entire contract year is complete.Answer (D) is incorrect because a portion of the related revenue should be recognized monthly rather than waitinguntil the entire fiscal year is complete.

[Fact Pattern #18]A company had the pre-closing trial balance at December 31 shown. Additional information:

The balance of opening inventory was 140,000.

The long-term debt pays interest at a rate of 10% per annum, payable every 12 months. The debt was issued on July 1 of thecurrent year and originally had 5 years to maturity.

The assets classified as property, plant, and equipment (exclusive of land) have a 10-year estimated useful life and were 1 year oldat the start of the current year. Straight-line depreciation is used by the company.

Cash 80,000Accounts receivable 100,000Inventory 230,000Gross property, plant, and equipment 600,000Accumulated depreciation 60,000Accounts payable 200,000Long-term debt 1,000,000Share capital 2,000,000Retained earnings (Jan. 1) 500,000Sales revenue 750,000Purchases 530,000Administrative expenses 200,000

[309] Gleim #: 3.48 -- Source: CIA 594 IV-2

(Refers to Fact Pattern #18)On the year-end financial statements, the company will report cost of goods sold of

440,000A.620,000B.670,000C.900,000D.

Gleim's CIA Test Prep: Part III: Business Analysis and Information Technology(1165 questions)

Copyright 2004 Gleim Publications, Inc. Page 114Printed for g j

Page 115: Part three cia_with_ answers

Answer (A) is correct. Cost of goods sold equals beginning inventory, plus purchases, minus ending inventory. Hence, costof goods sold is 440,000 (140,000 + 530,000 – 230,000).Answer (B) is incorrect because 620,000 is obtained by reversing the opening and closing inventory figures.Answer (C) is incorrect because 670,000 omits closing inventory from the calculation.Answer (D) is incorrect because 900,000 adds, rather than subtracting, closing inventory.

[310] Gleim #: 3.49 -- Source: CIA 594 IV-3

(Refers to Fact Pattern #18)Which is the correct order of the following four steps in the accounting cycle?

Step 1 prepare adjusting journal entriesStep 2 take a post-closing trial balanceStep 3 prepare an adjusted trial balanceStep 4 prepare reversing entries

1, 3, 2, 4A.4, 1, 3, 2B.3, 1, 2, 4C.1, 2, 3, 4D.

Answer (A) is correct. The accounting cycle may be summarized as consisting of nine steps: record the period’stransactions in the appropriate journals, post to the ledger(s) from the journals, prepare an unadjusted trial balance,prepare and post adjusting journal entries, prepare an adjusted trial balance, prepare the financial statements, prepareand post the closing entries, take a post-closing trial balance, and prepare reversing entries (optional).Answer (B) is incorrect because the preparation of reversing entries is the last step in the accounting cycle.Answer (C) is incorrect because the adjusted trial balance is prepared after adjusting entries are made.Answer (D) is incorrect because the post-closing trial balance is prepared after adjusting entries and the adjusted trialbalance are completed.

[311] Gleim #: 3.50 -- Source: CIA 591 IV-43

On 1/1/03, an enterprise acquired a plant asset for 300,000. Using the straight-line method of depreciation over a servicelife of 10 years with no residual value, the enterprise recorded depreciation of 30,000 for the year ending 12/31/03. The fairvalue of the asset at 12/31/03 is 310,000. If the enterprise uses the benchmark treatment for the measurement of property,plant, and equipment subsequent to initial recognition, the balance sheet prepared on 12/31/03 will

Report the excess of the fair value over the carrying amount as goodwill.A.Report the excess of the fair value over the carrying amount as part of property, plant, and equipment.B.Report the excess of the fair value over the carrying amount in equity as revaluation surplus.C.Not report the excess of the fair value over the carrying amount.D.

Answer (A) is incorrect because an excess of fair value over book value of a plant asset is not reflected inconventional financial statements under the benchmark treatment.Answer (B) is incorrect because an excess of fair value over book value of a plant asset is not reflected inconventional financial statements.Answer (C) is incorrect because the allowed alternative treatment described in IAS 16 is to carry the item of property,plant, and equipment at a revalued amount equal to fair value minus any subsequent accumulated depreciation andimpairment losses. An increase in the carrying amount after a revaluation is credited directly to equity as revaluationsurplus.Answer (D) is correct. According to the benchmark treatment for the measurement of property, plant, and equipmentsubsequent to initial recognition as an asset, the carrying amount should be cost minus any accumulated depreciationand impairment losses. Thus, the plant asset should be reported at its book value of 270,000 [300,000 – (300,000 ÷10 years)].

[312] Gleim #: 3.51 -- Source: CIA 1192 IV-30

The management of ABC Corporation is analyzing the financial statements of XYZ Corporation because ABC is stronglyconsidering purchasing a block of XYZ ordinary shares that would give ABC significant influence over XYZ. Whichfinancial statement should ABC primarily use to assess the amounts, timing, and certainty of future cash flows of XYZCompany?

Gleim's CIA Test Prep: Part III: Business Analysis and Information Technology(1165 questions)

Copyright 2004 Gleim Publications, Inc. Page 115Printed for g j

Page 116: Part three cia_with_ answers

Income statement.A.Statement of changes in equity.B.Statement of cash flows.C.Balance sheet.D.

Answer (A) is incorrect because the statement of income is prepared on an accrual basis and is not meant to report cashflows.Answer (B) is incorrect because the statement of changes in equity is prepared on the accrual basis.Answer (C) is correct. A statement of cash flows provides information about the cash receipts and cash payments of anenterprise during a period. This information helps investors, creditors, and other users to assess the enterprise’s ability togenerate cash and cash equivalents and the needs of the enterprise to use those cash flows. Historical cash flow data indicatethe amount, timing, and certainty of future cash flows. It is also a means of verifying past cash flow assessments and ofdetermining the relationship between profits and net cash flows and the effects of changing prices.Answer (D) is incorrect because the balance sheet reports on financial position at a moment in time.

[313] Gleim #: 3.52 -- Source: CIA 1191 IV-32

In a statement of cash flows (indirect method), depreciation expense should be presented as

An inflow of cash.A.An outflow of cash.B.An addition to net profit in converting net profit or loss to net cash flows from operating activities.C.A deduction from net profit in converting net profit or loss to net cash flows from operating activities.D.

Answer (A) is incorrect because depreciation does not involve an inflow or outflow of cash. The purchase and thesale of property, plant, and equipment constitute investing activities, but the process of depreciating such assets is notdefined as an investing activity. Depreciation is a noncash operating expense.Answer (B) is incorrect because depreciation is a noncash operating expense.Answer (C) is correct. Net cash flow from (used by) operating activities may be reported indirectly by removing fromnet profit or loss the effects of (1) deferrals of past operating cash flows, (2) accruals of expected future operatingcash flows, (3) income or expense items related to financing and investing cash flows, and (4) noncash transactions.These adjustments include such items as depreciation, amortization of patents, amortization of bond discount andbond premium, gains and losses, changes during the period in trade receivables, changes in inventory, and changes inaccounts payable and accrued liabilities. In the reconciliation of net profit or loss to net cash from (used by)operations, depreciation (a noncash expense) is added to net profit or loss.Answer (D) is incorrect because depreciation should be added.

[314] Gleim #: 3.53 -- Source: CIA 592 IV-35

A financial statement includes all of the following items: net profit, depreciation, operating activities, and financingactivities. What financial statement is this?

Balance sheet.A.Income statement.B.Statement of cash flows.C.Statement of changes in equity.D.

Answer (A) is incorrect because the balance sheet does not include periodic net profit or loss or depreciation expense.Answer (B) is incorrect because the income statement does not have captions for operating and financing activities.Answer (C) is correct. A statement of cash flows is a required financial statement. It provides information about cashreceipts and payments by reporting the cash effects of an enterprise’s operating, investing, and financing activities.Related disclosures report the effects of noncash investing and financing activities. If the statement is presented usingthe indirect method, it will reconcile net profit or loss to net cash from (used in) operating activities. Depreciation, anoncash expense, is included in this presentation.Answer (D) is incorrect because the statement of changes in equity does not include captions for operating andinvesting activities and depreciation.

[315] Gleim #: 3.54 -- Source: Publisher

Which of the following describes the proper treatment of a material loss resulting from a natural disaster that is notexpected to recur frequently or regularly?

Gleim's CIA Test Prep: Part III: Business Analysis and Information Technology(1165 questions)

Copyright 2004 Gleim Publications, Inc. Page 116Printed for g j

Page 117: Part three cia_with_ answers

Report as part of continuing operations.A.Report as part of discontinuing operations.B.Report as an extraordinary item.C.Report as a fundamental error.D.

Answer (A) is incorrect because the loss should be treated as extraordinary. It is from an event clearly different fromordinary activities.Answer (B) is incorrect because the facts do not state that operations will be discontinued.Answer (C) is correct. Net profit or loss includes profit or loss from ordinary activities and extraordinary items. Both aredisclosed on the face of the income statement. Extraordinary items result from events or transactions that are clearlydifferent from ordinary activities and are not expected to recur frequently and regularly. Examples are an expropriation ofassets or a natural disaster. Each extraordinary item is separately disclosed. However, a total may be disclosed on theincome statement if the nature and amount of each item are disclosed in the notes.Answer (D) is incorrect because a fundamental error relates to a prior period. The loss presumably is current.

[316] Gleim #: 3.55 -- Source: CIA 592 IV-36

An enterprise decided to sell a separate major line of its business. The assets were sold for 100,000 and had a net carryingamount of 70,000. The applicable tax rate was 20%. The result of this transaction may appear on the

Balance sheet as a fundamental error.A.Income statement as an extraordinary item.B.Income statement as discontinuing operations.C.Income statement as a change in accounting policy.D.

Answer (A) is incorrect because the gain or loss from disposing of assets or settling liabilities of a discontinuingoperation appears on the face of the income statement.Answer (B) is incorrect because IAS 35 states that a discontinuing operation is not presented as an extraordinaryitem.Answer (C) is correct. A discontinuing operation (DO) is part of an enterprise that is being disposed of in accordancewith a single plan. It constitutes a separate major line of business or geographical operating area that is distinct foroperational and reporting purposes. If an enterprise disposes of assets or settles liabilities of a DO or reaches abinding agreement to do so, it must disclose on the face of the financial statements the pre-tax gain or loss and relatedincome tax expense for any gain or loss recognized on the disposal or settlement. However, other required disclosuresmay be made in the notes or on the face of the financial statements. Consequently, the income statement may, but isnot required to, be presented by segregating continuing and discontinuing operations.Answer (D) is incorrect because accounting policies guide the preparation and presentation of financial statements.

[317] Gleim #: 3.56 -- Source: CIA 1191 IV-40

A company uses the percentage-of-completion method and reports the following:

  Year 1 Year 2 Construction costs 100 200Estimated cost to complete at year-end 300 0

The contract price is 1,000. What is the gross profit recognized in year 2?

150A.400B.550C.800D.

Answer (A) is incorrect because gross profit recognized in year 2 is 550.Answer (B) is incorrect because gross profit recognized in year 2 is 550.Answer (C) is correct. At the end of year 1, total cost was expected to be 400 (100 incurred + 300 estimated cost tocomplete), and estimated total gross profit was 600 (1,000 price – 400 estimated total cost). Hence, the amount ofgross profit recognized in year 1 was 150 [600 × (100 cost incurred ÷ 400 estimated total cost)]. The project wascompleted in year 2 at an additional cost of 200. Actual gross profit was therefore 700 (1,000 – 300 actual total cost).Gross profit recognized in year 2 is 550 (700 total – 150 recognized in year 1).Answer (D) is incorrect because gross profit recognized in year 2 is 550.

Gleim's CIA Test Prep: Part III: Business Analysis and Information Technology(1165 questions)

Copyright 2004 Gleim Publications, Inc. Page 117Printed for g j

Page 118: Part three cia_with_ answers

[318] Gleim #: 3.57 -- Source: CIA 594 IV-30

Which of the following is not an appropriate basis for measuring the historical cost of property, plant, and equipment?

Delivery and handling costs and installation costs of a productive asset should be included in the asset’s cost.A.The cost should include the purchase price without a deduction for trade discounts.B.The costs of improvements to equipment incurred after its acquisition should be added to the asset’s cost if theyprovide future economic benefits exceeding the originally assessed standard of performance.

C.

All costs incurred in the construction of a plant building, from excavation to completion, should be considered as partof the asset’s cost.

D.

Answer (A) is incorrect because directly attributable costs of bringing the asset to working condition for its intendeduse are included in its cost.Answer (B) is correct. An asset classified under property, plant, and equipment is measured initially at cost. Thisamount includes the purchase price and any directly attributable costs of bringing the asset to working condition forits intended use. Directly attributable costs include costs of, for example, site preparation, initial delivery andhandling, installation, professional fees (e.g., those of architects and engineers), and dismantling and removing theasset and restoring the site. The purchase price is determined by adding any import fees and nonrefundable purchasetaxes and subtracting any trade discounts and rebates.Answer (C) is incorrect because subsequent expenditures should be added to the carrying amount if it is probable thatthey will result in future economic benefits exceeding the originally assessed standard of performance.Answer (D) is incorrect because the costs of a self-constructed asset are determined in the same way as those of anacquired asset. Hence, all costs of construction of a building are included in its initially recorded cost because theyare directly attributable costs of bringing the asset to its working condition for its intended use.

[319] Gleim #: 3.58 -- Source: CIA 590 IV-31

DEF is the consignee for 1,000 units of product X for ABC Company. ABC should recognize the revenue from these 1,000units when

The agreement between DEF and ABC is signed.A.ABC ships the goods to DEF.B.DEF receives the goods from ABC.C.DEF sells the goods and informs ABC of the sale.D.

Answer (A) is incorrect because the revenue recognition criteria for sales of goods are not met in a consignment untilthe consignee sells the goods to a third party.Answer (B) is incorrect because the revenue recognition criteria for sales of goods are not met in a consignment untilthe consignee sells the goods to a third party.Answer (C) is incorrect because the revenue recognition criteria for sales of goods are not met in a consignment untilthe consignee sells the goods to a third party.Answer (D) is correct. Under a consignment arrangement, the consignor ships merchandise to the consignee who actsas agent for the consignor in selling the goods. The goods are in the physical possession of the consignee but remainthe property of the consignor and are included in the consignor’s inventory. Revenue and the related cost of goodssold from these consigned goods should only be recognized by the consignor when the merchandise is sold anddelivered to the final customer. Accordingly, recognition occurs when notification is received that the consignee hassold the goods. Only then are the recognition criteria for a sale of goods satisfied: the enterprise has transferred thesignificant risks and rewards of ownership, the enterprise has neither continuing managerial involvement to an extentassociated with ownership nor effective control over the goods, the amount can be reliably measured, it is probablethat the economic benefits will flow to the enterprise, and transactional costs can be reliably measured.

[320] Gleim #: 3.59 -- Source: CIA 1193 IV-37

ABC Manufacturing Company ships merchandise costing 40,000 on consignment to XYZ Stores. ABC pays 3,000 offreight costs to a transport company, and XYZ pays 2,000 for local advertising costs that are reimbursable from ABC. Bythe end of the period, three fourths of the consigned merchandise has been sold for 50,000 cash. XYZ notifies ABC of thesales, retains a 10% commission and the paid advertising costs, and remits the cash due ABC. Select the journal entry thatappropriately records the notification of sale and the receipt of cash by ABC.

Gleim's CIA Test Prep: Part III: Business Analysis and Information Technology(1165 questions)

Copyright 2004 Gleim Publications, Inc. Page 118Printed for g j

Page 119: Part three cia_with_ answers

A. Cash 40,000Advertising expense 2,000Commission expense 5,000Freight expense 3,000

Revenue from consignment sales 50,000B. Cash 43,000

Advertising expense 2,000Commission expense 5,000

Revenue from consignment sales 50,000C. Cash 50,000

Revenue from consignment sales 50,000D. Cash 45,000

Commission expense 5,000Revenue from consignment sales 50,000

Answer (A) is incorrect because the freight was paid earlier in the period and would have been recorded then by a credit tocash and a debit to inventory. Thus, the freight costs will be included in the determination of net profit when cost of goodssold is recognized.Answer (B) is correct. ABC debits the cash received 43,000 [50,000 sales – 2,000 advertising – (.10 × 50,000) salescommission]. The advertising and commission expenses are debited for 2,000 and 5,000, respectively. Finally, 50,000 ofgross revenue is credited.Answer (C) is incorrect because the 10% commission and the advertising costs are ignored in this answer.Answer (D) is incorrect because the reimbursable advertising costs are ignored in this answer.

[321] Gleim #: 3.60 -- Source: CIA 592 IV-34

On December 1, year 1, a company using the installment sales method sold goods that cost 1,000 for 1,500. The buyer paid100 down. Monthly payments start January 1, year 2. Interest accrues at 1% per month on the unpaid balance. To thenearest dollar, the effect on profit for year 1 is

33 increase.A.47 increase.B.67 increase.C.114 increase.D.

Answer (A) is incorrect because 33 is the net profit effect excluding interest revenue of 14 [.01 × (1,500 – 100)].Answer (B) is correct. The gross profit margin is 33 1/3% [(1,500 – 1,000) ÷ 1,500], so the amount of profit from the100 down payment recognizable in year 1 is 33 (rounded). Interest accrued on the 1,400 (1,500 – 100) balance for 1month is 14. Consequently, the effect on profit is 47 (33 + 14).Answer (C) is incorrect because 67 is the cost of goods sold [(1,000 ÷ 1,500) × 100].Answer (D) is incorrect because 114 {100 + [.01 × (1,500 – 100)]} is the total revenue, not the profit.

[322] Gleim #: 3.61 -- Source: CIA 1189 IV-9

On January 31, 2003, a company prepaid the 72,000 rental fee for a parking lot it leases. The rental fee covered a 3-yearperiod beginning February 1, 2003. What is the effect of this transaction on the December 31, 2001 financial statements foreach of the following?

  Current Prepaid  Expenses Expenses A. 0 72,000B. 22,000 50,000C. 24,000 48,000D. 72,000 0

Gleim's CIA Test Prep: Part III: Business Analysis and Information Technology(1165 questions)

Copyright 2004 Gleim Publications, Inc. Page 119Printed for g j

Page 120: Part three cia_with_ answers

Answer (A) is incorrect because the expenses relating to the current year need to be recognized.Answer (B) is correct. The 72,000 rental fee should be recognized as expense evenly over the 36-month duration of thelease. In 2003, 22,000 should be debited to current expenses (11 months × 72,000 ÷ 36), and 50,000 should be deferred asprepaid expense.Answer (C) is incorrect because 24,000 is the expense recognized for 12 months instead of the 11 months from January 31,2003 to December 31, 2003.Answer (D) is incorrect because only those expenditures related to the current year should be expensed.

[323] Gleim #: 3.62 -- Source: CIA 1189 IV-29

A department store sells gift certificates that may be redeemed for merchandise. Each certificate expires 3 years afterissuance. The revenue from the gift certificates should be recognized

Evenly over 3 years from the date of issuance.A.In the period the certificates are sold.B.In the period the certificates expire.C.In the period the certificates are redeemed or in the period they expire if they are allowed to lapse.D.

Answer (A) is incorrect because there is no basis for such an allocation.Answer (B) is incorrect because, when the certificates are sold, future economic benefits are neither sufficientlycertain nor reliably measurable. The store has not yet performed its obligations created by the sales of gift certificates.Answer (C) is incorrect because revenue is also recognized when certificates are redeemed.Answer (D) is correct. Income, which includes revenue and gains, should not be recognized until an increase infuture economic benefits related to an increase in an asset or a decrease in a liability is sufficiently certain and can bemeasured reliably. These criteria are met when the certificates are redeemed or expire because the liability forredemptions will then decrease and economic benefits will be reliably measurable.

[324] Gleim #: 3.63 -- Source: CIA 1185 IV-20

Which of the following costs is most likely to be expensed immediately rather than recognized as an asset?

Development.A.Equipment.B.Goodwill.C.Research.D.

Answer (A) is incorrect because an internally generated intangible asset may arise from the development (but not theresearch) phase of asset generation. An asset is recognized if the enterprise demonstrates the technical feasibility ofcompleting the asset, the intent to complete and use or sell the asset, the ability to use or sell it, how it will generateprobable future economic benefits, the availability of resources to complete development and to use or sell the asset,and the ability to make a reliable measurement of development expenditures for the asset.Answer (B) is incorrect because items of equipment that qualify for asset recognition are initially measured at cost.The depreciable amounts of these assets are then allocated as expense (unless included in the carrying amount ofother assets) over their useful lives.Answer (C) is incorrect because goodwill arising from a business combination that meets the definition of anacquisition is recognized as an asset and then subsequently amortized.Answer (D) is correct. Research expenditures should be expensed when incurred. No intangible asset resulting fromresearch should be recognized. The enterprise cannot demonstrate at this stage that future economic benefits areprobable.

[Fact Pattern #19]On January 1, 2003, a construction company signed a contract with a property management firm involving the construction of alarge urban office tower. The total price of constructing the tower was agreed to be 10 million with 2 million being paid on thedate of the agreement. Construction began immediately upon the signing of the contract and was expected to take 3 years tocomplete. The original estimate of total construction costs was 8 million.

During the year ended December 31, 2003, 4 million of construction costs were incurred, and engineering estimates indicated thatthe office tower was 30% complete at year-end. At year-end, the revised estimate of total construction costs was 8.5 million andthe property management firm had been billed a further 4 million, although only 1 million of that amount had been collected byyear-end.

Gleim's CIA Test Prep: Part III: Business Analysis and Information Technology(1165 questions)

Copyright 2004 Gleim Publications, Inc. Page 120Printed for g j

Page 121: Part three cia_with_ answers

[325] Gleim #: 3.64 -- Source: CIA 1194 IV-8

(Refers to Fact Pattern #19)If the construction company uses the percentage-of-completion (cost-to-cost basis) method of revenue recognition, in 2003the amount of revenue it will recognize on the long-term contract will be

3,000,000A.4,705,882B.5,000,000C.6,000,000D.

Answer (A) is incorrect because 3,000,000 is based on the engineering estimate of the stage of completion.Answer (B) is correct. Under the cost-to-cost approach to determining the stage of completion of the contract, thestage of completion equals contract costs incurred to date divided by the most recent estimate of total contract costs.Hence, the revenue to be recognized in the first year is 4,705,882 [(4,000,000 ÷ 8,500,000) × 10,000,000]. Thisamount equals costs incurred plus recognized profit.Answer (C) is incorrect because 5,000,000 is based on the original estimate of contract costs.Answer (D) is incorrect because 6,000,000 is based on the percentage of total revenue billed to date.

[326] Gleim #: 3.65 -- Source: CIA 1194 IV-9

(Refers to Fact Pattern #19)Assume that the construction company recognizes 2 million of revenue for the long-term contract in 2003. If the companyuses the percentage-of-completion method (cost-to-cost basis), the difference between revenue recognized to date andcontract billings at the end of 2003 will be shown on the December 31, 2003 balance sheet as a <List A> of <List B>.

  List A List B A. Current asset 1,000,000B. Current asset 4,000,000C. Current liability 1,000,000D. Current liability 4,000,000

Answer (A) is incorrect because an excess of billings over the sum of costs and recognized profits is a liability, and1,000,000 is the amount of cash collected.Answer (B) is incorrect because an excess of billings over the sum of costs and recognized profits is a liability.Answer (C) is incorrect because 1,000,000 equals cash collected.Answer (D) is correct. The gross amount due from (to) customers for contract work is an asset (liability). If theamount of costs incurred plus recognized profits minus recognized losses exceeds progress billings, the enterprisepresents an asset. If the amount of progress billings exceeds costs incurred plus recognized profits minus recognizedlosses, the enterprise presents a liability. At the end of 2003, the company had recognized 2,000,000 of revenue(costs to date + recognized profit) and had submitted billings of 6,000,000. Thus, the excess billings equal 4,000,000.Because the billings exceed revenue recognized, this amount is listed as a current liability. It represents unearnedrevenue.

NOTE: Given a 10 million fixed price and 8.5 million of total costs, the assumption that 2 million of revenue wasrecognized under the percentage-of-completion method (cost-to-cost basis) necessarily includes the assumption thatthe stage of completion was 20% (2 million ÷ 10 million), that recognized profit was 300,000 [20% × (10 million –8.5 million)], and that costs to date were 1.7 million (20% × 8.5 million).

Gleim's CIA Test Prep: Part III: Business Analysis and Information Technology(1165 questions)

Copyright 2004 Gleim Publications, Inc. Page 121Printed for g j

Page 122: Part three cia_with_ answers

[Fact Pattern #20]On January 1, a new landscaping firm acquired a fleet of vehicles, all the necessary tools and equipment, and a parking andstorage facility. It began operations immediately. It is now the end of the first year of operations, and the first set of year-endfinancial statements are being prepared. Several decisions have to be made regarding the appropriate accounting and reportingpractices for this company. Relevant information for several of these items is described in the following list of transactions andevents:

At year-end, the parking and storage facility that was purchased for 150,000 has a fair value of 250,000.

The physical flow of inventory is first in, first out, and the cost of materials has risen steadily over the year.

To promote sales for the coming year, maintenance contracts were sold in December at very reasonable prices, provided that thecustomers paid cash.

On April 1, the company arranged a 100,000, 10% bank loan. Interest payments of 5,000 are due on October 1 and April 1 ofeach year during the 5-year term of the loan.

During the first year of operations, the company experienced a 5% bad debt rate on credit sales. None of the bad debts areexpected to be recovered, given that 5% is the industry average level of bad debts. Total credit sales for the year were 400,000.The year-end balance of accounts receivable, which includes uncollected overdue accounts, is 100,000. Half of the uncollectedoverdue amounts are estimated to be uncollectible.

[327] Gleim #: 3.66 -- Source: CIA 1194 IV-25

(Refers to Fact Pattern #20)The company will recognize revenue from the December sale of the maintenance contracts in the <List A> year if it selects<List B> basis reporting.

  List A List B A. First CashB. First AccrualC. Second CashD. Third Accrual

Answer (A) is correct. Under the cash basis of accounting, revenue is recognized when the cash is received. Becausethe customers paid cash for the maintenance contracts in the first year, that is the year in which the revenue should berecognized.Answer (B) is incorrect because, if the accrual basis is used, revenue received for the maintenance contracts isrecognized after the first year. The services are provided and the revenue recognition criteria are met after year one.Answer (C) is incorrect because, under cash-basis accounting, the revenue received in the first year is recognized inthe first year.Answer (D) is incorrect because accrual-basis accounting recognizes the revenue in year two. Services will beprovided in year two.

[328] Gleim #: 3.67 -- Source: CIA 1194 IV-26

(Refers to Fact Pattern #20)The company will report a value of <List A> for the parking and storage facility if it prepares financial reports consistentwith the <List B> principle.

  List A List B A. 150,000 MatchingB. 150,000 Historical costC. 250,000 Going concernD. 250,000 Revenue recognition

Gleim's CIA Test Prep: Part III: Business Analysis and Information Technology(1165 questions)

Copyright 2004 Gleim Publications, Inc. Page 122Printed for g j

Page 123: Part three cia_with_ answers

Answer (A) is incorrect because the matching principle requires the recognition of related revenues and expenses in thesame accounting period.Answer (B) is correct. An asset classified under property, plant, and equipment is measured initially at cost. This amountincludes the purchase price and any directly attributable costs of bringing the asset to working condition for its intendeduse. Directly attributable costs include costs of, for example, site preparation, initial delivery and handling, installation,professional fees (e.g., those of architects and engineers), and dismantling and removing the asset and restoring the site. Thepurchase price is determined by adding any import fees and nonrefundable purchase taxes and subtracting any tradediscounts and rebates.Answer (C) is incorrect because the going-concern assumption is that the enterprise will continue in existence indefinitely.Answer (D) is incorrect because revenue recognition principles refer to the timing of revenue recognition, not to thevaluation basis for property, plant, and equipment.

[329] Gleim #: 3.68 -- Source: CIA 1194 IV-27

(Refers to Fact Pattern #20)If the company uses the <List A> approach to estimate bad debt expense, the estimated bad debt expense will be <List B>.

  List A List B A. Balance sheet 20,000B. Balance sheet 100,000C. Income statement 20,000D. Income statement 50,000

Answer (A) is incorrect because the balance sheet approach estimates bad debts as a portion of overdue accountsestimated to be uncollectible. Under this approach, the amount estimated to be uncollectible is 50,000 (100,000 ×50%).Answer (B) is incorrect because the balance sheet approach estimates bad debts as a portion of overdue accountsestimated to be uncollectible. Under this approach, the amount estimated to be uncollectible is 50,000 (100,000 ×50%).Answer (C) is correct. Using the income statement approach, the bad debt expense is determined using a percentageof total credit sales. Thus, bad debt expense is 20,000 (400,000 credit sales × 5% estimated bad debt rate).Answer (D) is incorrect because 50,000 is the estimated bad debt expense using the balance sheet approach.

[330] Gleim #: 3.69 -- Source: CIA 1194 IV-28

(Refers to Fact Pattern #20)If the company reports expenses on an accrual basis, interest expense for the first year of operations is

5,000A.7,500B.10,000C.12,500D.

Answer (A) is incorrect because 5,000 is the interest expense under the cash basis.Answer (B) is correct. The interest expense for the first year equals the 5,000 payment made on October 1 plus thethree months’ accrued interest at year-end, or 7,500 (5,000 + 2,500).Answer (C) is incorrect because 10,000 is the interest expense for a full year.Answer (D) is incorrect because 12,500 is equal to 15 months of interest.

[331] Gleim #: 3.70 -- Source: CIA 595 IV-1

To calculate net sales, <List A> must be <List B> cash receipts from customers.

  List A List B A. An increase in accounts receivable Added toB. An increase in accounts receivable Subtracted fromC. An increase in accounts payable Added toD. An increase in accounts payable Subtracted from

Gleim's CIA Test Prep: Part III: Business Analysis and Information Technology(1165 questions)

Copyright 2004 Gleim Publications, Inc. Page 123Printed for g j

Page 124: Part three cia_with_ answers

Answer (A) is correct. To convert from the cash basis (cash receipts) to the accrual basis (net sales), the increase in netaccounts receivable must be added to cash receipts from customers.Answer (B) is incorrect because a decrease in receivables would be subtracted from cash receipts.Answer (C) is incorrect because changes in accounts payable are not included in the calculation of net sales.Answer (D) is incorrect because changes in accounts payable are not included in the calculation of net sales.

[332] Gleim #: 3.71 -- Source: CIA 595 IV-3

Which of the following steps in the accounting cycle is completed later than the others?

Adjustments.A.Journalization.B.Posting.C.Identification and measurement of transactions.D.

Answer (A) is correct. The order of the steps in the accounting cycle is identification and measurement oftransactions and other events required to be recognized, journalization, posting from the journals to the ledgers, thedevelopment of a trial balance, adjustments to produce an adjusted trial balance, statement presentation, closing,taking a post-closing trial balance (optional), and making reversing entries (optional).Answer (B) is incorrect because, of the steps listed, journalization occurs second.Answer (C) is incorrect because, of the steps listed, posting occurs third.Answer (D) is incorrect because the identification and measurement of transactions is the first step in the accountingcycle.

[333] Gleim #: 3.72 -- Source: CIA 595 IV-4

A company purchased 50,000 worth of office supplies on January 1st and had 15,000 of office supplies still on hand atyear-end. If the initial purchase of supplies entry on January 1st was to debit office supplies expense and to credit cash for50,000, the adjusting entry on December 31 will be

A. Office supplies 15,000Office supplies expense 15,000

B. Office supplies expense 15,000Office supplies 15,000

C. Office supplies 35,000Office supplies expense 35,000

D. Office supplies expense 35,000Office supplies 35,000

Answer (A) is correct. Because the initial entry recorded the entire purchase to a nominal account, the year-endadjusting entry must debit a prepaid asset to reflect the remaining supplies on hand of 15,000. The adjusting entrymust also credit (reduce) expenses for the 15,000 of supplies that were not used during the period.Answer (B) is incorrect because the reversing entry is to debit expense and credit a prepaid asset.Answer (C) is incorrect because the appropriate amount is 15,000, not 35,000.Answer (D) is incorrect because the closing entry would have been to debit expense and credit a prepaid asset if theinitial entry had been to a real account.

[334] Gleim #: 3.73 -- Source: CIA 1196 IV-11

A company began work on a long-term construction contract in year 1. The contract price was 3,000,000. Year-endinformation related to the contract is as follows:

  Year 1 Year 2 Year 3 Estimated total cost 2,000,000 2,000,000 2,000,000Cost incurred 700,000 900,000 400,000Billings 800,000 1,200,000 1,000,000Collections 600,000 1,200,000 1,200,000

If the company uses the percentage-of-completion method of accounting for this contract, the gross profit to be recognizedin year 1 is

Gleim's CIA Test Prep: Part III: Business Analysis and Information Technology(1165 questions)

Copyright 2004 Gleim Publications, Inc. Page 124Printed for g j

Page 125: Part three cia_with_ answers

(100,000)A.100,000B.200,000C.350,000D.

Answer (A) is incorrect because (100,000) is the difference between costs incurred and collections.Answer (B) is incorrect because 100,000 is the difference between billings and costs incurred.Answer (C) is incorrect because 200,000 is the difference between billings and collections.Answer (D) is correct. When the outcome of a contract can be reasonably estimated, the percentage-of-completion methodrecognizes revenue based on the stage of completion of the contract. One typical method for estimating the stage ofcompletion is the calculation of ratio of the contract costs incurred to date to the estimated total costs. The percentage-of-completion at year-end on the cost-to-cost basis is 35% (700,000 ÷ 2,000,000). The gross profit for year 1 is the anticipatedgross profit on the contract times the completion percentage. Thus, gross profit for year 1 is 350,000 [(3,000,000 –2,000,000) × 35%].

[335] Gleim #: 3.74 -- Source: CIA 1196 IV-30

A vendor sells specialty inks on consignment to a manufacturer of colored paper at a price of 200 per barrel. Payment ismade to the vendor in the month the manufacturer uses the barrels in production. The vendor records revenues when thebarrels are shipped and makes no adjusting entries to record unearned revenues until the December 31st closing of thebooks. At the end of July, the manufacturer had 40 barrels of ink on consignment. During August, the vendor consigned 50barrels and received payment for 30 barrels. Another 5 barrels were returned to the vendor by the manufacturer for credit.At the end of August, what is the amount of unearned revenue contained in the vendor’s accounts receivable from themanufacturer?

3,000A.4,000B.11,000C.12,000D.

Answer (A) is incorrect because 3,000 does not include the 40 barrels consigned in July.Answer (B) is incorrect because 4,000 does not include the 40 barrels consigned in July or deduct the 5 barrelsreturned.Answer (C) is correct. Under a consignment arrangement, the consignor ships merchandise to the consignee who actsas agent for the consignor in selling the goods. The goods are in the physical possession of the consignee but remainthe property of the consignor and are included in the consignor’s inventory. Revenue and the related cost of goodssold from these consigned goods should only be recognized by the consignor when the merchandise is sold anddelivered to the final customer. Accordingly, recognition occurs when notification is received that the consignee hassold the goods. Only then are the recognition criteria for a sale of goods satisfied: the enterprise has transferred thesignificant risks and rewards of ownership, the enterprise has neither continuing managerial involvement to an extentassociated with ownership nor effective control over the goods, the amount can be reliably measured, it is probablethat the economic benefits will flow to the enterprise, and transactional costs can be reliably measured. In thisconsignment arrangement, 30 barrels have been paid for, so the revenue is recognized for these barrels. The 5 barrelsreturned are not included in unearned revenue because they constitute a return of consigned goods. Accordingly, theamount of inappropriately recognized revenue is 11,000 [(40 consigned + 50 consigned - 30 paid for - 5 returns) x200].Answer (D) is incorrect because 12,000 does not reflect the 5 barrels returned.

[336] Gleim #: 3.75 -- Source: CIA 591 IV-52

A project is expected to result in the following adjustments over the next year:

Cash sales increase by 400,000.Expenses (except depreciation) increase by 180,000.Depreciation increases by 80,000.

Assume the corporate tax rate is 34%. The total relevant net cash flows during that year are

92,400A.140,000B.172,400C.220,000D.

Gleim's CIA Test Prep: Part III: Business Analysis and Information Technology(1165 questions)

Copyright 2004 Gleim Publications, Inc. Page 125Printed for g j

Page 126: Part three cia_with_ answers

Answer (A) is incorrect because 92,400 equals the increase in after-tax net profit.Answer (B) is incorrect because 140,000 is the increase in pre-tax net profit.Answer (C) is correct. The increase in pre-tax net profit is 140,000 (400,000 cash sales increase – 180,000 nondepreciationexpenses increase – 80,000 depreciation). Thus, taxes will increase by 47,600 (34% × 140,000), and the increase in net cashinflows will be 172,400 (400,000 – 180,000 – 47,600).Answer (D) is incorrect because 220,000 equals cash sales minus expenses other than depreciation.

[337] Gleim #: 3.76 -- Source: CIA 590 IV-55

A company had 30 million in total sales last year and expects 40 million in total sales this year. Ten percent of each year’ssales are on credit that will be paid the following year. The company anticipates the following expenses for this year:

Depreciation of 5 million.Labor, materials, taxes, and other expenses of 51 million.

Assume the company begins this year with a zero cash balance. At the end of this year, the company will have a cash deficitof

8 million.A.12 million.B.15 million.C.17 million.D.

Answer (A) is incorrect because 8 million includes next year’s payments on this year’s sales in the estimation of thisyear’s cash inflows.Answer (B) is correct. The cash inflows from last year’s credit sales are estimated to be 3,000,000 (10% ×30,000,000). The cash inflows from this year’s sales are expected to be 36,000,000 (90% × 40,000,000), a total cashinflow of 39,000,000 for the current year. Ignoring depreciation, which is a noncash expense, cash outflows areestimated at 51,000,000. Hence, the net cash outflow is anticipated to be 12,000,000 (39,000,000 – 51,000,000).Answer (C) is incorrect because 15 million excludes the 3 million cash inflow that resulted from last year’s creditsales.Answer (D) is incorrect because 17 million includes depreciation when estimating cash outflows.

[338] Gleim #: 3.77 -- Source: CIA 597 IV-32

The management discussion and analysis section of a company’s annual report typically contains

A description of the company’s primary business and segments.A.A discussion of the company’s operating results.B.A discussion of the future prospects of the company.C.All of the answers are correct.D.

Answer (A) is incorrect because a description of the company’s primary business and segments, a discussion of thecompany’s operating results, and a discussion of the future prospects of the company are included.Answer (B) is incorrect because a description of the company’s primary business and segments, a discussion of thecompany’s operating results, and a discussion of the future prospects of the company are included.Answer (C) is incorrect because a description of the company’s primary business and segments, a discussion of thecompany’s operating results, and a discussion of the future prospects of the company are included.Answer (D) is correct. The MD&A section addresses capital resources, liquidity, and operating results. Managementmust also identify trends and discuss significant events and uncertainties. Thus, the MD&A section typically includesa description of the company’s primary business and segments. It reviews the operating results of the company,providing a breakdown of net sales and income by segment. It also contains prospective information on economictrends and market changes, and their potential effects on the company’s future performance.

[339] Gleim #: 3.78 -- Source: CIA R98 IV-36

The present value of 100,000 due at the end of eight years, at 10%, is 46,650. What is the approximate value of 100,000invested for the same length of time and at the same rate?

46,650A.100,000B.114,360C.214,360D.

Gleim's CIA Test Prep: Part III: Business Analysis and Information Technology(1165 questions)

Copyright 2004 Gleim Publications, Inc. Page 126Printed for g j

Page 127: Part three cia_with_ answers

Answer (A) is incorrect because 46,650 is the present value of 100,000 discounted at 10% for 8 years.Answer (B) is incorrect because 100,000 is the present value of 214,360 discounted at 10% for 8 years.Answer (C) is incorrect because 114,360 is the interest earned.Answer (D) is correct. The present value of a future amount equals that amount divided by the interest factor for the givendiscount period and discount rate. The interest factor for the present value of 1 due in 8 years and discounted at 10% is2.1436 (100,000 ÷ 46,650). Thus, the future value of 100,000 must be 214,360 (2.1436 × 100,000).

[340] Gleim #: 3.79 -- Source: CIA R98 IV-41

A company purchased a new machine on an installment payment plan and is to make equal annual payments beginning oneyear from the date of purchase. Using an interest rate of 10%, the cost of the machine can be determined by multiplying onepayment by the

Future value of one currency unit.A.Future value of an annuity due of one currency unit.B.Present value of one currency unit.C.Present value of an ordinary annuity of one currency unit.D.

Answer (A) is incorrect because the cost at the date of purchase is the present value of an ordinary annuity.Answer (B) is incorrect because the cost at the date of purchase is the present value of an ordinary annuity.Answer (C) is incorrect because the cost at the date of purchase is the present value of an ordinary annuity.Answer (D) is correct. The cost of the machine is the present value of the annuity (the series of equal periodicpayments). The annuity is an ordinary annuity because the payments begin one period after the date of purchase. Thecost therefore equals the periodic payment times the interest factor for an ordinary annuity of one currency unit for aspecified number of equal periodic payments discounted at a given interest rate.

[341] Gleim #: 3.80 -- Source: Publisher

IAS 34, Interim Financial Reporting, provides guidelines for interim reporting stating that enterprises

May use estimates based on sales margins for interim inventory valuation although a different method is used forannual reporting.

A.

Must determine income tax expense by applying progressive tax rates to income on a quarterly basis.B.May prorate extraordinary items over four quarters.C.Need not disclose basic and diluted earnings per share each quarter.D.

Answer (A) is correct. Essentially the same reporting methods should be used for interim and annual financialstatements. However, the preparation of interim financial reports ordinarily requires a greater use of estimates. Forexample, IAS 34 states that complete inventory-taking and valuation may not be required at interim dates. Estimatesbased on sales margins may suffice.Answer (B) is incorrect because income tax expense is based on the estimated annual effective income tax rate.Answer (C) is incorrect because the same accounting policies should be applied in the interim statements as in theannual statements. Accordingly, an extraordinary item is to be reported in the interim period in which the gain or lossoccurred.Answer (D) is incorrect because the same accounting policies should be applied in the interim statements as in theannual statements. Thus, BEPS and DEPS must be reported.

[342] Gleim #: 3.81 -- Source: CIA 1193 IV-30

An enterprise that sprays chemicals in residences to eliminate or prevent infestation of insects requires that customersprepay for 3 monthsrsquo; service at the beginning of each new quarter. Select the term that appropriately describes thissituation from the viewpoint of the enterprise.

Unearned income.A.Earned income.B.Accrued income.C.Prepaid expense.D.

Gleim's CIA Test Prep: Part III: Business Analysis and Information Technology(1165 questions)

Copyright 2004 Gleim Publications, Inc. Page 127Printed for g j

Page 128: Part three cia_with_ answers

Answer (A) is correct. The prepayment does not meet the income recognition criteria because the future inflow of economicbenefits is not sufficiently certain given that the enterprise has not done what is required to be entitled to those benefits.Thus, the amount received in advance is considered a liability (unearned income) because it represents an obligation toperform a service in the future arising from a past transaction.Answer (B) is incorrect because the income is not earned. The exterminator has not performed the related services for thecustomer.Answer (C) is incorrect because accrued income has met the recognition criteria but has not been received.Answer (D) is incorrect because the customer has a prepaid expense (expense paid but not incurred).

[343] Gleim #: 3.82 -- Source: CIA 1196 IV-4

The purpose of an entry that contains a debit to prepaid property taxes and a credit to property tax expense is to recognizea(n)

Prepaid expense.A.Accrued expense.B.Prepaid income.C.Expired cost.D.

Answer (A) is correct. An entry debiting prepaid property taxes and crediting property tax expense is needed at thebalance sheet date when prepayments are initially recorded in an expense account. This adjusting entry defersexpense recognition to the future period in which the property taxes apply.Answer (B) is incorrect because the entry records an asset.Answer (C) is incorrect because the entry records an asset.Answer (D) is incorrect because the entry records an unexpired cost (an asset).

[344] Gleim #: 3.83 -- Source: CIA 597 IV-3

What are the effects of an adjusting entry used to accrue revenue from credit sales?

  Assets Liabilities Equity A. Decrease No effect DecreaseB. Increase No effect IncreaseC. No effect Decrease IncreaseD. No effect Increase Decrease

Answer (A) is incorrect because the accrual of revenue increases assets, increases equity, and has no effect onliabilities.Answer (B) is correct. The journal entry to accrue revenue requires a debit to a receivable account and a credit to arevenue account. Thus, the accrual of revenue increases assets and equity.Answer (C) is incorrect because the accrual of revenue increases assets, increases equity, and has no effect onliabilities.Answer (D) is incorrect because the accrual of revenue increases assets, increases equity, and has no effect onliabilities.

[345] Gleim #: 3.84 -- Source: CIA 597 IV-14

A manufacturer receives an advance payment for special-order goods that are to be manufactured and delivered within thenext year. The advance payment should be reported in the manufacturer’s current-year balance sheet as a(n)

Current liability.A.Noncurrent liability.B.Contra asset amount.C.Accrued revenue.D.

Gleim's CIA Test Prep: Part III: Business Analysis and Information Technology(1165 questions)

Copyright 2004 Gleim Publications, Inc. Page 128Printed for g j

Page 129: Part three cia_with_ answers

Answer (A) is correct. Recognition of an element of financial statements (e.g., income, which includes revenue and gains)requires that two criteria be met. It must be probable that any future economic benefit associated with the item will flow toor from the enterprise, and the cost or value of the item must be measurable with reliability. The usual procedures forincome recognition, e.g., that income be earned, reflect these criteria. Thus, income is recognized when an increase in futureeconomic benefits is associated with an increase in an asset or a decrease in a liability. However, the enterprise has notsubstantially completed what it must do to be entitled to the benefits represented by the advance payment, and the receipt offuture economic benefits is not sufficiently certain to merit income recognition. Accordingly, a liability should berecognized because the entity has a current obligation arising from a past event that will require an outflow of economicbenefits, that is, to deliver goods or to refund the customerrsquo;s money. The delivery of goods is to take place within ayear of the balance-sheet date; therefore, the obligation is expected to be settled in the normal course of the operating cycleor is due to be settled within 12 months.Answer (B) is incorrect because the obligation is current.Answer (C) is incorrect because a contra account reduces the valuation of the related account.Answer (D) is incorrect because the payment has not met the income recognition criteria.

[346] Gleim #: 3.85 -- Source: CIA 597 IV-28

An enterprise that owns a new professional basketball team sells season tickets to its team’s games. The season lasts fromNovember through April, with 10 games played each month. In 2003, the enterprise collected 3 million from season-ticketsales for the 2003-2004 season. Its fiscal year-end is December 31. Based on this information, the enterprise should

Report a liability for unearned income of 2 million on its December 31, 2003 balance sheet.A.Report a liability for unearned income of 3 million on its December 31, 2003 balance sheet.B.Report income of 3 million on its 2003 income statement.C.Report income of 1 million on its 2004 income statement.D.

Answer (A) is correct. The collections represent a liability (unearned income). The income recognition criteria arenot met with respect to the season ticket collections until the receipt of future economic benefits is sufficientlycertain. Such certainty exists with respect to collections related to the games played as of the balance sheet date.Accordingly, the enterprise should recognize 1 million as income in 2003 [2 months × (3,000,000 ÷ 6 months)]. Theremaining 2 million should be reported as a liability.Answer (B) is incorrect because the liability at the end of 2003 is 2 million.Answer (C) is incorrect because the enterprise should report income of 1 million in 2003 and 2 million in 2004.Answer (D) is incorrect because the enterprise should report income of 1 million in 2003 and 2 million in 2004.

[Fact Pattern #21]An enterprise had the pre-closing trial balance at December 31 shown below. Additional information:

The balance of opening inventory was 140,000.

The long-term debt pays interest at a rate of 10% per annum, payable every 12 months. The debt was issued on July 1 of thecurrent year and originally had 5 years to maturity.

The assets classified as property, plant, and equipment have a 10-year estimated useful life and were 1 year old at the start of thecurrent year. Straight-line depreciation is used.

Cash 80,000Accounts receivable 100,000Inventory 230,000Gross property, plant, and equipment 600,000Accumulated depreciation 60,000Accounts payable 200,000Long-term debt 1,000,000Share capital 2,000,000Retained earnings (Jan. 1) 500,000Sales revenue 750,000Purchases 530,000Administrative expenses 200,000

Gleim's CIA Test Prep: Part III: Business Analysis and Information Technology(1165 questions)

Copyright 2004 Gleim Publications, Inc. Page 129Printed for g j

Page 130: Part three cia_with_ answers

[347] Gleim #: 3.86 -- Source: CIA 594 IV-1

(Refers to Fact Pattern #21)The enterprise uses straight-line depreciation for financial reporting purposes, but uses accelerated depreciation for taxpurposes. Which of the following account balances would be lower in the financial statements used for tax purposes than itwould be in the general purpose financial statements?

Accumulated depreciation.A.Cash.B.Retained earnings.C.Gross property, plant, and equipment.D.

Answer (A) is incorrect because the balance of accumulated depreciation is higher in the tax-basis financialstatements.Answer (B) is incorrect because depreciation expense is a noncash charge. The cash balance is unaffected by thedepreciation method used.Answer (C) is correct. Because the tax basis uses an accelerated method, depreciation expense and accumulateddepreciation will be greater. Moreover, taxable income will be lower than financial net income. Consequently, tax-basis retained earnings will be less than that in the general purpose financial statements.Answer (D) is incorrect because the historical cost of property, plant, and equipment is recorded in the grossproperty, plant, and equipment account. This amount is unaffected by depreciation.

[348] Gleim #: 3.87 -- Source: CIA 594 IV-5

(Refers to Fact Pattern #21)The enterprise will report year-end total assets of

800,000A.890,000B.950,000C.1,010,000D.

Answer (A) is incorrect because 800,000 uses the beginning balance of inventory.Answer (B) is correct. The year-end total assets can be determined by summing all of the assets and deductingaccumulated depreciation (including the current yearrsquo;s depreciation). Total accumulated depreciation at the endof the second year is 120,000 [(600,000 ÷ 10 years) × 2 years]. Total assets equal 890,000 (80,000 cash + 100,000A/R + 230,000 EI + 600,000 gross property, plant, and equipment – 120,000 accumulated depreciation).Answer (C) is incorrect because 950,000 omits second-year depreciation from the calculation.Answer (D) is incorrect because 1,010,000 omits total accumulated depreciation from the calculation.

[349] Gleim #: 3.88 -- Source: CIA 594 IV-4

(Refers to Fact Pattern #21)Which adjusting entry should be used at year-end to account for interest expense on the long-term debt?

A. Interest expense 100,000Interest payable 100,000

B. Interest expense 50,000Cash 50,000

C. Interest payable 100,000Interest expense 100,000

D. Interest expense 50,000Interest payable 50,000

Gleim's CIA Test Prep: Part III: Business Analysis and Information Technology(1165 questions)

Copyright 2004 Gleim Publications, Inc. Page 130Printed for g j

Page 131: Part three cia_with_ answers

Answer (A) is incorrect because the debt has been outstanding for only 6 months, so accrued interest is only 50,000.Answer (B) is incorrect because the debt pays annual interest on July 1, and no cash outlay is required at year-end.Answer (C) is incorrect because accrued interest is 50,000. Also, interest expense is debited and interest payable is credited.Answer (D) is correct. The debt was issued on July 1 and has been outstanding for only 6 months. Interest expense equalsthe face amount of the debt multiplied by the interest rate and the fraction of the year the debt was outstanding [1,000,000 ×10% × (6 ÷ 12) = 50,000]. Because interest is payable on July 1, 6 monthsrsquo; interest is accrued and expensed in thecurrent period. The payable is also recognized in the current period. Thus, the adjusting entry should be

Interest expense 50,000Interest payable 50,000

[350] Gleim #: 3.89 -- Source: CIA 594 IV-6

(Refers to Fact Pattern #21)Assume that the enterprise reports cost of goods sold of 200,000 and interest expense of 10,000 for the current period. Alsoassume a 50% tax rate on corporate earnings. The final closing entry required to ensure that current earnings areincorporated into year-end retained earnings is

A. Income summary 140,000Retained earnings 140,000

B. Retained earnings 280,000Income summary 280,000

C. Income summary 240,000Retained earnings 240,000

D. Retained earnings 240,000Income summary 240,000

Answer (A) is correct. Current period pretax net income equals 280,000 (750,000 sales – 200,000 CGS – 60,000depreciation – 10,000 interest – 200,000 administrative expenses). Thus, after-tax net income credited to retainedearnings equals 140,000 [(1.0 – .5) × 280,000].Answer (B) is incorrect because income taxes are omitted and the journal entry is reversed.Answer (C) is incorrect because administrative expenses were omitted.Answer (D) is incorrect because administrative expenses were omitted and the journal entry is reversed.

[351] Gleim #: 4.1 -- Source: CIA 1191 IV-34

An enterprise offers its customers credit terms of a 2% discount if paid within 10 days, or the full balance is due within 30days (2/10, n/30). If some customers take advantage of the cash discount and others do not, which of the followingaccounts will appear on the income statement if the net method of recording receivables is employed?

  Sales Discounts  Sales Discounts Forfeited A. Yes YesB. Yes NoC. No NoD. No Yes

Answer (A) is incorrect because the net method requires a sales discount forfeited but not a sales discount account.Answer (B) is incorrect because the net method requires a sales discount forfeited but not a sales discount account.Answer (C) is incorrect because the net method requires a sales discount forfeited.Answer (D) is correct. The gross method accounts for receivables at their face value. If a discount is taken, a salesdiscount is recorded and classified as an offset to sales in the income statement to yield net sales. The net methodrecords receivables net of the applicable discount. If the payment is not received during the discount period, aninterest revenue account, such as sales discounts forfeited, is credited at the end of the discount period or when thepayment is received. Accordingly, the application of the net method requires a sales discount forfeited but not a salesdiscount account.

Gleim's CIA Test Prep: Part III: Business Analysis and Information Technology(1165 questions)

Copyright 2004 Gleim Publications, Inc. Page 131Printed for g j

Page 132: Part three cia_with_ answers

[352] Gleim #: 4.2 -- Source: CIA 1193 IV-41An internal auditor is deriving cash flow data based on an incomplete set of facts. Bad debt expense was 2,000. Additionaldata for this period follows:

Net profit 100,000Accounts receivable beginning balance 5,000Allowance for bad debts beginning

balance (500)Accounts receivable written off 1,000Increase in net accounts receivable (after

subtraction of allowance for bad debts) 30,000

How much cash was collected this period?

67,000A.68,500B.68,000C.70,000D.

Answer (A) is incorrect because 67,000 results from subtracting the write-offs and the bad debt expense from the sumof net income and beginning net accounts receivable.Answer (B) is incorrect because 68,500 assumes a zero balance in the beginning allowance account and deducts baddebt expense from the sum of net profit and beginning net accounts receivable.Answer (C) is incorrect because 68,000 deducts bad debt expense from the sum of net profit and beginning netaccounts receivable.Answer (D) is correct. The cash collected equals net profit adjusted for the change in net accounts receivable (grossA/R – allowance for bad debts). An increase in net accounts receivable implies that cash collected was less than netprofit. Hence, cash collected was 70,000 (100,000 – 30,000 increase in net A/R). Write-offs (debit the allowance,credit A/R) do not affect the computation of cash collected because the allowance and gross accounts receivable arereduced by the same amount. Moreover, recognition of bad debt expense (debit bad debt expense, credit theallowance) is not included in this calculation because it is already reflected in the net accounts receivable balance.

[353] Gleim #: 4.3 -- Source: CIA 1196 IV-33

An analysis of an enterprise’s 150,000 accounts receivable at year-end resulted in a 5,000 ending balance for its allowancefor uncollectible accounts and a bad debt expense of 2,000. During the past year, recoveries on bad debts previouslywritten off were correctly recorded at 500. If the beginning balance in the allowance for uncollectible accounts was 4,700,what was the amount of accounts receivable written off as uncollectible during the year?

1,200A.1,800B.2,200C.2,800D.

Answer (A) is incorrect because 1,200 results from subtracting the recoveries instead of adding them.Answer (B) is incorrect because 1,800 results from subtracting bad debt expense from the allowance account.Answer (C) is correct. Under the allowance method, uncollectible accounts are written off by a debit to theallowance account and a credit to accounts receivable. The 500 of recovered bad debts is accounted for by a debit toaccounts receivable and a credit to the allowance account. The 2,000 bad debt expense is also credited to theallowance account. The amount of accounts receivable written off as uncollectible is 2,200 [5,000 ending allowance –(4,700 beginning allowance + 500 recoveries + 2,000 bad debt expense)].Answer (D) is incorrect because 2,800 results from subtracting the recoveries and bad debt expense from theallowance account.

Gleim's CIA Test Prep: Part III: Business Analysis and Information Technology(1165 questions)

Copyright 2004 Gleim Publications, Inc. Page 132Printed for g j

Page 133: Part three cia_with_ answers

[Fact Pattern #22]An enterprise sells goods on an installment basis. The table below includes information about the level of installment sales, thecost of the goods sold on installment, and the cash receipts on installment sales for 2003 through 2005. All cash receipt amountsshown are net of any interest charges.

  2003 2004 2005 Installment sales 10,000 5,000 20,000Cost of installment sales 6,000 4,000 10,000       Cash receipts on 2003 sales 2,000 4,000 4,000Cash receipts on 2004 sales 1,000 2,000Cash receipts on 2005 sales 4,000

[354] Gleim #: 4.4 -- Source: CIA 596 IV-1

(Refers to Fact Pattern #22)The rate of gross profit on 2004 installment sales is

20%A.40%B.50%C.80%D.

Answer (A) is correct. The rate of gross profit on 2004 installment sales is 20% [(5,000 of 2004 installment sales –4,000 cost of 2004 installment sales) ÷ 5,000 of 2004 installment sales].Answer (B) is incorrect because 40% is the gross profit on 2003 installment sales.Answer (C) is incorrect because 50% is the gross profit on 2005 installment sales.Answer (D) is incorrect because 80% is the ratio of the cost of 2004 installment sales to 2004 installment sales.

[355] Gleim #: 4.5 -- Source: CIA 596 IV-2

(Refers to Fact Pattern #22)The amount of gross profit to be recognized in 2003 on 2003 installment sales is

800A.2,000B.3,200C.4,000D.

Answer (A) is correct. In 2003, cash receipts were 2,000 from 2003 installment sales. The gross profit realized is thegross profit on the portion of sales for which payment has been received. This amount equals the 2003 gross profitpercentage multiplied by the cash receipts, or 800 {[(10,000 – 6,000) ÷ 10,000] × 2,000}.Answer (B) is incorrect because 2,000 is the amount of cash receipts during 2003 on 2003 installment sales.Answer (C) is incorrect because 3,200 is the amount of the total gross profit on 2003 installment sales that is deferredto future periods.Answer (D) is incorrect because 4,000 is the total gross profit on 2003 installment sales.

[356] Gleim #: 4.6 -- Source: CIA 596 IV-3

(Refers to Fact Pattern #22)The gross profit amount from 2005 sales to be deferred to future years would be

2,000A.3,000B.8,000C.10,000D.

Gleim's CIA Test Prep: Part III: Business Analysis and Information Technology(1165 questions)

Copyright 2004 Gleim Publications, Inc. Page 133Printed for g j

Page 134: Part three cia_with_ answers

Answer (A) is incorrect because 2,000 is the realized gross profit on 2005 sales.Answer (B) is incorrect because 3,000 equals total receipts for 2004 and 2005 on 2004 sales.Answer (C) is correct. The total gross profit on 2005 sales is 10,000 (20,000 sales – 10,000 cost), and the amount realizedis 2,000 {[(20,000 – 10,000) ÷ 20,000] × 4,000 of 2005 cash receipts}. Accordingly, the amount deferred is 8,000 (10,000– 2,000).Answer (D) is incorrect because 10,000 is the total gross profit on 2005 sales.

[357] Gleim #: 4.7 -- Source: CIA 1196 IV-42

At the end of September, an enterprise has outstanding accounts receivable of 350 on third-quarter credit sales, composedas follows:

    Still Outstanding    at the End of

Month Credit Sales SeptemberJuly 600 100August 900 170September 500 80

The percentage of receivables in the 31-to-60-day age group at the end of September is

22.86%A.28.57%B.48.57%C.71.43%D.

Answer (A) is incorrect because 22.86% is the proportion of receivables in the 0-to-30-day age group at the end ofSeptember.Answer (B) is incorrect because 28.57% is the proportion of receivables in the 61-to-90-day age group at the end ofSeptember.Answer (C) is correct. Receivables from August sales still outstanding at the end of September are in the 31-to-60-day age group. This group represents 48.57% of total receivables [170 ÷ (100 + 170 + 80)].Answer (D) is incorrect because 71.43% is the proportion of outstanding receivables that are from 0 to 60 days old atthe end of September.

[358] Gleim #: 4.8 -- Source: CIA 595 IV-12

An enterprise sells inventory for 80,000 that had an inventory cost of 40,000. The terms of the sale involve paymentsreceivable of 10,000 in the first year, 45,000 in the second year, and 25,000 in the third year. The buyer of the inventory isa new firm with no credit history. If the cost-recovery method of revenue recognition is used, the amount of gross profit tobe recognized in the second year is

0A.5,000B.15,000C.45,000D.

Answer (A) is incorrect because under the cost-recovery method, profit is recognized in the second year when cashpayments by the buyer exceed the seller’s cost of merchandise.Answer (B) is incorrect because 5,000 is the profit to be recognized without consideration of the payment received inthe first year.Answer (C) is correct. The profit recognized in the second year equals the cumulative payments received minus theseller’s cost, or 15,000 [(10,000 + 45,000) – 40,000].Answer (D) is incorrect because 45,000 is the payment received in the second year.

[359] Gleim #: 4.9 -- Source: CIA 1195 IV-15

When a right of return exists, an enterprise may recognize revenue from a sale of goods at the time of sale only if

Gleim's CIA Test Prep: Part III: Business Analysis and Information Technology(1165 questions)

Copyright 2004 Gleim Publications, Inc. Page 134Printed for g j

Page 135: Part three cia_with_ answers

The amount of future returns can be reliably estimated.A.The seller retains the risks and rewards of ownership.B.The buyer resells the goods.C.The seller believes returns will not be material.D.

Answer (A) is correct. One condition for recognition of revenue from the sale of goods is the transfer of the significant risksand rewards of ownership. Retention of significant risk may occur when, for example, the buyer may rescind the purchasefor a reason stipulated in the contract, and the buyer is uncertain about the probability of return. However, if the enterprisecan reliably estimate future returns and recognizes a liability for returns based on experience and other pertinentinformation, revenue may be recognized at the time of sale if the other conditions for revenue recognition are also met.Answer (B) is incorrect because the risks and rewards of ownership must be transferred.Answer (C) is incorrect because this contingency is an example of retention of significant risk.Answer (D) is incorrect because returns may be material if they can be reliably estimated.

[360] Gleim #: 4.10 -- Source: CMA 1287 3-26

If a transfer of receivables with recourse qualifies to be recognized as a sale, the proceeds from the sale are

Accounted for as a collateralized borrowing.A.Recorded at fair value for the assets obtained and liabilities incurred.B.Recorded at the historical cost of the assets obtained.C.Reduced by the fair value of the recourse obligation.D.

Answer (A) is incorrect because the proceeds of the sale are reduced by the fair value of the recourse obligation.Answer (B) is incorrect because the proceeds of the sale are reduced by the fair value of the recourse obligation.Answer (C) is incorrect because the proceeds of the sale are reduced by the fair value of the recourse obligation.Answer (D) is correct. When a transfer of receivables with recourse meets the criteria to be accounted for as a sale,the enterprise derecognizes the financial assets it no longer controls. After derecognition, periodic net profit or losswill include the difference between the carrying amount transferred and the proceeds, plus or minus any prioradjustment reflecting the fair value of the asset that had been reported in equity. If a new financial asset is created or anew financial liability is assumed, the calculation is adjusted for the fair value of the asset or liability. Thus, theproceeds of the sale are reduced by the fair value of the recourse obligation (a new financial liability). When thetransfer does not meet these criteria, the transfer is accounted for as a collateralized borrowing.

[361] Gleim #: 4.11 -- Source: Publisher

A transferor enterprise most likely should continue to recognize a transferred financial asset if

The transferor may reacquire the asset, and the asset is readily obtainable in the market.A.The transferee may sell or pledge the full fair value of the asset.B.The transferor may reacquire the asset, and the reacquisition price is fair value.C.The transferor is entitled and obligated to repurchase the asset, and the transferee receives a lender’s return.D.

Answer (A) is incorrect because a financial asset should be derecognized despite the transferor’s right to reacquire itif the asset is readily obtainable in the market.Answer (B) is incorrect because control is lost when the transferee has the ability to obtain the benefits of the asset,for example, when the transferee may sell or pledge the full fair value of the asset.Answer (C) is incorrect because a financial asset should be derecognized despite the transferor’s right to reacquire itif the price is fair value at the date of reacquisition.Answer (D) is correct. An enterprise derecognizes a financial asset if it loses control of the contractual rights theasset represents. Control is lost when the enterprise realizes the rights to the specified contractual benefits, the rightsexpire, or it surrenders the rights. A transferor has not lost control and does not derecognize the financial asset if it isentitled and obligated to repurchase or redeem the asset, and the terms of this transaction in effect allow the transfereeto obtain a lender’s return on the assets it receives in exchange for the transferred financial asset.

Gleim's CIA Test Prep: Part III: Business Analysis and Information Technology(1165 questions)

Copyright 2004 Gleim Publications, Inc. Page 135Printed for g j

Page 136: Part three cia_with_ answers

[362] Gleim #: 4.12 -- Source: CIA 597 IV-15A retail enterprise maintains a markup of 25% based on cost. The enterprise has the following information for the currentyear:

Purchases of merchandise 690,000Freight-in on purchases 25,000Sales 900,000Ending inventory 80,000

Beginning inventory was

40,000A.85,000B.110,000C.265,000D.

Answer (A) is incorrect because 40,000 is based on a 25% markup on sales.Answer (B) is correct. Cost of goods sold equals beginning inventory, plus purchases (including freight-in), minusending inventory. Given that sales reflect 125% of cost, cost of goods sold must equal 720,000 (900,000 sales ÷1.25). Consequently, the beginning inventory must have been 85,000 (720,000 CGS + 80,000 EI – 690,000purchases – 25,000 freight-in).Answer (C) is incorrect because 110,000 omits the freight-in from the computation of cost of goods available for sale.Answer (D) is incorrect because 265,000 uses the sales figure for cost of goods sold.

[363] Gleim #: 4.13 -- Source: CIA 597 IV-16

Which inventory pricing method generally approximates current cost for each of the following?

  Ending Cost of  Inventory Goods Sold A. FIFO FIFOB. LIFO FIFOC. FIFO LIFOD. LIFO LIFO

Answer (A) is incorrect because LIFO approximates current cost of goods sold.Answer (B) is incorrect because FIFO approximates current cost for ending inventory, and LIFO approximatescurrent cost of goods sold.Answer (C) is correct. FIFO assigns the most recent purchase prices to ending inventory and the earliest purchaseprices to cost of goods sold. LIFO uses the earliest acquisition costs to price the ending inventory. Thus, FIFOapproximates current cost for ending inventory, and LIFO approximates current cost of goods sold.Answer (D) is incorrect because FIFO approximates current cost for ending inventory.

[364] Gleim #: 4.14 -- Source: CIA 596 IV-11

The cost of materials has risen steadily over the year. Which of the following methods of estimating the ending balance ofthe materials inventory account will result in the highest net profit, assuming all other variables remain constant?

Last-in, first-out (LIFO).A.First-in, first-out (FIFO).B.Weighted average.C.Specific identification.D.

Gleim's CIA Test Prep: Part III: Business Analysis and Information Technology(1165 questions)

Copyright 2004 Gleim Publications, Inc. Page 136Printed for g j

Page 137: Part three cia_with_ answers

Answer (A) is incorrect because LIFO yields the lowest net profit.Answer (B) is correct. Net profit will be higher when cost of goods sold is lower, other factors held constant. Cost of goodssold equals beginning inventory, plus purchases, minus ending inventory. Accordingly, cost of goods sold will be lowestwhen the ending inventory is highest. Ending inventory is highest under FIFO because the older, less expensive items aredeemed to have been sold, leaving the more expensive items in the ending inventory.Answer (C) is incorrect because weighted average averages inventory, so it results in a lower net profit than FIFO.Answer (D) is incorrect because, under specific identification, the newest (most expensive) items are not necessarily in theending inventory. The result is a higher cost of goods sold and lower net profit than under FIFO.

[365] Gleim #: 4.15 -- Source: CIA 1193 IV-31

A physical inventory count showed an enterprise had inventory costing 1,000,000 on hand at December 31, 2003.Excluded from this amount were the following:

Goods costing 82,000, shipped to a customer free on board (FOB) shipping point on December 28, 2003. They wereexpected to be received by the customer on January 4, 2004.Goods costing 122,000, shipped to a customer free on board (FOB) destination December 30, 2003. They wereexpected to be received by the customer on January 5, 2004.

Compute the correct ending inventory to be reported on the shipper’s balance sheet at December 31, 2003.

1,000,000A.1,082,000B.1,122,000C.1,204,000D.

Answer (A) is incorrect because 1,000,000 excludes the goods shipped FOB destination.Answer (B) is incorrect because 1,082,000 excludes the goods shipped FOB destination and fails to exclude thegoods shipped FOB shipping point.Answer (C) is correct. The goods shipped FOB shipping point should be counted in the buyer’s, not the seller’s,inventory because title and risk of loss pass at the time and place of shipment. These goods were properly excludedfrom ending inventory. The goods shipped FOB destination were improperly excluded from the seller’s endinginventory. The title and risk of loss did not pass until the time and place where the goods reached their destinationand were duly tendered. Thus, the correct ending inventory is 1,122,000 (1,000,000 beginning balance + 122,000goods shipped FOB destination).Answer (D) is incorrect because 1,204,000 includes the goods shipped FOB shipping point.

[366] Gleim #: 4.16 -- Source: CIA 1191 IV-31

When a perpetual inventory system is used and a difference exists between the perpetual inventory amount balance and thephysical inventory count, a separate entry is needed to adjust the perpetual inventory amount. Which of the followingdemonstrates that adjusting entry?

A. Inventory over and shortInventory

B. Extraordinary loss due to write-down ofinventory

InventoryC. Extraordinary loss due to write-down of

inventoryAllowance for inventory shortages

D. Cost of goods soldRetained earnings appropriated for

shortages

Gleim's CIA Test Prep: Part III: Business Analysis and Information Technology(1165 questions)

Copyright 2004 Gleim Publications, Inc. Page 137Printed for g j

Page 138: Part three cia_with_ answers

Answer (A) is correct. The entry to record a write-down is a debit to inventory over and short and a credit to inventory.This amount is reported as an adjustment of cost of goods sold or as an other expense on the income statement.Answer (B) is incorrect because a difference between a physical count and a perpetual inventory balance is common.Reasons include normal and expected shrinkage, breakage, shoplifting, and faulty record keeping. Thus, it is not anextraordinary item.Answer (C) is incorrect because a difference between a physical count and a perpetual inventory balance is common.Reasons include normal and expected shrinkage, breakage, shoplifting, and faulty record keeping. Thus, it is not anextraordinary item.Answer (D) is incorrect because, although the debit to cost of goods sold is acceptable, the credit should be to inventory.Also, any appropriation of retained earnings would also have to involve the unappropriated retained earnings account.

[367] Gleim #: 4.17 -- Source: CIA 1192 IV-43

An enterprise had the following selected per-unit data relating to work-in-process:

Selling price 100Completion costs 10Historical cost 91Replacement cost 108Normal gross profit 20Selling cost 5

In comparison with historical cost, what will be the per-unit impact on gross profit of measuring ending inventory?

No effect.A.Reduction of 6.B.Reduction of 26.C.Increase of 5.D.

Answer (A) is incorrect because no effect implies that NRV is at least as high as cost.Answer (B) is correct. Inventories are measured at the lower of cost or net realizable value (NRV). NRV equalsselling price minus completion and selling costs. Given that historical cost is 91 and NRV is 85 (price of 100 – 10completion cost – 5 selling cost), the effect on per-unit gross profit is a reduction of 6. This amount is the writedownexpensed.Answer (C) is incorrect because a reduction of 26 results from treating normal gross profit as if it were a completionor selling cost.Answer (D) is incorrect because an increase is permissible only to the extent of a prior writedown.

[Fact Pattern #23]Illustrated below is a perpetual inventory card for the current year.

 Date Units Purchased UnitsSold

UnitsBalance

January 1 - 0 -January 12 1,000 @ 2.00 1,000March 15 300 700May 5 500 @ 2.20 1,200July 8 500 700November 24 1,000 @ 1.65 1,700

Additional Information:

The enterprise had no opening inventory.1.The items sold on March 15 were purchased on January 12.2.The items sold on July 8 were purchased on May 5.3.

[368] Gleim #: 4.18 -- Source: CIA 596 IV-17

(Refers to Fact Pattern #23)The ending inventory balance under the first-in, first-out (FIFO) method of inventory valuation is

Gleim's CIA Test Prep: Part III: Business Analysis and Information Technology(1165 questions)

Copyright 2004 Gleim Publications, Inc. Page 138Printed for g j

Page 139: Part three cia_with_ answers

3,050A.3,150B.3,230C.3,430D.

Answer (A) is incorrect because 3,050 is the ending inventory under the specific identification method.Answer (B) is correct. Under the FIFO method, the 1,700 units of ending inventory are valued at the most recent prices.Ending inventory is assumed to include 1,000 units purchased November 24, 500 units purchased May 5, and 200 unitspurchased January 12. Hence, the ending inventory is 3,150 [(1,000 × 1.65) + (500 × 2.20) + (200 × 2.00)].Answer (C) is incorrect because 3,230 is the ending inventory under the weighted-average method.Answer (D) is incorrect because 3,430 is the ending inventory under the LIFO method.

[369] Gleim #: 4.19 -- Source: CIA 596 IV-18

(Refers to Fact Pattern #23)The cost of goods sold under the specific identification method of inventory valuation is

1,320A.1,520B.1,600C.1,700D.

Answer (A) is incorrect because 1,320 is the cost of goods sold under the LIFO method.Answer (B) is incorrect because 1,520 is the cost of goods sold under the weighted-average method.Answer (C) is incorrect because 1,600 is the cost of goods sold under the FIFO method.Answer (D) is correct. Of the 800 units sold during the period, the 300 units sold on March 15 were purchased onJanuary 12 at a cost of 2.00 per unit. The remaining 500 units were purchased on May 5 at a cost of 2.20 per unit.The cost of goods sold under the specific identification method is therefore 1,700 [(300 units × 2.00) + (500 units ×2.20)].

[Fact Pattern #24]An enterprise started in 2004 with 200 scented candles on hand at a cost of 3.50 each. These candles sell for 7.00 each. Thefollowing schedule represents the purchases and sales of candles during 2004:

Transaction Quantity Unit QuantityNumber Purchased Cost Sold

1 --- --- 1502 250 3.30 ---3 --- --- 1004 200 3.10 ---5 --- --- 2006 350 3.00 ---7 --- --- 300

[370] Gleim #: 4.20 -- Source: CIA 1196 IV-21

(Refers to Fact Pattern #24)If the enterprise uses perpetual LIFO inventory pricing, the cost of goods sold for 2004 is

2,330A.2,805B.2,375C.2,445D.

Gleim's CIA Test Prep: Part III: Business Analysis and Information Technology(1165 questions)

Copyright 2004 Gleim Publications, Inc. Page 139Printed for g j

Page 140: Part three cia_with_ answers

Answer (A) is incorrect because 2,330 is based on periodic LIFO inventory pricing.Answer (B) is incorrect because 2,805 is the FIFO gross profit.Answer (C) is correct. LIFO assumes that the latest goods purchased are the first sold. In a perpetual system, purchases aredirectly recorded in the inventory account, and cost of goods sold is determined as each sale is made. Accordingly, the costof goods sold using perpetual LIFO is 2,375.

Units Sold Unit Cost150 × 3.50 525100 × 3.30 330200 × 3.10 620300 × 3.00 900

               2,375       

Answer (D) is incorrect because 2,445 is the FIFO cost of goods sold.

[371] Gleim #: 4.21 -- Source: CIA 1196 IV-22

(Refers to Fact Pattern #24)If the enterprise uses periodic FIFO inventory pricing, the gross profit for 2004 would be

2,755A.2,805B.2,854C.2,920D.

Answer (A) is incorrect because 2,755 equals sales minus purchases.Answer (B) is correct. The FIFO method assumes that the first goods purchased are the first goods sold and thatending inventory consists of the latest purchases. Moreover, whether the inventory system is periodic or perpetualdoes not affect FIFO valuation. The cost of goods sold is 2,445 {beginning inventory (200 units × 3.50) + purchases[(250 units × 3.30) + (200 units × 3.10) + (350 units × 3.00)] – ending inventory (250 units × 3.00)}. Thus, the grossprofit for 2004 using FIFO is 2,805 [sales (750 units × 7.00) – cost of goods sold of 2,445].Answer (C) is incorrect because 2,854 uses a weighted-average ending inventory and part of the cost of goods soldcalculation.Answer (D) is incorrect because 2,920 uses a periodic LIFO inventory value.

[372] Gleim #: 4.22 -- Source: CIA 1196 IV-9

The following data are available from the records of a department store for the year ended December 31, 2003:

  At cost At retail Merchandise inventory, as of

January 1, 2003 9,000 13,000Purchases 33,000 46,000Markups (net) 1,000Markdowns (net) 4,000Sales 48,000

Using the retail method to approximate valuation at lower of average cost or net realizable value, the department store’smerchandise inventory at December 31, 2003, is

8,400A.8,000B.6,000C.5,600D.

Gleim's CIA Test Prep: Part III: Business Analysis and Information Technology(1165 questions)

Copyright 2004 Gleim Publications, Inc. Page 140Printed for g j

Page 141: Part three cia_with_ answers

Answer (A) is incorrect because 8,400 neglects to subtract the net markdowns to compute the merchandise inventory atretail at December 31, 2003.Answer (B) is incorrect because 8,000 is the inventory at retail at December 31, 2003.Answer (C) is incorrect because 6,000 is computed by including the net markdowns in the cost-to-retail ratio.Answer (D) is correct. The version of the retail method that approximates a lower-of-average-cost-or-NRV valuationincludes markups but not markdowns in the cost-to-retail ratio. Thus, the cost of the inventory at December 31, 2003 is5,600.

  Cost Retail Inventory, January 1, 2003 9,000 13,000Purchases 33,000 46,000Markups, net 1,000   42,000 60,000Sales (48,000)Markdowns, net (4,000)   Inventory at retail, December 31, 2003 8,000Cost-to-retail ratio (42,000 ÷ 60,000) ×    .70   Inventory at cost, December 31, 2003 5,600   

[373] Gleim #: 4.23 -- Source: CIA 591 IV-34

When the equity method is used to account for the investment in an associate, the recording of the receipt of a cashdistribution from the investee will result in

The recognition of investment income.A.A reduction in the investment balance.B.An increase in a liability account.C.An increase in a special equity account.D.

Answer (A) is incorrect because, when the equity method is used, investment income (loss) is recognized for theinvestee’s share of the profits or losses of the associate. Dividends received from the investee are recorded as areduction of the investment account.Answer (B) is correct. When the equity method is used, the investment is initially recorded at cost on the enterprise’sbooks. The carrying amount is subsequently adjusted to recognize the profits or losses of the associate after the dateof acquisition. Dividends received from an associate reduce the carrying amount.Answer (C) is incorrect because the investment account is credited.Answer (D) is incorrect because the investment account is credited.

[374] Gleim #: 4.24 -- Source: Publisher

Derivatives that are not hedging instruments are always classified in which category of financial instruments?

Financial assets or liabilities held for trading.A.Held-to-maturity investments.B.Loans and receivables originated by the enterprise.C.Available-for-sale financial assets.D.

Gleim's CIA Test Prep: Part III: Business Analysis and Information Technology(1165 questions)

Copyright 2004 Gleim Publications, Inc. Page 141Printed for g j

Page 142: Part three cia_with_ answers

Answer (A) is correct. Financial assets or liabilities held for trading are intended to result in profits from short-termchanges in prices or dealer’s margins. Regardless of intent, however, a financial asset is held for trading if it is included in aportfolio with a recent pattern of short-term profit taking. Derivatives are also deemed to be held for trading unless they aredesignated and effective as hedging instruments.Answer (B) is incorrect because held-to-maturity investments have fixed or determinable payments and a fixed maturity.Moreover, the enterprise must have a positive intent and ability to hold such investments to maturity. However, thisclassification excludes loans and receivables originated by the enterprise.Answer (C) is incorrect because loans and receivables originated by the enterprise result from providing money, goods, orservices directly to a debtor, excluding those that are held for trading.Answer (D) is incorrect because available-for-sale financial assets are those that do not fall within one of the otherclassifications.

[375] Gleim #: 4.25 -- Source: Publisher

Subsequent to their initial recognition, which financial assets with quoted market prices in an active market are measured atfair value?

  Loans and  Receivables  Originated by Financial  Held-to- the Enterprise Available-for- Assets  Maturity and not Held Sale Financial Held for  Investments for Trading Assets Trading A. Yes Yes Yes NoB. Yes Yes No NoC. No No Yes YesD. No No No Yes

Answer (A) is incorrect because available-for-sale and held-for-trading financial assets are measured at fair valueunless they do not have quoted market prices in an active market and their fair values cannot be reliably measured.Answer (B) is incorrect because available-for-sale and held-for-trading financial assets are measured at fair valueunless they do not have quoted market prices in an active market and their fair values cannot be reliably measured.Answer (C) is correct. Subsequent measurement of financial assets depends on their type. Available-for-sale andheld-for-trading financial assets are measured at fair value unless they do not have quoted market prices in an activemarket and their fair values cannot be reliably measured. Loans and receivables originated by the enterprise and notheld for trading, held-to- maturity investments, and financial assets otherwise not qualifying for fair valuemeasurement are measured at amortized cost using the effective interest rate method if they have a fixed maturity. Ifthey do not, they are measured at cost. However, loans and receivables originated by the enterprise and not held fortrading are measured at amortized cost without consideration of whether the enterprise intended to hold them tomaturity.Answer (D) is incorrect because available-for-sale and held-for-trading financial assets are measured at fair valueunless they do not have quoted market prices in an active market and their fair values cannot be reliably measured.

[376] Gleim #: 4.26 -- Source: Publisher

Assuming an available-for-sale financial asset that is not part of a hedge is remeasured to fair value at the balance sheet, thegain or loss

Must be recognized in net profit or loss.A.Must be recognized directly in equity.B.May be recognized in net profit or loss or directly in equity.C.Must be recognized in net profit or loss if the result is a loss and directly in equity if the result is a gain.D.

Gleim's CIA Test Prep: Part III: Business Analysis and Information Technology(1165 questions)

Copyright 2004 Gleim Publications, Inc. Page 142Printed for g j

Page 143: Part three cia_with_ answers

Answer (A) is incorrect because an enterprise should choose either immediate recognition of remeasurement gain or loss innet profit or loss or recognition directly in equity. The chosen accounting policy should then be applied to all available-for-sale financial assets not part of a hedging relationship.Answer (B) is incorrect because an enterprise should choose either immediate recognition of remeasurement gain or loss innet profit or loss or recognition directly in equity. The chosen accounting policy should then be applied to all available-for-sale financial assets not part of a hedging relationship.Answer (C) is correct. Available-for-sale and held-for-trading financial assets are measured at fair value unless they do nothave quoted market prices in an active market and their fair values cannot be reliably measured. Remeasurement to fairvalue of financial assets or liabilities that are not part of a hedge results in recognition of a gain or loss in net profit or loss ifthe items are held for trading. A gain or loss on an available-for-sale financial asset may be recognized in net profit or losswhen it arises or directly in equity through the statement of changes in equity. In the latter case, the accumulatedremeasurement gain or loss is included in net profit or loss when the asset is disposed of or is impaired.Answer (D) is incorrect because an enterprise should choose either immediate recognition of remeasurement gain or loss innet profit or loss or recognition directly in equity. The chosen accounting policy should then be applied to all available-for-sale financial assets not part of a hedging relationship.

[377] Gleim #: 4.27 -- Source: CIA 595 IV-15

An enterprise is depreciating an asset with a 5-year useful life. It cost 100,000 and has no residual value. If the <List A>method is used, depreciation in the second year will be <List B>.

  List A List B A. Sum-of-years’-digits 20,000B. Sum-of-years’-digits 40,000C. 200% diminishing-balance 16,000D. 200% diminishing-balance 24,000

Answer (A) is incorrect because depreciation in year two will be 20,000 under the straight-line method.Answer (B) is incorrect because, under the SYD method, second-year depreciation will be 26,667 [(4 ÷ 15) ×100,000].Answer (C) is incorrect because 16,000 is based on the 100% diminishing-balance method.Answer (D) is correct. The 200% diminishing-balance method uses twice the straight-line rate. Accordingly, first-year depreciation expense is 40,000 (100,000 × 20% × 2). In the second year, the depreciation base is reduced by theamount of depreciation already taken in the first year. Thus, depreciation expense in year two is 24,000 [(2 × 20%) ×(100,000 – 40,000)].

[378] Gleim #: 4.28 -- Source: CIA 595 IV-10

Which of the following is not an appropriate basis for measuring the cost of property, plant, and equipment?

The purchase price, freight costs, and installation costs of a productive asset should be included in the asset’s cost.A.Proceeds obtained in the process of readying land for its intended purpose, such as from the sale of cleared timber,should be recognized immediately as income.

B.

The costs of improvements to equipment incurred after its acquisition should be added to the asset’s cost if theyincrease future service potential.

C.

All costs incurred in the construction of a plant building, from excavation to completion, should be considered as partof the asset’s cost.

D.

Answer (A) is incorrect because the purchase price, freight costs, and installation costs of a productive asset areincluded in the asset’s cost.Answer (B) is correct. Accordingly, items of property, plant, and equipment (PPE) that meet the recognition criteriaare initially measured at cost. The cost includes the purchase price (minus trade discounts and rebates, plus purchasetaxes) and the directly attributable costs of bringing the assets to working condition for their intended use. Directlyattributable costs include site preparation, installation, initial delivery and handling, architect and equipment fees,costs of removing the assets and restoring the site, etc. Accordingly, the cost of land includes the cost of obtaining theland and readying it for its intended uses, but it is inappropriate to recognize the proceeds related to site preparationimmediately as income. They should be treated as reductions in the price of the land.Answer (C) is incorrect because subsequent expenditures are added to the carrying amount of an item of PPE if it isprobable that, as a result, future economic benefits will exceed the originally assessed standard of performance, forexample, because of an extended useful life, improved output quality, or reduced operating costs.Answer (D) is incorrect because all costs of construction should be included as a part of the asset’s cost.

Gleim's CIA Test Prep: Part III: Business Analysis and Information Technology(1165 questions)

Copyright 2004 Gleim Publications, Inc. Page 143Printed for g j

Page 144: Part three cia_with_ answers

[379] Gleim #: 4.29 -- Source: CIA 1196 IV-10A depreciable asset has an estimated 20% residual value. At the end of the asset’s estimated useful life, the accumulateddepreciation will equal the original cost of the asset under which of the following depreciation methods?

  Diminishing- Sum-of-the-Years'-  Balance Digits (SYD) A. Yes YesB. Yes NoC. No YesD. No No

Answer (A) is incorrect because at the end of the estimated useful life of a depreciable asset, the amount ofaccumulated depreciation should equal the depreciable cost (original cost – estimated residual value), regardless ofthe depreciation method used.Answer (B) is incorrect because at the end of the estimated useful life of a depreciable asset, the amount ofaccumulated depreciation should equal the depreciable cost (original cost – estimated residual value), regardless ofthe depreciation method used.Answer (C) is incorrect because at the end of the estimated useful life of a depreciable asset, the amount ofaccumulated depreciation should equal the depreciable cost (original cost – estimated residual value), regardless ofthe depreciation method used.Answer (D) is correct. At the end of the estimated useful life of a depreciable asset, the amount of accumulateddepreciation should equal the depreciable cost (original cost – estimated residual value), regardless of thedepreciation method used. Periodic diminishing-balance depreciation is calculated without regard to residual value,but the asset is not depreciated below its residual value. The SYD method uses a depreciable base equal to cost minusresidual value.

[380] Gleim #: 4.30 -- Source: CIA 1196 IV-29

If an enterprise employs the sum-of-the-years’- digits (SYD) method of depreciation for an asset with an estimated usefullife of 4 years, the percentage of the total depreciable cost that will be expensed in the third year is

10%A.25%B.20%C.70%D.

Answer (A) is incorrect because 10% is the amount expensed in the last year.Answer (B) is incorrect because 25% is the percentage expensed each year under the straight-line method.Answer (C) is correct. Under the SYD method, the amount of the depreciable cost that is expensed each year is theremaining useful life at the beginning of that year divided by the sum of the years of useful life. For the third year, theportion expensed is 20% [2 ÷ (1 + 2 + 3 + 4)].Answer (D) is incorrect because 70% is the total amount expensed in the first 2 years.

[381] Gleim #: 4.31 -- Source: CIA 596 IV-22

An enterprise donated land to a municipality for a park. The acquisition cost of the land was 75,000, and the market valueat the time of the donation was 200,000 as determined by a professional appraisal. If the enterprise has adopted the allowedalternative treatment for measurement of property, plant, and equipment subsequent to initial recognition, the journal entryto record the disposition of the land is

Gleim's CIA Test Prep: Part III: Business Analysis and Information Technology(1165 questions)

Copyright 2004 Gleim Publications, Inc. Page 144Printed for g j

Page 145: Part three cia_with_ answers

A. Expense 75,000Land 75,000

B. Land 125,000Revaluation surplus 125,000

Revaluation surplus 125,000Income 125,000

Expense 200,000Land 200,000

C. Land 125,000Revaluation surplus 125,000

Expense 200,000Land 200,000

D. Donation expense 200,000Land 75,000Income 125,000

Answer (A) is incorrect because the revaluation of the land must be recognized.Answer (B) is incorrect because the revaluation surplus is not transferred through the income statement.Answer (C) is correct. According to the benchmark treatment prescribed by the IASs, an item of PPE is carried at costminus any accumulated depreciation and impairment losses. Under the allowed alternative treatment, an item of PPE may becarried at a revalued amount equal to fair value at the revaluation date minus any subsequent accumulated depreciation andimpairment losses. Land is not depreciated, so it should be carried at its revalued amount. This amount should bedetermined by a professional appraisal. The revaluation increased the carrying amount and is therefore credited directly toequity as revaluation surplus. Accordingly, the entry is to debit land and credit revaluation surplus for 125,000. The entiresurplus is realized at the date of disposition, at which time it may be transferred to retained earnings (but not through theincome statement). Upon disposal, the asset should be removed from the balance sheet, and the resulting loss (revaluedcarrying amount of 200,000 – 0 net disposal proceeds = 200,000) should be recognized as expense in the income statement.Answer (D) is incorrect because the revaluation surplus is not transferred through the income statement.

[382] Gleim #: 4.32 -- Source: CIA 1195 IV-22

An enterprise sells a piece of machinery, for cash, prior to the end of its estimated useful life. The sale price is less than thecarrying amount of the asset on the date of sale. The entry that the enterprise uses to record the sale is

A. CashAccumulated depreciation -- machineryExpense-disposal of machinery

MachineryB. Cash

Accumulated depreciation -- machineryIncome-disposal of machineryMachinery

C. CashExpense-disposal of machinery

Accumulated depreciation -- machineryMachinery

D. CashMachinery

Accumulated depreciation -- machineryExpense-disposal of machinery

Answer (A) is correct. The cash account is debited for the amount of the sale proceeds. The machinery account andthe related accumulated depreciation account are eliminated by a credit and a debit, respectively. Because the saleprice was less than the carrying amount of the asset on the date of sale, a loss on disposal should be recognized as anexpense of the income statement.Answer (B) is incorrect because a loss on disposal should be recognized as an expense of the income statement.Answer (C) is incorrect because accumulated depreciation should be debited.Answer (D) is incorrect because an expense and accumulated depreciation should be debited.

[383] Gleim #: 4.33 -- Source: CIA 1193 IV-42

In making a cash flow analysis of property, plant, and equipment (PPE), the internal auditor discovered that depreciationexpense for the period was 10,000. PPE with a cost of 50,000 and related accumulated depreciation of 30,000 was sold fora gain of 1,000. If the carrying amount of PPE increased by 80,000 during the period, how much PPE was purchased thisperiod?

Gleim's CIA Test Prep: Part III: Business Analysis and Information Technology(1165 questions)

Copyright 2004 Gleim Publications, Inc. Page 145Printed for g j

Page 146: Part three cia_with_ answers

91,000A.100,000B.110,000C.119,000D.

Answer (A) is incorrect because 91,000 ignores the carrying amount of items sold and includes the gain in the computation.Answer (B) is incorrect because 100,000 ignores the depreciation expense for the period.Answer (C) is correct. The carrying amount of the PPE account, net of accumulated depreciation, is increased by the cost ofpurchases and decreased by the carrying amount of items of PPE sold and depreciation. The net PPE decreased by thecarrying amount of items sold, or 20,000 (50,000 cost – 30,000 accumulated depreciation), and by the 10,000 ofdepreciation. If PPE still increased by 80,000, 110,000 (30,000 total decrease + 80,000 increase) of equipment must havebeen purchased.Answer (D) is incorrect because 119,000 double counts depreciation expense and deducts the gain.

[384] Gleim #: 4.34 -- Source: CIA 1193 IV-38

A theme park purchased a new, exciting ride and financed it through the manufacturer. The following facts pertain:

Purchase price 800,000Delivery cost 50,000Installation cost 70,000Cost of trial-runs 40,000Interest charges for first year 60,000

The straight-line method is to be used. Compute the depreciation on the equipment for the first year assuming an estimatedservice life of 5 years.

160,000A.184,000B.192,000C.204,000D.

Answer (A) is incorrect because 160,000 excludes the delivery, installation, and trial-run costs.Answer (B) is incorrect because 184,000 excludes the trial-run cost.Answer (C) is correct. Under the straight-line method, the annual depreciation expense for an asset equals the asset’samount (cost – residual value) divided by the asset’s estimated useful life. The cost of the asset includes its price andthe directly attributable costs of bringing it to working condition for intended use. Thus, the depreciation expense is192,000 [(800,000 purchase price + 50,000 delivery cost + 70,000 installation cost + 40,000 trial-run cost) ÷ 5-yearestimated service life]. Borrowing costs incurred after the asset is prepared for its intended use are expensed even ifthe allowed alternative treatment of such costs is followed and the asset otherwise satisfies the criteria forcapitalization expensed.Answer (D) is incorrect because 204,000 includes the borrowing costs.

[385] Gleim #: 4.35 -- Source: CIA 1196 IV-32

On January 1, 2000, an enterprise purchased an abandoned quarry for 1,200,000 to be used as a landfill to service its trashcollection contracts with nearby cities for the next 20 years. The enterprise depletes the quarry using the sum-of-the-unitsmethod based on a surveyor’s measurements of volume of the quarry’s pit. This amount was 500,000 cubic yards whenpurchased and 350,000 cubic yards at year-end 2004. What is the net amount that should be shown on the enterprise’sDecember 31, 2004, balance sheet for the quarry?

1,200,000A.900,000B.840,000C.360,000D.

Answer (A) is incorrect because 1,200,000 is the purchase price of the quarry.Answer (B) is incorrect because 900,000 results from using straight-line depletion.Answer (C) is correct. The sum-of-the-units method allocates cost based on output. The net amount reported as anasset for the quarry using this method is 840,000 [(350,000 cubic yards ÷ 500,000 total cubic yards) × 1,200,000].Answer (D) is incorrect because 360,000 equals {1,200,000 – [(350,000 ÷ 500,000) × 1,200,000]}. The 350,000cubic yards is the volume remaining in the quarry’s pit at year-end 2004, not the volume depleted.

Gleim's CIA Test Prep: Part III: Business Analysis and Information Technology(1165 questions)

Copyright 2004 Gleim Publications, Inc. Page 146Printed for g j

Page 147: Part three cia_with_ answers

[386] Gleim #: 4.36 -- Source: CIA 592 IV-28

An enterprise installed an assembly line in 2000. Four years later, 100,000 was invested to automate the line. Theautomation increased the market value and productive capacity of the assembly line but did not affect its useful life. Properaccounting for the cost of the automation should be to

Report it as an expense in 2004.A.Establish a separate account for the 100,000.B.Allocate the cost of automation between the asset and accumulated depreciation accounts.C.Debit the cost to the property, plant, and equipment account.D.

Answer (A) is incorrect because the cost should be capitalized.Answer (B) is incorrect because the same account should be used.Answer (C) is incorrect because allocation is not an accepted procedure.Answer (D) is correct. Subsequent expenditures are added to the carrying amount of an item of PPE if it is probablethat, as a result, future economic benefits will exceed the originally assessed standard of performance, for example,because of an extended useful life, improved output quantity or quality, or reduced operating costs.

[387] Gleim #: 4.37 -- Source: CIA 1196 IV-18

An enterprise purchased new equipment on July 1, 2004, having a list price of 52,500. The enterprise traded old equipmentthat was being depreciated using the straight-line method and paid 35,000 in cash. The following information pertains tothe old equipment:

Cost on January 1, 2001 38,900Estimated useful life 5 yearsResidual value 2,900Fair value on July 1, 2004 16,000

If the old and new equipment are dissimilar, the enterprise will record the new equipment at

45,100A.48,700B.51,000C.52,500D.

Answer (A) is incorrect because 45,100 equals the book value of the old equipment after being depreciated for 4years plus the cash paid for the new equipment.Answer (B) is incorrect because 48,700 equals the carrying amount of the old equipment after being depreciated for3.5 years plus cash paid for the new equipment.Answer (C) is correct. An exchange of dissimilar items of PPE is considered the culmination of an earning process.Thus, the transaction should be recorded at the fair value of the asset received, which is equivalent to the fair value ofthe asset surrendered adjusted for any cash transferred. Hence, the enterprise will record the new equipment at the fairvalue of the old asset plus cash paid, or 51,000 (16,000 + 35,000).Answer (D) is incorrect because 52,500 is the list price of the new equipment.

[388] Gleim #: 4.38 -- Source: CIA 595 IV-25

Which of the following is not considered to be an intangible asset?

Goods on consignment.A.Patents.B.Copyrights.C.Trademarks.D.

Answer (A) is correct. IAS 38, Intangible Assets, defines an intangible asset as “an identifiable nonmonetary assetwithout physical substance held for use in the production or supply of goods or services, for rental to others, or foradministrative purposes.”  Inventory is a tangible asset. Thus, goods on consignment are not intangible assets.Answer (B) is incorrect because patents are intangible assets.Answer (C) is incorrect because copyrights are intangible assets.Answer (D) is incorrect because trademarks are intangible assets.

Gleim's CIA Test Prep: Part III: Business Analysis and Information Technology(1165 questions)

Copyright 2004 Gleim Publications, Inc. Page 147Printed for g j

Page 148: Part three cia_with_ answers

[389] Gleim #: 4.39 -- Source: CIA 593 IV-32

The costs of start-up activities, including fees of attorneys, should be

Capitalized, but not amortized, because of the indefinite life of the business.A.Capitalized and amortized.B.Capitalized and deferred until liquidation of the business.C.Expensed when incurred.D.

Answer (A) is incorrect because start-up costs are expensed when incurred.Answer (B) is incorrect because start-up costs are expensed when incurred.Answer (C) is incorrect because start-up costs are expensed when incurred.Answer (D) is correct. Expenditures on start-up activities are expensed when incurred unless they are included in thecost of an item of property, plant, and equipment. They include the costs of establishing a new legal entity, such aslegal and secretarial costs; pre-opening costs of an enterprise’s new business facility; and the pre-operating costs ofnew operations, products, or processes.

[390] Gleim #: 4.40 -- Source: CIA 590 IV-25

The amortization of intangible assets over their useful lives is justified by the

Economic entity assumption.A.Going concern assumption.B.Monetary unit assumption.C.Historical cost assumption.D.

Answer (A) is incorrect because the economic entity assumption is that every enterprise’s affairs are separate fromthose of its owners.Answer (B) is correct. Every business is assumed to be a going concern that will continue operating indefinitely.Thus, liquidation values are not important. If an enterprise is not a going concern, its intangible assets are reported atliquidation values, not at historical cost net of amortization.Answer (C) is incorrect because the monetary unit assumption provides that all transactions and events can bemeasured in terms of money.Answer (D) is incorrect because the historical cost principle deems cost to be the most objective and reliable measure.

[391] Gleim #: 4.41 -- Source: CIA 595 IV-71

The restriction that manufacturers should not market a new product that is illegally similar to that of another company’sproduct is due to which public policy instrument?

Copyright.A.Minimum standards for product warranties.B.Anti-merger laws.C.Patent laws.D.

Answer (A) is incorrect because a copyright provides legal protection for tangible expressions of ideas, e.g., novels,songs, and software.Answer (B) is incorrect because laws establishing minimum warranty standards do not limit the similarity of productofferings.Answer (C) is incorrect because anti-merger laws can affect the ability of one enterprise to acquire another enterpriseproducing similar products if the result will be to lessen competition, but they do not affect the design of newproducts.Answer (D) is correct. A patent is the exclusive legal right to use or sell an invention, such as a device or process. Apatent may be given to any new and useful process, machine, manufacture, or composition of matter, and anyinfringement of a patent is a basis for a lawsuit. Thus, patent laws require that enterprises not design new productsthat are illegally similar to those of other enterprises that enjoy patent protection.

[392] Gleim #: 4.42 -- Source: CIA 593 IV-43

MNO purchased all of XYZ’s 100,000 outstanding common shares for 40 per share on August 31. On this date, XYZ’sbalance sheet showed total assets of 5,000,000 and total liabilities of 2,000,000. The fair value of XYZ’s identifiable assetson this date was 550,000 greater than their carrying amount. The amount that should be reported on MNO’s consolidatedbalance sheet on August 31 for goodwill is

Gleim's CIA Test Prep: Part III: Business Analysis and Information Technology(1165 questions)

Copyright 2004 Gleim Publications, Inc. Page 148Printed for g j

Page 149: Part three cia_with_ answers

0A.450,000B.550,000C.1,000,000D.

Answer (A) is incorrect because cost in excess of fair value of the identifiable assets and liabilities is reported as goodwill.Answer (B) is correct. Goodwill is recognized as an asset equal to the excess of the cost over the acquirer’s interest in thefair value of the identifiable assets and liabilities acquired. The cost of 4,000,000 (40 × 100,000) is in excess of the3,550,000 (5,000,000 + 550,000 – 2,000,000) fair value of the identifiable assets and liabilities by 450,000. This excess isgoodwill.Answer (C) is incorrect because 550,000 is the excess of fair value over the carrying amount of the identifiable assets andliabilities on the seller’s books.Answer (D) is incorrect because the purchase price of 4,000,000 exceeds the seller’s 3,000,000 carrying amount by1,000,000.

[393] Gleim #: 4.43 -- Source: Publisher

According to IAS 41, Agriculture, the usual measurement of

Agricultural produce is at fair value at the point of harvest.A.Agricultural produce is at fair value minus point-of-sale costs at the point of harvest and at each subsequent balancesheet date.

B.

Biological assets is at fair value minus estimated point-of-sale costs at initial recognition and at each subsequentbalance sheet date.

C.

Biological assets is at cost minus accumulated depreciation and impairment losses.D.

Answer (A) is incorrect because agricultural produce is measured at fair value minus estimated point-of-sale costs atthe point of harvest.Answer (B) is incorrect because agricultural produce is measured at fair value minus estimated point-of-sale costs atthe point of harvest. It is not remeasured for subsequent changes.Answer (C) is correct. Measurement of biological assets upon initial recognition and agricultural produce from theenterprise’s biological assets at the point of harvest ordinarily is at fair value minus estimated point-of-sale costs.Biological assets are also remeasured at subsequent balance sheet dates at fair value minus estimated point-of-salecosts.Answer (D) is incorrect because biological assets are measured at cost minus accumulated depreciation andimpairment losses only if the presumption that their fair value is reliably measurable is rebutted. Such rebuttal ispermissible only upon initial recognition.

[394] Gleim #: 4.44 -- Source: Publisher

A gain or loss may be included in the determination of net profit or loss for

Initial recognition of a biological asset at fair value minus estimated point-of-sale costsI.A change in fair value minus estimated point-of-sale costs of a biological assetII.Initial recognition of agricultural produce at fair value minus estimated point-of-sale costsIII.A change in fair value minus estimated point-of-sale costs of agricultural produceIV.

I and II only.A.II and IV only.B.I, II, and III only.C.I, II, III, and IV.D.

Answer (A) is incorrect because a gain or loss may be reported in net profit or loss for initial recognition ofagricultural produce at fair value minus estimated point-of-sale costs.Answer (B) is incorrect because agricultural produce is measured only at the point of harvest.Answer (C) is correct. Recognition of gains and losses on biological assets may result from initial recognition at, orfrom a change in, fair value minus estimated point-of-sale costs. These gains and losses should be included in netprofit or loss as they occur. A gain or loss from the initial recognition of agricultural produce is included in net profitor loss as it occurs. However, agricultural produce is measured only at the point of harvest.Answer (D) is incorrect because agricultural produce is measured only at the point of harvest.

Gleim's CIA Test Prep: Part III: Business Analysis and Information Technology(1165 questions)

Copyright 2004 Gleim Publications, Inc. Page 149Printed for g j

Page 150: Part three cia_with_ answers

[395] Gleim #: 4.45 -- Source: CIA 597 IV-29An enterprise introduced a new product that carries a 2-year warranty against defects. It estimates that warranty costs willbe 2% of sales in the year of sale and 3% of sales in the year following the year of sale. Sales in year 1 and year 2 were5 million and 7 million, respectively. Actual costs of servicing the warranty in year 1 and year 2 were 110,000 and 260,000,respectively. What provision for warranty costs must the enterprise recognize in year 2?

260,000A.290,000B.350,000C.370,000D.

Answer (A) is incorrect because 260,000 is the actual cost of servicing the warranty in year 2.Answer (B) is incorrect because 290,000 equals the sum of 2% of year 2 sales and 3% of year 1 sales.Answer (C) is correct. The warranty provision must be matched with revenue in the year of sale. Thus, the provisionrelated to year 2 sales must be recognized in year 2 even if actual expenditures will not occur until year 3. Theprovision related to year 2 sales equals 350,000 [7,000,000 × (2% for the year of sale + 3% for the year after the yearof sale)].Answer (D) is incorrect because 370,000 equals the actual cost of servicing the warranty in year 1 and year 2.

[396] Gleim #: 4.46 -- Source: CIA 590 IV-34

On December 31, 2000, XYZ issued 5-year bonds with a face amount of 1 million. The bonds carry a stated interest rate of10% and were sold at par. Interest is payable annually on December 31. According to the provisions of the bond indenture,XYZ was to make annual deposits into a bond sinking fund (beginning December 31, 2001) to accumulate the fundsnecessary to retire the bonds at their maturity. On December 31, 2004, all required interest payments and sinking-fundpayments due to date had been made on schedule. If the sinking-fund assets are properly classified as noncurrent, howshould the balance of bonds payable be classified on the December 31, 2004 balance sheet?

Current liability.A.Long-term liability.B.Contra to long-term investments.C.Deferred credit.D.

Answer (A) is correct. A current liability is an obligation that is expected to be settled within the normal operatingcycle or is due to be settled within 12 months of the balance sheet date. Any other liability is noncurrent. Somecurrent liabilities are included in the working capital employed in the normal operating cycle, e.g., trade payables andaccrued employee operating costs. Current liabilities not settled within the normal operating cycle include the currentpart of interest-bearing debt, dividends, income taxes, and bank overdrafts. Thus, the bonds payable should beclassified as current because they are due to be settled within 12 months. Under the IASs, the classification of thesinking-fund assets is irrelevant to the classification of the bond payable.Answer (B) is incorrect because the bonds should be classified as a current liability.Answer (C) is incorrect because offsetting assets and liabilities is rarely acceptable.Answer (D) is incorrect because the bonds are a liability and should not be put in an ambiguous category such asdeferred credits.

[397] Gleim #: 4.47 -- Source: CIA 1192 IV-31

On August 1, 2000, an enterprise issued 5-year bonds with a face amount of 10 million. The bonds carry a stated interestrate of 10% and interest is payable annually on July 31. Which is the appropriate classification of bonds payable and therelated accrued interest payable on the December 31, 2004 balance sheet?

Classification Table  Bonds Payable Interest Payable Classification A Current liability Current liabilityClassification B Current liability Long-term liabilityClassification C Long-term liability Current liabilityClassification D Long-term liability Long-term liability

Gleim's CIA Test Prep: Part III: Business Analysis and Information Technology(1165 questions)

Copyright 2004 Gleim Publications, Inc. Page 150Printed for g j

Page 151: Part three cia_with_ answers

Classification A.A.Classification B.B.Classification C.C.Classification D.D.

Answer (A) is correct. A current liability is an obligation that is expected to be settled within the normal operating cycle oris due to be settled within 12 months of the balance sheet date. Any other liability is noncurrent. Some current liabilities areincluded in the working capital employed in the normal operation cycle, e.g., trade payables and accrued employeeoperating costs. Current liabilities not settled within the normal operating cycle include the current part of interest-bearingdebt, dividends, income taxes, and bank overdrafts. Given that the bonds payable and interest payable are due within 12months, they should be classified and current.Answer (B) is incorrect because the interest payable should be classified as a current liability. It is due within a year afterthe balance sheet date.Answer (C) is incorrect because the balance of bonds payable should be classified as a current liability. The bonds are duewithin a year after the balance sheet date.Answer (D) is incorrect because both the balance of bonds payable and interest payable should be classified as currentliabilities.

[398] Gleim #: 4.48 -- Source: CIA 1193 IV-39

A manufacturer produces a quality product for which it charges a little more than some competing items but gives itsconsumers a more liberal warranty policy. The product carries a 5-year warranty that covers both labor and materialscharges. Which of the following defines the appropriate method of accounting for the warranty?

Cash basis.A.Recognition of a provision.B.Sales warranty.C.Tax basis.D.

Answer (A) is incorrect because the cash basis calls for recognizing warranty expense as labor and materials areexpended to satisfy the warranty.Answer (B) is correct. A provision is a liability of uncertain timing and amount. A liability is a present obligationarising from past events, the settlement of which is expected to result in an outflow of resources embodying economicbenefits. Whether a past event results in a present obligation is usually clear. Thus, it is clear from the circumstancesthat the enterprise’s sale of goods without warranty is an obligating event that resulted in a present obligation for theissuance of warranty costs. Recognition of provisions is appropriate when the enterprise has a legal or constructivepresent obligation resulting from a past event (called an obligating event), it is probable that an outflow of economicbenefits will be necessary to settle the obligation, and its amount can be reliably estimated. Assuming that the amountof warranty costs can be reliably estimated (although they are uncertain in timing and amount compared with a tradepayable, for example) and that the outflow is probable (in these circumstances, “more likely than not”), themanufacturer’s contractual present obligation should result in recognition of a provision.Answer (C) is incorrect because the sales warranty method is appropriate for situations when a warranty is soldseparately from the product.Answer (D) is incorrect because the method of accounting for warranties for tax purposes is the cash basis. The cashbasis is unacceptable for accounting purposes because it violates the matching principle.

[399] Gleim #: 4.49 -- Source: CIA 1191 IV-41

At its balance sheet date, an enterprise reliably estimates that the expected cost of compensated absences resulting fromshort-term disability will be 100,000. This unused entitlement arose from employee services rendered during the period andis accumulating. How should it be accounted for?

Only a disclosure should be made because recognition does not occur until the absences occur.A.An expense should be recorded for 100,000.B.An asset of 100,000 should be recognized.C.A direct reduction to retained earnings of 100,000 should occur.D.

Gleim's CIA Test Prep: Part III: Business Analysis and Information Technology(1165 questions)

Copyright 2004 Gleim Publications, Inc. Page 151Printed for g j

Page 152: Part three cia_with_ answers

Answer (A) is incorrect because recognition occurs at the time of the absences if the benefits are not accumulating.Answer (B) is correct. Short-term employee benefits expected to be paid as a result of service rendered during the periodordinarily should be recognized as an expense and a liability (accrued expense). For short-term compensated absences, thetiming of recognition depends on whether the benefits accumulate. If the benefits for compensated absences accumulate, theexpected cost of short-term compensated absences is recognized when services are rendered that increase the employees’entitlement to future compensated absences. The obligation is recognized whether it is vesting (the employee is entitled to acash payment for an unused entitlement upon leaving the enterprise) or not vesting. The amount should not be discounted. Itequals the additional amount expected to be paid as a result of the unused accumulated entitlement at the balance sheet date.Hence, the enterprise should debit expense and credit liability for 100,000 because the entitlement accumulates and theemployees have rendered services during the period that increase their future entitlement.Answer (C) is incorrect because a liability rather than an asset is recognized.Answer (D) is incorrect because the expense is recognized in the income statement.

[400] Gleim #: 4.50 -- Source: CIA 1190 IV-37

A company allows customers to redeem 20 coupons for a toy (cost 3.00). Estimates are that 40% of coupons distributedwill result in redemption. Since beginning the promotion this year, 4 million coupons were distributed and 1 millioncoupons redeemed. The adjusting entry to accrue for unredeemed coupons at year-end is

A. Premium expense 90,000Provision

for premiums 90,000B. Sales 90,000

Provisionfor premiums 90,000

C. Premium expense 1,800,000Provision

for premiums 1,800,000D. Sales 1,800,000

Provisionfor premiums 1,800,000

Answer (A) is correct. An expense and a provision should be accrued for the coupons still outstanding that areexpected to be redeemed. Of the 4 million coupons distributed, 40%, or 1.6 million, are estimated to be redeemable.Of those, 1 million have already been redeemed, and 600,000 more are expected to be redeemed. The promotionrequires 20 coupons to receive one toy, so 30,000 (600,000 ÷ 20) more toys will be required. Each toy costs 3.00,creating a provision of 90,000 (30,000 × 3.00).Answer (B) is incorrect because the debit should be to an expense.Answer (C) is incorrect because, although an expense should be accrued, the amount is incorrect.Answer (D) is incorrect because the debit should be to an expense, and the amount is incorrect.

[401] Gleim #: 4.51 -- Source: CIA 1192 IV-29

The publisher of a popular magazine offers a special discounted price for a 3-year subscription. At the balance sheet date,the amount that has already been collected but pertains to future periods is best referred to as

Accrued subscriptions revenue (an asset account).A.Deferred subscriptions revenue (a liability account).B.Earned subscriptions revenue (a revenue account).C.Precollected subscriptions receivable (a deferred asset account).D.

Gleim's CIA Test Prep: Part III: Business Analysis and Information Technology(1165 questions)

Copyright 2004 Gleim Publications, Inc. Page 152Printed for g j

Page 153: Part three cia_with_ answers

Answer (A) is incorrect because an accrued revenue is revenue that has met the recognition criteria but has not beenreceived.Answer (B) is correct. Income, which includes revenue and gains, is recognized in the income statement when an increasein future economic benefits related to an increase in an asset or a decrease in a liability can be reliably measured. Revenue isrecognized (reported as revenue) in the period in which the recognition criteria are met; therefore, when it is received inadvance, the amount applicable to future periods is deferred. This deferral reflects the uncertainty of the reliablemeasurement of the future economic benefits. The uncertainty arises because the enterprise still must satisfy an obligation toperform in the future before it is entitled to the future economic benefits. The amount received in advance is considered aliability because it represents a present obligation arising from a past event. Accordingly, deferred or unearned revenue is anamount that has been received but that has not met the recognition criteria for revenue.Answer (C) is incorrect because the revenue will be recognized in future periods when forthcoming issues of the magazineare published and distributed to the subscribers.Answer (D) is incorrect because there is no such thing as a precollected receivable. Precollected revenue is deferredrevenue, which is an amount received that has not met the recognition criteria (classified as a liability). A subscriptionreceivable (an asset) would arise from accrued revenue, which is revenue not yet received.

[402] Gleim #: 4.52 -- Source: CIA 592 IV-38

A cable television enterprise receives deposits from customers that are refunded when service is terminated. The averagecustomer stays with the enterprise 8 years. How should these deposits be shown on the financial statements?

Operating revenue.A.Other revenue.B.Share capital.C.Liability.D.

Answer (A) is incorrect because deposits meet the definition of liabilities, not revenue. Revenue is income that arisesin the ordinary activities of the enterprise. Income is an increase in economic benefits in the form of inflows orenhancements of assets or decreases of liabilities that result in an increase in equity (excluding transactions withowners).Answer (B) is incorrect because deposits meet the definition of liabilities, not revenue. Revenue is income that arisesin the ordinary activities of the enterprise. Income is an increase in economic benefits in the form of inflows orenhancements of assets or decreases of liabilities that result in an increase in equity (excluding transactions withowners).Answer (C) is incorrect because deposits are liabilities, not equity items. The equity of an enterprise is the residualinterest in the assets of an enterprise that remains after deducting its liabilities.Answer (D) is correct. Liabilities are present obligations arising from past events, the settlement of which is expectedto result in an outflow of resources embodying economic benefits. Customers’ deposits must be returned or creditedto their accounts. The deposits should therefore be recorded as liabilities.

[403] Gleim #: 4.53 -- Source: CIA 590 IV-28

In performing an audit, you encounter an adjusting journal entry recorded at year-end that contains a debit to rental revenueand a credit to deferred rental revenue. The purpose of this journal entry is to record

An accrued revenue.A.An unexpired expense.B.An expired expense.C.A liability.D.

Answer (A) is incorrect because an accrued revenue has met the recognition criteria but has not yet been received.The journal entry described indicates that collection has been made.Answer (B) is incorrect because the entry concerns a revenue rather than an expense transaction.Answer (C) is incorrect because the entry concerns a revenue rather than an expense transaction.Answer (D) is correct. A deferred revenue is a revenue item that has been received but has not met the recognitioncriteria. The journal entry described in the question is an adjusting entry to transfer an amount from the revenueaccount to a liability (deferred revenue) account. The initial collection of cash in advance from the tenant wasapparently recorded by a credit to revenue. An adjusting entry is therefore required at year-end to transfer anyremaining amount that does not qualify for revenue recognition.

Gleim's CIA Test Prep: Part III: Business Analysis and Information Technology(1165 questions)

Copyright 2004 Gleim Publications, Inc. Page 153Printed for g j

Page 154: Part three cia_with_ answers

[404] Gleim #: 4.54 -- Source: CIA 597 IV-36Because of a defect discovered in its seat belts in December 2004, an automobile manufacturer believes it is probable that itwill be required to recall its products. The final decision on the recall is expected to be made in March 2005. The cost ofthe recall is reliably estimated to be 2.5 million. How should this information be reported in the December 31, 2004financial statements?

As a loss of 2.5 million and a provision of 2.5 million.A.As an adjustment of the opening balance of retained earnings equal to 2.5 million.B.As an appropriation of retained earnings of 2.5 million.C.It should not be disclosed because it has not yet happened.D.

Answer (A) is correct. A provision is a liability of uncertain timing or amount. Recognition of provisions isappropriate when the enterprise has a legal or constructive present obligation resulting from a past event (called anobligating event), it is probable that an outflow of economic benefits will be necessary to settle the obligation, and itsamount can be reliably estimated. Consequently, the company must recognize a loss and a liability for 2.5 million.Answer (B) is incorrect because such an adjustment is appropriate for fundamental errors and changes in accountingpolicies (under the benchmark treatments).Answer (C) is incorrect because an appropriation of retained earnings is permissible although not required, but theenterprise must still recognize a loss and a provision. Moreover, no part of the appropriation may be transferred toincome, and no loss may be charged to an appropriation of retained earnings.Answer (D) is incorrect because, if the loss is probable and can be reliably estimated, it should be recognized by acharge to income.

[405] Gleim #: 4.55 -- Source: CIA 1192 IV-45

An enterprise is subject to warranty claims. A reliable estimate is that between 1 million and 3 million will probably bepaid out. No estimate of loss within this range is more likely than any other. The enterprise should

Make no journal entry at this time.A.Disclose only a possible loss.B.Defer a provision of 1 million to 3 million depending on the applicable national accounting standards.C.Recognize a provision of 1 million to 3 million depending on the applicable national accounting standards.D.

Answer (A) is incorrect because the criteria for recognition of a provision have been met, including a reliableestimate of the range of the obligation.Answer (B) is incorrect because the loss is probable.Answer (C) is incorrect because the loss is not deferred; it is accrued.Answer (D) is correct. A provision is a liability of uncertain timing or amount. Recognition of provisions isappropriate when the enterprise has a legal or constructive present obligation resulting from a past event (called anobligating event), it is probable that an outflow of economic benefits will be necessary to settle the obligation, and itsamount can be reliably estimated. Thus, a provision should be recognized. Moreover, the amount recognized shouldbe the best estimate of the expenditure required to settle the obligation. However, the amount within the reliableestimate of the range of the obligation that will be recognized will vary from country to country.

[406] Gleim #: 4.56 -- Source: CIA 596 IV-25

Which one of the following will usually be accounted for by recognizing a provision?

Just prior to the balance sheet date, the board decided to close a division. No implementation steps have been taken.A.As of the balance sheet date, the board was aware that a new law would require the enterprise to fit smoke filters to itsfactories within the next year. No such filters have been fitted.

B.

A law requires an airline to overhaul its aircraft once every 3 years.C.Premiums offered to customers.D.

Gleim's CIA Test Prep: Part III: Business Analysis and Information Technology(1165 questions)

Copyright 2004 Gleim Publications, Inc. Page 154Printed for g j

Page 155: Part three cia_with_ answers

Answer (A) is incorrect because no obligating event has occurred. The enterprise could avoid the future expenditure by itsfuture actions.Answer (B) is incorrect because no obligating event has occurred. The enterprise could avoid the future expenditure by itsfuture actions.Answer (C) is incorrect because no obligating event has occurred. The enterprise could avoid the future expenditure by itsfuture actions.Answer (D) is correct. When premiums are offered to customers, for example, upon redemption of coupons, the enterprisecan usually establish that it has a legal present obligation resulting from a past event and that an outflow of economicbenefits is probable. Furthermore, if the enterprise has prior experience with such offers or information about the experienceof similar enterprises, a reliable estimate of the obligation should be feasible.

[407] Gleim #: 4.57 -- Source: CIA 595 IV-20

Which of the following is not a factor, with respect to pending litigation, that must be considered in determining whether aprovision should be recognized?

The time period in which the underlying cause of action occurred.A.The probability of an unfavorable outcome.B.The ability to make a reliable estimate of the amount of loss.C.The number of parties involved in the litigation.D.

Answer (A) is incorrect because the time period in which the obligating event occurred is relevant. If it arose after thedate of the financial statements, a provision may not be recognized in those statements.Answer (B) is incorrect because a provision is not recognized unless it is probable that an outflow of resourcesembodying economic benefits will be required to settle a present obligation arising from a past event.Answer (C) is incorrect because a provision is not recognized unless it is probable that an outflow of resourcesembodying economic benefits will be required to settle a present obligation arising from a past event. Moreover, theamount of the obligation should be capable of reliable estimation.Answer (D) is correct. The number of parties involved in the litigation is irrelevant. For example, the sameaccounting treatment is applied whether a claim is brought by an individual or in a class action suit.

[408] Gleim #: 4.58 -- Source: CIA 589 IV-33

An enterprise has been sued for 100 million for producing and selling an unsafe product. Attorneys for the enterprisecannot reliably predict the outcome of the litigation. In its financial statements, the enterprise should

A. Make the following journal entry, and disclose the existence of the lawsuit in a footnote.Estimated loss

from litigation 100,000,000

Estimated provisionfor litigation loss 100,000,000

B. Disclose the existence of the lawsuit in a note without making a journal entry.C. Neither make a journal entry nor disclose the lawsuits in a note because bad publicity will hurt the enterprise.D. Make the following journal entry, and disclose the existence of the lawsuit in a note.

Cost of goodssold 100,000,000

Estimated provisionfor litigation loss 100,000,000

Answer (A) is incorrect because a journal entry is made when the outflow in settlement is probable and can bereliably estimated.Answer (B) is correct. In the very rare case in which a reliable estimate of an obligation that otherwise qualifies fortreatment as a provision cannot be determined, no liability is recognized. Instead, the existing liability is disclosed asa contingent liability (unless the possibility of any outflow in settlement is remote).Answer (C) is incorrect because a disclosure must be made of a contingent liability.Answer (D) is incorrect because a journal entry is made when the outflow in settlement is probable and can bereliably estimated.

[409] Gleim #: 4.59 -- Source: Publisher

An enterprise is currently being sued by a customer. A reliable estimate can be made of the costs that would result from aruling unfavorable to the enterprise, and the amount involved is material. The enterprise’s managers, lawyers, and auditorsagree that the likelihood of an unfavorable ruling is remote. This contingent liability

Gleim's CIA Test Prep: Part III: Business Analysis and Information Technology(1165 questions)

Copyright 2004 Gleim Publications, Inc. Page 155Printed for g j

Page 156: Part three cia_with_ answers

Should be disclosed in a note.A.Should be disclosed as a parenthetical comment in the balance sheet.B.Need not be disclosed.C.Should be disclosed by an appropriation of retained earnings.D.

Answer (A) is incorrect because the contingent liability is required to be neither recognized nor disclosed.Answer (B) is incorrect because the contingent liability is required to be neither recognized nor disclosed.Answer (C) is correct. A contingent liability includes a present obligation for which an outflow of resources embodyingeconomic benefits is not probable. A contingent liability is not recognized but is disclosed unless the possibility of theoutflow is remote.Answer (D) is incorrect because the contingent liability is required to be neither recognized nor disclosed.

[410] Gleim #: 4.60 -- Source: CIA 597 IV-19

In November of the current year, the vice-president of a local bank reviews the bank’s mortgage portfolio prior to theDecember 31 year-end. The bank’s largest client has mortgages on buildings in three cities. The client has incurred netlosses for the past 3 years and is now experiencing serious cash flow problems. For the past 6 months, no payments havebeen made on any of the three mortgages. The vice-president reluctantly concludes that it is probable that the full amountof principal and interest will not be collected. What is the impact of this conclusion on the local bank’s current yearfinancial statements?

No accounting or disclosure of a possible loss in value is necessary.A.Contingency note disclosure of a possible impairment is required.B.The carrying amount of the mortgages should be reduced, with a charge directly to retained earnings.C.The carrying amount of the mortgages should be reduced, with a charge to the income statement.D.

Answer (A) is incorrect because the carrying amount should be reduced, and the charge should be included in netprofit or loss.Answer (B) is incorrect because the carrying amount should be reduced, and the charge should be included in netprofit or loss.Answer (C) is incorrect because the carrying amount should be reduced, and the charge should be included in netprofit or loss.Answer (D) is correct. A financial asset, such as a lender’s mortgage receivable, is impaired if its carrying amount atthe balance sheet date exceeds its estimated recoverable amount. If it is probable that all amounts due on such a held-to-maturity investment (a financial asset carried at amortized cost) cannot be collected, the loss should be included innet profit or loss. The loss equals the difference between the carrying amount and the present value of the expectedfuture cash flows discounted at the original effective interest rate. The carrying amount of the asset should be reducedto its estimated recoverable amount directly or by crediting an allowance account.

[411] Gleim #: 4.61 -- Source: CIA 597 IV-30

If bonds are initially sold at a discount and the effective-interest method of amortization is used,

Interest expense in the earlier periods will be less than interest expense in the later periods.A.Interest expense in the earlier periods will be greater than interest expense in the later periods.B.Interest expense will equal the cash interest payment each period.C.Interest expense will be less than the cash interest payment each period.D.

Answer (A) is correct. Interest expense equals the carrying amount of the liability at the beginning of the periodtimes the effective interest rate. The carrying amount of the liability equals the face amount of the bond minus thediscount. As the discount is amortized over the life of the bond, the carrying amount increases. Consequently, theinterest expense increases over the term of the bond.Answer (B) is incorrect because interest expense will increase over the term of the bonds.Answer (C) is incorrect because interest expense exceeds the cash interest payment when bonds are issued at adiscount. The reason is that the effective rate is higher than the nominal rate. The excess of interest expense over thecash payment is the amount of discount amortized each period.Answer (D) is incorrect because interest expense exceeds the cash interest payment when bonds are issued at adiscount. The reason is that the effective rate is higher than the nominal rate. The excess of interest expense over thecash payment is the amount of discount amortized each period.

Gleim's CIA Test Prep: Part III: Business Analysis and Information Technology(1165 questions)

Copyright 2004 Gleim Publications, Inc. Page 156Printed for g j

Page 157: Part three cia_with_ answers

[412] Gleim #: 4.62 -- Source: CIA 595 IV-19An enterprise issues 10-year bonds with a face value of 1 million, dated January 1, 2004 and bearing interest at an annualrate of 12% payable semiannually on January 1 and July 1. The full interest amount will be paid each due date. The marketrate of interest on bonds of similar risk and maturity, with the same schedule of interest payments, is also 12%. If the bondsare issued on February 1, 2004, the amount the issuing enterprise receives from the buyers of the bonds on that date is

990,000A.1,000,000B.1,010,000C.1,020,000D.

Answer (A) is incorrect because 990,000 is the result if 1 month of accrued interest is deducted from, rather thanadded to, the amount received.Answer (B) is incorrect because the purchasers must pay for the accrued interest from the last interest date to theissue date. They will receive 6 months’ interest on July 1 despite holding the bonds for 5 months.Answer (C) is correct. The amount the issuing enterprise receives on 2/1/04 is the face amount of the issue plus 1month of accrued interest, or 1,010,000 {1,000,000 + [(1,000,000 × 12%) ÷ 12]}.Answer (D) is incorrect because 1,020,000 results from adding 2 months of accrued interest to the face amount.

[413] Gleim #: 4.63 -- Source: CIA 1191 IV-39

If bonds payable with a carrying amount equal to par value are refunded by use of a call provision, the call premium of therefunded issue should be

Amortized over the remaining original life of the extinguished issue.A.Amortized over the life of the new issue.B.Recognized currently in net profit or loss as an extraordinary item.C.Recognized currently in net profit or loss.D.

Answer (A) is incorrect because the excess of the reacquisition price over the net carrying amount of the old bonds isrecognized in full in net profit or loss for the period.Answer (B) is incorrect because the excess of the reacquisition price over the net carrying amount of the old bonds isrecognized in full in net profit or loss for the period.Answer (C) is incorrect because an extraordinary item is income or an expense arising from events or transactionsthat are clearly distinct from the ordinary activities of the enterprise and are not expected to recur frequently orregularly.Answer (D) is correct. An enterprise should remove a financial liability from its balance sheet only when it isextinguished, for example, when the creditor is paid. The difference between the carrying amount and the amountpaid should be included in net profit or loss for the period.

[414] Gleim #: 4.64 -- Source: CIA 591 IV-36

At December 31, 2003, an enterprise had the following obligations that were expected to be refinanced:

17% note payable 140,00015% note payable 200,000

The 17% note payable was issued on October 1, 2002 and matures on July 1, 2004. The 15% note payable was issued onMay 1, 2002 and matures on May 1, 2004. On February 1, 2004, the entire 140,000 balance of the 17% note payable wasrefinanced by issuance of a long-term debt instrument. On February 7, 2004, the enterprise entered into a noncancelableagreement with a lender to refinance the 15% note payable on a long-term basis. The financial statements were authorizedto be issued on March 1, 2004. The total amount of obligations that may be properly excluded from current liabilities onthe enterprise’s December 31, 2003 balance sheet is

0A.140,000B.200,000C.340,000D.

Gleim's CIA Test Prep: Part III: Business Analysis and Information Technology(1165 questions)

Copyright 2004 Gleim Publications, Inc. Page 157Printed for g j

Page 158: Part three cia_with_ answers

Answer (A) is incorrect because 340,000 may be excluded from current liabilities.Answer (B) is incorrect because the 15% note is also excluded from current liabilities.Answer (C) is incorrect because the 17% note is also excluded from current liabilities.Answer (D) is correct. Some noncurrent, interest-bearing liabilities due to be settled within 12 months should neverthelesscontinue to be classified as noncurrent if the original term exceeded 12 months, the enterprise intends to refinance on along-term basis, and its intent is supported by an agreement to refinance or reschedule payments. This agreement must beconsummated before the financial statements are authorized for issue. Thus, 340,000 (140,000 + 200,000) may be excludedfrom current liabilities.

[415] Gleim #: 4.65 -- Source: Publisher

An enterprise most likely may derecognize a financial liability if it

Transfers amounts to a trust to be used to repay the obligation.A.Exchanges debt instruments with the lender with substantially similar terms.B.Exchanges debt instruments with the lender with substantially different terms.C.Transfers amounts in a transaction that meets the requirements of an in-substance defeasance.D.

Answer (A) is incorrect because payment to a third party such as a trust (also known as an in-substance defeasance)does not by itself extinguish the obligation absent a legal release.Answer (B) is incorrect because the terms should be substantially different.Answer (C) is correct. Derecognition of a financial liability (or a part thereof) occurs only by means ofextinguishment. This condition is satisfied only when the debtor pays the creditor or is legally released from primaryresponsibility either by the creditor or through the legal process. An extinguishment and derecognition of the old debtand recognition of new debt occurs when the borrower and lender exchange debt instruments with substantiallydifferent terms, that is, when the respective discounted cash flows differ by at least 10%.Answer (D) is incorrect because payment to a third party such as a trust (also known as an in-substance defeasance)does not by itself extinguish the obligation absent a legal release.

[416] Gleim #: 4.66 -- Source: CIA 1192 IV-44

An enterprise has a 100,000 liability on the books. In 12 months, 110,000 will be due, including 10% interest. Theenterprise negotiates settlement of the debt today by exchanging customer receivables with a carrying amount of 90,000.What is the journal entry today?

A. Liability 110,000Receivables 99,000Gain 11,000

B. Liability 100,000Receivables 99,000Gain 1,000

C. Liability 110,000Receivables 90,000Gain 20,000

D. Liability 100,000Receivables 90,000Gain 10,000

Answer (A) is incorrect because the liability and receivables should not be increased by the 10% interest rate.Answer (B) is incorrect because the receivables should not be increased by the 10% interest rate.Answer (C) is incorrect because the liability should not be increased by the 10% interest rate.Answer (D) is correct. An enterprise may derecognize a financial liability when it is extinguished. This condition ismet when the debtor discharges the debt by paying the creditor, such as with cash, other financial assets (e.g.,receivables), goods, or services. Consequently, the liability should be debited for its 100,000 balance. Receivableswith a 90,000 balance are given up, so that account should be credited. The difference is a gain.

[417] Gleim #: 4.67 -- Source: CIA 1196 IV-19

How will net profit or loss be affected by the amortization of a premium on bonds payable?

Gleim's CIA Test Prep: Part III: Business Analysis and Information Technology(1165 questions)

Copyright 2004 Gleim Publications, Inc. Page 158Printed for g j

Page 159: Part three cia_with_ answers

Interest expense is decreased, so net profit or loss is increased.A.Interest expense is increased, so net profit or loss is decreased.B.Interest income is increased, so net profit or loss is increased.C.Interest income is decreased, so net profit or loss is decreased.D.

Answer (A) is correct. The entry is to debit interest expense, debit bond premium, and credit cash paid. Thus, theamortization of a premium on bonds payable reduces the interest expense, thereby increasing net profit or loss.Answer (B) is incorrect because the amortization of a premium on bonds payable reduces interest expense.Answer (C) is incorrect because interest income is not affected by the amortization of a premium on bonds payable.Answer (D) is incorrect because interest income is not affected by the amortization of a premium on bonds payable.

[418] Gleim #: 4.68 -- Source: Publisher

What is the preferred method of handling unamortized discount, unamortized issue costs, and the costs of implementing aconversion of debt into common shares?

Expense them in the period bonds are converted.A.Amortize them over the remaining life of the issue retired.B.Amortize them over a period not to exceed 40 years.C.Charge them to share premium in excess of the par value of the shares issued.D.

Answer (A) is incorrect because this amount is not expensed. In effect, it reduces the amount at which the shares areissued.Answer (B) is incorrect because this amount is not expensed. In effect, it reduces the amount at which the shares areissued.Answer (C) is incorrect because this amount is not expensed. In effect, it reduces the amount at which the shares areissued.Answer (D) is correct. The conversion of debt into common shares is ordinarily based upon the carrying amount ofthe debt at the time of issuance. Because the carrying amount is based on all related accounts, the debit balances ofunamortized bond discount, unamortized issue costs, and conversion costs should be considered reductions in the netcarrying amount at the time of conversion. Consequently, these items should be reflected as reductions in the sharepremium in excess of par account.

[419] Gleim #: 4.69 -- Source: CIA 1186 IV-30

On May 1, 2004, an enterprise issued, at 103 plus accrued interest, 500 of its 12%, 1,000 bonds. The bonds are datedJanuary 1, 2004 and mature on January 1, 2009. Interest is payable semiannually on January 1 and July 1. The journal entryto record the issuance of the bonds and the receipt of the cash proceeds is

A. Cash 515,000Interest payable 20,000

Bonds payable 500,000Premium on bonds payable 35,000

B. Cash 525,000Bonds payable 500,000Premium on bonds payable 15,000Interest payable 10,000

C. Cash 535,000Bonds payable 500,000Premium on bonds payable 15,000Interest payable 20,000

D. Cash 535,000Bonds payable 500,000Premium on bonds payable 35,000

Answer (A) is incorrect because the bond premium is 15,000 (500,000 × .03), and interest payable should becredited.Answer (B) is incorrect because interest payable should be 20,000 [500,000 × .12 × (4 ÷ 12)].Answer (C) is correct. The face amount of the 500 bonds is equal to 500,000 (500 × 1,000). The cash proceedsexcluding interest from the issuance of the bonds are 515,000 (103% × 500,000). The 15,000 premium is thedifference between the cash issuance proceeds and the face amount of the bonds. Because the bonds were issuedbetween interest payment dates, the issuer is also entitled to receive accrued interest for the 4 months between theprior interest date and the issuance date. The accrued interest is 20,000 [500 bonds × 1,000 face value × 12% statedrate × (4 ÷ 12)]. The issuing company will therefore receive 535,000 in cash (515,000 + 20,000). The resultingjournal entry includes a 535,000 debit to cash, a 500,000 credit to bonds payable, a 15,000 credit to premium, and a20,000 credit to either interest payable or interest expense.

Gleim's CIA Test Prep: Part III: Business Analysis and Information Technology(1165 questions)

Copyright 2004 Gleim Publications, Inc. Page 159Printed for g j

Page 160: Part three cia_with_ answers

[420] Gleim #: 4.70 -- Source: CIA 1185 IV-15

An enterprise received a 2-year, 190,000 note on January 1, 2002 in exchange for property it sold. According to the termsof the note, interest of 5% is payable annually on January 1, 2003 and January 1, 2004, when the face amount is also due.There was no established exchange price for the property. The prevailing rate of interest for a note of this type was 12% atthe beginning of 2002 and 14% at the beginning of 2003. What interest rates should be used to calculate the amount ofinterest income from this transaction for the years ended December 31, 2002 and 2003, respectively?

0% and 5%.A.5% and 5%.B.12% and 12%.C.12% and 14%.D.

Answer (A) is incorrect because the market rate of interest at the issuance date should be used to calculate the amountof interest income.Answer (B) is incorrect because the market rate of interest at the issuance date should be used to calculate the amountof interest income.Answer (C) is correct. When the nominal interest rate on a note is not equal to the prevailing market rate for this typeof note, the face amount of the note is not equal to its fair value or present value. In this case, the present value of thenote should be determined by discounting the 190,000 maturity value and the 9,500 annual interest payments usingan appropriately imputed rate of interest. Given that 12% was the prevailing rate of interest for a note of that type atthe issuance date, 12% should be used to determine both the fair value and the interest income during the life of thenote, regardless of fluctuations in prevailing interest rates.Answer (D) is incorrect because the market rate of interest at the issuance date should be used to calculate the amountof interest income.

[421] Gleim #: 4.71 -- Source: Publisher

Debtor owes Bank on a 10-year, 15% note in the amount of 100,000, plus 30,000 accrued interest. Because of financialdifficulty, Debtor has been unable to make annual interest payments for the past 2 years, and the note is due today.Accordingly, Bank legally agreed to restructure Debtor’s debt as follows:

The 30,000 of accrued interest was forgiven.Debtor was given 3 more years to pay off the debt at 8% interest. Payments are to be made annually at year-end. Thepresent value of the payments using the prevailing rate for similar instruments of an issuer with a similar credit ratingis 84,018.

At the date of the restructuring, Debtor properly records

A loss of 30,000.A.A gain of 30,000.B.A gain of 45,982.C.No gain or loss because no extinguishment occurred.D.

Answer (A) is incorrect because 30,000 is the difference between the sum of the existing liabilities and the faceamount of the note with modified terms. Moreover, a gain should be recognized.Answer (B) is incorrect because 30,000 is the difference between the sum of the existing liabilities and the faceamount of the note with modified terms. Moreover, a gain should be recognized.Answer (C) is correct. Derecognition of a financial liability (or a part thereof) occurs only by means ofextinguishment. This condition is satisfied only when the debtor pays the creditor or is legally released from primaryresponsibility either by the creditor or through the legal process. An extinguishment and derecognition of the old debtand recognition of new debt occurs when the borrower and lender exchange debt instruments with substantiallydifferent terms, that is, when the respective discounted cash flows differ by at least 10%. A substantial modificationof terms is also accounted for as an extinguishment. The difference between the carrying amount (includingunamortized costs) of a liability (or part thereof) that has been extinguished or transferred and the amount paid isincluded in net profit or loss. This transaction qualifies as an extinguishment based on a substantial modification ofterms because the discounted cash flow from the old debt (130,000 due immediately) and the new debt (given as84,018) differ by at least 10%. Hence, the amount included by Debtor in net profit or loss at the date of therestructuring is a 45,982 gain (130,000 – 84,018), that is, the difference between the carrying amount extinguishedand the amount paid (the present value of the new debt instrument determined by discounting the cash outflows at theprevailing rate for similar instruments of an issuer with a similar credit rating). The entry is to debit the extinguishedliability for accrued interest and principal (130,000), debit discount on note payable (15,982), credit note payable(100,000), and credit gain (45,982).Answer (D) is incorrect because the terms were substantially different. Thus, an extinguishment occurred.

Gleim's CIA Test Prep: Part III: Business Analysis and Information Technology(1165 questions)

Copyright 2004 Gleim Publications, Inc. Page 160Printed for g j

Page 161: Part three cia_with_ answers

[422] Gleim #: 4.72 -- Source: CIA 597 IV-2

During the year, an enterprise’s balance sheet accounts increased by the following amounts:

Assets 180,000Liabilities 50,000Common shares 90,000Share premium 15,000

Net profit for the year was 42,000. The only other change in retained earnings was for the declaration of cash dividends.The amount of dividends declared was

2,000A.17,000B.33,000C.67,000D.

Answer (A) is incorrect because 2,000 ignores the 15,000 increase in share premium.Answer (B) is correct. Assets equals liabilities plus equity. Given an increase of 180,000 in assets, the sum ofliabilities and equity must also have increased by 180,000. Because liabilities, share capital, and share premiumincreased by 155,000 (50,000 + 90,000 + 15,000), retained earnings must have increased by 25,000 (180,000 –155,000). Given net profit of 42,000, dividends declared must have been 17,000 (42,000 – 25,000).Answer (C) is incorrect because 33,000 equals the increase in assets minus the sum of the increases in share capitalaccounts and net profit.Answer (D) is incorrect because 67,000 equals net profit plus the increase in net assets other than from ownercontributions.

[423] Gleim #: 4.73 -- Source: CIA 591 IV-37

At December 31, year 1, an enterprise had the following equity accounts:

Common shares, 10 par, 100,000 sharesauthorized, 40,000 shares issuedand outstanding 400,000

Share premium from issuanceof common shares 640,000

Retained earnings 1,000,000 Total equity 2,040,000 

Each of the 40,000 common shares outstanding was issued at a price of 26. On January 2, year 2, 2,000 shares werereacquired for 30 per share. The cost method is used in accounting for these treasury shares. Which of the followingcorrectly describes the effect of the acquisition of the treasury shares?

Common shares is reduced by 20,000.A.Share premium from issuance of common shares is reduced by 32,000.B.The retained earnings account balance is reduced by 8,000.C.Total equity is reduced by 60,000.D.

Answer (A) is incorrect because the common shares account balance is not affected when treasury shares areacquired.Answer (B) is incorrect because share premium is not affected when treasury shares are acquired and accounted forby the cost method.Answer (C) is incorrect because retained earnings is not affected by treasury share acquisitions when the cost methodis used.Answer (D) is correct. Using the cost method, the journal entry to record the acquisition of the treasury sharesincludes a debit to treasury shares for 60,000. The balance of the treasury shares account is classified as a contraequity item. Thus, the acquisition of the treasury shares reduces total equity by 60,000 (30 × 2,000 shares = 60,000).

Gleim's CIA Test Prep: Part III: Business Analysis and Information Technology(1165 questions)

Copyright 2004 Gleim Publications, Inc. Page 161Printed for g j

Page 162: Part three cia_with_ answers

[424] Gleim #: 4.74 -- Source: CIA 1188 IV-36At December 31, year 1, an enterprise has the following account balances:

Common shares (10 par, 50,000 sharesissued) 500,000

8% preferred shares (50 par, 10,000shares issued) 500,000

Share premium on common shares 640,000Share premium on preferred shares 20,000Retained earnings 600,000

The preferred shares are cumulative and nonparticipating, and has a call price of 55 per share. The journal entry to recordthe redemption of all preferred shares on January 2, year 2 pursuant to the call provision is

A. Preferred shares 500,000Share premium:  preferred 20,000Discount on preferred

shares 30,000Cash 550,000

B. Preferred shares 500,000Share premium:  preferred 20,000Loss on redemption of

preferred shares 30,000Cash 550,000

C. Preferred shares 500,000Loss on redemption of

preferred shares 50,000Retained earnings 300,000

Cash 550,000Share premium:

preferred 300,000D. Preferred shares 500,000

Share premium:  preferred 20,000Retained earnings 30,000

Cash 550,000

Answer (A) is incorrect because the 30,000 excess of cash paid over the carrying amount of the redeemed sharesshould be debited to retained earnings.Answer (B) is incorrect because the 30,000 excess of cash paid over the carrying amount of the redeemed sharesshould be debited to retained earnings.Answer (C) is incorrect because the premium on the preferred shares must be debited for only 20,000. Moreover,retained earnings must also be debited for the difference of 30,000.Answer (D) is correct. The exercise of the call provision resulted in the redemption of the 10,000 preferred sharesissued and outstanding at the call price of 550,000 (10,000 shares × 55 call price per share). To eliminate the carryingamount of the preferred shares and recognize the cash paid in this transaction, the required journal entry is to debitpreferred shares for 500,000, debit share premium:  preferred for 20,000, and credit cash for 550,000. The differenceof 30,000 (550,000 cash – 520,000 carrying amount of the preferred shares) is charged to retained earnings. No lossis reported because an Interpretation of IAS 32 does not permit the recognition of a gain or loss on transactionsinvolving an enterprise’s own shares.

[425] Gleim #: 4.75 -- Source: Publisher

Enterprise X effects self-insurance against loss from fire by appropriating an amount of retained earnings each year equal tothe amount that would otherwise be paid out as fire insurance premiums. The procedure used by Enterprise X is

Prohibited for external reporting purposes.A.Acceptable provided that fire losses are not charged against the appropriation.B.Acceptable provided that fire losses are charged against the appropriation.C.Acceptable if the amount is shown outside the equity section of the balance sheet.D.

Gleim's CIA Test Prep: Part III: Business Analysis and Information Technology(1165 questions)

Copyright 2004 Gleim Publications, Inc. Page 162Printed for g j

Page 163: Part three cia_with_ answers

Answer (A) is incorrect because an appropriation of retained earnings for self-insurance is permissible.Answer (B) is correct. Accrual of an expense prior to the occurrence of the event for which an enterprise is self-insuredshould not be permitted. This rule holds because the fair value of the property diminishes only if the event actually occurs.But an appropriation of retained earnings is acceptable to disclose the self-insurance policy if, when a fire loss occurs, theentry appropriating retained earnings is reversed, and the loss is charged against income of the period of loss and not againstretained earnings.Answer (C) is incorrect because fire losses may never be charged against the appropriation of retained earnings.Answer (D) is incorrect because the procedure is acceptable only if the appropriation is shown within the equity section ofthe balance sheet.

[426] Gleim #: 4.76 -- Source: CIA 580 IV-8

Unlike a share split, a share dividend requires a formal journal entry in the financial accounting records because share

Dividends increase the relative carrying amount of an individual’s share holding.A.Splits increase the relative carrying amount of an individual’s share holdings.B.Dividends are payable on the date they are declared.C.Dividends represent a transfer from retained earnings to share capital.D.

Answer (A) is incorrect because share dividends have no effect on total equity or on the carrying amount of anindividual shareholder’s investment.Answer (B) is incorrect because share splits have no effect on total equity or on the carrying amount of an individualshareholder’s investment.Answer (C) is incorrect because dividends, whether of shares, cash, or property, are usually payable on a datedifferent from the declaration date.Answer (D) is correct. The purpose of a share dividend is to provide evidence to the shareholders of their interest inaccumulated earnings without distribution of cash or other property. Share dividends are typically accounted for by atransfer from retained earnings at fair value.

[427] Gleim #: 4.77 -- Source: CMA 695 1-11

Enterprise UK has 6,000 shares of 5% cumulative, 100 par value preferred shares outstanding and 200,000 common sharesoutstanding. The board of directors last declared dividends for the year ended May 31, 2002, and there were no dividendsin arrears. For the year ended May 31, 2004, UK had net profit of 1,750,000. The board of directors is declaring a dividendfor common shareholders equivalent to 20% of net profit. The total amount of dividends to be paid by UK at May 31, 2004is

350,000A.380,000B.206,000C.410,000D.

Answer (A) is incorrect because 350,000 is the common shares dividend.Answer (B) is incorrect because 380,000 omits the 30,000 of cumulative dividends for 2002.Answer (C) is incorrect because 206,000 is based on a flat rate of 1 per share.Answer (D) is correct. If an enterprise has cumulative preferred shares, all preferred dividends for the current and anyunpaid prior years must be paid before any dividends can be paid on common shares. The total preferred dividendsthat must be paid equal 60,000 (2 years × 5% × 100 par × 6,000 shares), and the common dividend is 350,000(1,750,000 × 20%), for a total of 410,000.

[428] Gleim #: 4.78 -- Source: Publisher

Early in its fiscal year, Starr purchased 1,000 shares of Pack common shares for 54,000. In the same transaction, Starracquired 2,000 detachable share purchase warrants. Two of the warrants are required to purchase one additional Packcommon share. The market price without the warrants was 49 per share. The market price of the warrants was 3.50 perwarrant. Starr sold 50% of the warrants several weeks later. If the proceeds received by Starr equaled 4,000, it recognized arealized gain of

3,000A.625B.500C.0D.

Gleim's CIA Test Prep: Part III: Business Analysis and Information Technology(1165 questions)

Copyright 2004 Gleim Publications, Inc. Page 163Printed for g j

Page 164: Part three cia_with_ answers

Answer (A) is incorrect because 3,000 is the excess of the fair value of 2,000 rights over the sale price of 1,000 rights.Answer (B) is correct. The recipient of share purchase warrants should allocate the carrying amount of the shares ownedbetween those shares and the rights based on their relative fair values. Thus, the amounts to be allocated to the commonshares and warrants are 47,250 ({(49 × 1,000) ÷ [(49 × 1,000) + (3.50 × 2,000)]} × 54,000) and 6,750 (54,000 – 47,250),respectively. The realized gain is therefore 625 [4,000 – (6,750 × 50%)].Answer (C) is incorrect because 500 equals the excess of the sale price of 1,000 rights over their fair value.Answer (D) is incorrect because Starr should recognize a realized gain for the excess of the price over the carrying amount.

[Fact Pattern #25]An enterprise has issued 1,000 common shares with a par value of 10, and its credit balance in retained earnings is 5,000. Twoproposals are under consideration. The first is a share split giving each shareholder two new shares for each share formerly held.The second is to declare and distribute a 10% share dividend.

[429] Gleim #: 4.79 -- Source: CIA 1194 IV-50

(Refers to Fact Pattern #25)The share split proposal will <List A> earnings per share by <List B> than will the share dividend proposal.

  List A List B A. Increase MoreB. Increase LessC. Decrease MoreD. Decrease Less

Answer (A) is incorrect because the split results in a greater number of shares outstanding and a lower EPS thanresults from the share dividend.Answer (B) is incorrect because the split results in a greater number of shares outstanding and a lower EPS thanresults from the share dividend.Answer (C) is correct. The share split will double the number of shares outstanding to 2,000. The 10% sharedividend will increase the number of outstanding shares to 1,100. The higher number of shares in the split will resultin a lower earnings per share than will result from the share dividend.Answer (D) is incorrect because the split results in a greater number of shares outstanding and a lower EPS thanresults from the share dividend.

[430] Gleim #: 4.80 -- Source: CIA 1194 IV-51

(Refers to Fact Pattern #25)Under the share <List A>, the par value per outstanding share will <List B>.

  List A List B A. Dividend IncreaseB. Split IncreaseC. Dividend DecreaseD. Split Decrease

Answer (A) is incorrect because par value per share does not change following a share dividend.Answer (B) is incorrect because par value per share decreases following a share split.Answer (C) is incorrect because par value per share does not change following a share dividend.Answer (D) is correct. A share split results in a lower par value per share because the total number of sharesincreases but the total par value of outstanding share does not change.

[431] Gleim #: 4.81 -- Source: CIA 597 IV-55

An enterprise has excess capacity in production-related property, plant, and equipment. If in a given year these assets arebeing used to only 80% of capacity and the sales level in that year is 2 million, the full capacity sales level is

1,600,000A.2,000,000B.2,500,000C.10,000,000D.

Gleim's CIA Test Prep: Part III: Business Analysis and Information Technology(1165 questions)

Copyright 2004 Gleim Publications, Inc. Page 164Printed for g j

Page 165: Part three cia_with_ answers

Answer (A) is incorrect because 1,600,000 is 80% of the actual sales.Answer (B) is incorrect because 2,000,000 equals the actual sales.Answer (C) is correct. Full capacity sales equals actual sales divided by the percentage of capacity at which PPEs wereoperated. Thus, full capacity sales equals 2,500,000 (2,000,000 ÷ 80%).Answer (D) is incorrect because 10,000,000 equals actual sales divided by the proportion of unused capacity.

[432] Gleim #: 4.82 -- Source: CIA 1191 IV-14

Actual and projected sales of an enterprise for September and October are as follows:

  Cash Sales Credit Sales September (actual) 20,000 50,000October (projected) 30,000 55,000

All credit sales are collected in the month following the month in which the sale is made. The September 30 cash balance is23,000. Cash disbursements in October are projected to be 94,000. To maintain a minimum cash balance of 15,000 onOctober 31, the enterprise will need to borrow

0A.6,000B.11,000C.16,000D.

Answer (A) is incorrect because the enterprise will need to borrow 6,000.Answer (B) is correct. This enterprise will collect 80,000 (50,000 from September credit sales + 30,000 fromOctober cash sales) in October. To reach its targeted cash balance on October 31, it will have to borrow 6,000.

September 30 cash balance 23,000Collections 80,000Disbursements (94,000) (14,000) Ending balance 9,000Necessary borrowing 6,000   Targeted ending balance 15,000   

Answer (C) is incorrect because the enterprise will need to borrow 6,000.Answer (D) is incorrect because the enterprise will need to borrow 6,000.

[433] Gleim #: 4.83 -- Source: CIA 597 IV-34

An enterprise purchased a machine on January 1, 2000 for 1,000,000. The machine had an estimated useful life of 9 yearsand a residual value of 100,000. The company uses straight-line depreciation. On December 31, 2003, the machine wassold for 535,000. The gain or loss that should be recorded on the disposal of this machine is

35,000 gain.A.65,000 loss.B.365,000 loss.C.465,000 loss.D.

Answer (A) is incorrect because 35,000 gain equals selling price, minus carrying amount, plus residual value.Answer (B) is correct. The accumulated depreciation was 400,000 {[(1,000,000 – 100,000) ÷ 9 years] × 4 years}, sothe carrying amount was 600,000 (1,000,000 – 400,000). Thus, the loss was 65,000 (600,000 – 535,000 sales price).Answer (C) is incorrect because 365,000 loss equals cost, minus selling price, minus residual value.Answer (D) is incorrect because 465,000 loss equals cost minus selling price.

Gleim's CIA Test Prep: Part III: Business Analysis and Information Technology(1165 questions)

Copyright 2004 Gleim Publications, Inc. Page 165Printed for g j

Page 166: Part three cia_with_ answers

[434] Gleim #: 4.84 -- Source: CIA 597 IV-33An enterprise purchased a machine for 700,000. The machine was depreciated using the straight-line method and had aresidual value of 40,000. The machine was sold on December 31, 2003. The accumulated depreciation related to themachine was 495,000 on that date. The enterprise reported a gain on the sale of the machine of 75,000 in its incomestatement for the fiscal year ending December 31, 2003. The selling price of the machine was

280,000A.240,000B.205,000C.115,000D.

Answer (A) is correct. The selling price minus the carrying amount of the machine equals the gain or loss. Thecarrying amount equals cost minus accumulated depreciation, or 205,000 (700,000 – 495,000). Thus, the selling pricewas 280,000 (205,000 + 75,000 gain).Answer (B) is incorrect because 240,000 equals carrying amount, plus gain, minus residual value.Answer (C) is incorrect because 205,000 is the carrying amount.Answer (D) is incorrect because 115,000 equals the gain plus the residual value.

[435] Gleim #: 4.85 -- Source: CIA R98 IV-39

A transfer of receivables, with recourse, in exchange for cash should

Not be recorded until all of the receivables have been collected.A.Be recorded as a sale or a borrowing transaction, depending on the provisions of the transfer agreement.B.Be recorded as a sale of receivables.C.Be recorded as a borrowing transaction.D.

Answer (A) is incorrect because the transferor must recognize cash and either a sale or a borrowing.Answer (B) is correct. An enterprise derecognizes a financial asset (or a part thereof) when it loses control of thecontractual rights the asset represents. Control is lost when the enterprise realizes the rights to the benefits of thecontract, the rights expire, or the enterprise surrenders the rights. Moreover, whether loss of control has occurreddepends on the positions of both the transferor and the transferee. Control is not lost, for example, if the entity mayreacquire the asset (unless it is readily obtainable in the market or the price is fair value) or when the transferor isentitled and obligated to repurchase or redeem at a price giving the transferee a lender’s return. Control ordinarily islost when the transferee may obtain the benefits of the asset, for example, if the transferee may freely sell or pledgethe full fair value of the asset. When the conditions above are met, the transfer of receivables with recourse isaccounted for as a sale, with the proceeds of the sale reduced by the fair value of the recourse obligation (a newfinancial liability).Answer (C) is incorrect because certain conditions must be met before a sale may be recognized.Answer (D) is incorrect because a sale may be recognized if certain conditions are met.

[436] Gleim #: 4.86 -- Source: CIA 1190 IV-46

On January 1, year 1, an enterprise recorded the purchase of an asset correctly at 275,132. A down payment of 200,000 wasmade with the balance of 100,000 due in 3 years at an imputed annual interest rate of 10%. What is the year 1 interest torecord on the payable?

7,513A.8,289B.10,000C.27,513D.

Answer (A) is correct. The balance outstanding during the year was 75,132 (275,132 – 200,000). At an interest rateof 10%, the enterprise should accrue 7,513.20 (10% × 75,132) of interest for the year.Answer (B) is incorrect because 8,289 is the balance due in 3 years minus the balance outstanding during the yeardivided by 3 [(100,000 – 75,132) ÷ 3].Answer (C) is incorrect because 10,000 equals 10% of the face amount due.Answer (D) is incorrect because 27,513 equals the interest rate times the cost of the asset (275,132 × 10%).

Gleim's CIA Test Prep: Part III: Business Analysis and Information Technology(1165 questions)

Copyright 2004 Gleim Publications, Inc. Page 166Printed for g j

Page 167: Part three cia_with_ answers

[437] Gleim #: 4.87 -- Source: CIA 589 IV-35The following facts are presented for an enterprise that uses the retail inventory method:

  Cost Retail Beginning inventory 28,000 40,000Purchases 126,000 180,000Sales 170,000Ending inventory (per physical 45,000

count) at retail prices

What is the forgone sales revenue from inventory shrinkage?

3,500A.5,000B.35,000C.45,000D.

Answer (A) is incorrect because 3,500 is the inventory shrinkage at cost.Answer (B) is correct. The retail inventory method converts ending inventory at retail to cost by applying a cost-retail ratio. The advantage is that a physical inventory can be taken at retail without the necessity of countingindividual items at cost. Because this method requires detailed inventory records to be kept at retail, lost salesrevenue (at retail) caused by inventory shrinkage can be calculated using the following retail data:

Beginning inventory 40,000Purchases 180,000 Goods available for sale 220,000Sales (170,000) Estimated ending inventory 50,000Actual ending inventory (45,000) Inventory shrinkage 5,000 

Answer (C) is incorrect because 35,000 equals the estimated ending inventory at cost.Answer (D) is incorrect because 45,000 equals the actual ending inventory at retail.

[438] Gleim #: 4.88 -- Source: CIA 592 IV-30

An enterprise purchased 1,000 gross amount of inventory on account with terms of 2% discount if paid within 10 days. Theseller was responsible for delivery to the shipping point, with freight of 30 prepaid by the seller. The enterprise recordspurchases at the net amount. The journal entry to record payment 8 days after the invoice date is

A. Accounts payable 1,010Cash 1,010

B. Accounts payable 980Freight-in 30

Cash 1,010C. Purchases 1,000

Freight-in 30Accounts payable 1,030

D. Purchases 980Freight-in 30

Accounts payable 1,010

Gleim's CIA Test Prep: Part III: Business Analysis and Information Technology(1165 questions)

Copyright 2004 Gleim Publications, Inc. Page 167Printed for g j

Page 168: Part three cia_with_ answers

Answer (A) is correct. Under the net method, the payable is initially credited at the discounted amount. Because thepayment was within the discount period and freight was prepaid, the buyer’s remittance to the seller includes the freight costof 30 and the discounted price of the merchandise [1,000 × (1.0 – .02) = 980], a total of 1,010.Answer (B) is incorrect because freight-in was debited at the invoice date (debit freight-in and purchases, credit accountspayable). Accounts payable needs to be debited for the entire amount (including freight of 30) owed to the seller.Answer (C) is incorrect because this entry would have been made at the invoice date if the gross method had been used.Answer (D) is incorrect because this entry was made at the invoice date.

[Fact Pattern #26]An enterprise with a December 31 year-end purchased 2,000 of inventory on account. The seller was responsible for delivery tothe shipping point, with freight of 50 paid at destination by the buyer. The invoice date was December 27, 2003, and the goodsarrived on January 3, 2004.

[439] Gleim #: 4.89 -- Source: CIA 592 IV-33

(Refers to Fact Pattern #26)Now assume the terms required the seller to deliver to the destination instead of the shipping point. What is the correctamount of inventory and freight-in relating to this purchase on the 2003 financial statements?

  Inventory Freight-In A. 0 0B. 2,050 0C. 0 50D. 2,000 50

Answer (A) is correct. Title and risk of loss passed to the buyer at the destination, and the seller incurred the expenseof delivery to that point. The goods did not arrive until after year-end, so they should not be included in 2003inventory. Freight-in should also not be recorded until 2004.Answer (B) is incorrect because no inventory should be included in the 2003 financial statements.Answer (C) is incorrect because freight-in should be recorded in 2004.Answer (D) is incorrect because no inventory should be included in the 2003 financial statements, and freight-inshould be recorded in 2004.

[440] Gleim #: 4.90 -- Source: CIA 594 IV-18

A merchandising company had the following inventory related transactions in its first year of operations:

  Purchases Sales in BalanceDate in Units Units in Units

Jan. 1 10,000 @ 5 10,000March 1 6,000 @ 6 16,000May 1 3,000 13,000July 1 8,000 @ 6.25 21,000Sept. 1 12,000 9,000Nov. 1 5,000 @ 7 14,000Dec. 1 2,000 12,000

If the company uses the first-in-first-out (FIFO) method of inventory valuation, its ending inventory balance (rounded) willbe

62,000A.70,759B.78,750C.84,000D.

Gleim's CIA Test Prep: Part III: Business Analysis and Information Technology(1165 questions)

Copyright 2004 Gleim Publications, Inc. Page 168Printed for g j

Page 169: Part three cia_with_ answers

Answer (A) is incorrect because 62,000 is the ending inventory balance under LIFO.Answer (B) is incorrect because 70,759 is the ending inventory balance under the weighted average method.Answer (C) is correct. The first-in-first-out (FIFO) method assumes that the oldest units are used first and the newest unitsremain in inventory. Because the company has 12,000 units remaining, ending inventory equals 78,750 [(5,000 × 7) +(7,000 × 6.25)].Answer (D) is incorrect because 84,000 computes all 12,000 units of ending inventory at the most recent purchase price of7.

[441] Gleim #: 4.91 -- Source: CIA 597 IV-13

An enterprise has appropriately used the installment method of accounting since it began operations at the beginning of thecurrent year. The following information pertains to its operations for this year:

Installment sales 1,200,000Cost of installment sales 840,000Collections on installment sales 480,000General and administrative expenses 120,000

The amount of gross profit deferred at the end of the current year should be

720,000A.288,000B.216,000C.144,000D.

Answer (A) is incorrect because 720,000 is the year-end balance of installment accounts receivable.Answer (B) is incorrect because 288,000 applies the gross profit percentage to the sum of cost of sales and generaland administrative expenses.Answer (C) is correct. The gross profit on installment sales is 360,000 (1,200,000 installment sales – 840,000 cost ofinstallment sales). Accordingly, the gross profit percentage is 30% (360,000 gross profit ÷ 1,200,000 installmentsales), and the amount of gross profit deferred at the end of the current year is 216,000 [30% × (1,200,000 installmentsales – 480,000 collections)]. General and administrative expenses have no effect on the computation of realizedgross profit or deferred gross profit. They are to be classified as operating expenses on the income statement in theperiod in which they are incurred.Answer (D) is incorrect because 144,000 treats general and administrative expenses as costs of installment sales.

[442] Gleim #: 4.92 -- Source: CIA 594 IV-17

The cost of materials has risen steadily over the year. The enterprise uses its newest materials first when removing itemsfrom inventory. Which of the following methods of estimating the ending balance of the materials inventory account willresult in the highest net profit, all other variables held constant?

Last-in-first-out (LIFO).A.First-in-first-out (FIFO).B.Weighted average.C.Specific identification.D.

Answer (A) is incorrect because the last-in-first-out (LIFO) method assumes that the most recent and costliest unitshave been removed from inventory. If costs rise steadily during the accounting period, this method will result in thelowest inventory balance, the highest cost of goods sold, and the lowest net profit.Answer (B) is correct. The first-in-first-out (FIFO) method assumes that the oldest and hence least costly units areused first, and the newest and most costly items remain in inventory. This method will result in the highest inventorybalance if costs rise steadily during the accounting period. Accordingly, FIFO results in the lowest cost of goods soldand the highest net profit.Answer (C) is incorrect because the weighted-average cost method averages the costs of all inventory items andresults in a lower inventory balance and net profit than does the FIFO method.Answer (D) is incorrect because specific identification charges the actual cost of each unit to cost of goods sold eachperiod, leaving as inventory the actual cost of all items still in inventory. Given that the newest and most costly itemsare removed from inventory first, the inventory balance and net profit will be lower than that obtained using FIFOestimation.

Gleim's CIA Test Prep: Part III: Business Analysis and Information Technology(1165 questions)

Copyright 2004 Gleim Publications, Inc. Page 169Printed for g j

Page 170: Part three cia_with_ answers

[443] Gleim #: 4.93 -- Source: CIA 593 IV-35An enterprise uses the retail method of inventory estimation for interim reporting purposes. Management expects somenormal shrinkage in the inventory because of theft. What effect will the failure to consider this shrinkage have on thecomputation of (1) the cost-retail ratio, and (2) the estimated ending inventory at retail?

  (1) (2)  Effect on Cost- Effect on Estimated  Retail Ratio Ending Inventory at Retail A. No effect No effectB. No effect OverstatementC. Overstatement OverstatementD. Overstatement Understatement

Answer (A) is incorrect because shrinkage has no effect on the ratio but should be deducted in arriving at theestimated ending inventory at retail.Answer (B) is correct. The retail method of inventory estimation applies a cost-retail ratio to the ending inventory atretail to determine ending inventory at cost. The ratio equals goods available at cost divided by goods available atretail. Normal inventory shrinkage is deducted from the retail amount of goods available because the goods are notavailable. However, abnormal amounts of theft, etc., are deducted in arriving at both the cost and retail amounts. Thereason for the difference in treatment is that normal but not abnormal inventory losses are anticipated and included inselling price (retail value). Accordingly, failure to account for normal inventory shrinkage has no effect on thecalculation of the cost-retail ratio but overstates ending inventory at retail.Answer (C) is incorrect because shrinkage has no effect on the ratio but should be deducted in arriving at theestimated ending inventory at retail.Answer (D) is incorrect because shrinkage has no effect on the ratio but should be deducted in arriving at theestimated ending inventory at retail.

[444] Gleim #: 4.94 -- Source: CIA 1193 IV-29

An entity purchases office equipment for 525,000 on account. Select the appropriate journal entry to record thistransaction.

A. Office expense 525,000Accounts payable 525,000

B. Office equipment 525,000Accounts payable 525,000

C. Accounts payable 525,000Office expense 525,000

D. Accounts payable 525,000Office equipment 525,000

Answer (A) is incorrect because the charge should be to an asset account rather than an expense account.Answer (B) is correct. The purchase of office equipment represents the acquisition of an asset. An increase in anasset is recorded by a debit. The purchase on account increases liabilities. An increase in a liability is recorded by acredit.Answer (C) is incorrect because an increase in accounts payable is recorded by a credit. The purchase of equipmentresults in an asset that is recorded by a debit to an asset account.Answer (D) is incorrect because an increase in a liability is recorded by a credit. An increase in an asset is recordedby a debit.

[445] Gleim #: 4.95 -- Source: CIA 592 IV-27

An enterprise acquired an item of property, plant, and equipment with an estimated useful life of 5 years for 15,000 at thebeginning of year 1. For financial statement purposes, how would the depreciation expense calculated using the 200%diminishing-balance method compare with that calculated using the sum-of-the-years’-digits method in year 1 and year 2,respectively?

  Year 1 Year 2 A. Lower LowerB. Lower HigherC. Higher LowerD. Higher Higher

Gleim's CIA Test Prep: Part III: Business Analysis and Information Technology(1165 questions)

Copyright 2004 Gleim Publications, Inc. Page 170Printed for g j

Page 171: Part three cia_with_ answers

Answer (A) is incorrect because 200% diminishing-balance depreciation is higher in year 1.Answer (B) is incorrect because 200% diminishing-balance depreciation is higher in year 1 and lower in year 2.Answer (C) is correct. 200% diminishing-balance is an accelerated depreciation method that determines periodicdepreciation expense by multiplying the carrying amount at the beginning of each period by a constant rate that is equal totwice the straight-line rate of depreciation. Each year the carrying amount of the asset decreases by the depreciation expenserecognized. Residual value is ignored in determining the carrying amount except as a floor beneath which the asset may notbe depreciated. SYD depreciation multiplies a constant depreciable amount (cost - residual value) by the SYD fraction. TheSYD fraction’s numerator is the number of years of remaining useful life (n). The formula to compute the denominator inthe SYD method is (n(n+1)) ÷ 2. For a 5-year estimated useful life, the denominator of the fraction is 15 [5(5 + 1) ÷ 2].

200% DB: Year 1 = 15,000(.4) = 6,000  Year 2 = 9,000(.4) = 3,600

SYD: Year 1 = 15,000(5 ÷ 15) = 5,000  Year 2 = 15,000(4 ÷ 15) = 4,000

Answer (D) is incorrect because 200% diminishing-balance depreciation is lower in year 2.

[446] Gleim #: 4.96 -- Source: CIA 1196 IV-36

Which of the following is a noncash item?

Administrative expense.A.Interest expense.B.Income tax expense.C.Depreciation expense.D.

Answer (A) is incorrect because administrative expense is a cash charge.Answer (B) is incorrect because interest expense is a cash charge.Answer (C) is incorrect because income tax expense is a cash charge.Answer (D) is correct. Depreciation expense is a noncash charge. It represents the allocation of the historical cost ofassets to the time periods in which they are used to generate revenues.

[447] Gleim #: 4.97 -- Source: CIA 590 IV-45

An enterprise purchased the following for 120,000:

  Seller’s Estimated Fair  Carrying Amount Value Land 50,000 60,000Building 100,000 90,000

The land should be recorded at

40,000A.48,000B.50,000C.60,000D.

Answer (A) is incorrect because 40,000 assumes the purchase price allocation is based on carrying amounts insteadof fair values.Answer (B) is correct. An item of PPE is initially measured at its cost. Accordingly, the cost, which equals the120,000 purchase price, should be allocated between the land and the building based on their respective fair values.The cost of the land should be recorded at 48,000 {120,000 × [60,000 ÷ (60,000 + 90,000)]}.Answer (C) is incorrect because 50,000 is the seller’s carrying amount of the land.Answer (D) is incorrect because 60,000 is the estimated fair value of the land.

[448] Gleim #: 4.98 -- Source: Publisher

Whenever an enterprise must use net realizable value rather than cost to value an inventory item, the inventory account isreduced and the account “expense due to decline of inventory to net realizable value” is increased. The balance of thisaccount is reflected as a separate item on the

Gleim's CIA Test Prep: Part III: Business Analysis and Information Technology(1165 questions)

Copyright 2004 Gleim Publications, Inc. Page 171Printed for g j

Page 172: Part three cia_with_ answers

Balance sheet as a deduction from inventory.A.Income statement as an extraordinary loss.B.Income statement as a deduction from gross profit on sales.C.Income statement as an operating expense.D.

Answer (A) is incorrect because the write-down is expensed in the period in which the writedown occurs.Answer (B) is incorrect because the loss is ordinary.Answer (C) is correct. If a separate expense account is not used, the ending inventory will be reduced directly and the resultwill be an increase in cost of goods sold. No separate disclosure of the inventory writedown will appear in the incomestatement. The effect is to hide the loss in cost of goods sold. If the separate expense account is used, it appears on theincome statement as a deduction from gross profit (sales – cost of goods sold). One advantage is that cost of goods sold isnot misstated.Answer (D) is incorrect because the writedown is not treated as a selling or administrative expense.

[449] Gleim #: 4.99 -- Source: CMA 1292 2-5

Aston Company acquired a new machine at a cost of 200,000 and incurred costs of 2,000 to have the machine shipped toits factory. Aston also paid 4,500 to construct and prepare a site for the new machine and 3,500 to install the necessaryelectrical connections. Aston estimates that the useful life of this new machine will be 5 years and that it will have aresidual value of 15,000 at the end of that period. Assuming that Aston acquired the machine on January 1 and will take afull year’s depreciation, the proper amount of depreciation expense to be recorded by Aston if it uses the 200%diminishing-balance method is

74,000A.84,000B.80,000C.80,800D.

Answer (A) is incorrect because 74,000 assumes that the depreciable amount is the invoice price minus residualvalue.Answer (B) is correct. Items of property, plant, and equipment (PPE) that meet the recognition criteria are initiallymeasured at cost. The cost includes the purchase price (minus trade discounts and rebates, plus purchase taxes) andthe directly attributable costs of bringing the assets to working condition for their intended use. Hence, thedepreciable amount is 210,000 (200,000 invoice price + 2,000 delivery expense + 4,500 site preparation + 3,500electrical work). Under the 200% DB method, residual value is ignored at the beginning. Thus, the full 210,000 willbe subject to depreciation. Given a 5-year life, the annual straight-line rate is 20%, and the 200% DB rate will be40%. Depreciation for the first year is therefore 84,000 (40% × 210,000).Answer (C) is incorrect because the depreciable amount of the machine was 210,000, not the 200,000 invoice price.Answer (D) is incorrect because 80,800 assumes a depreciable amount of 202,000, but the site preparation andelectrical costs are part of that cost.

[450] Gleim #: 4.100 -- Source: CMA 683 1-13

An enterprise often factors its accounts receivable. The finance company requires an 8% reserve and charges a 1.5%commission on the amount of the receivable. The remaining amount to be advanced is further reduced by an annual interestcharge of 16%. What proceeds (rounded to the nearest dollar) will the enterprise receive from the finance company at thetime a 110,000 account that is due in 60 days is turned over to the finance company?

81,950A.83,630B.96,895C.99,550D.

Answer (A) is incorrect because the proceeds would be determined by reducing the 110,000 by 9.5% (8% reserve +1.5% commission) and then reducing that amount by the interest expense [16% × (60 ÷ 360)].Answer (B) is incorrect because the proceeds would be determined by reducing the 110,000 by 9.5% (8% reserve +1.5% commission) and then reducing that amount by the interest expense [16% × (60 ÷ 360)].Answer (C) is correct. The factor will hold out 8,800 (8% × 110,000) as a reserve against returns and allowances and1,650 (1.5% × 110,000) as a commission. That leaves 99,550 to be advanced to the seller. However, interest at therate of 16% annually is also to be withheld. For 60 days that interest would amount to approximately 2,655 (assuminga 360-day year). The proceeds to be given to the seller equal 96,895 (99,550 – 2,655).Answer (D) is incorrect because the proceeds would be determined by reducing the 110,000 by 9.5% (8% reserve +1.5% commission) and then reducing that amount by the interest expense [16% × (60 ÷ 360)].

Gleim's CIA Test Prep: Part III: Business Analysis and Information Technology(1165 questions)

Copyright 2004 Gleim Publications, Inc. Page 172Printed for g j

Page 173: Part three cia_with_ answers

[451] Gleim #: 4.101 -- Source: Publisher

Under IAS 2, Inventories, all of the following should be disclosed when reporting inventories except

The use of the lower-of-cost-or-net-realizable-value method, if applicable.A.The cost formulas used.B.The carrying amount of inventories in classifications appropriate to the enterprise.C.An estimated amount of obsolete inventory included in the total inventory valuation.D.

Answer (A) is incorrect because disclosures should include the accounting policies applied, such as the lower-of-cost-or-NRV method.Answer (B) is incorrect because disclosures should include the cost formulas used.Answer (C) is incorrect because disclosures should include the amounts for classifications of inventory appropriate tothe enterprise.Answer (D) is correct. According to IAS 2, Inventories, disclosures about inventories should include the accountingpolicies applied in measuring inventories, including the cost formulas used; total carrying amount; carrying amountfor each classification appropriate to the enterprise; carrying amount of items reported at NRV; amount of anyreversal of writedowns recognized as income; reasons for such a reversal; and carrying amount of inventory pledgedas security. Thus, the disclosures under IAS 2 include the carrying amount of inventories carried at NRV, not theamount of obsolete inventory.

[452] Gleim #: 4.102 -- Source: Publisher

At the balance sheet date, receivables originated by the enterprise are valued at the

Current market value.A.Estimated recoverable amount.B.Original invoice amount when the asset was acquired.C.Amount receivable when due.D.

Answer (A) is incorrect because current market value is not used for receivables. It is difficult to determine and oftenis not relevant given that an enterprise has no immediate plans to market its receivables.Answer (B) is correct. According to IAS 39, financial assets include, among other things, receivables originated bythe enterprise, that is, created by providing money, goods, or services directly to a debtor. This classification ofreceivables excludes those originated with the intention of immediate or near-term sale. Receivables originated by theenterprise that are of short duration and have no stated interest rate (e.g., trade receivables) are usually measured atthe original invoice amount unless the effect of interest imputation would be significant. Furthermore, all financialassets are subject to review for impairment. This assessment should be made at the balance sheet date. If it is probablethat all amounts due cannot be collected, the recoverable amount should be estimated, and the impairment or bad debtloss should be included in the periodic net profit or loss. The loss is measured by the difference between the carryingamount and the present value of discounted future cash flows, but the cash flows of short-term receivables ordinarilyare not discounted. The carrying amount of the asset is reduced directly or by use of an allowance account to theestimated recoverable amount (realizable value).Answer (C) is incorrect because the original invoice amount of receivables must be adjusted for possible impairment.Answer (D) is incorrect because the amount receivable, in total, must be reduced either directly or by an allowancefor uncollectible accounts to reflect the estimated recoverable amount.

[453] Gleim #: 4.103 -- Source: CIA 1194 IV-18

On January 1, an enterprise has no opening inventory balance. The following purchases are made during the year:

Date Units Purchased Unit CostJanuary 1 5,000 10.00April 1 5,000 9.00July 1 5,000 8.00October 1 5,000 7.50     There are 10,000 units in inventory on December 31.

Cost of goods sold for the year, if the enterprise uses the last-in, first-out (LIFO) method of inventory valuation, will be

Gleim's CIA Test Prep: Part III: Business Analysis and Information Technology(1165 questions)

Copyright 2004 Gleim Publications, Inc. Page 173Printed for g j

Page 174: Part three cia_with_ answers

77,500A.86,250B.87,500C.95,000D.

Answer (A) is correct. LIFO assumes that the units sold during the year are those most recently purchased. Thus, the cost ofgoods sold for the year is 77,500 [(5,000 × 7.50) + (5,000 × 8)].Answer (B) is incorrect because 86,250 is based on the average cost method of calculating cost of goods sold.Answer (C) is incorrect because 87,500 is the cost of goods sold under the specific identification method if endinginventory was purchased on April 1 and July 1.Answer (D) is incorrect because 95,000 is the cost of goods sold using the FIFO.

[454] Gleim #: 4.104 -- Source: CIA 1194 IV-19

A new machine has an initial cost of 300,000, an estimated useful life of 2,000 hours of use over a 3-year period, and anestimated residual value of 70,000. Usage rates are estimated as 500 hours in the first year, 700 hours in the second year,and 800 hours in the third year. Depreciation expense in year two under the sum-of-the-units method of depreciation willbe

57,500A.75,000B.80,500C.105,000D.

Answer (A) is incorrect because 57,500 is the depreciation expense for year one.Answer (B) is incorrect because 75,000 is the expense for year one calculated without subtracting the residual value.Answer (C) is correct. Depreciation expense equals cost minus residual value, times the quotient of estimated hoursof use in year two divided by the total estimated hours of use. Thus, depreciation expense is 80,500 [(300,000 –70,000) × (700 hours ÷ 2,000 hours)].Answer (D) is incorrect because 105,000 results from not subtracting the residual value.

[455] Gleim #: 4.105 -- Source: CIA 595 IV-9

What is the journal entry recorded upon the sale of an item of property, plant, and equipment (PPE) that was sold for cashin excess of its carrying amount?

A. No journal entry is required.B. Debit cash

Debit accumulated depreciationDebit income on disposal of PPE

Credit PPEC. Debit cash

Debit PPECredit accumulated depreciationCredit income on disposal of PPE

D. Debit cashDebit accumulated depreciation

Credit PPECredit income on disposal of PPE

Answer (A) is incorrect because depreciation must be taken up to the date of disposition and all amounts relating tothe retired asset should be eliminated.Answer (B) is incorrect because the gain should be recorded as a credit.Answer (C) is incorrect because the PPE account should be credited, and accumulated depreciation should bedebited.Answer (D) is correct. The journal entry to record the sale of an item of PPE for cash in excess of its carrying amountshould debit the cash account to record the sale proceeds received. Accumulated depreciation should be eliminated bydebiting an amount equal to depreciation accumulated up to the start of the current accounting period plus anydepreciation that has accumulated between the start of the current period and the date of disposal. Finally, the PPEaccount should be credited to eliminate the original cost of the asset. The gain should be recorded as a credit andrecognized as income on the income statement.

Gleim's CIA Test Prep: Part III: Business Analysis and Information Technology(1165 questions)

Copyright 2004 Gleim Publications, Inc. Page 174Printed for g j

Page 175: Part three cia_with_ answers

[456] Gleim #: 4.106 -- Source: CIA 595 IV-13An enterprise had 1,000 units of opening inventory that cost 10 per unit. On May 1, 1,000 units were purchased at a cost of11 each, and on September 1 another 1,000 units were purchased at a cost of 12 each. If 2,000 units were sold during theyear, the company will report cost of goods sold of <List A> if the <List B> method of inventory valuation is used.

  List A List B A. 22,000 LIFOB. 23,000 Weighted averageC. 21,000 FIFOD. 22,000 FIFO

Answer (A) is incorrect because cost of goods sold is 22,000 under the weighted-average method. Under LIFO, costof goods sold is 23,000 (12,000 + 11,000). The 2,000 most recently purchased units are presumed to have been sold.Answer (B) is incorrect because the weighted-average unit cost of all items available for sale is 11 [(10,000 + 11,000+ 12,000) ÷ 3,000]. Given that 2,000 units were sold, cost of goods sold is 22,000 (2,000 × 11) under this method.Answer (C) is correct. Under FIFO, the first items purchased are presumed to be the first sold. Given that 3,000 unitswere available and 2,000 units were sold, FIFO cost of goods sold must have been 21,000 [(1,000 × 10) BI + (1,000× 11) May 1 purchase].Answer (D) is incorrect because FIFO cost of goods sold is 21,000. Under the weighted-average method, cost ofgoods sold is 22,000.

[457] Gleim #: 4.107 -- Source: CIA 595 IV-17

An enterprise sold a depreciable asset in the middle of the fifth year of its estimated 10-year useful life. The original cost ofthe asset was 100,000, and it was being depreciated on the straight-line basis. If the asset was sold for 80,000, the gain onthe sale will be

20,000A.25,000B.30,000C.35,000D.

Answer (A) is incorrect because 20,000 omits depreciation for the first half of the fifth year.Answer (B) is correct. The gain on the sale is the difference between the sale proceeds and the undepreciated cost ofthe asset. Depreciation must be taken up to the time of sale. Assuming that residual value is 0, annual depreciation is10,000 (100,000 ÷ 10 years). Thus, the gain is 25,000 {80,000 – [100,000 – (4.5 years × 10,000)]}.Answer (C) is incorrect because 30,000 results if 5 full years of depreciation are taken.Answer (D) is incorrect because 35,000 results from subtracting accumulated depreciation from the sale proceeds.

[458] Gleim #: 4.108 -- Source: CIA 595 IV-22

Which of the following changes in accounting policies resulting from a significant change in the expected pattern ofeconomic benefit will increase net profit?

A change from FIFO to LIFO inventory valuation when costs are rising.A.A change from FIFO to weighted-average inventory valuation when costs are falling.B.A change from accelerated to straight-line depreciation in the later years of the depreciable lives of the assets.C.A change from straight-line to accelerated depreciation in the early years of the depreciable lives of the assets.D.

Gleim's CIA Test Prep: Part III: Business Analysis and Information Technology(1165 questions)

Copyright 2004 Gleim Publications, Inc. Page 175Printed for g j

Page 176: Part three cia_with_ answers

Answer (A) is incorrect because, in a period of rising costs, LIFO results in the highest cost of goods sold and the lowestreported net profit.Answer (B) is correct. According to IAS 16, Property, Plant, and Equipment, a change in depreciation method resultingfrom a significant change in the expected pattern of economic benefits is accounted for as a change in estimate, that is,prospectively. In a period of falling costs, FIFO results in higher cost of goods sold than the weighted-average method.FIFO includes the higher, earlier costs in cost of goods sold, whereas the weighted-average method averages the later, lowercosts with the higher, earlier costs. Thus, a change from FIFO to weighted-average costing reduces cost of goods sold andincreases reported net profit.Answer (C) is incorrect because, in the later years of the depreciable life of an asset, accelerated depreciation results inlower depreciation expense than does the straight-line method. A change to straight line increases depreciation expense andreduces reported net profit.Answer (D) is incorrect because, in the early years of the depreciable life of an asset, straight-line depreciation results inlower depreciation expense than accelerated depreciation. A change to accelerated depreciation increases depreciationexpense and reduces reported net profit.

[459] Gleim #: 4.109 -- Source: CIA 595 IV-24

Enterprise A acquires Enterprise B for 1,000,000. At the time of the acquisition, Enterprise B’s identifiable net assets had acarrying amount of 900,000 and a fair value of 800,000. The amount of goodwill Enterprise A will record on theacquisition date is

0A.100,000B.200,000C.300,000D.

Answer (A) is incorrect because goodwill equal to the excess of the acquisition cost over the fair value of theidentifiable assets and liabilities acquired must be recorded.Answer (B) is incorrect because 100,000 is the excess of the acquisition cost over the carrying amount of theidentifiable assets and liabilities acquired.Answer (C) is correct. Goodwill equals the excess of the acquisition cost over the acquirer’s interest in the fair valueof the identifiable assets and liabilities acquired. Consequently, goodwill is 200,000 (1,000,000 – 800,000).Answer (D) is incorrect because 300,000 equals goodwill plus the excess of the carrying amount over fair value.

[Fact Pattern #27]An enterprise had the following account balances in the pre-closing trial balance:

Opening inventory 100,000Closing inventory 150,000Purchases 400,000Transportation-in 6,000Purchase discounts 40,000Purchase allowances 15,000Returned purchases 5,000

[460] Gleim #: 4.110 -- Source: CIA 1195 IV-5

(Refers to Fact Pattern #27)The enterprise had net purchases for the period of

340,000A.346,000B.370,000C.376,000D.

Gleim's CIA Test Prep: Part III: Business Analysis and Information Technology(1165 questions)

Copyright 2004 Gleim Publications, Inc. Page 176Printed for g j

Page 177: Part three cia_with_ answers

Answer (A) is incorrect because 340,000 omits transportation-in from the calculation.Answer (B) is correct. Purchase discounts, allowances, and returns are subtractions from purchases because they arereductions of cost. Transportation-in is an addition because it increases cost. Thus, net purchases equals 346,000 (400,000 +6,000 – 40,000 – 15,000 – 5,000).Answer (C) is incorrect because 370,000 omits transportation-in and adds, rather than subtracts, purchase allowances.Answer (D) is incorrect because 376,000 adds, rather than subtracts, purchase allowances.

[Fact Pattern #28]An enterprise has 8,000 units in inventory on January 1, valued at 10 per unit. During the year, the enterprise sold 25,000 unitsand purchased inventory as follows:

  QuantityDate Purchased Unit PriceApril 1 15,000 units 8July 1 10,000 units 9October 1 12,500 units 10

[461] Gleim #: 4.111 -- Source: CIA 1195 IV-18

(Refers to Fact Pattern #28)If the enterprise uses the weighted-average method of inventory valuation, cost of goods sold for the period will be

186,978A.197,000B.228,023C.235,000D.

Answer (A) is incorrect because 186,978 is the value of ending inventory under the weighted-average method.Answer (B) is incorrect because 197,000 is the ending inventory under the FIFO method.Answer (C) is correct. Under the weighted-average method, the weighted-average cost per unit is multiplied by thenumber of units sold to determine the cost of goods sold for the period. The total units available for sale equaled45,500 (8,000 + 15,000 + 10,000 + 12,500). The total cost of all units available for sale was 415,000 [(8,000 × 10) +(15,000 × 8) + (10,000 × 9) + (12,500 × 10)]. Thus, the weighted-average cost per unit of inventory was 9.1209(415,000 ÷ 45,500), and cost of goods sold was 228,023 (25,000 × 9.1209).Answer (D) is incorrect because 235,000 is the cost of goods sold under the LIFO method.

[462] Gleim #: 4.112 -- Source: CIA 1196 IV-8

The following information is available for an enterprise for the quarter ended March 31, of the current year:

Merchandise inventory, as ofJanuary 1 of the current year 30,000

Sales 200,000Purchases 190,000

The gross profit margin is normally 20% of sales. What is the estimated cost of the merchandise inventory at March 31, ofthe current year?

20,000A.40,000B.60,000C.180,000D.

Gleim's CIA Test Prep: Part III: Business Analysis and Information Technology(1165 questions)

Copyright 2004 Gleim Publications, Inc. Page 177Printed for g j

Page 178: Part three cia_with_ answers

Answer (A) is incorrect because 20,000 is the difference between sales for the period and cost of goods available for sale.Answer (B) is incorrect because 40,000 is the amount of gross profit.Answer (C) is correct. The gross profit margin percentage is given as 20% of sales. Hence, cost of goods sold must havebeen 160,000 [200,000 sales × (1.0 – .2)]. The estimated cost of the inventory at March 31, 2004 is computed as follows:

Inventory, 1/1 30,000Purchases 190,000Cost of goods available for sale, 3/31 220,000Estimated cost of goods sold (160,000)Estimated inventory at 3/31 60,000

Answer (D) is incorrect because 180,000 results from subtracting the gross profit from cost of goods available for sale.

[463] Gleim #: 4.113 -- Source: CIA 596 IV-27

On January 1 of year 1, an enterprise purchased a piece of equipment for 250,000 that was originally estimated to have auseful life of 10 years with no residual value. Depreciation has been recorded for 3 years on a straight-line basis.

On January 1 of year 4, the estimated useful life was revised so that the equipment is considered to have a total life of 20years. Assume that the depreciation method and the useful life for financial reporting and tax purposes are the same. Thedepreciation expense in year 4 on this equipment would be

8,750A.10,294B.12,500C.14,706D.

Answer (A) is incorrect because 8,750 is the result of depreciating the remaining carrying amount over 20 yearsrather than the remaining 17 years.Answer (B) is correct. In year 4, the carrying amount at the start of the period will be amortized over the revisedestimated years of useful life. The depreciation recognized during years 1 through 3 was 75,000 [3 years × (250,000 ÷10)]. Thus, the carrying amount at the beginning of year 4 was 175,000, and year four depreciation based on therevised estimated useful life is 10,294 [175,000 ÷ (20 – 3)].Answer (C) is incorrect because 12,500 results from accounting for the change in estimate retroactively.Answer (D) is incorrect because 14,706 results from depreciating the original carrying amount over the revisedestimate of remaining useful life.

[Fact Pattern #29]An enterprise had the following opening and closing inventory balances during the current year:

  1/1 12/31 Finished goods 90,000 260,000Raw materials 105,000 130,000Work-in-progress 220,000 175,000

The following transactions and events occurred duringthe current year:

300,000 of raw materials were purchased, of which20,000 were returned because of defects.600,000 of direct labor costs were incurred.750,000 of production overhead costs wereincurred.

 

[464] Gleim #: 4.114 -- Source: CIA 596 IV-14

(Refers to Fact Pattern #29)The cost of goods sold for the current year ended December 31 would be

Gleim's CIA Test Prep: Part III: Business Analysis and Information Technology(1165 questions)

Copyright 2004 Gleim Publications, Inc. Page 178Printed for g j

Page 179: Part three cia_with_ answers

1,480,000A.1,500,000B.1,610,000C.1,650,000D.

Answer (A) is correct. Cost of goods sold equals cost of goods produced (CGP) adjusted for the change in finished goods.CGP equals the sum of raw materials used, direct labor costs, and production overhead, adjusted for the change in work-in-progress. Raw materials used equals 255,000 (105,000 BI + 300,000 purchases – 20,000 returns – 130,000 EI). Thus, CGPequals 1,650,000 (255,000 RM + 600,000 DL + 750,000 OH + 220,000 BWIP – 175,000 EWIP), and CGS equals1,480,000 (1,650,000 CGP + 90,000 BFG – 260,000 EFG).Answer (B) is incorrect because 1,500,000 results from not subtracting purchase returns when calculating raw materialsused.Answer (C) is incorrect because 1,610,000 fails to deduct the ending balance of raw materials when calculating rawmaterials used.Answer (D) is incorrect because 1,650,000 is the cost of goods produced.

[465] Gleim #: 4.115 -- Source: CIA 596 IV-15

(Refers to Fact Pattern #29)Without prejudice to your answer to any other question, assume that cost of goods sold for the current year endedDecember 31 is 2,000,000. Inventory turnover on total inventory for the enterprise would be

2.04 times.A.3.54 times.B.4.08 times.C.4.82 times.D.

Answer (A) is incorrect because 2.04 times results from adding all inventory balances, opening and closing, to obtainthe denominator of the turnover ratio.Answer (B) is incorrect because 3.54 times results from using the year-end inventory balances only.Answer (C) is correct. Inventory turnover is the ratio of cost of goods sold to the average inventory balance. Thetotal average inventory is 490,000 [(90,000 BFG + 105,000 BRM + 220,000 BWIP + 260,000 EFG + 130,000 ERM+ 175,000 EWIP) ÷ 2]. Hence, total inventory turnover is 4.08 times (2,000,000 assumed CGS ÷ 490,000 averagetotal inventory).Answer (D) is incorrect because 4.82 times results from using the opening inventory balances only.

[466] Gleim #: 4.116 -- Source: CIA 596 IV-16

(Refers to Fact Pattern #29)If the enterprise’s raw materials inventory as of December 31 of the current year (ending inventory) was miscounted andthe true figure was higher than 130,000, one effect on the year-end financial statements would be that

Net profit is overstated.A.Cost of goods sold is overstated.B.Working capital is overstated.C.Cost of goods produced is understated.D.

Answer (A) is incorrect because, if the ending inventory of raw materials is understated, cost of goods sold isoverstated, and net income is understated.Answer (B) is correct. If the ending inventory of raw materials is understated, raw materials used is overstated, costof goods produced is overstated, and cost of goods sold is overstated.Answer (C) is incorrect because working capital is higher when the balances of current assets are higher. If the rawmaterials inventory balance is understated, working capital will also be understated.Answer (D) is incorrect because, if the ending inventory of raw materials is understated, raw materials used isoverstated, and cost of goods produced is overstated.

[467] Gleim #: 4.117 -- Source: CIA 590 IV-51

With regard to inventory management, an increase in the frequency of ordering will normally

Gleim's CIA Test Prep: Part III: Business Analysis and Information Technology(1165 questions)

Copyright 2004 Gleim Publications, Inc. Page 179Printed for g j

Page 180: Part three cia_with_ answers

Reduce the total ordering costs.A.Have no impact on total ordering costs.B.Reduce total carrying costs.C.Have no impact on total carrying costs.D.

Answer (A) is incorrect because total ordering costs will increase.Answer (B) is incorrect because total ordering costs will increase.Answer (C) is correct. Inventory carrying costs can sometimes be transferred to suppliers. If a seller has good enoughcontrol of demand schedules to know exactly when goods are needed, orders can be placed so that goods arrive no earlierthan when actually needed. This practice relies on a supplier who is willing to take the responsibility for storing the neededinventory and shipping it to arrive on time. Suppliers are more willing to provide this type of service when they have manycompetitors.Answer (D) is incorrect because total carrying costs are reduced.

[468] Gleim #: 4.118 -- Source: CIA 594 IV-37

An enterprise sells 1,500 units of a particular item each year and orders the items in equal quantities of 500 units at a priceof 5 per unit. No safety stocks are held. If the enterprise has a cost of capital of 12%, its annual cost of carrying inventory is

150A.180B.300C.900D.

Answer (A) is correct. The annual cost of carrying inventory equals the average inventory level times the cost perunit of inventory times the cost of capital. The average inventory level is the order quantity divided by 2. Thus, theannual cost of carrying inventory is 150 [(500 ÷ 2) × 5 × .12].Answer (B) is incorrect because 180 is obtained by using the total annual quantity rather than the average inventorylevel and by not multiplying by the unit price.Answer (C) is incorrect because 300 is obtained by using the order size rather than the average inventory level.Answer (D) is incorrect because 900 is based on the total annual quantity rather than the average inventory level.

[Fact Pattern #30]A company sells 10,000 skateboards a year at $66 each. All sales are on credit, with terms of 3/10, net 30, which means threepercent discount if payment is made within 10 days; otherwise full payment is due at the end of 30 days. One half of thecustomers are expected to take advantage of the discount and pay on day 10. The other half are expected to pay on day 30. Salesare expected to be uniform throughout the year for both types of customers.

[469] Gleim #: 4.119 -- Source: CIA 594 IV-35

(Refers to Fact Pattern #30)What is the expected average collection period for the company?

10 days.A.15 days.B.20 days.C.30 days.D.

Answer (A) is incorrect because 10 days assumes all customers take the discount.Answer (B) is incorrect because 15 days assumes half of the customers pay on day 30 but ignores the remaining halfof the customers who pay on day 10.Answer (C) is correct. The average collection period is the average time it takes to receive payment from customers.because one-half of the customers will pay on day 10 and half will pay on day 30, the average collection period is 20days [.5(10 days) + .5(30 days)].Answer (D) is incorrect because 30 days assumes all customers pay on day 30.

[470] Gleim #: 4.120 -- Source: CIA 594 IV-36

(Refers to Fact Pattern #30)Assume that the average collection period is 25 days. After the credit policy is well established, what is the expectedaverage accounts receivable balance for the company at any moment in time, assuming a 365-day year?

Gleim's CIA Test Prep: Part III: Business Analysis and Information Technology(1165 questions)

Copyright 2004 Gleim Publications, Inc. Page 180Printed for g j

Page 181: Part three cia_with_ answers

684.93A.1,808.22B.27,123.30C.45,205.48D.

Answer (A) is incorrect because 684.93 is based on annual credit sales of 10,000.Answer (B) is incorrect because 1,808.22 equals the credit sales per day.Answer (C) is incorrect because 27,123.30 is based on a 15-day average collection period.Answer (D) is correct. The expected average accounts receivable balance equals the average collection period times thecredit sales per day. Thus, the average accounts receivable balance is 45,205.48 {[(10,000 units sold on credit × 66 price) ÷365 days] × 25 days}. The foregoing calculation assumes that receivables are recorded at their gross amounts.

[471] Gleim #: 4.121 -- Source: CIA 1194 IV-21

(Refers to Fact Pattern #30)A corporation acquires an ownership interest in the common shares of another corporation. The ownership acquired is 30%of the outstanding common shares. In this situation, the long-term investment is generally accounted for on the investorcorporation’s books using which of the following reporting methods?

Lower-of-cost-or-market.A.Cost.B.Consolidated.C.Equity.D.

Answer (A) is incorrect because the lower-of-cost-or-market method is not generally used for equity investments.Answer (B) is incorrect because the cost basis is used when the investor cannot exercise significant influence over theinvestee (it has less than 20% of the voting power of the investee) and market prices are not readily available.Answer (C) is incorrect because consolidated reporting is ordinarily required only when the investor controls theinvestee.Answer (D) is correct. If an investor can exercise significant influence over an investee, the investment should beaccounted for by the equity method. When a corporation owns 20% or more of the voting power of the investee, theability to exercise significant influence is presumed.

[472] Gleim #: 4.122 -- Source: CIA 596 IV-30

A conglomerate company acquired 100% of the net assets of a target company for $900 cash. The target company’s balancesheet just prior to the acquisition is presented below.

Target Company (as of acquisition date)

  Carrying Fair  Amount Value Cash $ 100 $ 100Receivables 200 200Inventory 150 200Property, plant,

and equipment (net) 600 400 Total assets $1,050 $ 900      Current liabilities $ 200 $ 200Share capital 200Retained earnings 650 Total liabilities and equity $1,050 

The amount of goodwill to be recorded by the conglomerate company related to its purchase of the target company is

Gleim's CIA Test Prep: Part III: Business Analysis and Information Technology(1165 questions)

Copyright 2004 Gleim Publications, Inc. Page 181Printed for g j

Page 182: Part three cia_with_ answers

$(200)A.$50B.$200C.None of the answers are correct.D.

Answer (A) is incorrect because goodwill is positive, not negative.Answer (B) is incorrect because $50 is based on carrying amounts.Answer (C) is correct. Goodwill is the excess of the purchase price over the fair value of the acquired net identifiableassets. This fair value equals the sum of cash, receivables, inventory, and PPE, minus liabilities. Hence, the fair value of thenet identifiable assets is $700, and goodwill is $200 ($900 price – $700).Answer (D) is incorrect because the goodwill recorded is $200.

[473] Gleim #: 4.123 -- Source: CIA 1191 IV-46

Enterprise X owns 90% of Enterprise Y. Early in the year X lent Y 1,000,000. No payments have been made on the debt byyear-end. Proper accounting at year-end in the consolidated financial statements would

Eliminate 100% of the receivable, the payable, and the related interest.A.Eliminate 100% of the receivable and the payable but not any related interest.B.Eliminate 90% of the receivable, the payable, and the related interest.C.Eliminate 90% of the receivable and the payable but not any related interest.D.

Answer (A) is correct. In a consolidated balance sheet, reciprocal balances, such as receivables and payables,between a parent and a consolidated subsidiary should be eliminated in their entirety regardless of the portion of thesubsidiary’s shares held by the parent. Thus, all effects of the 1,000,000 loan should be eliminated in the preparationof the year-end consolidated balance sheet.Answer (B) is incorrect because the interest must be eliminated.Answer (C) is incorrect because all aspects of the transaction must be eliminated.Answer (D) is incorrect because all aspects of the transaction must be eliminated.

[474] Gleim #: 4.124 -- Source: CIA 593 IV-42

A plot of land is acquired in exchange for 250,000 cash and a noninterest-bearing note with a face amount of 1,000,000 onJanuary 1 of the current year. The 1,000,000 is payable in installments of 250,000 each, with the first installment dueDecember 31 of the current year. With regard to imputing interest on this note, (1) what market rate should be used toaccount for interest for the current year and (2) what should be done in future years when there is a change in prevailinginterest rates?

  (2)  Impact of Change in  Prevailing Interest  (1) Rates in Future  Market Rate Used to Compute Periods on Rate Used  Interest Expense for the current year to Account for This Note A. Rate prevailing at January 2 Ignore change in rateB. Rate prevailing at January 2 Use new market rateC. Rate prevailing at December 31 Ignore change in rateD. Rate revailing at December 31 Use new market rate

Answer (A) is correct. Determination of the imputed interest rate is made at the time the debt instrument is issued.Subsequent to initial recognition, all financial liabilities, other than those held for trading and liabilities that arederivatives, are measured at amortized cost. Consequently, a change in the prevailing market rate does not affect themeasurement of the noninterest-bearing note given for the land.Answer (B) is incorrect because any subsequent changes in prevailing interest rates are ignored.Answer (C) is incorrect because determination of the imputed interest rate is made at the time the debt instrument isissued.Answer (D) is incorrect because determination of the imputed interest rate is made at the time the debt instrument isissued, and any subsequent changes in prevailing interest rates are ignored.

Gleim's CIA Test Prep: Part III: Business Analysis and Information Technology(1165 questions)

Copyright 2004 Gleim Publications, Inc. Page 182Printed for g j

Page 183: Part three cia_with_ answers

[475] Gleim #: 4.125 -- Source: CIA 593 IV-37An enterprise issues bonds payable at a premium. You are analyzing the effects of using the effective interest rate (constantyield) method in accounting for the bonds over their 10-year life. Which of the following trends related to the reportedamounts for (1) interest expense and (2) carrying amount of the bonds would you expect to find?

  Interest Expense Carrying Amount A. Constant amount Constant amountB. Increasing amount Decreasing amountC. Decreasing amount Decreasing amountD. Decreasing amount Constant amount

Answer (A) is incorrect because the interest expense is a decreasing amount each period. It is computed by applying aconstant rate to a decreasing carrying amount.Answer (B) is incorrect because the interest expense is a decreasing amount each period. It is computed by applying aconstant rate to a decreasing carrying amount.Answer (C) is correct. When the effective interest rate method is used, interest expense equals the effective rate (aconstant rate) times the carrying amount at the beginning of the period. The carrying amount is the par value plus thebalance of the unamortized premium. The difference between interest expense and the nominal interest is thepremium amortization for the period. Thus, interest expense is a decreasing amount each period because a constantrate is applied to a decreasing carrying amount.Answer (D) is incorrect because the interest expense is a decreasing amount each period. It is computed by applying aconstant rate to a decreasing carrying amount.

[476] Gleim #: 4.126 -- Source: CIA 590 IV-41

At December 31 of year 1, an enterprise had a provision of 40,000 for the “Estimated Liability under Warranties” account.During year 2, the enterprise sold 1,000 new units under warranties reliably estimated at 100 per unit. The actual parts andlabor warranty expenditures for year 2 were 90,000. Warranty expense for year 3 should be

40,000A.50,000B.90,000C.100,000D.

Answer (A) is incorrect because 40,000 is the beginning liability balance.Answer (B) is incorrect because 50,000 equals actual expenditures minus the beginning liability.Answer (C) is incorrect because 90,000 equals current-year expenditures.Answer (D) is correct. Recognition of provisions is appropriate when the enterprise has a legal or constructivepresent obligation resulting from a past event (called an obligating event), it is probable that an outflow of economicbenefits will be necessary to settle the obligation, and its amount can be reliably estimated. Accordingly, a provisionshould be recognized for warranty expense in the period in which the matching revenue is recorded. Thus, the debitto warranty expense and the credit to provision for warranty expense in year 2 is 100,000 (100 × 1,000 units).

[477] Gleim #: 4.127 -- Source: CIA 594 IV-22

Which of the following is required in order for an enterprise to record a provision?

The exact payee must be known.A.The exact date payable must be known.B.It must be reasonably possible that an outflow of resources will be required to settle the obligation.C.It must be possible to determine a reliable estimate of the amount of the obligation.D.

Answer (A) is incorrect because the exact payee does not have to be known.Answer (B) is incorrect because the exact date payable does not have to be known.Answer (C) is incorrect because the resource outflow must be probable.Answer (D) is correct. A provision is a liability of uncertain timing and amount. Recognition of provisions isappropriate when the enterprise has a legal or constructive present obligation resulting from a past event (called anobligating event), it is probable that an outflow of economic benefits will be necessary to settle the obligation, and itsamount can be reliably estimated.

Gleim's CIA Test Prep: Part III: Business Analysis and Information Technology(1165 questions)

Copyright 2004 Gleim Publications, Inc. Page 183Printed for g j

Page 184: Part three cia_with_ answers

[478] Gleim #: 4.128 -- Source: CIA 593 IV-33An enterprise has the following contingencies at a balance sheet date:

An obligation for clean-up costs relating to environmental damage may, but probably will not, require an outflow ofresources.

I.

An obligation for decommissioning costs of a nuclear reactor may, but probably will not, require an outflow ofresources.

II.

The enterprise is jointly and severally liable for an obligation arising from a lawsuit; the likelihood of an outflow ofresources is remote.

III.

Which of the above items must be disclosed in the notes to the financial statements?

I and II only.A.II and III only.B.I and III only.C.I, II, and III.D.

Answer (A) is correct. Recognition of provisions is appropriate when the enterprise has a legal or constructivepresent obligation resulting from a past event (called an obligating event), it is probable that an outflow of economicbenefits will be necessary to settle the obligation, and its amount can be reliably estimated. Thus, none of thecircumstances in the question meet the recognition criteria for a provision. Hence, each is treated as a contingentliability, which requires disclosure unless the possibility of the outflow of resources is remote.Answer (B) is incorrect because items I and II but not III should be disclosed.Answer (C) is incorrect because items I and II but not III should be disclosed.Answer (D) is incorrect because items I and II but not III should be disclosed.

[479] Gleim #: 4.129 -- Source: CIA 593 IV-34

An enterprise had the following account balances at December 31 of year 1:

Common shares, 10 par, 100,000 sharesauthorized, 80,000 shares issued andoutstanding 800,000

Share premium 400,000Retained earnings 500,000

All shares outstanding were issued in a prior period for 15 per share. On January 5 of year 2, 1,000 shares were purchasedfor the treasury for 17 per share. These treasury shares were sold on February 6 of year 2, for 18 per share. The effect of thepurchase and sale of the 1,000 treasury shares was to

Increase equity by 1,000.A.Increase equity by 2,000.B.Increase equity by 3,000.C.Not change equity.D.

Gleim's CIA Test Prep: Part III: Business Analysis and Information Technology(1165 questions)

Copyright 2004 Gleim Publications, Inc. Page 184Printed for g j

Page 185: Part three cia_with_ answers

Answer (A) is correct. Using the cost method, the journal entry to record the purchase of the treasury shares is

Treasury shares 17,000Cash 17,000

   The journal entry to record the sale is   Cash 18,000

Treasury shares 17,000Share premium fromtreasury shares

1,000

Consequently, the net effect is to increase equity by 1,000.Answer (B) is incorrect because the purchase of treasury shares reduces equity by the cost of the shares, and the sale oftreasury shares increases equity by the amount received.Answer (C) is incorrect because the purchase of treasury shares reduces equity by the cost of the shares, and the sale oftreasury shares increases equity by the amount received.Answer (D) is incorrect because the purchase of treasury shares reduces equity by the cost of the shares, and the sale oftreasury shares increases equity by the amount received.

[480] Gleim #: 4.130 -- Source: CIA 1193 IV-44

An enterprise issued a noninterest bearing note payable due in 1 year in exchange for land. The fair value of the land is notreliably determinable. Which of the following statements is true concerning the accounting for the transaction?

The land should be recorded at the future value of the note, and interest should be imputed at the prevailing rate onsimilar notes.

A.

No interest should be recognized on the note, and the land should be recorded at the present value of the note.B.Interest on the note should be imputed at the prime rate, and the land should be recorded at the discounted value ofthe note.

C.

Interest on the note should be imputed at the prevailing rate for similar notes, and the land should be recorded at thepresent value of the note.

D.

Answer (A) is incorrect because the value of the land will be overstated if it is recorded at future value; present valueis the proper valuation.Answer (B) is incorrect because interest should be recognized on the note.Answer (C) is incorrect because the proper discount rate is the prevailing rate for similar notes, not necessarily theprime rate.Answer (D) is correct. When a financial liability is initially recognized, the enterprise should measure it at its cost,that is, the fair value of the consideration received. The fair value ordinarily is determined by reference to thetransaction price or other market prices. If these prices are not reliably determinable, the fair value is estimated bydiscounting the future cash payments at an imputed rate equal to the prevailing market rate for a similar instrument(e.g., similar as to currency, term, and type of rate) of an issuer with a similar credit rating.

[481] Gleim #: 4.131 -- Source: CIA 1186 IV-29

The proper classification of noncurrent, interest-bearing obligations scheduled to mature within 1 year of the balance sheetdate but that are expected to be refinanced on a long-term basis should be to

Exclude all such obligations from the current liabilities classification.A.Include in the current liabilities classification only those obligations that have actually been refinanced on a long-term basis after the balance sheet date but before the financial statements are authorized to be issued.

B.

Exclude from the current liabilities classification those obligations that have been refinanced on a long-term basisafter the balance sheet date but before the financial statements are authorized to be issued.

C.

Exclude from the current liabilities classification those obligations management wishes to exclude because theyintended to convert them to long-term obligations.

D.

Gleim's CIA Test Prep: Part III: Business Analysis and Information Technology(1165 questions)

Copyright 2004 Gleim Publications, Inc. Page 185Printed for g j

Page 186: Part three cia_with_ answers

Answer (A) is incorrect because exclusion is only appropriate when the intent to refinance is properly supported.Answer (B) is incorrect because exclusion is proper in these circumstances given that the intent to refinance has beensupported.Answer (C) is correct. Some noncurrent, interest-bearing liabilities due to be settled within 12 months should neverthelesscontinue to be classified as noncurrent if the original term exceeded 12 months, the enterprise intends to refinance on along-term basis, and its intent is supported by an agreement to refinance or reschedule payments. This agreement must beconsummated before the financial statements are authorized for issue.Answer (D) is incorrect because management’s intent is not sufficient for exclusion.

[482] Gleim #: 4.132 -- Source: CMA 688 3-25

In year 1, Voorhees introduced a new line of computer products that carry a 2-year warranty against defects andworkmanship. The enterprise estimates that the total warranty cost will be 10% of sales, with 40% of the expendituresoccurring during the first year and 60% during the second year. Sales and actual warranty expenditures for year 1 and year2 were as follows:

    Actual WarrantyYear Sales Expenditures

1 300,000 12,0002 400,000 30,000

At the end of year 2, the balance in the estimated accrued warranty liability account will be

24,000A.46,000B.58,000C.28,000D.

Answer (A) is incorrect because the account balance is found by subtracting the actual expenditures from the totalexpense.Answer (B) is incorrect because the account balance is found by subtracting the actual expenditures from the totalexpense.Answer (C) is incorrect because the account balance is found by subtracting the actual expenditures from the totalexpense.Answer (D) is correct. If the warranty expense is 10% of sales, the total expense for the 2 years is 70,000 (10% ×700,000). Of that 70,000, 12,000 was paid in year 1 and 30,000 in year 2. The 42,000 of payments leaves an unpaidbalance of 28,000 (70,000 – 42,000).

[483] Gleim #: 4.133 -- Source: CMA 689 3-11

On March 26, Zepher Enterprises contracted with a consultant for services to be performed during the period from March26 through April 30 in exchange for 10,000 treasury shares. The exchange took place on April 30. The treasury shares wereacquired in January, and recorded at cost when the market price was 25 per share. The market price on March 26 was 21.50per share. It was 23 per share on April 30. The per share amount recorded for the services should have been

21.50A.22.25B.23.00C.25.00D.

Answer (A) is correct. A transaction is typically measured at the fair value of the consideration given up unless thefair value of the consideration received is more clearly evident. No information is given about the value of theservices, so the market price of the treasury shares must be used. This price was 21.50 on March 26, the date of theagreement to trade the shares for services. Thus, the services to be received should have been measured on that datebased on the price of the shares, or 21.50 per share. Because no gain or loss is recognized on transactions in treasuryshares, the entry is to debit services for 21,500 (21.50 × 10,000), debit retained earnings or share premium fromtreasury share transactions for 3,500 [(25 – 21.50) × 10,000], and credit treasury shares for 25,000 (25 × 10,000).Answer (B) is incorrect because the services should be measured at the fair value on the date of the exchange.Answer (C) is incorrect because the services should be measured at the fair value on the date of the exchange.Answer (D) is incorrect because the services should be measured at the fair value on the date of the exchange.

Gleim's CIA Test Prep: Part III: Business Analysis and Information Technology(1165 questions)

Copyright 2004 Gleim Publications, Inc. Page 186Printed for g j

Page 187: Part three cia_with_ answers

[484] Gleim #: 4.134 -- Source: CMA 689 3-13Muncie sold 1,000 treasury shares at 33 per share. The shares had originally been issued at 12 per share and had beenrepurchased at 27 per share. The par value is 5 per share. The entry to record the reissuance using the cost method shouldinclude a credit to

Retained earnings of 6,000.A.Treasury shares of 28,000.B.Paid-in capital in excess of par of 28,000.C.Share premium of 6,000.D.

Answer (A) is incorrect because share premium should be credited when using the cost method, not retained earnings.Answer (B) is incorrect because the repurchase price was 27 per share.Answer (C) is incorrect because share premium should be credited for only 6,000.Answer (D) is correct. Under the cost method, treasury shares are carried at cost. In this case, cost is 27,000 (27 ×1,000 shares). The journal entry to record a sale at 33 per share is

Cash 33,000Treasury shares 27,000Share premium 6,000

[485] Gleim #: 4.135 -- Source: CMA 689 4-16

Trade accounts payable are measured on the balance sheet at the

Historical cost.A.Current cost.B.Settlement value.C.Present value of future cash flows.D.

Answer (A) is incorrect because many assets are measured at historical cost.Answer (B) is incorrect because the current cost is irrelevant if the enterprise has a contract to pay a stated monetaryamount.Answer (C) is correct. The most common financial liabilities are accounts payable or trade payables, which areobligations to sellers that are incurred when an enterprise purchases inventory, supplies, or services on credit. Theseliabilities arise when the goods or services have been received or supplied and have been invoiced or formally agreedwith the supplier. Trade payables are usually measured at settlement value, which is the undiscounted amount of cash(or the equivalent) expected to be paid to satisfy the obligations in the normal course of business.Answer (D) is incorrect because the expected cash outflows associated with current liabilities ordinarily are notdiscounted. The effect is usually not material because of the short period between the reporting date and the paymentdate.

[486] Gleim #: 4.136 -- Source: CIA 596 IV-24

The effective-interest method and the straight-line method of amortizing a bond discount differ in that the effective-interestmethod results in

Higher total interest expense over the term of the bonds.A.Escalating annual interest expense over the term of the bonds.B.Shrinking annual interest expense over the term of the bonds.C.Constant annual interest expense over the term of the bonds.D.

Answer (A) is incorrect because the two methods of amortization result in the same total interest expense over theterm of the bonds.Answer (B) is correct. Under the effective-interest method, interest expense for each period equals the effectiveinterest rate times the carrying value of the bond issue. As the discount is amortized, the carrying value rises andinterest expense increases.Answer (C) is incorrect because annual interest expense would decrease if a premium were being amortized.Answer (D) is incorrect because the straight-line method results in constant annual interest expense.

Gleim's CIA Test Prep: Part III: Business Analysis and Information Technology(1165 questions)

Copyright 2004 Gleim Publications, Inc. Page 187Printed for g j

Page 188: Part three cia_with_ answers

[487] Gleim #: 4.137 -- Source: CIA 596 IV-23On January 1, an enterprise issued a 10-year 500,000 bond at 96% of its face amount. The bond bears interest at 12%,payable on January 1 and July 1. The entry to record the issuance of the bond on January 1 would be

A.  Cash 480,000Bonds payable 480,000

B.  Cash 500,000Bonds payable 500,000

C.  Cash 480,000Discount on bonds payable 20,000

Bonds payable 500,000D.  Cash 500,000

Premium on bonds payable 20,000Bonds payable 480,000

Answer (A) is incorrect because the entry to bonds payable is based on the face, or maturity, amount of the bondissued. The difference between the amount received on issuance and the face amount is recorded as a premium ordiscount on bonds payable.Answer (B) is incorrect because the discount should be recognized.Answer (C) is correct. The enterprise received 480,000 cash on the issuance of the bond. Its face amount is 500,000,the amount to be paid at maturity. Hence, the credit to bonds payable is 500,000. The 20,000 difference is recorded asa discount on bonds payable (a debit) and is amortized over the life of the issue.Answer (D) is incorrect because the debit to cash is 480,000, a 20,000 discount should be debited, and the credit tobonds payable is 500,000.

[488] Gleim #: 4.138 -- Source: CIA 596 IV-21

The unit selling price of a new enterprise’s product is 10,000. The buyers are provided with a 2-year warranty that isexpected to cost the enterprise 250 per unit in the year of the sale and 750 per unit in the year following the sale. Theenterprise sold 80 units in the first year of operation and 100 units in the second year. Actual payments for warranty claimswere 10,000 and 65,000 in years one and two, respectively. The amount charged to warranty expense during the secondyear of operation is

25,000A.65,000B.85,000C.100,000D.

Answer (A) is incorrect because 25,000 is the expected amount of warranty claims for the first year of second-yearsales.Answer (B) is incorrect because 65,000 is the actual amount of claims in the second year.Answer (C) is incorrect because 85,000 is the expected amount of warranty claims in the second year.Answer (D) is correct. Under the accrual method, the total estimated warranty costs are charged to operating expensein the year of sale. The total estimated warranty cost per unit is 1,000 (250 + 750). In year two, 100 units were sold,so the warranty expense recognized is 100,000.

[489] Gleim #: 4.139 -- Source: CIA 1191 IV-59

Which of the following brings in additional capital to the firm?

Two-for-one share split.A.Conversion of convertible bonds to common shares.B.Exercise of warrants.C.Exercise of options purchased through an option exchange.D.

Answer (A) is incorrect because a share split is merely an accounting action that increases (or occasionally decreases)the number of shares outstanding. It does not generate additional capital.Answer (B) is incorrect because conversion of convertible bonds to common shares simply replaces debt withoutstanding common shares.Answer (C) is correct. Warrants are options that permit the holder to buy shares for a stated price. Their exerciseresults in inflows and the issuance of shares.Answer (D) is incorrect because options purchased and exercised through option exchanges are transactions betweenindividual investors not affecting the enterprise whose shares are involved.

Gleim's CIA Test Prep: Part III: Business Analysis and Information Technology(1165 questions)

Copyright 2004 Gleim Publications, Inc. Page 188Printed for g j

Page 189: Part three cia_with_ answers

[490] Gleim #: 4.140 -- Source: CMA 689 1-9A sound justification for an enterprise’s repurchase of its own shares (e.g., treasury shares) is to

Lower the debt to equity ratio of the firm.A.Increase the enterprise’s total assets.B.Reduce the idle cash and increase marketable securities.C.Meet the share availability needs of a potential merger.D.

Answer (A) is incorrect because the purchase of treasury shares increases the debt-to-equity ratio. Equity but not debtis reduced by the acquisition.Answer (B) is incorrect because the purchase of treasury shares decreases assets. Cash is paid out and shareholders’equity is decreased.Answer (C) is incorrect because treasury shares are recorded as a reduction of equity, not as a marketable security.The theory of this treatment is that an enterprise cannot own itself.Answer (D) is correct. An enterprise has many reasons to repurchase its own shares. These include meeting the needscreated by potential mergers or pension and profit-sharing plans. Also, management may want to buy out a dissidentshareholder. Sometimes, an enterprise has excess cash and can find no better investment than its own shares.Moreover, management may believe the shares are selling for a low price for no apparent reason. Thus, a purchasemay not only be a good investment but may also support the market price of the shares.

[491] Gleim #: 4.141 -- Source: CMA 1291 1-5

The purchase of treasury shares with an enterprise’s surplus cash

Increases an enterprise’s financial leverage.A.Increases an enterprise’s equity.B.Increases an enterprise’s interest coverage ratio.C.Dilutes an enterprise’s earnings per share.D.

Answer (A) is correct. A purchase of treasury share involves a decrease in assets (usually cash) and a correspondingdecrease in shareholders’ equity. Thus, equity is reduced and the debt-to-equity ratio and financial leverage increase.Answer (B) is incorrect because equity (assets – liabilities) declines. A treasury share purchase is equivalent to aspecial dividend because assets are paid out to one or more shareholders.Answer (C) is incorrect because a firm’s interest coverage ratio is unaffected. Earnings, interest expense, and taxeswill all be the same regardless of the transaction.Answer (D) is incorrect because the purchase of treasury share is antidilutive; the same earnings will be spread overfewer shares. Some firms purchase treasury share for this reason.

[492] Gleim #: 4.142 -- Source: CIA 595 IV-40

Compared with another bond with the same risk and maturity but without a conversion feature, a convertible bond has a

Higher face amount.A.Lower face amount.B.Higher coupon rate.C.Lower coupon rate.D.

Answer (A) is incorrect because the face amount is not a distinguishing feature of convertible bonds.Answer (B) is incorrect because the face amount is not a distinguishing feature of convertible bonds.Answer (C) is incorrect because convertible bonds have lower, not higher, coupon rates.Answer (D) is correct. Convertible bonds are convertible at the holder’s option into the issuer’s common shares at aspecified price. They have a lower coupon rate than nonconvertible bonds because they offer investors a chance forcapital gains.

[493] Gleim #: 4.143 -- Source: CMA 692 2-21

A contingent asset is

Gleim's CIA Test Prep: Part III: Business Analysis and Information Technology(1165 questions)

Copyright 2004 Gleim Publications, Inc. Page 189Printed for g j

Page 190: Part three cia_with_ answers

Recognized when condemnation awards are probable or can be reliably estimated.A.Recognized when damages to be awarded in a copyright infringement suit are highly probable.B.Recognized when disclosure in the notes to financial statements only could be misleading.C.Not recognized under any circumstances.D.

Answer (A) is incorrect because contingent assets are not recognized.Answer (B) is incorrect because contingent assets are not recognized.Answer (C) is incorrect because contingent assets are not recognized. If disclosure would be misleading, the disclosureshould not be made.Answer (D) is correct. A contingent asset is a possible asset arising from past events and the existence of which will beconfirmed only by uncertain future events not wholly within the enterprise’s control. An example is a potential recovery ona legal claim with an uncertain outcome. A contingent asset is not recognized but should be disclosed if an inflow ofeconomic benefits is probable. Disclosures include a description of the contingent asset and an estimate of its financialeffects. A contingent asset is not recognized because the income may not be realized. However, if realization is virtuallycertain, the asset is not contingent and may be recognized.

[494] Gleim #: 4.144 -- Source: Publisher

On the first day of the current fiscal year, Enterprise XXX issued 1,000 bonds, each of which is convertible into 100common shares of the issuer. The face amount of each bond is 1,000, the nominal annual interest rate is 7%, and the marketannual interest rate for similar nonconvertible debt is 10%. Interest is payable at the end of each year of the bonds' 5-yearterm. The present values of 1 for 5 periods at 7% and 9% are .713 and .650, respectively. The present values of an ordinaryannuity of 1 for 5 periods at 7% and 9% are 4.100 and 3.890, respectively. The fair value of the bonds' equity component isnot determinable. What is the entry for the initial recognition of the convertible bonds?

A.  Cash 1,000,000Bonds payable 1,000,000

B.  Cash 1,000,000Bond discount 77,700

Bonds payable 1,000,000Share capital-equity

component ofconvertible debt 77,700

C.  Cash 1,000,000Bonds payable 713,000Share capital-equity

component ofconvertible debt 287,000

D.  Cash 713,000Bond discount 287,000

Bonds payable 1,000,000

Answer (A) is incorrect because the IASs require that the debt and equity components of a compound instrument beseparately presented on the issuer's balance sheet.Answer (B) is correct. The initial total carrying amount of convertible debt or of debt instruments issued withdetachable share purchase warrants should be allocated between the debt instruments and the equity feature, andthese debt and equity components should be separately accounted for. The total assigned initially to the instrument asa whole equals the fair value of the consideration received. However, the IASs do not stipulate a method for makingthe allocation. One approach is to assign to the less easily measurable component a residual amount after determiningthe amount of the more readily measurable component. For example, the future payments on the financial liabilitymight be discounted at the market rate for a similar instrument without an equity feature. This amount would then besubtracted from the amount of the compound instrument as a whole to determine the carrying amount of the equityfeature. This method is appropriate when the fair value of the equity feature is not determinable. Thus, the presentvalue of the principal repayment discounted at the market rate of 9% is 650,000 (1,000,000 × .650), the present valueof the interest payments discounted at 9% is 272,300 [(.07 × 1,000,000) × 3.890], and their total is 922,300. Theresidual amount assignable to the equity feature is therefore 77,700 [(1,000 par × 1,000 bonds) proceeds – 922,300assigned to the debt component], which is also the bond discount given that the convertible bonds were issued at par.Answer (C) is incorrect because the cash flows from the bonds should be discounted at the market rate.Answer (D) is incorrect because the proceeds equaled 1,000,000, the cash flows from the bonds should be discountedat the market rate, and the equity feature should be separately presented.

[495] Gleim #: 5.1 -- Source: CIA 1195 IV-24

Changes in accounting estimates are viewed as

Gleim's CIA Test Prep: Part III: Business Analysis and Information Technology(1165 questions)

Copyright 2004 Gleim Publications, Inc. Page 190Printed for g j

Page 191: Part three cia_with_ answers

Extraordinary items.A.Errors in reported amounts in prior periods.B.Catch-up adjustments related to amounts reported in prior periods.C.Normal recurring revisions.D.

Answer (A) is incorrect because extraordinary items are clearly distinct from ordinary activities and not expected to recurfrequently or regularly. Changes in accounting estimates are normal and recurring.Answer (B) is incorrect because changes in accounting estimates are not errors. They are responses to new information,subsequent events, additional experience, or changes in circumstances.Answer (C) is incorrect because catch-up adjustments to prior reported amounts are retroactive. Changes in accountingestimates are accounted for prospectively.Answer (D) is correct. A change in accounting estimate is a normal, recurring revision resulting from changes incircumstances, additional experience, new information, or subsequent events. It is neither a fundamental error nor anextraordinary item. The effect of a change in estimate is included in the determination of net profit or loss in the period ofchange, if the change affects that period only, or in the period of change and in future periods, if the change affects both thecurrent and future periods. In other words, a change in accounting estimate should be reflected in the financial statementsprospectively.

[496] Gleim #: 5.2 -- Source: CIA 597 IV-22

The following financial statement notes are extracts from the audited financial statements of public enterprises. Which notedescribes a change in accounting estimate?

The enterprise changed its amortization of capital assets based on a reassessment of the useful lives of the assets.Accordingly, the enterprise changed its rate of amortization from 5% and 6% to 8% and 10%, for machinery andequipment.

A.

Prior to 2001, plant and equipment (other than customer service replacement parts) were depreciated using thediminishing-balance method. Plant and equipment are now depreciated on a straight-line basis.

B.

During the year, the enterprise changed its method of accounting for noninterest-bearing, nonrecourse loans due fromemployees, pursuant to a change in an International Accounting Standard.

C.

Effective January 1, 2001, the enterprise changed to the LIFO method of inventory valuation. Prior to 2001, the FIFOmethod was used.

D.

Answer (A) is correct. Accounting estimates, e.g., service lives, residual values, warranty costs, uncollectibleaccounts, and inventory obsolescence, are a necessary part of preparing financial statements. However, theyinevitably change as new events occur and as additional experience and information are obtained. When alteredconditions require a change in estimate, it is accounted for prospectively. Thus, a change in the estimate of the servicelives of depreciable assets is a change in accounting estimate.Answer (B) is incorrect because a change from diminishing-balance depreciation to straight-line depreciation is achange in accounting policy.Answer (C) is incorrect because changing an accounting method due to a change in an IAS is a change in accountingpolicy.Answer (D) is incorrect because a change from FIFO to LIFO inventory valuation is a change in accounting policy.

[497] Gleim #: 5.3 -- Source: CIA 597 IV-20

On January 1, 2000, an enterprise purchased a machine for 10,000. The estimated useful life was 10 years, with no residualvalue. The enterprise depreciates its property, plant, and equipment using the straight-line method. On January 1, 2004, itwas estimated that the machine had a remaining useful life of 3 years. Compute the enterprise’s 2004 depreciation expensefor the machine.

1,000A.2,000B.3,000C.6,000D.

Answer (A) is incorrect because 1,000 assumes no change in estimate.Answer (B) is correct. The machine’s net carrying amount at January 1, 2004 is 6,000 (10,000 cost – 4,000accumulated depreciation for 4 years). A change in accounting estimate is applied prospectively. Thus, depreciationexpense is 2,000 per year for the next 3 years.Answer (C) is incorrect because 3,000 assumes a 2-year remaining useful life.Answer (D) is incorrect because 6,000 is the carrying amount at January 1, 2004.

Gleim's CIA Test Prep: Part III: Business Analysis and Information Technology(1165 questions)

Copyright 2004 Gleim Publications, Inc. Page 191Printed for g j

Page 192: Part three cia_with_ answers

[498] Gleim #: 5.4 -- Source: CIA 596 IV-28When financial statements are being prepared, which of the following items requires that accountants estimate the effects offuture conditions and events?

The purchase price for an acquired building.A.The price of a marketable security.B.The amount of recoverable mineral reserves.C.The physical quantity of inventory.D.

Answer (A) is incorrect because the purchase price for an acquired building can be ascertained with certainty. Noestimate is required.Answer (B) is incorrect because the price of a marketable security can be calculated with certainty. No estimate isrequired.Answer (C) is correct. Changes in estimates used in accounting are necessary consequences of periodic presentationsof financial statements. Preparing financial statements requires estimating the effects of future events. Examples ofitems for which estimates are necessary are uncollectible receivables, inventory obsolescence, service lives andresidual values of depreciable assets, warranty costs, periods benefited by a deferred cost, and recoverable mineralreserves.Answer (D) is incorrect because the physical quantity of inventory as of the balance sheet date can be measured.Although some estimation of the correct amount may be required, the estimates will not depend on future conditionsand events but on current conditions and measurement methods.

[499] Gleim #: 5.5 -- Source: CIA 1195 IV-23

In the prior accounting period, an enterprise incorrectly expensed a newly purchased piece of equipment rather thanestablishing an asset balance and beginning to depreciate it over the estimated useful life of the item. To correct thisfundamental error in the single-period financial statements of the current period, the enterprise records which entry if ituses the benchmark treatment?

 A. Debit equipment  Credit retained earnings  Credit accumulated depreciation – equipmentB. Debit retained earnings  Debit accumulated depreciation – equipment  Credit equipmentC. Debit equipment  Debit retained earnings  Credit accumulated depreciation – equipmentD. Debit equipment  Debit accumulated depreciation – equipment  Credit equipment

Answer (A) is correct. The benchmark treatment of fundamental errors requires retrospective treatment. To correctthe fundamental error, the enterprise must debit equipment for its cost and credit accumulated depreciation for thedepreciation expense appropriate for the first year of the estimated useful life. Retained earnings must be creditedbecause the error understated net profit or overstated net loss in the prior period.Answer (B) is incorrect because this entry is the reverse of the correct entry.Answer (C) is incorrect because retained earnings should be credited.Answer (D) is incorrect because accumulated depreciation and retained earnings should be credited.

[500] Gleim #: 5.6 -- Source: CIA 1188 IV-45

A retrospective change in an accounting policy in the current period should be accounted for in comparative reports by

An adjustment directly to retained earnings and restatement of prior years’ statements.A.A line item on the current income statement for the cumulative effect of the change.B.Presentation of pro forma comparative information.C.Note disclosure only in the current period.D.

Gleim's CIA Test Prep: Part III: Business Analysis and Information Technology(1165 questions)

Copyright 2004 Gleim Publications, Inc. Page 192Printed for g j

Page 193: Part three cia_with_ answers

Answer (A) is correct. According to the benchmark treatment, a change in accounting policy is applied retrospectivelyunless any adjustment relating to prior periods is not reasonably determinable. The resulting adjustment is to the openingbalance of retained earnings for the earliest period presented. If practicable, comparative information is restated.Answer (B) is incorrect because reporting the cumulative effect is the allowed alternative treatment for a change in anaccounting policy.Answer (C) is incorrect because this answer describes one of the requirements under the allowed alternative treatment for achange in an accounting policy.Answer (D) is incorrect because a change in an accounting policy qualifying for retrospective application requires anadjustment to retained earnings.

[501] Gleim #: 5.7 -- Source: CMA 681 3-24

An accounting change requiring retrospective treatment is a change in

The residual value of equipment.A.Depreciation methods from straight-line to diminishing-balance.B.An accounting policy inseparable from a change in an accounting estimate.C.A provision for warranty costs.D.

Answer (A) is incorrect because the residual value of equipment is a change in estimate that is accounted for on aprospective basis (in the future).Answer (B) is correct. A change in depreciation methods is reported as a change in accounting policy. According tothe benchmark treatment, a change in accounting policy is applied retrospectively unless any adjustment relating toprior periods is not reasonably determinable. The resulting adjustment is to the opening balance of retained earningsfor the earliest period presented. If practicable, comparative information is restated.Answer (C) is incorrect because, if distinguishing between a change in estimate and a change in accounting policy isdifficult, the change is accounted for as a change in estimate and properly disclosed.Answer (D) is incorrect because a provision for warranty costs is a change in estimate that is accounted for on aprospective basis (in the future).

[502] Gleim #: 5.8 -- Source: CIA 1196 IV-2

If ending inventory is underestimated due to an error in the physical count of items on hand, the cost of goods sold for theperiod will be <List A> and net earnings will be <List B>.

  List A List B A. Underestimated UnderestimatedB. Underestimated OverestimatedC. Overestimated UnderestimatedD. Overestimated Overestimated

Answer (A) is incorrect because cost of goods sold will be overestimated.Answer (B) is incorrect because cost of goods sold will be overestimated and net earnings will be underestimated.Answer (C) is correct. Cost of goods sold equals beginning inventory, plus purchases, minus ending inventory. Ifending inventory is underestimated, cost of goods sold will be overestimated for the period. If cost of goods sold isoverestimated, the net earnings for the period will be underestimated.Answer (D) is incorrect because net earnings will be underestimated.

[503] Gleim #: 5.9 -- Source: CIA 1196 IV-3

The failure to record an accrued expense at year-end will result in which of the following overstatement errors in thefinancial statements prepared at that date?

  Net Profit Working Capital Cash A. No No YesB. No Yes NoC. Yes No NoD. Yes Yes Yes

Gleim's CIA Test Prep: Part III: Business Analysis and Information Technology(1165 questions)

Copyright 2004 Gleim Publications, Inc. Page 193Printed for g j

Page 194: Part three cia_with_ answers

Answer (A) is incorrect because the failure to record an accrued expense will result in an overstatement of net profit and anoverstatement of working capital, and will have no effect on cash.Answer (B) is incorrect because the failure to record an accrued expense will result in an overstatement of net profit.Answer (C) is incorrect because the failure to record an accrued expense will result in an overstatement of working capital.Answer (D) is correct. An accrued expense is an expense that has been incurred but not paid. The appropriate adjustingentry to record an accrued expense will increase an expense account and increase a liability account. The failure to record anaccrued expense will result in an understatement of expenses leading to an overstatement of net profit. The failure to recordthe increase in a liability account will result in an understatement of current liabilities leading to an overstatement ofworking capital. There will be no effect on cash.

[504] Gleim #: 5.10 -- Source: CIA 1196 IV-31

Which of the following errors is not self-correcting over two accounting periods?

Failure to record accrued wages.A.Failure to record depreciation.B.Overstatement of inventory.C.Failure to record prepaid expenses.D.

Answer (A) is incorrect because understatement of accrued wages is a self-correcting error. Future wage expense willbe overstated, future cost of goods sold will be overstated, and future expenses will be understated, respectively.Answer (B) is correct. A failure to record depreciation must be corrected because the effects of the error do notautomatically reverse in future periods. Expenses are understated in the year of the error, but no correspondingoverstatement of expenses occurs in later years.Answer (C) is incorrect because overstatement of inventory and the consequent understatement of cost of goods soldis a self-correcting error. Future wage expense will be overstated, future cost of goods sold will be overstated, andfuture expenses will be understated, respectively.Answer (D) is incorrect because understatement of prepaid expenses (overstatement of expenses) is a self-correctingerror. Future wage expense will be overstated, future cost of goods sold will be overstated, and future expenses willbe understated, respectively.

[505] Gleim #: 5.11 -- Source: CIA 597 IV-21

Which of the following statements is true for a defined contribution postemployment benefit plan?

The employer is required to contribute a certain amount each period based on the plan’s formula.A.The employer bears the risk of the plan’s investment performance.B.Postemployment benefits received by employees are defined by the plan’s formula.C.The employer and employees are required to contribute equal amounts to the fund.D.

Answer (A) is correct. A defined contribution plan provides benefits in exchange for services, provides an accountfor each participant, and specifies how contributions are to be determined. Postemployment benefits depend only oncontributions, returns on investment, and allocated forfeitures of other participants’ benefits. Thus, employees havethe benefit of gain and the risk of loss.Answer (B) is incorrect because the employees bear the risk of the plan’s investment performance.Answer (C) is incorrect because, under a defined benefit plan, the postemployment benefits received by employeesare defined by the plan’s formula.Answer (D) is incorrect because equal contributions are not required for a defined contribution plan.

[506] Gleim #: 5.12 -- Source: CIA 595 IV-16

The defined postemployment benefit obligation of an enterprise includes benefit obligations to <List A> employees at <ListB> salary levels.

  List A List B A. Vested CurrentB. Vested FutureC. Vested and nonvested CurrentD. Vested and nonvested Future

Gleim's CIA Test Prep: Part III: Business Analysis and Information Technology(1165 questions)

Copyright 2004 Gleim Publications, Inc. Page 194Printed for g j

Page 195: Part three cia_with_ answers

Answer (A) is incorrect because the defined postemployment benefit includes both vested and nonvested benefits and iscalculated at future levels.Answer (B) is incorrect because the defined postemployment benefit includes both vested and nonvested benefits and iscalculated at future levels.Answer (C) is incorrect because the defined postemployment benefit includes both vested and nonvested benefits and iscalculated at future levels.Answer (D) is correct. The measurement of a postemployment benefit obligation estimates of future salary increases, thebenefits defined in the plan, the benefits arising from any constructive obligation beyond the terms of the plan, andestimates of future changes in state benefits that affect the level of plan benefits. The possibility that nonvested projectedbenefits will not vest is a factor in the measurement of the DBO, but it does not affect the existence of the obligation.

[507] Gleim #: 5.13 -- Source: CIA 1189 IV-44

An employee’s right to obtain postemployment benefits regardless of whether (s)he remains employed is known as his/her

Past service cost.A.Defined benefit plan.B.Vested benefits.C.Additional minimum liability.D.

Answer (A) is incorrect because past service cost relates to benefits for employee service provided prior to theadoption or amendment of a defined benefit plan.Answer (B) is incorrect because a defined benefit plan provides a defined benefit based on one or more factors, suchas level of compensation, years of service, or age.Answer (C) is correct. Vested benefits are those earned postemployment benefits owed to an employee that are notcontingent upon the employee’s continued service. Whether benefits have vested affects the measurement of theemployer’s defined benefit obligation but not its existence. Moreover, vesting affects the accounting for past servicecost. Past service cost is amortized as an expense over the average period until the benefits are vested.Answer (D) is incorrect because, under IAS 19, the employer does not recognize an additional minimum liability.

[508] Gleim #: 5.14 -- Source: Publisher

An employer sponsors a defined postemployment benefit plan. If the given amount of the present value of the definedbenefit obligation exceeds the given amount of the fair value of plan assets, the defined benefit liability to be recognized inthe balance sheet is greatest when the employer has

Net unrecognized actuarial gains and no past service cost.A.Net unrecognized actuarial losses and no past service cost.B.Net unrecognized actuarial losses and unrecognized past service cost.C.No unrecognized actuarial gains or losses and unrecognized past service cost.D.

Answer (A) is correct. The amount of the defined benefit liability recognized equals the present value of the definedbenefit obligation at the balance sheet date, plus (minus) unrecognized actuarial gains (losses), minus unrecognizedpast service cost, minus the fair value of plan assets at the balance sheet date. If this amount is negative, it representsan asset. However, the maximum that may be recognized for such an asset is the sum of unrecognized actuariallosses, unrecognized past service cost, and the present value of future refunds from the plan or reductions in futurecontributions. Thus, if the excess of the DBO over the fair value of plan assets is constant, net unrecognized actuarialgains will increase the liability. Net unrecognized actuarial losses and unrecognized past service cost decrease theliability.Answer (B) is incorrect because net unrecognized actuarial losses decrease the liability.Answer (C) is incorrect because net unrecognized actuarial losses and unrecognized past service cost decrease theliability.Answer (D) is incorrect because net unrecognized actuarial gains increase the liability.

Gleim's CIA Test Prep: Part III: Business Analysis and Information Technology(1165 questions)

Copyright 2004 Gleim Publications, Inc. Page 195Printed for g j

Page 196: Part three cia_with_ answers

[509] Gleim #: 5.15 -- Source: PublisherThe following information relates to the activity of the defined postemployment benefit plan of Twain Publishers, Ltd.:

Current service cost 120,000Expected return on plan assets 30,000Interest cost on defined benefit obligation 40,000Amortization of net actuarial loss 10,000Past service cost 5,000

Twain’s expense recognized in the income statement is

120,000A.135,000B.140,000C.145,000D.

Answer (A) is incorrect because 120,000 includes only the current service cost component.Answer (B) is incorrect because 135,000 excludes the amortization of the actuarial loss.Answer (C) is incorrect because 140,000 excludes the past service cost.Answer (D) is correct. Components of the expense are current service cost, interest cost, the expected return on planassets, past service cost (recognition in full of vested amounts and amortization of nonvested amounts). Currentservice cost, interest cost, the amortization of actuarial loss, and the past service cost increase the expense. Theexpected return on plan assets decreases the expense.

Current service cost 120,000Return on plan assets (30,000)Interest cost 40,000Amortization of actuarial loss 10,000Past service cost 5,000 Expense 145,000 

[510] Gleim #: 5.16 -- Source: Publisher

At the start of its current fiscal year, Emper Corporation amended its defined postemployment benefit plan, resulting in anincrease in the present value of the DBO. The benefits become vested after 6 years of service. Past service cost arising fromthe plan amendment includes 400,000 of benefits that are already vested and 200,000 of nonvested benefits. If the averageperiod until vesting is 4 years, the minimum past service cost to be recognized in the first year is

50,000A.200,000B.450,000C.600,000D.

Answer (A) is incorrect because 50,000 is the periodic amortization of nonvested benefits.Answer (B) is incorrect because 200,000 is the amount of the nonvested benefits.Answer (C) is correct. Past service cost is the increase in the present value of the DBO related to prior employeeservice that arises in the current period from the introduction of, or an amendment to, postemployment benefits.Accordingly, 400,000 should be recognized immediately to reflect the vested benefits and amortization of thenonvested benefits equals 50,000 (200,000 ÷ 4), a total of 450,000.Answer (D) is incorrect because 600,000 includes nonvested benefits not yet required to be amortized.

[511] Gleim #: 5.17 -- Source: Publisher

At end of the year, Employer’s defined benefit obligation (DBO) was determined to be 1,500,000, which was 200,000higher than had been expected. The defined benefit plan’s assets had a fair value of 1,250,000. No other actuarial gains andlosses have occurred. If the average remaining working life is 20 years, the minimum required amortization of theunrecognized net actuarial gains and losses in the next year will be

Gleim's CIA Test Prep: Part III: Business Analysis and Information Technology(1165 questions)

Copyright 2004 Gleim Publications, Inc. Page 196Printed for g j

Page 197: Part three cia_with_ answers

20,000A.3,750B.2,500C.0D.

Answer (A) is incorrect because 20,000 is the result of using the full 200,000 liability loss without regard to the corridoramount and assumes an amortization period of 10 years instead of 20.Answer (B) is incorrect because 3,750 is the result of using 125,000 (10% × 1,250,000 plan assets) as the corridor amountinstead of 150,000.Answer (C) is correct. At a minimum, amortization of the cumulative unrecognized net actuarial gains and losses must beincluded as income or expense if, as of the close of the prior period, that unrecognized gain or loss exceeds 10% of thegreater of the present fair value of the DBO or the fair value of plan assets. At year-end, the present value of the DBO was200,000 greater than estimated (a 200,000 liability loss). Given that no other gain or loss has occurred, the unrecognizedactuarial net loss to be amortized beginning next year is 200,000. The corridor amount is 150,000 (10% of the greater of1,500,000 present value of the DBO or 1,250,000 fair value of plan assets). The amount outside the corridor is 50,000(200,000 – 150,000), and the amount to be amortized is therefore 2,500 (50,000 ÷ 20 years of average remaining workinglife).Answer (D) is incorrect because 50,000 of the liability loss must be amortized over the average remaining working lifebeginning the year following the loss.

[512] Gleim #: 5.18 -- Source: Publisher

For an enterprise with a defined postemployment benefit plan, the fair value of plan assets at the beginning of the year was500,000. No unrecognized net cumulative actuarial gain or loss existed. On the last day of the fiscal year, the fair value ofplan assets was 620,000. Benefits paid equaled 100,000, and the enterprise made 120,000 in contributions. The discountrate was 10%, and the expected long-term rate of return on plan assets was 12%. The actual return on plan assets was

50,000A.60,000B.75,000C.100,000D.

Answer (A) is incorrect because 50,000 equals 10% of the beginning fair value.Answer (B) is incorrect because 60,000 equals 12% of the beginning fair value.Answer (C) is incorrect because 75,000 is a nonsense number.Answer (D) is correct. The actual return on plan assets is equal to the difference between the fair value of plan assetsat the beginning and the end of the year adjusted for contributions and benefits paid. The actual return thus is100,000.

FV, end of year 620,000Benefits paid 100,000Contributions (120,000)FV, beginning of year (500,000) Actual return 100,000 

[513] Gleim #: 5.19 -- Source: Publisher

An enterprise that sponsors a defined benefit postemployment benefit plan has discontinued a segment of its business. As aresult, the employees of that segment will not be able to earn additional benefits. On the date of the discontinuance, thepresent value of the DBO was 10,000,000, the fair value of the plan assets was 8,500,000, the net unrecognized actuarialgain was 500,000, and the unrecognized past service cost was 100,000. The effect of the discontinuance was to reduce thepresent value of the DBO by 10%. The portions of the unrecognized actuarial gain and past service cost related to theportion of the DBO that was eliminated also equaled 10%. Accordingly, the enterprise should recognize what net definedbenefit liability immediately after the discontinuance?

410,000A.860,000B.900,000C.950,000D.

Gleim's CIA Test Prep: Part III: Business Analysis and Information Technology(1165 questions)

Copyright 2004 Gleim Publications, Inc. Page 197Printed for g j

Page 198: Part three cia_with_ answers

Answer (A) is incorrect because 410,000 omits the remaining unrecognized actuarial gain.Answer (B) is correct. A curtailment arises from a reduction in covered employees or amendment of the plan to reducebenefits for future service. A settlement is a transaction that eliminates the DBO for part or all of plan benefits. When acurtailment or settlement occurs, the gain or loss recognized encompasses the change in the present value of the DBO, thechange in the fair value of plan assets, and previously unrecognized related actuarial gains and losses and past service cost.Thus, the present value of the DBO after the curtailment was 9,000,000 [10,000,000 – (10% × 10,000,000)]. The fair valueof plan assets was given as 8,500,000. The unrecognized actuarial gain was reduced to 450,000. The unrecognized pastservice cost was reduced to 90,000. Accordingly, the defined benefit liability immediately after the discontinuance was860,000 (9,000,000 – 8,500,000 + 450,000 – 90,000).Answer (C) is incorrect because 900,000 is 10% of the present value of the remaining DBO.Answer (D) is incorrect because 950,000 omits past service cost from the calculation.

[514] Gleim #: 5.20 -- Source: CIA 1195 IV-28

If a lease agreement transfers substantially all of the risks and rewards of ownership of the asset to the lessee, the assetvalue is recognized on the lessee’s records as a(n) <List A> asset, and the lease is referred to as a <List B> lease.

  List A List B A. Tangible FinanceB. Intangible FinanceC. Tangible OperatingD. Intangible Operating

Answer (A) is correct. When a lease agreement transfers the risks and rewards of ownership of the asset to the lessee,the lease is treated as a finance lease because the transaction is in essence an installment purchase. Accordingly, thelessee records a depreciable asset and a liability. Moreover, IAS 38, Intangible Assets, specifically does not apply toleases that are within the scope of IAS 17, Leases. A finance lease is therefore regarded as a tangible asset.Answer (B) is incorrect because the recorded asset is a tangible asset.Answer (C) is incorrect because, if it transfers substantially all of the risks and rewards of ownership, the lease is afinance lease.Answer (D) is incorrect because, if it transfers substantially all of the risks and rewards of ownership, the lease is afinance lease.

[515] Gleim #: 5.21 -- Source: CIA 596 IV-32

Finance and operating leases differ in that the lessor

Obtains use of the asset only under a finance lease.A.Is using the lease as a source of financing only under an operating lease.B.Makes rent payments that are actually installment payments constituting a payment of both principal and interest onlyunder a finance lease.

C.

Finances the transaction through the leased asset only under a finance lease.D.

Answer (A) is incorrect because the lessee obtains use of the asset.Answer (B) is incorrect because the lessee uses the lease as a source of financing under a finance lease, not anoperating lease.Answer (C) is incorrect because the lessee makes payments to the lessor.Answer (D) is correct. A lease is a rental or sub-purchase arrangement between a lessor (the owner or seller of theproperty) and a lessee (the renter or purchaser). The issue in all leases is whether the risks and rewards of ownershiphave been transferred from the lessor to the lessee; if so, the lease should be accounted for as a sale-purchase, i.e., afinance lease. If the risks and rewards of ownership have not transferred, the lease is a rental arrangement and iscalled an operating lease. In effect, the lessor provides financing for an installment purchase, and the lessee’spayments include both principal and interest components.

[516] Gleim #: 5.22 -- Source: CIA 595 IV-27

Which of the following statements is false of a finance lease?

Gleim's CIA Test Prep: Part III: Business Analysis and Information Technology(1165 questions)

Copyright 2004 Gleim Publications, Inc. Page 198Printed for g j

Page 199: Part three cia_with_ answers

The lessor capitalizes the net investment in the lease.A.The lessor records the leased item as an asset.B.The lessee records depreciation or finance cost allowance on the leased asset.C.The lease arrangement represents a form of financing.D.

Answer (A) is incorrect because, under a finance lease, the lessor recognizes a net receivable equal to the net investment inthe lease:  gross investment (minimum lease payments from the lessor’s perspective plus unguaranteed residual value)minus unearned finance income.Answer (B) is correct. When a transaction meets the criteria of a finance lease, the lessor removes the leased item from thebooks and records lease payments receivable regardless of whether the lessor is a manufacturer or dealer. The lessee recordsand depreciates the leased item under a finance lease.Answer (C) is incorrect because the lessee records depreciation on the leased asset under a finance lease. This process isseparate from the accounting for the lease obligation.Answer (D) is incorrect because, in essence, the leased asset is being purchased when a lease meets the criteria forcapitalization. Hence, the lease agreement represents a form of financing.

[517] Gleim #: 5.23 -- Source: CIA 1191 IV-44

At the inception of a finance lease, how should the lessee account for guaranteed residual value?

As part of minimum lease payments at present value.A.As part of minimum lease payments at future value.B.As part of minimum lease payments at future value of an annuity due.C.As not a part of the lease contract.D.

Answer (A) is correct. The lessee records a finance lease as an asset and a liability at the inception of the lease at thefair value of the leased property (not to exceed the present value of the minimum lease payments). The lessee’sminimum lease payments include required payments (excluding contingent rent and costs for services and taxes to bepaid by and reimbursed to the lessor) during the lease term and the amount of a bargain purchase option. If nobargain purchase option exists, the minimum lease payments equal the sum of the minimum payments payable overthe lease term and any amounts guaranteed by the lessee or by a party related to the lessee.Answer (B) is incorrect because the guaranteed residual must be discounted to present value.Answer (C) is incorrect because the guaranteed residual must be discounted to present value.Answer (D) is incorrect because guaranteed residuals are part of the lease contract.

[518] Gleim #: 5.24 -- Source: CIA 596 IV-31

Which of the following leases ordinarily should be classified as a finance lease by the lessee?

  LeaseA

LeaseB

LeaseC

LeaseD

Contains a bargainpurchase option?

Yes No No No

Lease term is for themajor part of theeconomic life of theleased asset

No No Yes No

Present value of theminimum lease paymentsis substantially all of thefair value of the leasedasset

No No No Yes

Leased asset usable onlyby lessor without majormodification

No Yes Yes No

Gleim's CIA Test Prep: Part III: Business Analysis and Information Technology(1165 questions)

Copyright 2004 Gleim Publications, Inc. Page 199Printed for g j

Page 200: Part three cia_with_ answers

Lease A only.A.Lease B only.B.Leases A, C, and D.C.Leases C and D only.D.

Answer (A) is incorrect because Leases C and D are also finance leases.Answer (B) is incorrect because B is the only operating lease in the set. If it were usable by the lessee (not the lessor)without major modification, it would normally be classified as a finance lease.Answer (C) is correct. A lease should be classified as a finance lease by a lessee if it transfers substantially all of the risksand rewards of ownership. A lease is classified at its inception. It normally is classified as a finance lease if, for example, thelease provides for the transfer of ownership of the leased asset by the end of the lease term; the lease contains a bargainpurchase option, i.e., the lessee has the option to purchase at a price expected to be sufficiently below the fair value of theexercise date that, at the lease’s inception, exercise is reasonably certain; the lease term is for the major part of the economiclife of the leased asset; the present value of the minimum lease payments is at least substantially all of the fair value of theleased asset at the inception of the lease; or the leased asset is such that it can be used only by the lessee without majormodification. Lease A is a finance lease because the terms of the lease include a bargain purchase option. Lease C passes theeconomic life test, and lease D passes the recovery of investment test.Answer (D) is incorrect because Lease A contains a bargain purchase option, so it qualifies as a finance lease.

[519] Gleim #: 5.25 -- Source: CIA 585 IV-22

ABC Enterprises, a manufacturer lessor, leased a machine to XYZ on January 1. The lease meets the criteria for a financelease. Title to the asset will automatically pass to the lessee at the end of the lease term. Other details are as follows:

Lease term 10 yearsUseful life of the asset 10 yearsCost of the leased asset to the lessor 55,000Annual payment payable at the beginning

of each year, beginning January 1 10,000Implicit interest rate 10%Present value of an annuity due of 1

discounted for 10 years at 10% 6.7590Present value of 1 due in 10 years

discounted at 10% .3855

Assuming the fair value of the asset is at least equal to the present value of the minimum lease payments, the journal entryto record the inception of this lease on the lessor’s books at January 1 is

Gleim's CIA Test Prep: Part III: Business Analysis and Information Technology(1165 questions)

Copyright 2004 Gleim Publications, Inc. Page 200Printed for g j

Page 201: Part three cia_with_ answers

 A. Leased machine 67,590  Lease liability 57,590  Cash 10,000B. Lease payments  receivable 90,000  Cash 10,000  Cost of sales 55,000  Inventory 55,000  Unearned finance  income--leases 45,000  Sales 55,000C. Lease payments  receivable 90,000  Cash 10,000  Finance income 32,410  Gross profit on lease 12,590  Inventory 55,000D. Lease payments  receivable 90,000  Cash 10,000  Cost of sales 55,000  Sales 67,590  Inventory 55,000  Unearned finance  income--leases 32,410

Answer (A) is incorrect because it is the lessee’s journal entry.Answer (B) is incorrect because the sale should be recorded at the present value of the minimum lease payments, and theunearned finance income should be recorded as the difference between the gross lease payments receivable and the presentvalue of this gross investment.Answer (C) is incorrect because the lease should reflect both cost of goods sold and sales, not the netted gross profit on thelease.Answer (D) is correct. For this finance lease, the manufacturer- lessor should record

As gross investment, the minimum lease payments because there is no residual value1.As net investment, the difference between the gross investment in the lease (a debit to a receivable) and unearnedfinance income (a credit to a liability)

2.

As unearned finance income, the difference between the gross investment and its present value3.As sales revenue, the fair value of the asset or, if lower, the present value of the minimum lease payments computed atthe interest rate implicit in the lease

4.

As cost of goods sold, the cost of the leased asset5.Because the first payment is made at the inception of the lease, the payment structure is that of an annuity due. Salesrevenue is therefore equal to the 10,000 periodic payment times the present value of an annuity due of 1 discounted for 10years at 10% (10,000 × 6.7590 = 67,590).

Given that cash is paid at the beginning of the year, the initial 10,000 cash debit immediately decreases the gross investmentin the lease (lease payments receivable) from 100,000 to 90,000. The cost of the leased asset (55,000) must also be chargedto cost of sales and credited to inventory. Finally, at the inception of the lease, unearned finance income equals thedifference between the gross investment and the sales price (100,000 – 67,590 = 32,410).

[520] Gleim #: 5.26 -- Source: J.O. Hall

On August 1, Jones leased property to Smith for a 5-year period. The annual 20,000 lease payment is payable at the end ofeach year. The expected residual value at the end of the lease term is 10,000. Jones’s implicit interest rate is 12%. The costof the property to Jones was 50,000, which is the fair value at the lease date. The present value of an ordinary annuity of 1for five periods is 3.605. The present value of 1 at the end of five periods is .567. At the inception of the lease, the recordedgross investment is

Gleim's CIA Test Prep: Part III: Business Analysis and Information Technology(1165 questions)

Copyright 2004 Gleim Publications, Inc. Page 201Printed for g j

Page 202: Part three cia_with_ answers

110,000A.100,000B.72,100C.90,000D.

Answer (A) is correct. For a finance lease, the lessor should record the gross investment in the lease at the undiscountedsum of the minimum lease payments (the total of the lessee’s required payments, excluding contingent rent and costs forservices and taxes to be paid by and reimbursed to the lessor, and any guaranteed residual value) and any unguaranteedresidual value. The gross investment is the same regardless of whether any residual value is guaranteed. The five periodicpayments of 20,000 equal 100,000. The expected residual value including guaranteed and unguaranteed portions equals10,000. The gross investment should be 110,000 (100,000 + 10,000).Answer (B) is incorrect because it fails to include the residual value in the gross investment.Answer (C) is incorrect because the annual lease payments should be recorded at their undiscounted value.Answer (D) is incorrect because the residual value is added to, not subtracted from, the undiscounted lease payments.

[521] Gleim #: 5.27 -- Source: H.F. Bush

KW Ltd. leased equipment under a 4-year, noncancelable lease properly classified as a finance lease. The lease does nottransfer ownership or contain a bargain purchase option. The equipment had an estimated economic life of 5 years and anestimated residual value of 20,000. Terms of the lease included a guaranteed residual value of 50,000. KW initiallyrecorded the leased equipment at 240,000, and its depreciation policy for owned assets is to use the straight-line method.Thus, the amount of depreciation that should be charged each year is

55,000A.47,500B.44,000C.38,000D.

Answer (A) is incorrect because the guaranteed residual value, not the estimated residual value, must be subtractedfrom the amount initially recorded.Answer (B) is correct. Depreciation should be consistent with the accounting policy for owned assets. Absent areasonable certainty that the lessee will own the asset at the end of the lease term, it should be fully depreciated overthe shorter of the useful life or the lease term. The lease does not transfer ownership or contain a bargain purchaseoption. Accordingly, the period of amortization should be the lease term. In accordance with the straight-line methodused by KW for owned assets, the depreciable base for this finance lease is equal to the 240,000 initially recordedminus the 50,000 guaranteed residual value allocated equally over the 4-year lease term. Consequently, annualdepreciation expense is 47,500 [(240,000 – 50,000) ÷ 4 years].Answer (C) is incorrect because the guaranteed residual value, not the estimated residual value, must be subtractedfrom the initially recorded amount, and the term of the lease, not the estimated economic life, is used as thedenominator in the depreciation calculation.Answer (D) is incorrect because the term of the lease, not the estimated economic life, must be used as thedenominator in the depreciation calculation.

Gleim's CIA Test Prep: Part III: Business Analysis and Information Technology(1165 questions)

Copyright 2004 Gleim Publications, Inc. Page 202Printed for g j

Page 203: Part three cia_with_ answers

[Fact Pattern #31]

On January 1, Plantation Partners is planning to enter as thelessee into the two lease agreements described in theopposite column. Each lease is noncancelable, andPlantation does not receive title to either leased propertyduring or at the end of the lease term. All paymentsrequired under these agreements are due on January 1 eachyear.

Lessor Lease A Lease B

Type of property Oven ComputerYearly rental 15,000 4,000Lease term 10 years 3 yearsEconomic life 15 years 5 yearsPurchase option None 3,000Renewal option None NoneFair value at inception

of lease 125,000 10,200Unguaranteed residual value None 2,000Lessee’s incremental

borrowing rate 10% 10%Executory costs paid by Lessee LessorAnnual executory costs 800 500Present value factor at 10%

(of an annuity due) 6.76 2.74

[522] Gleim #: 5.28 -- Source: CMA 1293 2-27

(Refers to Fact Pattern #31)Plantation should treat Lease A as a(n)

Finance lease with an initial asset value of 101,400.A.Operating lease, charging 14,200 in rental expense and 800 in executory costs to annual operations.B.Operating lease, charging the present value of the yearly rental expense to annual operations.C.Operating lease, charging 15,000 in rental expense and 800 in executory costs to annual operations.D.

Answer (A) is incorrect because Lease A does not qualify as a finance lease.Answer (B) is incorrect because rental expense is 15,000.Answer (C) is incorrect because the actual cash outlay for rent, 15,000, is charged to expense.Answer (D) is correct. Lease A is an operating lease with a 15,000 annual rental expense with annual executory costs(e.g., maintenance, insurance, and taxes) of 800 to be paid by the lessee. An operating lease does not transfer the risksand rewards of ownership to the lessee. Lease A is nothing more than a rental arrangement. Circumstances in whichthe risks and rewards of ownership are normally deemed to be transferred include the following:  the lease transferstitle to the lessee, the lease has a bargain purchase option, the lease term is for the major part of the useful life of theleased asset, the present value of the minimum lease payments is at least substantially all of the asset’s fair value, orthe asset is usable only by the lessee without major modification.

[523] Gleim #: 5.29 -- Source: CMA 1293 2-28

(Refers to Fact Pattern #31)Plantation should treat Lease B as a(n)

Finance lease with an initial asset value of 10,960.A.Finance lease with an initial asset value of 10,200.B.Operating lease, charging 3,500 in rental expense and 500 in executory costs to annual operations.C.Finance lease with an initial asset value of 9,590.D.

Gleim's CIA Test Prep: Part III: Business Analysis and Information Technology(1165 questions)

Copyright 2004 Gleim Publications, Inc. Page 203Printed for g j

Page 204: Part three cia_with_ answers

Answer (A) is incorrect because the initial asset value cannot exceed the fair value of the leased asset. Moreover, 10,960includes the present value of the executory costs.Answer (B) is incorrect because 10,200 is the fair value of the leased asset.Answer (C) is incorrect because the lease meets the criteria of a finance lease.Answer (D) is correct. A finance lease is one in which the risks and rewards of ownership are transferred to the lessee. Foraccounting purposes, the lessee treats a finance lease as similar to the purchase of an asset capitalized at the fair value of theleased asset or, if lower, the present value of the minimum lease payments. The lessee’s minimum lease payments includethe required payments, excluding contingent rent and executory costs (e.g., taxes and insurance), plus any amountsguaranteed by the lessee or a related party. If a bargain purchase option exists, however, minimum lease payments equal therequired payments plus the amount of the option. If the present value of the minimum lease payments (calculated withoutguaranteed amounts or a bargain purchase option) is substantially all of the asset’s fair value, the lease normally isaccounted for as a finance lease. Given that the executory costs associated with the lease are to be paid by the lessor, aportion of the lease rental price is for those costs, not for the asset. Consequently, the annual minimum lease payment equalsthe annual payment minus the executory costs, or 3,500 (4,000 yearly rental – 500). The present value of the minimum leasepayments is therefore 9,590 (2.74 × 3,500), which is substantially all of the fair value of the asset. Thus, the lease should becapitalized. The appropriate amount of the initial asset value is the present value of the minimum lease payments calculatedabove.

[524] Gleim #: 5.30 -- Source: CIA 597 IV-24

An enterprise has a high fixed-assets turnover ratio. What conclusion can a financial analyst draw from this?

The enterprise may be overcapitalized.A.The enterprise may have a problem with employees converting inventory to personal use.B.The enterprise may be undercapitalized.C.The enterprise has favorable profitability.D.

Answer (A) is incorrect because the ratio may indicate undercapitalization.Answer (B) is incorrect because fluctuations in inventory do not affect fixed-assets turnover.Answer (C) is correct. The fixed-assets turnover ratio equals net sales divided by net fixed assets. A high ratioindicates either that the enterprise is undercapitalized, that is, it cannot afford to buy enough fixed assets, or that ituses fixed assets efficiently.Answer (D) is incorrect because the fixed-assets turnover ratio is not a profitability indicator. It measures theefficiency of asset management.

[525] Gleim #: 5.31 -- Source: CIA 597 IV-31

The times-interest-earned ratio is primarily an indication of

Solvency.A.Liquidity.B.Asset management.C.Profitability.D.

Answer (A) is correct. The times-interest-earned ratio equals net profit before taxes and interest divided by interest. Itmeasures the extent to which operating profit can decline before the enterprise is unable to meet its annual interestcost. Thus, it is a measure of debt-paying capacity (solvency).Answer (B) is incorrect because liquidity ratios, e.g., the current ratio, indicate the relationship of current assets tocurrent liabilities.Answer (C) is incorrect because asset management ratios indicate how effectively the enterprise is using its assets.Answer (D) is incorrect because profitability ratios measure operating results.

[526] Gleim #: 5.32 -- Source: CIA 597 IV-27

An enterprise’s receivables collection period is equal to

The inventory conversion period.A.The cash conversion cycle.B.The day’s sales outstanding.C.The inventory divided by average daily sales.D.

Gleim's CIA Test Prep: Part III: Business Analysis and Information Technology(1165 questions)

Copyright 2004 Gleim Publications, Inc. Page 204Printed for g j

Page 205: Part three cia_with_ answers

Answer (A) is incorrect because the inventory conversion period (days of inventory) is the average time required to convertmaterials into finished goods and then to sell them. This process typically occurs before the receivables collection period,and the amount of time in one period does not necessarily bear any relationship to the other.Answer (B) is incorrect because the cash conversion cycle equals the inventory conversion period, plus the receivablescollection period, minus the payables deferral period (average time between resource purchases and payment of cash forthem). It estimates the time between when the enterprise makes payments and when it receives cash inflows.Answer (C) is correct. The day’s sales outstanding (days of receivables) may be stated as the accounts receivable balancedivided by average credit sales per day or as days in the year divided by the receivables turnover. It is the average timerequired to convert the enterprise’s receivables into cash. Thus, it is also called the receivables collection period.Answer (D) is incorrect because the inventory divided by the sales per day is the inventory conversion period (days ofinventory).

[527] Gleim #: 5.33 -- Source: CIA 597 IV-35

Which of the following is true about the impact of price inflation on financial ratio analysis?

Inflation has no impact on financial ratio analysis.A.Inflation affects financial ratio analysis for one enterprise over time but not comparative analysis of enterprises ofdifferent ages.

B.

Inflation affects financial ratio analysis for one enterprise over time, as well as comparative analysis of enterprises ofdifferent ages.

C.

Inflation affects comparative analysis of enterprises of different ages but not financial ratio analysis for one enterpriseover time.

D.

Answer (A) is incorrect because inflation badly distorts balance sheets, depreciation charges, inventory costs, andprofits.Answer (B) is incorrect because inflation affects any financial ratio analysis involving comparisons of prior-periodwith current-period monetary amounts.Answer (C) is correct. Inflation is the diminution over time of the purchasing power of money. Because balancesheet amounts are expressed in terms of money, historical cost amounts for different periods are measured in unitsrepresenting different levels of purchasing power. Net profit is also distorted because of inflation’s impact ondepreciation expense and inventory costs. Inflation therefore impairs the comparability of financial statement items,whether for the same enterprise over time or for enterprises of differing ages.Answer (D) is incorrect because inflation affects any financial ratio analysis involving comparisons of prior-periodwith current-period monetary amounts.

[528] Gleim #: 5.34 -- Source: CIA 597 IV-37

An investor has been given several financial ratios for an enterprise but none of the financial reports. Which combination ofratios can be used to derive return on equity?

Market-to-book-value ratio and total-debt-to- total-assets ratio.A.Price-to-earnings ratio, earnings per share, and net profit margin.B.Price-to-earnings ratio and return-on-assets ratio.C.Net profit margin, total assets turnover, and equity multiplier.D.

Answer (A) is incorrect because the market-to-carrying- amount ratio and the total-debt-to-total-assets ratio do notprovide any information about net profit available to shareholders.Answer (B) is incorrect because the price-to- earnings ratio, EPS, and the net profit margin do not provideinformation about the carrying amount of common equity.Answer (C) is incorrect because the price-to-earnings ratio and the return-on-assets ratio do not provide informationabout the carrying amount of common equity.Answer (D) is correct. The net profit margin equals the net profit available to common shareholders divided by sales,the total assets turnover equals sales divided by total assets, and the product of these two ratios is the return on assets.This result is the basic Du Pont equation. In the extended Du Pont equation, the return on assets is multiplied by theleverage factor, also called the equity multiplier (total assets ÷ common equity at carrying amount). The extended DuPont equation gives the return on common equity. This result is obtained because the total assets and sales factorscancel in the multiplication of the three ratios.

Gleim's CIA Test Prep: Part III: Business Analysis and Information Technology(1165 questions)

Copyright 2004 Gleim Publications, Inc. Page 205Printed for g j

Page 206: Part three cia_with_ answers

[529] Gleim #: 5.35 -- Source: CIA 597 IV-38The following ratios relate to an enterprise’s financial situation compared with that of its industry:

  The Industry  Company Average Return on Assets (ROA) 7.9% 9.2%Return on Equity (ROE) 15.2% 12.9%

What conclusion could a financial analyst validly draw from these ratios?

The enterprise’s product has a high market share, leading to higher profitability.A.The enterprise uses more debt than does the average enterprise in the industry.B.The enterprise’s profits are increasing over time.C.The enterprise’s shares have a higher market value to carrying amount than does the rest of the industry.D.

Answer (A) is incorrect because the question gave no information about market share.Answer (B) is correct. The use of financial leverage has a multiplier effect on the return on assets. The extended DuPont formula illustrates this point by showing that the return on equity equals the return on assets times the leveragefactor, also called the equity multiplier (total assets ÷ common equity). Thus, greater use of debt increases the equitymultiplier and the return on equity. In this example, the equity multiplier is 1.92 (15.2% ROE ÷ 7.9% ROA), and theindustry average is 1.40 (12.9% ROE ÷ 9.2% ROA). The higher equity multiplier indicates that the enterprise usesmore debt than the industry average.Answer (C) is incorrect because this comparison is with an industry average, not over time.Answer (D) is incorrect because share valuation is a response to many factors. The higher-than-average return onequity does not mean that the enterprise has a more favorable market-to- carrying-amount ratio.

[530] Gleim #: 5.36 -- Source: CIA 1196 IV-53

All else being equal, an enterprise with a higher dividend-payout ratio will have a <List A> debt-to- assets ratio and a <ListB> current ratio.

  List A List B A. Higher HigherB. Higher LowerC. Lower HigherD. Lower Lower

Answer (A) is incorrect because the current ratio will be lower.Answer (B) is correct. An enterprise with a higher dividend-payout ratio is distributing more of its earnings asdividends to common shareholders. It will have less cash and less total assets than a comparable enterprise with alower payout ratio. The debt-to- assets ratio will be higher because total assets are lower, and the current ratio will belower because cash is lower.Answer (C) is incorrect because the debt-to-assets ratio will be higher and the current ratio will be lower.Answer (D) is incorrect because the debt-to-assets ratio will be higher.

Gleim's CIA Test Prep: Part III: Business Analysis and Information Technology(1165 questions)

Copyright 2004 Gleim Publications, Inc. Page 206Printed for g j

Page 207: Part three cia_with_ answers

[531] Gleim #: 5.37 -- Source: CIA 1193 IV-46The following account balances represent the December 31 balance sheet of an enterprise.

Accounts payable 67,000Accounts receivable (net) 115,000Accumulated depreciation -- building 298,500Accumulated depreciation -- equipment 50,500Cash 27,500Common shares (10 par value) 100,000Deferred income taxes payable 37,500Equipment 136,000Income taxes payable 70,000Inventory 257,000Land and building 752,000Long-term notes payable 123,000Financial assets held for trading 64,000Notes payable within 1 year 54,000Other current liabilities 22,500Share premium 150,000Prepaid expenses 27,000Retained earnings 403,500

The quick ratio for this year is

1.42A.1.08B.0.97C.0.82D.

Answer (A) is incorrect because 1.42 excludes the income taxes payable from the current liabilities.Answer (B) is incorrect because 1.08 includes prepaid expenses in the quick assets and excludes income taxespayable in the current liabilities.Answer (C) is correct. The acid test (quick) ratio equals quick assets (cash, financial assets held for trading, andaccounts receivable) divided by current liabilities. Quick assets total 206,500 (27,500 cash + 64,000 financial assetsheld for trading + 115,000 net accounts receivable). Given current liabilities of 213,500 (67,000 accounts payable +54,000 current notes payable + 70,000 income taxes payable + 22,500 other current liabilities), the quick ratio is0.967 (206,500 ÷ 213,500).Answer (D) is incorrect because 0.82 includes deferred income taxes payable in the current liabilities.

Gleim's CIA Test Prep: Part III: Business Analysis and Information Technology(1165 questions)

Copyright 2004 Gleim Publications, Inc. Page 207Printed for g j

Page 208: Part three cia_with_ answers

[Fact Pattern #32]An enterprise’s financial statements for the current year are presented below:

Balance Sheet

Cash 100Accounts receivable 200Inventory 50Net fixed assets 600 

Total 950 Accounts payable 140Long-term debt 300Share capital 260Retained earnings 250 

Total 950    

Statement of Income and Retained Earnings   Sales 3,000Cost of goods sold 1,600 Gross profit 1,400Operations expenses 970 Operating profit 430Interest expense 30 Profit before tax 400Income tax 200 Net profit 200Plus Jan. 1 retained earnings 150Minus dividends 100 Dec. 31 retained earnings 250 

[532] Gleim #: 5.38 -- Source: CIA 596 IV-36

(Refers to Fact Pattern #32)The enterprise has a dividend-payout ratio of

19.6%A.28.6%B.40.0%C.50.0%D.

Answer (A) is incorrect because 19.6% is the ratio of dividends paid to the December 31 carrying amount of commonequity.Answer (B) is incorrect because 28.6% is the ratio of dividends paid to the sum of beginning retained earnings andnet profit.Answer (C) is incorrect because 40.0% is the ratio of dividends paid to the December 31 retained earnings.Answer (D) is correct. The dividend-payout ratio is the ratio of dividends paid to net profit for the period. Hence, itequals 50.0% (100 dividends ÷ 200 net profit).

[533] Gleim #: 5.39 -- Source: CIA 596 IV-38

(Refers to Fact Pattern #32)The enterprise has return on assets of

Gleim's CIA Test Prep: Part III: Business Analysis and Information Technology(1165 questions)

Copyright 2004 Gleim Publications, Inc. Page 208Printed for g j

Page 209: Part three cia_with_ answers

21.1%A.39.2%B.42.1%C.45.3%D.

Answer (A) is correct. The return on assets is the ratio of net profit to total assets. It equals 21.1% (200 net profit ÷ 950total assets).Answer (B) is incorrect because 39.2% is the ratio of net profit to common equity.Answer (C) is incorrect because 42.1% is the ratio of profit before tax to total assets.Answer (D) is incorrect because 45.3% is the ratio of profit before interest and tax to total assets.

[534] Gleim #: 5.40 -- Source: CIA 596 IV-40

(Refers to Fact Pattern #32)The enterprise has a profit margin of

6.67%A.13.33%B.14.33%C.46.67%D.

Answer (A) is correct. The profit margin is the ratio of net profit to sales. It equals 6.67% (200 net profit ÷ 3,000sales).Answer (B) is incorrect because 13.33% is the ratio of profit before tax to sales.Answer (C) is incorrect because 14.33% is the ratio of profit before interest and taxes to sales.Answer (D) is incorrect because 46.67% is the ratio of gross profit to sales.

[535] Gleim #: 5.41 -- Source: CIA 1195 IV-32

An enterprise has 100,000 outstanding common shares with a market value of 20 per share. Dividends of 2 per share werepaid in the current year, and the enterprise has a dividend-payout ratio of 40%. The price-to-earnings ratio of the enterpriseis

2.5A.4B.10C.50D.

Answer (A) is incorrect because 2.5 equals EPS divided-by dividends per share.Answer (B) is correct. The P-E ratio equals the share price divided by EPS. If the dividends per share equaled 2 andthe dividend- payout ratio was 40%, EPS must have been 5 (2 ÷ .4). Accordingly, the P-E ratio is 4 (20 share price ÷5 EPS).Answer (C) is incorrect because 10 equals share price divided by dividends per share.Answer (D) is incorrect because 50 equals price per share divided by the dividend-payout percentage.

Gleim's CIA Test Prep: Part III: Business Analysis and Information Technology(1165 questions)

Copyright 2004 Gleim Publications, Inc. Page 209Printed for g j

Page 210: Part three cia_with_ answers

[536] Gleim #: 5.42 -- Source: CIA 1192 IV-52The following are the January 1 and June 30 balance sheets of an enterprise:

Assets (in millions) Jan. 1 June 30

Cash 3 4Accounts receivable 5 4Inventories 8 10Fixed assets 10 11 Total assets 26 29 

Accounts payable 2 3Notes payable 4 3Accrued wages 1 2Long-term debt 9 11Equity 10 10 Total liabilities and equity 26 29 

From January 1 to June 30, the net working capital

Decreased by 1 million.A.Stayed the same.B.Increased by 1 million.C.Increased by 2 million.D.

Answer (A) is incorrect because a decrease of 1,000,000 results from omitting inventories.Answer (B) is incorrect because the difference between all assets and all liabilities stayed the same.Answer (C) is correct. Net working capital equals current assets (cash, accounts receivable, inventories for thisenterprise) minus current liabilities (accounts payable, notes payable, accrued wages). From January 1 to June 30, thenet working capital increased by 1,000,000 {[(4 + 4 + 10) – (3 + 3 + 2)] – [(3 + 5 + 8) – (2 + 4 + 1)]}.Answer (D) is incorrect because an increase of 2,000,000 results from omitting accrued wages.

[Fact Pattern #33]

An enterprise has a current ratio of 1.4, a quick, or acid test,ratio of 1.2, and the following partial summary balancesheet:

Cash 10 Current liabilities ___    Long-termAccounts receivable ___ liabilities 40Inventory ___ Equity 30Fixed assets  ___ Total liabilitiesTotal assets 100 and equity ___

[537] Gleim #: 5.43 -- Source: CIA 1196 IV-34

(Refers to Fact Pattern #33)The enterprise has an accounts receivable balance of

12A.26B.36C.66D.

Gleim's CIA Test Prep: Part III: Business Analysis and Information Technology(1165 questions)

Copyright 2004 Gleim Publications, Inc. Page 210Printed for g j

Page 211: Part three cia_with_ answers

Answer (A) is incorrect because 12 equals current assets minus current liabilities.Answer (B) is correct. Total assets equal total liabilities and equity. Hence, if total assets equal 100, total liabilities andequity must equal 100, and current liabilities must equal 30 (100 – 40 – 30). Because the quick ratio equals the quick assets(cash + accounts receivable) divided by current liabilities, the quick assets must equal 36 (30 × 1.2 quick ratio), and theaccounts receivable balance is 26 (36 – 10 cash).Answer (C) is incorrect because 36 equals the quick assets.Answer (D) is incorrect because 66 equals the sum of the quick assets and current liabilities.

[538] Gleim #: 5.44 -- Source: CIA 1196 IV-35

(Refers to Fact Pattern #33)The enterprise has a fixed assets balance of

0A.16B.58C.64D.

Answer (A) is incorrect because the sum of cash, accounts receivable, and inventory is less than 100.Answer (B) is incorrect because 16 is the result of neglecting to subtract the equity balance when calculating thecurrent liability balance.Answer (C) is correct. Total assets (given as 100) equals the sum of cash (given as 10), accounts receivable (26),inventory, and fixed assets. Inventory can be determined because it is included in current, but not quick, assets, andthe current and quick ratios are known. Current assets equal 42 (1.4 current ratio × 30 current liabilities), and thequick assets equal 36 (1.2 quick ratio × 30 current liabilities). Thus, inventory, which is the only difference in thisquestion between current and quick assets, equals 6 (42 – 36). Fixed assets must then equal 58 (100 total assets – 10cash – 26 accounts receivable – 6 inventory).Answer (D) is incorrect because 64 assumes that inventory is 0.

[539] Gleim #: 5.45 -- Source: CIA 596 IV-53

A growing enterprise is assessing current working capital requirements. An average of 58 days is required to convert rawmaterials into finished goods and to sell them. Then an average of 32 days is required to collect on receivables. If theaverage time the enterprise takes to pay for its raw materials is 15 days after they are received, the total cash conversioncycle is

11 days.A.41 days.B.75 days.C.90 days.D.

Answer (A) is incorrect because 11 days results from subtracting the receivables collection period.Answer (B) is incorrect because 41 days results from subtracting the receivables collection period and adding thepayables deferral period.Answer (C) is correct. The cash conversion cycle is the length of time between paying for purchases and receivingcash from the sale of finished goods. It equals the inventory conversion period, plus the receivables collection period,minus the payables deferral period, or 75 days (58 days + 32 days – 15 days).Answer (D) is incorrect because 90 days omits the payables deferral period.

[540] Gleim #: 5.46 -- Source: CIA 596 IV-75

Which one of the following statements best describes the asset-liability method of accounting for deferred income taxes?

The amount of deferred tax is based on tax rates in effect when temporary differences originate.A.The amount of deferred tax is based on the tax rates expected to be in effect during the periods in which the deferredtax liability is settled or the deferred tax asset is realized.

B.

The tax effects of temporary differences are not reported separately but are reported as adjustments to the amounts ofspecific assets and liabilities and the related revenues and expenses.

C.

The appropriate tax rate to be reported on the income statement is the tax actually levied in that year, meaning nodeferred taxes would be reported.

D.

Gleim's CIA Test Prep: Part III: Business Analysis and Information Technology(1165 questions)

Copyright 2004 Gleim Publications, Inc. Page 211Printed for g j

Page 212: Part three cia_with_ answers

Answer (A) is incorrect because this statement describes the deferred method of accounting for deferred income taxes.Answer (B) is correct. A DTA or DTL is measured at the rates expected to apply when it is realized or settled, based on taxlaws and rates enacted or substantively enacted as of the balance sheet date. If different rates apply to different taxable profitlevels, a DTA or DTL is measured based on the average rates expected to apply in the periods when the TDs are expected toreverse. The tax rate or tax base may vary with the manner of recovery or settlement. For example, one tax rate may apply ifan asset is sold immediately and another may apply if it is to be recovered through continued use.Answer (C) is incorrect because this statement describes the net-of-tax method, which recognizes that future taxability anddeductibility are important factors in the valuation of individual assets and liabilities.Answer (D) is incorrect because this statement describes the nonallocation or flow-through approach, which does notsupport the calculation and reporting of deferred income tax.

[541] Gleim #: 5.47 -- Source: CMA 696 2-9

Which one of the following temporary differences will result in a deferred tax asset?

Use of the straight-line depreciation method for determining accounting profit and an accelerated method fordetermining taxable profit (tax loss).

A.

Installment sale profits accounted for on the accrual basis for determining accounting profit and on a cash basis fordetermining taxable profit (tax loss).

B.

Advance rental receipts accounted for on the accrual basis for financial statement purposes and on a cash basis for taxpurposes.

C.

Prepaid expenses accounted for on the accrual basis for determining accounting profit and on a cash basis fordetermining taxable profit (tax loss).

D.

Answer (A) is incorrect because using an accelerated depreciation method for determining taxable profit (tax loss)results in a deferred tax liability.Answer (B) is incorrect because recognizing installment income on the financial statements but not the tax returnresults in a taxable temporary difference.Answer (C) is correct. A deferred tax asset records the deferred tax consequences attributable to deductibletemporary differences and carryforwards. Advance rental receipts accounted for on the accrual basis for determiningaccounting profit and on a cash basis for determining taxable profit (tax loss) would give rise to a deferred tax asset.The financial statements would show no income and no related tax expense because the rental payments apply tofuture periods. The tax return, however, would show the rent as income when the cash was received, and a tax wouldbe due in the year of receipt. Because the tax is paid prior to recording accounting profits, it represents an asset thatwill be recognized as an expense when income is finally recorded.Answer (D) is incorrect because recognizing prepaid expenses earlier on the tax return than on the financialstatements (a situation akin to the accelerated depreciation of fixed assets) gives rise to a deferred tax liability.

[542] Gleim #: 5.48 -- Source: Publisher

At December 31, SCM Ltd., a calendar-year enterprise, reported the following accounts for which the carrying amountdiffered from the tax base:

  Carrying Tax  Amount Base Depreciable assets (net) 150,000 80,000Deferred rental income 40,000 0

What taxable and deductible amounts are related to these temporary differences?

  Taxable Deductible  Amounts Amounts A. 40,000 70,000B. 70,000 40,000C. 0 110,000D. 110,000 0

Gleim's CIA Test Prep: Part III: Business Analysis and Information Technology(1165 questions)

Copyright 2004 Gleim Publications, Inc. Page 212Printed for g j

Page 213: Part three cia_with_ answers

Answer (A) is incorrect because the taxable amount is 70,000 and the deductible amount is 40,000.Answer (B) is correct. The difference between the carrying amount of an asset or liability and its tax base is a temporarydifference (TD). A taxable (deductible) TD results in taxable (deductible) amounts in the future when the carrying amountof the asset or liability is recovered or settled. The tax base is the amount attributed for tax purposes to an asset or liability.The tax base of an asset is the amount deductible against future taxable economic benefits when the asset’s carrying amountis recovered. The tax base of a liability is the portion of the carrying amount that will not be deductible against futuretaxable economic benefits for tax purposes. The tax base of revenue received in advance (a liability) is the portion of thecarrying amount taxable in the future. Thus, the 70,000 temporary difference (150,000 carrying amount – 80,000 tax base)related to the depreciable assets is classified as a taxable amount. When income, such as rental income, is taxable beforebeing recognized in accounting profit, future sacrifices to provide the rental service or refund amounts paid will result infuture tax deductible amounts when the liability is settled. Thus, the 40,000 temporary difference (40,000 carrying amount –0 tax base) related to the deferred rental revenue is classified as a deductible amount.Answer (C) is incorrect because the taxable amount is 70,000 and the deductible amount is 40,000.Answer (D) is incorrect because the taxable amount is 70,000 and the deductible amount is 40,000.

[543] Gleim #: 5.49 -- Source: Publisher

Which of the following results in a tax base of zero?

Trade receivables have a carrying amount of 1,000, and the related revenue has been included in full in thedetermination of taxable profit.

A.

A loan receivable has a carrying amount of 1,000, and repayment has no tax effects.B.Unearned interest revenue has a carrying amount of 1,000, and the related interest revenue was included in full in thedetermination of taxable profit.

C.

Accrued expenses have a carrying amount of 1,000, and the related expense has been included in full in thedetermination of taxable profit.

D.

Answer (A) is incorrect because the tax base is 1,000.Answer (B) is incorrect because the tax base is 1,000.Answer (C) is correct. The difference between the carrying amount of an asset or liability and its tax base is atemporary difference (TD). A taxable (deductible) TD results in taxable (deductible) amounts in the future when thecarrying amount of the asset or liability is recovered or settled. The tax base is the amount attributed for tax purposesto an asset or liability. The tax base of an asset is the amount deductible against future taxable economic benefitswhen the asset’s carrying amount is recovered. The tax base of a liability is the portion of the carrying amount thatwill not be deductible against future taxable economic benefits for tax purposes. The tax base of revenue received inadvance (a liability) is the portion of the carrying amount taxable in the future. For unearned interest revenue forwhich the related interest revenue was taxed on a cash basis, the tax base equals zero (1,000 carrying amount – 1,000not taxable in the future).Answer (D) is incorrect because the tax base is 1,000.

[Fact Pattern #34]An enterprise has purchased an asset with a 10-year useful life. It will use an accelerated depreciation method for determiningtaxable profit or tax loss. For determining accounting profit, it will use straight-line depreciation.

[544] Gleim #: 5.50 -- Source: CIA 1194 IV-69

(Refers to Fact Pattern #34)During the 10-year life of the asset, the enterprise will report as deferred tax an amount that

Increases steadily for the 10 years.A.Is constant.B.Increases and then decreases.C.Decreases and then increases.D.

Answer (A) is incorrect because the deferred tax liability will increase and then decrease.Answer (B) is incorrect because the deferred tax liability will increase and then decrease.Answer (C) is correct. The cumulative deferred tax increases, peaks, and then decreases to zero over the life of theasset. In the early years, the asset is depreciated more quickly for tax purposes than for financial reporting purposes.This temporary difference reverses in later years. Hence, in the early years, actual taxes payable will be less than taxexpense reported in the financial statements, and a deferred tax liability will be recognized. By the end of the asset’suseful life, cumulative actual taxes paid will equal cumulative reported tax expense, so the deferred tax balance willbe zero.Answer (D) is incorrect because the deferred tax liability will increase and then decrease.

Gleim's CIA Test Prep: Part III: Business Analysis and Information Technology(1165 questions)

Copyright 2004 Gleim Publications, Inc. Page 213Printed for g j

Page 214: Part three cia_with_ answers

[545] Gleim #: 5.51 -- Source: CIA 1194 IV-70(Refers to Fact Pattern #34)When determining cash flows accruing to the enterprise, using financial statements prepared for tax purposes will result in

No effect on cash flow amounts.A.An overstatement of cash flows throughout the economic life of the asset.B.An understatement of cash flows throughout the economic life of the asset.C.An overstatement of cash flows in the early years and then an understatement of cash flows in the later years of theeconomic life of the asset.

D.

Answer (A) is correct. Cash flows are not affected by the basis of accounting used to prepare the financialstatements. Accordingly, whether the financial statements are prepared based on the tax basis, the cash basis, oraccounting principles generally accepted in a given country, cash flows should be the same. For example, the cashinflow or outflow resulting from using an accelerated depreciation method to determine actual tax expense or benefit(the amount paid to or refunded by the taxing authority) is completely unaffected by the depreciation method used inthe financial statements. However, if cash flows are derived indirectly by adjusting net profit or loss reported in thefinancial statements, different adjustments are necessary to arrive at the same cash flow amounts if different bases ofaccounting are used in the preparation of the financial statements.Answer (B) is incorrect because cash flows are not affected by the method of depreciation used for reportingpurposes.Answer (C) is incorrect because cash flows are not affected by the method of depreciation used for reportingpurposes.Answer (D) is incorrect because cash flows are not affected by the method of depreciation used for reportingpurposes.

[546] Gleim #: 5.52 -- Source: Publisher

Based on its current operating levels, Glucose Plc estimates that its annual level of taxable profit in the foreseeable futurewill be 200,000 annually. Enacted tax rates for the tax jurisdiction in which Glucose operates are 15% for the first 50,000of taxable profit, 25% for the next 50,000 of taxable profit, and 35% for taxable profit in excess of 100,000. Which tax rateshould Glucose use to measure a deferred tax liability or asset?

15%A.25%B.27.5%C.35%D.

Answer (A) is incorrect because 15% is the tax rate for the first 50,000 of taxable profit.Answer (B) is incorrect because 25% is the tax rate for taxable profit over 50,000 but less than 100,000.Answer (C) is correct. In measuring a deferred tax liability or asset, the objective is to use the enacted orsubstantively enacted tax rate(s) expected to apply to taxable profit in the periods in which the deferred tax liability orasset is expected to be settled or realized. If different rates apply to different taxable profit levels, a DTA or DTL ismeasured based on the average rates expected to apply in the periods when the TDs are expected to reverse.Accordingly, the applicable tax rate is 27.5%.

Taxable TaxProfit Rate50,000 × 15% 7,50050,000 × 25% 12,500

100,000 × 35% 35,000200,000 55,000

55,000 ÷ 200,000 = 27.5%

Answer (D) is incorrect because 35% is the tax rate for taxable profit over 100,000.

[547] Gleim #: 5.53 -- Source: Publisher

When an enterprise reports deferred tax assets and liabilities, deferred tax expense or income should be disclosed equal tothe

Decrease in the deferred tax assets.A.Sum of the net changes in deferred tax assets and deferred tax liabilities.B.Increase in the deferred tax liabilities.C.Amount of the total tax liability.D.

Gleim's CIA Test Prep: Part III: Business Analysis and Information Technology(1165 questions)

Copyright 2004 Gleim Publications, Inc. Page 214Printed for g j

Page 215: Part three cia_with_ answers

Answer (A) is incorrect because the deferred tax expense or income is equal to the sum of the net changes in the deferredtax assets and deferred tax liabilities.Answer (B) is correct. The deferred tax expense or income disclosed is the sum of the net changes in the deferred tax assetsand deferred tax liabilities. This amount is the deferred tax expense or income relating to the origination or reversal oftemporary differences. For example, the reduction in a deferred tax asset or an increase in a deferred tax liability increasesdeferred tax expense.Answer (C) is incorrect because the deferred tax expense or income is equal to the sum of the net changes in the deferredtax assets and deferred tax liabilities.Answer (D) is incorrect because the total tax liability includes both the current and deferred tax expense or income for theyear.

[548] Gleim #: 5.54 -- Source: Publisher

On December 31, year 1, Health Enterprises reported a 150,000 warranty expense in its income statement. The expense wasbased on actual warranty costs of 30,000 in year 1 and expected warranty costs of 35,000 in year 2, 40,000 in year 3, and45,000 in year 4. For tax purposes, warranty costs are not deductible until paid. At December 31, year 1, deferred taxesshould be based on a

120,000 deductible temporary difference.A.150,000 deductible temporary difference.B.120,000 taxable temporary difference.C.150,000 taxable temporary difference.D.

Answer (A) is correct. At year-end year 1, Health Enterprises should report a 120,000 warranty liability in its balancesheet. The warranty liability is equal to the 150,000 warranty expense minus the 30,000 warranty cost actuallyincurred in year 1. Because warranty costs are not deductible until paid, the tax base of the warranty liability is 0. Theresult is a 120,000 temporary difference (120,000 carrying amount – 0 tax base). When the liability is settled throughthe actual incurrence of warranty costs, the amounts will be deductible. Thus, the temporary difference should beclassified as a deductible temporary difference.Answer (B) is incorrect because 150,000 equals the warranty expense, not the payable.Answer (C) is incorrect because warranty costs will result in a deductible amount.Answer (D) is incorrect because the warranty costs will result in a deductible amount, and the 30,000 actual warrantycosts is currently deductible.

[549] Gleim #: 5.55 -- Source: CPA 595 F-32

On September 22, 20X2, Yumi Corp. purchased merchandise from an unaffiliated foreign company for 10,000 units of theforeign company’s local currency. On that date, the spot rate was $.55. Yumi paid the bill in full on March 20, 20X3, whenthe spot rate was $.65. The spot rate was $.70 on December 31, 20X2. What amount should Yumi report as a foreigncurrency transaction loss in its income statement for the year ended December 31, 20X2?

$0A.$500B.$1,000C.$1,500D.

Answer (A) is incorrect because a loss resulted when the spot rate increased.Answer (B) is incorrect because $500 results from using the spot rates at 12/31/X2 and 3/20/X3.Answer (C) is incorrect because $1,000 results from using the spot rates at 9/22/X2 and 3/20/X3.Answer (D) is correct. SFAS 52 requires that a receivable or payable denominated in a foreign currency be adjustedto its current exchange rate at each balance sheet date. The resulting gain or loss should ordinarily be included indetermining net income. It is the difference between the spot rate on the date the transaction originates and the spotrate at year-end. Thus, the 20X2 transaction loss for Yumi Corp. is $1,500 [($0.55 – $0.70) × 10,000 units].

[550] Gleim #: 5.56 -- Source: Publisher

XYZ Company purchased $10,000 of British pounds and $25,000 of land in euros on June 1 of the current year. At the endof XYZ Company’s fiscal year on December 31, the spot rate for the British pounds was $12,000. The land had a value of$27,500 in euros. XYZ Company records its nonmonetary items at historical cost. Which of the following is the correctpresentation of this transaction on XYZ Company’s balance sheet?

Gleim's CIA Test Prep: Part III: Business Analysis and Information Technology(1165 questions)

Copyright 2004 Gleim Publications, Inc. Page 215Printed for g j

Page 216: Part three cia_with_ answers

  British Pounds Land A. $10,000 $25,000B. $10,000 $27,500C. $12,000 $25,000D. $12,000 $27,500

Answer (A) is incorrect because the British pounds are reported at the spot rate on the date of the balance sheet.Answer (B) is incorrect because the British pounds are reported at the spot rate on the balance sheet date and the land isvalued at the date of purchase.Answer (C) is correct. At the balance sheet date, monetary items are reported at the closing rate which is often the spot rateon the balance sheet date. Nonmonetary items measured at historical cost are reported at the rate on the transaction date.Therefore, the British pounds are reported at $12,000 and the land is reported at $25,000.Answer (D) is incorrect because the land is reported based upon the historical cost on the purchase date.

[551] Gleim #: 5.57 -- Source: Publisher

Which of the following is true with regard to joint ventures?

The establishment of a separate financial structure is necessary when joint venturers own joint assets.A.Two venturers are not allowed to transfer assets and liabilities in a given line of business to a jointly controlled entity.B.When a jointly controlled entity becomes a subsidiary of the venturer, consolidation is required.C.An investor is a party to the venture who has joint control.D.

Answer (A) is incorrect because jointly controlled assets do not require the establishment of an organization orfinancial structure separate from the venturers.Answer (B) is incorrect because the venturers may establish a separate entity in which each has an interest and thatoperates in the same manner as other entities if they choose to have a jointly controlled entity.Answer (C) is correct. A joint venture “is a contractual arrangement whereby two or more parties undertake aneconomic activity that is subject to joint control.”  When a jointly controlled entity becomes a subsidiary of theventurer, consolidation is required.Answer (D) is incorrect because an investor is a party to the venture who lacks joint control.

[552] Gleim #: 5.58 -- Source: CPA 1192 T-33

To effect a business combination initiated on July 1, 2004, Proper Co. acquired all the outstanding common shares ofScapula Co. for cash equal to the carrying amount of Scapula’s net assets. The carrying amounts of Scapula’s assets andliabilities approximated their fair values, except that the carrying amount of its building was more than fair value. Inpreparing Proper’s December 31, 2004 consolidated income statement, what is the effect of recording the assets acquiredand liabilities assumed at fair value and should goodwill amortization be recognized?

  Depreciation Goodwill  Expense Amortization A. Lower YesB. Higher YesC. Lower NoD. Higher No

Answer (A) is incorrect because depreciation will decrease, and goodwill will be recognized but not amortized.Answer (B) is incorrect because depreciation will decrease, and goodwill will be recognized but not amortized.Answer (C) is correct. A business combination initiated after June 30, 2001 is accounted for as a purchase regardlessof the form of consideration given. Under purchase accounting, assets acquired and liabilities assumed should berecorded at their fair values. The differences between fair values and carrying amounts will affect net income whenrelated expenses are incurred. The effect of recording the building at fair value in the consolidated balance sheetinstead of its higher carrying amount on Scapula’s books will be to decrease future depreciation. If the building is tobe used, fair value is its current replacement cost for similar capacity unless expected use indicates a lower value tothe acquirer. If the building is to be sold, it should be reported at fair value minus cost to sell. The excess of the costover fair value of the net assets acquired will be recognized as goodwill, but, under SFAS 142, this amount will betested for impairment but not amortized.Answer (D) is incorrect because depreciation will decrease, and goodwill will be recognized but not amortized.

Gleim's CIA Test Prep: Part III: Business Analysis and Information Technology(1165 questions)

Copyright 2004 Gleim Publications, Inc. Page 216Printed for g j

Page 217: Part three cia_with_ answers

[553] Gleim #: 5.59 -- Source: CPA 593 II-8Pellew Corp. paid $600,000 for the outstanding common stock of Samos Co. in a business combination initiated andcompleted in December 2004. At that time, Samos had the following condensed balance sheet:

  Carrying  Amounts Current assets $ 80,000Plant and equipment, net 760,000Liabilities 400,000Equity 440,000

The fair value of the plant and equipment was $120,000 more than its carrying amount. The fair values and carryingamounts were equal for all other assets and liabilities. What amount of goodwill, related to Samos’s acquisition, shouldPellew report in its consolidated balance sheet?

$40,000A.$80,000B.$120,000C.$160,000D.

Answer (A) is correct. A business combination initiated after June 30, 2001 is accounted for as a purchase regardlessof the form of the consideration given. Under purchase accounting, assets acquired and liabilities assumed should berecorded at their fair values. Any excess of cost over the fair value of the net assets acquired is recorded as goodwill.After adjusting the net plant and equipment, and given that other items are stated at fair value, the fair value of the netassets acquired is $560,000 [$80,000 current assets + ($760,000 + $120,000) plant and equipment – $400,000liabilities]. Hence, goodwill is $40,000 ($600,000 cost – $560,000).Answer (B) is incorrect because $80,000 is the amount of current assets.Answer (C) is incorrect because $120,000 is the amount plant and equipment is undervalued.Answer (D) is incorrect because $160,000 is the difference between the $600,000 cost and the $440,000 carryingamount of the net assets.

[554] Gleim #: 5.60 -- Source: CPA 590 T-33

In a business combination, the sum of the amounts assigned by the acquiring entity to assets acquired and liabilitiesassumed exceeds the cost of the acquired entity. The excess should be reported as a

Deferred credit.A.Reduction of the amounts assigned to current assets and a deferred credit for any unallocated portion.B.Reduction of the amounts assigned to certain acquired assets and an extraordinary gain for any unallocated portion.C.Pro rata reduction of the amounts assigned to all acquired assets and an extraordinary gain for any unallocatedportion.

D.

Answer (A) is incorrect because a deferred credit is never recognized for the excess of the fair value of acquired netassets over cost.Answer (B) is incorrect because the allocated portion is reported as an extraordinary gain, not as a deferred credit.Answer (C) is correct. In a business combination, any excess of the fair value assigned to the net assets acquired overthe cost of the purchase must be allocated proportionately to reduce the amounts otherwise assignable to all of theacquired assets except (a) financial assets (excluding equity-method investments), (b) assets to be disposed of by sale,(c) deferred tax assets, (d) prepaid assets relating to post-retirement benefit plans, and (e) other current assets. Anyremainder after the amounts otherwise assignable to those assets have been reduced to zero is reported as anextraordinary gain (SFAS 141).Answer (D) is incorrect because the amounts assigned to certain acquired assets (most financial assets, assets to bedisposed of by sale, etc.) are not reduced.

Gleim's CIA Test Prep: Part III: Business Analysis and Information Technology(1165 questions)

Copyright 2004 Gleim Publications, Inc. Page 217Printed for g j

Page 218: Part three cia_with_ answers

[555] Gleim #: 5.61 -- Source: CIA 1193 IV-36A service enterprise’s working capital at the beginning of January was 70,000. The following transactions occurred duringJanuary:

Performed services on account 30,000Purchased supplies on account 5,000Consumed supplies 4,000Purchased office equipment for cash 2,000Paid short-term bank loan 6,500Paid salaries 10,000Accrued salaries 3,500

What is the amount of working capital at the end of January?

80,500A.78,500B.50,500C.47,500D.

Answer (A) is correct. Working capital is the excess of total current assets (CA) over total current liabilities (CL).Thus, working capital at the end of January equals 80,500 computed as follows:

    CA* CL*Beginning working capital 70,000Performed services on account 30,000 I NPurchased supplies on account -0- I IConsumed supplies (4,000) D NPurchased office equipment (2,000) D NPaid short-term bank loan -0- D DPaid salaries (10,000) D NAccrued salaries (3,500) N I Working capital, end of January 80,500 

* N = no effect; I = increase; D = decrease

Answer (B) is incorrect because 78,500 ignores the consumed supplies, the cash purchase of office equipment, andthe accrued salaries. However, it incorrectly considers the supplies purchased on account and the repayment of theshort-term bank loan.Answer (C) is incorrect because 50,500 does not include the services performed on account.Answer (D) is incorrect because 47,500 omits the services performed on account and accrued salaries but includes therepayment of short-term loan.

[556] Gleim #: 5.62 -- Source: CIA 593 IV-39

Which of the following financial statement analyses is most useful in determining whether the various expenses of a givenenterprise are higher or lower than industry averages?

Horizontal.A.Vertical.B.Activity ratio.C.Defensive-interval ratio.D.

Answer (A) is incorrect because a horizontal analysis indicates the proportionate change over a period of time and isuseful in trend analysis of an individual enterprise.Answer (B) is correct. Vertical analysis is the expression of each item on a financial statement in a given period inrelation to a base figure. On the income statement, each item is stated as a percentage of sales. Thus, the percentagesfor the enterprise in question can be compared with industry norms.Answer (C) is incorrect because activity ratio analysis includes the preparation of turnover ratios such as those forreceivables, inventory, and total assets.Answer (D) is incorrect because the defensive-interval ratio is part of a liquidity analysis.

Gleim's CIA Test Prep: Part III: Business Analysis and Information Technology(1165 questions)

Copyright 2004 Gleim Publications, Inc. Page 218Printed for g j

Page 219: Part three cia_with_ answers

[Fact Pattern #35]An enterprise reports the following account balances at year-end:

Account BalanceLong-term debt 200,000Cash 50,000Net sales 600,000Fixed assets (net) 320,000Tax expense 67,500Inventory 25,000Common shares 100,000Interest expense 20,000Administrative expense 35,000Retained earnings 150,000Accounts payable 65,000Accounts receivable 120,000Cost of goods sold 400,000Depreciation expense 10,000

Additional Information:

The opening balance of common shares was 100,000.The opening balance of retained earnings was 82,500.The enterprise had 10,000 common shares outstanding all year.No dividends were paid during the year.

[557] Gleim #: 5.63 -- Source: CIA 594 IV-12

(Refers to Fact Pattern #35)For the year just ended, the enterprise has times-interest-earned of:

3.375 times.A.6.75 times.B.7.75 times.C.9.5 times.D.

Answer (A) is incorrect because 3.375 times results from including in the numerator deductions for taxes and interest.Answer (B) is incorrect because 6.75 times results from including in the numerator a deduction for interest.Answer (C) is correct. The TIE ratio is a leverage ratio. It emphasizes the ability to pay interest expense. The ratioequals profit before interest and taxes divided by interest.

Answer (D) is incorrect because 9.5 times results from failing to deduct the administrative expenses from thenumerator.

[558] Gleim #: 5.64 -- Source: CIA 594 IV-13

(Refers to Fact Pattern #35)At year-end, the enterprise has a carrying amount per share of

10.00A.15.00B.21.63C.25.00D.

Gleim's CIA Test Prep: Part III: Business Analysis and Information Technology(1165 questions)

Copyright 2004 Gleim Publications, Inc. Page 219Printed for g j

Page 220: Part three cia_with_ answers

Answer (A) is incorrect because 10.00 excludes retained earnings from the numerator.Answer (B) is incorrect because 15.00 excludes common shares from the numerator.Answer (C) is incorrect because 21.63 is based on average equity.Answer (D) is correct. Carrying amount per share, based on balance sheet amounts, measures the per share amount thatwould be received if the enterprise were liquidated. The ratio is calculated as common equity divided by the number ofoutstanding shares.

[559] Gleim #: 5.65 -- Source: CIA 594 IV-14

(Refers to Fact Pattern #35)For the year just ended, the enterprise had a rate of return on equity, rounded to two decimals, of:

31.21%A.58.06%B.67.50%C.71.68%D.

Answer (A) is correct. Rate of return on equity, a profitability ratio, measures the rate of return on investment. Theratio equals net profit divided by average equity.

Answer (B) is incorrect because 58.06% excludes common shares from the denominator.Answer (C) is incorrect because 67.50% excludes retained earnings from the denominator.Answer (D) is incorrect because 71.68% excludes interest expense and tax expense from the numerator.

[560] Gleim #: 5.66 -- Source: CIA 594 IV-15

(Refers to Fact Pattern #35)Suppose that an enterprise has paid one of its liabilities twice during the year, in error. The effects of this mistake would be

Assets, liabilities, and equity are understated.A.Assets, net income, and equity are unaffected.B.Assets and liabilities are understated.C.Assets and net profit and equity are understated, and liabilities are overstated.D.

Gleim's CIA Test Prep: Part III: Business Analysis and Information Technology(1165 questions)

Copyright 2004 Gleim Publications, Inc. Page 220Printed for g j

Page 221: Part three cia_with_ answers

Answer (A) is incorrect because the double payment of a liability does not affect expenses of the period so it does not affectnet profit and equity.Answer (B) is incorrect because assets will be reduced.Answer (C) is correct. When a liability is paid, an entry debiting accounts payable and crediting cash is made. If anenterprise erroneously pays a liability twice, the accounts payable and cash accounts will be understated by the amount ofthe liability. Hence, assets and liabilities will be understated.Answer (D) is incorrect because both assets and liabilities will be understated, whereas net profit and equity will beunaffected.

[561] Gleim #: 5.67 -- Source: CIA 1191 IV-45

The present value of future benefits payable as a result of work done before the introduction of, or amendment to,postemployment benefits is the

Additional minimum liability.A.Fair value of plan assets.B.Defined benefit obligation.C.Past service cost.D.

Answer (A) is incorrect because the IASs provide for no additional minimum liability.Answer (B) is incorrect because plan assets are assets held by a separate legal entity (fund) to be used to settle theemployee benefit obligation.Answer (C) is incorrect because the defined benefit obligation (DBO) consists of future amounts required to settle theobligation arising from services provided by employees in the current and prior periods.Answer (D) is correct. Past service cost is the increase in the present value of the DBO related to prior employeeservice that arises in the current period from the introduction of, or an amendment to, postemployment benefits. Pastservice cost is expensed on a straight-line basis over the average period until vesting. To the extent it is vested uponintroduction of, or amendment to, a plan, past service cost is immediately recognized.

[562] Gleim #: 5.68 -- Source: CIA 593 IV-36

An audit of an enterprise records for its first year of operations determined that the following errors were made at thebalance sheet date:

Failed to accrue 50,000 interest expense.Failed to record depreciation expense on office equipment of 80,000.Failed to amortize prepaid rent expense of $100,000.Failed to defer recognition of prepaid advertising expense of 60,000.

The net effect of these errors was to overstate net profit by

130,000A.170,000B.230,000C.290,000D.

Gleim's CIA Test Prep: Part III: Business Analysis and Information Technology(1165 questions)

Copyright 2004 Gleim Publications, Inc. Page 221Printed for g j

Page 222: Part three cia_with_ answers

Answer (A) is incorrect because 130,000 ignores the errors related to prepaid rent and prepaid advertising expense.Answer (B) is correct. The computation is as follows:

Error Effect on Expense Effect on Net ProfitFailure to accrue

interest expense Understate 50,000 Overstate 50,000Failure to record

depreciation Understate 80,000 Overstate 80,000Failure to amortize

prepaid rent expense Understate 100,000 Overstate 100,000Failure to recognize

prepaid advertising Overstate 60,000 Understate 60,000 

Totals Understate 170,000 Overstate 170,000 

Answer (C) is incorrect because 230,000 ignores the error related to prepaid advertising expense.Answer (D) is incorrect because 290,000 treats the error related to prepaid advertising expense as an understatement ofexpenses.

[Fact Pattern #36]An audit of an enterprise has revealed the following four errors that have occurred but have not been corrected:

Inventory at December 31, 2003:  40,000, understated1.Inventory at December 31, 2004:  15,000, overstated2.Depreciation for 2003:  7,000, understated3.Accrued expenses at December 31, 2004:  10,000, understated4.

[563] Gleim #: 5.69 -- Source: CIA 591 IV-45

(Refers to Fact Pattern #36)The errors cause the reported net profit for the year ending December 31, 2004 to be

Overstated by 72,000.A.Overstated by 65,000.B.Understated by 28,000.C.Understated by 45,000.D.

Answer (A) is incorrect because the net effect of these errors was a 65,000 overstatement.Answer (B) is correct. The effect of the understatement of the 2003 year-end inventory (beginning inventory for2004) was to overstate 2004 net profit by 40,000. The reason is that beginning inventory is a component of cost ofsales. The overstatement of the December 31, 2004 inventory overstated 2004 net income by 15,000 because theamounts in ending inventory are excluded from cost of sales. The understatement of 2003 depreciation expense (anominal account) has no effect on 2004 net income. Finally, the failure to accrue 10,000 of expenses for 2004overstated 2004 net income. The net effect of these errors was a 65,000 (40,000 + 15,000 + 0 + 10,000)overstatement.Answer (C) is incorrect because the net effect of these errors was a 65,000 overstatement.Answer (D) is incorrect because the net effect of these errors was a 65,000 overstatement.

[564] Gleim #: 5.70 -- Source: CIA 591 IV-46

(Refers to Fact Pattern #36)The errors cause the reported retained earnings at December 31, 2004 to be

Overstated by 65,000.A.Overstated by 32,000.B.Overstated by 25,000.C.Understated by 18,000.D.

Gleim's CIA Test Prep: Part III: Business Analysis and Information Technology(1165 questions)

Copyright 2004 Gleim Publications, Inc. Page 222Printed for g j

Page 223: Part three cia_with_ answers

Answer (A) is incorrect because the net effect of the errors was a 32,000 overstatement.Answer (B) is correct. The 2003 inventory error reversed in 2004 (excluding tax considerations) and therefore had no effecton reported retained earnings at December 31, 2004. The 15,000 inventory error at year-end 2004 and the failure to accrue10,000 of expenses for 2004 both overstated retained earnings as well as 2004 net profit. The omission of 7,000 ofdepreciation overstated 2003 net income and 2003 and 2004 retained earnings. Hence, the net effect of the errors onDecember 31, 2004 retained earnings was a 32,000 (0 + 15,000 + 7,000 + 10,000) overstatement.Answer (C) is incorrect because the net effect of the errors was a 32,000 overstatement.Answer (D) is incorrect because the net effect of the errors was a 32,000 overstatement.

[565] Gleim #: 5.71 -- Source: CMA 690 3-1

For a finance lease, the gross investment, lease payments receivable, recorded by the lessor is equal to the

Present value of the minimum lease payments minus the unguaranteed residual value accruing to the lessor at the endof the lease term.

A.

Lower of 90% of the present value of the minimum lease payments or the fair value of the leased asset.B.Minimum lease payments plus the unguaranteed residual value accruing to the lessor at the end of the lease term.C.Difference between the present value of the minimum lease payments and the unearned finance income.D.

Answer (A) is incorrect because the gross investment is not adjusted for the time value of money or fair value.Answer (B) is incorrect because the gross investment is not adjusted for the time value of money or fair value.Answer (C) is correct. For a finance lease, the lessor should record as the gross investment in the lease the amount ofthe minimum lease payments (payments plus either any bargain purchase option or any residual value guaranteed bythe lessee, a party related to the lessee, or by a financially capable party unrelated to the lessor or the lessee) plus anyamounts of unguaranteed residual value. The net investment in the lease is equal to the gross investment, minusunearned finance income.Answer (D) is incorrect because the gross investment is not adjusted for the time value of money or fair value.

[566] Gleim #: 5.72 -- Source: CMA 690 3-2

Initial direct costs incurred by the lessor under a finance lease may be

Allocated over the economic life of the leased asset.A.Expensed in the period incurred.B.Deferred and allocated over the term of the lease in proportion to the recognition of rental income.C.Subtracted from the gross investment in the lease.D.

Answer (A) is incorrect because initial direct costs of a finance lease may be allocated over the lease term orrecognized immediately.Answer (B) is correct. The initial direct costs of entering into a finance lease, e.g., commissions and legal fees, maybe recognized immediately or allocated over the lease term. One possibility is to recognize the expense as incurredwhile also recognizing as income in the same period an equal amount of the unearned finance income.Answer (C) is incorrect because initial direct costs of an operating lease may be deferred and allocated over the termof the lease in proportion to the recognition of rental income.Answer (D) is incorrect because initial direct costs of a finance lease are recognized immediately in income orallocated over the lease term.

[567] Gleim #: 5.73 -- Source: CIA 589 IV-41

A new asset with a fair value of 100,000 and a useful life of 10 years is being leased by an enterprise (lessee). Which of thesituations below will most likely result in classifying the lease on the lessee’s books as a finance lease?

The lease term is 7 years.A.The lease term is 8 years.B.The present value of the minimum lease payments is 60,000.C.The present value of the minimum lease payments is 70,000.D.

Gleim's CIA Test Prep: Part III: Business Analysis and Information Technology(1165 questions)

Copyright 2004 Gleim Publications, Inc. Page 223Printed for g j

Page 224: Part three cia_with_ answers

Answer (A) is incorrect because a 7-year lease term is less likely to be for the major part of the economic life of the assetthan an 8-year lease term.Answer (B) is correct. A lease may be classified as either a finance lease or an operating lease by a lessee. A finance leasetransfers substantially all of the risks and rewards of ownership of the asset to the lessee. A lease is classified at itsinception. It normally is classified as a financial lease if, for example,

The lease provides for the transfer of ownership of the asset by the end of the lease term.1.The lease contains a bargain purchase option; i.e., the lessee has the option to purchase at a price expected to besufficiently below the fair value of the exercise date that, at the lease’s inception, exercise is reasonably certain.

2.

The lease term is for the major part of the economic life of the asset.3.The present value of the minimum lease payments is at least substantially all of the fair value of the leased asset at theinception of the lease.

4.

The leased assets are such that they can be used only by the lessee without major modification.5.

Thus, a lease term that is 80% (8 ÷ 10) of the economic life of the asset will most likely result in classifying the lease as afinance lease, assuming that 80% is a “major part” of the economic life of the asset.Answer (C) is incorrect because a present value of the minimum lease payments of only 60,000 is not substantially all of thefair value of the leased asset.Answer (D) is incorrect because a present value of the minimum lease payments of only 70,000 is not substantially all of thefair value of the leased asset.

[568] Gleim #: 5.74 -- Source: CMA 1291 2-27

On December 31, 2002, Johnson sold on account and shipped merchandise with a list price of 75,000 to Gibsen. The termsof the sale were n/30, FOB shipping point. The merchandise arrived at Gibsen on January 5, 2004. Because of confusionabout the shipping terms, the sale was not recorded until January of 2004 and the merchandise, sold at a markup of 25% ofcost, was included in Johnson’s inventory on December 31, 2003. Johnson uses a periodic inventory system. As a result ofthe above, Johnson’s profit before income taxes for the year ended December 31, 2003 was

Understated by 15,000.A.Understated by 75,000.B.Understated by 18,750.C.Overstated by 60,000.D.

Answer (A) is correct. The shipping term was FOB shipping point. Hence, title to the goods and risk of loss passedto the buyer on December 31, 2003, and the 75,000 sale should have been recorded on that date. Given a selling priceof 75,000 and a markup on cost of 25%, cost must have been 60,000 (75,000 ÷ 1.25), and profit before taxes musthave been 15,000 (75,000 – 60,000). Because the sale was unrecorded, the seller’s balance sheet reflected inventoryof 60,000 instead of an account receivable of 75,000. Thus, assets were understated by 15,000. Also, profit wasunderstated by 15,000 because of the failure to credit sales for 75,000 and debit cost of goods sold for 60,000.Answer (B) is incorrect because income was understated by 15,000.Answer (C) is incorrect because income was understated by 15,000.Answer (D) is incorrect because income was understated by 15,000.

[569] Gleim #: 5.75 -- Source: CIA 592 IV-52

Finance leases are differentiated from operating leases in that finance leases normally

Provide for maintenance service at no extra cost to the lessee.A.Are cancelable at any time upon request by the lessee.B.Have rental payments that are approximately equal to the amortized cost of the leased asset.C.Are used only by financial institutions.D.

Gleim's CIA Test Prep: Part III: Business Analysis and Information Technology(1165 questions)

Copyright 2004 Gleim Publications, Inc. Page 224Printed for g j

Page 225: Part three cia_with_ answers

Answer (A) is incorrect because finance leases do not normally provide for maintenance service.Answer (B) is incorrect because finance leases are not normally cancelable.Answer (C) is correct. A lease may be classified as either a finance lease or an operating lease by a lessee. A finance leasetransfers substantially all of the risks and rewards of ownership of the asset to the lessee. A lease is classified at itsinception. It normally is classified as a finance lease if, for example,

The lease provides for the transfer of ownership of the asset by the end of the lease term.1.The lease contains a bargain purchase option; i.e., the lessee has the option to purchase at a price expected to besufficiently below the fair value of the exercise date that, at the lease’s inception, exercise is reasonably certain.

2.

The lease term is for the major part of the economic life of the asset.3.The present value of the minimum lease payments is at least substantially all of the fair value of the leased asset at theinception of the lease.

4.

The leased assets are such that they can be used only by the lessee without major modification.5.

Thus, the rental payments tend to approximate the cost of the leased property plus a return on the investment.Answer (D) is incorrect because finance leases are used by enterprises in various industries.

[570] Gleim #: 5.76 -- Source: CMA 1293 2-10

For the past three years, Colbeth Enterprises has failed to accrue unpaid wages earned by workers during the last week ofthe year. The amounts omitted, which are considered material, were as follows.

December 31, 2001 56,000December 31, 2002 51,000December 31, 2003 64,000

The entry on December 31, 2003 to correct for these omissions includes a

Credit to wage expense for 64,000.A.Debit to wage expense for 51,000.B.Debit to wage expense for 13,000.C.Credit to retained earnings for 64,000.D.

Answer (A) is incorrect because 64,000 is the total accrued wages payable, not the amount of the adjustment.Answer (B) is incorrect because 51,000 was the correct wage accrual for 2002.Answer (C) is correct. Failing to record accrued wages is a self-correcting error. Expenses are understated in oneyear and overstated in the next, resulting in the correction of the error over the 2-year period. The 2001 erroroverstated 2001 earnings and understated 2002 earnings by 56,000. Consequently, no correction is necessary for the2001 error. The 2002 error overstated 2002 profit before taxes and understated 2003 profit before taxes by 51,000.The 2003 error overstated 2003 profit before taxes by 64,000. Thus, the net effect in 2003 of the 2002 and 2003errors is a 13,000 (64,000 – 51,000) overstatement. The correcting entry is to debit expense for 13,000, debit retainedearnings for 51,000, and credit wages payable for 64,000.Answer (D) is incorrect because retained earnings should be debited because of the overstatement of 2002 income.

Gleim's CIA Test Prep: Part III: Business Analysis and Information Technology(1165 questions)

Copyright 2004 Gleim Publications, Inc. Page 225Printed for g j

Page 226: Part three cia_with_ answers

[Fact Pattern #37]During the year ended December 31, 2003, an enterprise had 500,000 in sales revenue and purchased 150,000 of inventory. Thecost of goods sold was 250,000 for the year, and the enterprise incurred 25,000 of general and administrative expenses. TheJanuary 1, 2003 opening balance sheet was as follows:

Cash 120,000Accounts receivable 100,000Inventory 200,000Fixed assets (gross) 600,000

Accum. depreciation (100,000)Net fixed assets 500,000 Total assets 920,000 Accounts payable 220,000Long-term debt 400,000Common shares 100,000Retained earnings 200,000 Total liabilities

and equity 920,000 

The cost of long-term debt financing is 10% per annum, payable in one installment on December 31 of each year.

The enterprise has a 50% tax rate.

The enterprise has a dividend payout ratio of 25%.

The fixed assets were 1-year old at the start of the current year, were originally estimated to have a 6-year useful life, and arebeing depreciated on the straight-line basis.

[571] Gleim #: 5.77 -- Source: CIA 1194 IV-1

(Refers to Fact Pattern #37)The enterprise has a closing inventory balance of

100,000A.250,000B.300,000C.350,000D.

Answer (A) is correct. Ending inventory equals beginning inventory, plus purchases, minus cost of goods sold. Thus,ending inventory is 100,000 (200,000 + 150,000 – 250,000).Answer (B) is incorrect because 250,000 is the cost of goods sold.Answer (C) is incorrect because 300,000 results from adding cost of goods sold to beginning inventory andsubtracting purchases.Answer (D) is incorrect because 350,000 equals beginning inventory plus purchases.

[572] Gleim #: 5.78 -- Source: CIA 1194 IV-2

(Refers to Fact Pattern #37)If the enterprise had purchased a larger amount of inventory during the year, with the additional items being purchased onDecember 31, 2003, what is the effect on cost of goods sold and on net profit for the year?

  Cost of Goods Sold Net Profit A. Increase DecreaseB. Increase IncreaseC. Decrease IncreaseD. No effect No effect

Gleim's CIA Test Prep: Part III: Business Analysis and Information Technology(1165 questions)

Copyright 2004 Gleim Publications, Inc. Page 226Printed for g j

Page 227: Part three cia_with_ answers

Answer (A) is incorrect because cost of goods sold and net profits are unaffected.Answer (B) is incorrect because cost of goods sold and net profits are unaffected.Answer (C) is incorrect because cost of goods sold and net profits are unaffected.Answer (D) is correct. Cost of goods sold equals beginning inventory, plus purchases, minus ending inventory. If apurchase is made on December 31, purchases and ending inventory should increase by the same amount. Thus, cost ofgoods sold is unaffected. Because cost of goods sold is unaffected, net profit is also unaffected.

[573] Gleim #: 5.79 -- Source: CIA 1194 IV-3

(Refers to Fact Pattern #37)The enterprise will pay dividends for the year of

10,625A.21,250B.23,125C.42,500D.

Answer (A) is correct. The enterprise has a dividend payout ratio of 25% of net profit. Profit before taxes is 85,000[500,000 sales – 250,000 CGS – 25,000 G&A expenses – (10% interest rate × 400,000 LT debt) – (600,000 fixedassets ÷ 6 years)]. Hence, after-tax net profit is 42,500 [(1.0 – .5) × 85,000], and the dividend payout is 10,625 (25%× 42,500).Answer (B) is incorrect because 21,250 uses a 50% dividend payout ratio instead of 25%.Answer (C) is incorrect because 23,125 calculates net profit without subtracting the 100,000 depreciation expense.Answer (D) is incorrect because 42,500 is the after-tax net profit.

[574] Gleim #: 5.80 -- Source: CIA 1194 IV-6

(Refers to Fact Pattern #37)What would be the effect of a lower tax rate on the ending balance of common shares and on dividends paid for the year?

  Ending Balance  of Common Shares Dividends Paid A. Increase IncreaseB. Decrease DecreaseC. No effect IncreaseD. No effect Decrease

Answer (A) is incorrect because the common shares account is not affected, but dividends will increase.Answer (B) is incorrect because the common shares account is not affected, but dividends will increase.Answer (C) is correct. The balance in the common shares account represents share capital, i.e., capital contributed byowners, not earnings retained in the business. It is not affected by a change in tax rate. Dividends will increase,however, because they are a constant percentage of after-tax earnings.Answer (D) is incorrect because the common shares account is not affected, but dividends will increase.

[575] Gleim #: 5.81 -- Source: CIA 1194 IV-22

Which of the following accurately describes how fundamental errors are accounted for under the benchmark treatment?

They are included in the determination of net profit or loss for the current period.A.Prior-period financial statements are adjusted and reissued.B.They are charged or credited directly to the opening balance of retained earnings.C.They are charged or credited directly to the closing balance of retained earnings.D.

Gleim's CIA Test Prep: Part III: Business Analysis and Information Technology(1165 questions)

Copyright 2004 Gleim Publications, Inc. Page 227Printed for g j

Page 228: Part three cia_with_ answers

Answer (A) is incorrect because, under the benchmark treatment, fundamental errors are excluded from the determination ofcurrent period net profit or loss.Answer (B) is incorrect because prior-period financial statements need not be reissued. However, if comparative statementsare presented, the prior-period statements must be restated if it is practicable to do so.Answer (C) is correct. According to the benchmark treatment, the correction of a fundamental error is recognized as anadjustment of the opening balance of retained earnings to the extent it affects prior periods. Comparative information shouldbe restated if practicable. Furthermore, national law may require the amendment of statements filed with regulators orapproved by shareholders. Thus, under the benchmark treatment, the amount of the correction of a fundamental error thatpertains to a given prior period is included in net profit or loss for that period, provided that restated financial statements forthe period are presented comparatively. The amount of the correction related to periods prior to those for which restatedinformation is presented comparatively is an adjustment of the opening balance of retained earnings for the earliest periodpresented.Answer (D) is incorrect because, under the benchmark treatment, fundamental errors are adjustments of the opening balanceof retained earnings.

[576] Gleim #: 5.82 -- Source: CIA 1194 IV-40

A change in an accounting estimate is shown on the income statement

Only in the relevant account.A.In a separate section entitled extraordinary items.B.In a separate section after continuing operations but before extraordinary items.C.As a cumulative-effect adjustment.D.

Answer (A) is correct. A change in an accounting estimate is accounted for prospectively and is shown on theincome statement only in the relevant account. The effect of the change is included in the same income statementclassification as the previous estimate. However, prior-period statements and opening balances are not adjusted.Answer (B) is incorrect because changes in an accounting estimate are not extraordinary, that is, clearly distinct fromthe ordinary activities of the enterprise.Answer (C) is incorrect because changes in accounting estimates are shown only in the relevant accounts. The effectof the change is included in the same income statement classification as the previous estimate.Answer (D) is incorrect because a change in an accounting estimate is only shown in the relevant account. The effectof the change is included in the same income statement classification as the previous estimate. No cumulative effectof the change is recognized because estimates are accounted for prospectively.

[577] Gleim #: 5.83 -- Source: CIA 595 IV-26

An enterprise has 100 of current assets, 100 of fixed assets, 100 of debt, and 100 of equity. If it then leases a 75 asset ina(n) <List A> lease agreement, the debt-to-assets ratio of the enterprise will be <List B>

  List A List B A. Operating 64%B. Operating 00%C. Finance 50%D. Finance 64%

Answer (A) is incorrect because 64% is the debt-to-assets ratio if the lease is a finance lease.Answer (B) is incorrect because 100% is the debt-to-equity ratio if the lease is an operating lease. Operating leasesare treated as rental agreements, and neither the asset nor the obligation for future payments is recorded on thebalance sheet. The balance of debt remains at 100, so the debt-to-equity ratio is 100% (100 ÷ 100).Answer (C) is incorrect because 50% is the debt-to-assets ratio if the lease is an operating lease.Answer (D) is correct. If the lease is a finance lease, both the asset and the obligation for future payments arerecorded. The enterprise would have current assets of 100, debt of 175 (100 + 75), fixed assets of 175 (100 + 75), andequity of 100. Hence, the debt-to-assets ratio would be 64% (175 ÷ 275).

[578] Gleim #: 5.84 -- Source: CIA 1195 IV-1

If certain goods owned by an enterprise were not recorded as a purchase and were not counted in ending inventory, in error,then

Gleim's CIA Test Prep: Part III: Business Analysis and Information Technology(1165 questions)

Copyright 2004 Gleim Publications, Inc. Page 228Printed for g j

Page 229: Part three cia_with_ answers

Cost of goods sold for the period will be understated.A.Cost of goods sold for the period will be overstated.B.Net profit for the period will be understated.C.There will be no effect on cost of goods sold or net profit for the period.D.

Answer (A) is incorrect because the errors offset each other, so cost of goods sold is not understated or overstated for theperiod.Answer (B) is incorrect because the errors offset each other, so cost of goods sold is not understated or overstated for theperiod.Answer (C) is incorrect because cost of goods sold is unaffected by the error, so net profit is also unaffected.Answer (D) is correct. The effects of the errors on cost of goods sold are offsetting. Purchases, which increases cost ofgoods sold, and ending inventory, which decreases cost of goods sold, are understated by the same amount. Neither cost ofgoods sold nor net profit is affected.

[Fact Pattern #38]At the end of the accounting period, an enterprise has the partially completed financial statements shown as follows.

Income Statement:SalesCost of goods soldGross profitAdministrative expense 40,000Depreciation expenseProfit before interest and taxInterest expenseProfit before taxTax expenseNet profit 100,000

Balance Sheet:Current assetsOther assets 300,000Total assets

Current liabilitiesLong-term debt 1,000,000EquityTotal liabilities and equity

Additional Information:Gross profit percentage = 30%Tax rate = 40%Annual interest cost on long-term debt = 10%Return on total assets = 3%Fixed assets originally cost 500,000 and are being depreciated

over 10 years on a straight-line basis.Working capital = 2,000,000

[579] Gleim #: 5.85 -- Source: CIA 1195 IV-12

(Refers to Fact Pattern #38)The enterprise had sales for the period of

891,667A.1,100,000B.1,188,890C.1,466,667D.

Gleim's CIA Test Prep: Part III: Business Analysis and Information Technology(1165 questions)

Copyright 2004 Gleim Publications, Inc. Page 229Printed for g j

Page 230: Part three cia_with_ answers

Answer (A) is incorrect because 891,667 uses a gross profit percentage of 40%.Answer (B) is incorrect because 1,100,000 calculates profit before tax as net profit divided by the tax rate and alsocalculates sales as gross profit divided by the tax rate.Answer (C) is correct. Gross profit equals 30% of sales. Thus, dividing gross profit by 30% yields sales. Gross profit equalsprofit before interest and tax (PBIT) plus administrative expense (given as 40,000) and depreciation (500,000 fixed assets ÷10 years = 50,000). Interest equals 10% of the long-term debt, or 100,000 (10% × 1,000,000). Profit before tax equals166,667 [100,000 net profit ÷ (1.0 – 40% tax rate)]. Hence, PBIT must equal 266,667 (166,667 + 100,000), and gross profitmust be 356,667 (266,667 PBIT + 40,000 administrative expense + 50,000 depreciation). Sales is therefore 1,118,890(356,667 gross profit ÷ 30% gross profit rate).Answer (D) is incorrect because 1,466,667 calculates profit before tax as net profit divided by the tax rate.

[580] Gleim #: 5.86 -- Source: CIA 1195 IV-13

(Refers to Fact Pattern #38)The enterprise has current assets of

1,000,000A.3,033,333B.3,333,333C.3,633,333D.

Answer (A) is incorrect because 1,000,000 equals working capital minus long-term debt.Answer (B) is correct. Return on total assets (3%) equals net profit (100,000) divided by total assets. Consequently,total assets equals 3,333,333 (100,000 ÷ .03). Current assets must therefore equal 3,033,333 (3,333,333 total assets –300,000 other assets).Answer (C) is incorrect because 3,333,333 is the amount of total assets.Answer (D) is incorrect because 3,633,333 is the sum of total and other assets.

[581] Gleim #: 5.87 -- Source: CIA 1195 IV-25

Because changes in accounting estimates relate to changes in circumstances in the [List A] period, they should be reported[List B].

  List A List B A. Current Not at allB. Current ProspectivelyC. Prior RetroactivelyD. Prior Not at all

Answer (A) is incorrect because changes in accounting estimates should be reported.Answer (B) is correct. Changes in accounting estimates arise as new events occur, as more experience is obtained, oras additional evidence is acquired. A change should be reported in the period in which it occurs, as well asprospectively if future periods are affected. Retroactive reporting is impracticable because it would result in continualadjustments of prior years’ financial statements.Answer (C) is incorrect because changes in accounting estimates arise from changes in current, not prior,circumstances.Answer (D) is incorrect because changes in accounting estimates arise from changes in current, not prior,circumstances.

[582] Gleim #: 5.88 -- Source: CIA 1195 IV-26

Under a defined contribution postemployment benefit plan, a(n) [List A] is reported on the balance sheet only if the amountthe employer has contributed to the pension trust fund is [List B] the amount required.

  List A List B A. Asset Greater thanB. Asset Equal toC. Liability Greater thanD. Liability Equal to

Gleim's CIA Test Prep: Part III: Business Analysis and Information Technology(1165 questions)

Copyright 2004 Gleim Publications, Inc. Page 230Printed for g j

Page 231: Part three cia_with_ answers

Answer (A) is correct. Under a defined contribution plan, the employer recognizes an expense and a liability for thecontribution payable in exchange for an employee’s services performed during the period. The amount is determined aftersubtracting any contribution already made. However, if the contribution made exceeds the amount due, the excess is treatedas a prepaid expense.Answer (B) is incorrect because an asset is reported only if the contribution is in excess of the amount required.Answer (C) is incorrect because the employer reports a liability only if the contribution is less than the required amount.Answer (D) is incorrect because the employer reports a liability only if the contribution is less than the required amount.

[Fact Pattern #39]Enterprise A has 50,000 common shares and 10,000 preferred shares outstanding at the start of the year on January 1. Thepreferred shares are entitled to a 2 per share annual cash dividend, payable on December 31. The enterprise had net profit of150,000 for the year. On April 1, the enterprise issued 15,000 additional common shares for cash.

Enterprise B is identical to Enterprise A in all respects except that it had 75,000 common shares outstanding for the entire year.

[583] Gleim #: 5.89 -- Source: CIA 1195 IV-30

(Refers to Fact Pattern #39)Basic earnings per share for Enterprise B is approximately

1.73A.1.87B.2.00C.2.27D.

Answer (A) is correct. BEPS equals net profit minus preference dividends, divided by common shares outstanding.Thus, BEPS equals 1.73 {[150,000 – (2 × 10,000 preferred shares)] ÷ 75,000}.Answer (B) is incorrect because 1.87 assumes the preference dividend is 1.Answer (C) is incorrect because 2.00 does not subtract preference dividends from net profit.Answer (D) is incorrect because 2.27 adds the preference dividends to net profit.

[Fact Pattern #40]

An enterprise has a current ratio of 1.4, a quick, or acid test,ratio of 1.2, and the following partial summary balancesheet:

Cash 10 Current liabilities ___    Long-termAccounts receivable ___ liabilities 40Inventory ___ Equity 30Fixed assets  ___ Total liabilitiesTotal assets 100 and equity ___

[584] Gleim #: 5.90 -- Source: CIA 594 IV-33

(Refers to Fact Pattern #40)The enterprise has an accounts receivable balance of

12A.26B.36C.66D.

Answer (A) is incorrect because 12 equals current assets minus current liabilities.Answer (B) is correct. Total assets equal total liabilities and equity. Hence, if total assets equal 100, total liabilitiesand equity must equal 100, and current liabilities must equal 30 (100 – 40 – 30). Because the quick ratio equals thequick assets (cash + accounts receivable) divided by current liabilities, the quick assets must equal 36 (30 × 1.2 quickratio), and the accounts receivable balance is 26 (36 – 10 cash).Answer (C) is incorrect because 36 equals the quick assets.Answer (D) is incorrect because 66 equals the sum of the quick assets and current liabilities.

Gleim's CIA Test Prep: Part III: Business Analysis and Information Technology(1165 questions)

Copyright 2004 Gleim Publications, Inc. Page 231Printed for g j

Page 232: Part three cia_with_ answers

[585] Gleim #: 5.91 -- Source: CIA 594 IV-34(Refers to Fact Pattern #40)The enterprise has a fixed assets balance of

0A.16B.58C.64D.

Answer (A) is incorrect because the sum of cash, accounts receivable, and inventory is less than 100.Answer (B) is incorrect because 16 is the result of neglecting to subtract the equity balance when calculating thecurrent liability balance.Answer (C) is correct. Total assets (given as 100) equals the sum of cash (given as 10), accounts receivable (26 ascalculated in the preceding question), inventory, and fixed assets. Inventory can be determined because it is includedin current, but not quick, assets, and the current and quick ratios are known. Current assets equal 42 (1.4 current ratio× 30 current liabilities), and the quick assets equal 36 (1.2 quick ratio × 30 current liabilities). Thus, inventory, whichis the only difference in this question between current and quick assets, equals 6 (42 – 36). Fixed assets must thenequal 58 (100 total assets – 10 cash – 26 accounts receivable – 6 inventory).Answer (D) is incorrect because 64 assumes that inventory is 0.

Gleim's CIA Test Prep: Part III: Business Analysis and Information Technology(1165 questions)

Copyright 2004 Gleim Publications, Inc. Page 232Printed for g j

Page 233: Part three cia_with_ answers

[Fact Pattern #41]

RST Enterprises:  Income Stmt. for Yr. Five and Six

Year Six Year FiveSales (all are credit) 285,000 200,000Cost of goods sold 150,000 120,000Gross profit 135,000 80,000S and A exp 65,000 36,000Profit bef. int. & inc. tax 70,000 44,000Interest expense 3,000 3,000Profit before income tax 67,000 41,000Income tax expense 27,000 16,000Net profit 40,000 25,000

RST Enterprises:  Balance Sheets End of Yr. Five and Six

Assets Year Six Year FiveCurrent assets

Cash 5,000 4,000Financial assets held

for trading 3,000 2,000Accounts receivable (net) 16,000 14,000Inventory 30,000 20,000Total current assets 54,000 40,000

Noncurrent assets:Long-term investments 11,000 11,000PP&E 80,000 70,000Intangibles 3,000 4,000Total assets 148,000 125,000

Liabilities and EquityCurrent liabilities:

Accounts payable 11,000 7,000Accrued payables 1,000 1,000Total current liabilities 12,000 8,000

Bonds payable, 10%, due Yr 12 30,000 30,000Total liabilities 42,000 38,000

Equity:Common shares, 2,400 shares,

10 par 24,000 24,000Retained earnings 82,000 63,000Total equity 106,000 87,000

Total liabilities and equity 148,000 125,000

The market value of RST’s common stock at the end of Year Six was $100.00 per share.

[586] Gleim #: 5.92 -- Source: CIA 592 IV-40

(Refers to Fact Pattern #41)RST’s current ratio at the end of Year Six is

4.5 to 1A.2.4 to 1B.2.0 to 1C.1.5 to 1D.

Gleim's CIA Test Prep: Part III: Business Analysis and Information Technology(1165 questions)

Copyright 2004 Gleim Publications, Inc. Page 233Printed for g j

Page 234: Part three cia_with_ answers

Answer (A) is correct. The current ratio equals current assets divided by current liabilities. At the end of Year Six, it was4.5 (54,000 ÷ 12,000).Answer (B) is incorrect because 2.4 was derived by dividing current liabilities (12,000) by the amount of cash (5,000),which produces no meaningful ratio.Answer (C) is incorrect because 2.0 was derived by dividing the total of cash, financial assets held for trading, and netreceivables (the quick assets) by total current liabilities. This computation determines the acid-test or quick ratio [(5,000 +3,000 + 16,000) ÷ 12,000].Answer (D) is incorrect because 1.5 [12,000 ÷ (5,000 + 3,000)] was derived by dividing total current liabilities by the sumof cash and financial assets held for trading, which produces no meaningful ratio.

[587] Gleim #: 5.93 -- Source: CIA 592 IV-41

(Refers to Fact Pattern #41)RST’s acid-test (or quick) ratio at the end of Year Six is

2.40 to 1A.2.18 to 1B.2.00 to 1C.1.50 to 1D.

Answer (A) is incorrect because 2.4 was derived by dividing total current liabilities (12,000) by the amount of cash(5,000), which produces no meaningful ratio.Answer (B) is incorrect because 2.18 was derived by dividing quick assets (5,000 + 3,000 + 16,000) by accountspayable (11,000). The denominator should include all current liabilities other than accounts payable.Answer (C) is correct. Liquidity ratios measure the ability to meet short-term obligations. A commonly used liquidityratio is the acid-test or quick ratio, which equals the sum of the quick assets (net accounts receivable, financial assetsheld for trading, and cash) divided by current liabilities. This ratio at the end of Year Six is 2.0 [(5,000 + 3,000 +16,000) ÷ 12,000].Answer (D) is incorrect because 1.5 was derived by dividing total current liabilities (12,000) by the sum of cash(5,000) and financial assets held for trading (3,000), which produces no meaningful ratio.

[588] Gleim #: 5.94 -- Source: CIA 592 IV-42

(Refers to Fact Pattern #41)Based on a comparison of RST’s quick ratios in Year Five and Year Six, what is a likely conclusion?

RST has improved its management of long-term investments in Year Six.A.RST has written off obsolete inventory in Year Six.B.RST’s ability to meet short-term financing needs has declined since Year Five.C.RST’s ability to meet short-term financing needs has improved since Year Five.D.

Answer (A) is incorrect because the quick ratio compares current assets (minus inventory) with current liabilities; itdoes not provide a basis for conclusions about long-term investments.Answer (B) is incorrect because the quick ratio compares current assets (minus inventory) with current liabilities; itdoes not provide a basis for determining the effects or the existence of inventory write-offs.Answer (C) is correct. RST’s quick ratio decreased from 2.5 in Year Five [(4,000 cash + 2,000 financial assets heldfor trading + 14,000 net A/R) ÷ 8,000] to 2.0 in Year Six [(5,000 + 3,000 + 16,000) ÷ 12,000]. RST has fewer assetsthat are easily convertible to cash available to meet current liabilities. Thus, its ability to meet short-term financingneeds has declined.Answer (D) is incorrect because RST is less liquid.

[589] Gleim #: 5.95 -- Source: CIA 592 IV-43

(Refers to Fact Pattern #41)RST’s accounts receivable turnover for Year Six is

19 times.A.16 times.B.10 times.C.6 times.D.

Gleim's CIA Test Prep: Part III: Business Analysis and Information Technology(1165 questions)

Copyright 2004 Gleim Publications, Inc. Page 234Printed for g j

Page 235: Part three cia_with_ answers

Answer (A) is correct. The accounts receivable turnover equals net credit sales divided by average trade receivables (net).In Year Six, the accounts receivable turned over 19 times {285,000 ÷ [(16,000 ending A/R + 14,000 beginning A/R) ÷ 2]}.Answer (B) is incorrect because 16 times was derived by dividing average sales of Years Five and Six [(285,000 + 200,000)÷ 2] by average receivables (15,000), which produces no meaningful ratio.Answer (C) is incorrect because 10 times was derived by dividing cost of goods sold (150,000) for Year Six by averageaccounts receivable (15,000), which produces no meaningful ratio.Answer (D) is incorrect because 6 times is the inventory turnover ratio which equals the cost of goods sold for Year Six(150,000) divided by average inventory (25,000).

[590] Gleim #: 5.96 -- Source: CIA 592 IV-45

(Refers to Fact Pattern #41)Based on a comparison of RST’s times-interest-earned ratios in Year Five and Year Six, what is a likely conclusion?

RST’s long-run solvency has declined.A.RST’s long-run solvency has improved.B.RST’s liquidity has improved.C.RST’s liquidity has declined.D.

Answer (A) is incorrect because long-run solvency has improved.Answer (B) is correct. The times-interest-earned ratio increased from 14.67 times in Year Five (44,000 ÷ 3,000) to23.33 times in Year Six (70,000 ÷ 3,000). This increase signifies that RST has more profit available to meet theinterest payments on its debt, and that long-run solvency has improved. Consequently, creditors will view RST’sobligations as less risky.Answer (C) is incorrect because liquidity is a firm’s short-term solvency, that is, its ability to meet current liabilitieswith current assets by converting assets to cash. It is not related to the times-interest-earned ratio, which comparesprofit before interest and taxes with interest expense associated with long-term investments.Answer (D) is incorrect because the times-interest-earned ratio is not a liquidity measure.

Gleim's CIA Test Prep: Part III: Business Analysis and Information Technology(1165 questions)

Copyright 2004 Gleim Publications, Inc. Page 235Printed for g j

Page 236: Part three cia_with_ answers

[Fact Pattern #42]An enterprise’s balance sheet is presented in Table A. The enterprise has 175,000 common shares outstanding.

The following results of operations for the year ended December 31, 2003:

Sales 15,000,000Cost of goods sold 11,500,000Net operating profit 1,750,000Net profit 1,000,000

Table A -- Balance Sheet, 2003(in thousands)

AssetsCash 450Financial assets held for trading 50Accounts receivable 650Inventories 1,350Total current assets 2,500Gross plant and equipment 7,000Minus:  depreciation 2,000Net plant and equipment 5,000Goodwill 500

Total assets 8,000

Liabilities and EquityAccounts payable 250Notes payable 550Accrued wages 100Accrued taxes 200Total current liabilities 1,100

Long-term debt 3,500

Common shares 1,000Share premium 900Retained earnings 1,500Total equity 3,400

Total liabilities and equity 8,000

[591] Gleim #: 5.97 -- Source: CIA 592 IV-58

(Refers to Fact Pattern #42)Based on the information provided, the return on operating assets is

21.88%A.35.00%B.23.33%C.12.50%D.

Answer (A) is incorrect because 21.88% equals net operating profit divided by total assets.Answer (B) is incorrect because 35.00% equals net operating profit divided by net plant and equipment.Answer (C) is correct. Net operating assets equal total assets minus goodwill, or 7,500,000. The return on netoperating assets is therefore 23.33% (1,750,000 net operating profit ÷ 7,500,000 net operating assets).Answer (D) is incorrect because 12.50% equals net profit divided by total assets.

Gleim's CIA Test Prep: Part III: Business Analysis and Information Technology(1165 questions)

Copyright 2004 Gleim Publications, Inc. Page 236Printed for g j

Page 237: Part three cia_with_ answers

[Fact Pattern #43]

EXHIBIT ABalance Sheet

  2001Assets (millions of dollars)Cash 300Financial assets held for trading 400Accounts receivable 200Inventories 400Net plant and equipment 1,700 Total assets 3,000 

Liabilities and EquityAccounts payable 300Notes payable 400Accrued wages 50Accrued taxes 150Debentures 600Common shares 500Retained earnings 1,000 Total liabilities and equity 3,000 

[592] Gleim #: 5.98 -- Source: CIA 1193 IV-58

(Refers to Fact Pattern #43)Based on the balance sheet shown in EXHIBIT A, the debt ratio is

20%A.40%B.50%C.100%D.

Answer (A) is incorrect because 20% excludes short-term debt.Answer (B) is incorrect because 40% divides long-term debt by total equity.Answer (C) is correct. The debt ratio equals total debt divided by total assets, or .50 [(3,000,000,000 total debt andequity – 500,000,000 common shares – 1,000,000,000 retained earnings) ÷ 3,000,000,000 total assets].Answer (D) is incorrect because 100% divides total debt by total equity.

[593] Gleim #: 5.99 -- Source: CIA 1193 IV-59

(Refers to Fact Pattern #43)An enterprise experienced sales of 5 billion, profit before interest and taxes of 200 million, profit before taxes of 100million, and net profit of 70 million. Based on this information and the enterprise’s balance sheet information in Exhibit A,the return on equity (ROE) is

4.67%A.6.67%B.13.33%C.14%D.

Answer (A) is correct. The return on equity equals net profit (70,000,000) divided by equity (500,000,000 commonshares + 1,000,000,000 retained earnings).Answer (B) is incorrect because 6.67% uses profit before taxes instead of net profit.Answer (C) is incorrect because 13.33% uses profit before interest and taxes instead of net profit.Answer (D) is incorrect because 14% divides net profit by common shares.

Gleim's CIA Test Prep: Part III: Business Analysis and Information Technology(1165 questions)

Copyright 2004 Gleim Publications, Inc. Page 237Printed for g j

Page 238: Part three cia_with_ answers

[594] Gleim #: 5.100 -- Source: CIA 590 IV-59Which of the outcomes represented in the following table would result from an enterprise’s retirement of debt with excesscash?

  Following  Total Assets Period's Interest-  Turnover Ratio Earned Ratio A. Increase IncreaseB. Increase DecreaseC. Decrease IncreaseD. Decrease Decrease

Answer (A) is correct. Because total assets will decline without any impact on sales, the total assets turnover ratio(sales ÷ total assets) will increase. In addition, a reduced debt level should cause a reduction in annual interestpayments, so the times-interest-earned ratio [(net profit + interest + taxes) ÷ interest] should increase.Answer (B) is incorrect because the times-interest-earned ratio will increase.Answer (C) is incorrect because the total assets turnover ratio will increase.Answer (D) is incorrect because the total assets turnover ratio will increase and the times-interest-earned ratio willincrease.

[595] Gleim #: 5.101 -- Source: CIA 1188 IV-60

Heilman Plc purchased a new machine for 500,000 by borrowing the required funds from a bank for 180 days. The directimpact of this transaction will be to

Decrease the current ratio and increase the debt ratio.A.Increase the current ratio and decrease the debt ratio.B.Increase the current ratio and increase the debt ratio.C.Decrease the current ratio and decrease the debt ratio.D.

Answer (A) is correct. The borrowing of funds for 180 days constitutes short-term borrowing. The new machine is afixed asset. Current liabilities have increased, and current assets have remained constant. Consequently, the currentratio (current assets ÷ current liabilities) has decreased. Total debt and total assets increased by the same absoluteamount, and the debt ratio (total debt ÷ total assets) should have increased, assuming total debt is less than total asset.Answer (B) is incorrect because the current ratio decreased and the debt ratio increased.Answer (C) is incorrect because the current ratio decreased and the debt ratio increased.Answer (D) is incorrect because the current ratio decreased and the debt ratio increased.

[596] Gleim #: 5.102 -- Source: CMA 688 1-20

Which one of the following ratios would provide the best measure of liquidity?

Sales minus returns to total debt.A.Total assets minus goodwill to total equity.B.Current assets minus inventories to current liabilities.C.Net profit minus dividends to interest expense.D.

Answer (A) is incorrect because it does not refer to expenses that have to be paid out of the sales dollars. Also, thesales revenue available to pay liabilities is indeterminable. Thus, sales minus debt service is not a liquidity measure.Answer (B) is incorrect because fixed assets are included in the numerator of the ratio; hence, this ratio is not ameasure of liquidity.Answer (C) is correct. Liquidity is the degree to which assets can be converted to cash in the short run to meetmaturing obligations. The usual measures of liquidity are the current ratio and the quick (acid-test) ratio. The quickratio is the best measure of short-term liquidity because it uses only the most liquid assets (for example, cash,financial assets held for trading, and receivables) in the calculation; inventories are not included because they are twosteps away from cash (they have to be sold, and then the receivable has to be collected).Answer (D) is incorrect because it shows only the amount of profit available to pay interest expense and not otherobligations. Also, net profit is not a measure of liquidity because it includes items that did not result in an increase ofliquid assets.

Gleim's CIA Test Prep: Part III: Business Analysis and Information Technology(1165 questions)

Copyright 2004 Gleim Publications, Inc. Page 238Printed for g j

Page 239: Part three cia_with_ answers

[597] Gleim #: 5.103 -- Source: CMA 1290 1-30Lawson Ltd. has the opportunity to increase annual sales 100,000 by selling to a new, riskier group of customers. Based onsales, the uncollectible expense is expected to be 15%, and collection costs will be 5%. Manufacturing and selling expensesare 70% of sales, and the effective tax rate is 40%. If Lawson accepts this opportunity, the after-tax profit will increase by

4,000.A.6,000.B.10,000.C.9,000.D.

Answer (A) is incorrect because after-tax profit will increase by 6,000.Answer (B) is correct. Manufacturing and selling costs exclusive of bad debts equal 70% of sales. Hence, the grossprofit on the 100,000 increase in sales will be 30,000 (30% × 100,000). Assuming 15,000 of bad debts and 5,000 ofcollection expense, the increase in pre-tax profit will be 10,000 (30,000 – 20,000). Consequently, after-tax profit willincrease by 6,000 [10,000 – (40% × 10,000)].Answer (C) is incorrect because after-tax profit will increase by 6,000.Answer (D) is incorrect because after-tax profit will increase by 6,000.

[598] Gleim #: 5.104 -- Source: CMA 691 1-4

Which group of ratios would be useful in evaluating the effectiveness of working capital management?

Profit margin, acid-test ratio, and return on assets.A.Acid-test ratio, inventory turnover ratio, and average collection period ratio.B.Inventory turnover ratio, times interest earned, and debt-to-equity ratio.C.Acid-test ratio, current ratio, and return on equity.D.

Answer (A) is incorrect because profit margin and return on assets are measures of the management of all assets.Answer (B) is correct. Working capital equals current assets minus current liabilities. The acid-test ratio equals quickassets (current assets – inventory – prepaid expenses) divided by current liabilities. Inventory (a current asset)turnover equals cost of goods sold divided by average inventory. The average collection period (number of days’sales in accounts receivable, a current asset) equals the number of days in a year divided by the accounts receivableturnover (net credit sales ÷ average receivables). The foregoing are some of the many ratios that can be used toevaluate working capital management.Answer (C) is incorrect because times interest earned and the debt-to-equity ratio are measures of capital structuremanagement.Answer (D) is incorrect because return on equity is a measure of capital structure management.

[599] Gleim #: 5.105 -- Source: CMA 692 1-27

The following information applies to Brandon Enterprises.

  Purchases Sales January 160,000 100,000February 160,000 200,000March 160,000 240,000April 140,000 300,000May 140,000 260,000June 120,000 240,000

A cash payment equal to 40% of purchases is made at the time of purchase, and 30% is paid in each of the next 2 months.Purchases for the previous November and December were 150,000 per month. Payroll is 10% of sales in the month itoccurs, and operating expenses are 20% of the following month’s sales (July sales were 220,000). Interest payments were20,000 paid quarterly in January and April. Brandon’s cash disbursements for the month of April were

140,000.A.152,000.B.200,000.C.254,000.D.

Gleim's CIA Test Prep: Part III: Business Analysis and Information Technology(1165 questions)

Copyright 2004 Gleim Publications, Inc. Page 239Printed for g j

Page 240: Part three cia_with_ answers

Answer (A) is incorrect because the disbursements for the month are greater than the purchases for the month.Answer (B) is incorrect because 152,000 excludes operating expenses, payroll, and interest.Answer (C) is incorrect because 200,000 is based on a miscalculation of operating expenses and payroll.Answer (D) is correct. The cash disbursements for the month of April include 40% of April purchases, 30% of Marchpurchases, 30% of February purchases, April payroll (10% of April sales), operating expenses (20% of May sales), and aninterest payment of 20,000.

40% × 140,000 = 56,00030% × 160,000 = 48,00030% × 160,000 = 48,00010% × 300,000 = 30,00020% × 260,000 = 52,000Interest = 20,000   Total disbursements = 254,000   

[600] Gleim #: 5.106 -- Source: CIA 589 IV-32

An enterprise incurred the following costs during its first year of operation.

Legal fees in connection with establishingthe legal entity 100,000

Pre-opening costs of the new business 200,000Operating losses 400,000

During its second and later years, the enterprise became profitable. It amortizes start-up costs using the straight-line methodover a 5-year period. What is the correct amount of amortization expense for start-up costs during the second year of thelife of the enterprise?

20,000A.40,000B.60,000C.0D.

Answer (A) is incorrect because 20,000 start-up costs ordinarily are expensed as incurred.Answer (B) is incorrect because 40,000 start-up costs ordinarily are expensed as incurred.Answer (C) is correct. In some circumstances, expenditures may be made to provide future economic benefits, but nointangible asset results that is recognized. Accordingly, such expenditures should be expensed as incurred. Examplesinclude expenditures for start-up costs (start-up activities), such as legal and secretarial costs of establishing a legalentity, pre-opening costs of a new facility or business, or pre-operating costs of commencing new operations ormarketing new products. Thus, the enterprise should have expensed the full amount of the start-up costs in the firstyear. No amortization expense should be recognized in the second year. Moreover, to correct this fundamental error,an adjustment should be made to the opening balance of retained earnings, unless any resulting adjustment relating toprior periods is not reasonably determinable.Answer (D) is incorrect because an adjustment is required to correct the first year’s amortization of start-up costs.

[601] Gleim #: 5.107 -- Source: CMA 1288 1-11

An enterprise earned 10,000 before interest and taxes, has a 36% tax rate, and has the following debt outstanding:

First mortgage bond, 9.0% 5,000Debenture, 10.2% 10,000Subordinated bond, 12.0% 6,000 Total long-term debt 21,000 

The annual coverage of the enterprise’s debt is

Gleim's CIA Test Prep: Part III: Business Analysis and Information Technology(1165 questions)

Copyright 2004 Gleim Publications, Inc. Page 240Printed for g j

Page 241: Part three cia_with_ answers

4.57 times.A.2.92 times.B.11.85 times.C.5.46 times.D.

Answer (A) is correct. The times-interest-earned (interest coverage) ratio is computed by dividing the profit available forpaying interest (pretax, pre-interest profit) by the annual interest expense. The first step is to determine the annual interestexpense:

First Mortgage 9.0% × 5,000 = 450Debenture 10.2% × 10,000 = 1,020Subordinated 12.0% × 6,000 = 720         TOTAL EXPENSE 2,190         

Dividing the pretax, pre-interest profit of 10,000 by the 2,190 of interest expense produces an interest coverage ratio of4.57.Answer (B) is incorrect because the coverage of the debt is measured by the times-interest-earned ratio, which is found bydividing the pretax, pre-interest profit of 10,000 by the 2,190 of interest expense, yielding 4.57.Answer (C) is incorrect because the coverage of the debt is measured by the times-interest-earned ratio, which is found bydividing the pretax, pre-interest profit of 10,000 by the 2,190 of interest expense, yielding 4.57.Answer (D) is incorrect because the coverage of the debt is measured by the times-interest-earned ratio, which is found bydividing the pretax, pre-interest profit of 10,000 by the 2,190 of interest expense, yielding 4.57.

[602] Gleim #: 5.108 -- Source: CMA 1291 1-9

Which one of the following factors would likely cause an enterprise to increase its use of debt financing as measured by thedebt-to-total-capitalization ratio?

Increased economic uncertainty.A.An increase in the degree of operating leverage.B.An increase in the price-earnings ratio.C.An increase in the income tax rate.D.

Answer (A) is incorrect because increased economic uncertainty makes equity financing more desirable. There is nolegal mandate to make regular payments to equity holders.Answer (B) is incorrect because increased operating leverage (a greater degree of fixed production costs) increasesrisk. Hence, debt holders will require higher rates of interest. Higher interest costs will reduce the desirability of debtas a means of financing.Answer (C) is incorrect because an increase in the price-earnings ratio makes equity financing less costly and moredesirable. Hence, a given level of earnings will support a higher share price. The cost of equity capital declines whenthe share price increases.Answer (D) is correct. Debt financing usually has a lower cost than equity financing because interest payments areusually tax deductible. If tax rates rise, debt becomes even more desirable because the tax shield becomes morevaluable. The disadvantages of debt include the increase in fixed payments (interest). Thus, in an unstable economy,debt represents a greater risk to a firm than equity financing.

[603] Gleim #: 5.109 -- Source: CIA 587 III-42

An analysis of inventory turnover in a store’s clothing department indicated extremely low turnover. Which of thefollowing would most likely increase the turnover rate?

Increase inventories.A.Increase sales incentives.B.Increase selling prices.C.Decrease the frequency of purchases but maintain the same level of inventory.D.

Gleim's CIA Test Prep: Part III: Business Analysis and Information Technology(1165 questions)

Copyright 2004 Gleim Publications, Inc. Page 241Printed for g j

Page 242: Part three cia_with_ answers

Answer (A) is incorrect because increasing the denominator of the ratio decreases turnover.Answer (B) is correct. Inventory turnover equals cost of sales divided by average inventory. Reducing inventory thereforeincreases turnover. Sales incentives to improve sales should lower inventory levels.Answer (C) is incorrect because increasing price would probably decrease sales, increase inventory, and decrease turnover.Answer (D) is incorrect because a constant inventory level would not affect the turnover rate.

[604] Gleim #: 5.110 -- Source: CIA 588 IV-55

During a financial audit, an internal auditor discovers that the auditee’s maturing obligations are greater than the cashimmediately available to pay. What is the terminology that should be used in the audit report to describe this situation?

Bankruptcy.A.Economic failure.B.Insolvency.C.Illiquidity.D.

Answer (A) is incorrect because bankruptcy is the legal term for the status of any enterprise that has filed undercriteria established by legislation or that has been adjudged bankrupt by an appropriate court. Neither of theseconditions applies in the case cited.Answer (B) is incorrect because economic failure exists when an enterprise’s income is insufficient to cover bothexplicit and implicit costs. No such argument is made about the enterprise cited.Answer (C) is incorrect because insolvency exists when an entity’s total liabilities exceed the fair value of its totalassets. This question mentions nothing about the enterprise’s total liabilities or the fair value of its assets.Answer (D) is correct. Liquidity is the degree to which an enterprise’s assets can be readily converted into cash. Anenterprise is illiquid if its resources are held in forms that do not permit conversion for purposes of meeting itsobligations as they fall due. Thus, an enterprise may be solvent in the sense that its assets exceed its liabilities but beunable to meet its maturing obligations.

[Fact Pattern #44]Presented below are partial year-end financial statement data for enterprises A and B.

  A B Cash 100 200Accounts receivable unknown 100Inventories unknown 100Net fixed assets 200 100Accounts payable 100 50Long-term debt 200 50Common shares 100 200Retained earnings 150 100  A B Sales 600 5,800Cost of goods sold 300 5,000Administrative expenses 100 500Depreciation expense 100 100Interest expense 20 10Income tax expense 40 95Net profit 40 95

[605] Gleim #: 5.111 -- Source: CIA 595 IV-33

(Refers to Fact Pattern #44)Based on 365 days per year, B has days’ sales outstanding to the nearest full day of

0 days.A.3 days.B.6 days.C.7 days.D.

Gleim's CIA Test Prep: Part III: Business Analysis and Information Technology(1165 questions)

Copyright 2004 Gleim Publications, Inc. Page 242Printed for g j

Page 243: Part three cia_with_ answers

Answer (A) is incorrect because 0 days equals accounts receivable divided by sales.Answer (B) is incorrect because 3 days is the result if accounts payable is used rather than accounts receivable.Answer (C) is correct. Days’ sales of receivables (days’ sales outstanding) equals the days in the year divided by thereceivables turnover ratio (net credit sales ÷ average accounts receivable). Assuming sales are net credit sales and thatending accounts receivable equals average accounts receivable, the turnover ratio is 58 (5,800 ÷ 100), and the days’ sales ofreceivables equals 6 days (365 ÷ 58) (rounded).Answer (D) is incorrect because 7 days is the result if average cost of goods sold per day is used rather than sales.

[606] Gleim #: 5.112 -- Source: CIA 595 IV-34

(Refers to Fact Pattern #44)A has a times-interest-earned ratio of

Four times.A.Five times.B.10 times.C.15 times.D.

Answer (A) is incorrect because four times results if profit before tax rather than profit before interest and tax is usedin the formula.Answer (B) is correct. The times-interest-earned ratio for A equals profit before interest and tax divided by interestexpense. Profit before interest and tax is 100 (40 net profit + 20 interest + 40 tax expense). Hence, interest was earnedfive times (100 ÷ 20).Answer (C) is incorrect because 10 times results if depreciation expense is not deducted in calculating profit beforeinterest and tax.Answer (D) is incorrect because 15 times results if gross margin is used in the numerator rather than profit beforeinterest and tax.

[607] Gleim #: 5.113 -- Source: CIA 595 IV-35

(Refers to Fact Pattern #44)The degree of financial leverage of B, to two decimal places, is

1.03A.1.05B.1.12C.1.25D.

Answer (A) is incorrect because 1.03 results if depreciation expense is omitted from the calculation of profit beforeinterest and tax.Answer (B) is correct. The degree of financial leverage for B may be calculated as profit before interest and taxes(PBIT), divided by PBIT minus interest. PBIT is 200 (95 net profit + 10 interest + 95 tax expense). Thus, the DFL is1.05 [200 ÷ (200 – 10)].Answer (C) is incorrect because 1.12 results if net profit is used instead of PBIT.Answer (D) is incorrect because 1.25 is the degree of financial leverage for A.

[608] Gleim #: 5.114 -- Source: CIA 595 IV-36

(Refers to Fact Pattern #44)If A has 60 common shares outstanding, it has a carrying amount per share of approximately

1.67A.2.50B.4.17C.5.00D.

Answer (A) is incorrect because 1.67 results if retained earnings is omitted from the numerator.Answer (B) is incorrect because 2.50 results if the amount of common shares is omitted from the numerator.Answer (C) is correct. The carrying amount per share for A equals the sum of common shares and retained earnings,divided by the number of shares, or 4.17 [(100 + 150) ÷ 60].Answer (D) is incorrect because 5.00 is the carrying amount per share for B.

Gleim's CIA Test Prep: Part III: Business Analysis and Information Technology(1165 questions)

Copyright 2004 Gleim Publications, Inc. Page 243Printed for g j

Page 244: Part three cia_with_ answers

[Fact Pattern #45]An enterprise had the following number of shares outstanding during the year:

Period Shares OutstandingJanuary 1 - April 30 60,000May 1 - June 30 96,000July 1 - September 30 90,000October 1 - December 31 70,000

During the year, the enterprise had net profit of 600,000. It paid 100,000 in preference dividends, declared and paid 250,000 ofcommon share dividends, and paid 50,000 of interest to debt holders.

[609] Gleim #: 5.115 -- Source: CIA 595 IV-79

(Refers to Fact Pattern #45)The number of shares the enterprise had outstanding for the year, for the purpose of computing basic earnings per share(BEPS), is

65,000A.70,000B.76,000C.79,000D.

Answer (A) is incorrect because 65,000 is the simple average of the shares outstanding on January 1 and December31.Answer (B) is incorrect because 70,000 is the year-end number of shares outstanding.Answer (C) is correct. The weighted-average number of shares outstanding for the year (76,000 shares) is used in theBEPS calculation.

Dates Shares Fraction WeightedOutstanding Outstanding of Year Shares

January 1 - April 30 60,000 4 ÷ 12 20,000May 1 - June 30 96,000 2 ÷ 12 16,000

July 1 - September 30 90,000 3 ÷ 12 22,500October 1 - December 31 70,000 3 ÷ 12 17,500

           76,000     

Answer (D) is incorrect because 79,000 is the simple average of the shares outstanding at various times during theyear.

[610] Gleim #: 5.116 -- Source: Publisher

Mill, which began operations on January 1, 2001, recognizes income from long-term construction contracts under thepercentage-of-completion method in its financial statements and under the completed-contract method for tax reporting.Income under each method follows:

  Completed- Percentage-Year Contract of-Completion2001 -- 300,0002002 400,000 600,0002003 700,000 850,000

There are no other temporary differences. If the applicable tax rate is 25%, Mill should report in its balance sheet atDecember 31, 2003 a deferred income tax liability of

87,500A.125,000B.162,500C.37,500D.

Gleim's CIA Test Prep: Part III: Business Analysis and Information Technology(1165 questions)

Copyright 2004 Gleim Publications, Inc. Page 244Printed for g j

Page 245: Part three cia_with_ answers

Answer (A) is incorrect because 87,500 ignores the difference for 2001.Answer (B) is incorrect because 125,000 ignores the difference for 2003.Answer (C) is correct. In its financial statements issued through December 31, 2003, Mill has reported 1,750,000 (300,000+ 600,000 + 850,000) of income from long-term contracts. In its tax returns for the same period, it has reported 1,100,000(400,000 + 700,000) of income from the same sources. The result is a taxable temporary difference. Thus, Mill expects tohave future taxable amounts of 650,000 and should recognize a deferred tax liability of 162,500 (25% applicable tax rate ×650,000).Answer (D) is incorrect because 37,500 ignores the differences for 2001 and 2002.

[611] Gleim #: 5.117 -- Source: CIA 1195 IV-35

An enterprise has total asset turnover of 3.5 times and a total debt to total assets ratio of 70%. If the enterprise has totaldebt of 1,000,000, it has a sales level of

5,000,000.00A.2,450,000.00B.408,163.26C.200,000.00D.

Answer (A) is correct. If the debt-to-total assets ratio is 70% and debt is 1,000,000, total assets must be 1,428,571.4(1,000,000 ÷ .7). Given total asset turnover (sales ÷ total assets) of 3.5, sales must be 5,000,000 (3.5 × 1,428,571.4).Answer (B) is incorrect because 2,450,000.00 is obtained if total assets is calculated as total debt multiplied by .7.Answer (C) is incorrect because 408,163.26 results if sales is determined by dividing total assets by 3.5.Answer (D) is incorrect because 200,000.00 is obtained by calculating total assets as total debt multiplied by .7 andsales as total assets divided by 3.5.

[612] Gleim #: 5.118 -- Source: CIA 1195 IV-54

All other things held constant, external capital requirements are lower for enterprises with which one of the followingcharacteristics?

Lower retention ratios.A.Higher sales growth rates.B.Lower capital intensity ratios.C.Lower profit margins.D.

Answer (A) is incorrect because, if earnings retention is low (that is, if dividend payout is high), less internal capitalis available, so more external capital is needed.Answer (B) is incorrect because higher sales require more assets and more capital.Answer (C) is correct. The capital intensity ratio is the amount of assets required per dollar of sales. Externalfinancial requirements are lower if the capital intensity ratio is low because sales can grow rapidly without muchoutside capital, other things held constant.Answer (D) is incorrect because enterprises with lower profit margins have lower retained earnings and thereforerequire more external capital.

[613] Gleim #: 5.119 -- Source: CIA 1195 IV-52

An enterprise borrows funds from its bank for a one-year period. The bank charges interest at a nominal rate of 15% perannum, on a discount basis, and requires a 10% compensating balance. The effective annual interest rate on the loan is

16.67%A.17.65%B.20.00%C.25.00%D.

Gleim's CIA Test Prep: Part III: Business Analysis and Information Technology(1165 questions)

Copyright 2004 Gleim Publications, Inc. Page 245Printed for g j

Page 246: Part three cia_with_ answers

Answer (A) is incorrect because 16.67% does not adjust for the discount interest arrangement.Answer (B) is incorrect because 17.65% does not adjust for the compensating balance requirement.Answer (C) is correct. Discount interest is subtracted before the loan proceeds are paid to the borrower. A compensatingbalance is an amount that the borrower must keep on deposit with the lender. The effective annual interest rate is increasedby both the discount interest arrangement and by the compensating balance requirement. The effective rate equals thenominal rate dividend by one minus the sum of the nominal rate and the compensating balance percentage, or 20% [.15 ÷(1.0 – .15 – .10)].Answer (D) is incorrect because 25.00% is the sum of the nominal rate and the compensating balance percentage.

[614] Gleim #: 5.120 -- Source: CIA 1194 IV-12

An enterprise has 400 of current assets, composed of 200 of cash, 100 of accounts receivable, and 100 of inventory. Theenterprise has 200 of long-term debt, $100 of accounts payable, and 75 of notes payable. The notes payable are due in 6months. The acid-test ratio, to two decimal places, is

0.80A.1.29B.1.71C.2.29D.

Answer (A) is incorrect because 0.80 is based on total liabilities instead of current liabilities.Answer (B) is incorrect because 1.29 uses working capital in the numerator.Answer (C) is correct. The acid-test ratio equals current assets (400) minus inventories (100), divided by currentliabilities (100 + 75 = 175), or 1.71.Answer (D) is incorrect because 2.29 is the current ratio.

[Fact Pattern #46]An enterprise has the following income statements:

  2003 2002 Sales 1,500,000 1,400,000Cost of goods sold 800,000 750,000Gross profit 700,000 650,000Selling and

admin. expense 62,000 60,000Depreciation expense 50,000 50,000Profits before

interest and taxes 688,000 540,000Interest expense 100,000 100,000Profit before taxes 588,000 440,000Income tax (50%) 294,000 220,000Net profit 294,000 220,000

Selected balance sheet items are as follows:

  2003 2002  Year-end Year-end Accounts receivable 300,000 200,000Accounts payable 250,000 275,000

Assume a 365-day year in any calculations.

[615] Gleim #: 5.121 -- Source: CIA 1196 IV-37

(Refers to Fact Pattern #46)In 2003 the enterprise had cash provided by operations of

219,000A.244,000B.344,000C.469,000D.

Gleim's CIA Test Prep: Part III: Business Analysis and Information Technology(1165 questions)

Copyright 2004 Gleim Publications, Inc. Page 246Printed for g j

Page 247: Part three cia_with_ answers

Answer (A) is correct. Cash provided by operations equals net profit, plus depreciation, minus the increase in accountsreceivable, minus the decrease in accounts payable. The cash provided is 219,000 [294,000 + 50,000 – (300,000 – 200,000)– (275,000 – 250,000)]. An increase in receivables is a noncash component of net profit. A decrease in accounts payable isadded when adjusting cost of goods sold to reflect cash paid to suppliers. Thus, it is subtracted when adjusting net profit toarrive at cash provided by operations.Answer (B) is incorrect because 244,000 does not consider the changes in accounts receivable and payable and subtractsdepreciation.Answer (C) is incorrect because 344,000 does not consider the changes in accounts receivable and payable.Answer (D) is incorrect because 469,000 adds rather than subtracts the increase in receivables and the decrease in payables.

[616] Gleim #: 5.122 -- Source: CIA 1196 IV-38

(Refers to Fact Pattern #46)Based on the 2003 year-end balance of accounts receivable and the 2003 income statement, the company had an averagecollection period for 2003 of

49 days.A.52 days.B.73 days.C.78 days.D.

Answer (A) is incorrect because 49 days uses accounts receivable from 2002 instead of 2003.Answer (B) is incorrect because 52 days is the average collection period for 2002.Answer (C) is correct. The average collection period for 2003 is 73 days [300,000 accounts receivable ÷ (1,500,000sales ÷ 365 days)].Answer (D) is incorrect because 78 days uses 2002 annual sales.

[617] Gleim #: 5.123 -- Source: CIA 596 IV-34

In calculating diluted earnings per share when an enterprise has convertible bonds outstanding, the weighted-averagenumber of ordinary shares outstanding must be <List A> to adjust for the conversion feature of the bonds, and the net profitattributable to ordinary shareholders must be <List B> by the amount of interest expense on the bonds, net of tax.

  List A List B A. Increased IncreasedB. Increased DecreasedC. Decreased IncreasedD. Decreased Decreased

Answer (A) is correct. The weighted-average number of ordinary shares outstanding must be increased to reflect theshares into which the bonds could be converted. Also, the effect of the bond interest on net profit attributable toordinary shareholders (net profit after subtracting preference dividends) must be eliminated. In this way, dilutedearnings per share is calculated as if the bonds had been converted into common shares as of the start of the year.Answer (B) is incorrect because the net profit attributable to ordinary shareholders must be increased.Answer (C) is incorrect because the weighted-average number of ordinary shares outstanding must be increased, andthe net profit attributable to ordinary shareholders must be increased.Answer (D) is incorrect because the weighted-average number of ordinary shares outstanding must be increased, andthe net profit attributable to ordinary shareholders must be increased.

Gleim's CIA Test Prep: Part III: Business Analysis and Information Technology(1165 questions)

Copyright 2004 Gleim Publications, Inc. Page 247Printed for g j

Page 248: Part three cia_with_ answers

[Fact Pattern #47]An enterprise’s financial statements for the current year are presented below:

Balance Sheet

Cash 100Accounts receivable 200Inventory 50Net fixed assets 600 

Total 950 Accounts payable 140Long-term debt 300Share capital 260Retained earnings 250 

Total 950    

Statement of Income and Retained Earnings   Sales 3,000Cost of goods sold 1,600 Gross profit 1,400Operations expenses 970 Operating profit 430Interest expense 30 Profit before tax 400Income tax 200 Net profit 200Plus Jan. 1 retained earnings 150Minus dividends 100 Dec. 31 retained earnings 250 

[618] Gleim #: 5.124 -- Source: CIA 596 IV-37

(Refers to Fact Pattern #47)The net working capital is

160A.210B.350C.490D.

Answer (A) is incorrect because 160 equals net quick assets minus current liabilities.Answer (B) is correct. Net working capital is the difference between current assets and current liabilities. It equals210 (100 cash + 200 A/R + 50 inventory – 140 A/P).Answer (C) is incorrect because 350 equals current assets.Answer (D) is incorrect because 490 equals current assets plus current liabilities.

[619] Gleim #: 5.125 -- Source: CMA 1293 1-11

Environmental policies that attempt to weigh the costs and benefits of environmental improvement

Gleim's CIA Test Prep: Part III: Business Analysis and Information Technology(1165 questions)

Copyright 2004 Gleim Publications, Inc. Page 248Printed for g j

Page 249: Part three cia_with_ answers

Cannot work because business firms are not willing to cooperate.A.Grant tax relief to the firms who pollute the least.B.Would leave an acceptable amount of pollution.C.Cannot work if government is involved.D.

Answer (A) is incorrect because firms are more supportive of cost-benefit regulatory policies.Answer (B) is incorrect because granting tax relief to those firms which pollute the least is not a cost-benefit policy.Answer (C) is correct. If environmental policies weigh the costs and benefits of regulation, some pollution will existbecause the costs of eliminating all pollution will likely outweigh the benefits. Cost-benefit considerations are reflected in,for example, the establishment of air quality standards that do not attempt to eliminate the incidence of specified pollutantsand requirements for control technology that, for the most part, do not mandate use of devices that achieve the maximumpollution reduction. The drawback of a cost-benefit approach is the difficulty of quantifying benefits.Answer (D) is incorrect because such policies can only work if government is involved.

[620] Gleim #: 5.126 -- Source: Publisher

Under a value-added taxing system

Businesses must pay a tax only if they make a profit.A.The consumer ultimately bears the incidence of the tax through higher prices.B.Consumer savings are discouraged.C.The amount of value added is the difference between a firm’s sales and its cost of goods sold.D.

Answer (A) is incorrect because businesses must pay a value-added tax regardless of whether they make a profit.Answer (B) is correct. A value-added tax is based on consumption. The tax is levied on the value added to goods byeach business unit in the production and distribution chain. It is the consumer who ultimately bears the incidence ofthe tax because businesses subject to the tax will raise prices to offset the operating cost of the tax.Answer (C) is incorrect because a value-added tax encourages savings. This tax is paid only on consumption, notsavings.Answer (D) is incorrect because the amount of value added is the difference between a firm’s sales and its purchases.

[621] Gleim #: 5.127 -- Source: Publisher

A deferred tax asset is recognized for

Goodwill if its amortization is nondeductible for tax purposes.A.The initial recognition of an asset that is not part of a business combination and does not affect accounting profit ortaxable profit at the date of the transaction.

B.

A deductible temporary difference if the future availability of taxable profit is probable.C.Negative goodwill treated as deferred income.D.

Answer (A) is incorrect because a deferred tax liability (DTL) is recognized for most taxable TDs. However, no DTLis recognized when it arises from goodwill if the amortization of the goodwill is not deductible for tax purposes. Thereason is that the recognition of a DTL would increase goodwill. In a business combination that is an acquisition,goodwill equals the excess of the acquisition cost over the acquirer’s interest in the fair value of the identifiable netassets of the acquiree.Answer (B) is incorrect because neither a deferred tax liability nor a deferred tax asset is recognized in thesecircumstances.Answer (C) is correct. A deferred tax asset is recognized for most deductible TDs and for the carryforward of unusedtax losses and credits, but only to the extent it is probable that taxable profit will be available to permit the use ofthose amounts.Answer (D) is incorrect because no DTA is recognized when it arises from negative goodwill treated as deferredincome. The reason is that the recognition of a DTA would increase negative goodwill. In a business combinationthat is an acquisition, negative goodwill equals the excess of the acquirer’s interest in the fair value of the identifiablenet assets of the acquiree over the acquisition cost.

[622] Gleim #: 5.128 -- Source: Publisher

At the beginning of year 1, Cody Construction began work on a 3-year construction contract. This contract is accounted forby the percentage-of-completion method for financial accounting purposes and the completed-contract method for taxpurposes. During year 1, Cody reported 800,000 of income from this contract. However, because of cost overruns incurredin year 2, Cody reported a 500,000 loss from this contract. Cody’s year 2 balance sheet should include a

Gleim's CIA Test Prep: Part III: Business Analysis and Information Technology(1165 questions)

Copyright 2004 Gleim Publications, Inc. Page 249Printed for g j

Page 250: Part three cia_with_ answers

  Deferred Deferred  Tax Asset Tax Liability A. Yes YesB. Yes NoC. No NoD. No Yes

Answer (A) is incorrect because a taxable temporary difference usually results in a deferred tax liability in the currentperiod.Answer (B) is incorrect because a taxable temporary difference usually results in a deferred tax liability in the currentperiod.Answer (C) is incorrect because a taxable temporary difference usually results in a deferred tax liability in the currentperiod.Answer (D) is correct. A deferred tax liability is recognized for the deferred tax consequences attributable to most taxabletemporary differences. A deferred tax asset is recognized for the deferred tax consequences attributable to most deductibletemporary differences, but only to the extent it is probable that taxable profit will be available to permit the use of thoseamounts. At the end of year 2, the cumulative effect of the difference in accounting for financial reporting and tax purposesis a 300,000 (800,000 – 500,000) temporary difference. Because this 300,000 temporary difference will result in futuretaxable amounts, it is a taxable temporary difference for which a deferred tax liability is recognized.

[Fact Pattern #48]The following relates to Lally Plc for 2002 and 2003.

  2003 2002 Profit before income taxes 5,000,000 4,000,000Interest income included above that

was not subject to income taxes 100,000 100,000

Profit before income taxes in 2002 included rent income of 80,000 that was notsubject to income tax until its receipt in 2003.Lally was subject to an effective income tax rate of 40% in 2002 and 2003.

 

[623] Gleim #: 5.129 -- Source: CMA 686 3-7

(Refers to Fact Pattern #48)The tax expense that would have been reported on Lally’s income statement for the year ended December 31, 2002 is

1,560,000A.1,528,000B.1,600,000C.1,568,000D.

Answer (A) is correct. Tax expense (tax income) equals the sum of current tax expense (current tax income) anddeferred tax expense (deferred tax income). Current tax expense equals taxes paid or payable, and deferred taxexpense equals the change in the enterprise’s deferred tax assets and liabilities. Because 100,000 of interest incomeand 80,000 of rental income are not subject to tax in 2002, taxable profit is 3,820,000, and current tax expense is1,528,000. Deferred tax expense equals the deferred tax liability (40% × 80,000 = 32,000) at year-end, assuming nodeferred tax asset or liability balances existed at the beginning of the year. Hence, tax expense is 1,560,000(1,528,000 + 32,000).Answer (B) is incorrect because 1,528,000 does not include tax on the rent income.Answer (C) is incorrect because 1,600,000 includes tax on the tax-exempt interest.Answer (D) is incorrect because 1,568,000 includes tax on the tax-exempt interest but does not include tax on therent income.

[624] Gleim #: 5.130 -- Source: CMA 686 3-9

(Refers to Fact Pattern #48)Lally’s current tax expense for 2003 was

Gleim's CIA Test Prep: Part III: Business Analysis and Information Technology(1165 questions)

Copyright 2004 Gleim Publications, Inc. Page 250Printed for g j

Page 251: Part three cia_with_ answers

1,960,000A.1,992,000B.2,000,000C.2,032,000D.

Answer (A) is incorrect because 1,960,000 does not include the deferred tax liability from the rent.Answer (B) is correct. The 5,000,000 of pre-tax profit is reduced by the 100,000 of interest income not subject to tax.However, the rental income accrued in the previous year is taxable in 2003. Thus, the profit subject to tax is 4,980,000.Given a 40% rate, the current tax expense is 1,992,000.Answer (C) is incorrect because 2,000,000 includes the tax-exempt interest but does not include the deferred tax liabilityfrom the rent.Answer (D) is incorrect because 2,032,000 includes the tax-exempt interest.

[625] Gleim #: 5.131 -- Source: CMA 686 3-10

(Refers to Fact Pattern #48)The deferred tax asset or liability reported on Lally’s balance sheet on December 31, 2003 is

40,000A.32,000B.8,000C.0D.

Answer (A) is incorrect because the non-taxable interest income does not have deferred tax consequences. The rentalincome created a deferred tax liability in 2002 that reversed in 2003. Therefore, no deferred tax amount existed at theend of 2003.Answer (B) is incorrect because the non-taxable interest income does not have deferred tax consequences. The rentalincome created a deferred tax liability in 2002 that reversed in 2003. Therefore, no deferred tax amount existed at theend of 2003.Answer (C) is incorrect because the non-taxable interest income does not have deferred tax consequences. The rentalincome created a deferred tax liability in 2002 that reversed in 2003. Therefore, no deferred tax amount existed at theend of 2003.Answer (D) is correct. The correct answer is 0. Deferred tax assets arise from deductible temporary differences andthe carryforward of unused tax losses and credits. Deferred tax liabilities arise from taxable temporary differences.The interest income not subject to tax resulted in neither future taxable or future deductible amounts and therefore nodeferred tax item was recognized for it. The only item resulting in a temporary difference was the rental income. Adeferred tax liability would have been created at the end of 2002. Given that the difference reversed in 2003, nodeferred amount existed at the end of 2003.

[626] Gleim #: 5.132 -- Source: CIA 1195 IV-71

An increase in excise taxes will generate [List A] tax revenue if it is levied on products for which demand is [List B]elastic.

  List A List B A. More MoreB. More LessC. Negative LessD. Less Less

Answer (A) is incorrect because price elasticity of demand is inversely correlated with the revenue effects of anincrease in excise taxes.Answer (B) is correct. A higher excise tax increases the selling price of the product. This price increase will have aless negative effect on sales volume for products with less elastic demand. Examples of products with low elasticityof demand include gasoline, tobacco, and alcohol. The tax revenue generated by an increase in excise taxes istherefore higher if the tax is levied on products with less elastic demand.Answer (C) is incorrect because price elasticity of demand is inversely correlated with the revenue effects of anincrease in excise taxes.Answer (D) is incorrect because price elasticity of demand is inversely correlated with the revenue effects of anincrease in excise taxes.

Gleim's CIA Test Prep: Part III: Business Analysis and Information Technology(1165 questions)

Copyright 2004 Gleim Publications, Inc. Page 251Printed for g j

Page 252: Part three cia_with_ answers

[627] Gleim #: 5.133 -- Source: CIA 1194 IV-65

If the government were to increase corporate tax rates, after-tax cost-volume-profit relationships for individual firms wouldchange as follows

Breakeven points would increase.A.Breakeven points would decrease.B.There would be no change in the breakeven points.C.There would be no change whatsoever in cost-volume-profit relationships.D.

Answer (A) is incorrect because a tax rate increase would not affect the breakeven point. At the breakeven point,profits equal zero.Answer (B) is incorrect because a tax rate increase would not affect the breakeven point. At the breakeven point,profits equal zero.Answer (C) is correct. The firm owes no tax if it does not earn a profit. At the breakeven point, there is no profit.Thus, a tax increase would have no effect on a firm’s breakeven point, which is calculated by dividing fixed costs bythe contribution margin ratio. The contribution margin equals sales revenues minus total variable costs.Answer (D) is incorrect because the contribution margin per unit will be lower if the corporate tax rate is higher, forall companies operating above the breakeven point.

[628] Gleim #: 6.1 -- Source: CIA 597 IV-48

Common shareholders with preemptive rights are entitled to

Vote first at annual meetings.A.Purchase any additional bonds sold by the firm.B.Purchase any additional shares sold by the firm.C.Gain control of the firm in a proxy fight.D.

Answer (A) is incorrect because there is no prescribed order of shareholder voting.Answer (B) is incorrect because preemptive rights concern only equity ownership. Thus, they do not apply tononconvertible debt.Answer (C) is correct. Preemptive rights protect common shareholders, proportional ownership interests fromdilution in value. A secondary purpose is to maintain the shareholders control of the company. Accordingly, thepreemptive right, whether granted by statute or by the corporate charter, grants common shareholders the power toacquire on a pro rata basis any additional common shares sold by the firm. Preemptive rights also apply to debtconvertible into common shares.Answer (D) is incorrect because a proxy fight is an attempt to gain control of a company by persuading shareholdersto grant their voting rights to others.

[629] Gleim #: 6.2 -- Source: CIA 597 IV-49

Participating preferred shareholders are entitled to

Monitor any sinking funds for the purchase and retirement of debt.A.Vote at all annual meetings.B.Convert their shares into common shares.C.Share in the firm’s earnings beyond the stated dividend level.D.

Answer (A) is incorrect because the participation privilege is unrelated to monitoring privileges.Answer (B) is incorrect because preferred shareholders do not have voting rights except in circumstances in whichthe firm has not paid the preferred share dividends for a specified period.Answer (C) is incorrect because a conversion feature, not a participation feature, allows conversion to common stock.Answer (D) is correct. Participating preferred shareholders are entitled to share in the earnings of the firm. Theyparticipate in earnings distributions under set terms and conditions. Hence, after the stated preferred dividend is paid,and common shareholders receive an equal dividend, any remaining dividends are allocated to all shareholders inproportion to the par values of their shares.

[630] Gleim #: 6.3 -- Source: CIA 590 IV-57

A company has recently purchased some shares of a competitor as part of a long-term plan to acquire the competitor.However, it is somewhat concerned that the market price of the shares could decrease over the short run. The companycould hedge against the possible decline in the market price by

Gleim's CIA Test Prep: Part III: Business Analysis and Information Technology(1165 questions)

Copyright 2004 Gleim Publications, Inc. Page 252Printed for g j

Page 253: Part three cia_with_ answers

Purchasing a call option on the shares.A.Purchasing a put option on the shares.B.Selling a put option on the shares.C.Obtaining a warrant option on the shares.D.

Answer (A) is incorrect because a call option is the right to purchase shares at a given price within a specified period. Ithedges against a price increase.Answer (B) is correct. A put option is the right to sell shares at a given price within a certain period. If the market pricefalls, the put option may allow the sale of shares at a price above market, and the profit of the option holder will be thedifference between the price stated in the put option and the market price, minus the cost of the option, commissions, andtaxes.Answer (C) is incorrect because selling a put option could force the company to purchase additional shares if the option isexercised.Answer (D) is incorrect because a warrant gives the holder a right to purchase shares from the issuer at a given price (it isusually distributed along with debt).

[631] Gleim #: 6.4 -- Source: CIA 1196 IV-58

A call option on a common share is more valuable when there is a lower

Market value of the underlying share.A.Exercise price on the option.B.Time to maturity on the option.C.Variability of market price on the underlying share.D.

Answer (A) is incorrect because a call option is the right to purchase a common share at a set price for a set timeperiod. If the underlying share has a lower market value, the call option is less, not more, valuable.Answer (B) is correct. The lower the exercise price, the more valuable the call option. The exercise price is the priceat which the call holder has the right to purchase the underlying share.Answer (C) is incorrect because a call option is less, not more, valuable given less time to maturity. When the optionhas less time to maturity, the chance that the share price will rise is smaller.Answer (D) is incorrect because a call option is less, not more, valuable if the price of the underlying share is lessvariable. Less variability means a lower probability of a price increase.

[632] Gleim #: 6.5 -- Source: CIA 596 IV-46

Preferred shares are securities with characteristics of both common shares and bonds. Preferred shares have <List A> likecommon shares and <List B> like bonds.

  List A List B A. A maturity date A fixed periodic paymentB. No maturity date No fixed periodic paymentC. A maturity date No fixed periodic paymentD. No maturity date A fixed periodic payment

Answer (A) is incorrect because preferred shares do not have a maturity date.Answer (B) is incorrect because preferred shares have fixed periodic dividend payments.Answer (C) is incorrect because preferred shares do not have a maturity date but do have fixed periodic dividendpayments.Answer (D) is correct. Like common shares (but unlike bonds), preferred shares have no maturity date, althoughcertain preferred shares (transient preferred shares) must be redeemed within a short time (e.g., 5 to 10 years). Likebonds (but unlike common shares), preferred shares have a fixed periodic payment. The fixed payment is in the formof a stated dividend in the case of the preferred shares and interest payments in the case of bonds. However, preferreddividends, unlike interest, do not become an obligation unless declared.

[633] Gleim #: 6.6 -- Source: CIA 1195 IV-47

Which of the following is usually not a feature of cumulative preferred shares?

Gleim's CIA Test Prep: Part III: Business Analysis and Information Technology(1165 questions)

Copyright 2004 Gleim Publications, Inc. Page 253Printed for g j

Page 254: Part three cia_with_ answers

Has priority over common shares with regard to earnings.A.Has priority over common shares with regard to assets.B.Has voting rights.C.Has the right to receive dividends in arrears before common share dividends can be paid.D.

Answer (A) is incorrect because preferred shares have priority over common shares with regard to earnings, so dividendsmust be paid on preferred shares before they can be paid on common shares.Answer (B) is incorrect because preferred shares have priority over common shares with regard to assets. In the event ofliquidation, for example, because of bankruptcy, the claims of preferred shareholders must be satisfied in full before thecommon shareholders receive anything.Answer (C) is correct. Preferred shares do not usually have voting rights. Preferred shareholders are usually given the rightto vote for directors only if the company has not paid the preferred dividend for a specified period of time, such as 10quarters. Such a provision is an incentive for management to pay preferred dividends.Answer (D) is incorrect because cumulative preferred shares have the right to receive any dividends not paid in priorperiods before common share dividends are paid.

[634] Gleim #: 6.7 -- Source: CIA 596 IV-57

If a company has outstanding bonds with a sinking fund provision and if interest rates have <List A> since the bonds wereissued, the company would realize a savings in meeting its sinking fund obligations by <List B>.

  List A List B A. Increased Buying back bonds  in the open marketB. Remained constant Calling in a portion of  the bonds at face valueC. Increased Calling in a portion of  the bonds at face valueD. Decreased Buying back bonds in  the open market

Answer (A) is correct. If interest rates have increased, the prices of outstanding bonds must decrease so that theiryields will reach the market rate. Hence, the bonds will be selling at a discount (below face value) in the open market.Retiring a portion of the outstanding bonds by buying them in the open market at the discounted price will be cheaperthan calling a portion of the bonds at face value.Answer (B) is incorrect because, if interest rates have remained constant, the bonds will still be selling at face valuein the open market. The cost of buying bonds on the open market will be the same as the cost of calling a portion ofthe bonds at face value.Answer (C) is incorrect because, if interest rates have increased, buying bonds in the open market will be the cheaperstrategy.Answer (D) is incorrect because, if interest rates have decreased, the bonds will be selling at a premium in the openmarket. Retiring a portion of the bonds by purchasing them on the open market will be more costly than calling aportion of the bonds at face value.

[635] Gleim #: 6.8 -- Source: CIA 1195 IV-46

Convertible bonds and bonds issued with warrants differ in that

Convertible bonds have lower coupon rates than straight bonds, while bonds issued with warrants have highercoupon rates than straight bonds.

A.

Convertible bonds have higher coupon rates than straight bonds, while bonds issued with warrants have lowercoupon rates than straight bonds.

B.

Convertible bonds remain outstanding after the bondholder exercises the right to become a common shareholder,while bonds that are issued with warrants do not.

C.

Bonds that are issued with warrants remain outstanding after the bondholder exercises the right to become a commonshareholder, while convertible bonds do not.

D.

Gleim's CIA Test Prep: Part III: Business Analysis and Information Technology(1165 questions)

Copyright 2004 Gleim Publications, Inc. Page 254Printed for g j

Page 255: Part three cia_with_ answers

Answer (A) is incorrect because bonds issued with warrants and convertible bonds have lower coupon rates thanconventional bonds.Answer (B) is incorrect because bonds issued with warrants and convertible bonds have lower coupon rates thanconventional bonds.Answer (C) is incorrect because convertible bonds do not remain outstanding.Answer (D) is correct. Warrants are usually detachable. They are options to purchase equity securities and should beseparately accounted for. A capital gain results if the share price rises above the option price. The bonds remain outstandingif the warrants are exercised. Convertible bonds must be surrendered when the conversion privilege is exercised. The equityfeature of convertible bonds is not separately accounted for.

[636] Gleim #: 6.9 -- Source: CIA 1196 IV-20

Zero-coupon bonds

Sell for a small fraction of their face value because their yield is much lower than the market rate.A.Increase in value each year as they approach maturity, providing the owner with the total payoff at maturity.B.Are redeemable in measures of a commodity such as barrels of oil, tons of coal, or ounces of rare metal (e.g., silver).C.Are high-interest-rate, high-risk, unsecured bonds that have been used extensively to finance leveraged buyouts.D.

Answer (A) is incorrect because deep discount bonds pay interest significantly below the market rate, whereas zero-coupon bonds do not pay interest.Answer (B) is correct. Zero-coupon bonds sell at a deep discount and increase in value each year until maturity.These bonds do not pay interest.Answer (C) is incorrect because commodity-backed bonds are redeemable in measures of a commodity.Answer (D) is incorrect because junk bonds are high-interest-rate, high-risk, unsecured bonds.

[637] Gleim #: 6.10 -- Source: CIA 595 IV-39

Bondholders are assured of protection against inflation if they hold

Income bonds.A.Convertible bonds.B.Mortgage bonds.C.Indexed bonds.D.

Answer (A) is incorrect because income bonds pay interest to the holder only if the interest is earned. The interestpaid is not inflation adjusted.Answer (B) is incorrect because convertible bonds can be redeemed for the common shares of the issuer at the optionof the holder. Interest payments are not inflation adjusted.Answer (C) is incorrect because mortgage bonds are secured by fixed assets of the issuer. Thus, they provide greatersecurity to bondholders, but the interest payments are not inflation adjusted.Answer (D) is correct. The interest payments on indexed or purchasing power bonds are based on an inflation index,such as the consumer price index. Thus, interest paid to bondholders rises automatically when the inflation rate rises.

[638] Gleim #: 6.11 -- Source: CIA 1191 IV-50

From the viewpoint of the investor, which of the following securities provides the least risk?

Mortgage bond.A.Subordinated debenture.B.Income bond.C.Debentures.D.

Gleim's CIA Test Prep: Part III: Business Analysis and Information Technology(1165 questions)

Copyright 2004 Gleim Publications, Inc. Page 255Printed for g j

Page 256: Part three cia_with_ answers

Answer (A) is correct. A mortgage bond is secured with specific fixed assets, usually real property. Thus, under the rightsenumerated in the bond indenture, creditors will be able to receive payments from liquidation of the property in case ofdefault. In a bankruptcy proceeding, these amounts are paid before any transfers are made to other creditors, including thosepreferences. Hence, mortgage bonds are less risky than the others listed.Answer (B) is incorrect because a debenture is long-term debt that is not secured (collateralized) by specific property.Subordinated debentures have a claim on the debtor’s assets that may be satisfied only after senior debt has been paid infull. Debentures of either kind are therefore more risky than mortgage bonds.Answer (C) is incorrect because an income bond pays interest only if the debtor earns it. Such bonds are also more riskythan secured debt.Answer (D) is incorrect because unsecured debt is riskier than a mortgage bond.

[639] Gleim #: 6.12 -- Source: CIA 589 IV-49

Which of the following classes of securities are listed in order from lowest risk/opportunity for return to highestrisk/opportunity for return?

U.S. Treasury bonds; corporate first mortgage bonds; corporate income bonds; preferred shares.A.Corporate income bonds; corporate mortgage bonds; convertible preferred shares; subordinated debentures.B.Common shares; corporate first mortgage bonds; corporate second mortgage bonds; corporate income bonds.C.Preferred shares; common shares; corporate mortgage bonds; corporate debentures.D.

Answer (A) is correct. The general principle is that risk and return are directly correlated. For example,U.S. Treasury securities are backed by the full faith and credit of the U.S. government and are therefore the least riskyform of investment listed. However, their return is correspondingly lower. Corporate first mortgage bonds are lessrisky than income bonds or shares because they are secured by specific property. In the event of default, thebondholders can have the property sold to satisfy their claims. Holders of first mortgages have rights paramount tothose of any other parties, such as holders of second mortgages. Income bonds pay interest only in the event thecorporation earns income. Thus, holders of income bonds have less risk than shareholders because meeting thecondition makes payment of interest mandatory. Preferred shareholders receive dividends only if they are declared,and the directors usually have complete discretion in this matter. Also, shareholders have claims junior to those ofdebtholders if the enterprise is liquidated.Answer (B) is incorrect because the proper listing is mortgage bonds, subordinated debentures, income bonds, andpreferred shares. Debentures are unsecured debt instruments. Their holders have enforceable claims against the issuereven if no income is earned or dividends declared.Answer (C) is incorrect because the proper listing is first mortgage bonds, second mortgage bonds, income bonds,and common shares. The second mortgage bonds are secured, albeit junior, claims.Answer (D) is incorrect because the proper listing is mortgage bonds, debentures, preferred shares, and commonshares. Holders of common shares cannot receive dividends unless the holders of preferred shares receive thestipulated periodic percentage return, in addition to any arrearages if the preferred shares are cumulative.

[640] Gleim #: 6.13 -- Source: CIA 1195 IV-53

The correct equation for calculating the approximate percentage cost, on an annual basis, of not taking trade discounts is

A. 

B. 

C. 

D. 

Gleim's CIA Test Prep: Part III: Business Analysis and Information Technology(1165 questions)

Copyright 2004 Gleim Publications, Inc. Page 256Printed for g j

Page 257: Part three cia_with_ answers

Answer (A) is correct. The first term of the formula represents the periodic cost of the trade discount, calculated as the costper unit of trade credit (discount %) divided by the funds made available by not taking the discount (100 – discount %). Thesecond term represents the number of times per year this cost is incurred. The multiple of these terms is the approximateannual percentage cost of not taking the trade discount. A precise formula would incorporate the effects of compoundingwhen calculating the annual cost.Answer (B) is incorrect because the denominator of the first term should represent the funds made available by not takingthe discount (100 – discount %).Answer (C) is incorrect because the first term is the reciprocal of the correct term.Answer (D) is incorrect because the second term is the reciprocal of the correct term.

[641] Gleim #: 6.14 -- Source: CIA 597 IV-51

A company obtaining short-term financing with trade credit will pay a higher percentage financing cost, everything elsebeing equal, when

The discount percentage is lower.A.The items purchased have a higher price.B.The items purchased have a lower price.C.The supplier offers a longer discount period.D.

Answer (A) is incorrect because the lower the discount percentage, the lower the opportunity cost of forgoing thediscount and using the trade credit financing.Answer (B) is incorrect because percentage financing cost is unaffected by the purchase price of the items.Answer (C) is incorrect because percentage financing cost is unaffected by the purchase price of the items.Answer (D) is correct. If the discount period is longer, the days of extra credit obtained by forgoing the discount arefewer. Assuming other factors are constant, the result is that the cost of trade credit, that is, the cost of not taking thediscount, is greater.

[642] Gleim #: 6.15 -- Source: CIA 1192 IV-54

A company has accounts payable of $5 million with terms of 2% discount within 15 days, net 30 days (2/15 net 30). It canborrow funds from a bank at an annual rate of 12%, or it can wait until the 30th day when it will receive revenues to coverthe payment. If it borrows funds on the last day of the discount period in order to obtain the discount, its total cost will be

$51,000 less.A.$75,500 less.B.$100,000 less.C.$24,500 more.D.

Answer (A) is incorrect because $51,000 less is based on a 30-day borrowing period.Answer (B) is correct. The interest cost of borrowing $4,900,000 (98% × $5,000,000) to take advantage of thediscount is $24,500 [$4,900,000 × 12% × (15 ÷ 360)], and the total cost will be $4,924,500. The total cost if thediscount is not taken will be $5,000,000, a difference of $75,500.Answer (C) is incorrect because $100,000 less does not consider the interest paid.Answer (D) is incorrect because $24,500 more reflects interest paid but ignores the discounted price.

[643] Gleim #: 6.16 -- Source: CIA 1196 IV-52

A short-term bank loan will have a higher effective financing cost if it has which combination of characteristics?

A 10% compensating balance and regular interest.A.A 10% compensating balance and discount interest.B.A 20% compensating balance and regular interest.C.A 20% compensating balance and discount interest.D.

Gleim's CIA Test Prep: Part III: Business Analysis and Information Technology(1165 questions)

Copyright 2004 Gleim Publications, Inc. Page 257Printed for g j

Page 258: Part three cia_with_ answers

Answer (A) is incorrect because lower compensating balances and regular interest are less costly.Answer (B) is incorrect because lower compensating balances and regular interest are less costly.Answer (C) is incorrect because lower compensating balances and regular interest are less costly.Answer (D) is correct. The most costly combination of characteristics is a higher compensating balance and discountinterest. The higher the compensating balance, the higher the portion of the loan funds that must be left on deposit with thelender. Hence, the interest paid is charged on a smaller amount of funds available to be used by the borrower, and theeffective cost is higher. Also, discount interest is deducted from the loan funds in advance, resulting in a further increase inthe effective financing cost.

[644] Gleim #: 6.17 -- Source: CIA 596 IV-41

The credit instrument known as a banker’s acceptance

Calls for immediate payment upon delivery of the shipping documents to the bank’s customer and acceptance ofgoods by the bank.

A.

Involves an invoice being signed by the banker upon receipt of goods, after which both the banker and the sellerrecord the transaction on their respective books.

B.

Is a time draft payable on a specified date and guaranteed by the bank.C.Is a method of sales financing in which the bank retains title to the goods until the buyer has completed payment.D.

Answer (A) is incorrect because a sight draft calls for immediate payment upon delivery of the shipping documentsto, and the acceptance of the draft by, the buyer.Answer (B) is incorrect because an open account is a credit arrangement involving only the signing of an invoice bythe buyer.Answer (C) is correct. A time draft (trade acceptance) is a form of commercial draft because it is drawn by a seller onthe buyer; that is, it calls for the buyer to pay a specified amount. The draft and the shipping documents related to thegoods are then sent to the buyer’s bank, which transmits the draft to the buyer. The buyer accepts the draft by signingit. A time draft, however, is similar to a promissory note because it is payable at a specific time in the future ratherthan upon acceptance by the buyer, which is characteristic of a sight draft. If a seller is reluctant to ship goodsbecause of concern about the buyer’s ability to pay a time draft, the seller’s bank may, for a fee, guarantee payment.This banker’s acceptance is an assumption of the obligation to pay at the due date.Answer (D) is incorrect because the description is of a conditional sales contract except that the seller, not the bank,retains title to the goods until the buyer has completed payment.

[645] Gleim #: 6.18 -- Source: CIA 1191 IV-56

An example of secured short-term financing is

Commercial paper.A.A warehouse receipt.B.A revolving credit agreement.C.Trade credit.D.

Answer (A) is incorrect because commercial paper is a type of unsecured, short-term promissory note issued by largefirms to other firms, insurance companies, mutual funds, etc.Answer (B) is correct. A document of title is usually issued by a bailee covering goods in the bailee’s possession orcare (UCC 1-201). It represents ownership of the goods and is ordinarily needed to obtain the goods from the bailee.The two major types of documents of title are bills of lading (issued by carriers) and warehouse receipts. A warehousereceipt is issued by a person engaged in the business of storing goods for hire. Security for short-term inventoryfinancing can be arranged if the debtor places its inventory under the control of the lender or its agent (e.g., a publicwarehouse), and the lender holds the warehouse receipts.Answer (C) is incorrect because a revolving credit agreement is a formal line of credit, usually with a bank, that largefirms often use.Answer (D) is incorrect because accounts payable, or trade credit, is the most common source of unsecured short-term financing.

[646] Gleim #: 6.19 -- Source: CIA 590 IV-50

Factoring is the

Gleim's CIA Test Prep: Part III: Business Analysis and Information Technology(1165 questions)

Copyright 2004 Gleim Publications, Inc. Page 258Printed for g j

Page 259: Part three cia_with_ answers

Selling of accounts receivable by one company to another.A.Selling of inventory by one company to another.B.Conversion of accounts receivable to bad debt on financial statements for accounts that are long overdue.C.Adjustment of inventories on financial statements for supplies that have become obsolete.D.

Answer (A) is correct. A factor purchases a company’s accounts receivable and assumes the risk of collection. The sellerreceives money immediately to reinvest in new inventories. The financing cost is usually high:  about 2 points or moreabove prime, plus a fee for collection. Factoring has been traditional in the textile industry for years, and recently companiesin many industries have found it an efficient means of operation. A company that uses a factor can eliminate its creditdepartment, accounts receivable staff, and bad debts. These reductions in costs can more than offset the fee charged by thefactor, which can often operate more efficiently than its clients because of the specialized nature of its service.Answer (B) is incorrect because factoring is a source of short-term funds through sale of receivables.Answer (C) is incorrect because factoring is a source of short-term funds through sale of receivables.Answer (D) is incorrect because factoring is a source of short-term funds through sale of receivables.

[647] Gleim #: 6.20 -- Source: CMA 1286 1-35

The following forms of short-term borrowing are available to a firm:

Floating lienFactoringRevolving creditChattel mortgagesBankers’ acceptancesLines of creditCommercial paper

The forms of short-term borrowing that are unsecured credit are

Floating lien, revolving credit, chattel mortgage, and commercial paper.A.Factoring, chattel mortgage, bankers’ acceptances, and line of credit.B.Floating lien, chattel mortgage, bankers’ acceptances, and line of credit.C.Revolving credit, bankers’ acceptances, line of credit, and commercial paper.D.

Answer (A) is incorrect because a chattel mortgage is a loan secured by personal property (movable property such asequipment or livestock). Also, a floating lien is secured by property, such as inventory, the composition of which maybe constantly changing. Factoring is a form of financing in which receivables serve as security.Answer (B) is incorrect because a chattel mortgage is a loan secured by personal property (movable property such asequipment or livestock). Also, a floating lien is secured by property, such as inventory, the composition of which maybe constantly changing. Factoring is a form of financing in which receivables serve as security.Answer (C) is incorrect because a chattel mortgage is a loan secured by personal property (movable property such asequipment or livestock). Also, a floating lien is secured by property, such as inventory, the composition of which maybe constantly changing. Factoring is a form of financing in which receivables serve as security.Answer (D) is correct. An unsecured loan is a loan made by a bank based on credit information about the borrowerand the ability of the borrower to repay the obligation. The loan is not secured by collateral, but is made on thesignature of the borrower. Unsecured credit is not backed by collateral. Revolving credit, bankers’ acceptances, linesof credit, and commercial paper are all unsecured means of borrowing. A chattel mortgage is a loan secured bypersonal property (movable property such as equipment or livestock). A floating lien is also secured by property, suchas inventory, the composition of which may be constantly changing. Factoring is a form of financing in whichreceivables serve as security.

[648] Gleim #: 6.21 -- Source: CIA 597 IV-42

A company has made the decision to finance next year’s capital projects through debt rather than additional equity. Thebenchmark cost of capital for these projects should be

The before-tax cost of new-debt financing.A.The after-tax cost of new-debt financing.B.The cost of equity financing.C.The weighted-average cost of capital.D.

Gleim's CIA Test Prep: Part III: Business Analysis and Information Technology(1165 questions)

Copyright 2004 Gleim Publications, Inc. Page 259Printed for g j

Page 260: Part three cia_with_ answers

Answer (A) is incorrect because the cost of capital is a composite, or weighted average, of all financing sources in theirusual proportions. The cost of capital should also be calculated on an after-tax basis.Answer (B) is incorrect because the cost of capital is a composite, or weighted average, of all financing sources in theirusual proportions. The cost of capital should also be calculated on an after-tax basis.Answer (C) is incorrect because the cost of capital is a composite, or weighted average, of all financing sources in theirusual proportions. The cost of capital should also be calculated on an after-tax basis.Answer (D) is correct. A weighted average of the costs of all financing sources should be used, with the weights determinedby the usual financing proportions. The terms of any financing raised at the time of initiating a particular project do notrepresent the cost of capital for the firm. When a firm achieves its optimal capital structure, the weighted-average cost ofcapital is minimized. The cost of capital is a composite, or weighted average, of all financing sources in their usualproportions. The cost of capital should also be calculated on an after-tax basis.

[649] Gleim #: 6.22 -- Source: CIA 597 IV-50

When a company increases its degree of financial leverage (DFL),

The equity beta of the company falls.A.The systematic risk of the company falls.B.The systematic risk of the company rises.C.The standard deviation of returns on the equity of the company rises.D.

Answer (A) is incorrect because an increase in the DFL increases the riskiness of the firm’s shares. Thus, beta rises.Beta is a measure of the volatility of a firm’s share price relative to the average share.Answer (B) is incorrect because systematic risk, also known as market risk, is unrelated to the DFL. Systematic risk isnot specific to a company. It is the risk associated with a company’s shares that cannot be diversified because it arisesfrom factors that affect all shares.Answer (C) is incorrect because systematic risk, also known as market risk, is unrelated to the DFL. Systematic risk isnot specific to a company. It is the risk associated with a company’s shares that cannot be diversified because it arisesfrom factors that affect all shares.Answer (D) is correct. The DFL equals the percentage change in EPS (earnings available to common shareholders)divided by the percentage change in net operating profit. When the DFL rises, fixed interest charges and the riskinessof the firm rise. As a result, the variability of returns will increase. In other words, the standard deviation of returns ofthe company rises.

[650] Gleim #: 6.23 -- Source: CIA 597 IV-52

In its first year of operations, a firm had $50,000 of fixed operating costs. It sold 10,000 units at a $10 unit price andincurred variable costs of $4 per unit. If all prices and costs will be the same in the second year and sales are projected torise to 25,000 units, what will the degree of operating leverage (the extent to which fixed costs are used in the firm’soperations) be in the second year?

1.25A.1.50B.2.0C.6.0D.

Answer (A) is incorrect because 1.25 ignores variable costs.Answer (B) is correct. The DOL may be calculated as the contribution margin (sales – variable cost) divided by theexcess of the contribution margin over fixed costs. The contribution margin is $150,000 [25,000 units × ($10 unitprice – $4 unit variable cost)]. Hence, the DOL in the second year is 1.50 [$150,000 CM ÷ ($150,000 – $50,000FC)].Answer (C) is incorrect because 2.0 uses the first-year sales level and ignores variable costs.Answer (D) is incorrect because 6.0 uses the first-year sales level.

[651] Gleim #: 6.24 -- Source: CIA 597 IV-53

If a company has a higher dividend-payout ratio, then, if all else is equal, it will have

A higher marginal cost of capital.A.A lower marginal cost of capital.B.A higher investment opportunity schedule.C.A lower investment opportunity schedule.D.

Gleim's CIA Test Prep: Part III: Business Analysis and Information Technology(1165 questions)

Copyright 2004 Gleim Publications, Inc. Page 260Printed for g j

Page 261: Part three cia_with_ answers

Answer (A) is correct. The higher the dividend-payout ratio, the sooner retained earnings are exhausted and the companymust seek external financing. Assuming the same investments are undertaken, the result is a higher marginal cost of capitalbecause lower-cost capital sources will be used up earlier.Answer (B) is incorrect because the marginal cost of capital is higher.Answer (C) is incorrect because the existence of investment opportunities is unrelated to the dividend payout.Answer (D) is incorrect because the existence of investment opportunities is unrelated to the dividend payout.

[652] Gleim #: 6.25 -- Source: CIA 1196 IV-49

If two companies, company X and company Y, are alike in all respects except that company X employs more debtfinancing and less equity financing than company Y does, which of the following statements is true?

Company X has more net earnings variability than company Y.A.Company X has more operating earnings variability than company Y.B.Company X has less operating earnings variability than company Y.C.Company X has less financial leverage than company Y.D.

Answer (A) is correct. Given that company X is more highly leveraged, it has greater fixed financing charges thancompany Y. Interest payments are fixed financing charges, but common share dividends are not. As a result, companyX will be more risky and therefore will have a more volatile net income stream than company Y, if other factors areconstant.Answer (B) is incorrect because the level of fixed financing charges does not affect operating income variability.Answer (C) is incorrect because the level of fixed financing charges does not affect operating income variability.Answer (D) is incorrect because company X has greater, not less, financial leverage than company Y. Greater use ofdebt financing means that a company has greater financial leverage.

[653] Gleim #: 6.26 -- Source: CIA 1196 IV-25

The market value of a firm’s outstanding common shares will be higher, everything else equal, if

Investors have a lower required return on equity.A.Investors expect lower dividend growth.B.Investors have longer expected holding periods.C.Investors have shorter expected holding periods.D.

Answer (A) is correct. The dividend growth model is used to calculate the cost of equity. The simplified formula is

R is the required rate of return, D is the next dividend, P is the share’s price, and G is the growth rate in earnings pershare. The equation is also used to determine the share price.

Thus, when investors have a lower required return on equity, the denominator is smaller, which translates into ahigher market value.Answer (B) is incorrect because, if investors expect lower dividend growth, the market value of common shares willdecrease.Answer (C) is incorrect because the expected holding periods of investors are not related to the market value of thecommon shares.Answer (D) is incorrect because the expected holding periods of investors are not related to the market value of thecommon shares.

[654] Gleim #: 6.27 -- Source: CIA 593 IV-49

Assume that nominal interest rates just increased substantially but that the expected future dividends for a company overthe long run were not affected. As a result of the increase in nominal interest rates, the company’s share price should

Increase.A.Decrease.B.Stay constant.C.Change, but in no obvious direction.D.

Gleim's CIA Test Prep: Part III: Business Analysis and Information Technology(1165 questions)

Copyright 2004 Gleim Publications, Inc. Page 261Printed for g j

Page 262: Part three cia_with_ answers

Answer (A) is incorrect because a higher interest rate raises the required return of investors, which results in a lower shareprice.Answer (B) is correct. The dividend growth model is used to calculate the price of a share.

If: P0  D1  RS  G

Assuming that D1 and G remain constant, an increase in RS resulting from an increase in the nominal interest rate will causeP0 to decrease. A higher interest rate raises the required return of investors, which results in a lower share price.Answer (C) is incorrect because a higher interest rate raises the required return of investors, which results in a lower shareprice.Answer (D) is incorrect because a higher interest rate raises the required return of investors, which results in a lower shareprice.

[655] Gleim #: 6.28 -- Source: CIA 594 IV-80

A company has 10,000 outstanding shares with a market value of $25 each. It just paid a $1 per share dividend. Dividendsare expected to grow at a constant rate of 10%. If flotation costs are 5% of the selling price, the cost of new equityfinancing is calculated by the following formula:

A. 

B. 

C. 

D. 

Gleim's CIA Test Prep: Part III: Business Analysis and Information Technology(1165 questions)

Copyright 2004 Gleim Publications, Inc. Page 262Printed for g j

Page 263: Part three cia_with_ answers

Answer (A) is incorrect because the end-of-period expected dividend and the net proceeds of the issue should be included incalculating the dividend yield.Answer (B) is incorrect because the end-of-period expected dividend should be included in calculating the dividend yield.Answer (C) is incorrect because the net proceeds, not market price, should be included in calculating the dividend yield.Answer (D) is correct. The cost of new equity is calculated by adding the expected dividend yield, based on the netproceeds of the new issue, to the expected dividend growth rate. The expected dividend at the end of the period equals thedividend at time zero times one plus the expected dividend growth rate. Net proceeds received by the firm when issuing onecommon share equals the market price of a share times one minus the flotation cost percentage. Flotation costs includeitems such as underwriting fees, printing, and advertising. The calculation of the cost of new equity is as follows:

[656] Gleim #: 6.29 -- Source: CIA 594 IV-48

The marginal cost of debt for a firm is defined as the interest rate on <List A> debt minus the <List B>.

  List A List B A. New Firm’s marginal tax rateB. Outstanding Firm’s marginal tax rateC. New Interest rate times the firm’s

marginal tax rateD. Outstanding Interest rate times the firm’s

marginal tax rate

Answer (A) is incorrect because the marginal cost of debt financing is the interest rate on new debt minus the firm’smarginal tax rate multiplied by the interest rate. Moreover, the marginal or incremental cost of debt to the firm isbased on the cost of newly issued debt, not on the cost of outstanding debt.Answer (B) is incorrect because the marginal cost of debt financing is the interest rate on new debt minus the firm’smarginal tax rate multiplied by the interest rate. Moreover, the marginal or incremental cost of debt to the firm isbased on the cost of newly issued debt, not on the cost of outstanding debt.Answer (C) is correct. The marginal cost of debt must equal the cost of new debt minus the tax savings. Hence,marginal cost equals the cost of new debt times one minus the marginal tax rate, or Kd(1 – T). This expression equalsKd – KdT. The marginal cost of debt financing is the interest rate on new debt minus the firm’s marginal tax ratemultiplied by the interest rate. Moreover, the marginal or incremental cost of debt to the firm is based on the cost ofnewly issued debt, not on the cost of outstanding debt.Answer (D) is incorrect because the marginal cost of debt financing is the interest rate on new debt minus the firm’smarginal tax rate multiplied by the interest rate. Moreover, the marginal or incremental cost of debt to the firm isbased on the cost of newly issued debt, not on the cost of outstanding debt.

Gleim's CIA Test Prep: Part III: Business Analysis and Information Technology(1165 questions)

Copyright 2004 Gleim Publications, Inc. Page 263Printed for g j

Page 264: Part three cia_with_ answers

[Fact Pattern #49]

A company has the following three investment projectsavailable:

Project Cost Internal Rate ofReturn

A $  50 million 14%

B $  75 million 12%

C $125 million 8%

The company has a 40% debt and 60% equity capitalstructure. Each dollar of investment funds will be raised inthese proportions (40 cents of debt and 60 cents of equity).

The marginal cost of financing increases with the amount ofnew funds raised, as follows:

Interval Amount Raised Weighted-AverageCost of Capital

1 First $  75 million 6%

2 Next $100 million 10%

3 Over $175 million 12%

These investment opportunities and financing costs areshown in the graph below:

(Refer to Figure CIA3_6_46.)

MCC = Marginal cost of capital

IOS = Investment opportunity schedule

[657] Gleim #: 6.30 -- Source: CIA 596 IV-48

(Refers to Fact Pattern #49)The investment opportunity schedule (IOS) shows, in rank order, how much money the company would invest at differentrates of return. Such schedules can be drawn only for a set of projects that

Have the same investment cost.A.Are mutually exclusive.B.Have the same net present value.C.Are independent.D.

Answer (A) is incorrect because IOS schedules do not require that all projects have the same investment cost. Thesteps of the schedule can be of varying lengths.Answer (B) is incorrect because IOS schedules cannot be drawn for mutually exclusive projects.Answer (C) is incorrect because IOS schedules do not require that all projects have the same NPV. The NPV of eachproject depends on the investment cost and on the present value of the expected cash flows. Both costs and cashflows can vary for projects on an IOS.Answer (D) is correct. An IOS schedule is drawn for a set of independent projects. The decision to be made iswhether to accept or reject each project without regard to other investment opportunities. Thus, the cash flows of oneindependent project are not influenced by those of another. Independence should be distinguished from mutualexclusivity. Projects are mutually exclusive if acceptance of one requires rejection of the other.

[658] Gleim #: 6.31 -- Source: CIA 596 IV-49

(Refers to Fact Pattern #49)The company should invest in Project(s) <List A> and has an optimal capital budget of <List B> million dollars.

  List A List B A. B only 75B. A and B only 125C. A and C only 175D. C only 125

Gleim's CIA Test Prep: Part III: Business Analysis and Information Technology(1165 questions)

Copyright 2004 Gleim Publications, Inc. Page 264Printed for g j

Page 265: Part three cia_with_ answers

Answer (A) is incorrect because both A and B should be undertaken.Answer (B) is correct. The intersection of the IOS and MCC schedules determines the cost of capital and the optimalcapital budget. The company should begin with the project having the highest return and continue accepting projects as longas the IRR exceeds the MCC. The highest ranked project is A, with a $50 million cost and a 14% IRR. The MCC is only6% over this range of financing. The next highest ranked project is B, with a $75 million cost and a 12% IRR. When $125million has been invested, the marginal cost of the next dollar of capital is 10%, so Project B is also acceptable, bringing theoptimal capital budget to $125 million. Project C is not acceptable because it has an 8% return. The MCC is 10% for thefirst $50 million invested in this project and 12% for the remaining $75 million.Answer (C) is incorrect because A is acceptable, but C is not.Answer (D) is incorrect because C is not acceptable. It offers an IRR less than the marginal cost of financing the project.

[659] Gleim #: 6.32 -- Source: CIA 596 IV-50

(Refers to Fact Pattern #49)Without prejudice to your answer to any other question, assume that the optimal capital budget for the company is $150million. The marginal cost of capital and the appropriate discount rate to use in evaluating investment proposals for thiscompany would be

6%A.8%B.10%C.12%D.

Answer (A) is incorrect because 6% applies only to the first $75 million of new financing. Given that the optimalcapital budget exceeds $75 million, 6% cannot be the discount rate.Answer (B) is incorrect because 8% is the IRR of Project C.Answer (C) is correct. The appropriate discount rate (the cost of capital used in capital budgeting) theoretically isdetermined at the intersection of the IOS and MCC schedules. This intersection is at an MCC of 10% and an optimalcapital budget of $125 million. However, if the optimal capital budget is assumed to be $150 million, the company isstill in the second interval of the MCC schedule. The marginal cost of financing in this part of the schedule is 10%.Answer (D) is incorrect because at an investment level of $150 million, the MCC is 10%.

[660] Gleim #: 6.33 -- Source: CMA 691 1-9

The most direct way to prepare a cash budget for a manufacturing firm is to include

Projected sales, credit terms, and net income.A.Projected net income, depreciation, and goodwill amortization.B.Projected purchases, percentages of purchases paid, and net income.C.Projected sales and purchases, percentages of collections, and terms of payments.D.

Answer (A) is incorrect because net income includes noncash elements, e.g., goodwill amortization and depreciation.Answer (B) is incorrect because net income includes noncash elements, e.g., goodwill amortization and depreciation.Answer (C) is incorrect because collection percentages must be considered, and net income includes noncashelements.Answer (D) is correct. The most direct way of preparing a cash budget requires incorporation of sales projections andcredit terms, collection percentages, estimated purchases and payment terms, and other cash receipts anddisbursements. In other words, preparation of the cash budget requires consideration of both inflows and outflows.

[661] Gleim #: 6.34 -- Source: CMA Sample Q1-6

Cleveland Masks and Costumes Inc. (CMC) has a majority of its customers located in the states of California and Nevada.Keystone National Bank, a major west coast bank, has agreed to provide a lockbox system to CMC at a fixed fee of$50,000 per year and a variable fee of $0.50 for each payment processed by the bank. On average, CMC receives 50payments per day, each averaging $20,000. With the lockbox system, the company’s collection float will decrease by 2days. The annual interest rate on money market securities is 6%. If CMC makes use of the lockbox system, what would bethe net benefit to the company?  Use 365 days per year.

$59,125A.$60,875B.$50,000C.$120,000D.

Gleim's CIA Test Prep: Part III: Business Analysis and Information Technology(1165 questions)

Copyright 2004 Gleim Publications, Inc. Page 265Printed for g j

Page 266: Part three cia_with_ answers

Answer (A) is incorrect because $59,125 is the annual lockbox cost.Answer (B) is correct. If payments are collected 2 days earlier, the company can earn $120,000 ($20,000 × 50 payments perday × 2 days × .06) at a cost of $59,125 [$50,000 + (50 payments × 365 days × $.50)], a gain of $60,875.Answer (C) is incorrect because $50,000 is the annual fixed fee.Answer (D) is incorrect because $120,000 is the annual savings without regard to costs.

[662] Gleim #: 6.35 -- Source: Publisher

DLF is a retail mail order firm that currently uses a central collection system that requires all checks to be sent to its Bostonheadquarters. An average of 6 days is required for mailed checks to be received, 3 days for DLF to process them, and 2days for the checks to clear through its bank. A proposed lockbox system would reduce the mailing and processing time to2 days and the check clearing time to 1 day. DLF has an average daily collection of $150,000. If DLF adopts the lockboxsystem, its average cash balance will increase by

$1,200,000A.$750,000B.$600,000C.$450,000D.

Answer (A) is correct. Checks are currently tied up for 11 days (6 for mailing, 3 for processing, and 2 for clearing). Ifthat period were reduced to 3 days, DLF’s cash balance would increase by $1,200,000 (8 days × $150,000 per day).Answer (B) is incorrect because the decrease is 8 days, not 5.Answer (C) is incorrect because $600,000 represents only a 4-day savings.Answer (D) is incorrect because the lockbox system will result in an additional 8 days of savings, not 3.

[663] Gleim #: 6.36 -- Source: CMA 694 1-24

Assume that each day a company writes and receives checks totaling $10,000. If it takes 5 days for the checks to clear andbe deducted from the company’s account, and only 4 days for the deposits to clear, what is the float?

$10,000A.$0B.$(10,000)C.$50,000D.

Answer (A) is correct. The float period is the time between when a check is written and when it clears the payor’schecking account. Check float results in an interest-free loan to the payor because of the delay between payment bycheck and its deduction from the bank account. If checks written require one more day to clear than checks received,the net float equals one day’s receipts. The company will have free use of the money for one day. In this case, theamount is $10,000.Answer (B) is incorrect because the company enjoys one day’s net float because its checks clear more slowly than itsdeposits.Answer (C) is incorrect because the net float is positive. The company can write checks (up to $10,000) even when ithas no money because the checks do not clear until a day after deposits clear.Answer (D) is incorrect because the net float represents the difference between when deposits clear and whendisbursements clear.

[664] Gleim #: 6.37 -- Source: CMA 696 1-11

A company obtained a short-term bank loan of $250,000 at an annual interest rate of 6%. As a condition of the loan, thecompany is required to maintain a compensating balance of $50,000 in its checking account. The company’s checkingaccount earns interest at an annual rate of 2%. Ordinarily, the company maintains a balance of $25,000 in its checkingaccount for transaction purposes. What is the effective interest rate of the loan?

6.44%A.7.00%B.5.80%C.6.66%D.

Gleim's CIA Test Prep: Part III: Business Analysis and Information Technology(1165 questions)

Copyright 2004 Gleim Publications, Inc. Page 266Printed for g j

Page 267: Part three cia_with_ answers

Answer (A) is correct. The $50,000 compensating balance requirement is partially satisfied by the company’s practice ofmaintaining a $25,000 balance for transaction purposes. Thus, only $25,000 of the loan will not be available for current use,leaving $225,000 of the loan usable. At 6% interest, the $250,000 loan would require an interest payment of $15,000 peryear. This is partially offset by the 2% interest earned on the $25,000 incremental balance, or $500. Subtracting the $500interest earned from the $15,000 of expense results in net interest expense of $14,500 for the use of $225,000 in funds.Dividing $14,500 by $225,000 produces an effective interest rate of 6.44%.Answer (B) is incorrect because 7.00% fails to consider that the $25,000 currently being maintained counts toward thecompensating balance requirement.Answer (C) is incorrect because 5.8% fails to consider the compensating balance requirement.Answer (D) is incorrect because 6.66% fails to consider the interest earned on the incremental balance being carried.

[665] Gleim #: 6.38 -- Source: CMA 691 1-12

Which one of the following is not a characteristic of a negotiable certificate of deposit?  Negotiable certificates of deposit

Have a secondary market for investors.A.Are regulated by the Federal Reserve System.B.Are usually sold in denominations of a minimum of $100,000.C.Have yields considerably greater than bankers’ acceptances and commercial paper.D.

Answer (A) is incorrect because negotiable CDs do have a secondary market (i.e., they are negotiable).Answer (B) is incorrect because negotiable CDs are regulated.Answer (C) is incorrect because negotiable CDs are typically issued in a denomination of $100,000.Answer (D) is correct. A certificate of deposit (CD) is a form of savings deposit that cannot be withdrawn beforematurity without incurring a high penalty. A negotiable CD can be traded. CDs usually have a fairly high rate ofreturn compared with other savings instruments because they are for fixed, usually long-term periods. However, theiryield is less than that of commercial paper and bankers’ acceptances because they are less risky.

[666] Gleim #: 6.39 -- Source: Publisher

A firm has daily cash receipts of $300,000. A bank has offered to provide a lockbox service that will reduce the collectiontime by 3 days. The bank requires a monthly fee of $2,000 for providing this service. If money market rates are expected toaverage 6% during the year, the additional annual income (loss) of using the lockbox service is

$(24,000)A.$12,000B.$30,000C.$54,000D.

Answer (A) is incorrect because $(24,000) ignores the additional interest revenue from investing the increased funds.Answer (B) is incorrect because $12,000 is based on 2 days of accelerated inflows rather than 3.Answer (C) is correct. Because collections will be accelerated by 3 days at a rate of $300,000 per day, the companywill have an additional $900,000 to invest. At a rate of 6%, the interest earned will be $4,000 per year. However, thebank will charge $24,000 (12 months × $2,000 per month) for its services. Thus, the firm wil increase its income by$30,000 ($54,000 – $24,000).Answer (D) is incorrect because $54,000 ignores the $24,000 bank service charge.

[667] Gleim #: 6.40 -- Source: CMA 1295 1-12

When managing cash and short-term investments, a corporate treasurer is primarily concerned with

Maximizing rate of return.A.Minimizing taxes.B.Investing in Treasury bonds since they have no default risk.C.Liquidity and safety.D.

Gleim's CIA Test Prep: Part III: Business Analysis and Information Technology(1165 questions)

Copyright 2004 Gleim Publications, Inc. Page 267Printed for g j

Page 268: Part three cia_with_ answers

Answer (A) is incorrect because most companies are not in business to earn high returns on liquid assets (i.e., they are heldto facilitate operations).Answer (B) is incorrect because the holding of cash and cash-like assets is not a major factor in controlling taxes.Answer (C) is incorrect because investments in Treasury bonds do not have sufficient liquidity to serve as short-term assets.Answer (D) is correct. Cash and short-term investments are crucial to a firm’s continuing success. Sufficient liquidity mustbe available to meet payments as they come due. At the same time, liquid assets are subject to significant control risk.Therefore, liquidity and safety are the primary concerns of the treasurer when dealing with highly liquid assets. Cash andshort-term investments are held because of their ability to facilitate routine operations of the company. These assets are notheld for purposes of achieving investment returns.

[668] Gleim #: 6.41 -- Source: CMA 697 1-8

Garo Company, a retail store, is considering foregoing sales discounts in order to delay using its cash. Supplier credit termsare 2/10, net 30. Assuming a 360-day year, what is the annual cost of credit if the cash discount is not taken and Garo paysnet 30?

24.0%A.24.5%B.36.0%C.36.7%D.

Answer (A) is incorrect because the length of the extra credit period is 20 days, not 30 days.Answer (B) is incorrect because the length of the extra credit period is 20 days, not 30 days.Answer (C) is incorrect because 36.0% calculates the interest rate based on the full invoice price.Answer (D) is correct. On a $1,000 invoice, the company could save $20 by paying within the discount period. Thus,an immediate payment of $980 would save the company $20, and the interest rate charged for holding $980 anadditional 20 days (30 – 10) is 2.04% ($20/$980). Because the number of 20-day periods in a year is 18 (360/20), theannual rate is 36.7% (18 × 2.04%).

[669] Gleim #: 6.42 -- Source: CMA 697 1-14

The sales manager at Ryan Company feels confident that, if the credit policy at Ryan’s were changed, sales would increaseand, consequently, the company would utilize excess capacity. The two credit proposals being considered are as follows:

  Proposal A Proposal B Increase in sales $500,000 $600,000Contribution margin 20% 20%Bad debt percentage 5% 5%Increase in operating profits $75,000 $90,000Desired return on sales 15% 15%

Currently, payment terms are net 30. The proposed payment terms for Proposal A and Proposal B are net 45 and net 90,respectively. An analysis to compare these two proposals for the change in credit policy would include all of the followingfactors except the

Cost of funds for Ryan.A.Current bad debt experience.B.Impact on the current customer base of extending terms to only certain customers.C.Bank loan covenants on days’ sales outstanding.D.

Answer (A) is incorrect because the cost of funds is an obvious element in the analysis of any investment.Answer (B) is correct. All factors should be considered that differ between the two policies. Factors that do notdiffer, such as the current bad debt experience, are not relevant. Ryan must estimate the expected bad debt lossesunder each new policy.Answer (C) is incorrect because the impact on the current customer base of extending terms to only certain customersis relevant. The current customers may demand the same terms.Answer (D) is incorrect because existing loan agreements may require Ryan to maintain certain ratios at stated levels.Thus, Ryan’s ability to increase receivables and possible bad debt losses may be limited.

[670] Gleim #: 6.43 -- Source: CMA 1295 1-4

The average collection period for a firm measures the number of days

Gleim's CIA Test Prep: Part III: Business Analysis and Information Technology(1165 questions)

Copyright 2004 Gleim Publications, Inc. Page 268Printed for g j

Page 269: Part three cia_with_ answers

After a typical credit sale is made until the firm receives the payment.A.For a typical check to “clear” through the banking system.B.Beyond the end of the credit period before a typical customer payment is received.C.Before a typical account becomes delinquent.D.

Answer (A) is correct. The average collection period measures the number of days between the date of sale and the date ofcollection. It should be related to a firm’s credit terms. For example, a firm that allows terms of 2/15, net 30, should have anaverage collection period of somewhere between 15 and 30 days.Answer (B) is incorrect because it describes the concept of float.Answer (C) is incorrect because the average collection period includes the total time before a payment is received, includingthe periods both before and after the end of the normal credit period.Answer (D) is incorrect because it describes the normal credit period.

[671] Gleim #: 6.44 -- Source: CMA 1292 1-20

Best Computers believes that its collection costs could be reduced through modification of collection procedures. Thisaction is expected to result in a lengthening of the average collection period from 28 days to 34 days; however, there willbe no change in uncollectible accounts. The company’s budgeted credit sales for the coming year are $27,000,000, andshort-term interest rates are expected to average 8%. To make the changes in collection procedures cost beneficial, theminimum savings in collection costs (using a 360-day year) for the coming year would have to be

$30,000.A.$360,000.B.$180,000.C.$36,000.D.

Answer (A) is incorrect because $36,000 is the minimum savings required.Answer (B) is incorrect because $36,000 is the minimum savings required.Answer (C) is incorrect because $36,000 is the minimum savings required.Answer (D) is correct. Given sales of $27,000,000, the average amount of daily sales must be $75,000 ($27,000,000÷ 360 days). The increased accounts receivable balance is therefore $450,000 (6 days × $75,000). With an additional$450,000 of capital invested in receivables, the company’s interest cost will increase by $36,000 per year (8% ×$450,000). Thus, the company must save at least $36,000 per year to justify the change in procedures.

[672] Gleim #: 6.45 -- Source: CMA 1294 1-22

A firm averages $4,000 in sales per day and is paid, on an average, within 30 days of the sale. After they receive theirinvoice, 55% of the customers pay by check, while the remaining 45% pay by credit card. Approximately how much wouldthe company show in accounts receivable on its balance sheet on any given date?

$4,000A.$120,000B.$48,000C.$54,000D.

Answer (A) is incorrect because $4,000 is only one day’s sales.Answer (B) is correct. If sales are $4,000 per day, and customers pay in 30 days, 30 days of sales are outstanding, or$120,000. Whether customers pay by credit card or cash, collection requires 30 days.Answer (C) is incorrect because invoices are outstanding for 30 days, not 12 days.Answer (D) is incorrect because $54,000 is based on the 45% of collections via credit card.

[673] Gleim #: 6.46 -- Source: Publisher

A common mistake in valuing the firm to be acquired in a business combination is

Using market values in the valuation.A.Including incremental cash flows in the valuation.B.Using the acquiring firm’s discount rate when valuing the incremental cash flows.C.Including all related transaction costs associated with an acquisition.D.

Gleim's CIA Test Prep: Part III: Business Analysis and Information Technology(1165 questions)

Copyright 2004 Gleim Publications, Inc. Page 269Printed for g j

Page 270: Part three cia_with_ answers

Answer (A) is incorrect because market values is an essential element of the valuation.Answer (B) is incorrect because incremental cash flow is an essential element of the valuation.Answer (C) is correct. If the net incremental cash flows to the acquiring firm’s shareholders are to be valued, the discountrate used should be the cost of equity capital. Moreover, this rate should reflect the risk associated with the use of fundsrather than their source. The rate therefore should not be the cost of capital of the acquiring firm but rather the cost of equityof the combined firm after the combination. This calculation requires a new estimate of beta to be used in the Capital AssetPricing Model.Answer (D) is incorrect because transaction costs is an essential element of the valuation.

[674] Gleim #: 6.47 -- Source: Publisher

For the past several years, M.F.S. Company has invested in the common stock of Annabelle Company. M.F.S. currentlyowns approximately 13% of the total of Annabelle’s outstanding voting common stock. Recently, managements of the twocompanies have discussed a possible combination of the two entities. If they do decide to combine, the resultingcombination should be accounted for as a

Pooling of interests.A.Purchase.B.Part purchase, part pooling.C.Joint venture.D.

Answer (A) is incorrect because the pooling-of-interests method is no longer an acceptable method of accounting forbusiness combinations.Answer (B) is correct. SFAS 141, Business Combinations, requires that all business combinations be accounted forusing the purchase method.Answer (C) is incorrect because accounting for a business combination as part purchase and part pooling is notallowed.Answer (D) is incorrect because a business combination cannot be accounted for as a joint venture.

[675] Gleim #: 6.48 -- Source: CIA 590 IV-57

A company has recently purchased some stock of a competitor as part of a long-term plan to acquire the competitor.However, it is somewhat concerned that the market price of this stock could decrease over the short run. The companycould hedge against the possible decline in the stock’s market price by

Purchasing a call option on that stock.A.Purchasing a put option on that stock.B.Selling a put option on that stock.C.Obtaining a warrant option on that stock.D.

Answer (A) is incorrect because a call option is the right to purchase shares at a given price within a specified period.Answer (B) is correct. A put option is the right to sell stock at a given price within a certain period. If the marketprice falls, the put option may allow the sale of stock at a price above market, and the profit of the option holder willbe the difference between the price stated in the put option and the market price, minus the cost of the option,commissions, and taxes. The company that issues the stock has nothing to do with put (and call) options.Answer (C) is incorrect because selling a put option could force the company to purchase additional stock if theoption is exercised.Answer (D) is incorrect because a warrant gives the holder a right to purchase stock from the issuer at a given price(it is usually distributed along with debt).

[676] Gleim #: 6.49 -- Source: Publisher

If a call option is “out-of-the-money,”

The option has expired.A.The value of the underlying asset is less than the exercise price.B.The option no longer exists.C.The option has become a put option.D.

Gleim's CIA Test Prep: Part III: Business Analysis and Information Technology(1165 questions)

Copyright 2004 Gleim Publications, Inc. Page 270Printed for g j

Page 271: Part three cia_with_ answers

Answer (A) is incorrect because an out-of-the-money option may not have expired.Answer (B) is correct. When the value of the asset underlying a call option is less than the exercise price of the option, theoption is “out-of-money.”Answer (C) is incorrect because the option does exist; it is just not worth exercising.Answer (D) is incorrect because call options do not change into put options.

[677] Gleim #: 6.50 -- Source: Publisher

The type of option that does not have the backing of stock is called a(n)

Covered option.A.Unsecured option.B.Naked option.C.Put option.D.

Answer (A) is incorrect because a covered option is one that is written against stock held in the option writer’sportfolio.Answer (B) is incorrect because an unsecured option is a nonsense term.Answer (C) is correct. A naked or uncovered option is a call option that does not have the backing of stock. Thus,the option writer will have to purchase the underlying stock if the call option is exercised.Answer (D) is incorrect because a put option is an option that gives the owner the right to sell the underlying asset fora fixed price.

[678] Gleim #: 6.51 -- Source: Publisher

A contractual arrangement that gives the owner the right to buy or sell an asset at a fixed price at any moment in timebefore or on a specified date is a(n)

European option.A.Foreign option.B.Future option.C.American option.D.

Answer (A) is incorrect because a European option is exercisable only at the expiration date.Answer (B) is incorrect because a foreign option is a nonsense term.Answer (C) is incorrect because although an option can be exercised in the future, it is not called a future option.Answer (D) is correct. An American option is a contractual arrangement that gives the owner the right to buy or sellan asset at a fixed price at any moment in time before or on a specified date.

[679] Gleim #: 6.52 -- Source: Publisher

The use of derivatives to either hedge or speculate results in

Increased risk regardless of motive.A.Decreased risk regardless of motive.B.Offset risk when hedging and increased risk when speculating.C.Offset risk when speculating and increased risk when hedging.D.

Answer (A) is incorrect because hedging decreases risk by using offsetting commitments that avoid the impact ofadverse price movements.Answer (B) is incorrect because speculation involves the assumption of risk in the hope of gaining from pricemovements.Answer (C) is correct. Derivatives, including options and futures, are contracts between the parties who contract.Unlike stocks and bonds, they are not claims on business assets. A futures contract is entered into as either aspeculation or a hedge. Speculation involves the assumption of risk in the hope of gaining from price movements.Hedging is the process of using offsetting commitments to minimize or avoid the impact of adverse price movements.Answer (D) is incorrect because speculating increases risk while hedging offsets risk.

[680] Gleim #: 6.53 -- Source: Publisher

An automobile company that uses the futures market to set the price of steel to protect a profit against price increases is anexample of

Gleim's CIA Test Prep: Part III: Business Analysis and Information Technology(1165 questions)

Copyright 2004 Gleim Publications, Inc. Page 271Printed for g j

Page 272: Part three cia_with_ answers

A short hedge.A.A long hedge.B.Selling futures to protect the company from loss.C.Selling futures to protect against price declines.D.

Answer (A) is incorrect because a short hedge is a futures contract that is sold to protect against price declines. Theautomobile company wishes to protect itself against price increases.Answer (B) is correct. A change in prices can be minimized or avoided by hedging. Hedging is the process of usingoffsetting commitments to minimize or avoid the impact of adverse price movements. The automobile company desires tostabilize the price of steel so that its cost to the company will not rise and cut into profits. Accordingly, the automobilecompany uses the futures market to create a long hedge, which is a futures contract that is purchased to protect against priceincreases.Answer (C) is incorrect because the automobile company needs to purchase futures in order to protect itself from loss, notsell futures. Selling futures protects against price declines.Answer (D) is incorrect because it is the definition of a short hedge, which is used for avoiding price declines. Theautomobile company wants to protect itself against price increases.

[681] Gleim #: 6.54 -- Source: Publisher

If a corporation holds a forward contract for the delivery of U.S. Treasury bonds in 6 months and, during those 6 months,interest rates decline, at the end of the 6 months the value of the forward contract will have

Decreased.A.Increased.B.Remained constant.C.Any of the answers may be correct, depending on the extent of the decline in interest rates.D.

Answer (A) is incorrect because the value of the forward contract will increase when interest rates decrease.Answer (B) is correct. Interest rate futures contracts involve risk-free bonds, such as U.S. Treasury bonds. Wheninterest rates decrease over the period of a forward contract, the value of the bonds and the forward contract increase.Answer (C) is incorrect because the value of the forward contract will not remain constant if interest rates decline.Answer (D) is incorrect because any decline in interest rates increases the value of the bonds.

[682] Gleim #: 6.55 -- Source: Publisher

A company wishes to price a call option written on a nondividend-paying stock. The current stock price is $50, the exerciseprice is $48, the risk-free interest rate is 5.0%, the option expires in 1 year, and the cumulative probabilities used tocalculate the present values of the final stock price and the exercise price are .65 and .58, respectively. According to theBlack-Scholes Option Pricing Model, the current value of the call option is

$6.02A.$4.66B.$4.02C.$2.00D.

Gleim's CIA Test Prep: Part III: Business Analysis and Information Technology(1165 questions)

Copyright 2004 Gleim Publications, Inc. Page 272Printed for g j

Page 273: Part three cia_with_ answers

Answer (A) is correct. The basic formula is

If C is the current value of a call option with time t in years until expiration, S is the current stock price, N (d i ) is thecumulative probability that a i deviation less than di will occur in a standardized normal distribution [N (di) is an area to theleft of d under the curve for the standard normal distribution], E is the call’s exercise price, e is a constant (approximately2.7183), and r is the annualized continuous risk-free rate of return, the value of the call is

C ($50 × .65) – ($48 × 2.7183(–.05 × 1) × .58)$32.50 – ($48 × .9512 × .58)$32.50 – $26.48$6.02

Answer (B) is incorrect because $4.66 results from omitting the term e(-rt) power from the equation.Answer (C) is incorrect because $4.02 equals the estimated call price minus the difference between the current stock priceand the exercise price.Answer (D) is incorrect because $2.00 is the difference between the current stock price and the exercise price.

[683] Gleim #: 6.56 -- Source: Publisher

A company has purchased a $1,000, 7%, 5-year bond at par that pays interest annually. The discount factors for the presentvalue of $1 at 7% for five periods are as follows:

Period Factor1 .9352 .8733 .8164 .7635 .713

For purposes of duration hedging, the duration of the bond is

5.39 years.A.5.00 years.B.4.39 years.C.3.81 years.D.

Answer (A) is incorrect because 5.39 years results from adding $1,000 to the numerator of the calculation.Answer (B) is incorrect because 5.00 years is the term of the bond.Answer (C) is correct. Duration hedging involves hedging interest-rate risk by matching the duration and value ofassets with the duration and value of liabilities. Duration is the weighted average of the times to interest and principalpayments. If duration increases, the volatility of the price of the debt instrument increases. Duration is lower if thenominal rate on the instrument is higher because more of the return is received earlier. The formula for duration is asfollows if CT is the interest or principal payment, T is the time to the payment, n is the time to maturity, r is the yieldto maturity, and V is the value of the instrument:

Because the expression 1 ÷ (1 + r)T is the present value of $1, the weighted present values of the payments can becalculated as follows:  $65.45 (7% × $1,000 × 1 period × .935), $122.22 (7% × $1,000 × 2 periods × .873), $171.36(7% × $1,000 × 3 periods × .816), $213.64 (7% × $1,000 × 4 periods × .763), and $3,814.55 (107% × $1,000 × 5periods × .713). The total is $4,387.22. The value of the bond is $1,000 {[$70 × (.935 + .873 + .816 + .763 + .713)]+ ($1,000 × .713)}. Thus, the duration is approximately 4.39 years ($4,387.22 ÷ $1,000).Answer (D) is incorrect because 3.81 years equals the weighted present value of the final payment divided by $1,000.

Gleim's CIA Test Prep: Part III: Business Analysis and Information Technology(1165 questions)

Copyright 2004 Gleim Publications, Inc. Page 273Printed for g j

Page 274: Part three cia_with_ answers

[684] Gleim #: 6.57 -- Source: PublisherHerbert Corporation was a party to the following transactions during November and December 2002. Which of thesetransactions is most likely to be defined as a derivative?

Purchased 1,000 shares of common stock of a public corporation based on the assumption that the stock will increasein value.

A.

Purchased a term life insurance policy on the company’s chief executive officer to protect the company from theeffects of an untimely demise of this officer.

B.

Agreed to cosign the note of its 100%-owned subsidiary to protect the lender from the possibility that the subsidiarymight default on the loan.

C.

Based on its forecasted need to purchase 300,000 bushels of wheat in 3 months, entered into a 3-month forwardcontract to purchase 300,000 bushels of wheat to protect itself from changes in wheat prices during the period.

D.

Answer (A) is incorrect because it involves a net investment equal to the fair value of the stock.Answer (B) is incorrect because insuring the CEO’s life is a transaction based on identifiable events, not underlyings.Answer (C) is incorrect because cosigning a subsidiary’s note is a transaction based on identifiable events, notunderlyings.Answer (D) is correct. SFAS 133 defines a derivative as a financial instrument or other contract that (1) has (a) oneor more underlyings and (b) one or more notional amounts or payment provisions, or both; (2) requires either noinitial net investment or an immaterial net investment; and (3) requires or permits net settlement. An underlying maybe a specified interest rate, security price, commodity price, foreign exchange rate, index of prices or rates, or othervariable. A notional amount is a number of currency units, shares, bushels, pounds, or other units specified.Settlement of a derivative is based on the interaction of the notional amount and the underlying. The purchase of theforward contract as a hedge of a forecasted need to purchase wheat meets the criteria prescribed by SFAS 133.

[685] Gleim #: 6.58 -- Source: Publisher

On December 1, 2004, Lombardi Company, a calendar-year-end firm, entered into a derivative contract designed to hedgethe risk of cash flows associated with the forecast future sale of 300,000 bushels of wheat. The anticipated sales date isFebruary 1, 2005. The notional amount of the derivative contract is 300,000 bushels, the underlying is the price of the samevariety and grade of wheat that Lombardi expects to sell, and the settlement date of the derivative is February 1, 2005. Thefair value of the derivative contract on December 31, 2004 increased by $30,000, an amount equal to the decrease in thefair value of the wheat. The fair value of the derivative contract had increased by an additional $25,000 on February 1,2005, also an amount equal to the decrease in the fair value of the wheat. On February 1, 2005, the wheat was sold and thederivative contract was settled. The gains attributable to the increase in the fair value of the derivative that should berecognized in 2004 and 2005 earnings, respectively, are

  2004 2005 A. $30,000 $25,000B. $0 $55,000C. $55,000 $0D. $0 $0

Answer (A) is incorrect because given that the hedge was fully effective, the $55,000 gain should be recognized inthe period in which the forecasted transaction affects earnings.Answer (B) is correct. A cash flow hedge is a hedge of an exposure to variability in the cash flows of a recognizedasset or liability or a forecasted transaction. The accounting treatment of gains and losses arising from changes in fairvalue of a derivative designated as a cash flow hedge varies for the effective and ineffective portions. The effectiveportion initially is reported as other comprehensive income. It is reclassified into earnings when the forecastedtransaction affects earnings. The ineffective portion is immediately included in earnings. This hedge has noineffective portion. Given that the sale occurred in 2005, the $30,000 gain in 2004 is recognized as othercomprehensive income in 2004. It is reclassified and included in earnings in 2005. Thus, 2005 earnings include the$30,000 reclassified from other comprehensive income and the $25,000 gain attributable to the increase in fair valuein 2005. Given that the hedge was fully effective, the $55,000 gain should be recognized in the period in which theforecasted transaction affects earnings.Answer (C) is incorrect because given that the hedge was fully effective, the $55,000 gain should be recognized inthe period in which the forecasted transaction affects earnings.Answer (D) is incorrect because given that the hedge was fully effective, the $55,000 gain should be recognized inthe period in which the forecasted transaction affects earnings.

Gleim's CIA Test Prep: Part III: Business Analysis and Information Technology(1165 questions)

Copyright 2004 Gleim Publications, Inc. Page 274Printed for g j

Page 275: Part three cia_with_ answers

[686] Gleim #: 6.59 -- Source: PublisherAt the beginning of period 1, Forecast Corporation enters into a qualifying cash flow hedge of a transaction it expects tooccur at the beginning of period 4. Forecast assesses hedge effectiveness by comparing the change in present value (PV) ofthe expected cash flows associated with the forecasted transaction with all of the hedging derivative’s gain or loss (changein fair value). The change in those cash flows that occurs for any reason has been designated as the hedged risk. Thefollowing information about the periodic changes hedging relationship is available:

Change in PV of  Change in Expected Cash Flows  Fair Value of from the Forecasted

Period the Derivative Transactions1 $50,000 $(48,000)2 47,000 (51,000)3 (81,000) 80,000

Given that the hedge is effective to the extent it offsets the change in the present value of the expected cash flows on theforecasted transaction, Forecast should

Recognize a loss of $2,000 in earnings for period 1.A.Report a balance in other comprehensive income (OCI) of $16,000 at the end of period 3.B.Recognize a gain of $47,000 in earnings for period 2.C.Record other comprehensive income of $97,000 for period 2.D.

Answer (A) is incorrect because Forecast should recognize earnings for period 1 of $2,000. The increase in fair valueof the derivative exceeds the decrease in PV of the cash flows by $2,000. The derivative is adjusted to fair value by a$50,000 debit, OCI is credited for $48,000, and earnings is credited for $2,000.Answer (B) is correct. The effective portion of a cash flow hedge of a forecasted transaction is included in OCI untilperiods in which the forecasted transaction affects earnings. At the end of period 3, the net change in the hedgingderivative’s fair value is $16,000 ($50,000 + $47,000 – $81,000), and the change in the PV of the expected cashflows on the forecasted transaction is $-19,000 ($80,000 – $48,000 – $51,000). Thus, the hedge is effective at the endof period 3 to the extent it offsets $16,000 of the net $19,000 decrease in the cash flows of the forecasted transactionthat are expected to occur in period 4.Answer (C) is incorrect because the entry for period 2 is to debit the derivative for $47,000, debit earnings for$2,000, and credit OCI for $49,000 ($50,000 + $47,000 – $48,000 credit in period 1).Answer (D) is incorrect because, at the end of period 2, OCI should have a credit balance of $97,000 (the extent ofthe hedge’s effectiveness).

[Fact Pattern #50]

Techspace has been a successful stock over the past fewyears despite its riskiness. The state of the economy has atremendous effect on the expected returns for Techspace:

TechspaceProbability State of the Economy Returns

.05 Depression –45%

.15 Recession –10%

.20 Minimal Slowdown 5%

.40 Stable 10%

.15 Expansion 30%

.05 Significant Expansion 35%

[687] Gleim #: 6.60 -- Source: Publisher

(Refers to Fact Pattern #50)What is the expected rate of return on Techspace shares?

7.5%A.15%B.35%C.25%D.

Gleim's CIA Test Prep: Part III: Business Analysis and Information Technology(1165 questions)

Copyright 2004 Gleim Publications, Inc. Page 275Printed for g j

Page 276: Part three cia_with_ answers

Answer (A) is correct. The expected rate of return on an investment is determined using an expected value calculation. It isan average of the outcomes weighted according to their probabilities. For Techspace, the average is accomplished bymultiplying each probability by the corresponding return for each state of the economy and then calculating the sum of theproducts. Numerically, the calculation is performed as follows: .05(-.45) + .15(-.10) + .2(.05) + .4(.10) + .15(.30) + .05(.35)= .075 = 7.5%.Answer (B) is incorrect because 15% adds positive products instead of negative products for the first two states.Answer (C) is incorrect because 35% is the sum of the returns (assuming all are positive) minus 1.Answer (D) is incorrect because 25% is the sum of the returns.

[688] Gleim #: 6.61 -- Source: Publisher

(Refers to Fact Pattern #50)The variance of Techspace returns is

.1735A.

.0301B.

.075C.

.2738D.

Answer (A) is incorrect because .1735 is the standard deviation of Techspace returns.Answer (B) is correct. The variance (σ2) is calculated using the equation

If: ki is the return for the ith outcome, ␇ is the expected return, and pi is the probability of the ith outcome. 

or   .05(-.45) +.15(-.10)+ .2(.05)

+ .4(.10) + .15(.30) + .05(.35) = .075

σ2 = .05(-.45 – .075)2 + .15(-.1 –.075)2

+ .2(.05 – .075)2 + .4(.1 – .075)2 + .15(.30 – .075)2

+ .05(.35 – .075)2 = .0301

Answer (C) is incorrect because .075 is the expected return.Answer (D) is incorrect because .2738 is the square root of the expected return.

[689] Gleim #: 6.62 -- Source: Publisher

(Refers to Fact Pattern #50)The standard deviation of Techspace returns is

7.5%A.17.35%B.3.01%C.None of the answers are correct.D.

Answer (A) is incorrect because 7.5% is the expected return.Answer (B) is correct. The standard deviation (σ) gives an exact value for the tightness of the distribution and theriskiness of the investment. It is calculated by taking the square root of the variance. Given that the variance is .0301,the standard deviation is .1735, or 17.35%.Answer (C) is incorrect because 3.01% is the variance.Answer (D) is incorrect because the standard deviation is 17.35%.

[690] Gleim #: 6.63 -- Source: Publisher

If the covariance of share A with share B is -.0076, then what is the covariance of share B with share A?

Gleim's CIA Test Prep: Part III: Business Analysis and Information Technology(1165 questions)

Copyright 2004 Gleim Publications, Inc. Page 276Printed for g j

Page 277: Part three cia_with_ answers

+.0076A.-.0076B.Greater than .0076.C.Less than -.0076.D.

Answer (A) is incorrect because the covariance of share B with share A is the same as the covariance of share A with shareB.Answer (B) is correct. The covariance measures the volatility of returns together with their correlation with the returns ofother securities. It is calculated with the following equation:

The covariance of two shares is the same regardless of which share is compared with the other.Answer (C) is incorrect because the covariance of share B with share A is the same as the covariance of share A with shareB.Answer (D) is incorrect because the covariance of share B with share A is the same as the covariance of share A with shareB.

[691] Gleim #: 6.64 -- Source: Publisher

A feasible portfolio that offers the highest expected return for a given risk or the least risk for a given expected return isa(n)

Optimal portfolio.A.Desirable portfolio.B.Efficient portfolio.C.Effective portfolio.D.

Answer (A) is incorrect because an optimal portfolio is a portfolio selected from the efficient set of portfolios becauseit is tangent to the investor’s highest indifference curve.Answer (B) is incorrect because a desirable portfolio is a nonsense term.Answer (C) is correct. A feasible portfolio that offers the highest expected return for a given risk or the least risk fora given expected return is called an efficient portfolio.Answer (D) is incorrect because an effective portfolio is a nonsense term.

[692] Gleim #: 6.65 -- Source: CIA 1196 IV-51

In which stage of a firm's development is it most likely to seek and obtain external equity financing in the form of venturecapital?

Formation.A.Rapid growth.B.Growth to maturity.C.Maturity and industry decline.D.

Answer (A) is incorrect because, during the formation stage, personal savings, trade credit, and government agenciesare the main sources of financing. Prior to demonstrating initial success, a firm is not likely to attract venture capitalfinancing easily.Answer (B) is correct. At the rapid growth stage, if a company is reasonably profitable, it will experience financingneeds in excess of funds available either internally or from trade credit or bank credit. Additional debt financing oftenresults in an unreasonable amount of financial leverage at this stage, and public equity financing ordinarily is not yetavailable. Hence, a rapidly growing firm is most likely to seek and obtain venture capital financing.Answer (C) is incorrect because, in the growth to maturity stage of development, the firm is able to access formalmarkets for debt and equity. It has a record of success and a better balance between cash inflows and outflows than inthe rapid growth stage. Formal capital markets provide financing at lower cost than venture capitalists, so venturecapital is not likely to be sought at this stage.Answer (D) is incorrect because the decline phase is characterized by more than adequate cash flows, relative toavailable investment opportunities, so venture capital is not likely to be sought at this stage of development.

Gleim's CIA Test Prep: Part III: Business Analysis and Information Technology(1165 questions)

Copyright 2004 Gleim Publications, Inc. Page 277Printed for g j

Page 278: Part three cia_with_ answers

[693] Gleim #: 6.66 -- Source: CIA 1196 IV-45A company is arranging debt financing for the purchase of a new piece of equipment that has a 5-year expected useful life.Which of the following alternative financing arrangements has the lowest effective annual percentage rate if each has aquoted nominal rate of 9.5%?

A 5-year term loan with interest compounded annually.A.A 10-year term loan with interest compounded semiannually.B.A 5-year term loan with interest compounded quarterly.C.A 10-year term loan with interest compounded monthly.D.

Answer (A) is correct. For any given quoted nominal rate, the least frequent compounding is associated with thelowest effective annual percentage cost. Annual compounding is less frequent than semiannual, quarterly, or monthly.The term of the loan is not relevant to the calculation of the effective annual percentage cost of financing.Answer (B) is incorrect because, the more frequent the interest compounding, the more costly the loan. Semiannual,quarterly, and monthly compounding are all more frequent than annual compounding.Answer (C) is incorrect because, the more frequent the interest compounding, the more costly the loan. Semiannual,quarterly, and monthly compounding are all more frequent than annual compounding.Answer (D) is incorrect because, the more frequent the interest compounding, the more costly the loan. Semiannual,quarterly, and monthly compounding are all more frequent than annual compounding.

[694] Gleim #: 6.67 -- Source: CIA 1195 IV-65

Which of the following financial instruments can be traded in international money markets?

Mortgages.A.Preferred shares.B.U.S. Treasury bills.C.U.S. Treasury bonds.D.

Answer (A) is incorrect because mortgages are long-term, capital market securities.Answer (B) is incorrect because preferred shares are long-term, capital market securities.Answer (C) is correct. Funds are borrowed or lent for short periods (less than one year) in money markets. Examplesof instruments traded in money markets are U.S. Treasury bills, bankers' acceptances, commercial paper, negotiablecertificates of deposit, money market mutual funds, Eurodollar market time deposits, and consumer credit loans.Capital markets trade shares and long-term debt.Answer (D) is incorrect because U.S. Treasury bonds are long-term, capital market securities.

[695] Gleim #: 6.68 -- Source: CMA 689 1-13

Short-term, unsecured promissory notes issued by large firms are known as

Agency securities.A.Bankers acceptances.B.Commercial paper.C.Repurchase agreements.D.

Answer (A) is incorrect because an agency security is issued by a corporation or agency created by a government.Examples are government securities issued by the bodies that finance mortgages, such as the Federal NationalMortgage Association (Fannie Mae) in the U.S.Answer (B) is incorrect because bankers' acceptances are drafts drawn on deposits at a bank. The acceptance by thebank guarantees payment at maturity. They are normally used to finance a specific transaction.Answer (C) is correct. Commercial paper is the term for the short-term (typically less than 9 months), unsecured,large denomination (often over $100,000) promissory notes issued by large, creditworthy companies to othercompanies and institutional investors. In many instances, the maturity date is only a few days after issuance.Answer (D) is incorrect because a repurchase agreement involves a secured loan to a government securities dealer. Itallows the buyer to retain interest income although the seller-dealer can repurchase after a specified time.

[696] Gleim #: 6.69 -- Source: CIA 1191 IV-58

A firm must select from among several methods of financing arrangements when meeting its capital requirements. Toacquire additional growth capital while attempting to maximize earnings per share, a firm should normally

Gleim's CIA Test Prep: Part III: Business Analysis and Information Technology(1165 questions)

Copyright 2004 Gleim Publications, Inc. Page 278Printed for g j

Page 279: Part three cia_with_ answers

Attempt to increase both debt and equity in equal proportions, which preserves a stable capital structure and maintainsinvestor confidence.

A.

Select debt over equity initially, even though increased debt is accompanied by interest costs and a degree of risk.B.Select equity over debt initially, which minimizes risk and avoids interest costs.C.Discontinue dividends and use current cash flow, which avoids the cost and risk of increased debt and the dilution of EPSthrough increased equity.

D.

Answer (A) is incorrect because EPS is not a function of investor confidence and is not maximized by concurrentproportional increases in both debt and equity. EPS are usually higher if debt is used instead of equity to raise capital, atleast initially.Answer (B) is correct. Earnings per share will ordinarily be higher if debt is used to raise capital instead of equity, providedthat the firm is not over-leveraged. The reason is that the cost of debt is lower than the cost of equity because interest is taxdeductible. However, the prospect of higher EPS is accompanied by greater risk to the firm resulting from required interestcosts, creditors' liens on the firm's assets, and the possibility of a proportionately lower EPS if sales volume fails to meetprojections.Answer (C) is incorrect because equity capital is initially more costly than debt.Answer (D) is incorrect because using only current cash flow to raise capital is usually too conservative an approach for agrowth-oriented firm. Management is expected to be willing to take acceptable risks to be competitive and attain anacceptable rate of growth.

[697] Gleim #: 6.70 -- Source: CIA 596 IV-51

The marginal cost of capital (MCC) curve for this company rises twice, first when the company has raised $75 million andagain when $175 million of new funds has been raised. These increases in the MCC are caused by

Increases in the returns on the additional investments undertaken.A.Decreases in the returns on the additional investments undertaken.B.Decreases in the cost of at least one of the financing sources.C.Increases in the cost of at least one of the financing sources.D.

Answer (A) is incorrect because financing costs do not depend on rates of return on investment.Answer (B) is incorrect because financing costs do not depend on rates of return on investment.Answer (C) is incorrect because as additional funds are raised, an increase in the cost of a source of financing, not adecrease, will result in an increase in the MCC.Answer (D) is correct. The MCC is a weighted average of the costs of the different financing sources. If the cost ofany source of financing increases, the MCC curve will rise. The MCC curve is upward sloping because the lowestcost financing sources are assumed to be used first. Thus, as cumulative debt increases, the cost of debt alsoincreases.

[698] Gleim #: 6.71 -- Source: CMA 692 1-26

RLF Corporation had income before taxes of $60,000 for the year. Included in this amount were depreciation of $5,000, acharge of $6,000 for the amortization of bond discounts, and $4,000 for interest expense. The estimated cash flow for theperiod is

$60,000.A.$66,000.B.$49,000.C.$71,000.D.

Answer (A) is incorrect because the cash flow for the period is greater than net income given noncash expenses in theform of depreciation and bond discount amortization.Answer (B) is incorrect because $66,000 does not reflect the noncash expense for depreciation.Answer (C) is incorrect because the $5,000 of depreciation and the $6,000 for amortization should be added back to,not subtracted from, income.Answer (D) is correct. To determine cash flow for the period, all noncash expenses should be added back to netincome. Adding the $5,000 of depreciation and the $6,000 of discount amortization to the $60,000 of net incomeproduces a cash flow of $71,000.

Gleim's CIA Test Prep: Part III: Business Analysis and Information Technology(1165 questions)

Copyright 2004 Gleim Publications, Inc. Page 279Printed for g j

Page 280: Part three cia_with_ answers

[699] Gleim #: 6.72 -- Source: CMA 1295 1-8Shown below is a forecast of sales for Cooper Inc. for the first 4 months of the year (all amounts are in thousands ofdollars).

  January February March April Cash sales $  15 $  24 $18 $14Sales on credit 100 120 90 70

On average, 50% of credit sales are paid for in the month of sale, 30% in the month following the sale, and the remainder ispaid 2 months after the month of sale. Assuming there are no bad debts, the expected cash inflow for Cooper in March is

$138,000A.$122,000B.$119,000C.$108,000D.

Answer (A) is incorrect because $138,000 equals the sum of February credit sales and March cash sales.Answer (B) is incorrect because $122,000 equals 50% of January credit sales, 30% of February credit sales, 20% ofMarch credit sales, and 100% of March cash sales.Answer (C) is correct. Cash inflows for March would consist of 50% of March credit sales (50% × $90 = $45), plus30% of February credit sales (30% × $120 = $36), plus 20% of January credit sales (20% × $100 = $20), plus cashsales for March of $18. Consequently, total collections equal $119,000.Answer (D) is incorrect because $108,000 is the total sales for March, not the total cash collections for March.

[700] Gleim #: 6.73 -- Source: CMA 696 1-12

A firm has daily cash receipts of $100,000 and collection time of 2 days. A bank has offered to reduce the collection timeon the firm's deposits by 2 days for a monthly fee of $500. If money market rates are expected to average 6% during theyear, the net annual benefit (loss) from having this service is

$3,000A.$12,000B.$0C.$6,000D.

Answer (A) is incorrect because $3,000 miscalculates the annual service charge.Answer (B) is incorrect because $12,000 fails to deduct the annual service charge from the interest earned.Answer (C) is incorrect because $0 results from figuring the interest earned for only one day, not two.Answer (D) is correct. If collection time is 2 days, and average daily receipts are $100,000, the average cash balancewill increase by $200,000 if the bank's system is adopted. At a 6% interest rate, $200,000 will generate $12,000 ofinterest revenue annually. The $500 monthly charge by the bank will result in an annual expense of $6,000. Thus, thenet annual benefit is $6,000 ($12,000 – $6,000).

Gleim's CIA Test Prep: Part III: Business Analysis and Information Technology(1165 questions)

Copyright 2004 Gleim Publications, Inc. Page 280Printed for g j

Page 281: Part three cia_with_ answers

[701] Gleim #: 6.74 -- Source: CMA 697 1-15The treasury analyst for Garth Manufacturing has estimated the cash flows for the first half of next year (ignoring any short-term borrowings) as follows.

  Cash (millions)   Inflows Outflows January $2 $1February 2 4March 2 5April 2 3May 4 2June 5 3

Garth has a line of credit of up to $4 million on which it pays interest monthly at a rate of 1% of the amount utilized. Garthis expected to have a cash balance of $2 million on January 1 and no amount utilized on its line of credit. Assuming allcash flows occur at the end of the month, approximately how much will Garth pay in interest during the first half of theyear?

Zero.A.$61,000B.$80,000C.$132,000D.

Answer (A) is incorrect because interest must be paid monthly when the credit line is used in April, May, and June.Answer (B) is correct. The sum of the beginning balance and inflows exceeds the outflows for the first 2 months. Atthe end of March, however, Garth must use $2,000,000 of its line of credit ($2,000,000 beginning balance +$6,000,000 inflows – $10,000,000 outflows). Thus, interest for April is $20,000 (1% × $2,000,000). The net cashoutflow for April (ignoring short-term borrowings) is $1,000,000 of an additional $1,000,000 of the line of credit.However, the $20,000 of interest for April must also be paid, so the amount of the line of credit used in May is$3,020,000 ($2,000,000 + $1,000,000 + $20,000). Interest for May is therefore $30,200 (1% × $3,020,000). Giventhe net cash inflow for May of $2,000,000 (again ignoring short-term borrowings) and the borrowing of $30,200 topay the interest for May, the amount of the line of credit used in June is $1,050,200. Interest in June is $10,502 (1%× $1,050,200), and total interest is $60,702 ($20,000 + $30,200 + $10,502). Consequently, the closest answer is$61,000.Answer (C) is incorrect because the company would repay the credit line at the end of months with a positive cashflow.Answer (D) is incorrect because the company would repay the credit line at the end of months with a positive cashflow.

[702] Gleim #: 6.75 -- Source: CMA 697 1-20

Kemple is a newly established janitorial firm, and the owner is deciding what type of checking account to open. Kemple isplanning to keep a $500 minimum balance in the account for emergencies and plans to write roughly 80 checks per month.The bank charges $10 per month plus a $0.10 per check charge for a standard business checking account with no minimumbalance. Kemple also has the option of a premium business checking account that requires a $2,500 minimum balance buthas no monthly fees or per check charges. If Kemple's cost of funds is 10%, which account should Kemple choose?

Standard account, because the savings is $34 per year.A.Premium account, because the savings is $34 per year.B.Standard account, because the savings is $16 per year.C.Premium account, because the savings is $16 per year.D.

Answer (A) is incorrect because the relevant cost of the minimum premium account deposit is based on the $2,000incremental deposit, not the full $2,500.Answer (B) is incorrect because the savings on the premium account is $16.Answer (C) is incorrect because the savings on the premium account is $16.Answer (D) is correct. The standard account will cost $10 per month plus $8 in check charges ($.10 x 80 checks), fora total of $18 per month or $216 per year. The premium account has no check charges, but it will require thedepositor to maintain a balance of $2,000 more than desired. At a 10% cost of capital, the incremental $2,000minimum deposit will cost $200 per year. Thus, the premium account should be selected because it is cheaper by $16per year.

Gleim's CIA Test Prep: Part III: Business Analysis and Information Technology(1165 questions)

Copyright 2004 Gleim Publications, Inc. Page 281Printed for g j

Page 282: Part three cia_with_ answers

[703] Gleim #: 6.76 -- Source: CMA 697 1-13Newman Products has received proposals from several banks to establish a lockbox system to speed up receipts. Newmanreceives an average of 700 checks per day averaging $1,800 each, and its cost of short-term funds is 7% per year. Assumingthat all proposals will produce equivalent processing results and using a 360-day year, which one of the followingproposals is optimal for Newman?

A $0.50 fee per check.A.A flat fee of $125,000 per year.B.A fee of 0.03% of the amount collected.C.A compensating balance of $1,750,000.D.

Answer (A) is incorrect because the annual cost is $126,000.Answer (B) is incorrect because the annual cost is $125,000.Answer (C) is incorrect because the annual cost is $136,080.Answer (D) is correct. Multiplying 700 checks times 360 days results in a total of 252,000 checks per year.Accordingly, under (A), total annual cost is $126,000 ($.50 × 252,000), which is less desirable than the $125,000 flatfee in (B). Given that the annual collections equal $453,600,000 ($1,800 × 700 × 360), (C) is also less desirablebecause the annual fee would be $136,080 (.03% × $453,600,000). The best option is therefore to maintain acompensating balance of $1,750,000 when the cost of funds is 7%, resulting in a total cost of $122,500 (.07 ×$1,750,000).

[704] Gleim #: 6.77 -- Source: Publisher

A firm has daily cash receipts of $300,000 and is interested in acquiring a lockbox service in order to reduce collectiontime. Bank 1's lockbox service costs $3,000 per month and will reduce collection time by 3 days. Bank 2's lockbox servicecosts $5,000 per month and will reduce collection time by 4 days. Bank 3's lockbox service costs $500 per month and willreduce collection time by 1 day. Bank 4's lockbox service costs $1,000 per month and will reduce collection time by 2days. If money market rates are expected to average 6% during the year, and the firm wishes to maximize income, whichbank should the firm choose?

Bank 1.A.Bank 2.B.Bank 3.C.Bank 4.D.

Answer (A) is incorrect because Bank 1 will increase the firm's income by only $18,000.Answer (B) is incorrect because Bank 2 will increase the firm's income by only $12,000.Answer (C) is incorrect because Bank 3 will increase the firm's income by only $12,000.Answer (D) is correct. Because collections made using Bank 4's lockbox service will be accelerated by 2 days at arate of $300,000 per day, the firm will have an additional $600,000 to invest. At a rate of 6%, the interest earned willbe $36,000 per year. However, the bank will charge $12,000 (12 months × $1,000 per month) for its services. Thus,the firm will gain $24,000 ($36,000 – $12,000).

[705] Gleim #: 6.78 -- Source: Publisher

If the average age of inventory is 60 days, the average age of the accounts payable is 30 days, and the average age ofaccounts receivable is 45 days, the number of days in the cash flow cycle is

135 days.A.90 days.B.75 days.C.105 days.D.

Answer (A) is incorrect because the age of payables should be deducted from the sum of the other items.Answer (B) is incorrect because the payables are not added to the inventory period. They are deducted.Answer (C) is correct. The cash flow cycle begins when the firm pays for merchandise it has purchased and endswhen the firm receives cash from the sale of the merchandise. Inventory is held for an average of 60 days prior tosale, but the average age of accounts payable is 30 days. Consequently, the average time between outlay and sale is30 days. Receivables are collected an average of 45 days after sale, so the length of the cash flow cycle is 75 days (30+ 45).Answer (D) is incorrect because 105 days equals the sum of the inventory cycle and the receivables cycle.

Gleim's CIA Test Prep: Part III: Business Analysis and Information Technology(1165 questions)

Copyright 2004 Gleim Publications, Inc. Page 282Printed for g j

Page 283: Part three cia_with_ answers

[Fact Pattern #51]CyberAge Outlet, a relatively new store, is a cafe that offers customers the opportunity to browse the Internet or play computergames at their tables while they drink coffee. The customer pays a fee based on the amount of time spent signed on to thecomputer. The store also sells books, tee-shirts, and computer accessories. CyberAge has been paying all of its bills on the lastday of the payment period, thus forfeiting all supplier discounts.

Shown below are data on CyberAge's two major vendors, including average monthly purchases and credit terms.

  Average  Monthly

Vendor Purchases Credit TermsWeb Master $25,000 2/10, net 30Softidee 50,000 5/10, net 90

[706] Gleim #: 6.79 -- Source: CMA 1296 1-10

(Refers to Fact Pattern #51)Assuming a 360-day year and that CyberAge continues paying on the last day of the credit period, the company's weighted-average annual interest rate for trade credit (ignoring the effects of compounding) for these two vendors is

27.0%A.25.2%B.28.0%C.30.2%D.

Answer (A) is incorrect because 27.0% is based on weights of $25,000 and $50,000.Answer (B) is correct. If the company pays Web Master within 10 days, it will save $500 (2% × $25,000). Thus, thecompany is effectively paying $500 to retain $24,500 ($25,000 – $500) for 20 days (30 – 10). The annualized interestrate on this borrowing is 36.7346% [($500 ÷ $24,500) × (360 days ÷ 20 days)]. Similarly, the company is, in effect,paying Softidee $2,500 (5% × $50,000) to hold $47,500 ($50,000 – $2,500) for 80 days (90 – 10). The annualizedrate on this borrowing is 23.6842% [($2,500 ÷ $47,500) × (360 days/80 days)]. The average amount borrowed fromWeb Master is $16,333.33 [1 month × $24,500 × (20 days ÷ 30 days)], and the average amount borrowed fromSoftidee is $126,666.67 [3 months × $47,500 × (80 days ÷ 90 days)]. Thus, the weighted average of these two ratesbased on average borrowings is 25.2% {[36.7346% × $16,333.33) + (23.6842% × $126,666.67)] ÷ ($16,333.33 +$126,666.67)}. This calculation, however, understates the true cost of not taking the discount because it does notconsider the effects of compounding.Answer (C) is incorrect because 28.0% is based on weights of $24,500 and $47,500.Answer (D) is incorrect because 30.2% is an unweighted average of the two interest rates.

[707] Gleim #: 6.80 -- Source: CMA 1296 1-11

(Refers to Fact Pattern #51)Should CyberAge use trade credit and continue paying at the end of the credit period?

Yes, if the cost of alternative short-term financing is less.A.Yes, if the firm's weighted-average cost of capital is equal to its weighted-average cost of trade credit.B.No, if the cost of alternative long-term financing is greater.C.Yes, if the cost of alternative short-term financing is greater.D.

Gleim's CIA Test Prep: Part III: Business Analysis and Information Technology(1165 questions)

Copyright 2004 Gleim Publications, Inc. Page 283Printed for g j

Page 284: Part three cia_with_ answers

Answer (A) is incorrect because the company should continue the current practice unless alternative short-term financing isavailable at a lower rate.Answer (B) is incorrect because the weighted-average cost of capital is usually a concern in capital budgeting and is not asimportant in the decision process as the marginal cost of capital. Furthermore, trade credit is just one element in the firm'sfinancing structure. An optimal mix of financing sources may require that trade credit be obtained at less than the weighted-average cost of capital.Answer (C) is incorrect because the company should maintain its current practice if the cost of alternative long-termfinancing is higher.Answer (D) is correct. The company is currently paying an annual rate of 25.2% (as determined below) to obtain tradecredit and pay at the end of the credit period. This policy should be continued if trade credit is the only source of financing,or if other sources are available only at a higher rate. The annual rate is determined as follows: If the company pays WebMaster within 10 days, it will save $500 (2% × $25,000). Thus, the company is effectively paying $500 to retain $24,500($25,000 – $500) for 20 days (30 – 10). The annualized interest rate on this borrowing is 36.7346% [($500 ÷ $24,500) ×(360 days ÷ 20 days)]. Similarly, the company is, in effect, paying Softidee $2,500 (5% × $50,000) to hold $47,500($50,000 – $2,500) for 80 days (90 – 10). The annualized rate on this borrowing is 23.6842% [($2,500 ÷ $47,500) × (360days ÷ 80 days)]. The average amount borrowed from Web Master is $16,333.33 [1 month × $24,500 × (20 days ÷ 30days)], and the average amount borrowed from Softidee is $126,666.67 [3 months × $47,500 × (80 days ÷ 90 days)]. Thus,the weighted average of these two rates based on average borrowings is 25.2% {[36.7346% × $16,333.33) + (23.6842% ×$126,666.67)] ÷ ($16,333.33 + $126,666.67)}.

[708] Gleim #: 6.81 -- Source: CMA 1294 1-17

Troy Toys is a retailer operating in several cities. The individual store managers deposit daily collections at a local bank ina non-interest bearing checking account. Twice per week, the local bank issues a depository transfer check (DTC) to thecentral bank at headquarters. The controller of the company is considering using a wire transfer instead. The additional costof each transfer would be $25; collections would be accelerated by 2 days; and the annual interest rate paid by the centralbank is 7.2% (0.02% per day). At what amount of dollars transferred would it be economically feasible to use a wiretransfer instead of the DTC? Assume a 350-day year.

It would never be economically feasible.A.$125,000 or above.B.Any amount greater than $173.C.Any amount greater than $62,500.D.

Answer (A) is incorrect because the $25 transfer fee is covered by the interest on $62,500 for 2 days.Answer (B) is incorrect because $125,000 is required if collections are accelerated by only one day.Answer (C) is incorrect because the interest on $173 for 2 days is less than $.07.Answer (D) is correct. Given a $25 fee and an interest rate of 0.02% per day for 2 days, the breakeven amount is$62,500 [$25 transfer fee ÷ (2 × .02% interest rate)]. Thus, the interest earned on a transfer of any amount greaterthan $62,500 would exceed the $25 fee.

[709] Gleim #: 6.82 -- Source: CMA 1296 1-5

A working capital technique that increases the payable float and therefore delays the outflow of cash is

Concentration banking.A.A draft.B.Electronic Data Interchange (EDI).C.A lockbox system.D.

Answer (A) is incorrect because concentration banking, a lockbox system, and the use of a local post office box aretechniques used to accelerate cash receipts.Answer (B) is correct. Payment by draft, a three-party instrument in which the drawer orders the drawee to paymoney to the payee, is a means of slowing cash outflows. A check is the most common type of draft. Check floatarises from the delay between an expenditure and the clearing of the check through the banking system.Answer (C) is incorrect because EDI is the communication of electronic documents directly from a computer in oneentity to a computer in another entity. Thus, EDI expedites cash payments. The payee receives the money almostinstantaneously.Answer (D) is incorrect because concentration banking, a lockbox system, and the use of a local post office box aretechniques used to accelerate cash receipts.

[710] Gleim #: 6.83 -- Source: CMA 688 1-13

A compensating balance

Gleim's CIA Test Prep: Part III: Business Analysis and Information Technology(1165 questions)

Copyright 2004 Gleim Publications, Inc. Page 284Printed for g j

Page 285: Part three cia_with_ answers

Compensates a financial institution for services rendered by providing it with deposits of funds.A.Is used to compensate for possible losses on a marketable securities portfolio.B.Is a level of inventory held to compensate for variations in usage rate and lead time.C.Is the amount of prepaid interest on a loan.D.

Answer (A) is correct. Banks sometimes require a borrower to keep a certain percentage of the face amount of a loan in anoninterest-bearing checking account. This requirement raises the effective rate of interest paid by the borrower. Thisgreater rate compensates a bank for services provided and results in greater profitability for the financial institution. Fundskept as a compensating balance can often be withdrawn if a certain average balance is maintained.Answer (B) is incorrect because, in financial accounting, a valuation allowance is used to reflect losses on marketablesecurities.Answer (C) is incorrect because safety stock is held for such purposes.Answer (D) is incorrect because interest deducted in advance is discount interest.

[711] Gleim #: 6.84 -- Source: CMA 697 1-19

Hagar Company's bank requires a compensating balance of 20% on a $100,000 loan. If the stated interest on the loan is7%, what is the effective cost of the loan?

5.83%A.7.00%B.8.40%C.8.75%D.

Answer (A) is incorrect because the borrower has access to less, not more, than the face amount of the loan.Answer (B) is incorrect because the effective rate is higher than the contract rate as a result of the compensatingbalance requirement.Answer (C) is incorrect because 8.40% is 120% of the contract rate.Answer (D) is correct. Interest on the loan is $7,000 (7% × $100,000). Given that the borrower has to maintain a20% compensating balance, only $80,000 [$100,000 – (20% × $100,000)] is available for use. Thus, the company ispaying $7,000 for the use of $80,000 in funds at an effective cost of 8.75% ($7,000 ÷ $80,000).

[712] Gleim #: 6.85 -- Source: CMA 696 1-10

A company uses the following formula in determining its optimal level of cash.

If:   b = fixed cost per transactioni = interest rate on marketable securities

T = total demand for cash over a period of time

This formula is a modification of the economic order quantity (EOQ) formula used for inventory management. Assume thatthe fixed cost of selling marketable securities is $10 per transaction and the interest rate on marketable securities is 6% peryear. The company estimates that it will make cash payments of $12,000 over a one-month period. What is the average cashbalance (rounded to the nearest dollar)?

$1,000A.$2,000B.$3,464C.$6,928D.

Gleim's CIA Test Prep: Part III: Business Analysis and Information Technology(1165 questions)

Copyright 2004 Gleim Publications, Inc. Page 285Printed for g j

Page 286: Part three cia_with_ answers

Answer (A) is incorrect because $1,000 results from using 24% in the denominator.Answer (B) is incorrect because $2,000 results from using 6% in the denominator.Answer (C) is correct. The EOQ for inventory is a function of ordering cost per order, inventory demand, and carrying cost.In the cash model, the fixed cost per sale of securities is equivalent to the ordering cost, the demand for cash is similar to thedemand for inventory, and the interest rate is effectively the cost of carrying a dollar of cash for the period. Substituting inthe formula yields an optimal cash balance of about $6,928. Thus, the average cash balance is $3,464 ($6,928 ÷ 2).

Answer (D) is incorrect because $6,928 is the optimal cash balance.

[713] Gleim #: 6.86 -- Source: Publisher

Which of the following models is essentially an application to cash management of the inventory economic order quantity(EOQ) formula?

Miller-Orr cash management model.A.Baumol cash management model.B.Altman's Z-Score model.C.Arbitrage pricing model.D.

Answer (A) is incorrect because the Miller-Orr cash management model is not deterministic, but simply provides arange in which marketable securities will not be transacted.Answer (B) is correct. The Baumol cash management model is essentially the inventory EOQ model in that itminimizes the total of fixed trading costs and cash carrying costs. It is a deterministic model in the same way that theEOQ model is deterministic.Answer (C) is incorrect because Altman's Z-Score is a means of forecasting a firm's future solvency by adding theweighted amounts of five financial ratios.Answer (D) is incorrect because an arbitrage pricing model is a means of valuing an asset by means of using severalrisk factors.

[714] Gleim #: 6.87 -- Source: CMA 694 1-25

All of the following are alternative marketable securities suitable for investment except

U.S. Treasury bills.A.Eurodollars.B.Commercial paper.C.Convertible bonds.D.

Answer (A) is incorrect because U.S. Treasury bills are short-term marketable securities.Answer (B) is incorrect because Eurodollars are short-term marketable securities.Answer (C) is incorrect because commercial paper is a short-term marketable security.Answer (D) is correct. Marketable securities are near-cash items used primarily for short-term investment. Examplesinclude U.S. Treasury bills, Eurodollars, commercial paper, money-market mutual funds with portfolios of short-termsecurities, bankers' acceptances, floating rate preferred stock, and negotiable CDs of U.S. banks. A convertible bondis not a short-term investment because its maturity date is usually more than one year in the future and its price can beinfluenced substantially by changes in interest rates or by changes in the investee's stock price.

[715] Gleim #: 6.88 -- Source: CMA 1294 1-16

The term “short-selling” is the

Selling of a security that was purchased by borrowing money from a broker.A.Selling of a security that is not owned by the seller.B.Selling of all the shares you own in a company in anticipation that the price will decline dramatically.C.Betting that a stock will increase by a certain amount within a given period of time.D.

Gleim's CIA Test Prep: Part III: Business Analysis and Information Technology(1165 questions)

Copyright 2004 Gleim Publications, Inc. Page 286Printed for g j

Page 287: Part three cia_with_ answers

Answer (A) is incorrect because margin trading involves buying securities by borrowing from a broker.Answer (B) is correct. Short-selling is accomplished by borrowing securities from a broker and selling those securities. At alater time, the loan is repaid by buying securities on the open market and returning them to the broker. The seller speculatesthat the stock's market price will decline.Answer (C) is incorrect because the investor does not own the shares sold in a short-sale.Answer (D) is incorrect because the short-seller is betting that the stock will decrease in price.

[716] Gleim #: 6.89 -- Source: CMA 1295 1-6

Jackson Distributors sells to retail stores on credit terms of 2/10, net 30. Daily sales average 150 units at a price of $300each. Assuming that all sales are on credit and 60% of customers take the discount and pay on day 10 while the rest of thecustomers pay on day 30, the amount of Jackson's accounts receivable is

$1,350,000A.$990,000B.$900,000C.$810,000D.

Answer (A) is incorrect because 60% of the sales will be paid for within the 10-day discount period.Answer (B) is incorrect because $990,000 is based on a sales total of $1,500,000 for 30 days rather than $1,350,000.Answer (C) is incorrect because $900,000 is based on a sales total of $1,500,000 for 30 days rather than $1,350,000.Answer (D) is correct. The firm has daily sales of $45,000 consisting of 150 units at $300 each. For 30 days, salestotal $1,350,000. Forty percent of these sales, or $540,000, will be uncollected because customers do not take theirdiscounts. The remaining 60%, or $810,000, will be paid within the discount period. However, by the end of 30 days,only 2/3 of the $810,000 will be collected because the sales from days 21 through 30 are still within the discountperiod. Therefore, an additional $270,000 ($810,000 – $540,000) will still be uncollected after the 30th day, but willbe subject to a discount. In total, the average receivable balance is $810,000, consisting of $540,000 on which nodiscount will be taken and $270,000 that will be paid within the discount period.

[717] Gleim #: 6.90 -- Source: CMA 1289 1-15

A change in credit policy has caused an increase in sales, an increase in discounts taken, a decrease in the amount of baddebts, and a decrease in the investment in accounts receivable. Based upon this information, the company's

Average collection period has decreased.A.Percentage discount offered has decreased.B.Accounts receivable turnover has decreased.C.Working capital has increased.D.

Answer (A) is correct. An increase in discounts taken accompanied by declines in receivables balances and doubtfulaccounts all indicate that collections on the increased sales have been accelerated. Accordingly, the average collectionperiod must have declined. The average collection period is a ratio calculated by dividing the number of days in ayear (365) by the receivable turnover. Thus, the higher the turnover, the shorter the average collection period. Theturnover increases when either sales (the numerator) increase, or receivables (the denominator) decrease.Accomplishing both higher sales and a lower receivables increases the turnover and results in a shorter collectionperiod.Answer (B) is incorrect because a decrease in the percentage discount offered provides no incentive for earlypayment.Answer (C) is incorrect because accounts receivable turnover (sales ÷ average receivables) has increased.Answer (D) is incorrect because no information is given relative to working capital elements other than receivables.Both receivables and cash are elements of working capital, so an acceleration of customer payments will have noeffect on working capital.

[718] Gleim #: 6.91 -- Source: CMA 1294 1-24

A company plans to tighten its credit policy. The new policy will decrease the average number of days in collection from75 to 50 days and will reduce the ratio of credit sales to total revenue from 70% to 60%. The company estimates thatprojected sales will be 5% less if the proposed new credit policy is implemented. If projected sales for the coming year are$50 million, calculate the dollar impact on accounts receivable of this proposed change in credit policy. Assume a 360-dayyear.

Gleim's CIA Test Prep: Part III: Business Analysis and Information Technology(1165 questions)

Copyright 2004 Gleim Publications, Inc. Page 287Printed for g j

Page 288: Part three cia_with_ answers

$3,817,445 decrease.A.$6,500,000 decrease.B.$3,333,334 decrease.C.$18,749,778 increase.D.

Answer (A) is incorrect because the decrease will be $3,333,334.Answer (B) is incorrect because the decrease will be $3,333,334.Answer (C) is correct. If sales are $50 million, 70% of which are on credit, total credit sales will be $35 million. Thereceivables turnover equals 4.8 times per year (360 days ÷ 75-day collection period). Receivables turnover equals net creditsales divided by average receivables. Accordingly, average receivables equal $7,291,667 ($35,000,000 ÷ 4.8). Under thenew policy, sales will be $47.5 million (95% × $50,000,000), and credit sales will be $28.5 million (60% × $47,500,000).The collection period will be reduced to 50 days, resulting in a turnover of 7.2 times per year (360 ÷ 50). The averagereceivables balance will therefore be $3,958,333 ($28,500,000 ÷ 7.2), a reduction of $3,333,334 ($7,291,667 –$3,958,333).Answer (D) is incorrect because receivables will decrease.

[719] Gleim #: 6.92 -- Source: CMA 696 1-13

A company with $4.8 million in credit sales per year plans to relax its credit standards, projecting that this will increasecredit sales by $720,000. The company's average collection period for new customers is expected to be 75 days, and thepayment behavior of the existing customers is not expected to change. Variable costs are 80% of sales. The firm'sopportunity cost is 20% before taxes. Assuming a 360-day year, what is the company's benefit (loss) on the planned changein credit terms?

$0A.$28,800B.$144,000C.$120,000D.

Answer (A) is incorrect because the company benefits from the change in credit terms.Answer (B) is incorrect because $28,800 results from multiplying the contribution margin by the 20% interest rate.Answer (C) is incorrect because $144,000 overlooks the costs created by having funds invested in receivables for 75days.Answer (D) is correct. The incremental sales will produce an increased contribution margin of $144,000 (20% ×$720,000). However, that amount must be offset by the cost of funds invested in receivables. The variable costsassociated with the incremental sales are $576,000 (80% × $720,000). Given a 75-day credit period, the averageinvestment in receivables equals $120,000 [$576,000 × (75 ÷ 360)]. Accordingly, the cost of the investment inadditional receivables is $24,000 (20% opportunity cost × $120,000), and the net benefit of the planned change incredit terms is $120,000 ($144,000 – $24,000).

[720] Gleim #: 6.93 -- Source: CMA 1296 1-18

Which of the following represents a firm's average gross receivables balance?

Days' sales in receivables × accounts receivable turnover.I.Average daily sales × average collection period.II.Net sales/average gross receivables.III.

I only.A.I and II only.B.II only.C.II and III only.D.

Answer (A) is incorrect because Alternative I cannot be correct. Neither of the multiplicands is a dollar figure, so theproduct could not be the dollar balance of receivables.Answer (B) is incorrect because Alternative I cannot be correct. Neither of the multiplicands is a dollar figure, so theproduct could not be the dollar balance of receivables.Answer (C) is correct. A firm's average gross receivables balance can be calculated by multiplying average dailysales by the average collection period (days' sales outstanding). Alternatively, annual credit sales can be divided bythe accounts-receivable turnover (net credit sales/average accounts receivable) to obtain the average balance inreceivables.Answer (D) is incorrect because Alternative III cannot be correct. It contains average gross receivables, the amountbeing calculated.

Gleim's CIA Test Prep: Part III: Business Analysis and Information Technology(1165 questions)

Copyright 2004 Gleim Publications, Inc. Page 288Printed for g j

Page 289: Part three cia_with_ answers

[721] Gleim #: 6.94 -- Source: PublisherThe high cost of short-term financing has recently caused a company to reevaluate the terms of credit it extends to itscustomers. The current policy is 1/10, net 60. If customers can borrow at the prime rate, at what prime rate must thecompany change its terms of credit in order to avoid an undesirable extension in its collection of receivables?

2%A.5%B.7%C.8%D.

Answer (A) is incorrect because the prime rate must be greater than 7.37% to make the company's terms preferable tothose of a bank.Answer (B) is incorrect because the prime rate must be greater than 7.37% to make the company's terms preferable tothose of a bank.Answer (C) is incorrect because the prime rate must be greater than 7.37% to make the company's terms preferable tothose of a bank.Answer (D) is correct. Terms of 1/10, net 60 mean that a buyer can save 1% of the purchase price by paying 50 daysearly. In essence, not taking the discount results in the buyer's borrowing 99% of the invoice price for 50 days at atotal interest charge of 1% of the invoice price. Because a year has 7.3 50-day periods (365 ÷ 50), the credit terms1/10, net 60 yield an effective annualized interest charge of approximately 7.37% [(1% ÷ 99%) × 7.3]. If the primerate were higher than 7.37%, the buyer would prefer to borrow from the vendor (i.e., not pay within the discountperiod) rather than from a bank. Consequently, an 8% prime rate could cause the vendor's receivables to increase.

[722] Gleim #: 6.95 -- Source: Publisher

The appropriate discount rate to use in valuing a business combination is the

Combined firm's cost of debt.A.Acquiring firm's weighted average cost of capital.B.Acquiring firm's cost of equity.C.Combined firm's cost of equity.D.

Answer (A) is incorrect because the cost of equity, not the cost of debt, should be the discount rate.Answer (B) is incorrect because the discount rate should be that of the combined firm, not the acquiring firm.Answer (C) is incorrect because the discount rate should be that of the combined firm, not the acquiring firm.Answer (D) is correct. If the net incremental cash flows to the acquiring firm's shareholders are to be calculated, thediscount rate used should be the cost of equity capital. Moreover, this rate should reflect the risk associated with theuse of funds rather than their source. The rate therefore should not be the cost of capital of the acquiring firm butrather the cost of equity of the acquired firm after the combination. This calculation requires a new estimate of beta tobe used in the Capital Asset Pricing Model.

[723] Gleim #: 6.96 -- Source: Publisher

Business combinations are accomplished either through a direct acquisition of assets and liabilities by a survivingcorporation or by stock investments in one or more companies. A parent-subsidiary relationship always arises from a

Tax-free reorganization.A.Vertical combination.B.Horizontal combination.C.Greater than 50% stock investment in another company.D.

Answer (A) is incorrect because a tax-free reorganization may or may not be a combination, and it may or may notresult in a parent-subsidiary relationship.Answer (B) is incorrect because vertical and horizontal combinations may also be accomplished by a merger or aconsolidation, in which case the combining entities become one. A vertical combination combines a supplier or acustomer firm with the acquirer. A horizontal combination combines two firms in the same line of business.Answer (C) is incorrect because vertical and horizontal combinations may also be accomplished by a merger or aconsolidation, in which case the combining entities become one. A vertical combination combines a supplier or acustomer firm with the acquirer. A horizontal combination combines two firms in the same line of business.Answer (D) is correct. A parent-subsidiary relationship arises from an effective investment in the stock of anotherenterprise in excess of 50%. The financial statements for the two companies ordinarily should be presented on aconsolidated basis. To the extent the corporation is not wholly-owned, a minority interest is presented.

Gleim's CIA Test Prep: Part III: Business Analysis and Information Technology(1165 questions)

Copyright 2004 Gleim Publications, Inc. Page 289Printed for g j

Page 290: Part three cia_with_ answers

[724] Gleim #: 6.97 -- Source: CMA 1291 1-13

When a firm finances each asset with a financial instrument of the same approximate maturity as the life of the asset, it isapplying

Working capital management.A.Return maximization.B.Financial leverage.C.A hedging approach.D.

Answer (A) is incorrect because working capital management is short-term asset management.Answer (B) is incorrect because return maximization is more aggressive than maturity matching. It entails using thelowest cost forms of financing.Answer (C) is incorrect because financial leverage is the relationship between debt and equity financing.Answer (D) is correct. Maturity matching, or equalizing the life of an asset and the debt instrument used to financethat asset, is a hedging approach. The basic concept is that the company has the entire life of the asset to recover theamount invested before having to pay the lender.

[725] Gleim #: 6.98 -- Source: Publisher

A forward contract involves

A commitment today to purchase a product on a specific future date at a price to be determined some time in thefuture.

A.

A commitment today to purchase a product some time during the current day at its present price.B.A commitment today to purchase a product on a specific future date at a price determined today.C.A commitment today to purchase a product only when its price increases above its current exercise price.D.

Answer (A) is incorrect because the price of a future contract is determined on the day of commitment, not some timein the future.Answer (B) is incorrect because performance is deferred in a future contract, and the price of the product is notnecessarily its present price. The price can be any price determined on the day of commitment.Answer (C) is correct. A forward contract is an executory contract in which the parties involved agree to the terms ofa purchase and a sale, but performance is deferred. Accordingly, a forward contract involves a commitment today topurchase a product on a specific future date at a price determined today.Answer (D) is incorrect because a forward contract is a firm commitment to purchase a product. It is not based on acontingency. Also, a forward contract does not involve an exercise price (exercise price is in an option contract).

[726] Gleim #: 6.99 -- Source: Publisher

A firm is planning to issue a callable bond with an 8% coupon rate and 10 years to maturity. A straight bond with a similarrate is priced at $1,000. If the value of the issuer's call option is estimated to be $50, what is the value of the callable bond?

$1,000A.$950B.$1,050C.$900D.

Answer (A) is incorrect because the call feature reduces the value of the bond by the value of the call option.Answer (B) is correct. A callable bond is not as valuable to an investor as a straight bond. Thus, the $50 call optionis subtracted from the $1,000 value of a straight bond to arrive at a $950 value for the callable bond.Answer (C) is incorrect because the call feature reduces the value of the bond by the value of the call option.Answer (D) is incorrect because the call feature reduces the value of the bond by the value of the call option.

[727] Gleim #: 6.100 -- Source: Publisher

How much must the stock be worth at expiration in order for a call holder to break even if the exercise price is $60 and thecall premium was $3?

Gleim's CIA Test Prep: Part III: Business Analysis and Information Technology(1165 questions)

Copyright 2004 Gleim Publications, Inc. Page 290Printed for g j

Page 291: Part three cia_with_ answers

$57.00A.$60.00B.$61.50C.$63.00D.

Answer (A) is incorrect because $57 is the result of deducting the call premium from the exercise price.Answer (B) is incorrect because $60 is the result of failing to consider the impact of the call premium.Answer (C) is incorrect because the full call premium must be added to the exercise price.Answer (D) is correct. Because the call premium is $3, the stock price must be at least $63 ($60 exercise price + $3 callpremium).

[Fact Pattern #52]AA Company has purchased one share of QQ Company common stock and one put option. It has also sold one call option. Theoptions are written on one share of QQ Company common stock and have the same maturity date and exercise price. The exerciseprice ($40) is the same as the share price. Moreover, the options are exercisable only at the expiration date.

[728] Gleim #: 6.101 -- Source: Publisher

(Refers to Fact Pattern #52)Assume that the value of a share of QQ Company common stock at the expiration date is either $30 or $45. The differencein the net payoff on the portfolio because of a difference in the stock price at the maturity date is

$10.00A.$7.50B.$5.00C.$0D.

Answer (A) is incorrect because the portfolio has the same value at the maturity date regardless of the price of thestock.Answer (B) is incorrect because the portfolio has the same value at the maturity date regardless of the price of thestock.Answer (C) is incorrect because the portfolio has the same value at the maturity date regardless of the price of thestock.Answer (D) is correct. If the stock price at the maturity date is $30, AA Company will have a share of stock worth$30 and a put option worth $10 ($40 exercise price – $30 stock price). The call option will be worthless. Hence, thenet payoff is $40 ($30 + $10). If the stock price at the maturity date is $45, the share of stock will be worth $45, theput will be worthless, and the loss on the call will be $5 ($45 – $40). Thus, the net payoff will be $40 ($45 – $5).Consequently, the difference in the net payoff on the portfolio because of a difference in the stock price at thematurity date is $0 ($40 – $40).

[729] Gleim #: 6.102 -- Source: Publisher

(Refers to Fact Pattern #52)Assuming the present value of the exercise price is $36 and the value of the call is $4.50, the value of the put in accordancewith the put-call parity theorem is

$4.50A.$4.00B.$.50C.$0D.

Answer (A) is incorrect because $4.50 is the value of the call.Answer (B) is incorrect because $4.00 is the difference between the exercise price and its present value.Answer (C) is correct. For European options, given market equilibrium for all relevant prices (no arbitragepossibilities), equal exercise prices for the put and the call, and the same expiration date, the put-call parity theoremstates that a fixed relationship applies to the market values of the put and call options on a security. For example, astrategy of selling one call option, buying one share of the stock, and buying one put option should result in a risk-free return. The gain (loss) from the stock and the put should equal the gain (loss) on the call. If VS is the value of thestock, VP is the value of the put, VC is the value of the call, and PVE is the present value of the exercise price (thetime interval is the time to expiration), the formula for put-call parity may be stated as follows:

Accordingly, the value of the put is $.50 ($36 + $4.50 – $40).Answer (D) is incorrect because the put has a value of $.50.

Gleim's CIA Test Prep: Part III: Business Analysis and Information Technology(1165 questions)

Copyright 2004 Gleim Publications, Inc. Page 291Printed for g j

Page 292: Part three cia_with_ answers

[730] Gleim #: 6.103 -- Source: Publisher

The effective portion of a gain arising from an increase in the fair value of a derivative is included in earnings in the periodof change if the derivative is appropriately designated and qualifies as a hedge of

A foreign currency exposure of a net investment in a foreign operation.A.A foreign currency exposure of a foreign-currency-denominated forecasted transaction.B.A foreign currency exposure of an available-for-sale security.C.The variable cash flows of a forecasted transaction.D.

Answer (A) is incorrect because the effective portion of gains and losses on this hedge is reported as a component ofthe cumulative translation adjustment in other comprehensive income.Answer (B) is incorrect because the effective portion of gains and losses on these hedges is included in othercomprehensive income until periods in which the forecasted transaction affects earnings.Answer (C) is correct. A fair value hedge includes a hedge of an exposure to changes in the fair value of arecognized asset or liability or an unrecognized firm commitment. Such a hedge minimizes the risk associated withfixed cash flows. A foreign currency fair value hedge includes a hedge of a foreign currency exposure of either anunrecognized firm commitment or an available-for-sale security. Gains and losses arising from changes in fair valueof a derivative classified as either a fair value or a foreign fair value hedge are included in the determination ofearnings in the period of change. They are offset by losses or gains on the hedged item attributable to the risk beinghedged. Thus, earnings of the period of change are affected only by the net gain or loss attributable to the ineffectiveportion of the hedge.Answer (D) is incorrect because the effective portion of gains and losses on these hedges is included in othercomprehensive income until periods in which the forecasted transaction affects earnings.

[731] Gleim #: 6.104 -- Source: Publisher

Garcia Corporation has entered into a binding agreement with Hernandez Company to purchase 400,000 pounds ofColombian coffee at $2.53 per pound for delivery in 90 days. This contract is accounted for as a

Financial instrument.A.Firm commitment.B.Forecasted transaction.C.Fair value hedge.D.

Answer (A) is incorrect because a financial instrument does not involve the delivery of a product.Answer (B) is correct. A firm commitment is an agreement with an unrelated party, binding on both parties andusually legally enforceable, that specifies all significant terms and includes a disincentive for nonperformance.Answer (C) is incorrect because a forecasted transaction is a transaction that is expected to occur for which no firmcommitment exists.Answer (D) is incorrect because the purchase commitment is an exposure to risk, not a hedge of an exposure to risk.

[732] Gleim #: 6.105 -- Source: Publisher

On October 1, 2002, Bordeaux, Inc., a calendar- year-end firm, invested in a derivative designed to hedge the risk ofchanges in fair value of certain assets, currently valued at $1.5 million. The derivative is structured to result in an effectivehedge. However, some ineffectiveness may result. On December 31, 2002, the fair value of the hedged assets has decreasedby $350,000; and the fair value of the derivative has increased by $325,000. Bordeaux should recognize a net effect on2002 earnings of

$0A.$25,000B.$325,000C.$350,000D.

Answer (A) is incorrect because the effect on earnings is equal to the ineffective portion of the hedge.Answer (B) is correct. A hedge of an exposure to changes in the fair value of a recognized asset or liability isclassified as a fair value hedge. Gains and losses arising from changes in fair value of a derivative classified as a fairvalue hedge are included in the determination of earnings in the period of change. They are offset by losses or gainson the hedged item attributable to the risk being hedged. Thus, earnings of the period of change are affected only bythe net gain or loss attributable to the ineffective aspect of the hedge. The ineffective portion is equal to $25,000($350,000 – $325,000).Answer (C) is incorrect because the increase in the fair value of the derivative is a gross effect.Answer (D) is incorrect because the decrease in the fair value of the hedged assets is a gross effect.

Gleim's CIA Test Prep: Part III: Business Analysis and Information Technology(1165 questions)

Copyright 2004 Gleim Publications, Inc. Page 292Printed for g j

Page 293: Part three cia_with_ answers

[Fact Pattern #53]On November 15, 2003, Hector Corp., a calendar-year-end U.S. company, signed a legally binding contract to purchaseequipment from Diego Corp., a foreign company. The negotiated price is FC1,000,000. The scheduled delivery date is February15, 2004. Terms require payment by Hector Corp. upon delivery. The terms also impose a 10% penalty on Diego Corp. if theequipment is not delivered by February 15, 2004.

To hedge its commitment to pay FC1,000,000, Hector entered into a forward-exchange contract on November 15, 2002 to receiveFC1,000,000 on February 15, 2004 at an exchange rate of FC1.00 = U.S.$0.36. Additional exchange rate information:

    Forward Rates forDate Spot Rates February 15, 2004

11/15/02 1 FC = $0.35 U.S. 1 FC = $0.36 U.S.12/31/02 1 FC = $0.36 U.S. 1 FC = $0.38 U.S.02/15/03 1 FC = $0.39 U.S. 1 FC = $0.39 U.S.

Quotes obtained from dealers indicate the following incremental changes in the fair values of the forward-exchange contractbased on the changes in forward rates discounted on a net-present-value basis:

Date Gain/(Loss)11/15/02 $012/31/02 $19,60002/15/03 $10,400

Hector formally documented its objective and strategy for entering into this hedge. Hector also decided to assess hedgeeffectiveness based on an assessment of the difference between changes in value of the forward-exchange contract and the U.S.dollar equivalent of the firm commitment. Because both changes are based on changes in forward rates, Hector further determinedthat the hedge is 100% effective.

[733] Gleim #: 6.106 -- Source: Publisher

(Refers to Fact Pattern #53)The contract signed by Hector Corp. to purchase the equipment from Diego Corp. meets the definition of a

  Firm Forecasted  Commitment Transaction A. Yes YesB. No NoC. Yes NoD. No Yes

Answer (A) is incorrect because the contract meets the definition of a firm commitment. Thus, it cannot be aforecasted transaction.Answer (B) is incorrect because the contract meets the definition of a firm commitment. Thus, it cannot be aforecasted transaction.Answer (C) is correct. SFAS 133 defines a firm commitment as an agreement between unrelated parties, binding onboth and usually legally enforceable, that specifies all significant terms and includes a disincentive fornonperformance. SFAS 133 defines a forecasted transaction as a transaction that is expected to occur for which thereis no firm commitment.Answer (D) is incorrect because the contract meets the definition of a firm commitment. Thus, it cannot be aforecasted transaction.

[734] Gleim #: 6.107 -- Source: Publisher

(Refers to Fact Pattern #53)What are the amounts reported for the forward contract receivable and the firm commitment liability at December 31, 2002and February 15, 2003 (prior to the settlement of the contract)?

Gleim's CIA Test Prep: Part III: Business Analysis and Information Technology(1165 questions)

Copyright 2004 Gleim Publications, Inc. Page 293Printed for g j

Page 294: Part three cia_with_ answers

  12/31/02 02/15/03 A. $10,000 $40,000B. $19,600 $30,000C. $19,600 $10,400D. $20,000 $30,000

Answer (A) is incorrect because $10,000 and $40,000 reflect the undiscounted changes in the spot rate from 11/15/02 to12/31/02 and from 11/15/02 to 02/15/03, respectively.Answer (B) is correct. This hedge is a foreign currency fair value hedge because it hedges a foreign currency exposure ofan unrecognized firm commitment. SFAS 133 requires recognition of the forward contract receivable as an asset at fairvalue, with the changes in fair value recognized in earnings. SFAS 133 further requires recognition of the changes in thefair value of the firm commitment that are attributable to the changes in exchange rates. The changes in fair value arerecognized in earnings and as entries to a liability. Fair values should reflect changes in the forward exchange rates on a net-present-value basis. Thus, the forward contract receivable should be debited and a gain credited for $19,600 at 12/31/02. Aloss should be debited and a firm commitment liability should be credited in the same amount at the same date. At 2/15/03,a further $10,400 forward contract gain and firm commitment loss should be recorded. Because the changes in value of boththe forward-exchange contract and the U.S. dollar equivalent of the firm commitment are based on changes in forward rates,the hedge is completely effective; the changes in fair values ($19,600 and $10,400) of the forward contract receivable(gains) and the firm commitment (losses) offset each other in the income statement.Answer (C) is incorrect because $19,600 and $10,400 reflect the income statement effects.Answer (D) is incorrect because $20,000 and $30,000 reflect the undiscounted changes in the forward rates.

[735] Gleim #: 6.108 -- Source: Publisher

(Refers to Fact Pattern #53)As a result of this hedging transaction, at what amount should Hector recognize the equipment on February 15, 2003?

$350,000A.$360,000B.$390,000C.$420,000D.

Answer (A) is incorrect because $350,000 is the amount that should be recognized if the equipment had beendelivered on 11/15/02.Answer (B) is correct. The equipment should be recorded at $360,000. This amount equals $390,000 (FC1,000,000x $0.39 spot rate at 2/15/03) minus the $30,000 balance in the firm commitment liability account. The entry is todebit equipment for $360,000, debit the firm commitment liability for $30,000, and credit a payable for $390,000. Onthe same date, Hector will debit the payable for $390,000, credit the forward contract receivable for $30,000, andcredit cash for $360,000. The latter entry reflects settlement of the payable and of the forward contract.Answer (C) is incorrect because $390,000 is the amount that should be recognized if the firm commitment had notbeen hedged.Answer (D) is incorrect because $420,000 equals $390,000 plus the $30,000 balance in the firm commitment liabilityaccount.

[736] Gleim #: 6.109 -- Source: Publisher

According to SFAS 133, Accounting for Derivative Instruments and Hedging Activities, as amended by SFAS 138,Accounting for Certain Derivative Instruments and Certain Hedging Activities, the effective portion of a loss associatedwith a change in fair value of a derivative instrument shall be reported as a component of other comprehensive income onlyif the derivative is appropriately designated as a

Cash flow hedge of the foreign currency exposure of a forecasted transaction.A.Fair value hedge of the foreign currency exposure of an unrecognized firm commitment.B.Fair value hedge of the foreign currency exposure of a recognized asset or liability for which a foreign currencytransaction gain or loss is recognized in earnings.

C.

Speculation in a foreign currency.D.

Gleim's CIA Test Prep: Part III: Business Analysis and Information Technology(1165 questions)

Copyright 2004 Gleim Publications, Inc. Page 294Printed for g j

Page 295: Part three cia_with_ answers

Answer (A) is correct. The hedge of the foreign currency exposure of a forecasted transaction is designated as a cash flowhedge. The effective portion of gains and losses associated with changes in fair value of a derivative instrument designatedand qualifying as a cash flow hedging instrument is reported as a component of other comprehensive income.Answer (B) is incorrect because a hedge of the foreign currency exposure of either an unrecognized firm commitment or ofa recognized asset or liability for which a foreign currency transaction gain or loss is recognized in earnings may be a cashflow hedge (if cash flows are variable) or a fair value hedge. The effective portion of gains and losses arising from changesin fair value of a derivative classified as a fair value hedge is included in earnings of the period of change. It is offset bylosses and gains on the hedged item that are attributable to the risk being hedged.Answer (C) is incorrect because a hedge of the foreign currency exposure of either an unrecognized firm commitment or ofa recognized asset or liability for which a foreign currency transaction gain or loss is recognized in earnings may be a cashflow hedge (if cash flows are variable) or a fair value hedge. The effective portion of gains and losses arising from changesin fair value of a derivative classified as a fair value hedge is included in earnings of the period of change. It is offset bylosses and gains on the hedged item that are attributable to the risk being hedged.Answer (D) is incorrect because gains and losses associated with changes in fair value of a derivative used as a speculationin a foreign currency are included in earnings of the period of change.

[737] Gleim #: 6.110 -- Source: CIA 1187 IV-66

A measure that describes the risk of an investment project relative to other investments in general is the

Coefficient of variation.A.Beta coefficient.B.Standard deviation.C.Expected return.D.

Answer (A) is incorrect because the coefficient of variation compares risk with expected return (standard deviation ÷expected return).Answer (B) is correct. The required rate of return on equity capital in the Capital Asset Pricing Model is the risk-freerate, plus the product of the market risk premium times the beta coefficient. The market risk premium is the amountabove the risk-free rate that will induce investment in the market. The beta coefficient of an individual share is thecorrelation between the volatility (price variation) of the stock market and that of the price of the individual share.For example, if an individual share goes up 15% and the market only 10%, beta is 1.5.Answer (C) is incorrect because standard deviation measures dispersion (risk) of project returns.Answer (D) is incorrect because expected return does not describe risk.

[738] Gleim #: 6.111 -- Source: CIA 591 IV-48

A company uses portfolio theory to develop its investment portfolio. If the company wishes to obtain optimal risk reductionthrough the portfolio effect, it should make its next investment in

An investment that correlates negatively to the current portfolio holdings.A.An investment that is uncorrelated to the current portfolio holdings.B.An investment that is highly correlated to the current portfolio holdings.C.An investment that is perfectly correlated to the current portfolio holdings.D.

Answer (A) is correct. A common general definition is that risk is an investment with an unknown outcome but aknown probability distribution of returns (a known mean and standard deviation). An increase in the standarddeviation (variability) of returns is synonymous with an increase in the riskiness of a project. Risk is also increasedwhen the project's returns are positively (directly) correlated with other investments in the company's portfolio; thatis, risk increases when returns on all projects rise or fall together. Consequently, the overall risk is decreased whenprojects have low variability and are negatively correlated (the diversification effect).Answer (B) is incorrect because uncorrelated investments are more risky than negatively correlated investments.Answer (C) is incorrect because correlated investments are very risky.Answer (D) is incorrect because correlated investments are very risky.

[739] Gleim #: 6.112 -- Source: CIA 592 IV-49

The level of risk that concerns investors who supply capital to a diversified company is

Gleim's CIA Test Prep: Part III: Business Analysis and Information Technology(1165 questions)

Copyright 2004 Gleim Publications, Inc. Page 295Printed for g j

Page 296: Part three cia_with_ answers

Project risk (beta).A.Pure play risk (beta).B.The standard deviation of project risk (betas).C.The weighted average of project risk (betas).D.

Answer (A) is incorrect because, in project risk assessment, the firm attempts to assess the level of risk for a given projectby considering, for a given project, either pure play betas (see below) or accounting betas (beta values determined fromaccounting data rather than from regression of a company's share returns on a stock market index). Neither is a sufficientmeans for assessing the overall risk of a diversified firm.Answer (B) is incorrect because, in the pure play method of project risk assessment, the firm attempts to estimate beta for asingle product company in the same line of business as the project being evaluated.Answer (C) is incorrect because the standard deviation of project betas is not employed in firm risk assessment.Answer (D) is correct. The risk of concern to investors who supply capital to a diversified company is market risk. Beta ormarket risk is the risk that cannot be eliminated by diversification and is defined from the perspective of an investor whoviews the investment as part of a diversified portfolio. It is the average risk of the firm's investment projects calculated asthe weighted average of project betas.

[740] Gleim #: 7.1 -- Source: CIA 1192 IV-16

Presented below is a cost-volume-profit chart for a firm. Various reference points are marked on the chart with letters.

(Refer to Figure CIA3_07_05.)

The letters CEH on the chart represent the

Total sales.A.Total expenses.B.Area of the chart where total sales exceed total expenses.C.Area of the chart where total expenses exceed total sales.D.

Answer (A) is incorrect because line HEO represents total sales.Answer (B) is incorrect because line CED represents total expenses.Answer (C) is correct. A cost-volume-profit chart contains elements (lines, points, axes) that identify variable cost,fixed cost, the breakeven point, total revenue, profit, and volume in units. When the total sales revenue line risesabove the total expense line, a company will have positive net income.Answer (D) is incorrect because the loss area, i.e., the area of the chart where total expenses exceed sales, isrepresented by the area OED.

[741] Gleim #: 7.2 -- Source: CIA 1196 III-91

A company manufactures a single product. Estimated cost data regarding this product and other information for the productand the company are as follows:

Sales price per unit $40Total variable production cost per unit $22Sales commission (on sales) 5%Fixed costs and expenses:

Manufacturing overhead $5,598,720General and administrative $3,732,480

Effective income tax rate 40%

The number of units the company must sell in the coming year in order to reach its breakeven point is

388,800 units.A.518,400 units.B.583,200 units.C.972,000 units.D.

Gleim's CIA Test Prep: Part III: Business Analysis and Information Technology(1165 questions)

Copyright 2004 Gleim Publications, Inc. Page 296Printed for g j

Page 297: Part three cia_with_ answers

Answer (A) is incorrect because 388,800 units results from not subtracting the variable costs per unit from sales price.Answer (B) is incorrect because 518,400 units does not reflect the sales commissions in the total variable costs.Answer (C) is correct. The breakeven point is determined by dividing total fixed costs by the unit contribution margin. Thetotal fixed costs are $9,331,200 ($5,598,720 manufacturing overhead + $3,732,480 general and administrative). Thecontribution margin is $16.00 ($40 sales price – $22 variable production cost – $2.00 commission). Thus, the breakevenpoint is 583,200 units ($9,331,200 ÷ $16).Answer (D) is incorrect because 972,000 units includes taxes in the total variable costs, which understates the unitcontribution margin.

[Fact Pattern #54]

A company that sells its single product for $40 per unit usescost-volume-profit analysis in its planning. The company’safter-tax net income for the past year was $1,188,000 afterapplying an effective tax rate of 40%. The projected costsfor manufacturing and selling its single product in thecoming year are listed.

Variable costs per unit:

Direct material $  5.00Direct labor 4.00Manufacturing overhead 6.00Selling and administrative costs 3.00

 Total cost per unit $18.00 

Annual fixed operating costs:Manufacturing overhead $6,200,000Selling and administrative costs 3,700,000

 Total annual fixed cost $9,900,000 

[742] Gleim #: 7.3 -- Source: CIA 596 III-84

(Refers to Fact Pattern #54)The dollar sales volume required in the coming year to earn the same after-tax net profit as the past year is

$20,160,000A.$21,600,000B.$23,400,000C.$26,400,000D.

Answer (A) is incorrect because $20,160,000 does not adjust after-tax net profit to pretax net profit.Answer (B) is correct. The desired after-tax net profit is $1,188,000 (the past year’s amount). Given a 40% tax rate,the pretax equivalent is $1,980,000 [$1,188,000 ÷ (1.0 – .40)]. Pretax net profit equals dollar sales (unit sales × $40),minus total fixed costs, minus total variable costs (unit sales × unit variable cost). Hence, the contribution margin(sales – variable costs) is equated with the sum of fixed costs and the targeted pretax net profit. Unit sales (S) equal540,000, and sales dollars equal $21,600,000 ($40 × 540,000 units).

$40S – $9,900,000 – $18S$22S

S units

Answer (C) is incorrect because $23,400,000 results from adjusting after-tax net profit by dividing by the tax raterather than one minus the tax rate.Answer (D) is incorrect because $26,400,000 results from equating the sum of the desired pretax net profit and totalfixed costs with total variable costs instead of the contribution margin.

[743] Gleim #: 7.4 -- Source: CIA 596 III-85

(Refers to Fact Pattern #54)The company has learned that a new direct material is available that will increase the quality of its product. The newmaterial will increase the direct material costs by $3 per unit. The company will increase the selling price of the product to$50 per unit and increase its marketing costs by $1,575,000 to advertise the higher-quality product. The number of unitsthe company has to sell in order to earn a 10% before-tax return on sales would be

Gleim's CIA Test Prep: Part III: Business Analysis and Information Technology(1165 questions)

Copyright 2004 Gleim Publications, Inc. Page 297Printed for g j

Page 298: Part three cia_with_ answers

337,500 units.A.346,875 units.B.425,000 units.C.478,125 units.D.

Answer (A) is incorrect because 337,500 units results from using the wrong sign for the pretax net profit.Answer (B) is incorrect because 346,875 units results from subtracting, not adding, the incremental marketing costs todetermine total fixed costs.Answer (C) is incorrect because 425,000 units fails to adjust for the increase in direct materials costs.Answer (D) is correct. Pretax net profit (10% of dollar sales) equals dollar sales (unit sales × $50), minus total fixed costs(increased by $1,575,000 of marketing costs), minus total variable costs (increased by $3 per unit). Unit sales (S) thereforeequal 478,125 units.

.10($50S) S – ($9,900,000 + $1,575,000) – ($18 + $3)S$24S

S units

[744] Gleim #: 7.5 -- Source: CIA 1193 IV-11

A retail company determines its selling price by marking up variable costs 60%. In addition, the company uses frequentselling price markdowns to stimulate sales. If the markdowns average 10%, what is the company’s contribution marginratio?

27.5%A.30.6%B.37.5%C.41.7%D.

Answer (A) is incorrect because 27.5% omits markdowns from the denominator.Answer (B) is correct. The contribution margin equals revenues minus variable costs. The CMR equals the UCMdivided by the selling price. For example, if variable costs average $10 per unit, the average selling price is $16 (1.60× $10). However, the 10% markdown implies that the actual average selling price is $14.40 (.90 × $16). The CMR istherefore 30.6% [($14.40 – $10.00) ÷ $14.40].Answer (C) is incorrect because 37.5% ignores markdowns.Answer (D) is incorrect because 41.7% omits markdowns from the numerator.

[Fact Pattern #55]A company manufactures and sells a single product. Planned and actual production in its first year of operation was 100,000units. Planned and actual costs for that year were as follows:

  Manufacturing Nonmanufacturing

Variable $600,000 $500,000

Fixed 400,000 300,000

The company sold 85,000 units of product at a selling price of $30 per unit.

[745] Gleim #: 7.6 -- Source: CIA 595 III-87

(Refers to Fact Pattern #55)Using absorption costing, the company’s operating profit was

$750,000A.$900,000B.$975,000C.$1,020,000D.

Gleim's CIA Test Prep: Part III: Business Analysis and Information Technology(1165 questions)

Copyright 2004 Gleim Publications, Inc. Page 298Printed for g j

Page 299: Part three cia_with_ answers

Answer (A) is incorrect because $750,000 equals absorption costing net profit minus ending inventory ($15,000 units ×$10).Answer (B) is correct. Under absorption costing, product costs include fixed and variable manufacturing costs. The unitproduct cost under absorption costing is $10 [($600,000 + $400,000) ÷ 100,000 units produced]. All nonmanufacturingcosts are expensed in the period incurred. Thus, operating profit is $900,000.

Revenue (85,000 units × $30) $2,550,000Cost of goods sold (85,000 units × $10) (850,000)Nonmanufacturing costs

($500,000 + $300,000) (800,000) Operating profit $   900,000 

Answer (C) is incorrect because $975,000 treats the variable nonmanufacturing costs as manufacturing costs.Answer (D) is incorrect because $1,020,000 assumes that all costs are manufacturing costs.

[746] Gleim #: 7.7 -- Source: CIA 595 III-88

(Refers to Fact Pattern #55)Using variable costing, the company’s operating profit was

$750,000A.$840,000B.$915,000C.$975,000D.

Answer (A) is incorrect because $750,000 equals variable costing net profit minus ending inventory (15,000 units ×$6).Answer (B) is correct. Under variable costing, the product cost includes only variable manufacturing costs. All fixedcosts are expensed in the period incurred. Unit product cost under variable costing is $6 ($600,000 ÷ 100,000 unitsproduced).

Revenue (85,000 units × $30) $2,550,000Variable cost of goods sold

(85,000 units × $6) (510,000)Variable nonmanufacturing costs (500,000) Contribution margin $1,540,000Fixed costs (700,000) Operating profit $   840,000 

Answer (C) is incorrect because $915,000 treats all variable costs as manufacturing costs.Answer (D) is incorrect because $975,000 treats all variable costs and fixed manufacturing costs as product costs.

[747] Gleim #: 7.8 -- Source: CIA 1190 IV-12

During its first year of operations, a company produced 275,000 units and sold 250,000 units. The following costs wereincurred during the year:

Variable costs per unit:Direct materials $15.00Direct labor 10.00Manufacturing overhead 12.50Selling and administrative 2.50

Total fixed costs:Manufacturing overhead $2,200,000Selling and administrative 1,375,000

The difference between operating profit calculated on the absorption-costing basis and on the variable- costing basis is thatabsorption-costing operating profit is

Gleim's CIA Test Prep: Part III: Business Analysis and Information Technology(1165 questions)

Copyright 2004 Gleim Publications, Inc. Page 299Printed for g j

Page 300: Part three cia_with_ answers

$200,000 greater.A.$220,000 greater.B.$325,000 greater.C.$62,500 less.D.

Answer (A) is correct. Absorption-costing operating profit will exceed variable-costing operating income becauseproduction exceeds sales, resulting in a deferral of fixed manufacturing overhead in the inventory calculated using theabsorption method. The difference of $200,000 is equal to the fixed manufacturing overhead per unit ($2,200,000 ÷275,000 = $8.00) times the difference between production and sales (275,000 – 250,000 = 25,000, which is the inventorychange in units).Answer (B) is incorrect because units produced, not units sold, should be used as the denominator to calculate the fixedmanufacturing cost per unit.Answer (C) is incorrect because fixed selling and administrative costs are not properly inventoriable under absorptioncosting.Answer (D) is incorrect because variable selling and administrative costs are period costs under both variable- andabsorption-cost systems in the determination of operating profit.

[Fact Pattern #56]

A and B are autonomous divisions of a corporation. Theyhave no beginning or ending inventories, and the number ofunits produced is equal to the number of units sold.Following is financial information relating to the twodivisions.

A B

Sales $150,000 $400,000

Other revenue 10,000 15,000

Direct materials 30,000 65,000

Direct labor 20,000 40,000

Variable factory overhead 5,000 15,000

Fixed factory overhead 25,000 55,000

Variable S&A expense 15,000 30,000

Fixed S&A expense 35,000 60,000

Central corporate expenses(allocated)

12,000 20,000

[748] Gleim #: 7.9 -- Source: CIA 1193 IV-20

(Refers to Fact Pattern #56)What is the total contribution to corporate profits generated by Division A before allocation of central corporate expenses?

$18,000A.$20,000B.$30,000C.$90,000D.

Answer (A) is incorrect because $18,000 is the result of deducting the central corporate expenses.Answer (B) is incorrect because $20,000 is the result of excluding other revenue.Answer (C) is correct. Division A’s total contribution to corporate profits includes everything except the centralcorporate expense allocation. Thus, the total contribution is $30,000 ($150,000 sales + $10,000 other revenue –$30,000 direct materials – $20,000 direct labor – $5,000 variable overhead – $25,000 fixed overhead – $15,000variable S&A expense – $35,000 fixed S&A expense).Answer (D) is incorrect because $90,000 is the result of failing to deduct the fixed costs.

[749] Gleim #: 7.10 -- Source: CIA 1193 IV-21

(Refers to Fact Pattern #56)What is the contribution margin of Division B?

Gleim's CIA Test Prep: Part III: Business Analysis and Information Technology(1165 questions)

Copyright 2004 Gleim Publications, Inc. Page 300Printed for g j

Page 301: Part three cia_with_ answers

$150,000A.$205,000B.$235,000C.$265,000D.

Answer (A) is incorrect because $150,000 is the result of subtracting fixed costs.Answer (B) is incorrect because $205,000 is the result of subtracting fixed S&A costs.Answer (C) is incorrect because $235,000 is the result of subtracting fixed S&A costs but not variable S&A costs.Answer (D) is correct. The contribution margin equals revenue minus variable costs. Thus, Division B’s contributionmargin is $265,000 ($400,000 sales + $15,000 other revenue – $65,000 direct materials – $40,000 direct labor – $15,000variable overhead – $30,000 variable S&A expense).

[750] Gleim #: 7.11 -- Source: CIA 595 III-94

A company allocates overhead to jobs in process using direct labor costs, raw material costs, and machine hours. Theoverhead application rates for the current year are

100% of direct labor20% of raw materials$117 per machine hour

A particular production run incurred the following costs:

Direct labor, $8,000Raw materials, $2,000A total of 140 machine hours wererequired for the production run

What is the total cost that would be charged to the production run?

$18,000A.$18,400B.$24,780C.None of the answers are correct.D.

Answer (A) is incorrect because $18,000 includes only $8,000 for overhead (based on 100% of direct labor).Answer (B) is incorrect because $18,400 includes only $8,400 for overhead (based on 100% of direct labor and 20%of raw materials).Answer (C) is correct. The total cost is $34,780. It consists of direct labor ($8,000), raw materials ($2,000), andfactory overhead ($24,780). The overhead applied is calculated as follows:

100% × $8,000 of direct labor $  8,00020% × $2,000 of raw materials 400$117 × 140 machine hours 16,380       $24,780   

Answer (D) is incorrect because total cost is $34,780.

Gleim's CIA Test Prep: Part III: Business Analysis and Information Technology(1165 questions)

Copyright 2004 Gleim Publications, Inc. Page 301Printed for g j

Page 302: Part three cia_with_ answers

[751] Gleim #: 7.12 -- Source: CIA 1184 IV-21A company manufactures plastic products for the home and restaurant market. The company also does contract work forother customers and uses a job-order costing system. The flexible budget covering next year’s expected range of activity is

Direct labor hours 50,000 80,000 110,000 Machine hours 40,000 64,000 88,000 Variable O/H costs $100,000 $160,000 $220,000Fixed O/H costs 150,000 150,000 150,000 Total O/H costs $250,000 $310,000 $370,000 

A predetermined overhead rate based on direct labor hours is used to apply total overhead. Management has estimated that100,000 direct labor hours will be used next year. The predetermined overhead rate per direct labor hour to be used toapply total overhead to the individual jobs next year is

$3.36A.$3.50B.$3.70C.$3.88D.

Answer (A) is incorrect because $3.36 per direct labor hour is based on use of an activity level of 110,000 directlabor hours to determine the fixed overhead rate.Answer (B) is correct. The predetermined overhead rate is calculated by dividing the total fixed overhead by theactivity level to arrive at a unit fixed overhead cost that is added to the unit variable overhead cost. The unit variableoverhead rate is the same at each activity level. Thus, the predetermined overhead rate is $3.50 [($150,000 FOH ÷100,000 hrs.) + ($220,000 VOH ÷ 110,000 hrs.)].Answer (C) is incorrect because $3.70 is the result of assuming that $220,000 of variable overhead will be incurredfor 100,000 (not 110,000) direct labor hours.Answer (D) is incorrect because $3.88 (rounded) results from using an activity level of 80,000 direct labor hours todetermine the fixed overhead rate.

[752] Gleim #: 7.13 -- Source: Publisher

At the beginning of the year, Smith Inc. budgeted the following:

Units 10,000 Sales $100,000Minus:

Total variable expenses 60,000Total fixed expenses 20,000

 Net income $  20,000 Factory overhead:

Variable $  30,000Fixed 10,000

There were no beginning inventories. At the end of the year, no work was in process, total factory overhead incurred was$39,500, and underapplied factory overhead was $1,500. Factory overhead was applied on the basis of budgeted unitproduction. How many units were produced this year?

10,250A.10,000B.9,875C.9,500D.

Gleim's CIA Test Prep: Part III: Business Analysis and Information Technology(1165 questions)

Copyright 2004 Gleim Publications, Inc. Page 302Printed for g j

Page 303: Part three cia_with_ answers

Answer (A) is incorrect because 10,250 would have been produced if overhead had been overapplied by $1,500 [($39,500 +$1,500) ÷ $4].Answer (B) is incorrect because 10,000 is the result of dividing budgeted, not applied, overhead by the application rate.Answer (C) is incorrect because 9,875 units would have been produced if $39,500 had been the amount of appliedoverhead.Answer (D) is correct. Given actual overhead of $39,500 and underapplied overhead of $1,500, overhead applied was$38,000 ($39,500 – $1,500). Overhead is applied at the rate of $4 per unit ($40,000 budgeted overhead ÷ 10,000 budgetedunits). Accordingly, 9,500 units were produced ($38,000 applied overhead ÷ $4 per unit application rate).

[Fact Pattern #57]

Kimbeth Manufacturing uses a process cost system tomanufacture Dust Density Sensors for the mining industry.The following information pertains to operations for themonth of May:

  Units    Beginning work-in-process inventory,

May 1 16,000Started in production during May 100,000Completed production during May 92,000Ending work-in-process inventory,

May 31 24,000

The beginning inventory was 60% complete for materials and20% complete for conversion costs. The ending inventorywas 90% complete for materials and 40% complete forconversion costs.

Costs pertaining to the month of May are as follows:

Beginning inventory costs are materials, $54,560;direct labor, $20,320; and factory overhead, $15,240.Costs incurred during May are materials used,$468,000; direct labor, $182,880; and factoryoverhead, $391,160.

[753] Gleim #: 7.14 -- Source: CMA 695 3-1

(Refers to Fact Pattern #57)Using the first-in, first-out (FIFO) method, the equivalent units of production (EUP) for materials are

97,600 units.A.104,000 units.B.107,200 units.C.108,000 units.D.

Answer (A) is incorrect because 97,600 units omits the 6,400 EUP added to beginning work-in-process.Answer (B) is correct. Under FIFO, EUP are based solely on work performed during the current period. The EUPequals the sum of the work done on the beginning work-in-process inventory, units started and completed in thecurrent period, and the ending work-in-process inventory. Given that beginning work-in-process was 60% completeas to materials, the current period is charged for 6,400 EUP (40% × 16,000 units). Because 92,000 units werecompleted during the period, 76,000 (92,000 – 16,000 in BWIP) must have been started and completed during theperiod. They represent 76,000 EUP. Finally, the EUP for ending work-in-process equal 21,600 (90% × 24,000 units).Thus, total EUP for May are 104,000 (6,400 + 76,000 + 21,600).Answer (C) is incorrect because 107,200 units assumes beginning work-in-process was 40% complete.Answer (D) is incorrect because 108,000 units equals the sum of the physical units in beginning work-in-process andthe physical units completed.

[754] Gleim #: 7.15 -- Source: CMA 695 3-2

(Refers to Fact Pattern #57)Using the FIFO method, the equivalent units of production for conversion costs are

85,600 units.A.88,800 units.B.95,200 units.C.98,400 units.D.

Gleim's CIA Test Prep: Part III: Business Analysis and Information Technology(1165 questions)

Copyright 2004 Gleim Publications, Inc. Page 303Printed for g j

Page 304: Part three cia_with_ answers

Answer (A) is incorrect because 85,600 units omits the work done on beginning work-in-process.Answer (B) is incorrect because 88,800 units omits the work done on ending work-in-process.Answer (C) is incorrect because 95,200 units assumes the beginning work-in-process was 40% complete as to conversioncosts.Answer (D) is correct. The beginning inventory was 20% complete as to conversion costs. Hence, 12,800 EUP (80% ×16,000 units) were required for completion. EUP for units started and completed equaled 76,000 [100% × (92,000completed units – 16,000 units in BWIP)]. The work done on ending work-in-process totaled 9,600 EUP (40% × 24,000units). Thus, total EUP for May are 98,400 (12,800 + 76,000 + 9,600).

[755] Gleim #: 7.16 -- Source: CMA 695 3-3

(Refers to Fact Pattern #57)Using the FIFO method, the equivalent unit cost of materials for May is

$4.12A.$4.50B.$4.60C.$4.80D.

Answer (A) is incorrect because $4.12 is based on EUP calculated under the weighted-average method.Answer (B) is correct. Under the FIFO method, EUP for materials equal 104,000 [(16,000 units in BWIP × 40%) +(76,000 units started and completed × 100%) + (24,000 units in EWIP × 90%)]. Consequently, the equivalent unitcost of materials is $4.50 ($468,000 total materials cost in May ÷ 104,000 EUP).Answer (C) is incorrect because $4.60 is the weighted-average cost per equivalent unit.Answer (D) is incorrect because $4.80 omits the 6,400 EUP added to beginning work-in-process.

[756] Gleim #: 7.17 -- Source: CMA 695 3-4

(Refers to Fact Pattern #57)Using the FIFO method, the equivalent unit conversion cost for May is

$5.65A.$5.83B.$6.00C.$6.20D.

Answer (A) is incorrect because $5.65 is based on EUP calculated under the weighted-average method.Answer (B) is correct. Under the FIFO method, EUP for conversion costs equal 98,400 [(16,000 units in BWIP ×80%) + (76,000 units started and completed × 100%) + (24,000 units in EWIP × 40%)]. Conversion costs incurredduring the current period equal $574,040 ($182,880 DL + $391,160 FOH). Hence, the equivalent unit cost forconversion costs is $5.83 ($574,040 ÷ 98,400).Answer (C) is incorrect because $6.00 is the cost per equivalent unit calculated under the weighted-average method.Answer (D) is incorrect because $6.20 results from combining conversion costs for May with those in beginningwork-in-process and dividing by 98,400 EUP.

[757] Gleim #: 7.18 -- Source: CMA 695 3-5

(Refers to Fact Pattern #57)Using the FIFO method, the total cost of units in the ending work-in-process inventory at May 31 is

$153,168A.$154,800B.$155,328C.$156,960D.

Gleim's CIA Test Prep: Part III: Business Analysis and Information Technology(1165 questions)

Copyright 2004 Gleim Publications, Inc. Page 304Printed for g j

Page 305: Part three cia_with_ answers

Answer (A) is correct. The FIFO costs per equivalent unit for materials and conversion costs are $4.50 and $5.83,respectively. EUP for materials in ending work-in-process equal 21,600 (90% × 24,000). Thus, total FIFO materials cost is$97,200 (21,600 EUP × $4.50). EUP for conversion costs in ending work-in-process equal 9,600 (40% × 24,000). Totalconversion costs are therefore $55,968 (9,600 EUP × $5.83). Consequently, total work-in-process costs are $153,168($97,200 + $55,968).Answer (B) is incorrect because $154,800 is based on a FIFO calculation for materials and a weighted-average calculationfor conversion costs.Answer (C) is incorrect because $155,328 is based on a weighted-average calculation for materials and a FIFO calculationfor conversion costs.Answer (D) is incorrect because $156,960 is the weighted- average cost of ending work-in-process.

[Fact Pattern #58]Believing that its traditional cost system may be providing misleading information, an organization is considering an activity-based costing (ABC) approach. It now employs a full cost system and has been applying its manufacturing overhead on the basisof machine hours.

The organization plans on using 50,000 direct labor hours and 30,000 machine hours in the coming year. The following datashow the manufacturing overhead that is budgeted.

    Budgeted BudgetedActivity Cost Driver Activity Cost

Material handling No. of parts handled 6,000,000 $   720,000Setup costs No. of setups 750 315,000Machining costs Machine hours 30,000 540,000Quality control No. of batches 500 225,000       Total manufacturing overhead cost: $1,800,000     

Cost, sales, and production data for one of the organization’s products for the coming year are as follows:

Prime costs:

Direct material cost per unit $4.40Direct labor cost per unit .05 DLH @ $15.00/DLH .75 Total prime cost $5.15 

Sales and production data:

Expected sales 20,000 unitsBatch size 5,000 unitsSetups 2 per batchTotal parts per finished unit 5 partsMachine hours required 80 MH per batch

[758] Gleim #: 7.19 -- Source: CIA 1195 III-93

(Refers to Fact Pattern #58)If the organization uses the traditional full cost system, the cost per unit for this product for the coming year will be

$5.39A.$5.44B.$6.11C.$6.95D.

Answer (A) is incorrect because $5.39 assumes that 80 machine hours are required for the total production of 20,000units.Answer (B) is incorrect because $5.44 is based on the machining overhead rate ($18).Answer (C) is correct. Given that manufacturing overhead is applied on the basis of machine hours, the overhead rateis $60 per hour ($1,800,000 ÷ 30,000) or $.96 per unit [($60 × 80 machine hours per batch) ÷ 5,000 units per batch].Accordingly, the unit full cost is $6.11 ($5.15 unit price cost + $.96).Answer (D) is incorrect because $6.95 is based on the direct labor hour manufacturing overhead rate.

Gleim's CIA Test Prep: Part III: Business Analysis and Information Technology(1165 questions)

Copyright 2004 Gleim Publications, Inc. Page 305Printed for g j

Page 306: Part three cia_with_ answers

[759] Gleim #: 7.20 -- Source: CIA 1195 III-94

(Refers to Fact Pattern #58)If the organization employs an activity-based costing system, the cost per unit for the product described for the coming yearwill be

$6.00A.$6.08B.$6.21C.$6.30D.

Answer (A) is incorrect because $6.00 assumes one setup per batch and 80 total machine hours.Answer (B) is incorrect because $6.08 assumes that only 80 machine hours were used.Answer (C) is incorrect because $6.21 assumes one setup per batch.Answer (D) is correct. Materials handling cost per part is $.12 ($720,000 ÷ 6,000,000), cost per setup is $420($315,000 ÷ 750), machining cost per hour is $18 ($540,000 ÷ 30,000), and quality cost per batch is $450 ($225,000÷ 500). Hence, total manufacturing overhead applied is $22,920 [(5 parts per unit) × 20,000 units × $.12) + (4batches × 2 setups per batch × $420) + (4 batches × 80 machine hours per batch × $18) + (4 batches × $450)]. Thetotal unit cost is $6.296 [$5.15 prime cost + ($22,920 ÷ 20,000 units) overhead].

Gleim's CIA Test Prep: Part III: Business Analysis and Information Technology(1165 questions)

Copyright 2004 Gleim Publications, Inc. Page 306Printed for g j

Page 307: Part three cia_with_ answers

[Fact Pattern #59]This information was presented as part of Question 3 on Part 3 of the June 1992 CMA examination, which covered activity-basedcosting.Alaire Corporation manufactures several different types of printed circuit boards; however, two of the boards account for themajority of the company’s sales. The first of these boards, a television (TV) circuit board, has been a standard in the industry forseveral years. The market for this type of board is competitive and therefore price-sensitive. Alaire plans to sell 65,000 of the TVboards next year at a price of $150 per unit. The second high-volume product, a personal computer (PC) circuit board, is a recentaddition to Alaire’s product line. Because the PC board incorporates the latest technology, it can be sold at a premium price;plans include the sale of 40,000 PC boards at $300 per unit.

Alaire’s management group is meeting to discuss strategies, and the current topic of conversation is how to spend the sales andpromotion dollars for next year. The sales manager believes that the market share for the TV board could be expanded byconcentrating Alaire’s promotional efforts in this area. In response to this suggestion, the production manager said, “Why don’tyou go after a bigger market for the PC board?  The cost sheets that I get show that the contribution from the PC board is morethan double the contribution from the TV board. I know we get a premium price for the PC board; selling it should help overallprofitability.”

Alaire uses a standard cost system, and the following data apply to the TV and PC boards:

  TV Board PC Board Direct materials $80 $140Direct labor 1.5 hours 4 hoursMachine time .5 hours 1.5 hours

Variable factory overhead is applied on the basis of direct labor hours. For next year, variable factory overhead is budgeted at$1,120,000, and direct labor hours are estimated at 280,000. The hourly rates for machine time and direct labor are $10 and $14,respectively. Alaire applies a materials handling charge at 10% of materials cost; this materials handling charge is not included invariable factory overhead. Total annual expenditures for materials are budgeted at $10,600,000.

Ed Welch, Alaire’s controller, believes that, before the management group proceeds with the discussion about allocating sales andpromotional dollars to individual products, they should consider the activities involved in their production. As Welch explainedto the group, “Activity-based costing integrates the cost of all activities, known as cost drivers, into individual product costsrather than including these costs in overhead pools.”  Welch has prepared the schedule shown below to help the managementgroup understand this concept.

“Using this information,” Welch explained, “we can calculate an activity-based cost for each TV board and each PC board andthen compare it with the standard cost we have been using. The only cost that remains the same for both cost methods is the costof direct materials. The cost drivers will replace the direct labor, machine time, and overhead costs in the standard cost.”

Budgeted Cost

Materials overhead:Procurement $    400,000Production scheduling 220,000Packaging and shipping 440,000

   $1,060,000 Variable overhead:

Machine setup $   446,000Hazardous waste disposal 48,000Quality control 560,000General supplies 66,000

   $1,120,000 Manufacturing:

Machine insertion $1,200,000Manual insertion 4,000,000Wave soldering 132,000

   $5,332,000 

  Annual ActivityCost Driver for Cost Driver

Number of parts 4,000,000 partsNumber of boards 110,000 boardsNumber of boards 110,000 boards   Number of setups 278,750 setupsPounds of waste 16,000 poundsNumber of inspections 160,000 inspectionsNumber of boards 110,000 boards   Number of parts 3,000,000 partsNumber of parts 1,000,000 partsNumber of boards 110,000 boards

Gleim's CIA Test Prep: Part III: Business Analysis and Information Technology(1165 questions)

Copyright 2004 Gleim Publications, Inc. Page 307Printed for g j

Page 308: Part three cia_with_ answers

[760] Gleim #: 7.21 -- Source: Publisher

(Refers to Fact Pattern #59)On the basis of standard costs, the total contribution budgeted for the TV board is

$1,950,000A.$2,275,000B.$2,340,000C.$2,470,000D.

Answer (A) is correct. As calculated below, the budgeted standard unit cost of a TV board is $120. This amountincludes $6 of variable overhead [1.5 DLH × ($1,120,000 total VOH ÷ 280,000 DLH)]. Given a unit price of $150,the unit contribution margin is therefore $30. Total budgeted contribution is $1,950,000 ($30 UCM × 65,000budgeted units).

Direct materials $  80DM handling (10% × $80) 8Direct labor (1.5 hr. × $14) 21Machine time (.5 hr. × $10) 5Variable overhead (1.5 hr. × $4) 6 Budgeted unit cost $120 

Answer (B) is incorrect because $2,275,000 excludes the cost of machine time ($5 × 65,000 units = $325,000).Answer (C) is incorrect because $2,340,000 excludes the variable overhead ($6 × 65,000 units = $390,000).Answer (D) is incorrect because $2,470,000 excludes the direct materials handling cost ($8 × 65,000 units =$520,000).

[761] Gleim #: 7.22 -- Source: Publisher

(Refers to Fact Pattern #59)On the basis of activity-based costs (ABC), the total contribution budgeted for the TV board is

$1,594,000A.$1,950,000B.$2,037,100C.$2,557,100D.

Answer (A) is incorrect because $1,594,000 is the CM for the PC board based on an ABC calculation.Answer (B) is incorrect because $1,950,000 is the CM for the TV board based on a standard-cost calculation.Answer (C) is incorrect because $2,037,100 erroneously includes $8 per board for materials handling ($8 × 65,000units = $520,000).Answer (D) is correct. As calculated below, the budgeted activity-based unit cost of a TV board is $110.66. Given aunit price of $150, the unit contribution margin is $39.34. Total budgeted contribution is $2,557,100 ($39.34 UCM ×65,000 budgeted units).

Direct materials $  80.00Procurement [($400,000 ÷ 4,000,000 parts) × 25 parts] 2.50Scheduling ($220,000 ÷ 110,000 boards) 2.00Packaging and shipping ($440,000 ÷ 110,000 boards) 4.00Setups [($446,000 ÷ 278,750 setups) × 2 setups] 3.20Waste disposal [($48,000 ÷ 16,000 lb.) × .02] .06Quality control ($560,000 ÷ 160,000 inspections) 3.50General supplies ($66,000 ÷ 110,000 boards) .60Machine insertion [(1,200,000 ÷ 3,000,000 parts) × 24parts]

9.60

Manual insertion ($4,000,000 ÷ 1,000,000 parts) 4.00Soldering ($132,000 ÷ 110,000 boards) 1.20 Budgeted ABC unit cost $110.66 

Gleim's CIA Test Prep: Part III: Business Analysis and Information Technology(1165 questions)

Copyright 2004 Gleim Publications, Inc. Page 308Printed for g j

Page 309: Part three cia_with_ answers

[Fact Pattern #60]Fabricating and Finishing are the two production departments of a manufacturing company. Building Operations and InformationServices are service departments that provide support to the two production departments as well as to each other. The companyemploys departmental overhead rates in the two production departments to allocate the service department costs to the productiondepartments. Square footage is used to allocate building operations, and computer time is used to allocate information services.The costs of the service departments and relevant operating data for the departments are as follows:

  Building Operations Information Services Fabricating Finishing

Labor and benefit costs $200,000 $    300,000

Other traceable costs 350,000 900,000

Total $550,000 $1,200,000

Square feet occupied 5,000 10,000 16,000 24,000

Computer time (in hours) 200 1,200 600

[762] Gleim #: 7.23 -- Source: CIA 596 III-82

(Refers to Fact Pattern #60)If the company employs the direct method to allocate the costs of the service departments, then the amount of BuildingOperations costs allocated to Fabricating would be

$140,000A.$160,000B.$176,000C.$220,000D.

Answer (A) is incorrect because $140,000 is 40% of other traceable costs.Answer (B) is incorrect because $160,000 assumes an allocation base of 55,000 square feet.Answer (C) is incorrect because $176,000 assumes an allocation base of 50,000 square feet.Answer (D) is correct. The direct method does not allocate service costs to other service departments. Hence, theallocation base is the square footage in the two production departments. Fabricating’s share is 40% (16,000 ÷ 40,000)of the total cost incurred by Building Operations, or $220,000 (40% × $550,000).

[763] Gleim #: 7.24 -- Source: CIA 596 III-83

(Refers to Fact Pattern #60)If the company employs the step method to allocate the costs of the service departments and if Information Services costsare allocated first, then the total amount of service department costs (information services and building operations)allocated to Finishing would be

$657,000A.$681,600B.$730,000C.$762,000D.

Answer (A) is incorrect because $657,000 results from allocating Building Operations costs first.Answer (B) is incorrect because $681,600 results from allocating Building Operations costs to Information Services.Answer (C) is incorrect because $730,000 allocates the costs of both service departments according to the directmethod rather than the step method.Answer (D) is correct. The step method of service department cost allocation is a sequential (but not a reciprocal)process. These costs are allocated to other service departments as well as to users. The process usually starts with theservice department that renders the greatest percentage of its services to other service departments, the departmentthat provides services to the greatest number of other departments, or the department with the greatest costs ofservices provided to other service departments. If the $1,200,000 of Information Services costs is allocated first, theallocation base is 2,000 computer hours (200 + 1,200 + 600). Thus, $120,000 [(200 ÷ 2,000) × $1,200,000] will beallocated to Building Operations and $360,000 [(600 ÷ 2,000) × $1,200,000] to Finishing. The total of the BuildingOperations costs to be allocated to production equals $670,000 ($550,000 + $120,000). The allocation base will be40,000 square feet because no costs are allocated back to Information Services. Accordingly, the total of service costsallocated to Finishing equals $762,000 {$360,000 + [$670,000 × (24,000 ÷ 40,000)]}.

Gleim's CIA Test Prep: Part III: Business Analysis and Information Technology(1165 questions)

Copyright 2004 Gleim Publications, Inc. Page 309Printed for g j

Page 310: Part three cia_with_ answers

[764] Gleim #: 7.25 -- Source: CIA 1190 IV-8The step-down method of service department cost allocation often begins with allocation of the costs of the servicedepartment that

Provides the greatest percentage of its services to the production departments.A.Provides the greatest percentage of its services to other service departments.B.Provides the greatest total output of services.C.Has the highest total costs among the service departments.D.

Answer (A) is incorrect because the step-down method may start with the department that renders the highestpercentage of its total services to other service (not production) departments.Answer (B) is correct. The step-down method may start with the department that renders the highest percentage of itstotal services to other service departments. It then progresses in descending order to the service department renderingthe least percentage of its services to the other service departments. An alternative is to begin with the department thatrenders the highest dollar value of services to other service departments. A third possibility is to begin with thedepartment that renders service to the greatest number of other service departments.Answer (C) is incorrect because beginning with the service department with the greatest output is not customary.Answer (D) is incorrect because beginning with the service department with the highest costs is not customary.

[Fact Pattern #61]

Atlas Foods produces three supplemental food productssimultaneously through a refining process costing $93,000.

The joint products, Alfa and Betters, have a final sellingprice of $4 per pound and $10 per pound, respectively, afteradditional processing costs of $2 per pound of each productare incurred after the split-off point. Morefeed, a by-product,is sold at the split-off point for $3 per pound.

Alfa 10,000 pounds of Alfa, a popular butrelatively rare grain supplement having acaloric value of 4,400 calories per pound.

Betters 5,000 pounds of Betters, a flavoring materialhigh in carbohydrates with a caloric value of11,200 calories per pound.

Morefeed 1,000 pounds of Morefeed, used as a cattlefeed supplement with a caloric value of1,000 calories per pound.

[765] Gleim #: 7.26 -- Source: CMA 1293 3-3

(Refers to Fact Pattern #61)Assuming Atlas Foods inventories Morefeed, the by-product, the joint cost to be allocated to Alfa, using the net realizablevalue method is

$3,000A.$30,000B.$31,000C.$60,000D.

Answer (A) is incorrect because $3,000 is the value of the by-product.Answer (B) is correct. The NRV at split-off for each of the joint products must be determined. Given that Alfa has a$4 selling price and an additional $2 of processing costs, the value at split-off is $2 per pound. The total value atsplit-off for 10,000 pounds is $20,000. Betters has a $10 selling price and an additional $2 of processing costs. Thus,the value at split-off is $8 per pound. The total value of 5,000 pounds of Betters is therefore $40,000. The 1,000pounds of Morefeed has a split-off value of $3 per pound, or $3,000. Assuming that Morefeed (a by-product) isinventoried (recognized in the accounts when produced) and treated as a reduction of joint costs, the allocable jointcost is $90,000 ($93,000 – $3,000). (NOTE:  Other methods of accounting for by-products are possible.) The totalnet realizable value of the main products is $60,000 ($20,000 Alfa + $40,000 Betters). The allocation to Alfa is$30,000 [($20,000 ÷ $60,000) × $90,000].Answer (C) is incorrect because $31,000 fails to adjust the joint processing cost for the value of the by-product.Answer (D) is incorrect because $60,000 is the amount allocated to Betters.

[766] Gleim #: 7.27 -- Source: CMA 1293 3-4

(Refers to Fact Pattern #61)Assuming Atlas Foods inventories Morefeed, the by-product, the joint cost to be allocated to Alfa, using the physicalquantity method is

Gleim's CIA Test Prep: Part III: Business Analysis and Information Technology(1165 questions)

Copyright 2004 Gleim Publications, Inc. Page 310Printed for g j

Page 311: Part three cia_with_ answers

$3,000A.$30,000B.$31,000C.$60,000D.

Answer (A) is incorrect because $3,000 is the value of the by-product.Answer (B) is incorrect because $30,000 is based on the net realizable value method.Answer (C) is incorrect because $31,000 is based on the net realizable value method and fails to adjust the joint processingcost for the value of the by-product.Answer (D) is correct. Joint cost is $93,000, and Morefeed has a split-off value of $3,000. Assuming the latter amount istreated as a reduction in joint cost, the allocable joint cost is $90,000. The total physical quantity (volume) of the two jointproducts is 15,000 pounds (10,000 Alfa + 5,000 Betters). Hence, $60,000 [(10,000 ÷ 15,000) × $90,000] of the net jointcosts should be allocated to Alfa.

[767] Gleim #: 7.28 -- Source: CMA 1293 3-5

(Refers to Fact Pattern #61)Assuming Atlas Foods inventories Morefeed, the by-product, the joint cost to be allocated to Betters using the weightedquantity method based on caloric value per pound is

$39,208A.$39,600B.$50,400C.$52,080D.

Answer (A) is incorrect because $39,208 is the amount allocated to Alfa if the 1,000,000 calories attributable toMorefeed is included in the computation.Answer (B) is incorrect because $39,600 is the allocation to Alfa.Answer (C) is correct. The net allocable joint cost is $90,000. The caloric value of Alfa is 44,000,000 (4,400 ×10,000 pounds), the caloric value of Betters is 56,000,000 (11,200 × 5,000 pounds), and the total is 100,000,000. Ofthis total volume, Alfa makes up 44% and Betters 56%. Thus, $50,400 (56% × $90,000) should be allocated toBetters.Answer (D) is incorrect because $52,080 is the allocation to Betters if the joint cost is not adjusted for the value ofthe by-product.

[768] Gleim #: 7.29 -- Source: CMA 1293 3-6

(Refers to Fact Pattern #61)Assuming Atlas Foods inventories Morefeed, the by-product, the joint cost to be allocated to Alfa using the gross marketvalue method is

$36,000A.$40,000B.$41,333C.$50,000D.

Answer (A) is incorrect because $36,000 is based on 40%, not 4/9.Answer (B) is correct. Alfa has a gross sales value of $40,000 ($4 × 10,000 pounds), Betters has a gross sales valueof $50,000 ($10 × 5,000 pounds), and Morefeed has a split-off value of $3,000. If the value of Morefeed isinventoried and treated as a reduction in joint cost, the allocable joint cost is $90,000 ($93,000 – $3,000). The totalgross sales value of the two main products is $90,000 ($40,000 + $50,000). Of this total value, $40,000 should beallocated to Alfa [($40,000 ÷ $90,000) × $90,000].Answer (C) is incorrect because $41,333 fails to adjust the joint cost by the value of the by-product.Answer (D) is incorrect because $50,000 is the joint cost allocated to Betters.

Gleim's CIA Test Prep: Part III: Business Analysis and Information Technology(1165 questions)

Copyright 2004 Gleim Publications, Inc. Page 311Printed for g j

Page 312: Part three cia_with_ answers

[Fact Pattern #62]

A company produces a gasoline additive. The standard costsand input for a 500-liter batch of the additive are presentedbelow.

  Standard Standard  Input Quantity CostChemical in Liters per Liter Total Cost

Echol 200 $.200 $  40.00Protex 100 .425 42.50Benz 250 .150 37.50CT-40 50 .300 15.00

   600 $135.00 

The quantities purchased and used during the current periodare shown below. A total of 140 batches were made during thecurrent period.

  Quantity Pur- Total QuantityChemical chased (Liters) Purchase Price Used (Liters)

Echol 25,000 $  5,365 26,600Protex 13,000 6,240 12,880Benz 40,000 5,840 37,800CT-40 7,500 2,220 7,140

 Total 85,500 $19,665 84,420

 

[769] Gleim #: 7.30 -- Source: Publisher

(Refers to Fact Pattern #62)What is the materials mix variance for this operation?

$294 favorable.A.$388.50 favorable.B.$94.50 unfavorable.C.$219.50 favorable.D.

Answer (A) is incorrect because $294 favorable is the materials quantity variance.Answer (B) is correct. The materials mix variance equals the actual total quantity used times the difference betweenthe budgeted weighted- average standard unit cost for the budgeted mix and the budgeted weighted-average standardunit cost for the actual mix. This variance is favorable if the standard weighted-average cost for the actual mix is lessthan the standard weighted- average cost for the budgeted mix. The standard mix weighted-average standard unit costis $.225 per liter ($135 standard total cost ÷ 600 liters).

The standard cost of the actual quantity used was $18,606 (see below). Thus, the actual mix weighted-averagestandard unit cost was $.220398 ($18,606 ÷ 84,420 liters used), and the mix variance was $388.50 favorable[($.220398 – $.225) × 84,420 liters].

$.200 × 26,600  = $  5,320.00.425 × 12,880  = 5,474.00.150 × 37,800  = 5,670.00.300 ×   7,140  = 2,142.00

   $18,606.00 

Answer (C) is incorrect because $94.50 unfavorable is the materials yield variance.Answer (D) is incorrect because $219.50 favorable is based on the actual mix of purchases.

[770] Gleim #: 7.31 -- Source: Publisher

(Refers to Fact Pattern #62)What is the materials yield variance for this operation?

$294.50 favorable.A.$388.50 favorable.B.$94.50 unfavorable.C.$219.50 favorable.D.

Gleim's CIA Test Prep: Part III: Business Analysis and Information Technology(1165 questions)

Copyright 2004 Gleim Publications, Inc. Page 312Printed for g j

Page 313: Part three cia_with_ answers

Answer (A) is incorrect because $294.50 favorable is the materials quantity variance.Answer (B) is incorrect because $388.50 favorable is the materials mix variance.Answer (C) is correct. The materials yield variance equals the difference between the actual input and the standard inputallowed for the actual output, times the budgeted weighted-average standard cost per input unit at the standard mix. Thestandard input for the actual output was 84,000 liters (140 batches × 600 liters per batch). The standard mix budgetedweighted-average standard unit cost is $.225 per liter ($135 total cost ÷ 600 liters). Thus, the yield variance is $94.50unfavorable [(84,420 liters used – 84,000 liters allowed) × $.225].Answer (D) is incorrect because $219.50 favorable is based on the actual mix of purchases.

[771] Gleim #: 7.32 -- Source: CIA 592 IV-18

The following is a standard cost variance analysis report on direct labor cost for a division of a manufacturing company:

Job ActualHours atActualWages

ActualHours atStandardWages

StandardHours atStandardWages

213 $3,243 $3,700 $3,100

215 $15,345 $15,675 $15,000

217 $6,754 $7,000 $6,600

219 $19,788 $18,755 $19,250

221 $3,370 $3,470 $2,650

Totals $48,500 $48,600 $46,600

What is the total flexible budget direct labor variance for the division?

$100 favorable.A.$1,900 unfavorable.B.$1,900 favorable.C.$2,000 unfavorable.D.

Answer (A) is incorrect because the direct labor rate variance is $100 favorable ($48,500 – $48,600).Answer (B) is correct. The total flexible budget direct labor variance equals the difference between total actual directlabor cost and standard direct labor cost allowed for the actual output. It combines the direct labor rate and efficiencyvariances. For this company, the variance is $1,900 unfavorable ($48,500 actual wages at actual hours – $46,600standard wages at standard hours).Answer (C) is incorrect because the total labor variance is unfavorable.Answer (D) is incorrect because the direct labor efficiency variance is $2,000 unfavorable.

[Fact Pattern #63]

Water Control Inc. manufactures water pumps and uses astandard cost system. The standard factory overhead costs perwater pump are based on direct labor hours and are as follows:

Variable overhead (4 hours at $8/hour) $32Fixed overhead (4 hours at $5*/hour) 20

Total overhead cost per unit $52

*Based on a capacity of 100,000 direct labor hours per month.

The following additional information is available for themonth of November:

22,000 pumps were produced although 25,000 hadbeen scheduled for production.94,000 direct labor hours were worked at a total costof $940,000.The standard direct labor rate is $9 per hour.The standard direct labor time per unit is 4 hours.Variable overhead costs were $740,000.Fixed overhead costs were $540,000.

[772] Gleim #: 7.33 -- Source: CMA 1294 3-26

(Refers to Fact Pattern #63)The fixed overhead spending variance for November was

Gleim's CIA Test Prep: Part III: Business Analysis and Information Technology(1165 questions)

Copyright 2004 Gleim Publications, Inc. Page 313Printed for g j

Page 314: Part three cia_with_ answers

$40,000 unfavorable.A.$70,000 unfavorable.B.$460,000 unfavorable.C.$240,000 unfavorable.D.

Answer (A) is correct. The fixed overhead spending (budget) variance is the difference between actual and budgeted fixedfactory overhead. Actual fixed overhead was $540,000. Budgeted fixed overhead was $5 per hour based on a capacity of100,000 direct labor hours per month, or $500,000. Because these costs are fixed, the budgeted fixed overhead is the sameat any level of production. Hence, the variance is $40,000 unfavorable ($540,000 – $500,000).Answer (B) is incorrect because $70,000 unfavorable is the difference between actual fixed overhead and the product of thestandard rate and the actual direct labor hours.Answer (C) is incorrect because $460,000 unfavorable is the volume variance.Answer (D) is incorrect because $240,000 unfavorable is the difference between actual variable overhead and budgetedfixed overhead.

[773] Gleim #: 7.34 -- Source: CMA 1294 3-27

(Refers to Fact Pattern #63)The variable overhead spending variance for November was

$60,000 favorable.A.$12,000 favorable.B.$48,000 unfavorable.C.$40,000 unfavorable.D.

Answer (A) is incorrect because $60,000 favorable is based on 100,000 hours, not the actual hours of 94,000.Answer (B) is correct. The variable overhead spending variance is the difference between actual variable overheadand the variable overhead based on the standard rate and the actual activity level. Thus, the variable overheadspending variance was $12,000 favorable [$740,000 actual cost – ($8 standard rate × 94,000 actual hours)]. Becauseactual is less than standard, the variance was favorable.Answer (C) is incorrect because $48,000 unfavorable is the variable overhead efficiency variance.Answer (D) is incorrect because $40,000 unfavorable is the fixed overhead spending variance.

[774] Gleim #: 7.35 -- Source: CMA 1294 3-28

(Refers to Fact Pattern #63)The variable overhead efficiency variance for November was

$48,000 unfavorable.A.$60,000 favorable.B.$96,000 unfavorable.C.$200,000 unfavorable.D.

Answer (A) is correct. The variable overhead efficiency variance equals the standard price ($8 an hour) times thedifference between the actual hours and the standard hours allowed for the actual output. Thus, the variance is$48,000 {$8 × [94,000 actual hours – (4 standard hours per unit × 22,000 units produced)]}. The variance isunfavorable because actual hours exceeded standard hours.Answer (B) is incorrect because $60,000 favorable is the variable overhead spending variance calculated based oncapacity, not actual hours.Answer (C) is incorrect because $96,000 unfavorable is based on the difference between standard hours allowed forthe actual output and capacity hours.Answer (D) is incorrect because $200,000 unfavorable is the excess of actual direct labor costs over actual variableoverhead costs.

[775] Gleim #: 7.36 -- Source: CMA 1295 3-9

Individual budget schedules are prepared to develop an annual comprehensive or master budget. The budget schedule thatwould provide the necessary input data for the direct labor budget would be the

Gleim's CIA Test Prep: Part III: Business Analysis and Information Technology(1165 questions)

Copyright 2004 Gleim Publications, Inc. Page 314Printed for g j

Page 315: Part three cia_with_ answers

Sales forecast.A.Raw materials purchases budget.B.Schedule of cash receipts and disbursements.C.Production budget.D.

Answer (A) is incorrect because the sales forecast is insufficient for completion of the direct labor budget.Answer (B) is incorrect because the raw material purchases budget is not needed to prepare a direct labor budget.Answer (C) is incorrect because the schedule of cash receipts and disbursements cannot be prepared until after the directlabor budget has been completed.Answer (D) is correct. A master budget typically begins with the preparation of a sales budget. The next step is to prepare aproduction budget. Once the production budget has been completed, the next step is to prepare the direct labor, rawmaterial, and overhead budgets. Thus, the production budget provides the input necessary for the completion of the directlabor budget.

[Fact Pattern #64]

The Raymar Company is preparing its cash budget for themonths of April and May. The firm has established a$200,000 line of credit with its bank at a 12% annual rateof interest on which borrowings for cash deficits must bemade in $10,000 increments. There is no outstandingbalance on the line of credit loan on April 1. Principalrepayments are to be made in any month in which there is asurplus of cash. Interest is to be paid monthly. If there areno outstanding balances on the loans, Raymar will investany cash in excess of its desired end-of-month cashbalance in U.S. Treasury bills. Raymar intends to maintaina minimum balance of $100,000 at the end of each monthby either borrowing for deficits below the minimumbalance or investing any excess cash. Expected monthlycollection and disbursement patterns are shown in thecolumns to the right.

Collections.  50% of the current month’s sales budget and50% of the previous month’s sales budget.Accounts Payable Disbursements.  75% of the currentmonth’s accounts payable budget and 25% of theprevious month’s accounts payable budget.All other disbursements occur in the month in which theyare budgeted

Budget Information

March April MaySales $40,000 $50,000 $100,000Accounts payable 30,000 40,000 40,000Payroll 60,000 70,000 50,000Other disbursements 25,000 30,000 10,000

[776] Gleim #: 7.37 -- Source: CMA 1293 3-19

(Refers to Fact Pattern #64)In April, Raymar’s budget will result in

$45,000 in excess cash.A.A need to borrow $50,000 on its line of credit for the cash deficit.B.A need to borrow $100,000 on its line of credit for the cash deficit.C.A need to borrow $90,000 on its line of credit for the cash deficit.D.

Answer (A) is incorrect because $45,000 equals cash receipts.Answer (B) is incorrect because the cash deficit will be $92,500 without borrowing.Answer (C) is correct. Assuming Raymar maintained a $100,000 cash balance at the end of March, the amount to beborrowed or invested in April is the difference between cash receipts and disbursements. April’s cash collections are$45,000 [(50% × $50,000 April sales) + (50% × $40,000 March sales)]. Disbursements for accounts payable are$37,500 [(75% × $40,000 April payables) + (25% × $30,000 March payables)]. In addition to the accounts payabledisbursements, payroll and other disbursements will require an additional $100,000. Hence, total disbursements areestimated to be $137,500. The net negative cash flow (amount to be borrowed to reach the required minimum cashbalance of $100,000) is $92,500 ($137,500 – $45,000). Because the line of credit must be drawn upon in $10,000increments, the loan must be for $100,000.Answer (D) is incorrect because a loan of only $90,000 would still leave a negative cash balance of $2,500.

[777] Gleim #: 7.38 -- Source: CMA 1293 3-20

(Refers to Fact Pattern #64)In May, Raymar will be required to

Gleim's CIA Test Prep: Part III: Business Analysis and Information Technology(1165 questions)

Copyright 2004 Gleim Publications, Inc. Page 315Printed for g j

Page 316: Part three cia_with_ answers

Repay $20,000 principal and pay $1,000 interest.A.Repay $90,000 principal and pay $100 interest.B.Pay $900 interest.C.Borrow an additional $20,000 and pay $1,000 interest.D.

Answer (A) is incorrect because no funds are available to repay the loan. May receipts are less than May disbursements.Answer (B) is incorrect because the 1% interest is calculated on a $100,000 loan, not a $90,000 loan.Answer (C) is incorrect because the 1% interest is calculated on a $100,000 loan, not a $90,000 loan.Answer (D) is correct. The company will have to borrow $100,000 in April, which means that interest will have to be paidin May at the rate of 1% per month (12% annual rate). Consequently, interest expense is $1,000 (1% × $100,000). Mayreceipts are $75,000 [(50% × $100,000 May sales) + (50% × $50,000 April sales)]. Disbursements in May are $40,000[(75% × $40,000 May payables) + (25% × $40,000 April payables)]. In addition to the May accounts payabledisbursements, payroll and other disbursements are $60,000, bringing total disbursements to $101,000 ($60,000 + $40,000+ $1,000). Thus, disbursements exceed receipts by $26,000 ($101,000 – $75,000). However, cash has a beginning surplusbalance of $7,500 ($100,000 April loan – $92,500 negative cash flow for April). As a result, the company needs to borrowan additional $18,500 to eliminate its cash deficit. Given the requirement that loans be in $10,000 increments, the May loanmust be for $20,000.

[778] Gleim #: 7.39 -- Source: CMA 1294 3-13

When sales volume is seasonal in nature, certain items in the budget must be coordinated. The three most significant itemsto coordinate in budgeting seasonal sales volume are

Production volume, finished goods inventory, and sales volume.A.Direct labor hours, work-in-process inventory, and sales volume.B.Raw material inventory, direct labor hours, and manufacturing overhead costs.C.Raw material inventory, work-in-process inventory, and production volume.D.

Answer (A) is correct. The most difficult items to coordinate in any budget, particularly for a seasonal business, areproduction volume, finished goods inventory, and sales. Budgets usually begin with sales volume and proceed toproduction volume, but the reverse is sometimes used when production is more of an issue than generation of sales.Inventory levels are also important because sales cannot occur without inventory, and the maintenance of highinventory levels is costly.Answer (B) is incorrect because direct labor hours and work-in-process are only two components of a productionbudget.Answer (C) is incorrect because sales is usually the most important aspect of any budget.Answer (D) is incorrect because sales is usually the most important aspect of any budget.

[779] Gleim #: 7.40 -- Source: CMA 1292 3-11

Butteco has the following cost components for 100,000 units of product for 1998:

Raw materials $200,000Direct labor 100,000Manufacturing overhead 200,000Selling/administrative expense 150,000

All costs are variable except for $100,000 of manufacturing overhead and $100,000 of selling and administrative expenses.The total costs to produce and sell 110,000 units during 1998 are

$650,000A.$715,000B.$695,000C.$540,000D.

Gleim's CIA Test Prep: Part III: Business Analysis and Information Technology(1165 questions)

Copyright 2004 Gleim Publications, Inc. Page 316Printed for g j

Page 317: Part three cia_with_ answers

Answer (A) is incorrect because $650,000 is the cost at a production level of 100,000 units.Answer (B) is incorrect because $715,000 assumes a variable unit cost of $6.50 with no fixed costs.Answer (C) is correct. Raw materials unit costs are strictly variable at $2 ($200,000 ÷ 100,000 units). Similarly, directlabor has a variable unit cost of $1 ($100,000 ÷ 100,000 units). The $200,000 of manufacturing overhead for 100,000 unitsis 50%. The variable unit cost is $1. Selling costs are $100,000 fixed and $50,000 variable for production of 100,000 units,and the variable unit selling expense is $.50 ($50,000 ÷ 100,000 units). The total unit variable cost is therefore $4.50 ($2 +$1 + $1 + $.50). Fixed costs are $200,000. At a production level of 110,000 units, variable costs are $495,000 ($4.50 ×110,000 units). Hence, total costs are $695,000 ($495,000 + $200,000).Answer (D) is incorrect because total costs are $695,000 based on a unit variable cost of $4.50 each.

[780] Gleim #: 7.41 -- Source: CMA 1292 3-12

Barnes Corporation expected to sell 150,000 board games during the month of November, and the company’s masterbudget contained the following data related to the sale and production of these games:

Revenue $2,400,000Cost of goods sold

Direct materials 675,000Direct labor 300,000Variable overhead 450,000

 Contribution $   975,000

Fixed overhead 250,000Fixed selling/administration 500,000 

Operating income $   225,000 

Actual sales during November were 180,000 games. Using a flexible budget, the company expects the operating income forthe month of November to be

$225,000A.$270,000B.$420,000C.$510,000D.

Answer (A) is incorrect because $225,000 is the net income before the increase in sales.Answer (B) is incorrect because net income was originally $1.50 per game. The $270,000 figure simply extrapolatesthat amount to sales of 180,000 games.Answer (C) is correct. Revenue of $2,400,000 reflects a unit selling price of $16 ($2,400,000 ÷ 150,000 games). Thecontribution margin is $975,000, or $6.50 per game ($975,000 ÷ 150,000 games). Thus, unit variable cost is $9.50($16 – $6.50). Increasing sales will result in an increased contribution margin of $195,000 (30,000 × $6.50).Assuming no additional fixed costs, net income will increase to $420,000 ($225,000 originally reported + $195,000).Answer (D) is incorrect because $510,000 assumes that variable overhead is fixed. Variable overhead is a $3component ($450,000 ÷ 150,000 units) of unit variable cost.

[781] Gleim #: 7.42 -- Source: CIA 595 III-20

The major feature of zero-base budgeting (ZBB) is that it

Takes the previous year’s budgets and adjusts them for inflation.A.Questions each activity and determines whether it should be maintained as it is, reduced, or eliminated.B.Assumes all activities are legitimate and worthy of receiving budget increases to cover any increased costs.C.Focuses on planned capital outlays for property, plant, and equipment.D.

Gleim's CIA Test Prep: Part III: Business Analysis and Information Technology(1165 questions)

Copyright 2004 Gleim Publications, Inc. Page 317Printed for g j

Page 318: Part three cia_with_ answers

Answer (A) is incorrect because traditional or incremental budgeting takes the previous year’s budgets and adjusts them forinflation.Answer (B) is correct. ZBB is a planning process in which each manager must justify his/her department’s full budget foreach period. The purpose is to encourage periodic reexamination of all costs in the hope that some can be reduced oreliminated.Answer (C) is incorrect because traditional or incremental budgeting assumes all activities are legitimate and worthy ofreceiving budget increases to cover any increased costs.Answer (D) is incorrect because it is a definition of a capital budget.

[Fact Pattern #65]A firm with an 18% cost of capital is considering the following projects (on January 1, 2001):

  January 1, 2001 December 31, 2005  Cash Outflow Cash Inflow Project Internal  (000’s Omitted) (000’s Omitted) Rate of Return Project A $3,500 $7,400 16%Project B 4,000 9,950 ?

Present Value of $1 Due at the End of “N” Periods               

N 12% 14% 15% 16% 18% 20% 22%4 .6355 .5921 .5718 .5523 .5158 .4823 .42305 .5674 .5194 .4972 .4761 .4371 .4019 .34116 .5066 .4556 .4323 .4104 .3704 .3349 .2751

[782] Gleim #: 7.43 -- Source: CIA 597 IV-40

(Refers to Fact Pattern #65)Using the net-present-value (NPV) method, project A’s net present value is

$316,920A.$23,140B.$(265,460)C.$(316,920)D.

Answer (A) is incorrect because $316,920 discounts the cash inflow over a 4-year period.Answer (B) is incorrect because $23,140 assumes a 16% discount rate.Answer (C) is correct. The cash inflow occurs 5 years after the cash outflow, and the NPV method uses the firm’scost of capital of 18%. The present value of $1 due at the end of 5 years discounted at 18% is .4371. Thus, the NPVof project A is $(265,460) [(.4371 × $7,400,000 cash inflow) – $3,500,000 cash outflow].Answer (D) is incorrect because $(316,920) discounts the cash inflow over a 4-year period and also subtracts thepresent value of the cash inflow from the cash outflow.

[783] Gleim #: 7.44 -- Source: CIA 597 IV-41

(Refers to Fact Pattern #65)Project B’s internal rate of return is closest to

15%A.16%B.18%C.20%D.

Gleim's CIA Test Prep: Part III: Business Analysis and Information Technology(1165 questions)

Copyright 2004 Gleim Publications, Inc. Page 318Printed for g j

Page 319: Part three cia_with_ answers

Answer (A) is incorrect because 15% results in a positive NPV for project B.Answer (B) is incorrect because 16% is the approximate internal rate of return for project A.Answer (C) is incorrect because 18% is the company’s cost of capital.Answer (D) is correct. The internal rate of return is the discount rate at which the NPV is zero. Consequently, the cashoutflow equals the present value of the inflow at the internal rate of return. The present value of $1 factor for project B’sinternal rate of return is therefore .4020 ($4,000,000 cash outflow ÷ $9,950,000 cash inflow). This factor is closest to thepresent value of $1 for 5 periods at 20%.

[784] Gleim #: 7.45 -- Source: CIA 595 IV-37

Everything else being equal, the internal rate of return (IRR) of an investment project will be lower if

The investment cost is lower.A.Cash inflows are received later in the life of the project.B.Cash inflows are larger.C.The project has a shorter payback period.D.

Answer (A) is incorrect because the present value of the cash inflows is inversely related to the discount rate; that is,if the discount rate is higher, the present value of the cash inflows is lower. If the investment cost is lower, a higherdiscount rate (the IRR) will be required to set the net present value equal to zero.Answer (B) is correct. The IRR is the discount rate at which the net present value is zero. Because the present valueof a dollar is higher the sooner it is received, projects with later cash flows will have lower net present values for anygiven discount rate than will projects with earlier cash flows, if other factors are constant. Hence, projects with latercash flows will have a lower IRR.Answer (C) is incorrect because the larger the cash inflows, the higher the IRR will be. Higher cash inflows have ahigher present value at any given discount rate. A higher discount rate will be required to set the net present valueequal to zero.Answer (D) is incorrect because projects with shorter payback periods have higher cash inflows early in the life of theproject. Projects with earlier cash inflows have higher IRRs.

[785] Gleim #: 7.46 -- Source: CIA 596 III-80

A manufacturing company’s primary goals include product quality and customer satisfaction. The company sells a product,for which the market demand is strong, for $50 per unit. Due to the capacity constraints in the Production Department, only300,000 units can be produced per year. The current defective rate is 12% (i.e., of the 300,000 units produced, only264,000 units are sold and 36,000 units are scrapped). There is no revenue recovery when defective units are scrapped. Thefull manufacturing cost of a unit is $29.50, including

Direct materials $17.50Direct labor 4.00Fixed manufacturing overhead 8.00

The company’s designers have estimated that the defective rate can be reduced to 2% by using a different direct material.However, this will increase the direct materials cost by $2.50 per unit to $20 per unit. The net benefit of using the newmaterial to manufacture the product will be

$(120,000)A.$120,000B.$750,000C.$1,425,000D.

Answer (A) is incorrect because $(120,000) considers only the production costs of the good units sold. Moreover, itincludes fixed overhead, a cost that is not affected by the choice of materials.Answer (B) is incorrect because $120,000 considers only the variable costs of the good units produced.Answer (C) is correct. If a different direct material is used, incremental revenue will be $1,500,000 {[(12% defectrate – 2%) × 300,000 units] × $50}. Incremental cost will be $750,000 ($2.50 × 300,000 units). Thus, the net benefitwill be $750,000 ($1,500,000 – $750,000).Answer (D) is incorrect because $1,425,000 includes only the incremental direct materials cost of the increase in thenumber of good units produced.

Gleim's CIA Test Prep: Part III: Business Analysis and Information Technology(1165 questions)

Copyright 2004 Gleim Publications, Inc. Page 319Printed for g j

Page 320: Part three cia_with_ answers

[Fact Pattern #66]The segmented income statement for a retail company with three product lines is presented below:

Total Company Product Line 1 Product Line 2 Product Line 3Volume (in units) 20,000 28,000 50,000Sales revenue $2,000,000 $800,000 $700,000 $500,000Costs & expenses:

Administrative $   180,000 $  60,000 $  60,000 $  60,000Advertising 240,000 96,000 84,000 60,000Commissions 40,000 16,000 14,000 10,000Cost of sales 980,000 360,000 420,000 200,000Rent 280,000 84,000 140,000 56,000Salaries 110,000 54,000 32,000 24,000

Total costs & expenses $1,830,000 $670,000 $750,000 $410,000Operating income (loss) $   170,000 $130,000 $ (50,000) $  90,000

The company buys the goods in the three product lines directly from manufacturers’ representatives. Each product line is directedby a manager whose salary is included in the administrative expenses. Administrative expenses are allocated to the three productlines equally because the administration is spread evenly among the three product lines. Salaries represent payments to theworkers in each product line and therefore are traceable costs of each product line. Advertising promotes the entire companyrather than the individual product lines. As a result, the advertising is allocated to the three product lines in proportion to the salesrevenue. Commissions are paid to the salespersons in each product line based on 2% of gross sales. Rent represents the cost of theretail store and warehouse under a lease agreement with 5 years remaining. The product lines share the retail and warehousespace, and the rent is allocated to the three product lines based on the square footage occupied by each of the product lines.

[786] Gleim #: 7.47 -- Source: CIA 1196 III-98

(Refers to Fact Pattern #66)The company has an opportunity to promote one of its product lines by making a one-time $7,000 expenditure. Thecompany can choose only one of the three product lines to promote. The incremental sales revenue that would be realizedfrom this $7,000 promotion expenditure in each of the product lines is estimated as follows:

  Increase in  Sales Revenue Product Line 1 $15,000Product Line 2 20,000Product Line 3 14,000

In order to maximize profits, the promotion expenditure should be spent on <List A>, resulting in an increase in operatingincome of <List B>.

  List A List B A. Product Line 2 $13,000B. Product Line 2 $  5,000C. Product Line 3 $  1,400D. Product Line 3 $  1,120

Gleim's CIA Test Prep: Part III: Business Analysis and Information Technology(1165 questions)

Copyright 2004 Gleim Publications, Inc. Page 320Printed for g j

Page 321: Part three cia_with_ answers

Answer (A) is incorrect because Product Line 2 has an increased profit of $600.Answer (B) is incorrect because Product Line 2 has an increased profit of $600.Answer (C) is incorrect because $1,400 omits the commissions from the calculation.Answer (D) is correct. Fixed costs should be ignored. Thus, the increase in sales revenue should be multiplied by thecontribution margin ratio for each product line. The incremental promotion cost ($7,000) is subtracted from this amount todetermine the marginal benefit of promoting each product line. Hence, Product Line 3 has an increased profit of $1,120.

Calculation of Contribution Margin

  Total Product Product Product  Company Line 1 Line 2 Line 3 Sales revenue $2,000,000 $800,000 $700,000 $500,000 Variable costs

Commissions $     40,000 $  16,000 $  14,000 $  10,000Cost of sales 980,000 360,000 420,000 200,000

 Total $1,020,000 $376,000 $434,000 $210,000

 CM $   980,000 $424,000 $266,000 $290,000 CMR 49% 53% 38% 58% Revenue $  15,000 $  20,000 $  14,000CMR .53 .38 .58   CM $    7,950 $    7,600 $    8,120Promotion cost 7,000 7,000 7,000   Increased profits $      950 $      600 $    1,120   

[787] Gleim #: 7.48 -- Source: CIA 1196 III-99

(Refers to Fact Pattern #66)One company executive has expressed concern about the operating loss that has occurred in Product Line 2 and hassuggested that Product Line 2 be discontinued. If Product Line 2 is dropped, the manager of the line would be retained andassigned other duties with the company, but the other employees would not be retained. Management has indicated that thenature of the company’s advertising might change with the elimination of Product Line 2, but the total dollar amount wouldnot change. If Product Line 2 were to be dropped, the operating income of the company would

Increase by $50,000.A.Decrease by $94,000.B.Decrease by $234,000.C.Increase by $416,000.D.

Answer (A) is incorrect because an increase of $50,000 assumes the revenue will be lost and all of its costs will beavoided.Answer (B) is incorrect because a decrease of $94,000 results from treating rent as an avoidable cost.Answer (C) is correct. The operating income will decrease. Product Line 2 income will be lost, but only the traceablecosts of commissions, cost of sales, and salaries will be avoided. Accordingly, the decrease will be $234,000 [-$700,000 + ($14,000 + 420,000 + 32,000)]. The other shared costs will have to be absorbed by the two remainingproduct lines.Answer (D) is incorrect because an increase of $416,000 subtracts the costs that will not be avoided if Product Line 2is dropped from the lost sales revenue.

Gleim's CIA Test Prep: Part III: Business Analysis and Information Technology(1165 questions)

Copyright 2004 Gleim Publications, Inc. Page 321Printed for g j

Page 322: Part three cia_with_ answers

[788] Gleim #: 7.49 -- Source: CIA 1196 III-100(Refers to Fact Pattern #66)A customer, operating in an isolated foreign market, has approached the head salesperson for Product Line 1 and offered topurchase 4,000 units of a special-order product over the next 12 months. This product would be sold in the same manner asProduct Line 1’s other products except that the customer is hoping for a price break. Product Line 1’s cost to purchase thisproduct (cost of sales) would be $14.70. Product Line 1 has excess capacity, meaning that the rate or amount of theremaining operating costs would not change as a consequence of the purchase and sale of this special-order product. Theminimum selling price for this special-order product would be

$15.00A.$17.30B.$27.50C.$30.20D.

Answer (A) is correct. Product Line 1 needs to cover its variable out-of-pocket costs as a minimum on this special-order product; therefore, any selling price greater than the variable cost will contribute towards profits. Thus, theminimum selling price of the special-order product is the variable cost divided by 1 minus the commission rate, or$15 [$14.70 ÷ (1.0 – .02)].Answer (B) is incorrect because $17.30 includes the average cost of salaries (at the new volume level of 24,000 units)as a cost that needs to be covered when determining the minimum selling price.Answer (C) is incorrect because $27.50 is calculated based on a full cost approach.Answer (D) is incorrect because $30.20 adds all costs and expenses (except cost of sales) and divides them by theoriginal volume level of 20,000 units to determine the average operating costs. The new cost of sales is added to theaverage operating costs to determine the minimum selling price.

[789] Gleim #: 7.50 -- Source: CIA 593 IV-19

The ABC Company manufactures components for use in producing one of its finished products. When 12,000 units areproduced, the full cost per unit is $35, separated as follows:

Direct materials $  5Direct labor 15Variable overhead 10Fixed overhead 5

The XYZ Company has offered to sell 12,000 components to ABC for $37 each. If ABC accepts the offer, some of thefacilities currently being used to manufacture the components can be rented as warehouse space for $40,000. However, $3of the fixed overhead currently applied to each component would have to be covered by ABC’s other products. What is thedifferential cost to the ABC Company of purchasing the components from the XYZ Company?

$8,000A.$20,000B.$24,000C.$44,000D.

Answer (A) is incorrect because $8,000 assumes that $3 of fixed overhead is avoidable.Answer (B) is correct. Differential (incremental) cost is the difference in total cost between two decisions. Therelevant costs do not include unavoidable costs, such as the $3 of fixed overhead. It would cost ABC an additional$20,000 to purchase, rather than manufacture, the components.

Cost to purchase ($37 × 12,000) $444,000Minus rental income (40,000)   $404,000Cost to manufacture ($32 × 12,000) $384,000 Cost differential $  20,000 

Answer (C) is incorrect because $24,000 compares the full cost of manufacturing with cost to purchase.Answer (D) is incorrect because $44,000 ignores the opportunity cost.

Gleim's CIA Test Prep: Part III: Business Analysis and Information Technology(1165 questions)

Copyright 2004 Gleim Publications, Inc. Page 322Printed for g j

Page 323: Part three cia_with_ answers

[Fact Pattern #67]

A business needs a computer application that can be eitherdeveloped internally or purchased. Suitable software from avendor costs $29,000. Minor modifications and testing canbe conducted by the systems staff as part of their regularworkload.

If the software is developed internally, a systems analystwould be assigned full time, and a contractor would assumethe analyst’s responsibilities. The hourly rate for the regularanalyst is $25. The hourly rate for the contractor is $22. Thecontractor would occupy an empty office. The office has100 square feet, and occupancy cost is $45 per square foot.

Other related data are as follows. Computer time is chargedusing predetermined rates. The organization has sufficientexcess computer capacity for either software development ormodification/testing of the purchased software.

InternalDevelopment

PurchasedSoftware

Systems analyst time in hours

Development 1,000 N/A

Modifications and testing N/A 40

Computer charges $800 $250

Additional hardware purchases $3,200 N/A

Incidental supplies $500 $200

[790] Gleim #: 7.51 -- Source: CIA 1194 III-56

(Refers to Fact Pattern #67)When applying the cost-benefit approach to a decision, the primary criterion is how well management goals will beachieved in relation to costs. Costs include all expected

Variable costs for the courses of action but not expected fixed costs because only the expected variable costs arerelevant.

A.

Incremental out-of-pocket costs as well as all expected continuing costs that are common to all the alternative coursesof action.

B.

Future costs that differ among the alternative courses of action plus all qualitative factors that cannot be measured innumerical terms.

C.

Historical and future costs relative to the courses of action including all qualitative factors that cannot be measured innumerical terms.

D.

Answer (A) is incorrect because variable and fixed costs may be relevant or irrelevant.Answer (B) is incorrect because expected incremental out-of-pocket expenses should be considered, but commoncosts should not.Answer (C) is correct. The analysis of a make-or-buy decision is based on relevant costs. If costs do not vary withthe option chosen, they are irrelevant. Moreover, the decision may be based on nonquantitative factors, for example,the desire to maintain a relationship with a vendor or to assume control over development of a product.Answer (D) is incorrect because historical costs are not relevant to cost-benefit analysis because they are sunk costs.

[791] Gleim #: 7.52 -- Source: CIA 1194 III-57

(Refers to Fact Pattern #67)Based solely on the cost figures presented, the cost of developing the computer application will be

$3,500 less than acquiring the purchased software package.A.$500 less than acquiring the purchased software package.B.$1,550 more than acquiring the purchased software package.C.$3,550 more than acquiring the purchased software package.D.

Answer (A) is correct. Development cost equals the cost of the outside contractor plus the costs for hardware andsupplies. Computer charges are transfer prices and do not require additional expenditures, given idle capacity. Therelevant cost of supplies is $300 ($500 – $200 cost if the software is purchased). The contractor’s use of an otherwiseidle office is not relevant. Thus, the relevant cost of development is $25,500 [($22 hourly cost of the contractor ×1,000 hours) + $3,200 hardware purchases + $300 incremental cost of supplies]. This amount is $3,500 less than the$29,000 cost of purchase. A systems analyst’s work on the new software is not relevant. It is part of the regularworkload.Answer (B) is incorrect because the contractor is not paid $25 per hour.Answer (C) is incorrect because $550 in computer charges and $4,500 in occupancy charges should not be included.Answer (D) is incorrect because the contractor is not paid $25 per hour, and 40 hours of modification and testing,$550 of the computer charges, and the occupancy costs are irrelevant.

Gleim's CIA Test Prep: Part III: Business Analysis and Information Technology(1165 questions)

Copyright 2004 Gleim Publications, Inc. Page 323Printed for g j

Page 324: Part three cia_with_ answers

[792] Gleim #: 7.53 -- Source: CIA 592 IV-10

A company manufactures a product that is sold for $37.95. It uses an absorption-cost system. Plant capacity is 750,000units annually, but normal volume is 500,000 units. Costs at normal volume are given below.

Unit Cost Total Cost

Direct materials $  9.80 $4,900,000

Direct labor 4.50 2,250,000

Manufacturing overhead 12.00 6,000,000

Selling andadministrative:

Variable

2.50 1,250,000

Fixed 4.20 2,100,000

Total cost $33.00 $16,500,000

Fixed manufacturing overhead is budgeted at $4.5 million. A customer has offered to purchase 100,000 units at $25.00each to be packaged in large cartons, not the normal individual containers. It will pick up the units in its own trucks. Thus,variable selling and administrative expenses will decrease by 60%. The company should compare the total revenue to bederived from this order with the total relevant costs of

$1,830,000A.$1,880,000B.$2,930,000C.$3,150,000D.

Answer (A) is correct. The necessary assumptions are that all fixed costs and the unit variable costs of directmaterials, direct labor, and variable manufacturing overhead are not affected by the special order. Hence, the fixedcosts are not relevant. The unit costs of direct materials and direct labor are given as $9.80 and $4.50, respectively.The unit variable manufacturing overhead cost is $3.00 [($6,000,000 total manufacturing overhead – $4,500,000 totalfixed manufacturing overhead) ÷ 500,000 units normal volume]. The unit variable selling and administrative cost is$1.00 [$2.50 × (1.0 – .6)]. Consequently, the total relevant cost of the special order is $1,830,000 [100,000 units ×($9.80 + $4.50 + $3.00 + $1.00)].Answer (B) is incorrect because variable manufacturing per unit is determined by using normal volume, not plantcapacity, as the denominator level.Answer (C) is incorrect because the total relevant cost of the special order is $1,830,000 [100,000 units × ($9.80 +$4.50 + $3.00 + $1.00)].Answer (D) is incorrect because fixed selling and administrative expenses of $4.20 per unit should not be included.Furthermore, variable manufacturing of $3 per unit, not total manufacturing overhead of $12 per unit, should be usedin the calculation of relevant costs.

[793] Gleim #: 7.54 -- Source: CIA 1195 III-96

A large manufacturing company has several autonomous divisions that sell their products in perfectly competitive externalmarkets as well as internally to the other divisions of the company. Top management expects each of its divisionalmanagers to take actions that will maximize the organization’s goals as well as their own goals. Top management alsopromotes a sustained level of management effort of all of its divisional managers. Under these circumstances, for productsexchanged between divisions, the transfer price that will generally lead to optimal decisions for the manufacturing companywould be a transfer price equal to the

Full cost of the product.A.Full cost of the product plus a markup.B.Variable cost of the product plus a markup.C.Market price of the product.D.

Gleim's CIA Test Prep: Part III: Business Analysis and Information Technology(1165 questions)

Copyright 2004 Gleim Publications, Inc. Page 324Printed for g j

Page 325: Part three cia_with_ answers

Answer (A) is incorrect because a transfer at full cost means that the selling division will not make a profit. In addition, theselling division may be forgoing profits that could be obtained by selling to outside customers. Thus, full-cost transferprices can lead to suboptimal decisions.Answer (B) is incorrect because a transfer at full cost plus markup results in no incentive for the selling division to controlits costs. Hence, a sustained level of management effort may not be maintained.Answer (C) is incorrect because a transfer at variable cost plus markup has the same weaknesses as full cost plus markup.Answer (D) is correct. A market price transfer price promotes goal congruence and a sustained level of management effort.It is also consistent with divisional autonomy. A market transfer price is most appropriate when the market is competitive,interdivisional dependency is low, and buying in the market involves no marginal costs or benefits.

[794] Gleim #: 7.55 -- Source: CIA 1193 IV-19

The Eastern division sells goods internally to the Western division of the same company. The quoted external price inindustry publications from a supplier near Eastern is $200 per ton plus transportation. It costs $20 per ton to transport thegoods to Western. Eastern’s actual market cost per ton to buy the direct materials to make the transferred product is $100.Actual per ton direct labor is $50. Other actual costs of storage and handling are $40. The company president selects a$220 transfer price. This is an example of

Market-based transfer pricing.A.Cost-based transfer pricing.B.Negotiated transfer pricing.C.Cost-plus-20% transfer pricing.D.

Answer (A) is correct. A transfer price is the price charged by one segment of an organization for a product orservice supplied to another segment of the same organization. The three basic criteria that the transfer pricing systemin a decentralized company should satisfy are to (1) provide information allowing central management to evaluatedivisions with respect to total company profit and each division’s contribution to profit, (2) stimulate each manager’sefficiency without losing each division’s autonomy, and (3) motivate each divisional manager to achieve his/her ownprofit goal in a manner contributing to the company’s success. Because the $220 transfer price selected is based onthe quoted external price (market), it is an example of market-based transfer pricing.Answer (B) is incorrect because the cost-based price would be $210 ($100 + $50 + $40 + $20).Answer (C) is incorrect because no negotiations took place.Answer (D) is incorrect because cost plus 20% would be $252 ($210 × 1.20).

[795] Gleim #: 7.56 -- Source: CIA 589 IV-16

Division A of a company is currently operating at 50% capacity. It produces a single product and sells all its production tooutside customers for $13 per unit. Variable costs are $7 per unit, and fixed costs are $6 per unit at the current productionlevel. Division B, which currently purchases this product from an outside supplier for $12 per unit, would like to purchasethe product from Division A. Division A will operate at 80% capacity to meet outside customers’ and Division B’sdemand. What is the minimum price that Division A should charge Division B for this product?

$7.00 per unit.A.$10.40 per unit.B.$12.00 per unit.C.$13.00 per unit.D.

Answer (A) is correct. From the seller’s perspective, the price should reflect at least its incremental cash outflow(outlay cost) plus the contribution from an outside sale (opportunity cost). Because A has idle capacity, theopportunity cost is $0. Thus, the minimum price Division A should charge Division B is $7.00.Answer (B) is incorrect because $7.00 is the minimum that should be charged.Answer (C) is incorrect because Division A should not include any fixed costs in its transfer price because Division Ahas idle capacity.Answer (D) is incorrect because, since Division A has idle capacity, the minimum transfer price should recoverDivision A’s variable (outlay) costs.

[796] Gleim #: 7.57 -- Source: CMA 1291 3-10

A segment of an organization is referred to as a service center if it has

Gleim's CIA Test Prep: Part III: Business Analysis and Information Technology(1165 questions)

Copyright 2004 Gleim Publications, Inc. Page 325Printed for g j

Page 326: Part three cia_with_ answers

Responsibility for developing markets and selling the output of the organization.A.Responsibility for combining the raw materials, direct labor, and other factors of production into a final output.B.Authority to make decisions affecting the major determinants of profit including the power to choose its markets andsources of supply.

C.

Authority to provide specialized support to other units within the organization.D.

Answer (A) is incorrect because a service center has no responsibility for developing markets or selling.Answer (B) is incorrect because a production center is engaged in manufacturing.Answer (C) is incorrect because a profit center can choose its markets and sources of supply.Answer (D) is correct. A service center exists primarily and sometimes solely to provide specialized support to other unitswithin the organization. Service centers are usually operated as cost centers.

[797] Gleim #: 7.58 -- Source: CIA 595 III-96

Which of the following techniques would be best for evaluating the management performance of a department that isoperated as a cost center?

Return on assets ratio.A.Return on investment ratio.B.Payback method.C.Variance analysis.D.

Answer (A) is incorrect because return on assets cannot be computed for a cost center. The manager is notresponsible for revenue (return) or the assets available.Answer (B) is incorrect because return on investments cannot be computed for a cost center. The manager is notresponsible for revenue (return) or the assets available.Answer (C) is incorrect because the payback method is a means of evaluating alternative investment proposals.Answer (D) is correct. A cost center is a responsibility center that is responsible for costs only. Of the alternativesgiven, variance analysis is the only one that can be used in a cost center. Variance analysis involves comparing actualcosts with predicted or standard costs.

[798] Gleim #: 7.59 -- Source: CMA 694 3-24

Decentralized firms can delegate authority and yet retain control and monitor managers’ performance by structuring theorganization into responsibility centers. Which one of the following organizational segments is most like an independentbusiness?

Revenue center.A.Profit center.B.Cost center.C.Investment center.D.

Answer (A) is incorrect because a revenue center is responsible only for revenue generation, not for costs or capitalinvestment.Answer (B) is incorrect because a profit center is responsible for revenues and costs but not for invested capital.Answer (C) is incorrect because a cost center is evaluated only on the basis of costs incurred. It is not responsible forrevenues or invested capital.Answer (D) is correct. An investment center is the organizational type most like an independent business because itis responsible for its own revenues, costs incurred, and capital invested. The other types of centers do not incorporateall three elements.

Gleim's CIA Test Prep: Part III: Business Analysis and Information Technology(1165 questions)

Copyright 2004 Gleim Publications, Inc. Page 326Printed for g j

Page 327: Part three cia_with_ answers

[799] Gleim #: 7.60 -- Source: CIA 593 IV-5A company employs a process cost system using the first-in, first-out (FIFO) method. The product passes through bothDepartment 1 and Department 2 in order to be completed. Units enter Department 2 upon completion in Department 1.Additional direct materials are added in Department 2 when the units have reached the 25% stage of completion withrespect to conversion costs. Conversion costs are added proportionally in Department 2. The production activity inDepartment 2 for the current month was as follows:

Beginning work-in-process inventory (40%complete with respect to conversion costs) 15,000

Units transferred in from Department 1 80,000 

Units to account for 95,000 Units completed and transferred to finished

goods 85,000Ending work-in-process inventory (25%

complete with respect to conversion costs) 10,000 Units accounted for 95,000 

How many equivalent units for direct materials were added in Department 2 for the current month?

70,000 units.A.80,000 units.B.85,000 units.C.95,000 units.D.

Answer (A) is correct. Beginning inventory is 40% complete. Hence, direct materials have already been added.Ending inventory has not reached the 25% stage of completion, so direct materials have not yet been added to theseunits. Thus, the equivalent units for direct materials calculated on a FIFO basis are equal to the units started andcompleted in the current period (85,000 units completed – 15,000 units in BWIP = 70,000 units started andcompleted).Answer (B) is incorrect because 80,000 total units were transferred in from Department 1.Answer (C) is incorrect because 85,000 equals the equivalent units for direct materials calculated on a weighted-average basis.Answer (D) is incorrect because 95,000 equals the total units to be account for.

[800] Gleim #: 7.61 -- Source: CIA 1193 IV-3

A large manufacturing company has two service departments and two production departments. Each of the servicedepartments renders services to each other and to the two production departments. Which one of the following methodswould most accurately allocate the costs of the service departments to the production departments of this company?

The direct allocation method.A.The step-down allocation method.B.The reciprocal allocation method.C.The sequential allocation method.D.

Answer (A) is incorrect because the direct allocation method ignores any services that are rendered by one servicedepartment to another service department.Answer (B) is incorrect because the step-down allocation method allows for limited recognition of services renderedby service departments to other service departments.Answer (C) is correct. The reciprocal method uses simultaneous equations to allocate each service department’scosts. It allocates costs by explicitly including the mutual services rendered among all departments. When servicedepartments render services to each other, the use of the direct method or the step-down method would not betheoretically accurate. Accordingly, in such situations, the reciprocal method would result in the most accurateallocation.Answer (D) is incorrect because the sequential allocation method is another name for the step-down allocationmethod.

Gleim's CIA Test Prep: Part III: Business Analysis and Information Technology(1165 questions)

Copyright 2004 Gleim Publications, Inc. Page 327Printed for g j

Page 328: Part three cia_with_ answers

[801] Gleim #: 7.62 -- Source: CIA 592 IV-6A metal fabricating company uses a job-order cost system. The company expects to have small residual pieces of metalcuttings and shavings from all of its jobs. Although the metal pieces and shavings cannot be reused, they can be sold forscrap. The scrap metal is sold when a ton of scrap has been accumulated. The requisitions and the scrap recovery foraluminum during the current month are as follows.

Aluminum requisitions:100,000 lbs. at $1.50 per lb. $150,000

Aluminum scrap recovery: 800 lbs.

This amount of scrap is within normal allowances for the company’s operations. The market price for scrap aluminumfluctuates greatly and has ranged from $.25 to $.40 per pound during the last 12 months. The accumulated scrap aluminumwas sold last month for $.35 per pound.

The appropriate accounting treatment for the scrap aluminum recovered during the current month is to

Debit direct materials quantity variance for $1,200 (800 lbs. × $1.50 per lb.) and credit work-in-process inventorycontrol for $1,200, with postings to each job from which the scrap metal was recovered.

A.

Debit scrap inventory for $280 (800 lbs. × $.35 per lb.) and credit factory overhead control for $280.B.For materiality reasons, no entry is made until the scrap metal is sold. At that time, debit cash and credit factoryoverhead control for the quantity sold at the current market price.

C.

Debit direct materials quantity variance for $1,200 (800 lbs. × $1.50 per lb.) and credit factory overhead control for$1,200 at the time of recovery, and when the scrap is sold, debit cash and credit direct materials quantity variance forthe quantity sold at the current market price.

D.

Answer (A) is incorrect because a quantity variance is not recorded for scrap that is anticipated. Furthermore, work-in-process inventory is credited only when scrap is unique to a job.Answer (B) is incorrect because an accounting entry is not needed. The amount is not material.Answer (C) is correct. Making a memorandum entry at the time of recovery is appropriate. The value of the scrap isthen recognized at the time of sale. The factory overhead control account is credited because scrap is inevitable to thecompany’s production operations and not attributable to a specific job. This accounting method has the effect ofspreading the revenue from scrap sales over all jobs or products.Answer (D) is incorrect because normal scrap is not the basis for recording a variance.

[802] Gleim #: 7.63 -- Source: CIA 1193 IV-6

Some units of output failed to pass final inspection at the end of the manufacturing process. The production and inspectionsupervisors determined that the estimated incremental revenue from reworking the units exceeded the cost of rework. Therework of the defective units was authorized, and the following costs were incurred in reworking the units:

Materials requisitioned from stores:Direct materials $   5,000Miscellaneous supplies 300

Direct labor 14,000

The manufacturing overhead budget includes an allowance for rework. The predetermined manufacturing overhead rate is150% of direct labor cost. The account(s) to be charged and the appropriate charges for the rework cost would be

Work-in-process inventory control for $19,000.A.Work-in-process inventory control for $5,000 and factory overhead control for $35,300.B.Factory overhead control for $19,300.C.Factory overhead control for $40,300.D.

Gleim's CIA Test Prep: Part III: Business Analysis and Information Technology(1165 questions)

Copyright 2004 Gleim Publications, Inc. Page 328Printed for g j

Page 329: Part three cia_with_ answers

Answer (A) is incorrect because factory overhead should be charged for direct materials, supplies, direct labor, and appliedoverhead incurred for rework.Answer (B) is incorrect because factory overhead should be charged for direct materials, supplies, direct labor, and appliedoverhead incurred for rework.Answer (C) is incorrect because $19,300 excludes the predetermined manufacturing overhead.Answer (D) is correct. The rework charge for direct materials, indirect materials (supplies), direct labor, and overheadapplied on the basis of direct labor cost is $40,300 [$5,000 + $300 + $14,000 + (1.5 × $14,000)]. If an allowance forrework is included in a company’s manufacturing overhead budget, rework of defective units is spread over all jobs orbatches as part of the predetermined overhead application rate. Hence, the debit is to overhead control.

[803] Gleim #: 7.64 -- Source: CIA 1192 IV-10

A company uses a joint production process that produces three products at split-off. Joint production costs during Maywere $340,000. Product data for May were as follows:

  Product   X Y Z Tons produced 10,000 2,000 5,000Tons sold 8,000 1,700 4,300Selling price per ton $50 $125 $10       The company uses the net realizable value method for allocating joint costs.

Assume that product Z is treated as a by-product. Product X can be sold at split-off without further processing but productsY and Z have to be processed further before they can be sold. The separable cost of processing product Y (so it can be soldfor $165 per ton) is $30,000. The separable cost of product Z is $25,000. Product Z can be sold for $13 per ton after furtherprocessing is complete. What amount of the joint cost should be allocated to product Y?

$40,000A.$106,250B.$112,500C.$120,000D.

Gleim's CIA Test Prep: Part III: Business Analysis and Information Technology(1165 questions)

Copyright 2004 Gleim Publications, Inc. Page 329Printed for g j

Page 330: Part three cia_with_ answers

Answer (A) is incorrect because $40,000 is the NRV of by-product Z.Answer (B) is incorrect because $106,250 is the joint cost allocated to Y assuming that X, Y, and Z are joint products andno separable costs need to be incurred.Answer (C) is correct. By-products are joint products that have minor sales value compared with the sales value of the mainproduct(s). To be salable, the by-product may or may not require additional processing beyond the split-off point. Thus, theincurrence of separable costs beyond the split-off point may or may not be required. Practice with regard to recognizing by-products in the accounts is not uniform. The most cost-effective method for the initial recognition of by-products is toaccount for their value at the time of sale as a reduction in the joint cost or as a revenue. The alternative is to recognize thenet realizable value at the time of production, a method that results in the recording of by-product inventory. Assuming theNRV is recognized as a reduction in the joint cost, the joint cost allocated to Y is determined as follows:

Joint cost to be allocated to X and Y=  Joint cost – NRV of by-product Z=  $340,000 – [(5,000 tons × $13) – $25,000]=  $300,000

NRVX = 10,000 tons × $50 = $500,000

NRVY = (2,000 tons × $165) – $30,000 = $300,000

Joint cost allocated to Y

$300,000=  × 30,000

$300,000 + $500,000

=  $112,500

Answer (D) is incorrect because $120,000 is the joint cost of Y if X, Y, and Z are joint products, Y has separable costs of$30,000, and Y has a selling price of $165.

[804] Gleim #: 7.65 -- Source: CIA 593 IV-4

A new advertising agency serves a wide range of clients including manufacturers, restaurants, service businesses,department stores, and other retail establishments. The accounting system the advertising agency has most likely adoptedfor its record keeping in accumulating costs is

Job-order costing.A.Operating costing.B.Relevant costing.C.Process costing.D.

Answer (A) is correct. Job-order costing is used by organizations whose products or services are readily identified byindividual units or batches. The advertising agency accumulates its costs by client. Job-order costing is the mostappropriate system for this type of nonmanufacturing firm.Answer (B) is incorrect because operation costing would most likely be employed by a manufacturer producinggoods that have common characteristics plus some individual characteristics. This would not be an appropriatesystem for an advertising agency with such a diverse client base.Answer (C) is incorrect because relevant costing refers to expected future costs that are considered in decisionmaking.Answer (D) is incorrect because process costing is employed when a company mass produces a homogeneousproduct in a continuous fashion through a series of production steps.

Gleim's CIA Test Prep: Part III: Business Analysis and Information Technology(1165 questions)

Copyright 2004 Gleim Publications, Inc. Page 330Printed for g j

Page 331: Part three cia_with_ answers

[Fact Pattern #68]The standard direct labor cost to produce one pound of output for a company is presented below. Related data regarding theplanned and actual production activities for the current month for the company are also given below:

NOTE:  DLH = Direct labor hoursDirect labor standard:

0.4 DLH @ $12.00 per DLH = $4.80

Planned production 15,000 poundsActual prodcution 15,500 poundsActual direct labor costs

(6,250 DLH) $75,250

[805] Gleim #: 7.66 -- Source: CIA R98 IV-67

(Refers to Fact Pattern #68)The company’s direct labor rate variance for the current month is

$10 unfavorable.A.$240 unfavorable.B.$248 unfavorable.C.$250 unfavorable.D.

Answer (A) is incorrect because $10 unfavorable equals the difference between the planned and actual DLH times thedifference between the standard and actual rates.Answer (B) is incorrect because $240 unfavorable equals the planned DLH times the difference between the standardand actual rates.Answer (C) is incorrect because $248 unfavorable equals the DLH allowed for the actual output times the differencebetween the standard and actual rates.Answer (D) is correct. The direct labor rate variance equals actual DLH times the difference between the actual andstandard direct labor rates, or $250 unfavorable {6,250 actual DLH × [($75,250 actual costs ÷ 6,250 actual DLH) –$12.00 standard rate]}.

[806] Gleim #: 7.67 -- Source: CIA R98 IV-68

(Refers to Fact Pattern #68)The company’s direct labor efficiency variance for the current month is

$600 unfavorable.A.$602 unfavorable.B.$2,400 unfavorable.C.$3,000 unfavorable.D.

Answer (A) is correct. The direct labor efficiency variance equals the standard direct labor rate times the differencebetween the actual DLH and the standard DLH allowed for the actual output, or $600 unfavorable {$12 × [6,250actual DLH – (.4 DLH × 15,500 pounds actually produced)]}.Answer (B) is incorrect because $602 unfavorable is based on the actual direct labor rate.Answer (C) is incorrect because $2,400 unfavorable equals the standard direct labor rate times the difference betweenthe DLH allowed for the planned output and the DLH allowed for the actual output.Answer (D) is incorrect because $3,000 unfavorable equals the standard direct labor rate times the difference betweenthe DLH allowed for the planned output and the actual DLH.

[807] Gleim #: 7.68 -- Source: CIA 593 IV-11

A company has sales of $500,000, variable costs of $300,000 and pretax profit of $150,000. If the company increased thesales price per unit by 10%, reduced fixed costs by 20%, and left variable cost per unit unchanged, what would be the newbreakeven point in sales dollars?

Gleim's CIA Test Prep: Part III: Business Analysis and Information Technology(1165 questions)

Copyright 2004 Gleim Publications, Inc. Page 331Printed for g j

Page 332: Part three cia_with_ answers

$88,000A.$100,000B.$110,000C.$125,000D.

Answer (A) is correct. The breakeven point in sales dollars is equal to the sum of fixed cost plus any desired pretax profit,divided by contribution margin ratio [(sales – variable costs) ÷ sales]. Fixed cost was $50,000 ($500,000 sales – $300,000VC – $150,000 pretax profit). Given the increase in sales of 10% and decrease in fixed costs of 20%, the breakeven point insales is $88,000.

Fixed costs ($50,000 × .8) +Desired pretax profit 0 $40,000

=Sales [($500,000 × 1.1) – Variable costs $250,000 ÷ $550,000

$300,000] ÷ Sales ($500,000 × 1.1)

Answer (B) is incorrect because $100,000 ignores the 10% sales price increase.Answer (C) is incorrect because $110,000 ignores the 20% decrease in fixed costs.Answer (D) is incorrect because $125,000 ignores the changes in sales price and fixed costs.

[808] Gleim #: 7.69 -- Source: CIA R98 IV-66

Which of the following is a product cost for a manufacturing company?

Insurance on the corporate headquarters building.A.Property taxes on a factory.B.Depreciation on a salesperson’s automobile.C.The salary of a sales manager.D.

Answer (A) is incorrect because insurance on the corporate headquarters building is a general and administrativeexpense.Answer (B) is correct. Product (inventoriable) costs are incurred to produce units of output and are deferred to futureperiods to the extent output is not sold (kept on hand for sale in future periods). They are expensed in the period theproduct is sold. Hence, product costs are those that can be associated with specific revenues. Examples are directmaterials, direct labor, and factory (not general and administrative) overhead. Property taxes on a factory are includedin factory (manufacturing) overhead.Answer (C) is incorrect because depreciation on a salesperson’s automobile is a general and administrative expense.Answer (D) is incorrect because the salary of a sales manager is a general and administrative expense.

[Fact Pattern #69]A company manufactures and sells a single product. It takes two machine hours to produce one unit. Annual sales are expected tobe 75,000 units. Annual production capacity is 200,000 machine hours. Expected selling price is $10 per unit. Cost data formanufacturing and selling the product are as follows:

Variable Costs (per unit)Direct Materials $3.00Direct Labor 1.00Variable Manufacturing Overhead 0.80Variable Selling 2.00

Fixed Costs (per year)Fixed Manufacturing Overhead $90,000Fixed Selling 60,000

[809] Gleim #: 7.70 -- Source: CIA 595 III-85

(Refers to Fact Pattern #69)The company receives a special order for 10,000 units at $7.60. Variable selling cost for each of these 10,000 units will be$1.20. This special order will not affect regular sales of 75,000 units. If the company accepts this special order, its profitwill

Gleim's CIA Test Prep: Part III: Business Analysis and Information Technology(1165 questions)

Copyright 2004 Gleim Publications, Inc. Page 332Printed for g j

Page 333: Part three cia_with_ answers

Increase by $8,000.A.Increase by $16,000.B.Decrease by $4,000.C.Decrease by $12,000.D.

Answer (A) is incorrect because an increase of $8,000 assumes unit variable selling cost is $2.00.Answer (B) is correct. If the company accepts the special order, its revenue will increase by $76,000 ($7.60 × 10,000units). However, its incremental cost will include only the variable costs because fixed manufacturing and selling costs willbe unchanged. The increase in cost from accepting the special order is $60,000 [($3.00 + $1.00 + $0.80 + $1.20) × 10,000units]. Thus, acceptance of the special order will increase profits by $16,000 ($76,000 – $60,000).Answer (C) is incorrect because profit will increase.Answer (D) is incorrect because profit will increase.

[810] Gleim #: 7.71 -- Source: CIA 595 III-86

(Refers to Fact Pattern #69)The company estimates that by reducing its selling price to $9.30 per unit, it can increase sales to 90,000 units annually.Fixed costs per year and unit variable costs will remain unchanged. If the company reduces its selling price to $9.30 perunit, its profit will

Decrease by $5,000.A.Decrease by $15,000.B.Decrease by $45,000.C.Increase by $15,000.D.

Answer (A) is incorrect because profit will decrease by $15,000.Answer (B) is correct. Because total fixed costs are unaffected, the change in profit is the change in the contributionmargin. The contribution margin at the current selling price is $240,000 [($10 – $3 – $1 – $0.80 – $2) × 75,000units]. The contribution margin at the $9.30 selling price is $225,000 [($9.30 – $3 – $1 – $0.80 – $2) × 90,000units]. Hence, profit will be reduced by $15,000 ($240,000 – $225,000) if the selling price is lowered to $9.30.Answer (C) is incorrect because profit will decrease by $15,000.Answer (D) is incorrect because profit will decrease by $15,000.

[Fact Pattern #70]The power and maintenance departments of a manufacturing company are service departments that provide support to each otheras well as to the organization's two production departments, plating and assembly. The manufacturing company employs separatedepartmental manufacturing overhead rates for the two production departments requiring the allocation of the service departmentcosts to the two manufacturing departments. Square footage of area served is used to allocate the maintenance department costswhile percentage of power usage is used to allocate the power department costs. Department costs and operating data are asfollows:

Service Depts. Production Depts.Costs: Power Maintenance Plating AssemblyLabor $      60,000 $180,000Overhead 1,440,000 540,000Total Costs $1,500,000 $720,000

Operating Data:Square Feet 6,000 1,500 6,000 24,000Percent of Usage:

Long-run capacity 5% 60% 35%Expected actual use 4% 70% 26%

[811] Gleim #: 7.72 -- Source: CIA 1195 III-85

(Refers to Fact Pattern #70)The allocation method that would provide this manufacturer with the theoretically best allocation of service departmentcosts would be

Gleim's CIA Test Prep: Part III: Business Analysis and Information Technology(1165 questions)

Copyright 2004 Gleim Publications, Inc. Page 333Printed for g j

Page 334: Part three cia_with_ answers

A dual-rate allocation method allocating variable cost on expected actual usage and fixed costs on long-run capacity usage.A.The step-down allocation method.B.The direct allocation method.C.The reciprocal (or linear algebra) allocation method.D.

Answer (A) is incorrect because a dual-rate method is a refinement of the direct method permitting the allocation of thevariable and fixed costs in separate pools. Although this method is more precise than the direct method, it does notrecognize the mutual services rendered among all service departments.Answer (B) is incorrect because the step-down method provides for partial recognition of services rendered by other servicedepartments, but it does not recognize all mutual services.Answer (C) is incorrect because the direct method is not theoretically precise. It does not give any recognition to theservices rendered by other service departments.Answer (D) is correct. The reciprocal method is the theoretically most defensible allocation method. It recognizes themutual services rendered among all service departments.

[812] Gleim #: 7.73 -- Source: CIA 1195 III-86

(Refers to Fact Pattern #70)Assume that the manufacturing company employs the step-down allocation method to allocate service department costs. Ifit allocates the cost of the maintenance department first, then the amount of the maintenance department’s costs that aredirectly allocated to the plating department would be

$144,000A.$120,000B.$115,200C.$90,000D.

Answer (A) is incorrect because $144,000 is the allocation based on the direct method.Answer (B) is correct. The allocation base is 36,000 square feet [6,000 + 6,000 + 24,000]. Plating’s share is one-sixth [6,000 ÷ 36,000] of the total cost of maintenance services, or $120,000 ($720,000 ÷ 6).Answer (C) is incorrect because $115,200 is based on the total square footage of the entire plant.Answer (D) is incorrect because $90,000 results from allocating overhead costs only.

[813] Gleim #: 7.74 -- Source: CIA 1196 III-89

In a company, products pass through some or all of the production departments during manufacturing, depending upon theproduct being manufactured. Direct material and direct labor costs are traced directly to the products as they flow througheach production department. Manufacturing overhead is assigned in each department using separate departmentalmanufacturing overhead rates. The inventory costing method that the manufacturing company is using in this situation is

Absorption costing.A.Activity-based costing.B.Backflush costing.C.Variable costing.D.

Answer (A) is correct. Absorption costing inventories all direct manufacturing costs and both variable and fixedmanufacturing overhead (indirect) costs.Answer (B) is incorrect because activity-based costing develops cost pools for activities and then allocates those coststo cost objects based on the drivers of the activities.Answer (C) is incorrect because a backflush costing system applies costs based on output.Answer (D) is incorrect because variable costing excludes fixed manufacturing overhead costs from inventoriablecosts and treats them as period costs.

[814] Gleim #: 7.75 -- Source: CIA 596 IV-43

A company has $450,000 per year of fixed production costs, of which $150,000 are noncash outlays. The variable cost perunit is $15, and the unit selling price is $25. The breakeven volume in sales units for this company would be

Gleim's CIA Test Prep: Part III: Business Analysis and Information Technology(1165 questions)

Copyright 2004 Gleim Publications, Inc. Page 334Printed for g j

Page 335: Part three cia_with_ answers

18,000 units.A.30,000 units.B.45,000 units.C.60,000 units.D.

Answer (A) is incorrect because 18,000 units results from dividing fixed costs by unit price.Answer (B) is incorrect because 30,000 units equals fixed costs divided by unit variable cost.Answer (C) is correct. The breakeven volume in sales units equals fixed costs divided by the unit contribution margin (unitprice – unit variable cost). Hence, the breakeven point is 45,000 units [$450,000 ÷ ($25 – $15)].Answer (D) is incorrect because 60,000 units is calculated by adding $150,000 to the fixed costs.

[Fact Pattern #71]Data regarding 2001 operations for an enterprise that had no beginning or ending inventories are as follows:

Sales (150,000 units) $9,000,000Variable costs:

Direct materials $1,800,000Direct labor 720,000Manufacturing overhead 1,080,000Selling expenses 450,000

Fixed costs:Manufacturing overhead $   600,000Administrative expenses 567,840Selling expenses 352,800

Income tax rate 40%

The enterprise estimates that next year direct materials costs will increase by 10% and direct labor costs will increase by $0.60 perunit to $5.40 per unit. In addition, fixed selling expenses will increase by $29,520. All other costs will be incurred at the samerates or amounts as the current year.

[815] Gleim #: 7.76 -- Source: CIA R98 IV-57

(Refers to Fact Pattern #71)The total contribution margin for 2001 is

$2,970,000A.$4,950,000B.$5,400,000C.$6,030,000D.

Answer (A) is incorrect because $2,970,000 results from subtracting income tax.Answer (B) is correct. The total contribution margin equals sales minus variable costs, or $4,950,000 ($9,000,000 –$1,800,000 – $720,000 – $1,080,000 – $450,000).Answer (C) is incorrect because $5,400,000 omits variable selling costs from the calculation.Answer (D) is incorrect because $6,030,000 omits variable overhead from the calculation.

[816] Gleim #: 7.77 -- Source: CIA R98 IV-58

(Refers to Fact Pattern #71)The gross margin for 2001 is

$3,429,360A.$4,232,160B.$4,350,000C.$4,800,000D.

Gleim's CIA Test Prep: Part III: Business Analysis and Information Technology(1165 questions)

Copyright 2004 Gleim Publications, Inc. Page 335Printed for g j

Page 336: Part three cia_with_ answers

Answer (A) is incorrect because $3,429,360 equals pretax operating income (revenue minus total costs).Answer (B) is incorrect because $4,232,160 equals the gross margin minus fixed administrative expenses.Answer (C) is incorrect because $4,350,000 equals the gross margin minus the variable selling expenses.Answer (D) is correct. The gross margin equals revenue minus cost of goods sold (beginning finished goods inventory +cost of goods manufactured – ending finished goods inventory). Cost of goods manufactured equals all manufacturingadjusted for the change in work-in-process. Consequently, the gross margin was $4,800,000 ($9,000,000 – $1,800,000 DM– $720,000 DL – $1,080,000 VOH – $600,000 FOH), given no beginning or ending finished goods or work-in-processinventories.

[817] Gleim #: 7.78 -- Source: CIA R98 IV-59

(Refers to Fact Pattern #71)The breakeven point in unit sales for 2001 is

36,495 units.A.42,240 units.B.46,080 units.C.56,320 units.D.

Answer (A) is incorrect because 36,495 units includes fixed manufacturing overhead and excludes variable sellingexpenses from the UCM calculation. It also excludes fixed selling expenses from the total fixed costs.Answer (B) is incorrect because 42,240 units excludes variable selling expenses from the UCM calculation.Answer (C) is correct. The breakeven point in unit sales equals total fixed costs divided by the unit contributionmargin (UCM). Total fixed costs are $1,520,640 ($600,000 + $587,840 + $352,800), and the UCM is $33[$4,950,000 contribution margin (sales minus variable costs) ÷ 150,000 units sold]. Hence, the breakeven point inunit sales is 46,080 units ($1,520,640 ÷ $33).Answer (D) is incorrect because 56,320 units results from dividing total fixed costs by the total unit variable cost.

[818] Gleim #: 7.79 -- Source: CIA R98 IV-60

(Refers to Fact Pattern #71)What dollar sales volume, to the nearest dollar, would be required in 2002 to earn the same net income as in 2001?

$6,938,031A.$8,736,000B.$9,576,000C.$10,374,000D.

Answer (A) is incorrect because $6,938,031 includes net income instead of pretax operating income in thecalculation.Answer (B) is incorrect because $8,736,000 omits the unit variable selling expense from the calculation of the totalunit variable cost.Answer (C) is correct. To earn the same net income in 2002 as in 2001, the contribution margin must equal the sumof the 2001 pretax operating income and the 2002 fixed costs. The pretax operating income is calculated as$3,429,360 ($9,000,000 – $4,050,000 variable costs – $1,520,640 fixed costs). Fixed costs in 2002 will be the sameas in 2001 except for an increase of $29,520 in fixed selling expenses, a total of $1,550,160 ($1,520,640 aspreviously calculated + $29,520). The desired 2002 sales volume must equal the 2002 contribution margin of$4,979,520 ($3,429,360 desired pretax operating income + $1,550,160 fixed costs in 2002) divided by thecontribution margin ratio (CMR). The CMR may be determined from the unit price (constant at $9,000,000 ÷150,000 units = $60 per unit) and unit variable cost data. Unit direct materials cost in 2002 will increase by 10% to$13.20 [1.1 × ($1,800,000 ÷ 150,000 units)]. Unit direct labor cost in 2002 is given as $5.40. Unit variable overheadcost is constant at $7.20 ($1,080,000 ÷ 150,000 units). Unit variable selling expense is constant at $3.00 ($450,000 ÷150,000 units). Thus, total unit variable cost is $28.80 ($13.20 + $5.40 + $7.20 + $3.00), and the UCM is $31.20($60 unit selling price – $28.80). The CMR is therefore .52 ($31.20 ÷ $60.00), and the desired 2002 sales volume is$9,576,000 ($4,979,520 CM ÷ .52).Answer (D) is incorrect because $10,374,000 uses the variable cost ratio instead of the CMR.

[819] Gleim #: 7.80 -- Source: CIA R98 IV-61

(Refers to Fact Pattern #71)What selling price would the company have to charge for its product in 2002 to maintain the same contribution marginpercentage rate as in 2001?

Gleim's CIA Test Prep: Part III: Business Analysis and Information Technology(1165 questions)

Copyright 2004 Gleim Publications, Inc. Page 336Printed for g j

Page 337: Part three cia_with_ answers

$61.80A.$64.00B.$64.50C.$72.00D.

Answer (A) is incorrect because $61.80 equals the original unit selling price plus the increase in the unit variable costs.Answer (B) is correct. The CMR equals unit contribution margin (UCM) divided by unit selling price. The UCM equals theunit selling price minus unit variable cost. The original unit selling price was $60 ($9,000,000 revenue ÷ 150,000 units),and the original unit variable cost was $27 [($1,800,000 + $720,000 + $1,080,000 + $450,000) ÷ 150,000 units]. Theoriginal CMR was therefore .55 [($60 – $27) ÷ $60]. The new unit variable cost was calculated as $28.80. Accordingly, theunit selling price (X) is $64.

(X – $28.80) ÷ X = .55X – $28.80 = .55X

.45X = $28.80X = 28.8 ÷ .45X = $64

Answer (C) is incorrect because $64.50 omits the unit variable selling costs from all calculations.Answer (D) is incorrect because $72.00 omits the unit variable selling costs from the calculation of the original unit variablecost.

[Fact Pattern #72]A manufacturing company has the following information for its service departments, S1 and S2, and its production departments,P1 and P2.

S1 S2 P1 P2Overhead cost $4,000 $7,200 $8,000 $10,000Service provided by S1 -- 30% 30% 40%Service provided by S2 25% -- 30% 45%

[820] Gleim #: 7.81 -- Source: CIA 591 IV-1

(Refers to Fact Pattern #72)Using the direct method of service department cost allocation, how much is to be allocated from S2 to P2? Roundcalculations to the nearest dollar.

$3,240A.$4,000B.$4,320C.$5,040D.

Answer (A) is incorrect because S2’s overhead allocated to P2 is $4,320.Answer (B) is incorrect because S2’s overhead allocated to P2 is $4,320.Answer (C) is correct. Using the direct method, service use by other service departments is ignored. P2 accounted for60% [45% ÷ (30% + 45%)] of the service provided to production departments by S2. Hence, the amount of S2’soverhead allocated to P2 is $4,320 (60% × $7,200).Answer (D) is incorrect because S2’s overhead allocated to P2 is $4,320.

[821] Gleim #: 7.82 -- Source: CIA 591 IV-2

(Refers to Fact Pattern #72)Using the reciprocal method of service department allocation, how much is the total overhead cost for P1 for the period?Round calculations to the nearest dollar.

$12,560A.$12,594B.$16,594C.$12,605D.

Gleim's CIA Test Prep: Part III: Business Analysis and Information Technology(1165 questions)

Copyright 2004 Gleim Publications, Inc. Page 337Printed for g j

Page 338: Part three cia_with_ answers

Answer (A) is incorrect because P1 had total overhead costs of $12,605.Answer (B) is incorrect because P1 had total overhead costs of $12,605.Answer (C) is incorrect because P1 had total overhead costs of $12,605.Answer (D) is correct. The reciprocal method applies simultaneous equations to allocate each service department’s costsamong the departments providing mutual services before reallocation to other users. Thus, P1 had total overhead costs of$12,605 as shown below.

Determine costs:S1 = $4,000 + .25 S2S2 = $7,200 + .30 S1S1 = $4,000 + $1,800 + .075 S1S1 = $6,270

S2 = $7,200 + .30 ($6,270)S2 = $9,081

Allocate costs:  S1 S2 P1 P2 Overhead costs $4,000 $7,200 $8,000 $10,000Allocate S1 (6,270) 1,881 1,881 2,508Allocate S2 2,270 (9,081) 2,724 4,086 Balances $0 $0 $12,605 $16,594 

[822] Gleim #: 7.83 -- Source: CIA 592 IV-6

A metal fabricating company uses a job-order cost system. The company expects to have small residual pieces of metalcuttings and shavings from all of its jobs. Although the metal pieces and shavings cannot be reused, they can be sold forscrap. The scrap metal is sold when a ton of scrap has been accumulated. The requisitions and the scrap recovery foraluminum during the current month are as follows.

Aluminum requisitions:100,000 lbs. @ $1.50/lb. $150,000

Aluminum scrap recovery: 800 lbs.

This amount of scrap is within normal allowances for the company’s operations. The market price for scrap aluminumfluctuates greatly and has ranged from $.25 to $.40 per pound during the last 12 months. The accumulated scrap aluminumwas sold last month for $.35 per pound.

The appropriate accounting treatment for the scrap aluminum recovered during the current month is to

Debit direct materials quantity variance for $1,200 (800 lbs. @ $1.50/lb.) and credit work-in-process inventorycontrol for $1,200, with postings to each job from which the scrap metal was recovered.

A.

Debit scrap inventory for $280 (800 lbs. @ $.35/lb.) and credit factory overhead control for $280.B.For materiality reasons, no entry is made until the scrap metal is sold. At that time, debit cash and credit factoryoverhead control for the quantity sold at the current market price.

C.

Debit direct materials quantity variance for $1,200 (800 lbs. @ $1.50/lb.) and credit factory overhead control for$1,200 at the time of recovery, and when the scrap is sold, debit cash and credit direct materials quantity variance forthe quantity sold at the current market price.

D.

Answer (A) is incorrect because a quantity variance is not recorded for scrap that is anticipated. Furthermore, work-in-process inventory is credited only when scrap is unique to a job.Answer (B) is incorrect because an accounting entry is not needed. The amount is not material.Answer (C) is correct. Making a memorandum entry at the time of recovery is appropriate. The value of the scrap isthen recognized at the time of sale. The factory overhead control account is credited because scrap is inevitable to thecompany’s production operations and not attributable to a specific job. This accounting method has the effect ofspreading the revenue from scrap sales over all jobs or products.Answer (D) is incorrect because normal scrap is not the basis for recording a variance.

Gleim's CIA Test Prep: Part III: Business Analysis and Information Technology(1165 questions)

Copyright 2004 Gleim Publications, Inc. Page 338Printed for g j

Page 339: Part three cia_with_ answers

[Fact Pattern #73]A manufacturer has the following direct materials standard for one of its products.

Direct materials:  3 pounds @ $1.60/pound = $4.80

The company records all inventory at standard cost.

Data for the current period regarding the manufacturer's budgeted and actual production for the product as well as direct materialspurchases and issues to production for manufacture of the product are presented below.

Budgeted production for the period 8,000 unitsActual production for the period 7,500 unitsDirect materials purchases:

Pounds purchased 25,000 poundsTotal cost $38,750

Direct materials issued to production 23,000 pounds

[823] Gleim #: 7.84 -- Source: CIA 1192 IV-20

(Refers to Fact Pattern #73)The direct materials price variance for the current period is

$1,125 favorable.A.$1,150 favorable.B.$1,200 favorable.C.$1,250 favorable.D.

Answer (A) is incorrect because $1,125 favorable results from multiplying the standard direct materials that shouldhave been used in production [(3 × 7,500) = 22,500 lbs.] times the difference between standard price and actual price[($1.60 – $1.55) = $.05].Answer (B) is incorrect because $1,150 favorable results from multiplying the direct materials issued to production(23,000 lbs.) times the difference between standard price and actual price [($1.60 – $1.55) = $.05].Answer (C) is incorrect because $1,200 favorable results from multiplying the direct materials that should have beenused for budgeted production [(3 × 8,000) = 24,000 lbs.] times the difference between standard price and actual price[($1.60 – $1.55) = $.05].Answer (D) is correct. The materials price variance measures the difference between what was actually paid for thegoods purchased and the standard amount allowed for the goods purchased. Thus, it equals the difference betweenactual price and standard price, multiplied by the actual quantity purchased. This question assumes that pricevariances are isolated at the time of purchase. If they are isolated when the materials are used, the variance is thedifference between standard and actual price, times the amount used (not amount purchased). Accordingly, the directmaterials purchase price variance is $1,250 F {[($38,750 TC ÷ 25,000 lbs.) – $1.60 per lb.] × 25,000 lbs.}. Thevariance is favorable because actual price was less than standard price.

[824] Gleim #: 7.85 -- Source: CIA 1192 IV-21

(Refers to Fact Pattern #73)The materials efficiency variance for the current period is

$775 unfavorable.A.$800 unfavorable.B.$1,600 favorable.C.$3,200 favorable.D.

Gleim's CIA Test Prep: Part III: Business Analysis and Information Technology(1165 questions)

Copyright 2004 Gleim Publications, Inc. Page 339Printed for g j

Page 340: Part three cia_with_ answers

Answer (A) is incorrect because $775 unfavorable results from multiplying the difference between standard quantity [(3 ×7,500) = 22,500] and actual quantity used (23,000) times the actual price ($1.55).Answer (B) is correct. The efficiency (quantity or usage) variance for direct materials equals standard unit price times thedifference between actual usage and the standard usage for the actual output. Accordingly, the direct materials efficiencyvariance is $800 U {[23,000 lbs. issued – (3 lbs. × 7,500 units)] × $1.60}. The variance is unfavorable because actual usageexceeded standard usage.Answer (C) is incorrect because $1,600 favorable results from multiplying the difference between standard quantity thatshould have been used for budgeted production [(3 × 8,000) = 24,000] and actual quantity used (23,000) times the standardprice ($1.60).Answer (D) is incorrect because $3,200 favorable results from multiplying the difference between the actual quantitypurchased (25,000) and actual quantity used (23,000) times the standard price ($1.60).

[825] Gleim #: 7.86 -- Source: CIA 593 IV-14

The sales volume variance may be calculated for each product in the sales mix

The difference between actual and master budget sales volume, times actual unit contribution margin.A.Flexible budget and actual sales volume, times master budget unit contribution margin.B.Flexible budget and master budget sales volume, times actual budget unit contribution margin.C.Flexible budget and master budget sales volume, times master budget unit contribution margin.D.

Answer (A) is incorrect because budgeted, not actual, UCM is used to calculate this variance.Answer (B) is incorrect because the flexible budget volume is the actual volume, resulting in a zero variance.Answer (C) is incorrect because budgeted, not actual, UCM is used to calculate this variance.Answer (D) is correct. The sales volume variance is the difference between flexible budget (actual volume) andmaster budget sales quantity, times master budget unit contribution margin (UCM). This amount is calculated foreach product in the sales mix, and the results are added to determine the total sales volume variance. This variancemay be further divided into quantity and mix variances.

[826] Gleim #: 7.87 -- Source: CIA 1194 III-44

A company services office equipment. Some customers bring their equipment to the company’s service shop; othercustomers prefer to have the company’s service personnel come to their offices to repair their equipment. The mostappropriate costing method for the company is

A job-order costing system.A.An activity-based costing system.B.A process costing system.C.An operations costing system.D.

Answer (A) is correct. Job-order costing systems are designed to accumulate costs for tasks or projects that areunique and nonrepetitive. Service organizations are interested in identifying the costs applicable to each customerand/or each service call.Answer (B) is incorrect because activity-based costing systems focus on generating more accurate cost information byidentifying the activities that are cost drivers.Answer (C) is incorrect because process costing systems are designed for homogeneous products that are massproduced in continuous production runs.Answer (D) is incorrect because operations costing systems are designed for batches of homogeneous products;operations costing is a hybrid of job-order and process costing.

Gleim's CIA Test Prep: Part III: Business Analysis and Information Technology(1165 questions)

Copyright 2004 Gleim Publications, Inc. Page 340Printed for g j

Page 341: Part three cia_with_ answers

[Fact Pattern #74]A manufacturing company employs a process cost system. The company’s product passes through both Department 1 andDepartment 2 in order to be completed. Conversion costs are incurred uniformly throughout the process in Department 2. Thedirect material is added in Department 2 when conversion is 80% complete. This direct material is a preservative that does notchange the volume. Spoiled units are discovered at the final inspection and are recognized then for costing purposes. The physicalflow of units for the current month is presented below.

Beginning work-in-process in Department 2 (90%complete with respect to conversion costs) 14,000

Transferred in from Department 1 76,000Completed and transferred to finished goods 80,000Spoiled units – all normal 1,500Ending work-in-process in Department 2 (60%

complete with respect to conversion costs) 8,500

[827] Gleim #: 7.88 -- Source: CIA 1195 III-80

(Refers to Fact Pattern #74)If the manufacturing company uses the weighted-average method, the equivalent units for direct materials in Department 2for the current month would be

67,500A.80,000B.81,500C.90,000D.

Answer (A) is incorrect because 67,500 equivalent units of direct materials were added in accordance with the FIFOmethod.Answer (B) is incorrect because 80,000 units were transferred.Answer (C) is correct. The weighted-average method does not distinguish between work done currently and in theprior period. Given that materials are added when units are 80% complete, that ending work-in-process is 60%complete, and that goods are inspected when they are 100% complete, and assuming that equivalent units arecalculated for normal spoilage, the total weighted-average equivalent units for direct materials equals 81,500 (80,000units transferred + 1,500 normally spoiled units).Answer (D) is incorrect because 90,000 units equals the actual physical flow for the month.

[828] Gleim #: 7.89 -- Source: CIA 1195 III-81

(Refers to Fact Pattern #74)If the manufacturing company uses the FIFO (first-in, first-out) method, the equivalent units for conversion costs inDepartment 2 for the current month would be

72,500A.74,000B.85,200C.86,600D.

Answer (A) is incorrect because 72,500 ignores spoilage.Answer (B) is correct. The FIFO method distinguishes between work done in the prior period and work donecurrently. The total FIFO equivalent units equals the work done currently on beginning work-in-process, plus thework done on ending work-in-process, plus all units started and completed currently. Hence, total FIFO equivalentunits equals 74,000 {(10% × 14,000 units in BWIP) + (60% × 8,500 units in EWIP) + [100% × (81,500 spoiled andtransferred – 14,000 units in BWIP)]}.Answer (C) is incorrect because 85,200 includes 90% of the beginning work-in-process.Answer (D) is incorrect because 86,600 calculates the equivalent units for conversion costs according to theweighted-average method.

[829] Gleim #: 7.90 -- Source: CIA 594 III-70

Three commonly employed systems for product costing are termed job order costing, operations costing, and processcosting. Match the type of production environment with the costing method used.

Gleim's CIA Test Prep: Part III: Business Analysis and Information Technology(1165 questions)

Copyright 2004 Gleim Publications, Inc. Page 341Printed for g j

Page 342: Part three cia_with_ answers

  Job Order Operations Process  Costing Costing Costing A. Auto repair Clothing manufacturer Oil refiningB. Loan processing Drug manufacturing Custom printingC. Custom printing Print manufacturing Paper manufacturingD. Engineering design Auto assembly Motion picture

production

Answer (A) is correct. Job order costing is appropriate when producing products with individual characteristics and/orwhen identifiable groupings are possible. Process costing should be used to assign costs to similar products that are massproduced on a continuous basis. Operations costing is a hybrid of job order and process costing systems. It is used bycompanies that manufacture goods that undergo some similar and some dissimilar processes. Thus, job order costing wouldbe appropriate for auto repair, operations costing for clothing manufacturing, and process costing for oil refining.Answer (B) is incorrect because custom printing would not use process costing.Answer (C) is incorrect because paint manufacturing would not use operations costing.Answer (D) is incorrect because motion picture production would not use process costing.

[830] Gleim #: 7.91 -- Source: CIA 1194 III-46

A company produces stereo speakers for automobile manufacturers. The automobile manufacturers emphasize total qualitycontrol (TQC) in their production processes and reject approximately 3% of the stereo speakers received as being ofunacceptable quality. The company inspects the rejected speakers to determine which ones should be reworked and whichones should be discarded. The discarded speakers are classified as

Waste.A.Scrap.B.Spoilage.C.Rework costs.D.

Answer (A) is incorrect because waste is input material that is either lost in the production process or has no salesvalue.Answer (B) is incorrect because scrap is input material that has a relatively minor sales value at the end of theproduction process.Answer (C) is correct. Rejected units that are discarded are classified as spoilage. Spoilage is separated intoabnormal or normal spoilage. Normal spoilage is an inherent result of the normal production process. Abnormalspoilage is spoilage that is not expected to occur under normal, efficient operating conditions.Answer (D) is incorrect because rework costs are incurred to make unacceptable units appropriate for sale or use.

[831] Gleim #: 7.92 -- Source: CIA 595 III-93

A corporation manufactures two brands of barbed wire fencing for sale to wholesalers and large ranchers. Which of thefollowing would be the best type of costing system for such a company to use?

EOQ system.A.Job-order system.B.Process system.C.Retail inventory system.D.

Answer (A) is incorrect because an EOQ system is an inventory control tool, not a cost system.Answer (B) is incorrect because a job-order system is used when products are differentiated from one customer to thenext. Each job (customer) is a separate cost center.Answer (C) is correct. A process costing system is used when a company mass produces a standardized product on acontinuous basis. The production department becomes the cost center.Answer (D) is incorrect because this company is a manufacturer, not a retailer. The retail method converts endinginventory stated at retail to cost.

Gleim's CIA Test Prep: Part III: Business Analysis and Information Technology(1165 questions)

Copyright 2004 Gleim Publications, Inc. Page 342Printed for g j

Page 343: Part three cia_with_ answers

[832] Gleim #: 7.93 -- Source: CIA 1189 IV-15The following data apply to one item sold by a large retailer:

Purchase price $4.95Variable selling expenses $2.00Variable administrative expenses $1.00Fixed costs $7,500Expected annual sales 5,000 units

Using the contribution approach to pricing, which of the following prices is the lowest that can be charged in the short run?

$5.00A.$7.00B.$8.00C.$9.50D.

Answer (A) is incorrect because $5.00 covers the purchase price ($4.95) but not the variable selling ($2.00) andadministrative ($1.00) expenses.Answer (B) is incorrect because $7.00 covers the purchase price and the variable selling expenses ($4.95 + $2.00) butnot the variable administrative expenses ($1.00).Answer (C) is correct. In the short run, the fixed costs are sunk costs. They will be incurred whether or not the itemis produced. Accordingly, the retailer will suffer no additional loss if the price charged covers the incremental costs,which are the variable costs in this case. The lowest feasible price is the one that provides a zero (but not a negative)contribution margin (revenue – variable costs). Thus, the lowest short-run price is the unit variable cost ($4.95 +$2.00 + $1.00 = $7.95). The lowest price provided in the answer choices that will recover the unit variable cost is$8.00.Answer (D) is incorrect because $9.50 covers the fixed costs of $1.50 ($7,500 ÷ 5,000) and the variable costs.

[Fact Pattern #75]A manufacturing company is considering a new product for the coming year, an electric motor which the company can purchasefrom a reliable vendor for $21.00 per unit. The alternative is to manufacture the motor internally. The company has excesscapacity to manufacture the 30,000 motors needed in the coming year except for manufacturing space and special machinery. Themachinery can be leased for $45,000 annually. Finished goods warehouse space adjoining the main manufacturing facility, leasedfor $39,000 annually, can be converted and used to manufacture the motors. Additional off-site space can be leased at an annualcost of $54,000 to replace the finished goods warehouse. The estimated unit costs for manufacturing the motors internally,exclusive of the leasing costs itemized above, are:

Direct material $  8.00Direct labor 4.00Variable manufacturing overhead 3.00Allocated fixed manufacturing overhead 5.00 Total manufacturing cost per unit $20.00 

[833] Gleim #: 7.94 -- Source: CIA 1195 III-68

(Refers to Fact Pattern #75)A cost-benefit analysis would show that the manufacturing company would save

$54,000 by purchasing the motors from the outside vendor.A.$69,000 by purchasing the motors from the outside vendor.B.$81,000 by making the motors internally.C.$96,000 by making the motors internally.D.

Gleim's CIA Test Prep: Part III: Business Analysis and Information Technology(1165 questions)

Copyright 2004 Gleim Publications, Inc. Page 343Printed for g j

Page 344: Part three cia_with_ answers

Answer (A) is incorrect because saving $54,000 by purchasing assumes that fixed manufacturing overhead and the currentspace rental are relevant and that the new space rental is not.Answer (B) is incorrect because saving $69,000 by purchasing assumes that fixed manufacturing overhead is relevant.Answer (C) is correct. The relevant costs of making the motor are direct materials, direct labor, variable manufacturingoverhead, rental costs of the special machinery, and rental costs of the new warehouse. The fixed manufacturing overheadand the costs of the current warehouse space are not relevant because they will be incurred regardless of whether the motoris made or bought. Buying the motor will cost $630,000 ($21 × 30,000). Making the motor will cost $549,000 {[($8 DM +$4 DL + $3 VOH) × 30,000] + $54,000 + $45,000}. Hence, making the motor saves $81,000 ($630,000 – $549,000).Answer (D) is incorrect because saving $96,000 by making the motor assumes that the current space rental is relevant andthat the new space rental is not.

[834] Gleim #: 7.95 -- Source: CIA 1195 III-69

(Refers to Fact Pattern #75)If the manufacturing company decides to use its excess manufacturing capacity for the motors, the contribution or netbenefit that could have been obtained from other alternative uses is referred to as

Residual income.A.Sunk costs.B.Separable costs.C.Opportunity costs.D.

Answer (A) is incorrect because residual income equals net operating income minus an imputed interest charge forthe investment base.Answer (B) is incorrect because sunk costs are unavoidable. They cannot be changed regardless of the action taken.Answer (C) is incorrect because separable costs arise after a joint process. They are incurred beyond the split-offpoint and can be directly associated with individual products.Answer (D) is correct. Opportunity cost is the net benefit that could be obtained from the best alternative use of aresource. Accordingly, it is the benefit forgone by using that resource in a particular way.

[Fact Pattern #76]A company that annually reviews its investment opportunities and selects appropriate capital expenditures for the coming year ispresented with two projects, called Project A and Project B. Best estimates indicate that the investment outlay for Project A is$30,000 and for Project B is $1 million. The projects are considered to be equally risky. Project A is expected to generate cashinflows of $40,000 at the end of each year for 2 years. Proejct B is expected to generate cash inflows of $700,000 at the end of thefirst year and $500,000 at the end of the second year. The company has a cost of capital of 8%.

[835] Gleim #: 7.96 -- Source: CIA 1190 IV-54

(Refers to Fact Pattern #76)If the net present value (NPV) of Project A is known to be higher than the NPV of Project B, it can be concluded that

The internal rate of return (IRR) of Project A will definitely be higher than the IRR of Project B.A.The IRR of Project A will definitely be lower than the IRR of Project B.B.The ranking of IRRs is indeterminate based on the information provided.C.The payback period for Project A is definitely shorter than the payback period for Project B.D.

Answer (A) is incorrect because the ranking of IRRs is indeterminate.Answer (B) is incorrect because the ranking of IRRs is indeterminate.Answer (C) is correct. The IRR is the discount rate at which the net present value is zero. The NPV is the presentvalue of future cash flows minus the present value of the investment. Because of a possible difference in the scale ofthe projects and other factors, a higher NPV does not necessarily result in a higher IRR.Answer (D) is incorrect because the ranking for payback periods is indeterminate.

[836] Gleim #: 7.97 -- Source: CIA 594 IV-43

(Refers to Fact Pattern #76)If Projects A and B are mutually exclusive, which of the following statements is true?

Gleim's CIA Test Prep: Part III: Business Analysis and Information Technology(1165 questions)

Copyright 2004 Gleim Publications, Inc. Page 344Printed for g j

Page 345: Part three cia_with_ answers

The company should undertake Project A only.A.The company should undertake Project B only.B.The company should undertake both projects.C.The company should not undertake either project.D.

Answer (A) is incorrect because the NPV method ranks B over A.Answer (B) is correct. For two mutually exclusive projects, the company should undertake the project with the highestpositive net present value. Project B has a higher NPV than Project A. The NPVs of both projects are calculated using thefirm’s 8% cost of capital as follows:

$40,000 $40,000NPV of Project A =  –$30,000 +  +

(1.08) (1.08)2

=  $41,331

$700,000 $500,000NPV of Project A =  –$1,000,000 + +

(1.08) (1.08)2

=  $76,816

Answer (C) is incorrect because the projects are mutually exclusive.Answer (D) is incorrect because both projects have positive NPVs and one should be selected.

[837] Gleim #: 7.98 -- Source: CIA 594 IV-44

(Refers to Fact Pattern #76)If Projects A and B are independent, which of the following statements is true?

The company should undertake Project A only.A.The company should undertake Project B only.B.The company should undertake both projects.C.The company should not undertake either project.D.

Answer (A) is incorrect because each project has a positive NPV and both should be undertaken.Answer (B) is incorrect because each project has a positive NPV and both should be undertaken.Answer (C) is correct. Independent projects are those whose cash flows are not affected by the acceptance ornonacceptance of other projects. The company must decide whether to accept or to reject each of the projects.Because both projects have a positive NPV calculated using the firm’s 8% cost of capital, both should be accepted.Answer (D) is incorrect because each project has a positive NPV and both should be undertaken.

Gleim's CIA Test Prep: Part III: Business Analysis and Information Technology(1165 questions)

Copyright 2004 Gleim Publications, Inc. Page 345Printed for g j

Page 346: Part three cia_with_ answers

[838] Gleim #: 7.99 -- Source: CIA 1192 IV-18A manufacturer has been approached by a new customer who wants to place a one-time order for a component similar toone that the manufacturer makes for another customer. Existing sales will not be affected by acceptance of this order. Themanufacturer has a policy of setting its targeted selling price at 60% over full manufacturing cost. The manufacturing costsand the targeted selling price for the existing product are presented below.

Direct materials $2.30Direct labor 3.60Variable manufacturing overhead (applied at

75% of direct labor cost) 2.70Fixed manufacturing overhead (applied at

150% of direct labor cost) 5.40 

Total manufacturing cost $14.00Markup (60% of full manufacturing cost) 8.40 

Targeted selling price $22.40 

The manufacturer has excess capacity to produce the quantity of the component desired by the new customer. The directmaterials used in the component for the new customer would cost the manufacturer $0.25 less than the componentcurrently being made. The variable selling expenses (packaging and shipping) would be the same, or $0.90 per unit.

Under these circumstances, the minimum unit price at which the manufacturer would accept the special order is oneexceeding

$8.35A.$9.25B.$14.00C.$14.80D.

Answer (A) is incorrect because $8.35 does not consider the variable selling expenses.Answer (B) is correct. Because the manufacturer has excess capacity and existing sales will be unaffected, theminimum price the manufacturer should be willing to accept is anything above the total variable cost of the unit($2.05 + $3.60 + $2.70 + $0.90 = $9.25), an amount that includes the variable manufacturing cost and the variableselling expenses. The fixed costs are not relevant.Answer (C) is incorrect because $14.00 includes the fixed manufacturing overhead, which will be incurred whetherthe order is accepted or not. Additional fixed manufacturing overhead costs will not be incurred because themanufacturer is below full capacity. The fixed manufacturing overhead is a sunk cost that is not relevant to thisdecision.Answer (D) is incorrect because $14.80 does not consider that the manufacturer is below full capacity and that thecustomer is placing a one-time order. Under these circumstances, the manufacturer would not use its targeted sellingprice formula.

[839] Gleim #: 7.100 -- Source: CIA 1194 III-55

The procedure employed in zero-base budgeting is to

Budget from the ground up whereby every proposed expenditure for every unit is reviewed as though the budget wasbeing prepared for the very first time.

A.

Review the prior period’s budget along with actual results for that period and the expectations for the coming periodto develop more realistic budget amounts for the coming period.

B.

Require managers to establish priorities by including a description of what activities or changes would occur if thebudget were increased and decreased by a fixed percentage amount.

C.

Ascertain the outputs that are desired and work backward to determine the amount of inputs that will be required togenerate the desired outputs.

D.

Gleim's CIA Test Prep: Part III: Business Analysis and Information Technology(1165 questions)

Copyright 2004 Gleim Publications, Inc. Page 346Printed for g j

Page 347: Part three cia_with_ answers

Answer (A) is correct. Zero-base budgeting treats the budget as if it were being initiated for the first time. Thus, all costsare reexamined each period. Beginning with the smallest budgetary subunits, managers must determine objectives,operations, and costs for all activities. Alternative methods of conducting each activity are considered, different levels ofservice are evaluated for each activity, means of measuring work and performance are determined, and activities are rankedin order of importance to the organization.Answer (B) is incorrect because reviewing the prior period’s budget, actual results for that period, and the expectations forthe coming period is incremental budgeting.Answer (C) is incorrect because requiring managers to establish priorities in the event the budget is increased or decreasedby a fixed percentage is priority incremental budgeting.Answer (D) is incorrect because ascertaining desired outputs and necessary inputs is the input-output procedure for budgetdevelopment.

[840] Gleim #: 7.101 -- Source: CMA 695 3-17

Which one of the following is the best characteristic concerning the capital budget? The capital budget is a(n)

Plan to insure that there are sufficient funds available for the operating needs of the company.A.Exercise that sets the long-range goals of the company including the consideration of external influences caused byothers in the market.

B.

Plan that results in the cash requirements during the operating cycle.C.Plan that assesses the long-term needs of the company for plant and equipment purchases.D.

Answer (A) is incorrect because capital budgeting involves long-term investment needs, not immediate operatingneeds.Answer (B) is incorrect because strategic planning establishes long-term goals in the context of relevant factors in thefirm’s environment.Answer (C) is incorrect because cash budgeting determines operating cash flows. Capital budgeting evaluates the rateof return on specific investment alternatives.Answer (D) is correct. Capital budgeting is the process of planning expenditures for long-lived assets. It involveschoosing among investment proposals using a ranking procedure. Evaluations are based on various measuresinvolving rate of return on investment.

[841] Gleim #: 7.102 -- Source: CMA 694 3-7

Zohar’s budget for next year contains the following information:

Zohar  Units Beginning finished goods inventory 85Beginning work-in-process in equivalent units 10Desired ending finished goods inventory 100Desired ending work-in-process in equivalent units 40Projected sales for next year 1,800

How many equivalent units should Zohar plan to produce next year?

1,800A.1,565B.1,815C.1,845D.

Gleim's CIA Test Prep: Part III: Business Analysis and Information Technology(1165 questions)

Copyright 2004 Gleim Publications, Inc. Page 347Printed for g j

Page 348: Part three cia_with_ answers

Answer (A) is incorrect because 1,800 equals projected unit sales.Answer (B) is incorrect because 1,565 equals units needed for sales minus all inventory amounts.Answer (C) is incorrect because 1,815 equals finished units needed.Answer (D) is correct. The finished units needed equal 1,815:

Needed for sales 1,800Needed for ending inventory 100 Total finished units needed 1,900Minus beginning inventory 85 Finished units needed 1,815 The units to be produced equal 1,845:Finished units needed 1,815Needed for ending inventory 40 Total units in process 1,855Minus beginning WIP inventory 10 Units to be produced 1,845 

[842] Gleim #: 7.103 -- Source: CMA 696 3-14

Comparing actual results with a budget based on achieved volume is possible with the use of a

Monthly budget.A.Master budget.B.Rolling budget.C.Flexible budget.D.

Answer (A) is incorrect because comparing results using a monthly budget is no easier than using a budget of anyother duration.Answer (B) is incorrect because a master budget is the overall budget. It will not facilitate comparisons unless it isalso a flexible budget.Answer (C) is incorrect because a rolling (or continuous) budget is revised on a regular (continuous) basis. It will notfacilitate comparisons unless it is also a flexible budget.Answer (D) is correct. A flexible budget is essentially a series of several budgets prepared for many levels of sales orproduction. At the end of the period, management can compare actual costs or performance with the appropriatebudgeted level in the flexible budget. New columns can quickly be made by interpolation or extrapolation, ifnecessary. A flexible budget is designed to allow adjustment of the budget to the actual level of activity beforecomparing the budgeted activity with actual results.

[843] Gleim #: 8.1 -- Source: CMA 1287 1-24

In the modern world economy, balance-of- payments deficits and surpluses can be eliminated

Through the market mechanism of flexible exchange rates.A.If all nations adopt tight monetary policies.B.Only if trade between nations is curtailed.C.When the opportunity costs of production are made the same in all countries.D.

Gleim's CIA Test Prep: Part III: Business Analysis and Information Technology(1165 questions)

Copyright 2004 Gleim Publications, Inc. Page 348Printed for g j

Page 349: Part three cia_with_ answers

Answer (A) is correct. If exchange rates are allowed to fluctuate, the value of a particular currency will be determined inaccordance with the supply of and demand for that currency. For example, if U.S. exports to Japan are greater than imports,the dollar will be in great demand; thus, the dollar will be driven up in price relative to the Japanese yen. This increase inprice will discourage the Japanese from buying U.S. goods. The decrease in Japanese purchases will then result, inprinciple, in less demand for the dollar and a movement back toward equilibrium in the export/import ratio.Answer (B) is incorrect because tight money policies would not affect the balance of payments but would reduce economicactivity generally.Answer (C) is incorrect because deficits can be overcome by less drastic and counterproductive methods than eliminatingtrade.Answer (D) is incorrect because demand for particular products is as important as relative costs in creating trade deficits.

[844] Gleim #: 8.2 -- Source: CMA 688 1-30

What is the role of gold in the present international monetary system?

Gold is quoted in United States dollars only.A.All of the major currencies of the world, except the United States dollar, have a fixed value in terms of gold.B.Gold is like any other asset whose value depends upon supply and demand.C.Gold is the reserve asset of the International Monetary Fund.D.

Answer (A) is incorrect because, although most exchanges quote the price of gold in U.S. dollars, the dollar’s value isnot linked to that of gold.Answer (B) is incorrect because floating exchange rates have existed since about 1973. Tying currency values to agold standard, in effect, fixes exchange rates.Answer (C) is correct. Gold has no special role in the modern international monetary system. The present system isbased upon managed floating currency exchange rates. Consequently, gold is treated as a commodity, the price ofwhich depends upon supply and demand.Answer (D) is incorrect because the only reserves of the IMF are international currencies.

[845] Gleim #: 8.3 -- Source: CIA 595 IV-59

Interest rates received by depositors on Eurodollar deposits tend to be higher than domestic U.S. rates on equivalentinstruments because

Borrowers pay higher rates than domestic U.S. rates on equivalent instruments.A.The deposits involve different currencies.B.Eurodollar deposits are for smaller amounts.C.The Eurodollar market is outside the direct control of the U.S. monetary authorities and has lower costs.D.

Answer (A) is incorrect because Eurodollar borrowers tend to pay lower, not higher, rates. Borrowers and depositorscan both receive more favorable rates because, with its lower costs, the Eurodollar market can offer smaller spreadsbetween borrowing and lending rates.Answer (B) is incorrect because U.S. dollars are on deposit in both cases.Answer (C) is incorrect because Eurodollar deposits tend to be for larger, not smaller, amounts. Furthermore, smallerdeposits tend to earn lower, not higher, rates than larger deposits.Answer (D) is correct. Eurodollars are U.S. dollars deposited in banks outside the U.S. Because it is outside thedirect control of the U.S. monetary authorities, the Eurodollar market has lower costs. For example, U.S. reserverequirements and FDIC premium payments do not apply in this market. A lower cost market can offer depositorshigher interest rates.

[846] Gleim #: 8.4 -- Source: CIA 1195 IV-66

Of the following, a characteristic of Eurobonds is that they are

Always denominated in Eurodollars.A.Always sold in some country other than the one in whose currency the bond is denominated.B.Sold outside the country of the borrower but are denominated in the currency of the country in which the issue issold.

C.

Generally issued as registered bonds.D.

Gleim's CIA Test Prep: Part III: Business Analysis and Information Technology(1165 questions)

Copyright 2004 Gleim Publications, Inc. Page 349Printed for g j

Page 350: Part three cia_with_ answers

Answer (A) is incorrect because Eurobonds are not always denominated in Eurodollars, which are U.S. dollars depositedoutside the U.S.Answer (B) is correct. Eurobonds are, by definition, always sold in some country other than the one in whose currency thebond issue is denominated. Their advantage is that they are customarily less stringently regulated than most other bonds.Hence, transaction costs are lower.Answer (C) is incorrect because foreign bonds are denominated in the currency of the country in which they are sold.Answer (D) is incorrect because Eurobonds are usually issued not as registered bonds but as bearer bonds, so names andnationalities of the investors are not recorded.

[847] Gleim #: 8.5 -- Source: CMA 687 1-21

The World Trade Organization

Introduced fixed exchange rates among the United States, Canada, and members of the European Union.A.Created the International Monetary Fund.B.Encourages reductions in trade barriers between countries.C.Introduced exchange rates that adjust in response to changes in trade deficits and surpluses.D.

Answer (A) is incorrect because the WTO is a worldwide agreement concerning trade barriers, not exchange rates.Today, moreover, exchange rates are not pegged (fixed) but are allowed to float.Answer (B) is incorrect because the IMF was founded in 1944 to stabilize exchange rates.Answer (C) is correct. International trade agreements provide regulatory authority for businesses in internationaltrade. The WTO, which was established on January 1, 1995, is the product of the Uruguay Round of internationaltrade negotiations. It is a permanent body with a secretariat based in Geneva, Switzerland The WTO Agreement is apermanent set of commitments by more than 120 nations designed to prohibit trade discrimination among membernations and between imported and domestic products.Answer (D) is incorrect because the WTO is a worldwide agreement concerning trade barriers, not exchange rates.

[848] Gleim #: 8.6 -- Source: Publisher

What law prohibits U.S. companies from making bribes to foreign officials for the purpose of obtaining or retainingbusiness?

Federal Ethical Standards Act.A.Robinson-Patman Act.B.Foreign Corrupt Practices Act.C.North American Free Trade Agreement.D.

Answer (A) is incorrect because the Federal Ethical Standards Act does not deal with international payments.Answer (B) is incorrect because the Robinson-Patman Act of 1936 prohibits price discrimination.Answer (C) is correct. The Foreign Corrupt Practices Act of 1977 prohibits bribes to foreign officials for purposes ofobtaining or retaining business. The Act also requires companies to maintain effective systems of internal control.Answer (D) is incorrect because the North American Free Trade Agreement (NAFTA), passed in 1993, provides forfree trade among the nations of Canada, Mexico, and the U.S.

[849] Gleim #: 8.7 -- Source: Publisher

Which of the following is not an aspect of the Foreign Corrupt Practices Act of 1977?

It subjects management to fines and imprisonment.A.It prohibits bribes to foreign officials.B.It requires the establishment of independent audit committees.C.It requires an internal control system to be developed and maintained.D.

Answer (A) is incorrect because this is a provision of the Act.Answer (B) is incorrect because this is a provision of the Act.Answer (C) is correct. The Foreign Corrupt Practices Act of 1977 prohibits bribes to foreign officials and requiresfirms to have adequate systems of internal control. Violation of the act subjects individual managers to fines and/orimprisonment. The Act does not specifically require the establishment of audit committees, but many firms haveestablished audit committees as one means of dealing with the internal control provisions of the Act.Answer (D) is incorrect because this is a provision of the Act.

Gleim's CIA Test Prep: Part III: Business Analysis and Information Technology(1165 questions)

Copyright 2004 Gleim Publications, Inc. Page 350Printed for g j

Page 351: Part three cia_with_ answers

[850] Gleim #: 8.8 -- Source: CIA 596 IV-74

An individual had taxable income of $23,000 per year and paid $8,000 in income tax. The individual’s taxable income thenincreased to $30,000 per year resulting in a $10,000 income tax liability. The personal tax system being applied to thisindividual is

Progressive.A.Regressive.B.Marginal.C.Proportional.D.

Answer (A) is incorrect because, under a progressive tax system, both the amount of tax and the percentage of incomepaid in tax (the average tax rate) rise as income increases. In the case described, the individual pays a higher amountof tax but a lower percentage of income in tax.Answer (B) is correct. The average tax rate of the individual has decreased from 34.8% ($8,000 ÷ $23,000) to 33.3%($10,000 ÷ $30,000). Under a regressive tax system, the average tax rate falls as income rises, although the amount oftax paid may rise.Answer (C) is incorrect because marginal is not a type of tax system but a type of tax rate. The marginal tax rate is thetax rate paid on incremental income.Answer (D) is incorrect because, under a proportional income tax system, the average tax rate is the same for alllevels of income. The average tax rate of this individual falls as income rises.

[851] Gleim #: 8.9 -- Source: CIA 1195 IV-70

In most countries <List A> taxes tend to be <List B> with respect to income.

  List A List B A. General sales ProportionalB. Property RegressiveC. Personal income ProportionalD. Personal income Regressive

Answer (A) is incorrect because general sales taxes tend to be regressive. The lower the taxpayer’s income, the higherthe proportion that is usually paid in sales taxes, which are collected at a flat rate per dollar. Low-income taxpayersare unable to save as high a portion of their income as high-income taxpayers. Thus, the latter are less exposed togeneral sales taxes because they avoid the tax on the amount saved.Answer (B) is correct. Property taxes tend to be regressive. Taxpayers with lower incomes must pay a higher portionof their incomes for necessities, such as housing.Answer (C) is incorrect because personal income taxes tend to be progressive. Higher tax rates are charged on higherincomes.Answer (D) is incorrect because personal income taxes tend to be progressive. Higher tax rates are charged on higherincomes.

[852] Gleim #: 8.10 -- Source: CIA 595 IV-70

In a <List A> personal tax system, an individual’s marginal tax rate is normally <List B> the average tax rate.

  List A List B A. Progressive Greater thanB. Progressive Equal toC. Regressive Equal toD. Regressive Greater than

Gleim's CIA Test Prep: Part III: Business Analysis and Information Technology(1165 questions)

Copyright 2004 Gleim Publications, Inc. Page 351Printed for g j

Page 352: Part three cia_with_ answers

Answer (A) is correct. The marginal tax rate is the tax applicable to the last unit of income, whereas the average tax rate isthe total tax paid divided by taxable income. In a progressive tax system, higher incomes attract higher tax rates, so themarginal tax rate paid on the last unit of income exceeds the average tax rate.Answer (B) is incorrect because, in a progressive tax system, the tax rate varies with the level of income, so the marginaland average tax rates differ.Answer (C) is incorrect because, in a regressive tax system, the tax rate varies with the level of income, so the marginal andaverage tax rates differ.Answer (D) is incorrect because, in a regressive tax system, the marginal tax rate is less than, not greater than, the averagetax rate.

[853] Gleim #: 8.11 -- Source: CIA 1195 IV-69

In order to index a progressive tax system to inflation, the government must

Adjust only tax deductions and exemptions.A.Adjust only tax brackets.B.Adjust deductions, exemptions, and tax brackets.C.Ensure that nominal tax receipts grow more slowly than inflation.D.

Answer (A) is incorrect because, if only deductions and exemptions are indexed, real taxes will still increase withinflation as taxpayers experience bracket creep.Answer (B) is incorrect because, if only tax brackets are adjusted, deductions and exemptions will have less realvalue. Real taxes will rise.Answer (C) is correct. If tax brackets are not adjusted for inflation, increases in nominal but not real income maypush taxpayers into tax brackets with higher (progressive) rates. Additionally, deductions and exemptions must beindexed so that the real incidence of taxation is not increased by inflation. Increases in nominal but not real incomewill result in higher real taxable income if deductions and exemptions have not been adjusted for inflation.Answer (D) is incorrect because indexing does not result in nominal tax receipts growing more slowly than the rate ofinflation. Rather, indexing maintains the same real rate of taxation by ensuring that nominal tax receipts keep pacewith inflation.

[854] Gleim #: 8.12 -- Source: CIA 596 IV-77

A value-added tax is collected on the basis of

The difference between the value of a firm’s sales and the value of its purchases from other domestic firms.A.The difference between the selling price of a real estate property and the amount the firm originally paid for theproperty.

B.

The value of a firm’s sales to related companies.C.The profit earned on a firm’s sales.D.

Answer (A) is correct. A value-added tax (VAT) is collected on the basis of the value created by the firm. This tax ismeasured by the difference between the value of the firm’s sales and the value of its purchases. A VAT is in effect aretail sales tax. Because a consumer can avoid the tax by not purchasing, a VAT encourages saving and discouragesconsumption.Answer (B) is incorrect because the difference between the selling price of a real estate property and the amount thefirm originally paid for the property is a capital gain.Answer (C) is incorrect because the value of a firm’s sales to related companies is the internal transfer price.Answer (D) is incorrect because the profit earned on a firm’s sales is subject to the income tax.

[855] Gleim #: 8.13 -- Source: CIA 1196 IV-76

A company purchases $150,000 of inputs from other firms and incurs $500,000 of labor costs in manufacturing itsproducts. It also incurs $100,000 of interest expense. The company sells all of its output for $2.5 million. Rather thanpaying income tax, the company must pay a 25% value-added tax. How much tax will be due for this year’s activities?

$437,500A.$462,500B.$587,500C.$625,000D.

Gleim's CIA Test Prep: Part III: Business Analysis and Information Technology(1165 questions)

Copyright 2004 Gleim Publications, Inc. Page 352Printed for g j

Page 353: Part three cia_with_ answers

Answer (A) is incorrect because $437,500 is 25% of the earnings before tax.Answer (B) is incorrect because $462,500 deducts labor costs as well as purchased inputs.Answer (C) is correct. The value added is the difference between the value of the output and the value of the purchasedinputs. Value-added tax payable equals the value-added tax rate times value added, or $587,500 [.25 × ($2,500,000 –$150,000)].Answer (D) is incorrect because $625,000 is the value-added tax rate multiplied by the output of the firm.

[856] Gleim #: 8.14 -- Source: CIA 594 IV-70

Which of the following designations refers to taxes that will not necessarily take a larger absolute amount of income asincome rises?

Progressive.A.Proportional.B.Regressive.C.Regenerative.D.

Answer (A) is incorrect because progressive taxes, for which the average tax rate rises as income rises, take both alarger percentage of income and a larger absolute amount of income as income rises.Answer (B) is incorrect because proportional taxes, for which the average tax rate is constant for all income levels,always take a larger absolute amount of income as income rises.Answer (C) is correct. Regressive taxes are those for which the average tax rate falls as income rises. They take asmaller percentage of income as income rises, so they will not necessarily take a larger absolute amount of income asincome rises.Answer (D) is incorrect because regenerative is not a term used to designate types of taxes.

[857] Gleim #: 8.15 -- Source: CIA 594 IV-71

A taxpayer who earns $50,000 during the year and pays a 15% tax rate on the first $30,000 of income and a 30% tax rateon all earnings over $30,000 has a(n)

Marginal tax rate of 15%.A.Marginal tax rate of 21%.B.Average tax rate of 21%.C.Average tax rate of 22.5%.D.

Answer (A) is incorrect because the marginal tax rate is 30%. The marginal tax rate equals the highest rate paid.Answer (B) is incorrect because the marginal tax rate is 30%. The marginal tax rate equals the highest rate paid.Answer (C) is correct. The average tax rate is calculated using the weighted-average method. The weight assigned toeach rate is determined by the proportion of taxable income subject to it. The average tax rate is 21% [.15 × ($30,000÷ $50,000) + .30 × ($20,000 ÷ $50,000)].Answer (D) is incorrect because 22.5% is a simple numerical average.

[858] Gleim #: 8.16 -- Source: CMA 1283 1-13

The narrow definition of money supply, M1, consists only of

Current and demand deposits.A.Currency, demand deposits, other checkable deposits, and travelers’ checks.B.Currency, demand deposits, and small time deposits.C.Currency, demand deposits, small time deposits, and Money Market Mutual Fund balances.D.

Answer (A) is incorrect because it omits travelers’ checks.Answer (B) is correct. The narrow definition of money supply includes coins, currency, and checking accountdeposits (including travelers’ checks).Answer (C) is incorrect because it includes small time deposits.Answer (D) is incorrect because the broader definition adds small time deposits (e.g., little CDs owned byindividuals) and money market funds.

[859] Gleim #: 8.17 -- Source: CIA 597 IV-58

Some economic indicators lead the economy into a recovery or recession, and some lag it. An example of a lag variablewould be

Gleim's CIA Test Prep: Part III: Business Analysis and Information Technology(1165 questions)

Copyright 2004 Gleim Publications, Inc. Page 353Printed for g j

Page 354: Part three cia_with_ answers

Chronic unemployment.A.Orders for consumer and producer goods.B.Housing starts.C.Consumer expectations.D.

Answer (A) is correct. Economists use a variety of economic indicators to forecast turns in the business cycle. Economicindicators are variables that in the past have had a high correlation with aggregate economic activity. The best known arethe composite indexes calculated by The Conference Board, a private research group with more than 2,700 corporate andother members worldwide. Indicators may lead, lag, or coincide with economic activity. The Conference Board’s laggingindicators include average duration of unemployment in weeks, the change in the index of labor cost per unit of output, theaverage prime rate charged by banks, the ratio of manufacturing and trade inventories to sales, the commercial andindustrial loans outstanding, the ratio of consumer installment credit outstanding to personal income, and the change in theCPI for services.Answer (B) is incorrect because orders for consumer and producer goods are leading indicators.Answer (C) is incorrect because housing starts are leading indicators.Answer (D) is incorrect because consumer expectations are leading indicators.

[860] Gleim #: 8.18 -- Source: CIA 597 IV-59

The two main variables that contribute to increases in real gross domestic product (GDP) derived from labor inputs arelabor productivity and

The potential labor force.A.The inflation rate.B.Quality of output.C.Total worker hours.D.

Answer (A) is incorrect because the potential labor force is not a factor in the calculation. Rather, real GDP isdetermined by actual inputs and their productivity.Answer (B) is incorrect because real GDP is adjusted for inflation.Answer (C) is incorrect because national income accounting does not address the quality of output.Answer (D) is correct. Real GDP increases when resource inputs and their productivity increase. Thus, to the extentthat real GDP depends on labor inputs, real GDP equals total worker hours (labor input) times labor productivity (realoutput per worker per hour).

[861] Gleim #: 8.19 -- Source: CIA 596 IV-68

If the government of a country uses its foreign currency reserves to <List A> its own currency in the foreign currencymarket, the effect is to <List B> domestic aggregate demand.

  List A List B A. Purchase Not affectB. Purchase DecreaseC. Sell Not affectD. Sell Decrease

Answer (A) is incorrect because, if a government increases the demand for its own currency, the currency willappreciate. This action will alter the trade balance and the level of aggregate demand.Answer (B) is correct. Aggregate demand is the amount of real domestic output that domestic consumers, foreignbuyers, governments, and businesses will want to purchase at each price level. One factor that changes aggregatedemand is net export spending. Exchange rates are among the determinants of net export spending. When agovernment intervenes in the foreign currency market to purchase its own currency, it causes an appreciation of thatcurrency. One result is that the trade balance will be affected. Exports will fall as domestic goods become more costlyfrom the perspective of foreign consumers. Imports will rise as foreign goods become less costly for domesticconsumers. Consequently, net exports will decline and domestic aggregate demand will also decline.Answer (C) is incorrect because a sale of the currency by the domestic government will cause the currency todepreciate. This action will alter the trade balance and the level of aggregate demand.Answer (D) is incorrect because, if the government intervenes and causes a depreciation of the domestic currency,exports will rise and imports will fall. Net exports will therefore rise if the currency appreciates, so aggregate demandwill increase.

Gleim's CIA Test Prep: Part III: Business Analysis and Information Technology(1165 questions)

Copyright 2004 Gleim Publications, Inc. Page 354Printed for g j

Page 355: Part three cia_with_ answers

[862] Gleim #: 8.20 -- Source: CIA 595 IV-57The sale of final goods is <List A> the gross domestic product, and the sale of intermediate goods is <List B> the grossdomestic product.

  List A List B A. Included in Included inB. Included in Excluded fromC. Excluded from Included inD. Excluded from Excluded from

Answer (A) is incorrect because the sale of final goods is included in GDP, and the sale of intermediate goods isexcluded.Answer (B) is correct. The sale of final goods is included in, and the sale of intermediate goods is excluded from,GDP. The purpose of this treatment is to avoid double counting. The value of final goods already includes anyintermediate transactions involved in their production.Answer (C) is incorrect because the sale of final goods is included in GDP, and the sale of intermediate goods isexcluded.Answer (D) is incorrect because the sale of final goods is included in GDP, and the sale of intermediate goods isexcluded.

[863] Gleim #: 8.21 -- Source: CIA 596 IV-60

Under the income approach, gross domestic product (GDP) is measured as

Depreciation charges and indirect business taxes + Wages + Rents + Interest + Profits adjusted for net incomeearned abroad.

A.

Wages + Rents + Interest + Profits.B.Depreciation charges and indirect business taxes + Wages + Rents – Interest + Profits.C.Wages + Rents + Interest – Profits adjusted for net income earned abroad.D.

Answer (A) is correct. GDP is the total value of goods and services produced within the boundaries of a country. Itmay be measured using an expenditures approach or an income approach. Under the income approach, GDP equalsall income derived from the production of the year’s output, with an adjustment for net income earned abroad (apositive or negative amount in theory). Two types of nonincome charges or allocations must be added to incomes(wages, rents, interest, and profits). Depreciation reflects the consumption of fixed capital during the period. It is thepart of the year’s receipts that must be allocated to replace the machinery, plant, etc., used up in the production ofGDP. Indirect business taxes, such as sales, excise, and property taxes, are treated by businesses as a cost ofproduction and form part of the total price of goods and services. Thus, they are not paid as wages, rents, interest, andprofits. Accordingly, GDP may be measured as the sum of consumption of fixed capital, indirect business taxes,wages, rents, interest, and profits (proprietors’ income, corporate taxes, dividends, and undistributed corporateprofits), with an adjustment for net income earned abroad.Answer (B) is incorrect because nonincome charges and the adjustment for net income earned abroad must also beincluded in the calculation.Answer (C) is incorrect because interest income is added, not subtracted. Also, the adjustment for net income earnedabroad must be considered.Answer (D) is incorrect because profits are added, not subtracted. Also, nonincome charges must be included in thecalculation.

[864] Gleim #: 8.22 -- Source: CIA 1196 IV-59

Net domestic product is composed of the total market value of

All final goods and services produced in the economy in 1 year.A.All goods and services produced in the economy in 1 year.B.All final goods and services produced in the economy in 1 year minus the capital consumption allowance.C.All goods and services produced in the economy in 1 year minus the capital consumption allowance.D.

Answer (A) is incorrect because net domestic product is calculated net of the capital consumption allowance.Answer (B) is incorrect because net domestic product includes only final goods. The inclusion of intermediate goodswould involve double counting. Also, net domestic product is calculated net of the capital consumption allowance.Answer (C) is correct. Net domestic product is the market value of all final goods and services produced within theboundaries of a country within 1 year minus the capital consumption allowance.Answer (D) is incorrect because net domestic product does not include intermediate goods.

Gleim's CIA Test Prep: Part III: Business Analysis and Information Technology(1165 questions)

Copyright 2004 Gleim Publications, Inc. Page 355Printed for g j

Page 356: Part three cia_with_ answers

[865] Gleim #: 8.23 -- Source: CIA 1194 IV-67

(Refer to Figure CIA3_8_01.)

Curves ABC and DEF on the above graph represent production possibility curves for a nation. If point E on curve DEFrepresents the current combination of goods #1 and #2 consumed by that nation, the nation can reach point B on curveABC by

Discovering more or better resources.A.Discovering improved production techniques.B.Incurring a trade deficit.C.Incurring a trade surplus.D.

Answer (A) is incorrect because the discovery of more or better resources causes an outward shift of the productionpossibilities curve.Answer (B) is incorrect because the discovery of improved production techniques causes an outward shift of theproduction possibilities curve.Answer (C) is incorrect because incurring a trade deficit results in consumption of a combination of goods outside thecurrent production possibilities curve.Answer (D) is correct. If a nation’s exports are greater than its imports, the combination of goods consumed will beless than the combination produced. Hence, the production possibilities curve will shift inward, for example, to pointB on curve ABC.

[866] Gleim #: 8.24 -- Source: CIA 1193 IV-67

A nation’s unemployment rate increased from 4% to 6%. The economic cost of this increase in unemployment can bedescribed as the amount by which

Actual gross domestic product falls short of potential gross domestic product.A.Aggregate expenditures fall short of the full-employment level of net domestic product.B.Aggregate spending exceeds the full-employment level of net domestic product.C.Merchandise imports exceed exports.D.

Answer (A) is correct. In macroeconomic terms, the cost of unemployment is lost production. This lost output ismeasured in terms of the GDP gap, that is, the difference between actual and potential GDP. According to Okun’sLaw, a 2.5% GDP gap results from every 1% excess of the actual unemployment rate over the natural rate.Answer (B) is incorrect because the cost of unemployment is estimated in terms of the GDP gap.Answer (C) is incorrect because the cost of unemployment is estimated in terms of the GDP gap.Answer (D) is incorrect because the cost of unemployment is estimated in terms of the GDP gap.

[867] Gleim #: 8.25 -- Source: CMA 1293 1-25

In relation to the balance of trade, all international transactions involving the purchase or sale of physical products betweendomestic and foreign countries are reflected in

The balance of the capital account.A.Official reserves held by the central banks.B.The official financing account.C.The trade balance in the current account.D.

Answer (A) is incorrect because the capital account includes capital movements only; the direction of capitalmovements is influenced by the prevailing interest rates in each nation.Answer (B) is incorrect because official reserves are assets held by central banks and are not necessarily related tocurrent transactions.Answer (C) is incorrect because this is a nonsense answer.Answer (D) is correct. The balance of payments represents all international payments made by one nation to another,including those for imports, exports, investments, unilateral transfers such as pensions and gifts, and capitalmovements. The principal accounts are the current account and the capital account. The current account includes anet trade balance in goods, net investment receipts or payments, net receipts or payments for services, and the balanceof unilateral transfers.

Gleim's CIA Test Prep: Part III: Business Analysis and Information Technology(1165 questions)

Copyright 2004 Gleim Publications, Inc. Page 356Printed for g j

Page 357: Part three cia_with_ answers

[868] Gleim #: 8.26 -- Source: CMA 1288 1-19When analyzing a country’s balance of payments accounts, the

“Current account” refers only to merchandise exports and imports.A.“Current account” and “trade balance” are the same.B.“Capital account” refers to the transactions related to the international movement of financial capital.C.Country will be in financial jeopardy unless each component in the balance of payments accounts balances at the endof the year.

D.

Answer (A) is incorrect because the current account includes exports and imports of services as well as goods.Answer (B) is incorrect because the trade balance concerns goods only.Answer (C) is correct. The balance of payments accounts include all international payments made by one nation toanother, including capital movements, imports, exports, and unilateral transfers. The net of exports and imports is thecurrent account balance. The current account balance does not include capital transactions. The capital accountreflects movements of financial capital (investments).Answer (D) is incorrect because the various components are never in balance. In reality, the important considerationis the total, and that is more of a long-run than an annual problem.

[869] Gleim #: 8.27 -- Source: CMA 691 1-18

Which one of the following items represents a credit in the U.S. balance of payment accounts?

U.S. imports of crude oil.A.Expenditures of American tourists abroad.B.Earnings belonging to foreign businesses that have U.S. plants.C.Loans to Americans by foreigners.D.

Answer (A) is incorrect because imports are debited.Answer (B) is incorrect because expenditures of American tourists abroad are treated as imports and are thereforedebited.Answer (C) is incorrect because earnings of foreigners represent outflows of foreign exchange and are debited.Answer (D) is correct. Basically, exports of goods and services are credited to the U.S. balance of payments accountsand imports are debited. Similarly, capital movements may be debited or credited. For example, transfers of capitalfrom foreigners to Americans, such as loans, are credits in the accounts. In effect, these loans constitute exports ofdebt instruments and increase the supply of foreign exchange available for the U.S.

[870] Gleim #: 8.28 -- Source: CMA 1285 1-30

The dominant reason countries devalue their currencies is to

Improve the balance of payments.A.Discourage exports without having to impose controls.B.Curb inflation by increasing imports.C.Slow what is regarded as too rapid an accumulation of international reserves.D.

Answer (A) is correct. Currency devaluations result in a change in the balance of payments. A devaluation meansthat other currencies will buy more of the devaluing nation’s currency, and the prices of goods denominated in thedevalued currency are therefore cheaper. A devaluation usually results in an increase in exports, a decrease in imports(caused by higher relative input prices), and an improved balance of trade.Answer (B) is incorrect because a devaluation will encourage exports.Answer (C) is incorrect because a devaluation will discourage imports and may encourage domestic inflation. Foreigngoods will be more expensive.Answer (D) is incorrect because devaluation most likely occurs when a country is losing its reserves.

[871] Gleim #: 8.29 -- Source: Publisher

Hearsay

Could not consist of evidence of statements made in a court.A.Is evidence of statements made verbally or nonverbally.B.Is admissible but not discoverable.C.Is discoverable but not admissible.D.

Gleim's CIA Test Prep: Part III: Business Analysis and Information Technology(1165 questions)

Copyright 2004 Gleim Publications, Inc. Page 357Printed for g j

Page 358: Part three cia_with_ answers

Answer (A) is incorrect because hearsay might involve a statement made in a courtroom at an unrelated trial or hearing.Answer (B) is correct. Hearsay is a statement other than one made by the declarant while testifying at the trial or hearing,offered in evidence to prove the truth of the matter asserted. The statement alluded to may be a verbal or a nonverbalstatement. It might include body language or written statements.Answer (C) is incorrect because anything which is admissible is also discoverable under the rules of discovery.Answer (D) is incorrect because hearsay is discoverable only if it relates to the subject matter of the pending litigation and isnot privileged. Hearsay may be admissible if one of the numerous exceptions to the hearsay rule is applicable.

[872] Gleim #: 8.30 -- Source: CIA 592 II-13

Which of the following statements describes an internal control questionnaire?  It

Provides detailed evidence regarding the substance of the control system.A.Takes less of the engagement client’s time to complete than other control evaluation devices.B.Requires that the internal auditor be in attendance to properly administer it.C.Provides indirect evidence that might need corroboration.D.

Answer (A) is incorrect because questionnaires usually provide for yes/no responses and therefore provide lessdetailed evidence than some other procedures.Answer (B) is incorrect because questionnaires tend to be lengthy, and their completion is time-consuming.Answer (C) is incorrect because an auditor need not be present.Answer (D) is correct. An internal control questionnaire consists of a series of questions about the controls designedto prevent or detect errors or irregularities. Answers to the questions help the internal auditor to identify specificinternal control policies and procedures relevant to specific assertions and to design tests of controls to evaluate theeffectiveness of their design and operation. The questionnaire provides a framework to assure that specific concernsare not overlooked, but it is not a sufficient means of understanding the entire system. Thus, the evidence obtained isindirect and requires corroboration by means of observation, interviews, flowcharting, examination of documents,etc.

[873] Gleim #: 8.31 -- Source: CIA 592 II-25

During interviews with the inventory management personnel, an internal auditor learned that salespersons often orderinventory for stock without receiving the approval of the vice president of sales. Also, detail testing showed that there areno written approvals on purchase orders for replacement parts. The detail testing is a good example of

Indirect evidence.A.Circumstantial evidence.B.Corroborative evidence.C.Subjective evidence.D.

Answer (A) is incorrect because detail testing provides direct evidence that the approvals were not received. Indirectevidence establishes immediately collateral facts from which the main fact may be inferred.Answer (B) is incorrect because circumstantial evidence tends to prove a fact by proving other events orcircumstances that afford a basis for a reasonable inference of the occurrence of the fact. Thus, it is also indirectevidence.Answer (C) is correct. Corroborative evidence is evidence from a different source that supplements and confirmsother evidence. For example, oral testimony that a certain procedure was not performed may be corroborated by theabsence of documentation.Answer (D) is incorrect because subjective evidence is opinion-oriented and is not dependable for reachingengagement conclusions. No subjective evidence is present in this situation.

[874] Gleim #: 8.32 -- Source: CIA 1188 I-28

A contract dispute has arisen between an organization and a major supplier. To resolve the dispute, the most competentevidence is

Oral testimony of contracting parties.A.The original contract.B.Actions by parties to the contract.C.A letter from the supplier’s attorney.D.

Gleim's CIA Test Prep: Part III: Business Analysis and Information Technology(1165 questions)

Copyright 2004 Gleim Publications, Inc. Page 358Printed for g j

Page 359: Part three cia_with_ answers

Answer (A) is incorrect because, if the original writing is available, oral testimony cannot contradict the content of thewriting.Answer (B) is correct. The best (primary) evidence is the most persuasive evidence. Reliability and the best evidence ruleare closely related. The best evidence rule is ordinarily applied only to documentary evidence, especially to proof of thecontent of a writing. If the original writing is available, the best evidence rule prohibits a party from proving the content of awriting through oral testimony. Therefore, the original writing is the most competent evidence.Answer (C) is incorrect because the contract itself is the best evidence.Answer (D) is incorrect because the contract itself is the best evidence.

[875] Gleim #: 8.33 -- Source: CIA 1182 I-7

Much of the internal auditor’s work involves accumulation of engagement information. A duplicate of a contract ratherthan the original is an example of what kind of evidence?

Secondary.A.Circumstantial.B.Hearsay.C.Opinion.D.

Answer (A) is correct. Secondary evidence according to the legal view is acceptable if primary evidence (thestrongest evidence, e.g., original documents) has been destroyed or is not reasonably procurable. Secondary evidencemust be a proper representation of primary evidence, e.g., copies of a contract.Answer (B) is incorrect because circumstantial evidence inferentially establishes one fact by proving anothercollateral fact.Answer (C) is incorrect because hearsay is an out-of-court statement offered in evidence to prove the truth of thematter asserted.Answer (D) is incorrect because except for testimony by experts, witnesses may normally testify as to facts only.

[876] Gleim #: 8.34 -- Source: Publisher

Which of the following is not a required element of a contract?

Legality.A.Consideration.B.Legal capacity.C.A writing.D.

Answer (A) is incorrect because legality is a required element of a contract.Answer (B) is incorrect because consideration is a required element of a contract.Answer (C) is incorrect because legal capacity is a required element of a contract.Answer (D) is correct. The four essential elements of a contract are an agreement (offer and acceptance),consideration, legal capacity of the parties to contract, and a legal objective or purpose. A writing is not required toenter into a contract. However, some contracts are not enforceable unless a writing evidences the contract.

[877] Gleim #: 8.35 -- Source: Publisher

The necessary elements of a contract include

Some form of writing, equal consideration, and legal capacity.A.Formal execution, definite terms, and a valid offer and acceptance.B.Offer and acceptance, consideration, legal capacity, and mutual assent.C.Bilateral promises, legal capacity, and legality of purpose.D.

Gleim's CIA Test Prep: Part III: Business Analysis and Information Technology(1165 questions)

Copyright 2004 Gleim Publications, Inc. Page 359Printed for g j

Page 360: Part three cia_with_ answers

Answer (A) is incorrect because an oral contract is usually enforceable. Consideration must be legally sufficient and must bebargained for but need not have equal market value.Answer (B) is incorrect because most contracts are informal (simple), and if a term is missing, it can be implied by thecourt, with the exception of a quantity term.Answer (C) is correct. Contracts require each of the following:

Offer and acceptance1.Mutual assent (meeting of the minds)2.Consideration (bargained-for exchange)3.Legality (legal purpose)4.Capacity of parties (legal ability)5.

Answer (D) is incorrect because promises can be unilateral or divisible.

[878] Gleim #: 8.36 -- Source: Publisher

Consideration consists of

Something with monetary value.A.Each party’s receiving an actual benefit only.B.Two promises.C.Legal sufficiency and bargained-for exchange.D.

Answer (A) is incorrect because a promise satisfies the element of legal sufficiency. Monetary value is relativelyunimportant.Answer (B) is incorrect because consideration is an exchange of legal benefit that may not have actual benefit.Answer (C) is incorrect because contracts can be unilateral, which involves only one promise.Answer (D) is correct. Consideration must be legally sufficient and intended as a bargained-for exchange. Apromisee has provided legally sufficient consideration if (s)he incurs a legal detriment or if the promisor receives alegal benefit. An essential aspect of consideration is that it be bargained for, and given in exchange for, theconsideration provided by the other party. That is, consideration is mutual.

[879] Gleim #: 8.37 -- Source: Publisher

Lamar became homeless at a very young age and was taken in by Aunt and Uncle. Many years later, Lamar became adetective in the city police department. When Aunt disappeared and was not heard from for a month, the case was assignedto Lamar. Uncle also came to Lamar and asked him to promise to find Aunt in return for the years of support. Lamar agreedto Uncle’s request. Which of the following is true?

Lamar’s contractual duty to find Aunt is based on past consideration.A.Lamar has no contractual duty to find Aunt.B.If Uncle had also promised Lamar $1,000 for finding Aunt, he would be liable when Lamar found her.C.Lamar will be liable for breach of contract if he does not find Aunt.D.

Answer (A) is incorrect because past consideration does not satisfy the consideration requirement for the formation ofa contract.Answer (B) is correct. Lamar has a preexisting legal duty to find Aunt. Consideration does not exist if an existingduty was imposed by law or a person is already under a contractual agreement to render a specified performance.Lamar will suffer no new legal detriment by finding Aunt; thus, no contractual obligation exists.Answer (C) is incorrect because Lamar has a preexisting legal duty to find Aunt.Answer (D) is incorrect because Lamar has not made a valid contract with Uncle.

[880] Gleim #: 8.38 -- Source: E. Rahm

Which of the following creates a valid contract between the parties?

A reward was posted for the capture of Tom Jones. Maypole Burden, the county sheriff, captured Jones and claimedthe reward.

A.

Slim Polestone promised to give Mabel Abbot a wedding present.B.Anxious Father promised to pay Albert Niceguy $4,000 to take his daughter to the annual Mulestone Dance. Albertagreed.

C.

Festival Fanny was drowning in her hot tub. Her boyfriend pulled her out. After the rescue, Festival’s husbandpromised to pay her boyfriend $4 for rescuing his wife.

D.

Gleim's CIA Test Prep: Part III: Business Analysis and Information Technology(1165 questions)

Copyright 2004 Gleim Publications, Inc. Page 360Printed for g j

Page 361: Part three cia_with_ answers

Answer (A) is incorrect because the sheriff had a preexisting legal duty to capture the criminal.Answer (B) is incorrect because a promise to make a gift is not enforceable. It lacks the bargain element.Answer (C) is correct. A valid contract exists because Anxious Father offered to pay Albert $4,000 for taking his daughterto the dance. This is a unilateral contract supported by the consideration of $4,000. Albert will be entitled to the $4,000after he takes the daughter to the dance.Answer (D) is incorrect because past consideration is not consideration.

[881] Gleim #: 8.39 -- Source: Publisher

Which of the following is a true statement about the legality requirement for a contract?

Only agreements to commit a crime or a tort or to violate a statute fail to meet the legality requirement.A.All contracts in restraint of trade are considered illegal.B.The illegality that invalidates the contract must be in the consideration for the promise.C.The general rule is that neither party may be permitted to enforce an illegal agreement.D.

Answer (A) is incorrect because agreements that violate public policy lack legality.Answer (B) is incorrect because the rule of reason is applied in antitrust cases to allow certain contracts to restraincompetition.Answer (C) is incorrect because lack of legality in formation or performance renders a contract void.Answer (D) is correct. Legal purpose is requisite to enforceability of a contract. An agreement that cannot beperformed without violating a statute is void, as are agreements that violate public policy. The state, not a privatecitizen, prosecutes violations of criminal statutes. The interest protected is that of society.

[882] Gleim #: 8.40 -- Source: Publisher

Which of the following types of contracts require an internal auditor to thoroughly review the economy and efficiency ofperformance of the contract?

Lump-sum contracts.A.Cost-plus contracts.B.Unit-price contracts.C.Fixed-price contracts.D.

Answer (A) is incorrect because the internal auditor usually has little to evaluate because the work is performed inaccordance with the terms of the contract.Answer (B) is correct. Cost-plus are used in situations where the costs of a project are uncertain. The price paid tothe contractor is usually the cost to perform the contract and a fixed amount or fixed percentage cost. A problem thatmay be encountered is that the contractor may not have an incentive to perform the contract efficiently or cost-effectively. Therefore the internal auditor should review these contracts with more care.Answer (C) is incorrect because a unit-contract is used when there is a convenient measure of how much work iscompleted.Answer (D) is incorrect because a fixed-price contract is another name for a lump-sum contract.

[883] Gleim #: 8.41 -- Source: Publisher

Which of the following types of contracts would be most appropriate to measure the work completed on a contract to clear100 acres of farmland?

Fixed-price contract.A.Lump-sum contract.B.Cost-plus contract.C.Unit-price contract.D.

Answer (A) is incorrect because a fixed-price contract does not make it easier to measure work completed when aconvenient measure of work is available.Answer (B) is incorrect because a lump-sum contract is another name for a fixed-price contract and neither contractprovide for an easy measure of work available on the 100 acres of farmland.Answer (C) is incorrect because a cost-plus contract is often used when costs are unknown. However, clearing 100acres of farmland is not an unknown cost and a cost-plus contract will not help measure the progress on theperformance of the contract.Answer (D) is correct. Unit-price contracts are often used when a convenient measure of work is available on acontract. The 100 acres of farmland cleared can be easily measured per acre as the contract is performed.

Gleim's CIA Test Prep: Part III: Business Analysis and Information Technology(1165 questions)

Copyright 2004 Gleim Publications, Inc. Page 361Printed for g j

Page 362: Part three cia_with_ answers

[884] Gleim #: 8.42 -- Source: CMA 684 1-20

A tight monetary policy is frequently cited as an important policy instrument for fighting inflation. Keynesian economistsbelieve that one of the possible undesirable side effects of such a policy is

Reduced business investment due to higher interest rates.A.Reduced business investment due to lower interest rates.B.Increased business investment due to decreased government spending.C.Increased business investment because of reduced confidence in business.D.

Answer (A) is correct. A tight monetary policy means that little money is available for borrowing. When supply isreduced, the price increases. Thus, interest rates are increased when the money supply contracts. Because of highinterest rates, the cost of investment is increased and investment is discouraged.Answer (B) is incorrect because lower interest rates would lead to higher business investment.Answer (C) is incorrect because increased business investment is a desirable effect.Answer (D) is incorrect because increased business investment is a desirable effect.

[Fact Pattern #77]The table below concerns the money supply for the economy of the hypothetical country of Petasus.

  BillionsItem of Dollars

Checkable deposits 2,400Small time deposits 500Currency 60Large time deposits 1,000Noncheckable savings deposits 950Money market deposits accounts 300Money market mutual funds 550

[885] Gleim #: 8.43 -- Source: Publisher

(Refers to Fact Pattern #77)The size of the M1 money supply is

$2,400A.$2,460B.$2,960C.$3,010D.

Answer (A) is incorrect because $2,400 omits currency.Answer (B) is correct. M1 and M2 are the most common measures of the money supply. M1, or the narrow moneystock, includes coins, currency, and checking deposits. Thus, M1 equals $2,460 ($2,400 + $60).Answer (C) is incorrect because $2,960 erroneously includes small time deposits.Answer (D) is incorrect because $3,010 includes money market mutual funds.

[886] Gleim #: 8.44 -- Source: Publisher

(Refers to Fact Pattern #77)The size of the M2 money supply is

$2,960A.$3,910B.$4,760C.$5,760D.

Gleim's CIA Test Prep: Part III: Business Analysis and Information Technology(1165 questions)

Copyright 2004 Gleim Publications, Inc. Page 362Printed for g j

Page 363: Part three cia_with_ answers

Answer (A) is incorrect because $2,960 fails to include noncheckable savings deposits and money market accounts andfunds.Answer (B) is incorrect because $3,910 fails to include money market accounts and funds.Answer (C) is correct. M2 includes coins and currency, checking deposits, nonchecking savings, and small (less than$100,000) time deposits. Also included are money market accounts and money market mutual funds. Therefore, M2 equals$4,760 ($2,400 + $60 + $950 + $500 + $300 + $550).Answer (D) is incorrect because $5,760 includes large time deposits, which are an element of M3, but not M2.

[887] Gleim #: 8.45 -- Source: Publisher

(Refers to Fact Pattern #77)The size of the M3 money supply is

$2,460A.$4,760B.$5,210C.$5,760D.

Answer (A) is incorrect because $2,460 is M1.Answer (B) is incorrect because $4,760 is M2.Answer (C) is incorrect because $5,210 fails to include money market mutual funds.Answer (D) is correct. M3 is the broadest definition of the money supply and includes M2 and large time deposits.Thus, M3 equals $5,760 ($2,400 + $500 + $60 + $1,000 + $950 + $300 + $550).

[888] Gleim #: 8.46 -- Source: Publisher

Assuming the price level increases by 50%, the value of money decreases

10%A.33.33%B.50%C.55%D.

Answer (A) is incorrect because money declines in value by 33.33%.Answer (B) is correct. If the price level increases 50%, an item that used to cost $1.00 will increase to $1.50. Aperson with $1.50 could have previously purchased 1.5 units of product. After the price level increases, that personcan buy only 1 unit, or a decline of .5 units. Thus, the value of money declines by 33.33% (.5 ÷ 1.5).Answer (C) is incorrect because 50% is the increase in price level, which is calculated using a different base from theone used to calculate the decline in monetary value.Answer (D) is incorrect because money declines in value by 33.33%.

[889] Gleim #: 8.47 -- Source: Publisher

A regressive tax is a tax in which

Individuals with higher incomes pay a higher percentage of their income in tax.A.The burden for payment falls disproportionately on lower-income persons.B.The individual pays a constant percentage in taxes, regardless of income level.C.Individuals with lower incomes pay a lower percentage of their income in tax.D.

Answer (A) is incorrect because a progressive tax is a tax in which individuals with higher (lower) incomes pay ahigher (lower) percentage of their income in tax. For example, income taxes are progressive.Answer (B) is correct. With a regressive tax, the percentage paid in taxes decreases as income increases. Forexample, excise taxes and payroll taxes are both regressive taxes. An excise tax is regressive because its burden fallsdisproportionally on lower-income persons. As personal income increases, the percentage of income paid declinesbecause an excise tax is a flat amount per quantity of the good or service purchased.Answer (C) is incorrect because a proportional tax is a tax in which the individual pays a constant percentage intaxes, regardless of income level. A sales tax is a proportional tax.Answer (D) is incorrect because a progressive tax is a tax in which individuals with higher (lower) incomes pay ahigher (lower) percentage of their income in tax. For example, income taxes are progressive.

Gleim's CIA Test Prep: Part III: Business Analysis and Information Technology(1165 questions)

Copyright 2004 Gleim Publications, Inc. Page 363Printed for g j

Page 364: Part three cia_with_ answers

[890] Gleim #: 8.48 -- Source: CMA 676 1-34Which of the following economic policies would not tend to correct a balance of payments deficit in the U.S.?

Increase productivity in the manufacturing of U.S. exports.A.More effective use of monetary and fiscal policies to reduce inflation.B.A redistribution of economic aid and mutual defense burdens toward western European countries.C.Increase value of U.S. currency in relation to foreign currencies.D.

Answer (A) is incorrect because increasing productivity will lower the price of U.S. exports which will increase theamount of exports demanded, and thus reduce the balance of payments deficit.Answer (B) is incorrect because as the rate of inflation slows down, prices charged to overseas buyers are reduced,which will increase exports (and reduce the balance of payments deficit).Answer (C) is incorrect because if more money were given to countries the U.S. trades with, they would buy moreU.S. exports, thus correcting a balance of payments deficit.Answer (D) is correct. A balance of payments deficit exists when the fixed or managed exchange rate is too high.“Too high” is when the fixed price is higher than the equilibrium price would be if market forces were at work. Tocorrect a balance of payments deficit, the price of dollars must decrease or other means must be undertaken to raisethe real value to the fixed level. If the value of U.S. currency is increased, the deficit will grow.

[891] Gleim #: 8.49 -- Source: CMA 680 1-17

The value of the U.S. dollar in relation to other foreign currencies is

Determined directly by the price of gold because the value of the U.S. dollar is tied to the price of gold.A.Set by the U.S. government in consultation with other foreign governments.B.Set along with the value of other currencies held by the International Monetary Fund.C.Determined by the forces of supply and demand on the foreign exchange markets.D.

Answer (A) is incorrect because the value of the dollar is not formally tied to gold. While there may be a long-termrelationship between gold and the value of the dollar, there are often inverse (or random) short-term fluctuations.Answer (B) is incorrect because an exchange rate set by the government is called a fixed exchange rate. The oldinternational monetary system which used fixed exchange rates collapsed because of its inefficient handling ofcurrency prices.Answer (C) is incorrect because the International Monetary Fund has little effect on the valuation of currencies.Answer (D) is correct. Exchange rates are determined by the forces of supply and demand on the exchange markets.Often other forces try to intervene in this process of exchange rate determination, but these reflect only short-runpolicies. An example of this type of policy would be government or central bank intervention in the internationalmoney markets.

[892] Gleim #: 8.50 -- Source: CMA 680 1-20

When the U.S. dollar is expected to rise in value against foreign currencies, a U.S. company with foreign currencydenominated receivables and payables should

Slow down collections and speed up payments.A.Slow down collections and slow down payments.B.Speed up collections and speed up payments.C.Speed up collections and slow down payments.D.

Answer (A) is incorrect because this would raise the real cost to the company.Answer (B) is incorrect because this would not maximize the amount of U.S. dollars the company will have.Answer (C) is incorrect because this would not maximize the amount of U.S. dollars the company will have.Answer (D) is correct. The proper action would be to increase collections and decrease payments. Collections shouldbe made quickly and converted into dollars to sustain the increase in their value as the dollar appreciates. Decreasingpayments would be profitable because, as the company exchanges dollars for foreign currency at a later date, it willreceive more of the foreign currency, thus lowering its real cost.

[893] Gleim #: 8.51 -- Source: CMA 1281 1-16

In most recent years, the U.S. balance of payments has registered a deficit. This balance of payments deficit is a measure ofthe excess of

Gleim's CIA Test Prep: Part III: Business Analysis and Information Technology(1165 questions)

Copyright 2004 Gleim Publications, Inc. Page 364Printed for g j

Page 365: Part three cia_with_ answers

Exports over imports.A.Imports over exports.B.Imports, private capital outflows, grants, and remittances over exports and private capital inflows.C.Goods imports over services imports.D.

Answer (A) is incorrect because it excludes capital inflows and outflows, grants, and remittances.Answer (B) is incorrect because it excludes capital inflows and outflows, grants, and remittances.Answer (C) is correct. The balance of payments is defined as the excess of imports, private capital outflows, grants, andremittances over exports and private capital inflows. When there is a surplus in the balance of payments, more domesticgoods may have been sold abroad than were imported, and/or foreigners may have invested more capital in the domesticcountry than domestic citizens invested abroad. For this reason, a surplus is considered a favorable balance of payments.Just the opposite is true for a deficit in the balance of payments.Answer (D) is incorrect because it excludes exports completely.

[894] Gleim #: 8.52 -- Source: CMA 682 1-12

Of the following transactions, the one that would result in a debit entry in the U.S. balance of payments account is the

Receipt of dividends by an American corporation from its German subsidiary.A.Buying of IBM shares by a Kuwaiti investor.B.U.S. export of military equipment to Saudi Arabia.C.Expenditure of a U.S. resident vacationing in France.D.

Answer (A) is incorrect because it increases the demand for U.S. currency.Answer (B) is incorrect because it represents an increase in demand for U.S. assets by foreigners.Answer (C) is incorrect because it reflects an increase in U.S. goods and services demanded by foreigners.Answer (D) is correct. Since debits represent unfavorable items, expenditures by U.S. residents vacationing abroadrequire debits. Buying French goods and services means using dollars to acquire francs, thus increasing the demandfor French currency.

[895] Gleim #: 8.53 -- Source: CMA 682 1-13

Which one of the following transactions would result in a credit entry in the U.S. balance of payments account?

A New York bank pays $5,000 in interest to foreigners.A.Volkswagen’s U.S. subsidiary remits a dividend of $1 million to its parent company in Germany.B.A U.S. exporter buys marine insurance from a British insurance company.C.An Iowa farmer exports grain to Turkey.D.

Answer (A) is incorrect because it reflects an increase in demand for foreign currency by Americans.Answer (B) is incorrect because it is a private capital outflow and would appear as a debit in the balance of paymentsaccount.Answer (C) is incorrect because it represents imports by the U.S. of insurance services.Answer (D) is correct. Exports help a country’s balance of payments and are therefore considered a credit in the U.S.balance of payments account. Therefore, the Iowa farmer’s export of grain to Turkey would be recorded as a credit.

[896] Gleim #: 8.54 -- Source: CMA 1282 1-12

One may characterize the current international monetary system developed by the industrialized countries as a

Clean float. Freely floating exchange rates are determined solely by the forces of demand and supply.A.Managed or dirty float. Central banks intervene in the foreign exchange market to influence the exchange rates.B.Stable-rate system.C.Gold-based system.D.

Gleim's CIA Test Prep: Part III: Business Analysis and Information Technology(1165 questions)

Copyright 2004 Gleim Publications, Inc. Page 365Printed for g j

Page 366: Part three cia_with_ answers

Answer (A) is incorrect because a clean float system does not exist owing to central bank intervention.Answer (B) is correct. Today’s international monetary system usually permits exchange rates to float freely. However,central banks occasionally intervene to avoid large fluctuations. Accordingly, the system is called a managed or dirty floatsystem.Answer (C) is incorrect because as a result of the floating exchange rate system, rates are not stable.Answer (D) is incorrect because the gold-based system was abandoned in 1973.

[897] Gleim #: 8.55 -- Source: CMA 1282 1-13

An overvalued currency can be considered as

A tax on exports and a subsidy to imports.A.A tax on imports and a subsidy to exports.B.A tax on both exports and imports.C.A subsidy to both exports and imports.D.

Answer (A) is correct. If a currency is overvalued relative to a foreign currency, it will purchase more of thatcurrency, thereby encouraging imports of foreign goods. Similarly, exports are discouraged because the foreigncurrency will purchase less of the overvalued currency. Therefore, an overvalued currency operates as a tax onexports and a subsidy to imports.Answer (B) is incorrect because it describes the effect of an undervalued currency.Answer (C) is incorrect because overvaluation does not operate as a tax on imports.Answer (D) is incorrect because overvaluation does not operate as a subsidy to exports.

[898] Gleim #: 8.56 -- Source: CMA 1285 1-31

The U.S. balance of trade is decreased by

Foreign investments in the United States.A.U.S. investments in foreign countries.B.U.S. exports.C.U.S. imports.D.

Answer (A) is incorrect because foreign investments in the United States is a factor in the balance of payments butnot trade.Answer (B) is incorrect because U.S. investments in foreign countries is a factor in the balance of payments but nottrade.Answer (C) is incorrect because exports increase the balance of trade.Answer (D) is correct. The balance of payments embraces all payments made by one nation to another, includingcapital movements. The balance of trade is the difference between imports and exports of goods and services over agiven period. In T-account form, exports are credits and imports debits. Assuming that a credit balance reflects apositive balance of trade, imports will decrease a positive balance while exports will increase it.

[899] Gleim #: 8.57 -- Source: CMA 1285 1-35

The basic objective of the International Monetary Fund is to

Promote multilateral tariff reductions among nations.A.Provide long-term loans to member countries that want to improve their infrastructure.B.Provide a means of financing food imports so that more domestic resources can be shifted to industrialization.C.Provide short-term loans to member nations experiencing temporary balance of payments difficulties.D.

Answer (A) is incorrect because the World Trade Organization (WTO) is an attempt by the signatory nations toreduce tariffs and import quotas.Answer (B) is incorrect because the World Bank provides credits for development purposes.Answer (C) is incorrect because the U.S. Food for Peace program permits other countries to purchase U.S. farmproducts with other currencies.Answer (D) is correct. The resources of the IMF consist of a pool of currency from which short-term loans can bemade to member nations who are experiencing temporary balance of payments problems. Special drawing rights(SDRs) have also been created on the books of IMF to serve as an additional reserve for member nations to use insettling accounts.

Gleim's CIA Test Prep: Part III: Business Analysis and Information Technology(1165 questions)

Copyright 2004 Gleim Publications, Inc. Page 366Printed for g j

Page 367: Part three cia_with_ answers

[900] Gleim #: 8.58 -- Source: CMA 686 1-23The economic reasoning dictating that each nation specialize in the production of goods that it produces relatively moreefficiently than other nations and import those goods that are produced relatively more efficiently by other nations is calledthe doctrine of

Efficient trade.A.Diminishing returns.B.Relative competition.C.Comparative advantage.D.

Answer (A) is incorrect because efficient trade is not meaningful in this context.Answer (B) is incorrect because diminishing returns is not meaningful in this context.Answer (C) is incorrect because relative competition is not meaningful in this context.Answer (D) is correct. The doctrine of comparative advantage relates to comparative costs within one country. Itholds that a country should produce those products in which it has a comparative advantage, not necessarily thoseproducts in which it has an absolute advantage. The doctrine suggests that a country should produce those productsfor which the greatest efficiencies are attainable even if it could also produce other goods more efficiently thananother nation. In the long run, importing a product in which a country has an absolute advantage but not acomparative advantage will result in an overall increase in global production.

[901] Gleim #: 8.59 -- Source: CMA 1286 1-16

The balance of trade is the

Same as the balance of the current account.A.Balance on the capital account.B.Balance on all international transactions.C.Balance on the goods transactions in the current account.D.

Answer (A) is incorrect because the balance in the current account contains financial balances, but the balance oftrade is concerned only with the balance of transactions in goods and services.Answer (B) is incorrect because the capital account records net capital movements, not the balance of trade.Answer (C) is incorrect because the balance of all international transactions includes items that are not goods orservices, such as net capital movements, government transactions, and remittances.Answer (D) is correct. The balance of payments embraces all payments made by one nation to another, includingcapital movements. The balance of trade is the difference between imports and exports of goods and services over agiven period. In T-account form, exports are credits and imports debits. Assuming that a credit balance reflects apositive balance of trade, imports will decrease a positive balance while exports will increase it.

[Fact Pattern #78]Suppose that 2 worker-hours are required to produce a clock radio in Japan, while 3 are required to do the same in Germany. Inaddition, 4 worker-hours are required to produce a television in Japan, while 5 are required to do the same in Germany.

[902] Gleim #: 8.60 -- Source: CMA 1286 1-21

(Refers to Fact Pattern #78)Under conditions appropriate for free trade, both Japan and Germany would be better off if

Japan produces only televisions while Germany produces only clock radios and both use trade to meet the needs forthe item not produced locally.

A.

Japan produces only clock radios while Germany produces only televisions and both use trade to meet the needs ofthe item not produced locally.

B.

Both produce some of each item and use trade to meet additional needs of a product.C.Both Japan and Germany produce both products for internal use only with no need for international trade.D.

Gleim's CIA Test Prep: Part III: Business Analysis and Information Technology(1165 questions)

Copyright 2004 Gleim Publications, Inc. Page 367Printed for g j

Page 368: Part three cia_with_ answers

Answer (A) is incorrect because Japan and Germany should specialize in producing the product which they have acomparative advantage.Answer (B) is correct. Japan has a comparative advantage in radio production because its cost of producing radios is asmaller fraction of its cost of producing TV sets (2/4 = .5) than is true for Germany (its fraction is 3/5 = .6). Germany has acomparative advantage with regard to TV set production because its costs of producing TV sets (5/3 = 1.67) is a smallerfraction of its cost of producing radios than is true for Japan (its fraction is 4/2 = 2). Under the theory of comparativeadvantage, Japan should manufacture radios, Germany should make TV sets, and both should trade for what they do notproduce.Answer (C) is incorrect because Japan and Germany should specialize in producing the product which they have acomparative advantage.Answer (D) is incorrect because Japan and Germany should specialize in producing the product which they have acomparative advantage.

[903] Gleim #: 8.61 -- Source: CMA 1293 1-27

The economic term used to describe the situation in which each nation specializes in the production of goods that itproduces relatively more efficiently than other nations and imports those goods that are produced relatively more efficientlyby other nations is called

Balance of trade.A.Diminishing returns.B.Relative competition.C.Comparative advantage.D.

Answer (A) is incorrect because the balance of trade is the difference between imports and exports of goods.Answer (B) is incorrect because beyond some level of output, returns diminish as more and more units of an input areadded to the production process.Answer (C) is incorrect because relative competition is not a term relevant to international economics.Answer (D) is correct. The relevant concept is comparative advantage, which compares the costs of inputs within asingle country. In contrast, the concept of absolute advantage compares the costs of inputs between countries. It ispossible that a country might have an absolute advantage with respect to every product, but comparative advantage isdifferent from absolute advantage. A particular nation can have a comparative advantage even though it does not havean absolute advantage. For example, assume that Country A can produce Item X for $100 and Item Y for $200 andthat Country B can produce Item X for $50 and Item Y for $150. B has an absolute advantage in the production ofboth products; however, B has a comparative advantage in producing Item X (50 ÷ 100, or 50% of the A cost,compared with 150 ÷ 200, or 75% of the A cost for Item Y). A has a comparative advantage in producing Item Y(200 ÷ 150, or 133% of the B cost, versus 100 ÷ 50, or 200% for Item X). A nation will benefit by exporting goods inwhich it has a comparative advantage and importing goods in which it does not have a comparative advantage. Totaloutput will be maximized when each nation specializes in the products in which it has the greatest comparativeadvantage or the least comparative disadvantage.

[904] Gleim #: 8.62 -- Source: CIA 1193 IV-68

Which of the following provides the best justification for reducing trade barriers among nations?

The military self-sufficiency argument.A.Diversification for stability argument.B.The infant industry argument.C.Increased total world output argument.D.

Answer (A) is incorrect because military self-sufficiency is an argument for increasing trade barriers.Answer (B) is incorrect because diversification for stability is an argument for increasing trade barriers by promotingindustrial diversification and less dependence on other nations for certain products.Answer (C) is incorrect because protection of infant industries is an argument for increasing trade barriers for thepurpose of allowing new domestic industries to establish themselves.Answer (D) is correct. The general effect of free trade would be to maximize world output because resources in eachcountry would be deployed most efficiently according to the principle of comparative advantage. Comparativeadvantage means that a country can produce a greater output of certain goods for a given level of input than othergoods. Thus, that country should specialize in and export the goods it can produce most efficiently. Total worldoutput will increase in these circumstances.

[905] Gleim #: 8.63 -- Source: CIA 594 IV-58

Which of the following statements does not properly describe a Eurodollar Deposit?

Gleim's CIA Test Prep: Part III: Business Analysis and Information Technology(1165 questions)

Copyright 2004 Gleim Publications, Inc. Page 368Printed for g j

Page 369: Part three cia_with_ answers

Eurodollar Deposits are U.S. dollar deposits in banks outside of the U.S.A.Eurodollar Deposits are outside the direct control of the U.S. monetary authorities.B.Eurodollar Deposit rates tend to be lower than domestic U.S. rates on equivalent instruments.C.Interest rates on Eurodollar Deposits are tied to the London Interbank Offer Rate (LIBOR).D.

Answer (A) is incorrect because Eurodollar Deposits are U.S. dollar deposits in banks outside of the U.S.Answer (B) is incorrect because Eurodollar Deposits are outside the direct control of the U.S. monetary authorities.Answer (C) is correct. Eurodollars are U.S. dollars on deposit in a foreign bank. These deposits are created when a check isdrawn on a dollar deposit in a U.S. bank and then deposited in a bank outside the U.S. This amount is then available forlending by the foreign bank to its customers. However, the depositors still hold claims denominated in dollars. BecauseEurodollars are outside the direct control of the U.S. monetary authorities, U.S. banking regulations with respect toreserves, insurance, interest ceilings, etc., do not apply. The absence of these costs means that Eurodollar deposit rates tendto be higher, not lower, than domestic U.S. rates on equivalent instruments.Answer (D) is incorrect because interest rates on Eurodollar Deposits are tied to the London Interbank Offer Rate (LIBOR).

[906] Gleim #: 8.64 -- Source: CIA 1190 IV-58

A U.S. company and a German company purchased the same stock on the German stock exchange and held the stock for 1year. The value of the German mark weakened against the dollar over this period. Comparing the returns of the twocompanies, the United States company’s return will be

Lower.A.Higher.B.The same.C.Higher in the short-run but lower in the long-run.D.

Answer (A) is correct. The returns on the stock are presumably paid in marks. Hence, the change in the value of themark relative to the dollar does not affect the German company’s return. However, the weakening of the mark reducesthe number of dollars it will buy, and the U.S. company’s return in dollars is correspondingly reduced.Answer (B) is incorrect because the return to the U.S. company is adversely affected by the exchange rate movement.Answer (C) is incorrect because the return to the U.S. company is adversely affected by the exchange rate movement.Answer (D) is incorrect because the return to the U.S. company was directly and adversely affected by the exchangerate movement in the short-run, but the return to the German company was not.

[907] Gleim #: 8.65 -- Source: CMA 694 1-5

All of the following are true about international trade except that

The gains from international trade depend on specialization with comparative advantage.A.Absolute advantage without comparative advantage does not result in gains from international trade.B.Absolute advantage is defined as the ability of one nation to produce a product at a relatively lower opportunity costthan another nation.

C.

If there is reciprocal absolute advantage between two countries, specialization will make it possible to produce moreof each product.

D.

Answer (A) is incorrect because world output will be maximized if each country specializes in those products inwhich they have a comparative advantage.Answer (B) is incorrect because comparative advantage, not absolute advantage, determines the products in which acountry should specialize.Answer (C) is correct. The concept of comparative advantage compares costs within a single country. It is the abilityof one nation to produce a product at a relatively lower opportunity cost (benefits forgone) than another nation.Absolute advantage compares the costs of inputs between countries. One country might have an absolute advantagewith respect to every product. However, total output will be maximized when each nation specializes in the productsin which it has the lowest opportunity costs, that is, a comparative advantage.Answer (D) is incorrect because given a reciprocal absolute advantage, a corresponding comparative advantage willexist.

[908] Gleim #: 8.66 -- Source: CIA 597 IV-76

General sales taxes tend to be regressive with respect to income because

Gleim's CIA Test Prep: Part III: Business Analysis and Information Technology(1165 questions)

Copyright 2004 Gleim Publications, Inc. Page 369Printed for g j

Page 370: Part three cia_with_ answers

A larger portion of a lower income person's income is subject to the tax.A.A smaller portion of a lower income person's income is subject to the tax.B.The tax rate is higher for person with lower income.C.The tax claims an increasing amount of income as income rises.D.

Answer (A) is correct. A sales tax is regressive with respect to income even though the rate is the same regardless of thebuyer's income. The reason is that a greater percentage of a low-income individual's income is exposed to the tax. A higher-income individual should be able to save more and therefore shield a greater percentage of his/her income from the tax.Answer (B) is incorrect because a larger portion of a lower-income person's income is subject to the tax.Answer (C) is incorrect because the general sales tax rate is uniform for all taxpayers.Answer (D) is incorrect because, if the tax claims an increasing amount of income as income rises, it is progressive, notregressive.

[909] Gleim #: 8.67 -- Source: Publisher

Under a value-added taxing system

Businesses must pay a tax only if they make a profit.A.The consumer ultimately bears the incidence of the tax through higher prices.B.Consumer savings are discouraged.C.The amount of value added is the difference between a firm’s sales and its cost of goods sold.D.

Answer (A) is incorrect because businesses must pay a value-added tax regardless of whether they make a profit.Answer (B) is correct. A value-added tax is based on consumption. The tax is levied on the value added to goods byeach business unit in the production and distribution chain. It is the consumer who ultimately bears the incidence ofthe tax because businesses subject to the tax will raise prices to offset the operating cost of the tax.Answer (C) is incorrect because a value-added tax encourages savings. This tax is paid only on consumption, notsavings.Answer (D) is incorrect because the amount of value added is the difference between a firm’s sales and its purchases.

[910] Gleim #: 8.68 -- Source: CIA 594 IV-57

The gross domestic product (GDP) does not measure:

National economic performance.A.Market-oriented activity.B.Improvements in product quality.C.The size of national output.D.

Answer (A) is incorrect because GDP does measure national economic performance. GDP can be measured as thetotal of all output in the economy or total income.Answer (B) is incorrect because GDP does measure market-oriented activity. GDP can be measured as the total of alloutput in the economy or total income.Answer (C) is correct. GDP is the total market value of all final goods and services produced within the boundariesof the U.S. in one year, without regard to ownership of the means of production. It is a monetary measure ofquantitative data, not qualitative characteristics. Thus, improvements in product quality are not measured by GDP.Answer (D) is incorrect because GDP does measure the size of national output. GDP can be measured as the total ofall output in the economy or total income.

[911] Gleim #: 8.69 -- Source: CIA 1193 IV-64

The Gross Domestic Product (GDP) will fall following an increase in

Consumption expenditures.A.Imports.B.Exports.C.Inflation.D.

Gleim's CIA Test Prep: Part III: Business Analysis and Information Technology(1165 questions)

Copyright 2004 Gleim Publications, Inc. Page 370Printed for g j

Page 371: Part three cia_with_ answers

Answer (A) is incorrect because an increase in consumption expenditures increases GDP.Answer (B) is correct. By definition, GDP determined using the expenditures approach equals the sum of consumption,investment, government purchases of goods and services, and net exports (exports – imports). Thus, a rise in imports willcause a fall in net exports and GDP.Answer (C) is incorrect because an increase in exports increases GDP.Answer (D) is incorrect because an increase in inflation increases GDP.

[912] Gleim #: 8.70 -- Source: CIA 1194 IV-59

Under the “output” approach, gross domestic product (GDP) is measured as

Non-income charges or allocations + wages + rents + interest.A.Non-income charges or allocations + wages + rents + interest + profits.B.Consumption expenditures + investment + government purchases + interest.C.Consumption expenditures + investment + government purchases + expenditures by foreigners.D.

Answer (A) is incorrect because the income or allocations approach adds the money income from production of theyear’s output. This approach to GDP measurement adds nonincome charges or allocations, wages, rents, interest, andprofits. From this amount, net American income earned abroad is subtracted.Answer (B) is incorrect because the income or allocations approach adds the money income from production of theyear’s output. This approach to GDP measurement adds nonincome charges or allocations, wages, rents, interest, andprofits. From this amount, net American income earned abroad is subtracted.Answer (C) is incorrect because the output approach does not include interest, but includes expenditures byforeigners.Answer (D) is correct. GDP is the total value of all goods and services produced in the U.S., regardless of theownership of the means of production. GDP can be measured using an income or output (expenditures) approach.The output approach measures the amount spent to purchase the year’s total output. Using this approach, GDP iscalculated by adding consumption expenditures by households, investment expenditures by businesses, governmentpurchases of goods and services, and expenditures by foreigners.

[913] Gleim #: 8.71 -- Source: CIA 1194 IV-60

Which of the following is not a supply factor in economic growth?

The stock of technology.A.The provision of full production from a nation’s resources.B.The quantity and quality of natural resources.C.The quantity and quality of human resources.D.

Answer (A) is incorrect because technology is a supply factor in economic growth.Answer (B) is correct. The four supply factors in economic growth are 1) the quantity and quality of naturalresources, 2) the quantity and quality of human resources, 3) the supply or stock of capital goods, and 4) technology.The provision of full production from a nation’s resources relates to the use or allocation of resources, which is anallocative factor, not a supply factor, in economic growth.Answer (C) is incorrect because the quantity and quality of natural resources is a supply factor in economic growth.Answer (D) is incorrect because the quantity and quality of human resources is a supply factor in economic growth.

[914] Gleim #: 8.72 -- Source: CIA 1195 IV-57

In the output or “expenditures” approach to measuring a country’s Gross Domestic Product, which of the followingcalculations is used?

Consumption + Investment + Government Purchases + Expenditures by Foreigners.A.Consumption + Investment + Government Purchases – Expenditures by Foreigners.B.Consumption + Investment – Government Purchases – Expenditures by Foreigners.C.Consumption – Investment – Government Purchases – Expenditures by Foreigners.D.

Gleim's CIA Test Prep: Part III: Business Analysis and Information Technology(1165 questions)

Copyright 2004 Gleim Publications, Inc. Page 371Printed for g j

Page 372: Part three cia_with_ answers

Answer (A) is correct. GDP can be calculated using an income approach or an expenditures approach because what is spenton a product or service is income to those who contributed resources to its production and marketing. Thus, the amountspent to purchase the GDP is identical with the income derived from its production and sale. Under the output orexpenditures approach, GDP equals the sum of consumption expenditures by households, investment by business,government purchases of goods and services, and expenditures by foreigners (net exports).Answer (B) is incorrect because consumption, investment, government purchases, and expenditures by foreigners (netexports) are added to determine GDP.Answer (C) is incorrect because consumption, investment, government purchases, and expenditures by foreigners (netexports) are added to determine GDP.Answer (D) is incorrect because consumption, investment, government purchases, and expenditures by foreigners (netexports) are added to determine GDP.

[915] Gleim #: 8.73 -- Source: Publisher

Which of the following does the internal auditor not have to review as thoroughly in a lump-sum contract?

Progressive payments.A.Adjustments to labor costs.B.Work completed in accordance with the contract.C.Incentives associated with the contract.D.

Answer (A) is incorrect because the internal auditor should ensure that the contractor is receiving payment in order tomeet expenses and complete the contract.Answer (B) is incorrect because adjustments to labor cost may change the profitability of the contract and are of greatimportance to the internal auditor.Answer (C) is correct. The internal auditor usually has little to evaluate when the work is performed in accordancewith the contract. Further, the internal auditor may lack the technical expertise to know if the contract is beingcompleted according to the terms.Answer (D) is incorrect because incentives such as a bonus for early completion effect the overall profitability of thecontract and are frequently reviewed by the internal auditor.

[916] Gleim #: 8.74 -- Source: CIA 1194 IV 71

A nation has a progressive tax structure with income tax brackets that are not indexed to inflation. An individual taxpayerexperiences an income increase just equal to the general inflation rate and moves into a higher tax bracket. The individualtaxpayer will then experience <List A> nominal taxes payable and <List B> real taxes payable.

  List A List B A. Increasing IncreasingB. Increasing DecreasingC. Decreasing IncreasingD. Decreasing Decreasing

Answer (A) is correct. Because the taxpayer’s income increase is equal to the general inflation rate, no real increasein income has occurred. However, some or all of the increase is subject to a higher tax rate. Accordingly, the taxpayerwill experience an increase in both nominal taxes payable and in real taxes payable.Answer (B) is incorrect because real taxes payable and nominal taxes payable will increase.Answer (C) is incorrect because real taxes payable and nominal taxes payable will increase.Answer (D) is incorrect because real taxes payable and nominal taxes payable will increase.

[917] Gleim #: 8.75 -- Source: CIA 1194 IV 72

A value-added tax is collected on the basis of

The difference between the value of a firm’s sales and the value of its purchases from other domestic firms.A.The difference between the selling price of a real estate property and the amount the firm originally paid for it.B.The value of a firm’s sales to related companies.C.The profit earned on a firm’s sales.D.

Gleim's CIA Test Prep: Part III: Business Analysis and Information Technology(1165 questions)

Copyright 2004 Gleim Publications, Inc. Page 372Printed for g j

Page 373: Part three cia_with_ answers

Answer (A) is correct. A value-added tax is similar to a retail sales tax on consumer goods. It is applied to the differencebetween the value of a firm’s sales and the value of its purchases from other firms, that is, to the value added to the input. Ineffect, a value-added tax is a national tax that penalizes consumption.Answer (B) is incorrect because the difference between the selling price of a real estate property and the amount the firmoriginally paid for it is taxed as capital gain.Answer (C) is incorrect because the value of a firm’s sales to related companies is the internal transfer price.Answer (D) is incorrect because the profit earned on a firm’s sales is subject to income tax.

[918] Gleim #: 8.76 -- Source: CIA 595 IV 66

An enterprise in a 40% tax bracket needs <List A> of operating income to pay 1 of interest and <List B> of pretax incometo pay 1 of dividends.

  List A List B A. .60 .60B. 1.00 1.00C. 1.00 1.67D. 1.67 1.00

Answer (A) is incorrect because at least 1.00 of operating income will be needed to pay 1.00 of either interest ordividends.Answer (B) is incorrect because 1.67 of pretax income is required to pay 1.00 of dividends.Answer (C) is correct. Interest is deductible from operating income to obtain taxable income but dividends are not.Thus, only 1.00 of pretax income is needed to pay 1.00 of interest, but 1.67 of pretax income [1.00 ÷ (1.0 – .4)] isneeded to pay 1.00 of dividends if the enterprise is in a 40% tax bracket.Answer (D) is incorrect because 1.00 of operating income is needed to pay 1.00 of interest, and 1.67 of pretax incomeis needed to pay 1.00 of dividends.

[919] Gleim #: 8.77 -- Source: CIA 595 IV 68

An enterprise produces a good in country A and sells some of its output in country B. Selling prices are identical in the twocountries. The corporate tax rates are 40% in country A and 20% in country B. Assuming that the enterprise does notincrease or decrease production, it should <List A> sales in country B and set as <List B> a transfer price as possible, inorder to minimize global taxes.

  List A List B A. Maximize HighB. Maximize LowC. Minimize HighD. Minimize Low

Answer (A) is incorrect because, if selling prices are identical, the tax-minimizing strategy involves maximizing salesin country B. However, to report the highest possible profits in the lower tax country, input costs must be minimized.The transfer price paid to the production facility in country A must therefore be set as low as possible.Answer (B) is correct. The tax-minimizing strategy is to minimize taxable income where tax rates are high and tomaximize taxable income where tax rates are low. Consequently, the company should sell more in country B but set alow transfer price. This dual strategy minimizes sales and profits in country A, minimizes cost of sales in country B,and maximizes sales and profits in country B.Answer (C) is incorrect because a strategy of minimizing sales in country B and maximizing reported cost of goodssold in country B would result in the lowest reported profit in the lower tax country, thereby maximizing taxes paid.Answer (D) is incorrect because the tax-minimizing strategy involves maximizing sales revenue in the lower taxcountry, not minimizing it.

[920] Gleim #: 8.78 -- Source: CIA 595 IV-69

Value-added taxes are levied on

Interest income received by individual owners of publicly traded bonds.A.Dividend income received by corporations owning shares of other corporations.B.The value of a firm’s assets.C.The difference between a firm’s sales and its purchases from other firms.D.

Gleim's CIA Test Prep: Part III: Business Analysis and Information Technology(1165 questions)

Copyright 2004 Gleim Publications, Inc. Page 373Printed for g j

Page 374: Part three cia_with_ answers

Answer (A) is incorrect because value-added taxes are not levied on personal interest income.Answer (B) is incorrect because value-added taxes are not levied on corporate dividend income.Answer (C) is incorrect because value-added taxes are not levied on the value of a firm’s assets.Answer (D) is correct. A value-added tax is levied on the value a firm adds to a good or service. This amount is measuredas the difference between the value of a firm’s sales and the value of its purchases from other firms. A value-added tax istherefore similar to a retail sales tax. It is equivalent to a national sales tax on consumer goods. By penalizing consumption,it encourages saving and investment.

[921] Gleim #: 8.79 -- Source: CIA 1195 IV 71

An increase in excise taxes will generate <List A> tax revenue if it is levied on products for which demand is <List B>elastic.

  List A List B A. More MoreB. More LessC. Negative LessD. Less Less

Answer (A) is incorrect because price elasticity of demand is inversely correlated with the revenue effects of anincrease in excise taxes.Answer (B) is correct. A higher excise tax increases the selling price of the product. This price increase will have aless negative effect on sales volume for products with less elastic demand. Examples of products with low elasticityof demand include gasoline, tobacco, and alcohol. The tax revenue generated by an increase in excise taxes istherefore higher if the tax is levied on products with less elastic demand.Answer (C) is incorrect because price elasticity of demand is inversely correlated with the revenue effects of anincrease in excise taxes.Answer (D) is incorrect because price elasticity of demand is inversely correlated with the revenue effects of anincrease in excise taxes.

[922] Gleim #: 9.1 -- Source: CIA 1189 I-24

Which of the following statements accurately describes the impact that automation has on the controls normally present in amanual system?

Transaction trails are more extensive in a computer-based system than in a manual system because there is always aone-for-one correspondence between data entry and output.

A.

Responsibility for custody of information assets is more concentrated in user departments in a computer-based systemthan it is in a manual system.

B.

Controls must be more explicit in a computer-based system because many processing points that presentopportunities for human judgment in a manual system are eliminated.

C.

The quality of documentation becomes less critical in a computer-based system than it is in a manual system becausedata records are stored in machine-readable files.

D.

Answer (A) is incorrect because the “paper trail” is less extensive in an information system. Combining processingand controls within the system reduces documentary evidence.Answer (B) is incorrect because information assets are more likely to be under the control of the information systemfunction.Answer (C) is correct. Using a computer does not change the basic concepts and objectives of control. However, theuse of computers may modify the control techniques used. The processing of transactions may be combined withcontrol activities previously performed separately, or control function may be combined within the informationsystem activity.Answer (D) is incorrect because documentation is more important in an information system. Information is morelikely to be stored in machine-readable form than in hard copy.

[923] Gleim #: 9.2 -- Source: Publisher

The two broad groupings of information systems control activities are general controls and application controls. Generalcontrols include controls

Gleim's CIA Test Prep: Part III: Business Analysis and Information Technology(1165 questions)

Copyright 2004 Gleim Publications, Inc. Page 374Printed for g j

Page 375: Part three cia_with_ answers

Relating to the correction and resubmission of faulty data.A.For developing, modifying, and maintaining computer programs.B.Designed to assure that only authorized users receive output from processing.C.Designed to ensure that all data submitted for processing have been properly authorized.D.

Answer (A) is incorrect because control over report distribution (output), correction of input errors, and authorization ofinput are application controls.Answer (B) is correct. General controls are policies and procedures that relate to many information systems application andsupport the effective functioning of application controls by helping to ensure the continued proper operation of informationsystems. General controls include controls over (1) data center and network operations, (2) systems software acquisition andmaintenance, (3) access security, and (4) application systems acquisition, development, and maintenance (AU 319).Answer (C) is incorrect because control over report distribution (output), correction of input errors, and authorization ofinput are application controls.Answer (D) is incorrect because control over report distribution (output), correction of input errors, and authorization ofinput are application controls.

[924] Gleim #: 9.3 -- Source: CIA 590 I-20

The practice of maintaining a test program library separate from the production program library is an example of

An organizational control.A.Physical security.B.An input control.C.A concurrency control.D.

Answer (A) is correct. This separation is an organizational control. Organizational controls concern the propersegregation of duties and responsibilities within the information systems department. Although proper segregation isdesirable, functions that would be considered incompatible if performed by a single individual in a manual activityare often performed through the use of an information systems program or series of programs. Thus, compensatingcontrols may be necessary, such as library controls, effective supervisions, and rotation of personnel. Segregating testprograms makes concealment of unauthorized changes in production programs more difficult.Answer (B) is incorrect because physical security (e.g., climate control and restrictions on physical access) is anotheraspect of organizational control.Answer (C) is incorrect because input controls validate the completeness, accuracy, and appropriateness of input.Answer (D) is incorrect because concurrency controls manage situations in which two or more programs attempt touse a file or database at the same time.

[925] Gleim #: 9.4 -- Source: CIA 596 III-75

In traditional information systems, computer operators are generally responsible for backing up software and data files on aregular basis. In distributed or cooperative systems, ensuring that adequate backups are taken is the responsibility of

User management.A.Systems programmers.B.Data entry clerks.C.Tape librarians.D.

Answer (A) is correct. In distributed or cooperative systems, the responsibility for ensuring that adequate backups aretaken is the responsibility of user management. The systems are under the control of users, not a central informationprocessing department.Answer (B) is incorrect because distributed environments have no systems programmers comparable to those atcentral sites for traditional systems.Answer (C) is incorrect because distributed environments may not have data entry clerks. Users typically performtheir own data entry.Answer (D) is incorrect because, in distributed environments, there are no tape librarians.

[926] Gleim #: 9.5 -- Source: CIA 1196 III-55

An organization’s computer help-desk function is usually a responsibility of the

Gleim's CIA Test Prep: Part III: Business Analysis and Information Technology(1165 questions)

Copyright 2004 Gleim Publications, Inc. Page 375Printed for g j

Page 376: Part three cia_with_ answers

Applications development unit.A.Systems programming unit.B.Computer operations unit.C.User departments.D.

Answer (A) is incorrect because applications development is responsible for developing systems, not providing help to end-users.Answer (B) is incorrect because the responsibility of systems programming is to implement and maintain system-levelsoftware such as operating systems, access control software, and database systems software.Answer (C) is correct. Help desks are usually a responsibility of computer operations because of the operational nature oftheir functions. A help desk logs reported problems, resolves minor problems, and forwards more difficult problems to theappropriate information systems resources, such as a technical support unit or vendor assistance.Answer (D) is incorrect because user departments usually lack the expertise to solve computer problems.

[927] Gleim #: 9.6 -- Source: CIA1195 I-32

Most large-scale computer systems maintain at least three program libraries:  production library (for running programs);source code library (maintains original source coding); and test library (for programs which are being changed). Which ofthe following statements is correct regarding the implementation of sound controls over computer program libraries?

Only programmers should have access to the production library.A.Users should have access to the test library to determine whether all changes are properly made.B.Only the program librarian should be allowed to make changes to the production library.C.The computer operator should have access to both the production library and the source code library to assist indiagnosing computer crashes.

D.

Answer (A) is incorrect because effective control requires that programmers not be able to make undetected,unrecorded changes in data or programs. Thus, programmers should not have access to the production library.Answer (B) is incorrect because programmers should be responsible for making program changes, and users shouldbe responsible for testing the changes. Hence, users should not have access to the test library. Accountability forchanges would be diminished. Moreover, users may lack the competence to make appropriate changes.Answer (C) is correct. The program librarian is accountable for, and has custody of, the programs in the productionlibrary.Answer (D) is incorrect because, if the operator has access to both program libraries, (s)he may be able to makeunauthorized and undetected changes to the computer programs.

[928] Gleim #: 9.7 -- Source: CIA 597 III-46

Which of the following would not be appropriate to consider in the physical design of a data center?

Evaluation of potential risks from railroad lines and highways.A.Use of biometric access systems.B.Design of authorization tables for operating system access.C.Inclusion of an uninterruptible power supply system and surge protection.D.

Answer (A) is incorrect because external risks should be evaluated to determine the center’s location.Answer (B) is incorrect because biometric access systems control physical access to the data center. These devicesidentify such unique physical qualities as fingerprints, voice patterns, and retinal patterns.Answer (C) is correct. Authorization tables for operating system access address logical controls, not physicalcontrols.Answer (D) is incorrect because power supply systems and surge protection are included in data center design. Thus,two separate power lines, line conditioning equipment, and backup battery power or a generator are typical elementsin the design.

[929] Gleim #: 9.8 -- Source: CIA 1193 III-20

What type of information system uses communications capabilities to make needed data and computing capability availableto end users at separate locations?

Distributed processing system.A.Time-sharing system.B.Online processing system.C.Personal computing system.D.

Gleim's CIA Test Prep: Part III: Business Analysis and Information Technology(1165 questions)

Copyright 2004 Gleim Publications, Inc. Page 376Printed for g j

Page 377: Part three cia_with_ answers

Answer (A) is correct. The advent of less expensive and smaller computers has permitted the development of a differentalternative to centralization or decentralization. In a distributed data processing system, the organization’s processing needsare examined in their totality. The decision is not whether an application should be done centrally or locally but, rather,which parts of the application are better performed by smaller local computers and which parts are better performed at someother, possibly centralized, site. In essence, the best distribution of processing tasks within application areas is sought. Thekey distinction between decentralized and distributed systems is the interconnection among the nodes (sites) in the network.Answer (B) is incorrect because time-sharing systems are terminal-oriented systems that are connected to a centralprocessing site.Answer (C) is incorrect because an online processing system operates under direct control of the CPU.Answer (D) is incorrect because a personal computing system is a microcomputer resource dedicated to a single user,usually in a stand-alone configuration.

[930] Gleim #: 9.9 -- Source: CIA 1196 III-51

Even though an organization is committed to using its mainframe for its manufacturing plant operations, it has beenlooking for ways to downsize other applications. The purpose of downsizing is to

Improve reliability.A.Improve security.B.Reduce complexity.C.Decrease costs.D.

Answer (A) is incorrect because client-server technology used in downsizing is less reliable.Answer (B) is incorrect because security is usually better on a mainframe.Answer (C) is incorrect because downsizing applications often increases their complexity. The data files becomefragmented across multiple systems.Answer (D) is correct. The purpose of downsizing is to reduce costs of applications by abandoning larger, moreexpensive systems in favor of smaller, less expensive systems that are more versatile. However, downsizedapplications are less reliable than their mainframe predecessors because they are new and have not been usedextensively. Typically, downsized client-server implementations lack the monitoring and control features that permitrecovery from minor processing interruptions.

[931] Gleim #: 9.10 -- Source: CMA 695 4-13

In distributed data processing, a ring network

Has all computers linked to a host computer, and each linked computer routes all data through the host computer.A.Links all communication channels to form a loop, and each link passes communications through its neighbor to theappropriate location.

B.

Attaches all channel messages along one common line with communication to the appropriate location via directaccess.

C.

Organizes itself along hierarchical lines of communication usually to a central host computer.D.

Answer (A) is incorrect because a star network routes all data through the host computer.Answer (B) is correct. In a distributed system, an organization’s processing needs are examined in their totality. Thedecision is not whether an application should be done centrally or locally, but rather which parts are better performedby small local computers as intelligent terminals, and which parts are better performed at some other, possiblycentralized, site. The key distinction between decentralized and distributed systems is the interconnection among thenodes in the network. A ring network links all communication channels to form a loop and each link passescommunications through its neighbor to the appropriate location.Answer (C) is incorrect because a bus network attaches all channel messages along one common line withcommunication to the appropriate location via direct access.Answer (D) is incorrect because a tree configuration is organized along hierarchical lines to a host computer.

[932] Gleim #: 9.11 -- Source: D. Payne

Regardless of the language in which an application program is written, its execution by a computer requires that primarymemory contain

A utility program.A.An operating system.B.Compiler.C.Assembly.D.

Gleim's CIA Test Prep: Part III: Business Analysis and Information Technology(1165 questions)

Copyright 2004 Gleim Publications, Inc. Page 377Printed for g j

Page 378: Part three cia_with_ answers

Answer (A) is incorrect because utility programs are application programs that are usually attached to larger programs. Theyperform various activities, such as sorting data, merging files, converting data from one medium to another, and printing.Answer (B) is correct. An operating system (e.g., UNIX or Windows) is required in all computerized systems to oversee theelements of the CPU and the interaction of the hardware components.Answer (C) is incorrect because a compiler converts (compiles) a program written in a source language, such as FORTRAN,into machine language.Answer (D) is incorrect because an assembler translates an assembly language program into machine language. Assemblylanguage uses mnemonic codes for each machine language instruction.

[933] Gleim #: 9.12 -- Source: CPA 1185 A-52

What type of computer processing system is characterized by data that are assembled from more than one location andrecords that are updated immediately?

Personal computer systems.A.Data compression systems.B.Batch processing systems.C.Online, real-time systems.D.

Answer (A) is incorrect because access from multiple locations is more typical of larger computer systems than ofpersonal computer systems.Answer (B) is incorrect because data compression systems encode data to take up less storage space.Answer (C) is incorrect because batching of transactions requires assembly of data at one place and a delay inupdating.Answer (D) is correct. Real-time processing involves processing an input record and receiving the output soonenough to affect a current decision-making process. In a real-time system, the user interacts with the system to controlan ongoing activity. Online indicates that the decision maker is in direct communication with the computer. Online,real-time systems usually permit access to the main computer from multiple remote terminals.

[934] Gleim #: 9.13 -- Source: CPA 1194 A-37

Misstatements in a batch computer system caused by incorrect programs or data may not be detected immediately because

Errors in some transactions may cause rejection of other transactions in the batch.A.The identification of errors in input data typically is not part of the program.B.There are time delays in processing transactions in a batch system.C.The processing of transactions in a batch system is not uniform.D.

Answer (A) is incorrect because the transactions within the batch are typically not contingent upon one another.Answer (B) is incorrect because edit checks can be incorporated into batch processing environments. However, theedit checks are used to test the transactions in batches.Answer (C) is correct. Transactions in a batch computer system are grouped together, or batched, prior to processing.Batches may be processed either daily, weekly, or even monthly. Thus, considerable time may elapse between theinitiation of the transaction and the discovery of an error.Answer (D) is incorrect because a batch of transactions is typically processed uniformly.

[935] Gleim #: 9.14 -- Source: CPA 594 A-16

Which of the following statements most likely represents a disadvantage for an entity that keeps data files on a server ratherthan on a manual system?

Attention is focused on the accuracy of the programming process rather than errors in individual transactions.A.It is usually easier for unauthorized persons to access and alter the files.B.Random error associated with processing similar transactions in different ways is usually greater.C.It is usually more difficult to compare recorded accountability with the physical count of assets.D.

Gleim's CIA Test Prep: Part III: Business Analysis and Information Technology(1165 questions)

Copyright 2004 Gleim Publications, Inc. Page 378Printed for g j

Page 379: Part three cia_with_ answers

Answer (A) is incorrect because the focus on programming is an advantage of using a server. A software program allowstransactions to be processed uniformly.Answer (B) is correct. In a manual system, one individual is usually assigned responsibility for maintaining andsafeguarding the records. However, in a server environment, the data files may be subject to change by others withoutdocumentation or an indication of who made the changes.Answer (C) is incorrect because it describes a disadvantage of a manual system.Answer (D) is incorrect because the method of maintaining the files is independent of the ability to compare thisinformation in the file with the physical count of assets.

[936] Gleim #: 9.15 -- Source: Publisher

Computers containing more than one central processing unit (CPU) are increasingly common. This feature enables acomputer to execute multiple instructions from multiple programs simultaneously. This process is

Time sharing.A.Multitasking.B.Multiprocessing.C.Batch processing.D.

Answer (A) is incorrect because in time sharing, the CPU spends a fixed amount of time on each program.Answer (B) is incorrect because multitasking is multiprogramming on a single-user operating system. It is the processof having multiple programs active at a given time, although the CPU is executing instructions from only oneprogram at a time.Answer (C) is correct. Multiprocessing greatly increases system efficiency by executing multiple programs on thesame computer at the same time. In systems with only one CPU, although multiple programs may be activesimultaneously, program instructions can only be executed for one of these programs at a time.Answer (D) is incorrect because batch processing entails execution of a list of instructions from beginning to endwithout interruption.

[937] Gleim #: 9.16 -- Source: Publisher

Which of the following statements about personal computers, midrange computers, and mainframe computers is true?

Personal computers usually cost more than midrange computers but less than mainframes.A.Because of the increased use of personal computers, there will be little need for mainframes in the near future.B.Midrange computers must be programmed directly in machine language while mainframes use higher-level language.C.The cost per transaction to process on each type of computer has decreased in recent years.D.

Answer (A) is incorrect because personal computers may cost less than $1,000. Relative to personal computers,midrange computers are more costly, more powerful, have more memory, and are able to interface with moreperipheral equipment. Mainframes are large computers with many peripheral devices and large memories. There isvirtually no upper limit on the cost of a mainframe.Answer (B) is incorrect because, although personal computers have become extremely popular, e.g., for wordprocessing, databases, other business-related activities, and Internet use, large mainframes are still necessary forsimulations and processing not possible on other smaller computers. Midrange computers fill the gap betweenpersonal and mainframe computers, and a relatively strong demand also exists for these types of processors,particularly for servers used in networks.Answer (C) is incorrect because all three computers ordinarily may be programmed in higher-level languages.Answer (D) is correct. Advances in technology have resulted in less expensive computers and increased computingpower. The cost to process transactions on all kinds of computers has therefore decreased.

[938] Gleim #: 9.17 -- Source: CIA 1196 III-76

In the accounting department of a large organization, the most likely use of a CD-ROM would be to

Create permanent audit trails of EDI transactions.A.Store images of documents received in the department.B.Record the front and back of checks returned from the bank.C.Provide a way to look up accounting standards and guidelines.D.

Gleim's CIA Test Prep: Part III: Business Analysis and Information Technology(1165 questions)

Copyright 2004 Gleim Publications, Inc. Page 379Printed for g j

Page 380: Part three cia_with_ answers

Answer (A) is incorrect because creating permanent audit trails of EDI transaction sequences is likely to be accomplishedwith write once, read many times (WORM) devices.Answer (B) is incorrect because maintaining images of documents with graphical components is likely to be done withredundant arrays of inexpensive disks (RAID). This technology is a magnetic medium that provides a primary storagemethod for imaging systems.Answer (C) is incorrect because recording the front and back of checks in banking applications is likely to be done with amicroform such as microfilm.Answer (D) is correct. CD-ROM (compact disk, read-only memory) is a fixed optical medium appropriate for storage ofvery large quantities of unchanging information. Researching standards is the best use of CD-ROM technology for anaccounting department because the data are static enough for periodic updates to remain sufficiently current. CD-ROMscommonly use indexing and searching facilities that make reference works usable. However, the use of CD-ROMs willdecline as the prices of erasable optical disks become more attractive.

[939] Gleim #: 9.18 -- Source: CIA 594 III-21

Internal auditors often encounter different personal computer platforms in separate operating divisions or geographiclocations. Which of the following statements is true?

Most data and programs from one personal computer platform are transferable to another environment only throughtranslation and emulation programs.

I.

Neither data nor programs are transferable when the hardware is not identical.II.Neither data nor programs are transferable when the operating systems are not identical.III.Most data and many programs are transferable among environments through shareware programs.IV.

I.A.I, IV.B.III.C.II, III.D.

Answer (A) is correct. An emulator is a hardware device that permits one system to imitate another, that is, to use thesame data and programs and obtain the same results as the other system. A translator is a program that translates fromone programming language into another.Answer (B) is incorrect because shareware does not transfer data. Shareware is a program that can be freely copiedand tested before purchase. If the party obtaining the shareware continues to use it, there is an obligation to sendpayment to the author. Shareware typically is found on bulletin boards and online information systems.Answer (C) is incorrect because there are facilities to transfer data and programs between disparate operating systems.Answer (D) is incorrect because there are facilities to transfer data and programs between some environments.

[940] Gleim #: 9.19 -- Source: CIA 597 III-67

Response time on a local area network (LAN) was so slow that programmers working on applications kept their code ontheir own workstations rather than on the server. As a result, daily backups of the server did not contain the current sourcecode. The best approach to detect deteriorating response time is

Parallel testing.A.Integrated test facility.B.Performance monitoring.C.Program code comparison software.D.

Answer (A) is incorrect because parallel testing is an approach to implementing a new system.Answer (B) is incorrect because an ITF is an audit tool that uses a fictitious entity against which data transactions areprocessed.Answer (C) is correct. Performance monitoring is the systematic measurement and evaluation of operating resultssuch as transaction rates, response times, and incidence of error conditions. Performance monitoring will revealtrends in capacity usage so that capacity can be upgraded before response deteriorates to the point that users behavein unintended or undesirable ways.Answer (D) is incorrect because program code comparison software is used to detect unauthorized changes inprograms.

[941] Gleim #: 9.20 -- Source: CIA 597 III-75

A manufacturer is considering using bar-code identification for recording information on parts used by the manufacturer. Areason to use bar codes rather than other means of identification is to ensure that

Gleim's CIA Test Prep: Part III: Business Analysis and Information Technology(1165 questions)

Copyright 2004 Gleim Publications, Inc. Page 380Printed for g j

Page 381: Part three cia_with_ answers

The movement of all parts is recorded.A.The movement of parts is easily and quickly recorded.B.Vendors use the same part numbers.C.Vendors use the same identification methods.D.

Answer (A) is incorrect because any identification method may fail to record the movement of some parts.Answer (B) is correct. Bar-code scanning is a form of optical character recognition. Bar codes are a series of bars ofdifferent widths that represent critical information about the item. They can be read and the information can be instantlyrecorded using a scanner. Thus, bar coding records the movement of parts with minimal labor costs.Answer (C) is incorrect because each vendor has its own part-numbering scheme.Answer (D) is incorrect because each vendor has its own identification method, although vendors in the same industry oftencooperate to minimize the number of bar-code systems they use.

[942] Gleim #: 9.21 -- Source: CIA 597 III-79

A manufacturer of complex electronic equipment such as oscilloscopes and microscopes has been shipping its productswith thick paper manuals but wants to reduce the cost of producing and shipping this documentation. Of the following, thebest medium for the manufacturer to use to accomplish this result is

Write once/read many (WORM).A.Digital audiotape (DAT).B.Compact disk/read-only memory (CD-ROM).C.Computer output to microfilm (COM).D.

Answer (A) is incorrect because WORM is an optical storage technique often used as an archival medium.Answer (B) is incorrect because DAT is primarily used as a backup medium in imaging systems and as a master forCD-ROM.Answer (C) is correct. CD-ROM is cheaper to produce and ship than the existing paper, yet it permits large volumesof text and images to be reproduced. Users of the electronic equipment are likely to have access to CD-ROM readerson PCs for using such documentation.Answer (D) is incorrect because COM is used for frequent access to archived documents, such as canceled checks inbanking applications.

[943] Gleim #: 9.22 -- Source: CMA 1287 5-4

The process of monitoring, evaluating, and modifying a system as needed is referred to as systems

Analysis.A.Feasibility study.B.Maintenance.C.Implementation.D.

Answer (A) is incorrect because systems analysis is the process of determining user problems and needs, surveyingthe organization’s present system, and analyzing the facts.Answer (B) is incorrect because a feasibility study determines whether a proposed system is technically,operationally, and economically feasible.Answer (C) is correct. Systems maintenance must be undertaken by systems analysts and applications programmerscontinually throughout the life of a system. Maintenance is the redesign of the system and programs to meet newneeds or to correct design flaws. These changes should be part of a regular program of preventive maintenance.Answer (D) is incorrect because implementation involves training and educating users, testing, conversion, andfollow-up.

[944] Gleim #: 9.23 -- Source: CISA

Effective internal control for application development should provide for which of the following?

A project steering committee to initiate and oversee the systemI.A technical systems programmer to evaluate systems softwareII.Feasibility studies to evaluate existing systemsIII.The establishment of standards for systems design and programmingIV.

Gleim's CIA Test Prep: Part III: Business Analysis and Information Technology(1165 questions)

Copyright 2004 Gleim Publications, Inc. Page 381Printed for g j

Page 382: Part three cia_with_ answers

I and III.A.I, II, and IV.B.I, III, and IV.C.II, III, and IV.D.

Answer (A) is incorrect because standards must be established.Answer (B) is incorrect because a technical systems programmer has a role in the development and modification of theoperating system but not necessarily in applications development. The technical support in this area would be provided bysystems analysts rather than programmers.Answer (C) is correct. Effective systems development requires participation by top management. This can be achievedthrough a steering committee composed of higher-level representatives of system users. The committee approves orrecommends projects and reviews their progress. Studies of the economic, operational, and technical feasibility of newapplications necessarily entail evaluations of existing systems. Another necessary control is the establishment of standardsfor system design and programming. Standards represent user and system requirements determined during systems analysis.Answer (D) is incorrect because a technical systems programmer has a role in the development and modification of theoperating system but not necessarily in applications development.

[945] Gleim #: 9.24 -- Source: CIA 592 I-29

Which of the following should be emphasized before designing any system elements in a top-down approach to newsystems development?

Types of processing systems being used by competitors.A.Computer equipment to be used by the system.B.Information needs of managers for planning and control.C.Controls in place over the current system.D.

Answer (A) is incorrect because the needs of the organization should be the overriding factor in systemsdevelopment.Answer (B) is incorrect because the equipment selection should be a function of the processing needs, not vice versa.Answer (C) is correct. The top-down method begins with analysis of broad organizational goals, objectives, andpolicies as a basis for the design process. This step requires an understanding of the entity’s environment andsignificant activities. The next step is to determine the decisions made by managers and the information required tomake them. The necessary reports, databases, inputs, processing methods, and equipment specifications can then bedefined. The weakness of the top-down approach is that it tends to concentrate on managers’ information needs at theexpense of the design of efficient transaction processing at the operational level.Answer (D) is incorrect because functional controls should be designed for the new system.

[946] Gleim #: 9.25 -- Source: CIA 1195 III-65

A benefit of using computer-aided software engineering (CASE) technology is that it can ensure that

No obsolete data fields occur in files.A.Users become committed to new systems.B.All programs are optimized for efficiency.C.Data integrity rules are applied consistently.D.

Answer (A) is incorrect because obsolete data fields must be recognized by developers or users. Once recognized,obsolete data fields can be treated consistently in CASE procedures.Answer (B) is incorrect because using CASE will not ensure user commitment to new systems if they are poorlydesigned or otherwise do not meet users’ needs.Answer (C) is incorrect because, although it has the potential to accelerate system development, CASE cannot ensurethat all programs are optimized for efficiency. In fact, some CASE-developed modules may need to be optimized byhand to achieve acceptable performance.Answer (D) is correct. CASE is an automated technology (at least in part) for developing and maintaining softwareand managing projects. A benefit of using CASE technology is that it can ensure that data integrity rules, includingthose for validation and access, are applied consistently across all files.

[947] Gleim #: 9.26 -- Source: CIA 592 III-38

CASE (computer-aided software engineering) is the use of the computer to aid in the development of computer-basedinformation systems. Which of the following could not be automatically generated with CASE tools and techniques?

Gleim's CIA Test Prep: Part III: Business Analysis and Information Technology(1165 questions)

Copyright 2004 Gleim Publications, Inc. Page 382Printed for g j

Page 383: Part three cia_with_ answers

Information requirements determination.A.Program logic design.B.Computer program code.C.Program documentation.D.

Answer (A) is correct. CASE applies the computer to software design and development. It maintains on the computer alibrary of standard program modules and all of the system documentation, e.g., data flow diagrams, data dictionaries, andpseudocode (structured English); permits development of executable input and output screens; and generates program codein at least skeletal form. Thus, CASE facilitates the creation, organization, and maintenance of documentation and permitssome automation of the coding process. However, information requirements must be determined prior to using CASE.Answer (B) is incorrect because CASE may generate program logic design.Answer (C) is incorrect because CASE may generate computer program code.Answer (D) is incorrect because CASE may generate program documentation.

[948] Gleim #: 9.27 -- Source: CIA 597 III-71

Object technology is likely to become more important in companies’ strategic use of information systems because of itspotential to

Permit quicker and more reliable development of systems.A.Maintain programs written in procedural languages.B.Minimize data integrity violations in hierarchical databases.C.Streamline the traditional “waterfall” systems development methodology.D.

Answer (A) is correct. An object-oriented approach is intended to produce reusable code. Because code segmentscan be reused in other programs, the time and cost of writing software should be reduced.Answer (B) is incorrect because object technology has the potential to support faster maintenance of programs writtenin object-oriented, but not procedural, languages.Answer (C) is incorrect because object technology is being applied to relational, but not hierarchical, databases.Answer (D) is incorrect because object technology is typically implemented in a prototyping environment.

[949] Gleim #: 9.28 -- Source: CIA 597 III-72

User acceptance testing is more important in an object-oriented development process than in a traditional environmentbecause of the implications of the

Absence of traditional design documents.A.Lack of a tracking system for changes.B.Potential for continuous monitoring.C.Inheritance of properties in hierarchies.D.

Answer (A) is incorrect because, instead of traditional design documents, items such as the business model, narrativesof process functions, iterative development screens, computer processes and reports, and product descriptions guidesare produced in object-oriented development.Answer (B) is incorrect because, in general, object-oriented development systems include tracking systems forchanges made in objects and hierarchies.Answer (C) is incorrect because object-oriented systems are usually developed in client-server environments, so thepotential exists for continuous monitoring of system use. However, continuous monitoring typically occurs duringsystem operation, not during development.Answer (D) is correct. In object-oriented development, all objects in a class inherit the properties of higher classes inthe hierarchy. Thus, changes in one object may affect many other objects, and the extent and effects of errorssignificantly increase. Testing one object provides no assurance that the objects are properly coordinated.Accordingly, user acceptance testing to verify correct functioning of the whole system becomes more important.

[950] Gleim #: 9.29 -- Source: CIA 596 III-70

A systems development approach used to quickly produce a model of user interfaces, user interactions with the system, andprocess logic is called

Gleim's CIA Test Prep: Part III: Business Analysis and Information Technology(1165 questions)

Copyright 2004 Gleim Publications, Inc. Page 383Printed for g j

Page 384: Part three cia_with_ answers

Neural networking.A.Prototyping.B.Reengineering.C.Application generation.D.

Answer (A) is incorrect because neural networking involves hardware or software that imitates the processing activities ofthe human brain.Answer (B) is correct. Prototyping produces the first model(s) of a new system. This technique usually employs a softwaretool for quick development of a model of the user interface (such as by report or screen), interaction of users with thesystem (for example, a menu-screen approach or data entry), and processing logic (the executable module). Prototypingstimulates user participation because the model allows quick exploration of concepts and development of solutions withquick results.Answer (C) is incorrect because reengineering salvages reusable components of existing systems and restructures them todevelop new systems or to improve the old systems.Answer (D) is incorrect because an application generator is software that can be used to develop an application simply bydescribing its requirements to the computer rather than by writing a procedural program.

[951] Gleim #: 9.30 -- Source: CISA

An MIS manager has only enough resources to install either a new payroll system or a new data security system, but notboth. Which of the following actions is most appropriate?

Giving priority to the security system.A.Leaving the decision to the MIS manager.B.Increasing MIS staff output in order for both systems to be installed.C.Having the information systems steering committee set the priority.D.

Answer (A) is incorrect because not enough information is given to conclude that priority should be given to thesecurity system.Answer (B) is incorrect because the MIS manager should not be the only decision maker.Answer (C) is incorrect because the question indicates that development of both systems is not possible.Answer (D) is correct. The needs assessment and cost-benefit analysis should be conducted by those responsible formaking the decision. In this case, the information systems steering committee is the appropriate decision maker.

[952] Gleim #: 9.31 -- Source: CIA 1193 I-26

Which of the following is the most appropriate activity for an internal auditor to perform during a review of systemsdevelopment activity?

Serve on the MIS steering committee that determines what new systems are to be developed.A.Review the methodology used to monitor and control the system development function.B.Recommend specific automated procedures to be incorporated into new systems that will provide reasonableassurance that all data submitted to an application are converted to machine-readable form.

C.

Recommend specific operational procedures that will ensure that all data submitted for processing are converted tomachine-readable form.

D.

Answer (A) is incorrect because service on a management decision-making committee is an operating responsibilityand would impair audit objectivity.Answer (B) is correct. Auditor objectivity is not impaired when (s)he recommends standards of control for systemsor reviews procedures before implementation. However, drafting procedures for systems and designing, installing,and operating systems are not audit functions. Thus, reviewing the methodology used by an organization is anappropriate activity that enables the internal auditor to determine whether (s)he can rely on the systems developmentactivity to design and implement appropriate automated controls within applications.Answer (C) is incorrect because making recommendations for automated procedures is an operating responsibility.Answer (D) is incorrect because recommending operational procedures is an operating responsibility.

[953] Gleim #: 9.32 -- Source: CIA 1189 II-29

The purpose of input controls is to ensure the

Gleim's CIA Test Prep: Part III: Business Analysis and Information Technology(1165 questions)

Copyright 2004 Gleim Publications, Inc. Page 384Printed for g j

Page 385: Part three cia_with_ answers

Authorization of access to data files.A.Authorization of access to program files.B.Completeness, accuracy, and validity of updating.C.Completeness, accuracy, and validity of input.D.

Answer (A) is incorrect because access controls authorize access to data files.Answer (B) is incorrect because access controls authorize access to program files.Answer (C) is incorrect because processing controls ensure the completeness, accuracy, and validity of updating.Answer (D) is correct. Input controls provide reasonable assurance that data received for computer processing have beenproperly authorized and are in a form suitable for processing, i.e., complete, accurate, and valid. Input controls also relate torejection, correction, and resubmission of data that were initially incorrect.

[954] Gleim #: 9.33 -- Source: CIA 1193 I-29

A mail-order retailer of low-cost novelty items is receiving an increasing number of complaints from customers about thewrong merchandise being shipped. The order code for items has the format wwxxyyzz. The major category is ww, xx is theminor category, yy identifies the item, and zz identifies the catalog. In many cases, the wrong merchandise was sent becauseadjacent characters in the order code had been transposed. The best control for decreasing the number of orders with thewrong merchandise is to

Require customers to specify the name for each item they order.A.Add check-digits to the order codes and verify them for each order.B.Separate the parts of the order code with hyphens to make the characters easier to read.C.Use a master file reference for all order codes to verify the existence of items.D.

Answer (A) is incorrect because having customers specify the name for each item they order would let the companycorrect erroneous order codes once they had been detected, but would not, in general, detect erroneous codes.Answer (B) is correct. Self-checking digits may be used to detect incorrect codes. The digit is generated by applyingan algorithm to the code. During the input process, the check digit is recomputed by applying the same algorithm tothe code actually entered.Answer (C) is incorrect because separating the parts of the order code with hyphens would make the characters easierto read, but would not cure the problem of transposed characters.Answer (D) is incorrect because using a master file reference for all order codes would verify the existence of items,but would not detect erroneous order codes in which transposed characters in an order code match other items.

[955] Gleim #: 9.34 -- Source: CIA 597 I-19

Which of the following computerized control procedures would be most effective in ensuring that data uploaded frompersonal computers to a mainframe are complete and that no additional data are added?

Self-checking digits to ensure that only authorized part numbers are added to the database.A.Batch control totals, including control totals and hash totals.B.Passwords that effectively limit access to only those authorized to upload the data to the mainframe computer.C.Field-level edit controls that test each field for alphanumerical integrity.D.

Answer (A) is incorrect because self-checking digits detect inaccurate identification numbers. They are an effectivecontrol to ensure that the appropriate part has been identified. However, the control objective is to ensure that datatransfer is complete.Answer (B) is correct. Batch control totals for the data transferred can be reconciled with the batch control totals inthe existing file. This comparison provides information on the completion of the data transfer. Batch totals mayinclude record counts, totals of certain critical amounts, or hash totals. A hash total is a control total without a definedmeaning, such as the total of employee numbers or invoice numbers, that is used to verify the completeness of data.Thus, the hash total for the employee listing by the personnel department could be compared with the total generatedduring the payroll run.Answer (C) is incorrect because passwords help ensure that only authorized personnel make the transfer, not that datatransfer is complete.Answer (D) is incorrect because field checks are effective input controls, but they do not ensure completeness of datatransfer.

[956] Gleim #: 9.35 -- Source: CIA 596 I-58

When assessing application controls, which one of the following input controls or edit checks is most likely to be used todetect a data input error in the customer account number field?

Gleim's CIA Test Prep: Part III: Business Analysis and Information Technology(1165 questions)

Copyright 2004 Gleim Publications, Inc. Page 385Printed for g j

Page 386: Part three cia_with_ answers

Limit check.A.Validity check.B.Control total.C.Hash total.D.

Answer (A) is incorrect because reasonableness, limit, and range checks are based upon known limits for given information.For example, the hours worked per week is not likely to be greater than 45.Answer (B) is correct. Validity checks are tests of identification numbers or transaction codes for validity by comparisonwith items already known to be correct or authorized. For example, Social Security numbers on payroll input records can becompared with Social Security numbers authorized by the personnel department.Answer (C) is incorrect because a record count is a control total of the number of records processed during the operation ofa program. Financial totals summarize dollar amounts in an information field in a group of records.Answer (D) is incorrect because a hash total is the number obtained from totaling the same field value for each transactionin a batch. The total has no meaning or value other than as a comparison with another hash total.

[957] Gleim #: 9.36 -- Source: CIA 1192 II-31

Payroll master file updates are sent from a remote terminal to a mainframe program on a real-time system. A control thatworks to ensure accuracy of the transmission is a(n)

Echo check.A.Protection ring.B.Hash total.C.Integrated test facility.D.

Answer (A) is correct. An echo check is a hardware control that provides for a peripheral device to return (echo) asignal sent by the CPU. For example, the CPU sends a signal to the printer, and the printer, just prior to printing,sends a signal back to the CPU verifying that the proper print position has been activated.Answer (B) is incorrect because a protection ring prevents accidental writing on a tape file. A real-time system wouldnot use tape files.Answer (C) is incorrect because hash totals are used to control data sent to a batch system, not a real-time system.Answer (D) is incorrect because integrated test facilities are useful in testing real-time systems but cannot be used toensure completeness of data transmissions.

[958] Gleim #: 9.37 -- Source: CIA 590 II-20

Detecting errors in real memory is a function of

Memory protection.A.Parity checking.B.Validity checking.C.Range checking.D.

Answer (A) is incorrect because memory protection prohibits programs from accessing memory outside theirdesignated ranges.Answer (B) is correct. A parity check adds the bits in a character or message and checks the sum to determine if it isodd or even, depending on whether the computer has odd or even parity. This check verifies that all data have beentransferred without loss. For example, if the computer has even parity, a bit will be added to a binary coded characteror message that contains an odd number of bits. No bit is added if a character or message in binary form has an evennumber of bits.Answer (C) is incorrect because for hardware, validity checking verifies that a machine-level instruction is a validinstruction; for applications, validity checking verifies that transaction data are complete, authorized, and reasonable.Answer (D) is incorrect because range checking verifies that input data values are within pre-determined ranges.

[959] Gleim #: 9.38 -- Source: CMA 680 5-15

Omen Company is a manufacturer of men’s shirts. It distributes weekly sales reports to each sales manager. The quantity2R5 appeared in the quantity sold column for one of the items on the weekly sales report for one of the sales managers. Themost likely explanation for what has occurred is that

Gleim's CIA Test Prep: Part III: Business Analysis and Information Technology(1165 questions)

Copyright 2004 Gleim Publications, Inc. Page 386Printed for g j

Page 387: Part three cia_with_ answers

The output quantity has been stated in hexadecimal numbers.A.The computer has malfunctioned during execution.B.The printer has malfunctioned and the “R” should have been a decimal point.C.The program did not contain a data checking routine for input data.D.

Answer (A) is incorrect because R is not a hexadecimal character. Hexadecimal characters are 0-9 and A-F representing 0 to15 in decimal.Answer (B) is incorrect because the probability of a computer malfunction’s resulting in the printing of an R is slight.Answer (C) is incorrect because 2.5 would not be appropriate for a quantity of shirts sold.Answer (D) is correct. The probable explanation for reporting a quantity using a character other than a digit is that the datawere incorrectly encoded and the computer program did not perform a field check, which would have detected the error. Afield check tests whether a field consists of the proper characters, whether alphabetic, numeric, special, or combinationsthereof.

[960] Gleim #: 9.39 -- Source: CMA 687 5-5

The online data entry control called preformatting is

A program initiated prior to regular input to discover errors in data before entry so that the errors can be corrected.A.A check to determine if all data items for a transaction have been entered by the terminal operator.B.A series of requests for required input data that requires an acceptable response to each request before a subsequentrequest is made.

C.

The display of a document with blanks for data items to be entered by the terminal operator.D.

Answer (A) is incorrect because an edit routine is a program initiated prior to regular input to discover errors in databefore entry so that the errors can be corrected.Answer (B) is incorrect because a completeness check tests whether all data items for a transaction have been enteredby the terminal operator.Answer (C) is incorrect because the dialogue approach is another screen prompting method for data entry. It is mostappropriate when information is received orally, e.g., by phone.Answer (D) is correct. To avoid data entry errors in online systems, a preformatted screen approach may be used. It isa screen prompting approach that involves the display on a monitor of a set of boxes for entry of specified data items.The format may even be in the form of a copy of a transaction document. This technique is best suited to conversionof data from a source document.

[961] Gleim #: 9.40 -- Source: CIA 595 I-27

A catalog company has been experiencing an increasing incidence of problems in which the wrong products have beenshipped to the customer. Most of the customer orders come in over the telephone, and an operator enters the data into theorder system immediately. Which of the following control procedures, if properly implemented, would address theproblem?

Have the computer automatically assign a sequential order number to each customer order.I.Implement a self-checking digit algorithm for each product number and request entries by product number.II.Request entries by product number, have the computer program identify the product and price, and require theoperator to orally verify the product description with the customer.

III.

II only.A.I, II, and III.B.II and III.C.I and II.D.

Answer (A) is incorrect because oral verification also would address the problem.Answer (B) is incorrect because assigning a sequential number to the customer’s order helps build an audit trail butdoes not address the product identification issue.Answer (C) is correct. A self-checking digit detects incorrect codes. The digit is generated by applying an algorithmto the code. During input, the digit is recomputed by applying the algorithm to the code actually entered. Oralverification also addresses the problem of incorrectly identifying the product number. Assigning a sequential numberto the customer’s order helps build an audit trail but does not address the product identification issue.Answer (D) is incorrect because assigning a sequential number to the customer’s order helps build an audit trail butdoes not address the product identification issue.

Gleim's CIA Test Prep: Part III: Business Analysis and Information Technology(1165 questions)

Copyright 2004 Gleim Publications, Inc. Page 387Printed for g j

Page 388: Part three cia_with_ answers

[962] Gleim #: 9.41 -- Source: CIA 596 I-57

Which one of the following input controls or edit checks would catch certain types of errors within the payment amountfield of a transaction?

Record count.A.Echo check.B.Check digit.C.Limit check.D.

Answer (A) is incorrect because a record count determines the number of documents entered into a process.Answer (B) is incorrect because an echo check tests the reliability of computer hardware. For example, the CPUsends a signal to a printer that is echoed just prior to printing. The signal verifies that the proper print position hasbeen activated.Answer (C) is incorrect because a self-checking number is generated by applying an algorithm to an identificationnumber.Answer (D) is correct. A limit, reasonableness, or range test determines whether an amount is within a predeterminedlimit for given information. It can only detect certain errors (i.e., those that exceed the acceptable limit).

[963] Gleim #: 9.42 -- Source: CIA 582 IV-12

The key verification process associated with keying computer records for input to a computer system is

Effectively used to detect the erroneous recording of data on source documents.A.Inexpensive and therefore widely used.B.Used to detect errors introduced by the keying process.C.Ordinarily used with a computer program written to check the data.D.

Answer (A) is incorrect because key verification does not detect errors in the source documents.Answer (B) is incorrect because, although widely used, key verification effectively doubles the work and isexpensive.Answer (C) is correct. Key verification is a procedure to determine if the keying process was performed properly.Information from source documents is rekeyed on a special keyboard by another operator and compared with thatpreviously recorded.Answer (D) is incorrect because key verification is a manual process.

[964] Gleim #: 9.43 -- Source: CIA 597 III-37

Output controls ensure that the results of computer processing are accurate, complete, and properly distributed. Which ofthe following is not a typical output control?

Reviewing the computer processing logs to determine that all of the correct computer jobs executed properly.A.Matching input data with information on master files and placing unmatched items in a suspense file.B.Periodically reconciling output reports to make sure that totals, formats, and critical details are correct and agree withinput.

C.

Maintaining formal procedures and documentation specifying authorized recipients of output reports, checks, or othercritical documents.

D.

Answer (A) is incorrect because reviewing the computer processing logs is an output control.Answer (B) is correct. Output controls often include comparing output totals with input and processing totals;reviewing computer logs; auditing output reports to ensure that totals, formats, and details are accurate andreconcilable; and specifying authorized recipients by formal means. The data control group also performs importantoutput control functions. Matching the input data with information held on master or suspense files is a processingcontrol, not an output control. It ensures that data are complete and accurate during updating.Answer (C) is incorrect because periodically reconciling output reports is an output control.Answer (D) is incorrect because maintaining formal procedures and documentation specifying authorized recipientsis an output control.

[965] Gleim #: 9.44 -- Source: CIA 1195 III-36

The marketing department’s proposal was finally accepted, and the marketing employees attended a class in using themainframe report writer. Soon, the marketing analysts found that it was easier to download the data and manipulate it ontheir own microcomputers than to perform all the data manipulation with the mainframe report writer directly. One analystbecame highly skilled at downloading and wrote downloading command sequences for the other employees. When theanalyst left the company for a better job, the department had problems making modifications to these command sequences.The department’s problems are most likely due to inadequate

Gleim's CIA Test Prep: Part III: Business Analysis and Information Technology(1165 questions)

Copyright 2004 Gleim Publications, Inc. Page 388Printed for g j

Page 389: Part three cia_with_ answers

Documentation.A.Data backup.B.Program testing.C.Anti-virus software.D.

Answer (A) is correct. One risk of end-user computing is that documentation may be poor and that important knowledgemay be limited to one person. The command sequences should have been documented so that other analysts could use andmodify them readily.Answer (B) is incorrect because the inability of other analysts to understand the command sequences is not a function ofinadequate data backup procedures.Answer (C) is incorrect because the inability of other analysts to understand the command sequences is not a function ofinadequate testing.Answer (D) is incorrect because the inability of other analysts to understand the command sequences is not a function ofinadequate use of anti-virus software.

[966] Gleim #: 9.45 -- Source: CIA 1195 III-34

After using the mainframe report writer for several months, the marketing analysts gained confidence in using it, but themarketing department manager became concerned. Whenever analysts revised reports they had written earlier, the codingerrors kept reappearing in their command sequences. The manager was sure that all the analysts knew what the errors wereand how to avoid them. The most likely cause of the reappearance of the same coding errors is inadequate

Backups.A.Change control.B.Access control.C.Testing.D.

Answer (A) is incorrect because inadequate backups are not the cause of reuse of erroneous code.Answer (B) is correct. Change control manages changes in information system resources and procedures. It includesa formal change request procedure; assessments of change requests on technical and business grounds; schedulingchanges; testing, installing, and monitoring changes; and reporting the status of recorded changes. The analysts werereusing erroneous code that should have been but was not corrected.Answer (C) is incorrect because inadequate access control is not the cause of reuse of erroneous code.Answer (D) is incorrect because inadequate testing is not the cause of reuse of erroneous code.

[967] Gleim #: 9.46 -- Source: CIA 593 III-46

A control for ensuring that the source code and the executable code for a program match is

Verifying that the program move request is authorized.A.Requiring program, system, and parallel testing of the code.B.Authorizing programmer access to test libraries only.C.Recompiling source code into the production load library.D.

Answer (A) is incorrect because verifying that the program move request is authorized ensures that a change wasauthorized, not that the source and executable code match.Answer (B) is incorrect because requiring program, system, and parallel testing of the code ensures that the codemeets test specifications, not that the source and executable code match.Answer (C) is incorrect because authorizing programmer access to test libraries only ensures that programmers do nothave access to production libraries, not that the source and executable code match.Answer (D) is correct. Recompiling source code into the production load library ensures that the source andexecutable codes match because the executable code is created from the source code.

[968] Gleim #: 9.47 -- Source: CIA 1196 III-36

Traditional information systems development procedures that ensure proper consideration of controls may not be followedby users developing end-user computing (EUC) applications. Which of the following is a prevalent risk in the developmentof EUC applications?

Gleim's CIA Test Prep: Part III: Business Analysis and Information Technology(1165 questions)

Copyright 2004 Gleim Publications, Inc. Page 389Printed for g j

Page 390: Part three cia_with_ answers

Management decision making may be impaired due to diminished responsiveness to management’s requests forcomputerized information.

A.

Management may be less capable of reacting quickly to competitive pressures due to increased application developmenttime.

B.

Management may place the same degree of reliance on reports produced by EUC applications as it does on reports producedunder traditional systems development procedures.

C.

Management may incur increased application development and maintenance costs for EUC systems, compared withtraditional (mainframe) systems.

D.

Answer (A) is incorrect because EUC systems typically increase flexibility and responsiveness to management’sinformation requests. Such systems are more easily modified.Answer (B) is incorrect because EUC systems typically reduce application development cycle time.Answer (C) is correct. End-user developed applications may not be subject to an independent outside review by systemsanalysts and are not created in the context of a formal development methodology. These applications may lack appropriatestandards, controls, quality assurance procedures, and documentation. A risk of end-user applications is that managementmay rely on them as much as traditional applications.Answer (D) is incorrect because EUC systems typically result in reduced application development and maintenance costs.

[969] Gleim #: 9.48 -- Source: CIA 1196 III-37

Traditional information systems development and operational procedures typically involve four functional areas. Thesystems analysis function focuses on identifying and designing systems to satisfy organizational requirements. Theprogramming function is responsible for the design, coding, testing, and debugging of computer programs necessary toimplement the systems designed by the analysis function. The computer operations function is responsible for datapreparation, program/job execution, and system maintenance. The user function provides the input and receives the outputof the system. Which of these four functions is often poorly implemented or improperly omitted in the development of anew end-user computing (EUC) application?

Systems analysis function.A.Programming function.B.Computer operations function.C.User function.D.

Answer (A) is correct. Systems analysis is one step that is not absolutely required in the development of a system.The desire to produce a system quickly may result in this step being eliminated or poorly implemented. A system isoften produced and then analyzed to see if it will satisfy the needs of the organization. In an EUC application, thesystems analysis is often incomplete or omitted.Answer (B) is incorrect because, without programming, there would be no system.Answer (C) is incorrect because, without computer operations, the system would not be able to do anything.Answer (D) is incorrect because, without users, there would be no need for the system.

[970] Gleim #: 9.49 -- Source: CIA 1194 I-37

Responsibility for the control of end-user computing exists at the organizational, departmental, and individual user level.Which of the following should be a direct responsibility of the individual users?

Acquisition of hardware and software.A.Taking equipment inventories.B.Strategic planning of end-user computing.C.Physical security of equipment.D.

Answer (A) is incorrect because the acquisition of hardware and software is an organizational- and departmental-levelresponsibility.Answer (B) is incorrect because taking equipment inventories is an organizational-level responsibility.Answer (C) is incorrect because strategic planning is an organizational- and departmental-level responsibility.Answer (D) is correct. End-user computing involves user-created or -acquired systems that are maintained andoperated outside of traditional information systems controls. In this environment, an individual user is ordinarilyresponsible for the physical security of the equipment (s)he uses.

[971] Gleim #: 9.50 -- Source: CIA 586 II-39

Which of the following represents a limitation on the use of generalized audit software (GAS)?

Gleim's CIA Test Prep: Part III: Business Analysis and Information Technology(1165 questions)

Copyright 2004 Gleim Publications, Inc. Page 390Printed for g j

Page 391: Part three cia_with_ answers

It requires lengthy detailed instructions in order to accomplish specific tasks.A.It has limited application without significant modification.B.It requires significant programming knowledge to be used effectively.C.It can only be used on hardware with compatible operating systems.D.

Answer (A) is incorrect because the use of GAS is normally more efficient. Less time is required to write instructions toaccomplish a function than to manually select and examine items.Answer (B) is incorrect because the program is generalized, i.e., designed to be used on a variety of systems withoutsignificant modifications.Answer (C) is incorrect because an advantage is that GAS requires minimal knowledge of computer technology.Answer (D) is correct. Diversity of programming languages, computers, systems designs, and differing data structuresmakes generalized audit software impossible to apply in certain situations.

[972] Gleim #: 9.51 -- Source: Publisher

Specialized audit software

Is written to interface with many different client systems.A.May be written while its purposes and users are being defined.B.Requires the auditor to have less computer expertise than generalized audit software.C.May be written in a procedure-oriented language.D.

Answer (A) is incorrect because generalized audit software is written to interface with many different client systems.Answer (B) is incorrect because the purposes and users of this software must be defined before it is written.Answer (C) is incorrect because generalized audit software purchased “off the shelf” requires less computer expertisethan specialized software created by the auditor.Answer (D) is correct. Specialized audit software is written in a procedure- or problem-oriented language to fulfill aspecific set of audit tasks. The purposes and users of the software are well defined before the software is written.Auditors develop specialized audit software for the following reasons:

Unavailability of alternative software1.Functional limitations of alternative software2.Efficiency considerations3.Increased understanding of systems4.Opportunity for easy implementation5.Increased auditor independence and prestige6.

[973] Gleim #: 9.52 -- Source: CPA 1188 A-57

The two requirements crucial to achieving audit efficiency and effectiveness with a personal computer are selecting

The appropriate audit tasks for personal computer applications and the appropriate software to perform the selectedaudit tasks.

A.

The appropriate software to perform the selected audit tasks and data that can be accessed by the auditor’s personalcomputer.

B.

Company data that can be accessed by the auditor’s personal computer and the appropriate audit tasks for personalcomputer applications.

C.

The appropriate sample of company data to test with the auditor’s personal computer and the appropriate software toperform the selected audit tasks.

D.

Answer (A) is correct. The question relates to using the computer as an audit tool. To use a personal computer forthis purpose effectively and efficiently, the auditor must have the appropriate hardware and software.Answer (B) is incorrect because access to data does not relate directly to the efficient and effective use of a personalcomputer.Answer (C) is incorrect because access to data does not relate directly to the efficient and effective use of a personalcomputer.Answer (D) is incorrect because the selection of the appropriate sample size does not apply when using softwareapplications because the entire population can be tested.

[974] Gleim #: 9.53 -- Source: CPA 593 A-40

An auditor is least likely to use computer software to

Gleim's CIA Test Prep: Part III: Business Analysis and Information Technology(1165 questions)

Copyright 2004 Gleim Publications, Inc. Page 391Printed for g j

Page 392: Part three cia_with_ answers

Construct parallel simulations.A.Access client data files.B.Prepare spreadsheets.C.Assess computer control risk.D.

Answer (A) is incorrect because parallel simulation involves using an auditor’s program to reproduce the logic ofmanagement’s program.Answer (B) is incorrect because computer software makes accessing company files much faster and easier.Answer (C) is incorrect because many audit spreadsheet programs are available.Answer (D) is correct. The auditor is required to evaluate the adequacy and effectiveness of the system of internal controland to assess risk to plan the audit. This assessment is a matter of professional judgment that cannot be accomplished with acomputer.

[975] Gleim #: 9.54 -- Source: CIA 586 I-34

When an auditor performs tests on a computerized inventory file containing over 20,000 line items, that auditor canmaintain independence and perform most efficiently by

Asking the console operator to print every item that cost more than $100.A.Using a generalized audit software package.B.Obtaining a printout of the entire file and then selecting each nth item.C.Using the systems department’s programmer to write an extraction program.D.

Answer (A) is incorrect because independence is jeopardized when an operator is involved in the process.Answer (B) is correct. Independence can be preserved when the auditor acquires general audit software (GAS) froman external source rather than relying on auditee-developed audit software. Also, efficiency is enhanced to the extentGAS can be used (as compared to manual auditing or writing special audit programs).Answer (C) is incorrect because printing out the entire file is both unnecessary and inefficient.Answer (D) is incorrect because overreliance on an auditee’s programmer impairs independence.

[976] Gleim #: 9.55 -- Source: Publisher

Which of the following cannot be performed by an auditor using computer-assisted audit techniques (CAATs) software?

Identifying missing check numbers.A.Extracting data files containing only a two-digit year date field and changing it to hold four digits.B.Matching identical product information in separate data files.C.Aging accounts receivable.D.

Answer (A) is incorrect because identifying gaps is a function of major CAATs software packages.Answer (B) is correct. CAATs software has the ability to extract data files containing specified criteria, such as thesize of a data field, but it does not enable the auditor to change the format of this information within management’scomputer system. However, the auditor can specify the format of data fields in CAATs software-produced reports.Answer (C) is incorrect because merging files is a function of major CAATs software packages.Answer (D) is incorrect because aging is a function of major CAATs software packages.

[977] Gleim #: 9.56 -- Source: Publisher

Which of the following is not true about audit use of the Internet?

It is a useful research tool for gathering audit-related information.A.It provides a secure medium to transmit confidential information.B.Electronic communication is the major use of the Internet by internal auditors.C.An electronic record of a user’s web browsing activities is created.D.

Gleim's CIA Test Prep: Part III: Business Analysis and Information Technology(1165 questions)

Copyright 2004 Gleim Publications, Inc. Page 392Printed for g j

Page 393: Part three cia_with_ answers

Answer (A) is incorrect because the Internet is a useful audit tool for gathering and disseminating audit-related information.Answer (B) is correct. Users transmitting sensitive information across the Internet must understand the threats that arisethat could compromise the confidentiality of the data. Security measures, such as encryption technology, need to be taken toensure that the information is viewed only by those authorized to view it.Answer (C) is incorrect because the major use of the Internet by internal auditors is electronic communication.Answer (D) is incorrect because web browsing leaves an electronic record of the user’s search path.

[978] Gleim #: 9.57 -- Source: CIA 1196 III-61

A company has a very large, widely dispersed internal auditing department. Management wants to implement acomputerized system to facilitate communications among auditors. The specifications require that each auditor have theability to send, file, and answer messages at his/her convenience. Transmission of messages must be quick and thepreparation of messages must not be costly. Which type of system would best meet these specifications?

Utility program.A.Electronic bulletin board system (BBS).B.Electronic mail system.C.Private branch exchange (PBX).D.

Answer (A) is incorrect because utility programs are provided by manufacturers of equipment to perform routineprocessing tasks.Answer (B) is incorrect because an electronic bulletin board system is a centralized information source and messageswitching system and therefore does not provide a repository to store messages for each auditor.Answer (C) is correct. Electronic mail permits transfer, receipt, and storage of messages within or between computersystems. The “mail” consists of electronically transmitted messages. A user’s “mailbox” is the storage area allocatedfor messages. The advantages of electronic mail are high-speed transmission, reduction of message preparation costs,and the possibility of sending or reading messages at a convenient time.Answer (D) is incorrect because a PBX is a telecommunications system that routes calls to particular extensionswithin an organization.

[979] Gleim #: 9.58 -- Source: CIA 1193 II-23

Image processing can be used to convert paper documents into electronic images. Which of the following conceptsdistinguishes the retention of computerized audit documents from the traditional hard copy form?

Analyses, conclusions, and recommendations are filed on electronic media and are therefore subject to computersystem controls and security procedures.

A.

Evidential support for all findings is copied and provided to local management during the closing conference and toeach person receiving the final report.

B.

Computerized data files can be used in computer audit procedures.C.Audit programs can be standardized to eliminate the need for a preliminary survey at each location.D.

Answer (A) is correct. The only difference between the computerized and hard copy form is how the working papersare stored. Electronic working papers are saved on either disks or hard drive, whereas hard copies are stored in a filecabinet. Unlike computerized working papers, hard copies are not subject to computer controls and securityprocedures.Answer (B) is incorrect because evidential support is retained and provided on the basis of the nature of the findingand not the media used for storing working papers.Answer (C) is incorrect because this capability is not an exclusive function of computerized working papers.Answer (D) is incorrect because, though the nature of the preliminary survey may change in some cases, therequirement for this phase of the audit is not eliminated by computerized working papers.

[980] Gleim #: 9.59 -- Source: Publisher

Which of the following issues would be of most concern to an auditor relating to an organization’s Internet security policy?

Auditor documentation.A.System efficiency.B.Data integrity.C.Rejected and suspense item controls.D.

Gleim's CIA Test Prep: Part III: Business Analysis and Information Technology(1165 questions)

Copyright 2004 Gleim Publications, Inc. Page 393Printed for g j

Page 394: Part three cia_with_ answers

Answer (A) is incorrect because auditor documentation is not as crucial as data integrity.Answer (B) is incorrect because efficiency does not affect the basis for critical auditor decisions using information providedby the system.Answer (C) is correct. Controls are intended to ensure the integrity, confidentiality, and availability of information. Anauditor relies on the integrity of the system’s data and programs in making critical decisions throughout the audit process.Answer (D) is incorrect because rejected and suspense item controls represent a portion of the techniques used to ensuredata integrity.

[981] Gleim #: 9.60 -- Source: CIA 1195 I-31

Management’s enthusiasm for computer security seems to vary with changes in the environment, particularly theoccurrence of other computer disasters. Which of the following concepts should be addressed when making acomprehensive recommendation regarding the costs and benefits of computer security?

Potential loss if security is not implementedI.Probability of occurrencesII.Cost and effectiveness of the implementation and operation of computer securityIII.

I only.A.I and II only.B.III only.C.I, II, and III.D.

Answer (A) is incorrect because potential loss, the probability thereof, and the cost and effectiveness of securitymeasures are important elements of the analysis.Answer (B) is incorrect because potential loss, the probability thereof, and the cost and effectiveness of securitymeasures are important elements of the analysis.Answer (C) is incorrect because potential loss, the probability thereof, and the cost and effectiveness of securitymeasures are important elements of the analysis.Answer (D) is correct. Potential loss is the amount of dollar damages associated with a security problem or loss ofassets. Potential loss times the probability of occurrence is an estimate (expected value) of the exposure associatedwith lack of security. It represents a potential benefit associated with the implementation of security measures. Toperform a cost-benefit analysis, the costs should be considered. Thus, all three items need to be addressed.

Gleim's CIA Test Prep: Part III: Business Analysis and Information Technology(1165 questions)

Copyright 2004 Gleim Publications, Inc. Page 394Printed for g j

Page 395: Part three cia_with_ answers

[982] Gleim #: 9.61 -- Source: CIA 1195 I-28As organizations become more computer integrated, management is becoming increasingly concerned with the quality ofaccess controls to the computer system. Which of the following provides the most accountability?

  Option I Option II Option III Option IV Restrict accessby: Individuals Groups Individuals Departments

         Identify Field Workstation Workstation Individualcomputer level record leveldata at:

         Restrict Need to Right to Normal Items identifiedaccess: know know processing as processed

      by employee by department      type         Identify Password Password Key access to Departmentalusers by: workstation, password

      or password       on workstation         Limit Delete, Add or Add, delete, Add, delete,ability to: add, or delete or modify or modify data

  modify files data stored normally  data at workstation processed by      department

Option I.A.Option II.B.Option III.C.Option IV.D.

Answer (A) is correct. Access should be limited to those whose activities necessitate access to the computer system.Moreover, the degree of access allowed should be consistent with an individual’s responsibilities. Restricting accessto particular individuals rather than groups or departments clearly establishes specific accountability. Not everyone ina group will need access or the same degree of access. Thus, passwords assigned to individuals should be required foridentification of users by the system. Furthermore, data should be restricted at the field level, not the workstationlevel. It may be possible to limit access to a workstation, but most workstations are connected to larger mainframe ornetwork databases. Thus, the security at the workstation level only would be insufficient.Answer (B) is incorrect because access should be restricted to particular individuals on a need-to-know basis, datashould be restricted at the field level, and use should be limited to necessary functions performed by the accountableindividual.Answer (C) is incorrect because access should be restricted to particular individuals on a need-to-know basis, datashould be restricted at the field level, and use should be limited to necessary functions performed by the accountableindividual.Answer (D) is incorrect because access should be restricted to particular individuals on a need-to-know basis, datashould be restricted at the field level, and use should be limited to necessary functions performed by the accountableindividual.

[983] Gleim #: 9.62 -- Source: J. Brooks

Which of the following is a control that will prevent accessing the accounts receivable files from a hardwired terminallocated in a manufacturing department?

An echo check.A.A device authorization table.B.Providing only dial-up terminals.C.Using data encryption.D.

Gleim's CIA Test Prep: Part III: Business Analysis and Information Technology(1165 questions)

Copyright 2004 Gleim Publications, Inc. Page 395Printed for g j

Page 396: Part three cia_with_ answers

Answer (A) is incorrect because an echo check relates to the accuracy of signals sent from or to a terminal.Answer (B) is correct. A device authorization table restricts file access to those physical devices that logically need access.Because it is illogical for anyone to access the accounts receivable file from a manufacturing terminal, the deviceauthorization table will deny access even when a valid password is used.Answer (C) is incorrect because dial-up terminals provide less security than hardwired terminals. Any terminal may dial intothe communications port using public telephones.Answer (D) is incorrect because although data encryption (transmitting data in code form) might make the data unusable, itwould not prevent access.

[984] Gleim #: 9.63 -- Source: CIA 592 III-27

An equipment manufacturer maintains dial-up ports into its order-entry system for the convenience of its customersworldwide so they may order parts as they need them. The manufacturer promises 48-hour delivery anywhere in the worldfor 95% of these parts orders. Because of the cost and sensitive nature of certain electronic parts, the manufacturer needs tomaintain secure access to its order-entry system. The best technique for monitoring the security of access is

Integrated test facility for the order-entry system.A.Tracing of transactions through the order-entry system.B.Transaction selection of order-entry transactions.C.Logging of unsuccessful access attempts.D.

Answer (A) is incorrect because use of an integrated test facility (ITF) is a technique by which an auditor selectstransactions and processing functions and applies the transactions to a fictitious entity during a normal processingcycle along with regular transactions. This technique cannot determine whether the data themselves are legitimate.Answer (B) is incorrect because tracing follows the path of a transaction during processing but is inadequate todetermine whether a transaction is legitimate.Answer (C) is incorrect because transaction selection uses an independent computer program to monitor and selecttransactions for internal audit review. Like tracing, it fails to determine whether a transaction is legitimate. It wouldbe an appropriate technique to apply to transactions suspected to be illegitimate.Answer (D) is correct. An access log should be used to record all attempts to use the system. The date and time,codes used, mode of access, and data involved are recorded. The system should monitor unsuccessful attemptsbecause repeated attempts could suggest that someone is trying random or patterned character sequences in order toidentify a password.

[985] Gleim #: 9.64 -- Source: CIA 593 II-23

Data access security related to applications may be enforced through all the following except

User identification and authentication functions incorporated in the application.A.Utility software functions.B.User identification and authentication functions in access control software.C.Security functions provided by a database management system.D.

Answer (A) is incorrect because, although there is a migration of control of this type away from applications to othersoftware, the large bulk of these controls still reside in application software.Answer (B) is correct. Utility programs perform routine functions (e.g., sorting and copying), are available to allusers, and are promptly available for many different applications. Utility programs are one of the more seriousweaknesses in data access security because some can bypass normal access controls.Answer (C) is incorrect because access control software has as one of its primary objectives improving data accesssecurity for all data on the system.Answer (D) is incorrect because most database management systems provide for improved data access security whilethey are running.

[986] Gleim #: 9.65 -- Source: CIA 1195 I-36

Most organizations are concerned about the potential compromise of passwords. Which of the following procedures wouldbe the most effective in controlling against a perpetrator obtaining someone else’s password?

Allow only the users to change their passwords and encourage them to change passwords frequently.A.Implement a computer program that tests to see that the password is not easily guessed.B.Implement the use of “see-through” authentication techniques whereby the user uses a card to generate a passwordand verifies both the key and the generated password to the system.

C.

Limit password authorization to time of day and location.D.

Gleim's CIA Test Prep: Part III: Business Analysis and Information Technology(1165 questions)

Copyright 2004 Gleim Publications, Inc. Page 396Printed for g j

Page 397: Part three cia_with_ answers

Answer (A) is incorrect because users often choose passwords that are easily guessed.Answer (B) is incorrect because a program to test passwords is useful but less effective than see-through authentication.Answer (C) is correct. See-through authentication techniques, such as the one described, require the user to have two of thethree important elements to authenticate oneself to the system, i.e., a possession (the card used to generate the password),knowledge (the new password), or a personal characteristic (e.g., fingerprints).Answer (D) is incorrect because limiting access to times and locations is helpful in certain environments but not when thesystem allows dial-up access.

[987] Gleim #: 9.66 -- Source: CIA 594 I-32

Passwords for personal computer software programs are designed to prevent

Inaccurate processing of data.A.Unauthorized access to the computer.B.Incomplete updating of data files.C.Unauthorized use of the software.D.

Answer (A) is incorrect because passwords concern authorization, not accuracy of data.Answer (B) is incorrect because passwords do not prevent physical access to the computer.Answer (C) is incorrect because passwords concern authorization, not completeness of data.Answer (D) is correct. The use of passwords is an effective control in an online system to prevent unauthorizedaccess to computer files. Lists of authorized users are maintained in the computer. The entry of passwords or IDnumbers; a prearranged act of personal questions; and use of badges, magnetic cards, or optically scanned cards maybe combined to avoid unauthorized access.

[988] Gleim #: 9.67 -- Source: CIA 1195 III-32

The primary objective of security software is to

Control access to information system resources.A.Restrict access to prevent installation of unauthorized utility software.B.Detect the presence of viruses.C.Monitor the separation of duties within applications.D.

Answer (A) is correct. The objective of security software is to control access to information system resources, such asprogram libraries, data files, and proprietary software. Security software identifies and authenticates users, controlsaccess to information, and records and investigates security related events and data.Answer (B) is incorrect because security software will control the use of utilities, not their installation.Answer (C) is incorrect because antivirus software detects the presence of viruses.Answer (D) is incorrect because security software may be a tool to establish, but does not monitor, separation ofduties.

[989] Gleim #: 9.68 -- Source: CIA 597 I-5

A controller became aware that a competitor appeared to have access to the company’s pricing information. The internalauditor determined that the leak of information was occurring during the electronic transmission of data from branchoffices to the head office. Which of the following controls would be most effective in preventing the leak of information?

Asynchronous transmission.A.Encryption.B.Use of fiber-optic transmission lines.C.Use of passwords.D.

Answer (A) is incorrect because asynchronous transmission is a method of data transmission, not a means ofsafeguarding data. It is used for slow, irregular transmissions, such as from a keyboard terminal. Each character ismarked by a start and stop code.Answer (B) is correct. Encryption software uses a fixed algorithm to manipulate plain text and an encryption key (aset of random data bits used as a starting point for application of the algorithm) to introduce variation. Although datamay be accessed by tapping into the transmission line, the encryption key is necessary to understand the data beingsent.Answer (C) is incorrect because, although fiber-optic transmission lines are difficult to tap, their use will not preventtheft of unencrypted data by someone who has access to them.Answer (D) is incorrect because use of passwords will control access at the sending location and the head-officecomputer. However, passwords will not prevent someone from tapping the transmission line.

Gleim's CIA Test Prep: Part III: Business Analysis and Information Technology(1165 questions)

Copyright 2004 Gleim Publications, Inc. Page 397Printed for g j

Page 398: Part three cia_with_ answers

[990] Gleim #: 9.69 -- Source: CIA 1196 III-78

The use of message encryption software

Guarantees the secrecy of data.A.Requires manual distribution of keys.B.Increases system overhead.C.Reduces the need for periodic password changes.D.

Answer (A) is incorrect because no encryption approach absolutely guarantees the secrecy of data.Answer (B) is incorrect because keys may also be distributed electronically via secure key transporters.Answer (C) is correct. Encryption software uses a fixed algorithm to manipulate plain text and an encryption key (aset of random data bits used as a starting point for application of the algorithm) to introduce variation. The machineinstructions necessary to encrypt and decrypt data constitute system overhead. As a result, processing speed may beslowed.Answer (D) is incorrect because periodic password changes are needed. Passwords are the typical means of validatingusers’ access to unencrypted data.

[991] Gleim #: 9.70 -- Source: CIA 1190 I-34

All administrative and professional staff in a corporate legal department prepare documents on terminals connected to ahost LAN file server. The best control over unauthorized access to sensitive documents in the system is

Required entry of passwords for access to the system.A.Physical security for all disks containing document files.B.Periodic server backup and storage in a secure area.C.Required entry of passwords for access to individual documents.D.

Answer (A) is incorrect because password security for access to the system permits all departmental employees accessto all documents in the system.Answer (B) is incorrect because this system uses no floppy disks.Answer (C) is incorrect because periodic server backup and storage in a secure area is a good security/backupprocedure, but it would not prevent access to sensitive documents online.Answer (D) is correct. Different passwords may be required to access the system, to read certain files, and to performcertain other functions. Required entry of passwords for access to individual documents is the best single control overunauthorized access to sensitive documents.

[992] Gleim #: 9.71 -- Source: CIA 1196 III-62

An auditor has just completed a physical security audit of a data center. Because the center engages in top-secret defensecontract work, the auditor has chosen to recommend biometric authentication for workers entering the building. Therecommendation might include devices that verify all of the following except

Fingerprints.A.Retina patterns.B.Speech patterns.C.Password patterns.D.

Answer (A) is incorrect because authentication using fingerprints is a biometric measure.Answer (B) is incorrect because authentication using retina patterns is a biometric measure.Answer (C) is incorrect because authentication using speech patterns is a biometric measure.Answer (D) is correct. Biometric technologies are automated methods of establishing an individual’s identity usingphysiological or behavioral traits. These characteristics include fingerprints, retina patterns, hand geometry, signaturedynamics, speech, and keystroke dynamics.

[993] Gleim #: 9.72 -- Source: Publisher

An Internet firewall is designed to provide adequate protection against which of the following?

Gleim's CIA Test Prep: Part III: Business Analysis and Information Technology(1165 questions)

Copyright 2004 Gleim Publications, Inc. Page 398Printed for g j

Page 399: Part three cia_with_ answers

A computer virus.A.Unauthenticated logins from outside users.B.Insider leaking of confidential information.C.A Trojan horse application.D.

Answer (A) is incorrect because a firewall cannot adequately protect a system against computer viruses.Answer (B) is correct. A firewall is a device that separates two networks and prevents passage of specific types of networktraffic while maintaining a connection between the networks. Generally, an Internet firewall is designed to protect a systemfrom unauthenticated logins from outside users, although it may provide several other features as well.Answer (C) is incorrect because industrial spies need not leak information through the firewall. A telephone or floppy diskare much more common means of sharing confidential information.Answer (D) is incorrect because a firewall cannot adequately protect against a Trojan horse (a program, such as a game, thatappears friendly but that actually contains applications destructive to the computer system) or any other program that can beexecuted in the system by an internal user.

[994] Gleim #: 9.73 -- Source: CIA 1195 III-37

An organization installed antivirus software on all its personal computers. The software was designed to prevent initialinfections, stop replication attempts, detect infections after their occurrence, mark affected system components, and removeviruses from infected components. The major risk in relying on antivirus software is that antivirus software may

Not detect certain viruses.A.Make software installation overly complex.B.Interfere with system operations.C.Consume too many system resources.D.

Answer (A) is correct. Antivirus software designed to identify and remove known viruses is sometimes known as avaccine. A vaccine works only for known viruses and may not be completely effective for variants of those viruses.Answer (B) is incorrect because having antivirus software is unlikely to make software installation overly complex.Answer (C) is incorrect because antivirus software need not interfere with system operations. Its execution can bescheduled in advance so as not to interfere with running programs.Answer (D) is incorrect because antivirus software can be set to execute at times when it would not consume toomany system resources, e.g., at startup.

[995] Gleim #: 9.74 -- Source: CIA 1193 I-32

Each day, after all processing is finished, a bank performs a backup of its online deposit files and retains it for 7 days.Copies of each day’s transaction files are not retained. This approach is

Valid, in that having a week’s worth of backups permits recovery even if one backup is unreadable.A.Risky, in that restoring from the most recent backup file would omit subsequent transactions.B.Valid, in that it minimizes the complexity of backup/recovery procedures if the online file has to be restored.C.Risky, in that no checkpoint/restart information is kept with the backup files.D.

Answer (A) is incorrect because the practice of not retaining daily transaction data is unsound in that the bank loses aday’s transactions for each backup that is unreadable.Answer (B) is correct. At appropriate intervals, the disk files should be copied on magnetic tape so that restartprocedures can begin at those points if data are lost or destroyed. However, not retaining each day’s transaction filesis risky because information processed since the last backup file was created will be lost.Answer (C) is incorrect because the practice of not retaining daily transaction data certainly minimizes complexitybut at the expense of losing transaction data if the online file must be restored from the backup.Answer (D) is incorrect because checkpoint/restart information is not needed. The backups are created after allprocessing is finished for the day.

[996] Gleim #: 9.75 -- Source: CIA 589 II-25

A company updates its accounts receivable master file weekly and retains the master files and corresponding updatetransactions for the most recent 2-week period. The purpose of this practice is to

Gleim's CIA Test Prep: Part III: Business Analysis and Information Technology(1165 questions)

Copyright 2004 Gleim Publications, Inc. Page 399Printed for g j

Page 400: Part three cia_with_ answers

Verify run-to-run control totals for receivables.A.Match internal labels to avoid writing on the wrong volume.B.Permit reconstruction of the master file if needed.C.Validate groups of update transactions for each version.D.

Answer (A) is incorrect because comparison of batch totals is a control over the completeness of processing, not a recoveryprocedure.Answer (B) is incorrect because internal labels may avoid destruction of data but do not aid in recovery.Answer (C) is correct. The grandfather-father-son approach normally employs magnetic tapes to furnish backup in a batchprocessing system. The procedure involves creation and retention of three generations of master files so that lost ordestroyed data may be regenerated from the remaining master files and transaction data. In this case, a master file (thegrandfather) and the first week’s transactions are used to generate a second master file (the father). This file and the secondweek’s transactions are the basis for the current master file (the son). Online systems employ rollback and recoveryprocedures; i.e., the master file is periodically dumped onto a storage medium. Reconstruction is then possible using thebackup copy and the transactions log.Answer (D) is incorrect because validation may avoid destruction of data but does not aid in recovery.

[997] Gleim #: 9.76 -- Source: CIA 596 III-39

Good planning will help an organization restore computer operations after a processing outage. Good recovery planningshould ensure that

Backup/restart procedures have been built into job streams and programs.A.Change control procedures cannot be bypassed by operating personnel.B.Planned changes in equipment capacities are compatible with projected workloads.C.Service level agreements with owners of applications are documented.D.

Answer (A) is correct. The disaster plan should embrace data center recovery, critical application recovery, andnetwork recovery. It should be updated and current with regard to recent test results and new applications, equipment,and network configurations. The plan should also ensure that backup facilities are still able to process criticalapplications and that end-user responsibility is established. Another essential component of a disaster recovery plan isthat backup/restart procedures have been anticipated and provided for in the application systems.Answer (B) is incorrect because whether change control procedures can be bypassed is not usually a consideration indisaster recovery planning.Answer (C) is incorrect because planned rather than actual changes in equipment capacities are not relevant indisaster recovery planning.Answer (D) is incorrect because ensuring that service level agreements with owners of critical applications areadequate is not a function of disaster recovery planning.

[998] Gleim #: 9.77 -- Source: CIA 593 I-42

Contingency plans for information systems should include appropriate backup agreements. Which of the followingarrangements would be considered too vendor-dependent when vital operations require almost immediate availability ofcomputer resources?

A “hot site” arrangement.A.A “cold site” arrangement.B.A “cold and hot site” combination arrangement.C.Using excess capacity at another data center within the organization.D.

Answer (A) is incorrect because a “hot site” has all needed assets in place and is not vendor-dependent.Answer (B) is correct. Organizations should maintain contingency plans for operations in the case of a disaster.These plans usually include off-site storage of important backup data and an arrangement for the continuation ofoperations at another location. A “cold site” has all needed assets in place except the needed computer equipment andis vendor-dependent for timely delivery of equipment.Answer (C) is incorrect because a “cold and hot site” combination allows the “hot site” to be used until the “coldsite” is prepared and is thus not too vendor-dependent.Answer (D) is incorrect because excess capacity would ensure that needed assets are available and would not bevendor-dependent.

Gleim's CIA Test Prep: Part III: Business Analysis and Information Technology(1165 questions)

Copyright 2004 Gleim Publications, Inc. Page 400Printed for g j

Page 401: Part three cia_with_ answers

[999] Gleim #: 9.78 -- Source: CPA 1188 A-36Which of the following characteristics distinguishes computer processing from manual processing?

Computer processing virtually eliminates the occurrence of computational error normally associated with manualprocessing.

A.

Errors or fraud in computer processing will be detected soon after their occurrence.B.The potential for systematic error is ordinarily greater in manual processing than in computerized processing.C.Most computer systems are designed so that transaction trails useful for audit purposes do not exist.D.

Answer (A) is correct. Computer processing uniformly subjects like transactions to the same processing instructions.A computer program defines the processing steps to accomplish a task. Once the program is written and testedappropriately, it will perform the task repetitively and without error. However, if the program contains an error, alltransactions will be processed incorrectly.Answer (B) is incorrect because, when an error does occur, for example, in input, it may not be discovered on atimely basis. Ordinarily, much less human intervention occurs once the transaction is processed.Answer (C) is incorrect because systematic (repetitive) errors will occur in computerized processing if an error existsin the program.Answer (D) is incorrect because adequately designed systems maintain transaction, console, and error logs that createuseful audit trails.

[1000] Gleim #: 9.79 -- Source: CIA 1193 I-24

Your firm has recently converted its purchasing cycle from a manual process to an online computer system. Which of thefollowing is a probable result associated with conversion to the new automatic system?

Processing errors are increased.A.The firm’s risk exposures are reduced.B.Processing time is increased.C.Traditional duties are less segregated.D.

Answer (A) is incorrect because a computer system decreases processing errors.Answer (B) is incorrect because the conversion to a new system does not reduce the number of risk exposures.Answer (C) is incorrect because processing time is decreased.Answer (D) is correct. In a manual system with appropriate internal control, separate individuals are responsible forauthorizing transactions, recording transactions, and custody of assets. These checks and balances prevent fraud anddetect inaccurate or incomplete transactions. In a computer environment, however, this segregation of duties is notalways feasible. For example, a computer may print checks, record disbursements, and generate information forreconciling the account balance.

[1001] Gleim #: 9.80 -- Source: CPA 586 A-58

A small client recently put its cash disbursements system on a server. About which of the following internal controlfeatures would an auditor most likely be concerned?

Programming of the applications are in BASIC, although COBOL is the dominant, standard language for businessprocessing.

A.

The server is operated by employees who have cash custody responsibilities.B.Only one employee has the password to gain access to the cash disbursement system.C.There are restrictions on the amount of data that can be stored and on the length of time that data can be stored.D.

Answer (A) is incorrect because the choice of language would have little effect on internal control.Answer (B) is correct. Segregation of duties is a basic category of control activities (AU 319). Functions areincompatible if a person is in a position both to perpetrate and conceal fraud or errors. Hence, the duties ofauthorizing transactions, recording transactions, and custody of assets should be assigned to different people. Thoseemployees that operate the server may be able to override the controls to change records to conceal a theft of cash.Answer (C) is incorrect because the limitation on access would be considered a strength.Answer (D) is incorrect because restrictions on the amount of data that can be stored and on the length of time thatdata can be stored do not constitute a control weakness.

Gleim's CIA Test Prep: Part III: Business Analysis and Information Technology(1165 questions)

Copyright 2004 Gleim Publications, Inc. Page 401Printed for g j

Page 402: Part three cia_with_ answers

[1002] Gleim #: 9.81 -- Source: CPA 1195 A-14Which of the following controls most likely could prevent computer personnel from modifying programs to bypassprogrammed controls?

Periodic management review of computer utilization reports and systems documentation.A.Segregation of duties within computer for computer programming and computer operations.B.Participation of user department personnel in designing and approving new systems.C.Physical security of computer facilities in limiting access to computer equipment.D.

Answer (A) is incorrect because, although periodic management review is appropriate, reports and systemsdocumentation will not prevent or detect unauthorized modifications.Answer (B) is correct. Programmers and analysts can modify programs, data files, and controls. Thus, they shouldhave no access to programs used to process transactions. Segregation of programming and operations is a controlnecessary to prevent unauthorized modifications of programs.Answer (C) is incorrect because user participation relates to new systems, not modification of existing systems.Answer (D) is incorrect because programmers may have access through data communications. Thus, physical securityis not sufficient to prevent unauthorized modifications.

[1003] Gleim #: 9.82 -- Source: CPA 586 A-1

Matthews Corp. has changed from a system of recording time worked on clock cards to a computerized payroll system inwhich employees record time in and out with magnetic cards. The computer system automatically updates all payrollrecords. Because of this change

A generalized computer audit program must be used.A.Part of the audit trail is altered.B.The potential for payroll-related fraud is diminished.C.Transactions must be processed in batches.D.

Answer (A) is incorrect because use of generalized audit software is only one of many ways of auditing through acomputer.Answer (B) is correct. In a manual payroll system, a paper trail of documents is created to provide audit evidencethat controls over each step in processing are in place and functioning. One element of a computer system thatdifferentiates it from a manual system is that a transaction trail useful for auditing purposes might exist only for abrief time or only in computer-readable form.Answer (C) is incorrect because conversion to a computer system may actually increase the chance of fraud byeliminating segregation of incompatible functions and other controls.Answer (D) is incorrect because automatic updating indicates that processing is not in batch mode.

[1004] Gleim #: 9.83 -- Source: Publisher

Operating controls include all of the following except

Halt and error controls.A.Batch controls.B.Library security and use of proper file labels.C.Duplicate files and backup procedures.D.

Answer (A) is incorrect because halt and error condition controls include explicit instructions that should appear inrun manuals for handling halts and error messages for programs.Answer (B) is correct. A batch total is the sum of information that results from adding a field from each record in abatch or group. It provides for control over the movement or processing of the batch and is considered an applicationcontrol.Answer (C) is incorrect because library security controls include the organization and operation of a library topreclude misplacement or theft of tapes, programs, and other file media. In addition, internal and external labelsshould be used so that the proper files are processed.Answer (D) is incorrect because operating controls also include provisions for backup and reconstruction of files ifdata are lost as a result of processing errors or catastrophes.

[1005] Gleim #: 9.84 -- Source: CIA 1189 II-25

Which of the following terms best describes the type of control practice evidenced by a segregation of duties betweencomputer programmers and computer operators?

Gleim's CIA Test Prep: Part III: Business Analysis and Information Technology(1165 questions)

Copyright 2004 Gleim Publications, Inc. Page 402Printed for g j

Page 403: Part three cia_with_ answers

Systems development control.A.Hardware control.B.Applications control.C.Organizational control.D.

Answer (A) is incorrect because systems development controls concern systems analysis, design, and implementation.Answer (B) is incorrect because hardware controls are incorporated into the equipment.Answer (C) is incorrect because applications controls pertain to specific programs. They include input, processing, andoutput controls.Answer (D) is correct. Organizational control concerns the proper segregation of duties and responsibilities within theinformation systems department. For example, programmers should not have access to the equipment, and operators shouldnot have programming ability. Although proper segregation is desirable, functions that would be considered incompatible ifperformed by a single individual in a manual activity are often performed through the use of an information systemsprogram or series of programs. Therefore, compensating controls may be necessary, such as library controls, effectivesupervision, and rotation of personnel.

[1006] Gleim #: 9.85 -- Source: CIA 595 III-74

When a new application is being created for widespread use in a large organization, the principal liaison between theinformation systems (IS) department and the rest of an organization is normally a(n)

End user.A.Application programmer.B.Maintenance programmer.C.Systems analyst.D.

Answer (A) is incorrect because the end user never acts as a liaison between the IS department and the rest of theorganization.Answer (B) is incorrect because programmers design, write, test, and document the specific programs developed bysystems analysts. It is not their responsibility to act as a liaison between the IS department and the rest of theorganization.Answer (C) is incorrect because programmers design, write, test, and document the specific programs developed bysystems analysts. It is not their responsibility to act as a liaison between the IS department and the rest of theorganization.Answer (D) is correct. Systems analysts are specifically qualified to analyze and design computer informationsystems. They survey the existing system, analyze the organization’s information requirements, and design newsystems to meet those needs. Systems analysts communicate with the entire organization and act as a liaison betweenthe organization and the IS department.

[1007] Gleim #: 9.86 -- Source: CMA 693 4-14

Systems analysts are the personnel within an organization who are responsible for the development of the company’sinformation system. Which one of the following functions is least likely to be performed by a systems analyst?

Design of computer applications.A.Preparation of specifications for computer programming.B.Developing, coding, and testing computer programs.C.Examining user information requirements.D.

Answer (A) is incorrect because systems analysts design computer applications.Answer (B) is incorrect because analysts prepare specifications for computer programmers.Answer (C) is correct. A systems analyst plans and designs computer applications, often with the assistance ofsystems flowcharts. Systems analysis usually involves examining user information requirements, surveying theexisting system, and preparing specifications for computer programming. The concept of systems analysis, however,does not include the development, coding, and testing of individual computer programs. The work of the systemsanalyst provides the design specifications that will guide the preparation of specific programs by programmers. In agood system of internal control, the systems analyst should not perform programming tasks or have access tocomputer equipment, production programs, data files, or input-output controls.Answer (D) is incorrect because the systems analyst must examine user information requirements to learn what typeof application is needed.

[1008] Gleim #: 9.87 -- Source: CIA 588 II-31

Which of the following represents an internal control weakness in a computer-based system?

Gleim's CIA Test Prep: Part III: Business Analysis and Information Technology(1165 questions)

Copyright 2004 Gleim Publications, Inc. Page 403Printed for g j

Page 404: Part three cia_with_ answers

Computer programmers write and revise programs designed by analysts.A.The data control group is solely responsible for distributing reports and other output.B.The computer librarian maintains custody and record keeping for computer application programs.C.Computer operators have access to operator instructions and the authority to change programs.D.

Answer (A) is incorrect because writing and revising computer programs are appropriate functions for programmers.Answer (B) is incorrect because distributing computer reports is an appropriate function of the control group.Answer (C) is incorrect because maintaining custody and related record keeping for computer programs is appropriate for acomputer librarian.Answer (D) is correct. Computer operators need access to operator instructions. Otherwise, they could not perform theirduties. Operators, however, should not have the authority to change computer programs.

[1009] Gleim #: 9.88 -- Source: CPA 585 A-26

For control purposes, which of the following should be organizationally segregated from the computer operations function?

Data conversion.A.Surveillance of screen display messages.B.Systems development.C.Minor maintenance according to a schedule.D.

Answer (A) is incorrect because data conversion may be assigned to computer operations.Answer (B) is incorrect because surveillance of screen display messages may be assigned to computer operations.Answer (C) is correct. Systems analysts are specifically qualified to analyze and design computer informationsystems. They survey the existing system, analyze the organization’s information requirements, and design newsystems to meet those needs. These design specifications will guide the preparation of specific programs by computerprogrammers. The console operator should not be assigned programming duties, much less responsibility for systemsdesign, and thus not have the opportunity to make changes in programs and systems as (s)he operates the equipment.Answer (D) is incorrect because minor maintenance according to a schedule may be assigned to computer operations.

[1010] Gleim #: 9.89 -- Source: CMA 695 4-22

In the organization of the information systems function, the most important separation of duties is

Not allowing the data librarian to assist in data processing operations.A.Assuring that those responsible for programming the system do not have access to data processing operations.B.Having a separate information officer at the top level of the organization outside of the accounting function.C.Using different programming personnel to maintain utility programs from those who maintain the applicationprograms.

D.

Answer (A) is incorrect because librarians maintain control over documentation, programs, and data files; they shouldhave no access to equipment, but they can assist in data processing operations.Answer (B) is correct. Separation of duties is a general control that is vital in a computerized environment. Someseparation of duties common in noncomputerized environments may not be feasible in a computer environment.However, certain tasks should not be combined. Systems analysts and programmers should be separate fromcomputer operators. Both programmers and analysts may be able to modify programs, files, and controls, and shouldtherefore have no access to those programs nor to computer equipment. Operators should not be assignedprogramming duties or responsibility for systems design, and should have no opportunity to make changes inprograms and systems.Answer (C) is incorrect because a separate information officer outside of the accounting function would not be ascritical a separation of duties as that between programmers and processors.Answer (D) is incorrect because programmers usually handle all types of programs.

[1011] Gleim #: 9.90 -- Source: CMA 1290 4-24

Information processing made possible by a network of computers dispersed throughout an organization is called

Online processing.A.Interactive processing.B.Time sharing.C.Distributed data processing.D.

Gleim's CIA Test Prep: Part III: Business Analysis and Information Technology(1165 questions)

Copyright 2004 Gleim Publications, Inc. Page 404Printed for g j

Page 405: Part three cia_with_ answers

Answer (A) is incorrect because online processing is a method of processing data that permits both immediate posting(updating) and inquiry of master files as transactions occur.Answer (B) is incorrect because interactive processing is a method of processing data immediately upon input.Answer (C) is incorrect because time sharing is the processing of a program by the CPU until an input or output operation isrequired. In time sharing, the CPU spends a fixed amount of time on each program.Answer (D) is correct. Distributed processing is characterized by a merger of computer and telecommunicationstechnology. Distributed systems permit not only remote access to a computer but also the performance of local processing atlocal sites. The result is greater flexibility in systems design and the possibility of an optimal distribution of processingtasks.

[1012] Gleim #: 9.91 -- Source: CMA 693 4-15

Transaction processing systems frequently support the inquiry of online database users. Inquiry processing includes all ofthe following characteristics except that

Either batch or real-time processing may be used.A.It is dependent on the use of telecommunication networks and database management query languages.B.Responses are in a prespecified format displayed on the end-user’s terminal.C.End-users are allowed to make changes to the records retrieved.D.

Answer (A) is incorrect because either batch or real-time processing may be used to query a database system.Answer (B) is incorrect because a query system is dependent on the use of telecommunication networks and databasemanagement query languages.Answer (C) is incorrect because responses are in a prespecified format.Answer (D) is correct. Inquiry processing in a database system can be the result of either batch or real-timeprocessing. An inquiry system requires the use of sophisticated hardware and software, including a database querylanguage. Responses are in a prespecified format. End users receive responses concerning the results of transactionactivities but are not typically allowed to make changes to the records retrieved.

[1013] Gleim #: 9.92 -- Source: CMA 691 4-26

The concept of timeliness of data availability is most relevant to

Payroll systems.A.Manual systems.B.Computerized systems.C.Online systems.D.

Answer (A) is incorrect because timeliness is not necessarily an element of this type of system.Answer (B) is incorrect because timeliness is not necessarily an element of this type of system.Answer (C) is incorrect because timeliness is not necessarily an element of this type of system.Answer (D) is correct. An online processing system is in direct communication with the computer, giving it thecapability to handle transactions as they are entered. An online system permits both immediate posting (updating) andinquiry of master files as transactions occur. In an online system, data are immediately available to users upon entry.

[1014] Gleim #: 9.93 -- Source: CMA 692 4-1

Batch processing

Is not used by most businesses because it reduces the audit trail.A.Allows users to inquire about groups of information contained in the system.B.Accumulates transaction records into groups for processing against the master file on a delayed basis.C.Can only be performed on a centralized basis.D.

Answer (A) is incorrect because batch processing provides as much of an audit trail as any computerized operation.Answer (B) is incorrect because batch processing refers to the input of data, not inquiry.Answer (C) is correct. Batch processing is the accumulation and grouping of transactions for processing on a delayedbasis. The batch approach is suitable for applications that can be processed against the master file at intervals andinvolve large volumes of similar items, such as payroll, sales, inventory, and billing.Answer (D) is incorrect because batch processing can also be performed on a decentralized basis.

Gleim's CIA Test Prep: Part III: Business Analysis and Information Technology(1165 questions)

Copyright 2004 Gleim Publications, Inc. Page 405Printed for g j

Page 406: Part three cia_with_ answers

[1015] Gleim #: 9.94 -- Source: CMA 1290 4-23An interactive system environment is best characterized by

Data files with records arranged sequentially.A.The processing of groups of data at regular intervals.B.Sorting the transaction file before processing.C.The processing of data immediately on input.D.

Answer (A) is incorrect because an interactive system requires direct-access files.Answer (B) is incorrect because an interactive system permits immediate, online processing of single transactions.Answer (C) is incorrect because the transaction file need not be sorted before processing.Answer (D) is correct. In an interactive (inquiry) system, users employ interactive terminals to converse directly withthe system. The system is characterized by online entry and processing, direct access, and timesharing.

[1016] Gleim #: 9.95 -- Source: CPA 1185 A-52

What type of computer system is characterized by data that are assembled from more than one location and records that areupdated immediately?

Personal computer systems.A.Midrange computer systems.B.Batch processing systems.C.Online, real-time systems.D.

Answer (A) is incorrect because, although personal computer systems may be part of a local area network (LAN),they need not be.Answer (B) is incorrect because, although midrange computer systems may be part of a local area network (LAN),they need not be.Answer (C) is incorrect because batching of transactions requires assembly of data at one place and a delay inupdating.Answer (D) is correct. Real-time processing involves processing an input record and receiving the output soonenough to affect a current decision-making process. In a real-time system, the user interacts with the system to controlan ongoing activity. Online indicates that the decision maker is in direct communication with the computer. Online,real-time systems usually permit access to the server from multiple remote terminals.

[1017] Gleim #: 9.96 -- Source: CIA 589 III-24

If a workstation contains a processor, monitor, printer, storage, and communications capabilities, it is said to be a(n)

Dumb workstation.A.Intelligent workstation.B.Noninteractive workstation.C.Desktop publishing workstation.D.

Answer (A) is incorrect because a dumb workstation usually lacks data processing capacity.Answer (B) is correct. The workstation is usually said to be intelligent if it has data processing capacity, and storedprograms can be modified. Such a workstation usually includes a personal computer, an output device, disk storage,and communications links. Intelligent workstations are often connected in a local area network and can thereforecommunicate with each other. Thus, they are not mere stand-alone personal computers.Answer (C) is incorrect because a noninteractive workstation does not permit the user to direct system activities.Answer (D) is incorrect because desktop publishing is the capability to produce high-quality printing usingspecialized software.

[1018] Gleim #: 9.97 -- Source: CIA 1191 III-31

A major disadvantage of the life cycle approach to system development is that it is not well-suited for projects that are

Structured.A.Large.B.Complex.C.Unstructured.D.

Gleim's CIA Test Prep: Part III: Business Analysis and Information Technology(1165 questions)

Copyright 2004 Gleim Publications, Inc. Page 406Printed for g j

Page 407: Part three cia_with_ answers

Answer (A) is incorrect because the life cycle approach is better suited for structured projects.Answer (B) is incorrect because the life cycle approach is better suited for large projects.Answer (C) is incorrect because the life cycle approach is better suited for complex projects.Answer (D) is correct. The life cycle approach is best employed when systems are large and highly structured, usersunderstand the tasks to be performed by the information system, and the developers have directly applicable experience indesigning similar systems. In the life cycle process, each stage of development is highly structured, and requirements areclearly defined. However, when the task is unstructured, prototyping may be the better approach. Prototyping (anexperimental assurance process) is costly and time-consuming and thus is not currently the most common approach. Itentails developing and putting into operation successively more refined versions of the system until sufficient information isobtained to produce satisfactory design.

[1019] Gleim #: 9.98 -- Source: CIA 1192 III-39

An insurance firm that follows the systems development life cycle concept for all major information system projects ispreparing to start a feasibility study for a proposed underwriting system. Some of the primary factors the feasibility studyshould include are

Possible vendors for the system and their reputation for quality.A.Exposure to computer viruses and other intrusions.B.Methods of implementation such as parallel or cut-over.C.Technology and related costs.D.

Answer (A) is incorrect because possible vendors for the system and their reputation for quality would be determinedafter the feasibility study.Answer (B) is incorrect because exposure to computer viruses and other intrusions is part of the informationrequirements phase.Answer (C) is incorrect because methods of implementation such as parallel or cut-over would be determined duringthe implementation and operations stage.Answer (D) is correct. The feasibility study should consider the activity to be automated, the needs of the user, thetype of equipment required, the cost, and the potential benefit to the specific area and the company in general. Thus,technical feasibility and cost are determined during this stage.

[1020] Gleim #: 9.99 -- Source: CIA 1192 III-87

Compared with prototyping, life cycle methodologies are appropriate for problems involving

High user understanding of tasks and large project size.A.Low user understanding of tasks and small project size.B.Low user understanding of tasks and uncertainty of requirements.C.Uncertainty of requirements and large project size.D.

Answer (A) is correct. The life cycle approach is best employed when systems are large and highly structured, usersunderstand the tasks to be performed by the information system, and the developers have directly applicableexperience in designing similar systems. In the life cycle process, each stage of development is highly structured, andrequirements are clearly defined.Answer (B) is incorrect because the lower the user understanding of tasks and the smaller the project size, the lessappropriate life cycle methodologies are for the problem.Answer (C) is incorrect because the lower the user understanding of tasks and the more uncertain the requirements,the less appropriate life cycle methodologies are for the problem.Answer (D) is incorrect because the more uncertain the requirements, the less appropriate life cycle methodologiesare for the problem.

[1021] Gleim #: 9.100 -- Source: CIA 1191 III-29

To be more responsive to its customers, a bank wants a system that will permit account representatives to consolidateinformation about all the accounts belonging to individual customers. Bank management is willing to experiment withdifferent approaches because the requirements are evolving rapidly. The best development approach for this system is

Prototyping.A.System development life cycle model.B.Structured analysis and design technique.C.Hierarchy-input-process-output.D.

Gleim's CIA Test Prep: Part III: Business Analysis and Information Technology(1165 questions)

Copyright 2004 Gleim Publications, Inc. Page 407Printed for g j

Page 408: Part three cia_with_ answers

Answer (A) is correct. Prototyping (an experimental assurance process) is costly and time-consuming and thus is notcurrently the most common approach. It entails developing and putting into operation successively more refined versions ofthe system until sufficient information is obtained to produce satisfactory design. Prototyping is the best approach in thesecircumstances because the requirements are difficult to specify in advance and are likely to change significantly duringdevelopment.Answer (B) is incorrect because the system development life cycle model is appropriate for highly structured operationalapplications whose requirements can be defined in advance. Thus, it is not suitable for the bank’s application.Answer (C) is incorrect because structured analysis and design technique is a specific application of the systemdevelopment life cycle model.Answer (D) is incorrect because hierarchy-input-process-output is a specific application of the system development lifecycle model.

[1022] Gleim #: 9.101 -- Source: CIA 593 III-47

A bank was considering its first use of computer-aided software engineering (CASE) to develop an inquiry system foraccount representatives to access consolidated profiles of customers’ accounts. A benefit of using CASE in this situation isthat

No new software development tools would be needed.A.No training of programmers would be required.B.Management of the development process would be improved.C.The need for testing would be reduced.D.

Answer (A) is incorrect because the first time CASE is used, the bank will have to acquire all the CASE developmentsoftware.Answer (B) is incorrect because the first time CASE is used, the bank will have to train programmers in its uses.Answer (C) is correct. CASE applies computers to software design and development. It permits creation andmaintenance of systems documentation on the computer and the automation of a part of the programming effort.Using CASE would improve management of the development process because the CASE software maintains the linksbetween the different components, provides built-in project management tools, and supplies automated testing aids.Answer (D) is incorrect because using CASE does not reduce the need for testing to verify that program code matchesspecifications. However, the testing itself is more manageable because of automated testing aids provided by CASE.

[1023] Gleim #: 9.102 -- Source: CIA 1192 III-90

Advantages of life cycle methodologies are

Lower overall development costs when requirements change frequently.A.Ability to give users a functioning system quickly.B.Reduced application development time to achieve a functioning system.C.Enhanced management and control of the development process.D.

Answer (A) is incorrect because overall development costs are higher when requirements change frequently in a lifecycle methodology.Answer (B) is incorrect because life cycle methodologies are unable to give users a functioning system quickly.Answer (C) is incorrect because life cycle methodologies require lengthy application development time to achieve afunctioning system.Answer (D) is correct. The systems development life cycle approach is the most common methodology applied to thedevelopment of large, highly structured application systems. The life cycle approach is based on the idea that aninformation system has a finite life span that is limited by the changing needs of the organization. This cycle isanalytically divisible into stages. A new system life cycle begins when the inadequacy of the current system leads to adecision to develop a new or improved system. This method is a structured process for controlling the creativeactivity required to devise, develop, and implement an information system. The process is described in varying termsby different writers, but the nature and sequence of the steps are essentially the same. Life cycle methodologiesprovide enhanced management and control of the development process because they provide structure for a creativeprocess by dividing it into manageable steps and specifying what must be produced in each phase.

[1024] Gleim #: 9.103 -- Source: CISA

Application control objectives do not normally include assurance that

Gleim's CIA Test Prep: Part III: Business Analysis and Information Technology(1165 questions)

Copyright 2004 Gleim Publications, Inc. Page 408Printed for g j

Page 409: Part three cia_with_ answers

Authorized transactions are completely processed once and only once.A.Transaction data are complete and accurate.B.Review and approval procedures for new systems are set by policy and adhered to.C.Processing results are received by the intended user.D.

Answer (A) is incorrect because an objective of application controls is that authorized transactions are completely processedonce and only once.Answer (B) is incorrect because an objective of application controls is that transaction data is complete and accurate.Answer (C) is correct. Application controls provide reasonable assurance that the recording, processing, and reporting ofdata are properly performed. Review and approval procedures for new systems are among the general controls known assystem software acquisition and maintenance controls.Answer (D) is incorrect because an objective of application controls is that processing results are received by the intendeduser.

[1025] Gleim #: 9.104 -- Source: CMA 693 4-6

Data processed by a computer system are usually transferred to some form of output medium for storage. However, thepresence of computerized output does not, in and of itself, assure the output’s accuracy, completeness, or authenticity. Forthis assurance, various controls are needed. The major types of controls for this area include

Transaction controls, general controls, and printout controls.A.Activity listings, echo checks, and pre-numbered forms.B.Tape and disk output controls and printed output controls.C.Input controls, tape and disk output controls, and printed output controls.D.

Answer (A) is incorrect because transaction controls and general controls do not ensure the completeness andauthenticity of the output.Answer (B) is incorrect because activity listings and prenumbered forms do not ensure control of the output in asystem, and output controls are needed to ensure the accuracy of the output.Answer (C) is incorrect because input controls are needed to ensure the completeness and authenticity of the output.Answer (D) is correct. Input controls provide reasonable assurance that data received for processing have beenproperly authorized, converted into machine-sensible form, and identified, and that data have not been lost,suppressed, added, duplicated, or otherwise improperly changed. Output controls provide assurance that theprocessing result (such as reports, tape and disk, magnetic files, invoices, or printed output) is accurate and that onlyauthorized personnel receive the output.

[1026] Gleim #: 9.105 -- Source: CIA 579 II-5

Which of the following should the auditor recommend as the most economical point at which to correct input errors in anonline system?

Input data are balanced with computer-produced control totals.A.Entry of data into each field of a record is completed.B.Output data are balanced with computer-produced control totals and delivered to the user.C.Entry of data into each record is completed.D.

Answer (A) is incorrect because the most economical point is the earliest time after data has been entered into thesystem.Answer (B) is correct. The most economical point at which to correct input errors is at the earliest time after the datahas been entered into the system. The earliest point is the entry of data into each field of a record. Errors can easily berectified because the proper information is already available.Answer (C) is incorrect because the most economical point is the earliest time after data has been entered into thesystem.Answer (D) is incorrect because the most economical point is the earliest time after data has been entered into thesystem.

[1027] Gleim #: 9.106 -- Source: CPA 1192 A-20

If a control total were to be computed on each of the following data items, which would best be identified as a hash totalfor a payroll computer application?

Hours worked.A.Total debits and total credits.B.Net pay.C.Department numbers.D.

Gleim's CIA Test Prep: Part III: Business Analysis and Information Technology(1165 questions)

Copyright 2004 Gleim Publications, Inc. Page 409Printed for g j

Page 410: Part three cia_with_ answers

Answer (A) is incorrect because hours worked is an example of a financial (amount) total.Answer (B) is incorrect because total debits and total credits is a financial total.Answer (C) is incorrect because net pay is a financial total.Answer (D) is correct. The three types of control totals are record counts, financial (amount) totals, and hash totals. Recordcounts establish the number of source documents and reconcile it to the number of output records. Financial (amount) totalscompute dollar or amount totals from source documents (e.g., the total dollar amount of invoices processed) and reconcilethem with the output records. Hash totals add numbers on input documents that are not normally added (e.g., departmentnumbers for payroll processing) and reconcile them with output records.

[1028] Gleim #: 9.107 -- Source: CPA AUD R98-4

An entity has the following invoices in a batch:

Invoice Number Product Quantity Unit Price201 F10 150 $  5.00202 G15 200 $10.00203 H20 250 $25.00204 K35 300 $30.00

Which of the following most likely represents a hash total?

FGHK80A.4B.204C.810D.

Answer (A) is incorrect because a hash total is ordinarily the sum of a numeric field.Answer (B) is incorrect because 4 is a record count.Answer (C) is incorrect because 204 is an invoice number.Answer (D) is correct. Input controls in batch computer systems are used to determine that no data are lost or addedto the batch. Depending on the sophistication of a particular system, control may be accomplished by using recordcounts, batch totals, or hash totals. The hash total is a control total without a defined meaning, such as the total ofemployee numbers or invoice numbers, that is used to verify the completeness of data. The hash total of the invoicenumbers is 810.

[1029] Gleim #: 9.108 -- Source: CIA 1190 III-23

In an order-entry system, in which manually-prepared source documents are entered online for immediate processing,which of the following is an example of an appropriate input-output control?

Password authorization procedure.A.Check-digit validation procedure.B.Hash total verification.C.Backup and recovery procedures.D.

Answer (A) is incorrect because password authorization is a general control over access to the system or parts of it.Answer (B) is correct. Self-checking digits may be used to detect incorrect identification numbers. The digit isgenerated by applying an algorithm to the ID number. During the input process, the check digit is recomputed byapplying the same algorithm to the code actually entered.Answer (C) is incorrect because hash totals (otherwise meaningless control totals, e.g., of identification numbers) areappropriate for batch processing.Answer (D) is incorrect because backup and recovery procedures are general controls.

[1030] Gleim #: 9.109 -- Source: CPA AUD R98-5

A customer intended to order 100 units of product Z96014, but incorrectly ordered nonexistent product Z96015. Which ofthe following controls most likely would detect this error?

Check digit verification.A.Record count.B.Hash total.C.Redundant data check.D.

Gleim's CIA Test Prep: Part III: Business Analysis and Information Technology(1165 questions)

Copyright 2004 Gleim Publications, Inc. Page 410Printed for g j

Page 411: Part three cia_with_ answers

Answer (A) is correct. Check digit verification is used to identify incorrect identification numbers. The digit is generated byapplying an algorithm to the ID number. During input, the check digit is recomputed by applying the same algorithm to theentered ID number.Answer (B) is incorrect because a record count is a control total of the number of transactions in a batch.Answer (C) is incorrect because a hash total is a control total that is the sum of a field without a defined meaning.Answer (D) is incorrect because a redundant data check searches for duplicate information in a database.

[1031] Gleim #: 9.110 -- Source: CIA 1195 III-63

A validation check used to determine if a quantity ordered field contains only numbers is an example of a(n)

Input control.A.Audit trail control.B.Processing control.C.Data security control.D.

Answer (A) is correct. A validation check at data entry verifying that a quantity field contains only numbers is anexample of a programmatic means of ensuring the accuracy of an input value. Thus, it is an input control.Answer (B) is incorrect because the purpose of an audit trail control is to ensure that a chronological record of allrelevant events in a system has been recorded.Answer (C) is incorrect because a processing control ensures that data are complete and accurate during updating.Answer (D) is incorrect because a data security control ensure that only authorized individuals are permitted to accessand use a system.

[1032] Gleim #: 9.111 -- Source: CIA 1187 I-30

An accounts payable program posted a payable to a vendor not included in the online vendor master file. A control thatwould prevent this error is a

Validity check.A.Range check.B.Reasonableness test.C.Parity check.D.

Answer (A) is correct. Validity checks are tests of identification numbers or transaction codes for validity bycomparison with items already known to be correct or authorized. For example, Social Security numbers on payrollinput records can be compared with Social Security numbers authorized by the personnel department.Answer (B) is incorrect because a range check is based on known limits for given information.Answer (C) is incorrect because a reasonableness test is based on known limits for given information.Answer (D) is incorrect because a parity check adds the bits in a character or message and checks the sum todetermine if it is odd or even, depending on whether the computer has odd or even parity.

[1033] Gleim #: 9.112 -- Source: CMA 687 5-4

Which one of the following input validation routines is not likely to be appropriate in a real-time operation?

Sign check.A.Reasonableness check.B.Sequence check.C.Redundant data check.D.

Answer (A) is incorrect because sign checks test data for the appropriate arithmetic sign. For instance, hours workedin a payroll should always be a positive number.Answer (B) is incorrect because reasonableness tests verify that amounts fall within predetermined limits.Answer (C) is correct. The program controls listed prescreen or edit data prior to processing, but the sequence checkis most likely to be used only in batch processing. A sequence check tests to determine that records are in properorder. For example, a payroll input file can be sorted into Social Security number order. A sequence check can thenbe performed to verify record order. This control would not apply in a real-time operation because records are notprocessed sequentially.Answer (D) is incorrect because a redundancy check requires sending additional data items to serve as a check on theother transmitted data; for example, part of a customer name can be matched against the name associated with thetransmitted customer number.

Gleim's CIA Test Prep: Part III: Business Analysis and Information Technology(1165 questions)

Copyright 2004 Gleim Publications, Inc. Page 411Printed for g j

Page 412: Part three cia_with_ answers

[1034] Gleim #: 9.113 -- Source: CMA 1287 5-16

An employee in the receiving department keyed in a shipment from a remote terminal and inadvertently omitted thepurchase order number. The best systems control to detect this error is

Completeness test.A.Sequence check.B.Reasonableness test.C.Compatibility test.D.

Answer (A) is correct. A completeness test checks that all data elements are entered before processing. An interactivesystem can be programmed to notify the user to enter the number before accepting the receiving report.Answer (B) is incorrect because a sequence check tests for the ordering, not omission, of records.Answer (C) is incorrect because a limit or reasonableness test checks the values of data items against establishedlimits.Answer (D) is incorrect because a compatibility test (field check) determines whether characters are appropriate to afield.

[1035] Gleim #: 9.114 -- Source: CIA 1193 II-25

To ensure the completeness of update in an online system, separate totals are accumulated for all transactions processedthroughout the day. The computer then agrees these totals to the total of items accepted for processing. This is an exampleof

Run-to-run controls.A.Computer matching.B.Computer sequence check.C.One-for-one checking.D.

Answer (A) is correct. During each program run in a series, the computer accumulates the totals of transactions thathave been processed. The run-to-run check reconciles them with the totals forwarded from the previous program run.Run-to-run totals thus ensure completeness of update.Answer (B) is incorrect because computer matching compares transaction data with referenced fields or records.Answer (C) is incorrect because computer sequence checks identify changes or breaks in a numerical sequence.Answer (D) is incorrect because one-for-one checking usually requires manual comparisons of input data elementswith processing results.

[1036] Gleim #: 9.115 -- Source: CIA 1189 II-31

Reconciling input with processing control totals to ensure that all transactions have been processed is an example of

An input control.A.A processing control.B.An output control.C.An access control.D.

Answer (A) is incorrect because input controls are designed to provide reasonable assurance that data received forprocessing have been properly authorized and are in a suitable form. Input controls also include those that relate torejection, correction, and resubmission of data that were initially incorrect.Answer (B) is incorrect because processing controls provide reasonable assurance that all transactions are processedas authorized, no authorized transactions are omitted, and no unauthorized transactions are added.Answer (C) is correct. Application controls include input, processing, and output controls. Output controls aredesigned to assure the accuracy of processing results and that only authorized personnel receive the output. Forexample, comparing the control total of input documents with the total documents processed is an output control.Answer (D) is incorrect because access controls prevent unauthorized and improper use of data and programs.

[1037] Gleim #: 9.116 -- Source: CMA 685 5-14

Program documentation is a control designed primarily to ensure that

Gleim's CIA Test Prep: Part III: Business Analysis and Information Technology(1165 questions)

Copyright 2004 Gleim Publications, Inc. Page 412Printed for g j

Page 413: Part three cia_with_ answers

Programmers have access to the tape library or information on disk files.A.Programs do not make mathematical errors.B.Programs are kept up to date and perform as intended.C.No one has made use of the computer hardware for personal reasons.D.

Answer (A) is incorrect because programmers should not have access to operational materials.Answer (B) is incorrect because editing routines check for arithmetic errors prior to processing, and debugging shoulduncover errors in programs.Answer (C) is correct. Complete, up-to-date documentation of all programs and associated operating procedures isnecessary for efficient operation of a computer installation. Maintenance of programs is important to provide for continuityand consistency of data processing services to users. Program documentation (the program run manual) consists of problemstatements, systems flowcharts, operating instructions, record layouts, program flowcharts, program listings, test data, andapproval and change sheets.Answer (D) is incorrect because documentation cannot ensure computer security.

[1038] Gleim #: 9.117 -- Source: CIA 593 II-27

The accountant who prepared a spreadsheet model for workload forecasting left the company, and the accountant’ssuccessor was unable to understand how to use the spreadsheet. The best control for preventing such situations fromoccurring is to ensure that

Use of end-user computing resources is monitored.A.End-user computing efforts are consistent with strategic plans.B.Documentation standards exist and are followed.C.Adequate backups are made for spreadsheet models.D.

Answer (A) is incorrect because monitoring concerns controlling the use of resources.Answer (B) is incorrect because consistency with strategic plans concern evaluation of the system.Answer (C) is correct. The accountant’s successor could not use the forecasting model because of inadequatedocumentation. By requiring that documentation standards exist and are followed, the company will enable newemployees to understand internally developed programs when the developer leaves the organization.Answer (D) is incorrect because maintaining adequate backups for spreadsheet models is necessary, but lack ofadequate backup is not the reason the accountant’s successor could not use the forecasting model.

[1039] Gleim #: 9.118 -- Source: CIA 596 III-36

Change control typically includes procedures for separate libraries for production programs and for test versions ofprograms. The reason for this practice is to

Promote efficiency of system development.A.Segregate incompatible duties.B.Facilitate user input on proposed changes.C.Permit unrestricted access to programs.D.

Answer (A) is incorrect because production and test programs can be separated only if a specific procedure exists forplacing programs in production libraries. Thus, maintaining the separation requires its own procedure, which maydecrease development efficiency.Answer (B) is correct. Separating production and test versions of programs facilitates restricting access to productionprograms to the individuals, such as computer operators, who need access. The effect is to separate the incompatiblefunctions of operators and programmers.Answer (C) is incorrect because separating production and test versions of programs is independent of facilitatinguser input on proposed changes.Answer (D) is incorrect because separating production and test versions of programs restricts access to programs.

[1040] Gleim #: 9.119 -- Source: CIA 1196 III-49

A company often revises its production processes. The changes may entail revisions to processing programs. Ensuring thatchanges have a minimal impact on processing and result in minimal risk to the system is a function of

Gleim's CIA Test Prep: Part III: Business Analysis and Information Technology(1165 questions)

Copyright 2004 Gleim Publications, Inc. Page 413Printed for g j

Page 414: Part three cia_with_ answers

Security administration.A.Change control.B.Problem tracking.C.Problem-escalation procedures.D.

Answer (A) is incorrect because security administration is concerned with access to data.Answer (B) is correct. Change control is the process of authorizing, developing, testing, and installing coded changes so asto minimize the impact on processing and the risk to the system.Answer (C) is incorrect because problem tracking is concerned with collecting data to be analyzed for corrective action.Answer (D) is incorrect because problem escalation-procedures are a means of categorizing problems so that the leastskilled person can address them.

[1041] Gleim #: 9.120 -- Source: CIA 597 III-49

Minimizing the likelihood of unauthorized editing of production programs, job control language, and operating systemsoftware can best be accomplished by

Database access reviews.A.Compliance reviews.B.Good change-control procedures.C.Effective network security software.D.

Answer (A) is incorrect because database reviews determine whether users have gained access to database areas forwhich they have no authorization or whether they are employing programs that provide them with unauthorizedprivileges to view or change information.Answer (B) is incorrect because compliance reviews determine whether an organization has complied with applicableinternal and external procedures and regulations.Answer (C) is correct. Program change control includes (1) maintaining records of change authorizations, codechanges, and test results; (2) adhering to a systems development methodology (including documentation); (3)authorizing changeovers of subsidiary and headquarters’ interfaces; and (4) restricting access to authorized sourceand executable codes.Answer (D) is incorrect because network security audits review the physical and logical controls over the network.

[1042] Gleim #: 9.121 -- Source: CIA 591 I-36

Rejection of unauthorized modifications to application systems could be accomplished through the use of

Programmed checks.A.Batch controls.B.Implementation controls.C.One-for-one checking.D.

Answer (A) is incorrect because programmed checks determine the potential accuracy of input data (e.g., a rangecheck).Answer (B) is incorrect because batch control is used to ensure the completeness and accuracy of input and updating.Answer (C) is correct. General controls include organizational controls, such as a policy (an implementation control)that requires new programs and changes in programs (after adequate testing) to be formally approved before beingput into operation (implemented). This policy is reflected in the maintenance of approval and change sheets withappropriate authorizations.Answer (D) is incorrect because one-for-one checking is a technique used to check individual documents for accuracyand completeness of data input or update.

[1043] Gleim #: 9.122 -- Source: CIA 596 III-61

Which of the following risks is more likely to be encountered in an end-user computing (EUC) environment as comparedwith a mainframe computer system?

Inability to afford adequate uninterruptible power supply systems.A.User input screens without a graphical user interface (GUI).B.Applications that are difficult to integrate with other information systems.C.Lack of adequate utility programs.D.

Gleim's CIA Test Prep: Part III: Business Analysis and Information Technology(1165 questions)

Copyright 2004 Gleim Publications, Inc. Page 414Printed for g j

Page 415: Part three cia_with_ answers

Answer (A) is incorrect because inability to afford adequate uninterruptible power supply systems is a risk in all computingenvironments.Answer (B) is incorrect because lack of a GUI is a risk in all computing environments.Answer (C) is correct. The risk of allowing end users to develop their own applications is decentralization of control. End-user developed applications may not be subject to an independent outside review by systems analysts and are not created inthe context of a formal development methodology. These applications may lack appropriate standards, controls, and qualityassurance procedures. Moreover, when end users create their own applications and files, private information systems mayproliferate in which data are largely uncontrolled. These systems may contain the same information, but end-userapplications may update and define the data in different ways. Thus, determining the location of data and ensuring dataconsistency become more difficult because the applications are difficult to integrate.Answer (D) is incorrect because lack of adequate utility programs is a risk in all computing environments.

[1044] Gleim #: 9.123 -- Source: CIA 1194 I-37

Responsibility for the control of end-user computing exists at the organizational, departmental, and individual user level.Which of the following should be a direct responsibility of the individual users?

Acquisition of hardware and software.A.Taking equipment inventories.B.Strategic planning of end-user computing.C.Physical security of equipment.D.

Answer (A) is incorrect because the acquisition of hardware and software is an organizational- and departmental-levelresponsibility.Answer (B) is incorrect because taking equipment inventories is an organizational-level responsibility.Answer (C) is incorrect because strategic planning is an organizational- and departmental-level responsibility.Answer (D) is correct. End-user computing involves user-created or -acquired systems that are maintained andoperated outside of traditional information systems controls. In this environment, an individual user is ordinarilyresponsible for the physical security of the equipment (s)he uses.

[1045] Gleim #: 9.124 -- Source: CIA 595 III-40

Which of the following security controls might prevent unauthorized access to sensitive data via an unattended workstationdirectly connected to a mainframe?

Use of a screen saver.A.Use of passwords to identify users.B.Encryption of data files.C.Automatic log-off of inactive users.D.

Answer (A) is incorrect because a screen saver is software used to prevent the burning of an image onto the monitor’sscreen.Answer (B) is incorrect because the unattended workstation has likely already had the required password provided togain access.Answer (C) is incorrect because data is only encrypted when stored in a file. While the data or file is being modifiedby a user, the data is not encrypted. Thus, an unattended workstation would leave data susceptible to unauthorizedaccess.Answer (D) is correct. Automatic log-off of inactive users is a utility that disconnects a workstation from themainframe or server after a certain amount of time. Once the workstation has been disconnected, the user must logback into the system.

[1046] Gleim #: 9.125 -- Source: CIA 595 III-69

In general, mainframe computer production programs and data are adequately protected against unauthorized access.Certain utility software may, however, have privileged access to software and data. To compensate for the risk ofunauthorized use of privileged software, information systems (IS) management can

Prevent privileged software from being installed on the mainframe.A.Restrict privileged access to test versions of applications.B.Limit the use of privileged software.C.Keep sensitive programs and data on an isolated machine.D.

Gleim's CIA Test Prep: Part III: Business Analysis and Information Technology(1165 questions)

Copyright 2004 Gleim Publications, Inc. Page 415Printed for g j

Page 416: Part three cia_with_ answers

Answer (A) is incorrect because privileged software may be needed on the mainframe to modify programs and data.Answer (B) is incorrect because privileged access may be necessary to modify the final versions of applications.Answer (C) is correct. Since certain utility software may have privileged access to software and data stored on themainframe, management must control the use of this utility software. Management should limit the use of this software toonly those individuals with appropriate authority.Answer (D) is incorrect because authorized users must access sensitive programs and data through their workstations thatare connected to the mainframe.

[1047] Gleim #: 9.126 -- Source: CIA 595 III-37

Which of the following is an objective of logical security controls for information systems?

To ensure complete and accurate recording of data.A.To ensure complete and accurate processing of data.B.To restrict access to specific data and resources.C.To provide an audit trail of the results of processing.D.

Answer (A) is incorrect because it is not an objective of logical security control.Answer (B) is incorrect because it is not an objective of logical security control.Answer (C) is correct. The primary objective of security controls for information systems is to restrict access to dataand resources (both hardware and software) to only authorized individuals.Answer (D) is incorrect because it is not an objective of logical security control.

[1048] Gleim #: 9.127 -- Source: Publisher

Which of the following is the most effective user account management control in preventing the unauthorized use of acomputer system?

Management enforces an aggressive password policy that requires passwords to be 10 characters long, to benonreusable, and to be changed weekly.

A.

An account manager is responsible for authorizing and issuing new accounts.B.The passwords and usernames of failed log-in attempts are logged and documented in order to cite attemptedinfiltration of the system.

C.

Employees are required to renew their accounts semiannually.D.

Answer (A) is incorrect because, although passwords should be changed periodically, changing a long, non-reusablepassword weekly encourages employees to write each new password down in order to remember it, a practice notconsidered conducive to the control structure.Answer (B) is incorrect because the duties of authorization and control over assets should be separated.Answer (C) is incorrect because, although failed log-in attempts should be logged, it is considered bad practice torecord the password of failed log-in attempts. Employees often mistype their passwords, and therefore access to thelog by an infiltrator could facilitate breaking into a user’s account.Answer (D) is correct. Management’s network security policy should include measures to ensure that old and unusedaccounts are removed promptly. If employees’ accounts expire semiannually, reasonable assurance is provided thataccounts in use by unauthorized employees do not exist.

[1049] Gleim #: 9.128 -- Source: Publisher

Assigning passwords to computer users is a control to prevent unauthorized access. Because a password does notconclusively identify a specific individual, it must be safeguarded from theft. A method used to protect passwords is to

Require that they be displayed on computer screens but not printed on hard copy output.A.Set maximum character lengths.B.Require a minimum retention period.C.Eliminate all records of old passwords.D.

Gleim's CIA Test Prep: Part III: Business Analysis and Information Technology(1165 questions)

Copyright 2004 Gleim Publications, Inc. Page 416Printed for g j

Page 417: Part three cia_with_ answers

Answer (A) is incorrect because a password should not be displayed.Answer (B) is incorrect because a minimum length requirement is more likely.Answer (C) is correct. Security measures include changing passwords frequently, that is, establishing a relatively shortmaximum retention period; not displaying or printing passwords; setting minimum lengths; prohibiting the use of certainwords, character strings, or names; mandating a minimum retention period, so users cannot promptly change passwordsback to their old and convenient values; and retaining old passwords to prevent their use.Answer (D) is incorrect because retention of old passwords prevents their reuse.

[1050] Gleim #: 9.129 -- Source: CIA 1196 III-59

An insurance firm uses a wide area network (WAN) to allow agents away from the home office to obtain current rates andclient information and to submit approved claims using notebook computers and dial-in modems. In this situation, which ofthe following methods would provide the best data security?

Dedicated phone lines.A.Call-back features.B.Frequent changes of user IDs and passwords.C.End-to-end data encryption.D.

Answer (A) is incorrect because dedicated phone lines are not available to agents in the field.Answer (B) is incorrect because call-back features are used to authenticate the user but do not otherwise protect thetransmitted data.Answer (C) is incorrect because frequent changes of user IDs and passwords are used to authenticate the user but donot otherwise protect the transmitted data.Answer (D) is correct. Encryption of data is a security procedure in which a program encodes data prior totransmission and another program decodes the data after transmission. Encoding is important when confidential datathat can be electronically monitored are transmitted between geographically separated locations.

[1051] Gleim #: 9.130 -- Source: CIA 595 III-45

A control feature designed to negate the use of utility programs to read files that contain all authorized access user codesfor the network is

Internally encrypted passwords.A.A password hierarchy.B.Logon passwords.C.A peer-to-peer network.D.

Answer (A) is correct. Utility programs can be used to gain access to almost any file. However, gaining access to afile of passwords would be useless if those passwords were encrypted prior to placing them in the file.Answer (B) is incorrect because the utility program could gain access to all passwords in the file, regardless of thehierarchy.Answer (C) is incorrect because if the user has access to utility programs, (s)he already has access (is logged on) tothe system.Answer (D) is incorrect because a peer-to-peer network relates to connectivity of similar networks.

[1052] Gleim #: 9.131 -- Source: CISA

The telecommunication control of dial-up/disconnect/dial-back can be circumvented by using

Dedicated line technology.A.Automatic call forwarding.B.Encryption algorithms.C.High baud rate lines.D.

Answer (A) is incorrect because dedicated line technology strengthens controls, not circumvents them.Answer (B) is correct. Automatic call forwarding can circumvent the control if the authorized user has this telephoneservice and the computer callback is forwarded to an unauthorized user.Answer (C) is incorrect because encryption algorithms strengthen controls, not circumvent them.Answer (D) is incorrect because the speed of the lines does not directly contribute to the strength or weakness oftelecommunication controls.

Gleim's CIA Test Prep: Part III: Business Analysis and Information Technology(1165 questions)

Copyright 2004 Gleim Publications, Inc. Page 417Printed for g j

Page 418: Part three cia_with_ answers

[1053] Gleim #: 9.132 -- Source: CIA 1194 I-27

An auditor reviewed access security over the company’s various computer applications. The auditor found that securityconsisted of access controls programmed into each application. The best recommendation for management in the situationis

Eliminate the built-in access controls.A.Consider the use of access control software.B.Consider the use of utility software.C.Expand the use of the built-in access controls to new applications.D.

Answer (A) is incorrect because the built-in access controls should be retained until replaced with a morecomprehensive and cost-effective system.Answer (B) is correct. Access control software provides comprehensive and coordinated security. It permitsauthorized users to gain access only for purposes of performing their assigned duties and restricts employees fromperforming incompatible functions. A comprehensive system is more cost-effective than programming access controlsinto each application.Answer (C) is incorrect because utility software does not usually perform security functions.Answer (D) is incorrect because a comprehensive system is more cost-effective than programming access controlsinto each application.

[1054] Gleim #: 9.133 -- Source: CIA 1194 I-28

The best source of evidence to determine if ex-employees continue to have access to a company’s automated databases is

Discussing the password removal process with the database administrator.A.Reviewing computer logs of access attempts.B.Reconciling current payroll lists with database access lists.C.Reviewing access control software to determine whether the most current version is implemented.D.

Answer (A) is incorrect because discussing the password removal process does not determine whether ex-employeesare still using or are able to use their passwords to access the databases.Answer (B) is incorrect because the computer logs should be compared with current payroll lists.Answer (C) is correct. To determine if ex-employees are accessing the company’s automated database, the auditorshould obtain the log showing database accesses. This log should be compared with current payroll lists to see ifanyone not on the payroll is still accessing or is able to access the databases.Answer (D) is incorrect because reviewing the access control software does not indicate whether ex-employees canaccess or are accessing the databases.

[1055] Gleim #: 9.134 -- Source: Publisher

A client installed the most sophisticated controls using biometric attributes of employees to gain access to their computersystem. This technology most likely replaced which of the following controls?

Use of security specialists.A.Reasonableness tests.B.Passwords.C.Virus protection software.D.

Answer (A) is incorrect because biometric technologies do not eliminate the need for specialists who evaluate andmonitor security needs.Answer (B) is incorrect because checking to see if information is reasonable is related to input controls, not accesscontrols.Answer (C) is correct. The purpose of passwords is to prevent access by unauthorized users just as the moresophisticated control of employee biometric attributes. The use of passwords is an effective control in an onlinesystem to prevent unauthorized access to computer systems. However, biometric technologies are more sophisticatedand difficult to compromise.Answer (D) is incorrect because virus protection software prevents damage to data in a system, not access to asystem.

[1056] Gleim #: 9.135 -- Source: CPA AUD R02-20

Which of the following passwords would be most difficult to crack?

Gleim's CIA Test Prep: Part III: Business Analysis and Information Technology(1165 questions)

Copyright 2004 Gleim Publications, Inc. Page 418Printed for g j

Page 419: Part three cia_with_ answers

O?Ca!FlSiA.languageB.12 HOUSE 24C.pass56wordD.

Answer (A) is correct. To be effective, passwords should consist of random letters, symbols, and numbers and should notcontain words or phrases. Accordingly, computer system users should avoid employing words for or in their passwords.Answer (B) is incorrect because words should not be used for or in passwords.Answer (C) is incorrect because words should not be used for or in passwords.Answer (D) is incorrect because words should not be used for or in passwords.

[1057] Gleim #: 9.136 -- Source: CIA 1190 III-33

A company has a dial-up facility allowing the employees to gain access to the computer when they are not on the premises.The most effective way to prevent unauthorized access to the computer is to use

Call-back.A.Modem.B.Echo check.C.Console log.D.

Answer (A) is correct. The call-back technique prevents unauthorized access to the computer in a dial-upenvironment. The call-back technique is a two-step control. First, the connection is broken after the caller hasidentified him/herself and given the call number allowing reconnection. The system then checks for authorization. Ifit is verified, the computer is reconnected.Answer (B) is incorrect because the modem (modulator/demodulator) is a device that allows a connection between acomputer and a terminal to be made from a remote location through the use of telephone lines.Answer (C) is incorrect because the echo check is a control used to verify that information sent is identical to theinformation received. The information sent is echoed back by the recipient to the sender. If the message received bythe sender is not identical to what was sent, the transmission is attempted again.Answer (D) is incorrect because the console log has nothing to do with controlling access to the computer. The loglists all operating system activity, maintains an equipment use record, and identifies operator-initiated actions.

[1058] Gleim #: 9.137 -- Source: CIA 1192 I-33

Computer program libraries should be kept secure by

Installing a logging system for program access.A.Monitoring physical access to program library media.B.Restricting physical and logical access.C.Denying remote access via terminals.D.

Answer (A) is incorrect because installing a logging system for program access would permit detection ofunauthorized access but not prevent it.Answer (B) is incorrect because monitoring physical access to program library media would control onlyunauthorized physical access.Answer (C) is correct. An important operating control is to establish a library to preclude misplacement, misuse, ortheft of data files, programs, and documentation. A librarian should perform this custodianship function and beappropriately accountable. Restricting physical and logical access secures programs from unauthorized use, whetherin person or remotely via terminals.Answer (D) is incorrect because denying all remote access via terminals would likely be inefficient and would notsecure program libraries against physical access.

[1059] Gleim #: 9.138 -- Source: CPA AUD R02-9

Which of the following is an encryption feature that can be used to authenticate the originator of a document and ensurethat the message is intact and has not been tampered with?

Gleim's CIA Test Prep: Part III: Business Analysis and Information Technology(1165 questions)

Copyright 2004 Gleim Publications, Inc. Page 419Printed for g j

Page 420: Part three cia_with_ answers

Heuristic terminal.A.Perimeter switch.B.Default settings.C.Digital signatures.D.

Answer (A) is incorrect because the term "Heuristic terminal" is not meaningful in this context.Answer (B) is incorrect because the term "Perimeter switch" is not meaningful in this context.Answer (C) is incorrect because, in a computer program, a default setting is a value that a parameter will automaticallyassume unless specifically overridden.Answer (D) is correct. Businesses and others require that documents sent over the Internet be authentic. To authenticate adocument, a company or other user may transmit a complete plaintext document along with an encrypted portion of thesame document or another standard text that serves as a digital signature. If the plaintext document is tampered with, thetwo will not match.

[1060] Gleim #: 9.139 -- Source: CIA 1196 III-77

The encryption technique that requires two keys, a public key that is available to anyone for encrypting messages and aprivate key that is known only to the recipient for decrypting messages, is

Rivest, Shamir, and Adelman (RSA).A.Data encryption standard (DES).B.Modulator-demodulator.C.A cypher lock.D.

Answer (A) is correct. RSA is an encryption standard licensed to hardware and software vendors. Public-keyencryption requires management of fewer keys for a given client-server environment than does private-keyencryption. However, compared with DES, RSA entails more complex computations and therefore has a higherprocessing overhead. RSA requires two keys: The public key for encrypting messages is widely known, but theprivate key for decrypting messages is kept secret by the recipient.Answer (B) is incorrect because DES is a shared private-key method developed by the U.S. government. It encryptsdata into 64-bit blocks using a 56-bit key. DES requires only a single key for each pair of parties that want to sendeach other encrypted messages. DES is being replaced by AES, Advanced Encryption Standard, as the method ofchoice by the U.S. government.Answer (C) is incorrect because a modem is used for telecommunications.Answer (D) is incorrect because a cypher lock is a physical device.

[1061] Gleim #: 9.140 -- Source: Publisher

A client communicates sensitive data across the Internet. Which of the following controls would be most effective toprevent the use of the information if it were intercepted by an unauthorized party?

A firewall.A.An access log.B.Passwords.C.Encryption.D.

Answer (A) is incorrect because a firewall tries to prevent access from specific types of traffic to an internal network.After someone has obtained information from the site, a firewall cannot prevent its use.Answer (B) is incorrect because an access log only records attempted usage of a system.Answer (C) is incorrect because passwords prevent unauthorized users from accessing the system. If information hasalready been obtained, a password cannot prevent its use.Answer (D) is correct. Encryption technology converts data into a code. Encoding data before transmission overcommunications lines makes it more difficult for someone with access to the transmission to understand or modify itscontents.

Gleim's CIA Test Prep: Part III: Business Analysis and Information Technology(1165 questions)

Copyright 2004 Gleim Publications, Inc. Page 420Printed for g j

Page 421: Part three cia_with_ answers

[1062] Gleim #: 9.141 -- Source: CISATo ensure privacy in a public key encryption system, knowledge of which of the following keys would be required todecode the received message?

PrivateI.PublicII.

I.A.II.B.Both I and II.C.Neither I nor II.D.

Answer (A) is correct. In a public key system, the public key is used to encrypt the message prior to transmission,whereas the private key is needed to decrypt (decode) the message.Answer (B) is incorrect because the private key, not the public key, is needed to decrypt (decode) the message.Answer (C) is incorrect because the public key is not needed.Answer (D) is incorrect because the private key is needed to decrypt (decode) the message.

[1063] Gleim #: 9.142 -- Source: CIA 593 II-23

Data access security related to applications may be enforced through all the following except

User identification and authentication functions incorporated in the application.A.Utility software functions.B.User identification and authentication functions in access control software.C.Security functions provided by a database management system.D.

Answer (A) is incorrect because, although the trend is for this type of control function to be performed by othersoftware, most such controls still reside in application software.Answer (B) is correct. Utility programs perform routine functions (e.g., sorting and copying), are available to allusers, and are promptly available for many different applications. Utility programs can actually be a serious weaknessin data access security because some can bypass normal access controls.Answer (C) is incorrect because access control software has as one of its primary objectives improving data accesssecurity for all data on the system.Answer (D) is incorrect because most database management systems provide for improved data access security whilethey are running.

[1064] Gleim #: 9.143 -- Source: CIA 596 III-44

The duties properly assigned to an information security officer could include all of the following except

Developing an information security policy for the organization.A.Maintaining and updating the list of user passwords.B.Commenting on security controls in new applications.C.Monitoring and investigating unsuccessful access attempts.D.

Answer (A) is incorrect because developing an information security policy for the organization is an appropriate dutyof the information security officer.Answer (B) is correct. The information security officer should not know user passwords. They are normally stored ona computer in encrypted format, and users change them directly.Answer (C) is incorrect because commenting on security controls in new applications is an appropriate duty of theinformation security officer.Answer (D) is incorrect because monitoring and investigating unsuccessful access attempts is an appropriate duty ofthe information security officer.

[1065] Gleim #: 9.144 -- Source: CMA 693 4-10

Online access controls are critical for the successful operation of today’s computer systems. To assist in maintainingcontrol over such access, many systems use tests that are maintained through an internal access control matrix that consistsof

Gleim's CIA Test Prep: Part III: Business Analysis and Information Technology(1165 questions)

Copyright 2004 Gleim Publications, Inc. Page 421Printed for g j

Page 422: Part three cia_with_ answers

Authorized user code numbers, passwords, lists of all files and programs, and a record of the type of access each user isentitled to have to each file and program.

A.

Authorized user code numbers and passwords.B.A list of controls in the online system and a list of those individuals authorized to change and adjust these controls alongwith a complete list of files in the system.

C.

A completeness test, closed loop verification, and a compatibility test.D.

Answer (A) is correct. The list or tables of authorized users or devices are sometimes called access control matrices;however, they are more commonly referred to as device authorization tables. A device authorization table restricts fileaccess to those physical devices that logically need access. A device authorization table can contain such items asauthorized user code numbers, passwords, lists of all files and programs, and a record of the type of access each user isentitled to have to each file and program.Answer (B) is incorrect because it is not a complete list of the contents of an access control matrix.Answer (C) is incorrect because it is not information stored in an access control matrix.Answer (D) is incorrect because it is not a list of items stored in an access control matrix.

[1066] Gleim #: 9.145 -- Source: CMA 678 5-19

In an online, real-time system, which of the following is most likely to be used as backup for an application’s master filemaintained on magnetic disk?

At specified periods, the disk files are dumped to (copied on) magnetic tape; a transaction log is maintained as inputsare received and processed.

A.

A duplicate disk file is maintained and all activity is copied on magnetic tape continuously.B.The grandfather-father-son technique is employed to retain disk files.C.All source documents for transactions are retained.D.

Answer (A) is correct. When an application’s master file is maintained on magnetic disk in an online, real-timesystem, the backup procedure most likely to be used is the rollback and recovery method. This method involves thedumping of the master file’s contents and associated data structures onto a backup file. In the event of a loss of data,the dump is used together with the transaction log or file to reconstruct the master file.Answer (B) is incorrect because it describes dual logging.Answer (C) is incorrect because the grandfather-father-son technique is used as a backup procedure in batch-processing systems.Answer (D) is incorrect because it is not a sufficient backup procedure for an online, real-time system. Often, tangiblesource documents do not exist in an online system.

[1067] Gleim #: 9.146 -- Source: CMA 687 5-6

A checkpoint/restart procedure is primarily designed to recover from

Programming errors.A.Data input errors.B.Computer operator errors.C.Hardware failures.D.

Answer (A) is incorrect because it is not an error that a checkpoint/restart procedure is primarily designed to recoverfrom.Answer (B) is incorrect because it is not an error that a checkpoint/restart procedure is primarily designed to recoverfrom.Answer (C) is incorrect because it is not an error that a checkpoint/restart procedure is primarily designed to recoverfrom.Answer (D) is correct. Checkpoint/restart procedures involve capturing all the values of data and program indicatorsat specified points and storing these values in another file. If processing is interrupted, it can be resumed at the lastcheckpoint rather than at the beginning of the run.

[1068] Gleim #: 9.147 -- Source: CPA AUD R02-6

Which of the following procedures would an entity most likely include in its disaster recovery plan?

Convert all data from EDI format to an internal company format.A.Maintain a Trojan horse program to prevent illicit activity.B.Develop an auxiliary power supply to provide uninterrupted electricity.C.Store duplicate copies of files in a location away from the computer center.D.

Gleim's CIA Test Prep: Part III: Business Analysis and Information Technology(1165 questions)

Copyright 2004 Gleim Publications, Inc. Page 422Printed for g j

Page 423: Part three cia_with_ answers

Answer (A) is incorrect because converting all data from EDI format to an internal company format would reduce the abilityto process transactions with outside companies.Answer (B) is incorrect because a Trojan horse program is a computer program that appears to be legitimate but performssome illicit activity when it is run.Answer (C) is incorrect because the use of an uninterruptible power supply assures continued processing rather thanrecovery from a disaster.Answer (D) is correct. Off-site storage of duplicate copies of critical files protects them from a fire or other disaster at thecomputing facility. The procedure is part of an overall disaster recovery plan.

[Fact Pattern #79]An automobile and personal property insurer has decentralized its information processing to the extent that headquarters has lessprocessing capacity than any of its regional processing centers. These centers are responsible for initiating policies,communicating with policyholders, and adjusting claims. The company uses leased lines from a national telecommunicationscompany. Initially, the company thought there would be little need for interregion communication, but that has not been the case.The company underestimated the number of customers that would move between regions and the number of customers withclaims arising from accidents outside their regions. The company has a regional center in an earthquake-prone area and isplanning how to continue processing if that center, or any other single center, were unable to perform its processing.

[1069] Gleim #: 9.148 -- Source: CIA 595 III-71

(Refers to Fact Pattern #79)The company has considered several alternatives for replacing the hardware required for a regional center’s processing. Anadvantage of using a third-party cold site is that

Personnel employed at the site would be familiar with company operations.A.Travel expenses would be minimized for company personnel.B.No additional equipment would be required at the regional centers.C.The replacement site could be up and running in a few hours.D.

Answer (A) is incorrect because personnel employed at the site are not related to the company, and would thereforenot be familiar with company operations.Answer (B) is incorrect because depending on the location of the cold site, traveling expenses could increasedramatically.Answer (C) is correct. A cold site backup facility is a shell facility where the user can quickly install computerequipment and resume operations in the event of a disaster. The facility has all of the needed assets in place exceptthe needed computer equipment and is vendor-dependent for timely delivery of equipment. Accordingly, anadvantage of using a third-party cold site is the elimination of the need for additional equipment.Answer (D) is incorrect because it is not an advantage of using a third-party cold site; a few hours of downtime couldbe very costly to the organization.

[1070] Gleim #: 9.149 -- Source: CIA 595 III-72

(Refers to Fact Pattern #79)Unfortunately, the company has not revised its contingency plan since the time when its data processing was mostlycentralized at headquarters. The existing plan is likely to be out of date because of

Changes in equipment, data, and software.A.Inadequate processing capability at headquarters.B.Lack of arrangements for a backup site for headquarters.C.Personnel turnover at regional centers.D.

Answer (A) is correct. When a company decentralizes its information processing, many changes in equipment, data,and software occur as a result of the different processing methods. Since the company has not revised its contingencyplan since the time processing was centralized, the plan is most likely out of date as a result of the changes inequipment, data, and software.Answer (B) is incorrect because it is not a reason that would cause a contingency plan to be out of date.Answer (C) is incorrect because it is not a reason that would cause a contingency plan to be out of date.Answer (D) is incorrect because it is not a reason that would cause a contingency plan to be out of date.

[1071] Gleim #: 9.150 -- Source: CIA 595 III-73

(Refers to Fact Pattern #79)The company considered mirroring the data stored at each regional center at another center. A disadvantage of such anarrangement is

Gleim's CIA Test Prep: Part III: Business Analysis and Information Technology(1165 questions)

Copyright 2004 Gleim Publications, Inc. Page 423Printed for g j

Page 424: Part three cia_with_ answers

Lack of awareness at headquarters of the state of processing.A.Increased cost and complexity of network traffic.B.Interference of the mirrored data with original source data.C.Confusion on the part of insurance agents about where customer data are stored.D.

Answer (A) is incorrect because headquarters could monitor the network on a real-time basis and have complete awarenessof the state of processing.Answer (B) is correct. If data stored at one regional center is to mirror the data stored at another center, the most efficientmethod to ensure each center has the most current data is to transfer data across a network. Consequently, the cost ofnetwork traffic would increase dramatically. The complexity of the network would also increase as the network would needto provide a great deal of security when transferring data.Answer (C) is incorrect because adequate controls will ensure that the mirrored data will not affect the source data.Answer (D) is incorrect because the location of the data is not relevant to the insurance agents. The agents will retrievecustomer data through the network.

[1072] Gleim #: 9.151 -- Source: CPA AUD R98-6

A client is concerned that a power outage or disaster could impair the computer hardware’s ability to function as designed.The client desires off-site backup hardware facilities that are fully configured and ready to operate within several hours.The client most likely should consider a

Cold site.A.Cool site.B.Warm site.C.Hot site.D.

Answer (A) is incorrect because a cold site is a shell facility suitable for quick installation of computer equipment.Installing computer equipment would take more time in a cold site than in a hot site.Answer (B) is incorrect because it is a fabricated term that does not describe actual facilities.Answer (C) is incorrect because it is a fabricated term that does not describe actual facilities.Answer (D) is correct. A hot site is a service bureau that is a fully operational processing facility and is promptlyavailable in the case of a power outage or disaster.

[1073] Gleim #: 9.152 -- Source: Publisher

If High Tech Corporation’s disaster recovery plan requires fast recovery with little or no downtime, which of the followingbackup sites should it choose?

Hot site.A.Warm site.B.Cold site.C.Quick site.D.

Answer (A) is correct. A company uses a hot site backup when fast recovery is critical. The hot site includes allsoftware, hardware, and other equipment necessary for a company to carry out operations. Hot sites are expensive tomaintain and may be shared with other organizations with similar needs.Answer (B) is incorrect because a warm site provides an intermediate level of backup and causes more downtime thana hot site.Answer (C) is incorrect because a cold site is a shell facility suitable for quick installation of computer equipment.Disaster recovery would take more time in a cold site than a hot site.Answer (D) is incorrect because there is no backup site called a quick site.

[1074] Gleim #: 9.153 -- Source: CIA 1190 III-17

The system requiring the most extensive backup and recovery procedures is

A batch system for payroll processing.A.A database system for online order entry.B.A file-oriented system for billing clients.C.An indexed sequential access method file system for fixed asset accounting.D.

Gleim's CIA Test Prep: Part III: Business Analysis and Information Technology(1165 questions)

Copyright 2004 Gleim Publications, Inc. Page 424Printed for g j

Page 425: Part three cia_with_ answers

Answer (A) is incorrect because it does not require extensive backup and recovery procedures.Answer (B) is correct. Database systems require a more elaborate backup procedure than other systems. A database systemfor online entry would require almost continuous backup if data loss is to be minimized as transactions are processed on acontinuous basis, and without tangible source documentation. Backup procedures that could be employed for online orderentry include dual logging and rollback and recovery.Answer (C) is incorrect because it does not require extensive backup and recovery procedures.Answer (D) is incorrect because it does not require extensive backup and recovery procedures.

[1075] Gleim #: 9.154 -- Source: CIA 591 I-39

The total interruption of processing throughout a distributed information system is minimized by a control or conceptreferred to as

The system log.A.Fail-soft protection.B.Backup and recovery.C.Data file security.D.

Answer (A) is incorrect because a system log maintains a listing of the activity of the system.Answer (B) is correct. Fail-soft is a term associated with the ability to fail with minimum destruction. For example, adisk drive can be built to automatically park the heads when power fails. Although this procedure does not correct theproblem, it minimizes destruction and downtime.Answer (C) is incorrect because backup and recovery is a general term associated with the ability to recover lost files.Answer (D) is incorrect because data file security relates to the safeguard and protection of files.

[1076] Gleim #: 9.155 -- Source: CIA 1194 III-32

The primary purpose of a controller is to

Compensate for the differences in the rates of flow of data from input/output devices to the central processing unit(CPU).

A.

Compensate for the differences in the lengths of records in a file.B.Compensate for the differences in the execution time in various program instructions.C.Compensate for the differences in the volume of data that can be transmitted between communication devices.D.

Answer (A) is correct. Controllers are hardware units designed to operate (control) specific input or output units,e.g., terminals and printers. These devices eliminate the need for the CPU to operate such devices. The term “buffersand channels” is also sometimes used to describe devices that link the CPU with input/output devices and storageunits. Controllers may hold large blocks of logical records to compensate for the differences in the rates of flow ofdata between input/output devices and the central processing unit.Answer (B) is incorrect because compensating for the differences in the lengths of records in a file is achieved bysetting aside space in the file control block for the longest record that will be encountered.Answer (C) is incorrect because there is no compensating for the differences in the execution time in various programinstructions.Answer (D) is incorrect because multiplexors and concentrators compensate for the differences in the volume of datathat can be transmitted between communication devices.

[1077] Gleim #: 9.156 -- Source: CIA 1188 III-28

Many personal computers (PCs) have both a floppy disk drive and a hard disk drive. The major difference between the twotypes of storage is that a hard disk

Has a much larger storage capacity than a floppy disk and can also access information much more quickly.A.Is a direct-access storage medium, whereas a floppy disk is a sequential-access storage medium.B.Provides an automatic audit trail, whereas a floppy disk does not.C.Is suitable for an online system, whereas a floppy disk is not.D.

Gleim's CIA Test Prep: Part III: Business Analysis and Information Technology(1165 questions)

Copyright 2004 Gleim Publications, Inc. Page 425Printed for g j

Page 426: Part three cia_with_ answers

Answer (A) is correct. Floppy disks were developed by IBM out of plastic film that rotates the desired stored data to anopening in its cover where it is accessed by the computer’s read-write device. Hard disks are used in larger systems as wellas in personal computers. They require much less power and maintenance than floppies and can store far more data. Also,the read-write operations and disk rotation are much faster with hard disks.Answer (B) is incorrect because both floppy disk and hard disk are direct-access storage media.Answer (C) is incorrect because neither hard disk nor floppy disk provides an automatic audit trail.Answer (D) is incorrect because both floppy disk and hard disk are suited for an online system.

[1078] Gleim #: 9.157 -- Source: CIA 1191 III-32

Which of the following measures would indicate the computational power of a microprocessor?

Capacity of the hard disk.A.Main memory storage capacity.B.Number of bits processed per second.C.Read only memory.D.

Answer (A) is incorrect because capacity of the hard disk is a measure of memory.Answer (B) is incorrect because main memory storage capacity is a measure of memory.Answer (C) is correct. Processing speed is commonly calculated in terms of arithmetic-logic operations performedper second. This method is a function of the main processing chip’s cycle speed stated in gigahertz.Answer (D) is incorrect because read only memory is main memory that ordinarily cannot be modified by the user. Itis not a performance measure.

[1079] Gleim #: 9.158 -- Source: CIA 590 III-28

Which of the following computer hardware devices allows for an immediate update of merchandise inventory in a retailenvironment?

Inventory control terminal.A.Monitor.B.Video display terminal.C.Point-of-sale terminal.D.

Answer (A) is incorrect because it is not meaningful in this context.Answer (B) is incorrect because a monitor is a video display device that is the most common device used for human-computer interaction.Answer (C) is incorrect because a video display terminal is a video display device that is the most common deviceused for human-computer interaction.Answer (D) is correct. Point-of-sale terminals capture data by optical scanning or by keying. The data are thentransmitted to a CPU. The system permits collection of sales data, updating and ordering of inventory, pricing at thepoint of sale, and checking of customer credit cards.

[1080] Gleim #: 9.159 -- Source: CIA R98 III-60

What technology is needed in order to convert a paper document into a computer file?

Optical character recognition.A.Electronic data interchange.B.Bar-code scanning.C.Joining and projecting.D.

Answer (A) is correct. Optical character recognition (OCR) hardware scans special marks (e.g., bar codes),characters, or other codes and converts them into digital format.Answer (B) is incorrect because EDI is the communication of electronic documents directly from a computer in oneentity to a computer in another entity, for example, to order goods from a supplier or to transfer funds.Answer (C) is incorrect because bar-code scanning is just one form of OCR.Answer (D) is incorrect because joining and projecting are basic operations in a relational database.

Gleim's CIA Test Prep: Part III: Business Analysis and Information Technology(1165 questions)

Copyright 2004 Gleim Publications, Inc. Page 426Printed for g j

Page 427: Part three cia_with_ answers

[1081] Gleim #: 9.160 -- Source: CMA 1294 4-11A characteristic of a personal computer system that displays more than one program on the screen at the same time, placeseach program is in its own area of the screen, but permits only one program to be active, is

Windowing.A.Distributed processing.B.Context switching.C.File extension.D.

Answer (A) is correct. A graphical user interface is part of an operating system with which users may interact. It usesgraphic icons to represent activities, programs, and files. The computer mouse is used to make selections. Windows isa graphical user interface shell initially developed by Microsoft to run in conjunction with DOS. Newer operatingsystems also have this feature. Thus, windowing is the characteristic that allows a computer to display more than oneprogram on the screen at the same time. Each program has its own section of the screen, but only one program isactive.Answer (B) is incorrect because distributed processing is a means of assigning computer processing to varioussegments of a business, with some aspects centralized and some decentralized.Answer (C) is incorrect because context switching does not relate to the various segments on a computer screen.Answer (D) is incorrect because a file extension is a means of extending a logical collection of records.

[1082] Gleim #: 9.161 -- Source: CISA

Authentication is the process by which the

System verifies that the user is entitled to enter the transaction requested.A.System verifies the identity of the user.B.User identifies him/herself to the system.C.User indicates to the system that the transaction was processed correctly.D.

Answer (A) is incorrect because authentication involves verifying the identity of the user. This process does notnecessarily confirm the functions the user is authorized to perform.Answer (B) is correct. Identification is the process of uniquely distinguishing one user from all others.Authentication is the process of determining that individuals are who they say they are. For example, a password mayidentify but not authenticate its user if it is known by more than one individual.Answer (C) is incorrect because user identification to the system does not imply that the system has verified theidentity of the user.Answer (D) is incorrect because this procedure is an application control for accuracy of the transaction.

[1083] Gleim #: 10.1 -- Source: CISA, adapted

Advantages of using fiber optic cable are that

The signal is attenuated.I.Data are transmitted rapidly.II.It is small and flexible.III.It is unaffected by electrical interference.IV.

I and III.A.I and IV.B.I, II, and III.C.II, III, and IV.D.

Answer (A) is incorrect because attenuation of the signal is not an advantage of using fiber optics.Answer (B) is incorrect because attenuation of the signal is not an advantage of using fiber optics.Answer (C) is incorrect because attenuation of the signal is not an advantage of using fiber optics.Answer (D) is correct. A fiber optic cable uses light impulses that travel through clear flexible tubing half the size ofa human hair. Fiber optic cables are not subject to electrical interference and are highly reliable. They provide forextremely flexible and fast data transmission. The signal remains strong across long distances; i.e., it does not tend toweaken (attenuate).

Gleim's CIA Test Prep: Part III: Business Analysis and Information Technology(1165 questions)

Copyright 2004 Gleim Publications, Inc. Page 427Printed for g j

Page 428: Part three cia_with_ answers

[1084] Gleim #: 10.2 -- Source: CIA 592 III-37When two devices in a data communications system are communicating, there must be agreement as to how both data andcontrol information are to be packaged and interpreted. Which of the following terms is commonly used to describe thistype of agreement?

Asynchronous communication.A.Synchronous communication.B.Communication channel.C.Communication protocol.D.

Answer (A) is incorrect because asynchronous communication is a mode of transmission. Communication is indisjointed segments, typically character by character, preceded by a start code and ended by a stop code.Answer (B) is incorrect because synchronous communication is a mode of transmission in which a continuous streamof blocks of characters result in faster communications.Answer (C) is incorrect because a communication channel is a transmission link between devices in a network. Theterm is also used for a small processor that controls input-output devices.Answer (D) is correct. A protocol is a set of formal rules or conventions governing communication between asending and a receiving device. It prescribes the manner by which data are transmitted between these communicationsdevices. In essence, a protocol is the envelope within which each message is transmitted throughout a datacommunications network.

[1085] Gleim #: 10.3 -- Source: CIA 1195 III-58

A real estate brokerage firm is moving into a building that is already equipped with extensive telephone wiring. The firm isconsidering the installation of a digital private branch exchange (PBX) to connect computers and other office devices suchas copying machines, printers, and facsimile machines. A limitation of using a PBX-based system for this network is that

The firm would be dependent on others for system maintenance.A.The system cannot easily handle large volumes of data.B.Coaxial cabling would have to be installed throughout the building.C.Relocating devices in the office would be difficult and expensive.D.

Answer (A) is incorrect because the company would be responsible for all maintenance of the equipment, although itcould contract for service.Answer (B) is correct. A PBX has the advantage of using existing telephone lines and therefore not needing specialwiring. Moreover, equipment can be moved without necessitating rewiring. However, because PBX-based systemsuse telephone wiring (most often copper wire), they cannot easily handle large volumes of data.Answer (C) is incorrect because PBXs use telephone wiring. LANs typically require their own coaxial cabling.Answer (D) is incorrect because PBX-based systems do not require rewiring when devices are moved.

[1086] Gleim #: 10.4 -- Source: CIA 1195 III-40

Large organizations often have their own telecommunications networks for transmitting and receiving voice, data, andimages. Very small organizations, however, are unlikely to be able to make the investment required for their own networksand are more likely to use

Public switched lines.A.Fast-packet switches.B.Standard electronic mail systems.C.A WAN.D.

Answer (A) is correct. Companies can use public switched lines (phone lines) on a per-transmission basis. Thisoption is the most cost-effective way for low-volume users to conduct telecommunications.Answer (B) is incorrect because fast-packet switches receive transmissions from various devices, break the data intopackets, and route them over a network to their destination. They are typically installed by telecommunication utilitycompanies and other large companies that have their own networks.Answer (C) is incorrect because electronic mail systems do not allow for voice transmissions.Answer (D) is incorrect because large organizations would use a wide area network.

[1087] Gleim #: 10.5 -- Source: CIA 1196 III-72

Using a telecommunications provider affects in-house networks. To prepare for changes resulting from enhanced externalnetwork services, management should

Gleim's CIA Test Prep: Part III: Business Analysis and Information Technology(1165 questions)

Copyright 2004 Gleim Publications, Inc. Page 428Printed for g j

Page 429: Part three cia_with_ answers

Optimize in-house networks to avoid bottlenecks that would limit the benefits offered by the telecommunications provider.A.Plan for rapid implementation of new capabilities in anticipation of ready acceptance of the new technology.B.Downsize the company’s disaster recovery plan to recognize the increasing role of the telecommunications provider.C.Enhance the in-house network management to minimize dependence on the telecommunications provider for networkmanagement.

D.

Answer (A) is correct. To prepare the company for changes resulting from the enhanced external network services,management should take appropriate action. A number of bottlenecks may limit the benefits that can be derived from theexternal network. For example, conversion from analog to digital technology is necessary to achieve rapid improvements inbandwidth and speed and to improve access to telecommunications services. Furthermore, applications, systems software,and communications protocols must be able to process information in a format and in a manner acceptable to end users.Communications security also has heightened importance as greater amounts of data are transmitted from remote sites.Answer (B) is incorrect because resistance to change, inflexible organizational structures, and skepticism about thetechnology should be expected and must be successfully managed if the company is to reap the benefits.Answer (C) is incorrect because a company’s disaster recovery plan should be enhanced to ensure the reliability of thenetwork.Answer (D) is incorrect because network management may now be primarily a function, yet it will become more of apartnership arrangement with the communications carrier.

[1088] Gleim #: 10.6 -- Source: CMA 695 4-19

A local area network (LAN) is best described as a(n)

Computer system that connects computers of all sizes, workstations, terminals, and other devices within a limitedproximity.

A.

System to allow computer users to meet and share ideas and information.B.Electronic library containing millions of items of data that can be reviewed, retrieved, and analyzed.C.Method to offer specialized software, hardware, and data-handling techniques that improve effectiveness and reducecosts.

D.

Answer (A) is correct. A LAN is a local distributed computer system, often housed within a single building.Computers, communication devices, and other equipment are linked by cable. Special software facilitates efficientdata communication among the hardware devices.Answer (B) is incorrect because a LAN is more than a system to allow computer users to share information; it is aninterconnection of a computer system.Answer (C) is incorrect because a LAN is not a library.Answer (D) is incorrect because a LAN does not require specialized hardware.

[1089] Gleim #: 10.7 -- Source: Publisher

Which of the following statements about voice communications is true?

Modern voice recognition input devices have large vocabularies and short training periods.A.A voice output device converts speech into digital data.B.Cell phones and PCS services use the same frequency radio waves.C.Pagers can alert users to the receipt of messages, but cannot transmit text.D.

Answer (A) is correct. Voice recognition input devices provide an alternative to keyboard input. These systemscompare the speaker’s voice patterns with prerecorded patterns. Advanced systems now have large vocabularies andshorter training periods. They allow for dictation and are not limited to simple commands.Answer (B) is incorrect because a voice output device converts digital data into speech using prerecorded sounds.Answer (C) is incorrect because PCS services use lower-power, higher-frequency radio waves than cell phones.Answer (D) is incorrect because newer pager systems permit transmission of text messages.

[1090] Gleim #: 10.8 -- Source: CIA 1196 III-56

The Internet consists of a series of networks that include

Gateways to allow personal computers to connect to mainframe computers.A.Bridges to direct messages through the optimum data path.B.Repeaters to physically connect separate local area networks (LANs).C.Routers to strengthen data signals between distant computers.D.

Gleim's CIA Test Prep: Part III: Business Analysis and Information Technology(1165 questions)

Copyright 2004 Gleim Publications, Inc. Page 429Printed for g j

Page 430: Part three cia_with_ answers

Answer (A) is correct. The Internet facilitates information transfer between computers. Gateways are hardware or softwareproducts that allow translation between two different protocol families. For example, a gateway can be used to exchangemessages between different e-mail systems.Answer (B) is incorrect because routers are used to determine the best path for data.Answer (C) is incorrect because bridges connect LANs.Answer (D) is incorrect because repeaters strengthen signals.

[1091] Gleim #: 10.9 -- Source: Publisher

Which of the following is true concerning HTML?

The acronym stands for HyperText Material Listing.A.The language is among the most difficult to learn.B.The language is independent of hardware and software.C.HTML is the only language that can be used for Internet documents.D.

Answer (A) is incorrect because HTML is the acronym for HyperText Markup Language.Answer (B) is incorrect because the language is relatively easy to learn. Almost anyone can learn and use HTML, notjust computer programmers.Answer (C) is correct. HTML is the most popular language for authoring Web pages. It is hardware and softwareindependent, which means that it can be read by several different applications and on many different kinds ofcomputer operating systems. HTML uses tags to mark information for proper display on Web pages.Answer (D) is incorrect because a number of other languages can be used for Internet transmissions, including JAVAand XML.

[1092] Gleim #: 10.10 -- Source: CIA 592 III-28

Of the following, the greatest advantage of a database (server) architecture is

Data redundancy can be reduced.A.Conversion to a database system is inexpensive and can be accomplished quickly.B.Multiple occurrences of data items are useful for consistency checking.C.Backup and recovery procedures are minimized.D.

Answer (A) is correct. Data organized in files and used by the organization’s various applications programs arecollectively known as a database. In a database system, storage structures are created that render the applicationsprograms independent of the physical or logical arrangement of the data. Each data item has a standard definition,name, and format, and related items are linked by a system of pointers. The programs therefore need only to specifydata items by name, not by location. A database management system handles retrieval and storage. Because separatefiles for different applications programs are unnecessary, data redundancy can be substantially reduced.Answer (B) is incorrect because conversion to a database is often costly and time consuming.Answer (C) is incorrect because a traditional flat-file system, not a database, has multiple occurrences of data items.Answer (D) is incorrect because given the absence of data redundancy and the quick propagation of data errorsthroughout applications, backup and recovery procedures are just as critical in a database as in a flat-file system.

[1093] Gleim #: 10.11 -- Source: CIA 1188 III-36

An internal auditor encounters a batch-processed payroll in which each record contains the same type of data elements, inthe same order, with each data element needing the same number of storage spaces. Which file structure would mostappropriately be used to support this set of records?

Single flat file structure.A.Hierarchical structure.B.Network structure.C.Relational structure.D.

Gleim's CIA Test Prep: Part III: Business Analysis and Information Technology(1165 questions)

Copyright 2004 Gleim Publications, Inc. Page 430Printed for g j

Page 431: Part three cia_with_ answers

Answer (A) is correct. In a single flat file structure, all attributes and field lengths in a record are identical to those in theother records. The structure is typically a table or spreadsheet with records for rows and attributes for columns.Answer (B) is incorrect because a hierarchical or tree structure is used to express relationships in which one attribute oritem is related to many others in layers of subordinate records.Answer (C) is incorrect because a network structure expresses complex relationships in which many attributes are related tomany others.Answer (D) is incorrect because a relational structure is not unlike the flat structure but is far more sophisticated. It givesthe system the ability to handle many data relationships that were not anticipated by the designers. It uses a series of tablesin which each table defines a relationship.

[1094] Gleim #: 10.12 -- Source: CIA 1186 III-33

In an inventory system on a database management system (DBMS), one stored record contains part number, part name, partcolor, and part weight. These individual items are called

Fields.A.Stored files.B.Bytes.C.Occurrences.D.

Answer (A) is correct. A record is a collection of related data items (fields). A field (data item) is a group ofcharacters representing one unit of information.Answer (B) is incorrect because a file is a group or set of related records ordered to facilitate processing.Answer (C) is incorrect because a byte is a group of bits (binary digits). It represents one character.Answer (D) is incorrect because occurrences is not a meaningful term in this context.

[1095] Gleim #: 10.13 -- Source: CIA 592 III-32

An inventory clerk, using a computer terminal, views the following on screen: part number, part description, quantity on-hand, quantity on-order, order quantity and reorder point for a particular inventory item. Collectively, these data make up a

Field.A.File.B.Database.C.Record.D.

Answer (A) is incorrect because field refers to a single data item.Answer (B) is incorrect because file refers to multiple records.Answer (C) is incorrect because database refers to multiple files.Answer (D) is correct. A record is a collection of related data items (fields). A field (data item) is a group ofcharacters representing one unit of information. The part number, part description, etc., are represented by fields.

[1096] Gleim #: 10.14 -- Source: CPA AUD R98-3

An entity has the following invoices in a batch:

InvoiceNumber Product Quantity Unit Price

201 F10 150 $  5.00202 G15 200 $10.00203 H20 250 $25.00204 K35 300 $30.00

Which of the following numbers represents the record count?

1A.4B.810C.900D.

Gleim's CIA Test Prep: Part III: Business Analysis and Information Technology(1165 questions)

Copyright 2004 Gleim Publications, Inc. Page 431Printed for g j

Page 432: Part three cia_with_ answers

Answer (A) is incorrect because 1 is the number of batches.Answer (B) is correct. Input controls in batch computer systems are used to determine that no data are lost or added to thebatch. Depending on the sophistication of a particular system, control may be accomplished by using record counts, batchtotals, or hash totals. A record count establishes the number of source documents and reconciles it to the number of outputrecords. The total number of invoices processed is an example of a record count. In this case, the record count is 4.Answer (C) is incorrect because 810 is a hash total of the invoice numbers.Answer (D) is incorrect because 900 is the total quantity of items.

[1097] Gleim #: 10.15 -- Source: CIA 593 III-57

Which of the following is the elementary unit of data storage used to represent individual attributes of an entity?

Database.A.Data field.B.File.C.Record.D.

Answer (A) is incorrect because a database is an organized collection of files.Answer (B) is correct. A data item (or field) is a group of characters. It is used to represent individual attributes of anentity, such as an employee’s address. A field is an item in a record.Answer (C) is incorrect because a file is a collection of records.Answer (D) is incorrect because a record is a collection of data items.

[1098] Gleim #: 10.16 -- Source: CIA 594 III-17

A file-oriented approach to data storage requires a primary record key for each file. Which of the following is a primaryrecord key?

The vendor number in an accounts payable master file.A.The vendor number in a closed purchase order transaction file.B.The vendor number in an open purchase order master file.C.All of the answers are correct.D.

Answer (A) is correct. The primary record key uniquely identifies each record in a file. Because there is only onerecord for each vendor in an accounts payable master file, the vendor number would be the appropriate key.Answer (B) is incorrect because purchase order files can have multiple purchase orders made out to the same vendor.The primary key in purchase order files would be the purchase order number because it is the only unique identifierfor the record.Answer (C) is incorrect because purchase order files can have multiple purchase orders made out to the same vendor.The primary key in purchase order files would be the purchase order number because it is the only unique identifierfor the record.Answer (D) is incorrect because not all of the answer choices are correct.

[1099] Gleim #: 10.17 -- Source: CIA 1194 III-29

When required to read customer data, the operating system finds the primary key value in a file whose records contain keyvalues and their corresponding physical addresses. In this situation, the most likely file organization for customer data is

A direct-access file.A.An indexed sequential file.B.A sequential file.C.A text file.D.

Answer (A) is incorrect because a direct-access file has no index file.Answer (B) is correct. If records are stored sequentially on a direct-access storage device, the records can beaccessed directly using the indexed-sequential-access method. An index of key fields is maintained that lists thephysical location of the record corresponding to each key field.Answer (C) is incorrect because a sequential file has no index file.Answer (D) is incorrect because a text file would probably not be used in this situation.

Gleim's CIA Test Prep: Part III: Business Analysis and Information Technology(1165 questions)

Copyright 2004 Gleim Publications, Inc. Page 432Printed for g j

Page 433: Part three cia_with_ answers

[1100] Gleim #: 10.18 -- Source: CIA 1188 III-31A business has decided to use magnetic disks to store accounts receivable information. What data file concepts should beused to provide the ability to answer customer inquiries as they are received?

Sequential storage and chains.A.Sequential storage and indexes.B.Record keys, indexes, and pointers.C.Inverted file structure, indexes, and internal labels.D.

Answer (A) is incorrect because the ability to respond immediately to customers requires direct access.Answer (B) is incorrect because the ability to respond immediately to customers requires direct access.Answer (C) is correct. A record key is an attribute that uniquely identifies or distinguishes each record from theothers. An index is a table listing storage locations for attributes, often including those other than the unique recordkey attribute. A pointer is a data item that indicates the physical address of the next logically related record.Answer (D) is incorrect because internal labels are used to indicate various things to the computer, such as thecontents of various types of data storage media, the beginning of each file (with identification information), and theend of each file. However, they do not provide information for locating specific records in a file. An inverted filestructure (inverted list) is an index based on a secondary key, for example, years of experience rather than anemployee number (the primary key).

[1101] Gleim #: 10.19 -- Source: CIA 595 III-75

Auditors making database queries often need to combine several tables to get the information they want. One approach tocombining tables is known as

Extraction.A.Joining.B.Sorting.C.Summarization.D.

Answer (A) is incorrect because extraction selects data containing specified criteria from a data file; it does notcombine tables.Answer (B) is correct. In CAAT software packages, joining is the combining of data files based on a common dataelement. For example, if rows in a table containing information about specified parts have been selected, the resultcan be joined with a table that contains information about suppliers. The join operation may combine the two tablesusing the supplier number (assuming both tables contained this element) to provide information about the suppliersof particular parts.Answer (C) is incorrect because sorting allows the auditor to organize data by any data field, not combine tables.Answer (D) is incorrect because summarization reports on the information contained in several tables but does notcombine the tables.

[1102] Gleim #: 10.20 -- Source: CIA 597 III-50

Users making database queries often need to combine several tables to get the information they want. One approach tocombining tables is

Joining.A.Merging.B.Projecting.C.Pointing.D.

Answer (A) is correct. Joining is the combining of two or more relational tables based on a common data element.For example, if a supplier table contains information about suppliers and a parts table contains information aboutparts, the two tables can be joined using the supplier number (assuming both tables contain this attribute) to giveinformation about the supplier of particular parts.Answer (B) is incorrect because the three basic operations in a relational database are selecting, joining, andprojecting.Answer (C) is incorrect because projecting is the basic operation in a relational database that results in a subsetconsisting of columns (fields) in a table. This operation creates a new table containing only the required information.Answer (D) is incorrect because a pointer is a data element attached to a record that gives the address of anotherrecord.

[1103] Gleim #: 10.21 -- Source: CIA 596 III-65

All of the following are methods for distributing a relational database across multiple servers except

Gleim's CIA Test Prep: Part III: Business Analysis and Information Technology(1165 questions)

Copyright 2004 Gleim Publications, Inc. Page 433Printed for g j

Page 434: Part three cia_with_ answers

Snapshot (making a copy of the database for distribution).A.Replication (creating and maintaining replica copies at multiple locations).B.Normalization (separating the database into logical tables for easier user processing).C.Fragmentation (separating the database into parts and distributing where they are needed).D.

Answer (A) is incorrect because the snapshot technique makes duplicates to be stored at multiple locations.Answer (B) is incorrect because the replication technique makes duplicates to be stored at multiple locations. Changes areperiodically copied and sent to each location. If a database is small, storing multiple copies may be cheaper than retrievingrecords from a central site.Answer (C) is correct. A distributed database is stored in two or more physical sites. The two basic methods of distributinga database are partitioning and replication. However, normalization is a process of database design, not distribution.Normalization is the term for determining how groups of data items in a relational structure are arranged in records in adatabase. This process relies on “normal forms,” that is, conceptual definitions of data records and specified design rules.Normalization is intended to prevent inconsistent updating of data items. It is a process of breaking down a complex datastructure by creating smaller, more efficient relations, thereby minimizing or eliminating the repeating groups in eachrelation.Answer (D) is incorrect because fragmentation or partitioning stores specific records where they are most needed. Forexample, a financial institution may store a particular customer’s data at the branch where (s)he usually transacts his/herbusiness. If the customer executes a transaction at another branch, the pertinent data is retrieved via communications lines.

[1104] Gleim #: 10.22 -- Source: CIA 1196 III-75

In a database system, locking of data helps preserve data integrity by permitting transactions to have control of all the dataneeded to complete the transactions. However, implementing a locking procedure could lead to

Inconsistent processing.A.Rollback failures.B.Unrecoverable transactions.C.Deadly embraces (retrieval contention).D.

Answer (A) is incorrect because inconsistent processing occurs when a transaction has different effects depending onwhen it is processed. Data locking ensures consistent processing.Answer (B) is incorrect because rollback failure is the inability of the software to undo the effects of a transaction thatcould not be run to completion. A rollback failure is not caused by data locking. However, data locking may lead tosituations in which rollback is required.Answer (C) is incorrect because unrecoverable transactions are not a typical symptom of locking procedures.Answer (D) is correct. In a distributed processing system, the data and resources a transaction may update or useshould be held in their current status until the transaction is complete. A deadly embrace occurs when twotransactions need the same resource at the same time. If the system does not have a method to cope with the problemefficiently, response time worsens or the system eventually fails. The system should have an algorithm for undoingthe effects of one transaction and releasing the resources it controls so that the other transaction can run tocompletion.

[1105] Gleim #: 10.23 -- Source: CMA 695 4-20

One advantage of a database management system (DBMS) is

That each organizational unit takes responsibility and control for its own data.A.The cost of the data processing department decreases as users are now responsible for establishing their own datahanding techniques.

B.

A decreased vulnerability as the database management system has numerous security controls to prevent disasters.C.The independence of the data from the application programs, which allows the programs to be developed for theuser’s specific needs without concern for data capture problems.

D.

Gleim's CIA Test Prep: Part III: Business Analysis and Information Technology(1165 questions)

Copyright 2004 Gleim Publications, Inc. Page 434Printed for g j

Page 435: Part three cia_with_ answers

Answer (A) is incorrect because each organizational unit develops programs to make use of elements of a broad database.Answer (B) is incorrect because data handling techniques are still the responsibility of the data processing department; it isthe use of the data that is departmentalized.Answer (C) is incorrect because the DBMS is no safer than any other database system.Answer (D) is correct. A fundamental characteristic of databases is that applications are independent of the databasestructure; when writing programs or designing applications to use the database, only the name of the desired item isnecessary. Programs can be developed for the user’s specific needs without concern for data capture problems. Referencecan be made to the items using the data manipulation language, after which the DBMS takes care of locating and retrievingthe desired items. The physical or logical structure of the database can be completely altered without having to change anyof the programs using the data items; only the schema requires alteration.

[1106] Gleim #: 10.24 -- Source: CISA, adapted

Which of the following is a false statement about a database management system application environment?

Data are used concurrently by multiple users.A.Data are shared by passing files between programs or systems.B.The physical structure of the data is independent of user needs.C.Data definition is independent of any one program.D.

Answer (A) is incorrect because the advantage of a DBMS is that data can be used concurrently by multiple users.Answer (B) is correct. In this kind of system, applications use the same database. There is no need to pass filesbetween applications.Answer (C) is incorrect because when a DBMS is used, the physical structure of the data is independent of userneeds.Answer (D) is incorrect because when a DBMS is used, the data are defined independently of the needs of any oneprogram.

[1107] Gleim #: 10.25 -- Source: CIA 588 II-29

Which of the following should not be the responsibility of a database administrator?

Design the content and organization of the database.A.Develop applications to access the database.B.Protect the database and its software.C.Monitor and improve the efficiency of the database.D.

Answer (A) is incorrect because designing the content and organization of the database is a responsibility of thedatabase administrator.Answer (B) is correct. The database administrator (DBA) is the person who has overall responsibility for developingand maintaining the database. One primary responsibility is for designing the content of the database. Anotherresponsibility of the DBA is to protect and control the database. A third responsibility is to monitor and improve theefficiency of the database. The responsibility of developing applications to access the database belongs to systemsanalysts and programmers.Answer (C) is incorrect because protecting the database and its software is a responsibility of the databaseadministrator.Answer (D) is incorrect because monitoring and improving the efficiency of the database is a responsibility of thedatabase administrator.

[1108] Gleim #: 10.26 -- Source: Publisher

The responsibilities of a data administrator (DA) include monitoring

The database industry.A.The performance of the database.B.Database security.C.Backup of the system.D.

Answer (A) is correct. The DA handles administrative issues that arise regarding the database. The DA acts as anadvocate by suggesting new applications and standards. One of the DA’s responsibilities is to monitor the databaseindustry for new developments. In contrast, the database administrator (DBA) deals with the technical aspects of thedatabase.Answer (B) is incorrect because it is an example of the database administrator’s responsibilities.Answer (C) is incorrect because it is an example of the database administrator’s responsibilities.Answer (D) is incorrect because it is an example of the database administrator’s responsibilities.

Gleim's CIA Test Prep: Part III: Business Analysis and Information Technology(1165 questions)

Copyright 2004 Gleim Publications, Inc. Page 435Printed for g j

Page 436: Part three cia_with_ answers

[1109] Gleim #: 10.27 -- Source: CIA 589 III-39To trace data through several application programs, an auditor needs to know what programs use the data, which filescontain the data, and which printed reports display the data. If data exist only in a database system, the auditor couldprobably find all of this information in a

Data dictionary.A.Database schema.B.Data encryptor.C.Decision table.D.

Answer (A) is correct. The data dictionary is a file (possibly manual but usually computerized) in which the recordsrelate to specified data items. It contains definitions of data items, the list of programs used to process them, and thereports in which data are found. Only certain persons or entities are permitted to retrieve data or to modify data items.Accordingly, these access limitations are also found in the data dictionary.Answer (B) is incorrect because the schema describes the structure of the database.Answer (C) is incorrect because an encryptor encodes data.Answer (D) is incorrect because a decision table is a type of logic diagram that presents in matrix form the decisionpoints and related actions reflected in a computer program.

[1110] Gleim #: 10.28 -- Source: Publisher

All of the following are potential security issues for e-commerce except:

Correct identification of transacting parties.A.Proliferation of computer viruses.B.Determining who may rightfully make transaction decisions.C.Verification of payment data.D.

Answer (A) is incorrect because authentication is a security issue related to e-commerce.Answer (B) is correct. E-commerce is the purchase and sale of goods and services by electronic means. E-commercemay occur via online transactions on public networks, electronic data interchange (EDI), and e-mail. Security for e-commerce issues include the correct identification of transacting parties (authentication), determining who mayrightfully make decisions (authorization), and verification of payment data. While proliferation of computer viruses isa general security issue with regard to information systems, it is not a specific risk associated with e-commerce.Answer (C) is incorrect because authorization is a security issue related to e-commerce.Answer (D) is incorrect because verification of payment data is a security issue related to e-commerce.

[1111] Gleim #: 10.29 -- Source: CIA 594 III-26

Companies now can use electronic transfers to conduct regular business transactions. Which of the following terms bestdescribes a system in which an agreement is made between two or more parties to electronically transfer purchase orders,sales orders, invoices, and/or other financial documents?

Electronic mail (E-mail).A.Electronic funds transfer (EFT).B.Electronic data interchange (EDI).C.Electronic data processing (EDP).D.

Answer (A) is incorrect because e-mail can send text or document files, but the term encompasses a wide range oftransfers. EDI specifically applies to the system described in the question.Answer (B) is incorrect because electronic funds transfer (EFT) refers to the transfer of money.Answer (C) is correct. Electronic data interchange is the electronic transfer of documents between businesses. EDIwas developed to enhance just-in-time (JIT) inventory management. Advantages include speed, reduction of clericalerrors, and elimination of repetitive clerical tasks and their costs.Answer (D) is incorrect because electronic data processing (EDP) is a generic term for computerized processing oftransaction data within organizations.

[1112] Gleim #: 10.30 -- Source: CPA AUD R99-9

Which of the following is usually a benefit of transmitting transactions in an electronic data interchange (EDI)environment?

Gleim's CIA Test Prep: Part III: Business Analysis and Information Technology(1165 questions)

Copyright 2004 Gleim Publications, Inc. Page 436Printed for g j

Page 437: Part three cia_with_ answers

A compressed business cycle with lower year-end receivables balances.A.A reduced need to test computer controls related to sales and collections transactions.B.An increased opportunity to apply statistical sampling techniques to account balances.C.No need to rely on third-party service providers to ensure security.D.

Answer (A) is correct. EDI transactions are typically transmitted and processed in real time. Thus, EDI compresses thebusiness cycle by eliminating delays. The time required to receive and process an order, ship goods, and receive payment isgreatly reduced compared with that of a typical manual system. Accordingly, more rapid receipt of payment minimizesreceivables and improves cash flow.Answer (B) is incorrect because use of a sophisticated processing system would increase the need to test computer controls.Answer (C) is incorrect because computer technology allows all transactions to be tested rather than just a sample.Answer (D) is incorrect because EDI often uses a VAN (value-added network) as a third-party service provider, andreliance on controls provided by the VAN may be critical.

[Fact Pattern #80]A multinational company has an agreement with a value-added network (VAN) that provides the encoding and communicationstransfer for the company’s electronic data interchange (EDI) and electronic funds transfer (EFT) transactions. Before transfer ofdata to the VAN, the company performs online preprocessing of the transactions. The internal auditor is responsible for assessingpreprocessing controls. In addition, the agreement between the company and the VAN states that the internal auditor is allowed toexamine and report on the controls in place at the VAN on an annual basis. The contract specifies that access to the VAN canoccur on a surprise basis during the second or third quarter of the company’s fiscal year. This period was chosen so it would notinterfere with processing during the VAN’s peak transaction periods. This provision was not reviewed with internal auditing. Theannual audit plan approved by the board of directors specifies that a full audit would be done during the current year.

[1113] Gleim #: 10.31 -- Source: CIA 596 I-53

(Refers to Fact Pattern #80)Which one of the following would not be included as a reason for the company to use EFT with the EDI system?

To take advantage of the time lag associated with negotiable instruments.A.To allow the company to negotiate discounts with EDI vendors based upon prompt payment.B.To improve its cash management program.C.To reduce input time and input errors.D.

Answer (A) is correct. The float period is the time lag between transmittal of a regular check (a negotiableinstrument) and its clearance through regular banking channels. Float is eliminated by EFT.Answer (B) is incorrect because payment schedules may be based on the time required to process invoices, preparechecks, and transmit checks. Using EFT, payment is instantaneous, and payment schedules can be based on othercriteria, e.g., discounts for prompt payment.Answer (C) is incorrect because EFT allows for more effective control of payments and transfers among accounts.Answer (D) is incorrect because integration of EDI and EFT eliminates manual input of transaction data, a processthat introduces errors into the accounting system.

[1114] Gleim #: 10.32 -- Source: CIA 596 I-54

(Refers to Fact Pattern #80)Which one of the following is least likely to be recommended by the auditor when an EDI-EFT system is being designed?

The identity of the individual approving an electronic document should be stored as a data field.A.Disaster recovery plans should be established.B.Data security procedures should be written to prevent changes to data by unauthorized individuals.C.Remote access to electronic data should be denied.D.

Answer (A) is incorrect because approval information is needed to provide an audit trail.Answer (B) is incorrect because disaster recovery plans are needed to ensure that the company can continue tofunction if the system crashes.Answer (C) is incorrect because individuals should not be able to update data without proper identification andauthentication.Answer (D) is correct. One of the benefits of an EDI-EFT system is that it can provide remote access at any timefrom any place if telecommunications links are available. However, appropriate controls should prevent unauthorizedaccess.

Gleim's CIA Test Prep: Part III: Business Analysis and Information Technology(1165 questions)

Copyright 2004 Gleim Publications, Inc. Page 437Printed for g j

Page 438: Part three cia_with_ answers

[1115] Gleim #: 10.33 -- Source: CIA 593 III-38The emergence of electronic data interchange (EDI) as standard operating practice increases the risk of

Unauthorized third-party access to systems.A.Systematic programming errors.B.Inadequate knowledge bases.C.Unsuccessful system use.D.

Answer (A) is correct. EDI is the communication of electronic documents directly from a computer in one entity to acomputer in another entity. EDI for business documents between unrelated parties has the potential to increase therisk of unauthorized third-party access to systems because more outsiders will have access to internal systems.Answer (B) is incorrect because systematic programming errors are the result of misspecification of requirements orlack of correspondence between specifications and programs.Answer (C) is incorrect because inadequate knowledge bases are a function of lack of care in building them.Answer (D) is incorrect because a benefit of EDI is to improve the efficiency and effectiveness of system use.

[1116] Gleim #: 10.34 -- Source: CIA 596 III-64

Which of the following risks is not greater in an electronic funds transfer (EFT) environment than in a manual system usingpaper transactions?

Unauthorized access and activity.A.Duplicate transaction processing.B.Higher cost per transaction.C.Inadequate backup and recovery capabilities.D.

Answer (A) is incorrect because unauthorized access and activity is a risk specific to EFT.Answer (B) is incorrect because inaccurate transaction processing (including duplication) is a risk specific to EFT.Answer (C) is correct. EFT is a service provided by financial institutions worldwide that is based on EDI technology.EFT transaction costs are lower than for manual systems because documents and human intervention are eliminatedfrom the transactions process.Answer (D) is incorrect because inadequate backup and recovery capabilities is a risk specific to EFT.

[1117] Gleim #: 10.35 -- Source: CPA AUD R99-14

Which of the following are essential elements of the audit trail in an electronic data interchange (EDI) system?

Network and sender/recipient acknowledgments.A.Message directories and header segments.B.Contingency and disaster recovery plans.C.Trading partner security and mailbox codes.D.

Answer (A) is correct. An audit trail allows for the tracing of a transaction from initiation to conclusion. Networkand sender/recipient acknowledgments relate to the transaction flow and provide for the tracking of transactions.Answer (B) is incorrect because message directories and header segments provide information controlling themessage, such as originating and destination stations, message type and priority level, which are part of the messageand not the audit trail.Answer (C) is incorrect because, although contingency and disaster recovery plans are important controls, they do notrelate to the audit trail.Answer (D) is incorrect because, although maintaining control over security and mailbox codes is an importantcontrol, it does not relate to the audit trail.

[1118] Gleim #: 10.36 -- Source: Publisher

Which of the following is a false statement about XBRL?

XBRL is freely licensed.A.XBRL facilitates the automatic exchange of information.B.XBRL is used primarily in the U.S.C.XBRL is designed to work with a variety of software applications.D.

Gleim's CIA Test Prep: Part III: Business Analysis and Information Technology(1165 questions)

Copyright 2004 Gleim Publications, Inc. Page 438Printed for g j

Page 439: Part three cia_with_ answers

Answer (A) is incorrect because the AICPA-led consortium that developed XBRL has promoted the application as a freelylicensed product.Answer (B) is incorrect because XBRL will facilitate the exchange of information, for example, for reporting to the SEC.Answer (C) is correct. XBRL stands for eXtensible Business Reporting Language. It is being developed for business andaccounting applications. It is an XML-based application used to create, exchange, and analyze financial reportinginformation and is being developed for worldwide use.Answer (D) is incorrect because XBRL will allow exchange of data across many platforms and will soon be integrated intoaccounting software applications and products.

[1119] Gleim #: 10.37 -- Source: CPA AUD R99-11

Which of the following statements is true concerning internal control in an electronic data interchange (EDI) system?

Preventive controls generally are more important than detective controls in EDI systems.A.Control objectives for EDI systems generally are different from the objectives for other information systems.B.Internal controls in EDI systems rarely permit control risk to be assessed at below the maximum.C.Internal controls related to the segregation of duties generally are the most important controls in EDI systems.D.

Answer (A) is correct. In general, preventive controls are more important than detective controls because the benefitstypically outweigh the costs. In electronic processing, once a transaction is accepted, there is often little opportunityto apply detective controls. Thus, it is important to prevent errors or frauds before they happen.Answer (B) is incorrect because the basic control objectives are the same regardless of the nature of theprocessing:  to ensure the integrity of the information and to safeguard the assets.Answer (C) is incorrect because to gather sufficient evidence in a sophisticated computer system it is often necessaryto rely on the controls. Control risk may be assessed at below the maximum if relevant controls are identified andtested and if the resulting evidential matter provides the degree of assurance necessary to support the assessed level ofcontrol risk.Answer (D) is incorrect because the level of segregation of duties achieved in a manual system is usually not feasiblein a computer system.

[1120] Gleim #: 10.38 -- Source: CIA 1193 III-43

A control that a company can use to detect forged EDI messages is to

Acknowledge all messages initiated externally with confirming messages.A.Permit only authorized employees to have access to transmission facilities.B.Delay action on orders until a second order is received for the same goods.C.Write all incoming messages to a write-once/read-many device for archiving.D.

Answer (A) is correct. If the company acknowledges messages initiated externally, the alleged sender will have theopportunity to recognize that it had not sent the message and will then be able to notify the company of the potentialforgery. Then corrective action can be taken by the company.Answer (B) is incorrect because permitting only authorized employees to have access to transmission facilitiescontrols for unauthorized access to the facilities but would not detect forged EDI messages.Answer (C) is incorrect because delaying action on orders until a second order is received for the same goods defeatsthe purpose of using EDI, namely, rapid communication followed by rapid response.Answer (D) is incorrect because writing all incoming messages to a write-once/read-many device is a good practice,but it will not detect forgeries.

[1121] Gleim #: 10.39 -- Source: Publisher

Which of the following statements about SET (Secured Electronic Transaction) is false?

SET is a trademarked protocol that provides individualized security standards.A.SET is based on a hierarchy of certificate authorities.B.SET authenticates users with digital certificates and digital signatures.C.SET is supported through an organization called SETCo.D.

Gleim's CIA Test Prep: Part III: Business Analysis and Information Technology(1165 questions)

Copyright 2004 Gleim Publications, Inc. Page 439Printed for g j

Page 440: Part three cia_with_ answers

Answer (A) is correct. SET (Secured Electronic Transaction) is a trademarked protocol that provides a common securitystandards, especially with regard to Internet card purchases. It is supported by Visa and MasterCard through an organizationcalled SETCo. SET encrypts the details of payment transactions at all times so as to ensure privacy and data integrity. It alsoensures that the identities of buyers and sellers are authenticated using digital signatures and digital certificates. SET isbased on a hierarchy of certificate authorities that parallels financial relationships outside cyberspace.Answer (B) is incorrect because SET is based in a hierarchy of certificate authorities that parallels financial relationshipsoutside cyberspace.Answer (C) is incorrect because SET authenticates sellers and buyers using digital signatures and certificates.Answer (D) is incorrect because SET is supported by Visa and Mastercard through an organization called SETCo.

[1122] Gleim #: 10.40 -- Source: CIA 591 I-40

Inefficient use of excess computer equipment can be controlled by

Contingency planning.A.System feasibility studies.B.Capacity planning.C.Exception reporting.D.

Answer (A) is incorrect because contingency planning concerns the arrangements for alternative processing facilitiesin the event of equipment failure.Answer (B) is incorrect because feasibility study is one of the phases in the systems development life cycle.Answer (C) is correct. Planning is as important for the information systems function as for any other part of theorganization. The master plan for this function should be consistent with the strategic plan for the organization andinclude goals and objectives, an inventory of current capacity, and a forecast of future needs. The plan is the basis fordetermining hardware needs.Answer (D) is incorrect because exception reports are meant to list errata and bring them to the attention ofmanagement.

[1123] Gleim #: 10.41 -- Source: CIA 593 III-42

The best plan for responding to quickly changing information requirements is to foster

Greater online access to information systems.A.Competitive pressures for enhanced functions in systems.B.Closer linkage between organizational strategy and information.C.More widespread use of automated controls.D.

Answer (A) is incorrect because greater online access may or may not be helpful, depending on the userorganization’s needs.Answer (B) is incorrect because marketplace creates competitive pressures for enhanced functions in systems.Answer (C) is correct. An important management challenge is to integrate the planning, design, and implementationof complex application systems with the strategy of the organization, which will permit the best possible response toquickly changing information requirements.Answer (D) is incorrect because more pervasive use of automated controls may be independent of responding quicklyto changing information requirements.

[1124] Gleim #: 10.42 -- Source: Publisher

An enterprise resource planning (ERP) system integrates the organization’s computerized subsystems and may also providelinks to external parties. An advantage of ERP is that

The reengineering needed for its implementation should improve business processes.A.Customizing the software to suit the unique needs of the organization will facilitate upgrades.B.It can be installed by organizations of all sizes.C.The comprehensiveness of the system reduces resistance to change.D.

Gleim's CIA Test Prep: Part III: Business Analysis and Information Technology(1165 questions)

Copyright 2004 Gleim Publications, Inc. Page 440Printed for g j

Page 441: Part three cia_with_ answers

Answer (A) is correct. The benefits of ERP may significantly derive from the business process reengineering that is neededfor its implementation. Using ERP software that reflects the best practices forces the linked subunits in the organization notonly to redesign and improve their processes but also to conform to one standard.Answer (B) is incorrect because the disadvantages of ERP are its extent and complexity, which make customization of thesoftware difficult and costly.Answer (C) is incorrect because ERP software is costly and complex. It is usually installed only by the largest enterprises.Answer (D) is incorrect because implementing an ERP system is likely to encounter significant resistance because of itscomprehensiveness.

[1125] Gleim #: 10.43 -- Source: Publisher

A manufacturing resource planning (MRP II) system

Performs the same back-office functions for a manufacturer as an ERP system.A.Uses a master production schedule.B.Lacks the forecasting and budgeting capabilities typical of an ERP system.C.Performs the same front-office functions for a manufacturer as an ERP system.D.

Answer (A) is incorrect because an MRP II system does not integrate all the subsystems internal to the organization(back-office functions), such as human resources and customer service.Answer (B) is correct. Manufacturing resource planning (MRP II) continued the evolution begun with MRP I. It is aclosed-loop manufacturing system that integrates all facets of manufacturing, including production, sales, inventories,schedules, and cash flows. The same system is used for accounting and finance functions, which use the sametransactions and numbers. MRP II uses an MPS (master production schedule), a statement of the anticipatedmanufacturing schedule for selected items for selected periods. MRP also uses the MPS. Thus, MRP is a componentof an MRP II system.Answer (C) is incorrect because MRP II includes forecasting and planning capacities for generating cash and otherbudgets.Answer (D) is incorrect because MRP, MRP II and traditional ERP do not provide for front-office functions, that is,connections with customers, suppliers, owners, creditors, and strategic allies.

[1126] Gleim #: 10.44 -- Source: Publisher

In a traditional ERP system, the receipt of a customer order may result in

Customer tracking of the order’s progressI.Automatic replenishment of inventory by a supplierII.Hiring or reassigning of employeesIII.Automatic adjustment of output schedulesIV.

I, II, and IV only.A.I and III only.B.III and IV only.C.I, II, III, and IV.D.

Answer (A) is incorrect because ERP II but not traditional ERP is capable of customer tracking of the order’sprogress and automatic replenishment of inventory by a supplier.Answer (B) is incorrect because ERP II but not traditional ERP is capable of customer tracking of the order’sprogress and automatic replenishment of inventory by a supplier.Answer (C) is correct. The traditional ERP system is one in which subsystems share data and coordinate theiractivities. Thus, if marketing receives an order, it can quickly verify that inventory is sufficient to notify shipping toprocess the order. Otherwise, production is notified to manufacture more of the product, with a consequent automaticadjustment of output schedules. If materials are inadequate for this purpose, the system will issue a purchase order. Ifmore labor is needed, human resources will be instructed to reassign or hire employees. However, the subsystems in atraditional ERP system are internal to the organization. Hence, they are often called back-office functions. Theinformation produced is principally (but not exclusively) intended for internal use by the organization’s managers.

The current generation of ERP software (ERP II) has added front-office functions. Consequently, ERP II but nottraditional ERP is capable of customer tracking of the order’s progress and automatic replenishment of inventory by asupplier.Answer (D) is incorrect because ERP II but not traditional ERP is capable of customer tracking of the order’sprogress and automatic replenishment of inventory by a supplier.

Gleim's CIA Test Prep: Part III: Business Analysis and Information Technology(1165 questions)

Copyright 2004 Gleim Publications, Inc. Page 441Printed for g j

Page 442: Part three cia_with_ answers

[1127] Gleim #: 10.45 -- Source: PublisherWhat are the possible characteristics of a client-server configuration in a current ERP system?

Thin clients, local area network, single serverI.Fat clients, wide area network, multiple serversII.Fat clients, connection via Internet, and single serverIII.

I, II, and III.A.II and III only.B.II only.C.I only.D.

Answer (A) is correct. Current ERP systems have a client-server configuration with, possibly, scores or hundreds ofclient (user) computers. Clients may be thin or fat. So-called thin clients have little processing ability, but fat clientsmay have substantial processing power. The system may have multiple servers to run applications and containdatabases. The network architecture may be in the form of a local area network (LAN) or wide area network (WAN),or users may connect with the server(s) via the Internet. An ERP system may use almost any of the availableoperating systems and database management systems.Answer (B) is incorrect because an ERP system also may have thin clients connected via a LAN.Answer (C) is incorrect because an ERP system also may have thin clients connected via a LAN or the Internet withone server.Answer (D) is incorrect because an ERP system also may have fat clients connected via a WAN or the Internet tomultiple servers.

[1128] Gleim #: 10.46 -- Source: Publisher

A principal advantage of an ERP system is

Program-data dependence.A.Data redundancy.B.Separate data updating for different functions.C.Centralization of data.D.

Answer (A) is incorrect because an ERP system uses a central database and a database management system. Afundamental characteristic of a database is that applications are independent of the physical structure of the database.Writing programs or designing applications to use the database requires only the names of desired data items, nottheir locations.Answer (B) is incorrect because an ERP system eliminates data redundancy.Answer (C) is incorrect because an ERP system is characterized by one-time data updating for all organizationalfunctions.Answer (D) is correct. An advantage of an ERP system is the elimination of data redundancy through the use of acentral database. In principle, information about an item of data is stored once, and all functions have access to it.Thus, when the item (such as a price) is updated, the change is effectively made for all functions. The result isreliability (data integrity).

[1129] Gleim #: 10.47 -- Source: Publisher

The current generation of ERP software (ERP II) has added front-office functions like

Inventory control.A.Human resources.B.Purchasing.C.Customer service.D.

Answer (A) is incorrect because inventory control is a back-office function.Answer (B) is incorrect because human resources is a back-office function.Answer (C) is incorrect because purchasing is a back-office function.Answer (D) is correct. The current generation of ERP software (ERP II) has added front-office functions. Customerrelationship management applications in ERP II extend to customer service, finance-related matters, sales, anddatabase creation and maintenance. Integrated data are helpful in better understanding customer needs, such asproduct preference or location of retail outlets. Thus, the organization may be able to optimize its sales forecasts,product line, and inventory levels.

Gleim's CIA Test Prep: Part III: Business Analysis and Information Technology(1165 questions)

Copyright 2004 Gleim Publications, Inc. Page 442Printed for g j

Page 443: Part three cia_with_ answers

[1130] Gleim #: 10.48 -- Source: PublisherThe current generation of ERP software (ERP II) may include an advanced planning and scheduling system that

Determining the location of retail outlets.A.Connecting the organization with other members of a joint venture.B.Controls the flow of a manufacturer’s materials and components through the supply chain.C.Permits tracking of orders by customers.D.

Answer (A) is incorrect because customer relationship management applications in ERP II extend to customerservice, finance-related matters, sales, and database creation and maintenance. Integrated data are helpful in betterunderstanding customer needs, such as product preference or location of retail outlets.Answer (B) is incorrect because partner relationship management applications connect the organization not only withsuch partners as customers and suppliers but also with owners, creditors, and strategic allies (for example, othermembers of a joint venture).Answer (C) is correct. An advanced planning and scheduling system may be an element of a supply chainmanagement application for a manufacturer. It controls the flow of materials and components within the chain.Schedules are created given projected costs, lead times, and inventories.Answer (D) is incorrect because an advanced planning scheduling system is used by a manufacturer to control flowsthrough the supply chain. Other software permits customers to obtain information about order availability.

[1131] Gleim #: 10.49 -- Source: CPA AUD R02-5

Which of the following is a computer program that appears to be legitimate but performs some illicit activity when it isrun?

Hoax virus.A.Web crawler.B.Trojan horse.C.Killer application.D.

Answer (A) is incorrect because a hoax virus is a false notice about the existence of a computer virus. It is usuallydisseminated through use of distribution lists and is sent by e-mail or via an internal network.Answer (B) is incorrect because a web crawler (a spider or bot) is a computer program created to access and readinformation on websites. The results are included as entries in the index of a search engine.Answer (C) is correct. A Trojan horse is a computer program that appears friendly, for example, a game, but thatactually contains an application destructive to the computer system.Answer (D) is incorrect because a killer application is one that is so useful that it may justify widespread adoption ofa new technology.

[1132] Gleim #: 10.50 -- Source: CIA 591 III-24

The best preventive measure against a computer virus is to

Compare software in use with authorized versions of the software.A.Execute virus exterminator programs periodically on the system.B.Allow only authorized software from known sources to be used on the system.C.Prepare and test a plan for recovering from the incidence of a virus.D.

Answer (A) is incorrect because comparing software with authorized versions is a detective control used to determinewhether only authorized versions of the software are being used on the system.Answer (B) is incorrect because executing virus exterminator programs is a corrective control against a computervirus.Answer (C) is correct. Preventive controls are designed to prevent errors before they occur. Detective and correctivecontrols attempt to identify and correct errors. Preventive controls are usually more cost beneficial than detective orcorrective controls. Allowing only authorized software from known sources to be used on the system is a preventivemeasure. The authorized software from known sources is expected to be free of viruses.Answer (D) is incorrect because preparing and testing a plan for virus recovery is a corrective control against acomputer virus.

Gleim's CIA Test Prep: Part III: Business Analysis and Information Technology(1165 questions)

Copyright 2004 Gleim Publications, Inc. Page 443Printed for g j

Page 444: Part three cia_with_ answers

[1133] Gleim #: 10.51 -- Source: CIA 595 III-67Managers at a consumer products company purchased personal computer software from only recognized vendors, andprohibited employees from installing nonauthorized software on their personal computers. To minimize the likelihood ofcomputer viruses infecting any of its systems, the company should also

Restore infected systems with authorized versions.A.Recompile infected programs from source code backups.B.Institute program change control procedures.C.Test all new software on a stand-alone personal computer.D.

Answer (A) is incorrect because, if viruses infect a system, the company should restore the system with authorizedsoftware, but this procedure does not minimize the likelihood of initial infection.Answer (B) is incorrect because, if viruses infect programs that the company created, it should recompile theprograms from source code backups, but this procedure does not minimize the likelihood of initial infection.Answer (C) is incorrect because instituting program change control procedures is good practice but does notminimize the likelihood of the system’s being infected initially.Answer (D) is correct. Software from recognized sources should be tested in quarantine (for example, in atest/development machine or a stand-alone personal computer) because even vendor-supplied software may beinfected with viruses. The software should be run with a vaccine program and tested for the existence of logic bombs,etc.

[1134] Gleim #: 10.52 -- Source: CIA 1196 III-40

Which of the following is an indication that a computer virus is present?

Frequent power surges that harm computer equipment.A.Unexplainable losses of or changes to data.B.Inadequate backup, recovery, and contingency plans.C.Numerous copyright violations due to unauthorized use of purchased software.D.

Answer (A) is incorrect because power surges are caused by hardware or power supply problems.Answer (B) is correct. The effects of computer viruses range from harmless messages to complete destruction of alldata within the system. A symptom of a virus would be the unexplained loss of or change to data.Answer (C) is incorrect because inadequate back-up, recovery, and contingency plans are operating policyweaknesses.Answer (D) is incorrect because copyright violations represent policy or compliance problems.

[1135] Gleim #: 10.53 -- Source: CIA 1196 III-41

Which of the following operating procedures increases an organization’s exposure to computer viruses?

Encryption of data files.A.Frequent backup of files.B.Downloading public-domain software from websites.C.Installing original copies of purchased software on hard disk drives.D.

Answer (A) is incorrect because viruses are spread through the distribution of contaminated programs.Answer (B) is incorrect because backing up files does not increase the chances of a virus entering the computersystem.Answer (C) is correct. Viruses are spread through shared data. Downloading public-domain software carries a riskthat contaminated data may enter the computer.Answer (D) is incorrect because original copies of purchased software on hard disk drives should be free of viruses.

[1136] Gleim #: 10.54 -- Source: CIA 1195 III-37

An organization installed antivirus software on all its personal computers. The software was designed to prevent initialinfections, stop replication attempts, detect infections after their occurrence, mark affected system components, and removeviruses from infected components. The major risk in relying on antivirus software is that antivirus software may

Not detect certain viruses.A.Make software installation overly complex.B.Interfere with system operations.C.Consume too many system resources.D.

Gleim's CIA Test Prep: Part III: Business Analysis and Information Technology(1165 questions)

Copyright 2004 Gleim Publications, Inc. Page 444Printed for g j

Page 445: Part three cia_with_ answers

Answer (A) is correct. Antivirus software designed to identify and remove known viruses is sometimes known as a vaccine.A vaccine works only for known viruses and may not be effective for variants of those viruses or new viruses.Answer (B) is incorrect because having antivirus software is unlikely to make software installation overly complex.Answer (C) is incorrect because antivirus software need not interfere with system operations. Its execution can be scheduledin advance so as not to interfere with running programs.Answer (D) is incorrect because antivirus software can be set to execute at times when it would not consume too manysystem resources, e.g., at startup.

[1137] Gleim #: 10.55 -- Source: CIA 594 III-29

What is the best course of action to take if a program takes longer than usual to load or execute?

Test the system by running a different application program.A.Reboot the system.B.Run antivirus software.C.Back up the hard disk files to floppies.D.

Answer (A) is incorrect because running a different program as a test may cause the virus to spread and do additionaldamage.Answer (B) is incorrect because rebooting the system may cause the virus to spread and do additional damage.Answer (C) is correct. The described condition is a symptom of a virus. Many viruses will spread and causeadditional damage. Use of an appropriate antivirus program may identify and even eliminate a viral infection. Waysto minimize computer virus risk in a networked system include restricted access, regularly updated passwords,periodic testing of systems with virus detection software, and the use of anti-virus software on all shareware prior tointroducing it into the network.Answer (D) is incorrect because backing up hard disk files may cause the virus to spread and do additional damage.

[1138] Gleim #: 10.56 -- Source: CIA 1190 III-19

Six months after a disgruntled systems programmer was fired and passwords disabled, the company’s mainframe computerwas brought to a halt when it suddenly erased all of its own files and software. The most likely way the programmeraccomplished this was by

Returning to the computer center after 6 months.A.Planting a computer virus through the use of telephone access.B.Having an accomplice in the computer center.C.Implanting a virus in the operating system and executing it via a back door.D.

Answer (A) is incorrect because the programmer would most likely be denied access to the center.Answer (B) is incorrect because the programmer would not know the necessary passwords.Answer (C) is incorrect because collusion is less likely than individual wrongdoing.Answer (D) is correct. Viruses are a form of computer sabotage. They are programs hidden within other programsthat have the capacity to duplicate themselves and infect other systems. Sharing of storage media or participation incomputer networks creates exposure to viruses. Viruses may result in actions ranging from harmless pranks to erasureof files and programs. A back door is a shortcut created in an operating system that permits a programmer simpleaccess to the system.

[1139] Gleim #: 10.57 -- Source: CIA 1195 III-61

Because of competitive pressures to be more responsive to their customers, some organizations have connected theirinternal personal computer networks through a host computer to outside networks. A risk of this practice is that

Viruses may gain entry to one or more company systems.A.Uploaded files may not be properly edited and validated.B.Data downloaded to the personal computers may not be sufficiently timely.C.Software maintenance on the personal computers may become more costly.D.

Gleim's CIA Test Prep: Part III: Business Analysis and Information Technology(1165 questions)

Copyright 2004 Gleim Publications, Inc. Page 445Printed for g j

Page 446: Part three cia_with_ answers

Answer (A) is correct. Viruses are harmful programs that disrupt memory and processing functions and may destroy data.They spread from network to network, from infected diskettes, or from infected machines. Hence, connecting all networkedpersonal computers through a host computer to outside networks increases the exposure of all of a company’s computers toviruses.Answer (B) is incorrect because whether uploaded files are properly edited and validated is independent of whether externallinks to other networks exist.Answer (C) is incorrect because whether data downloaded to the personal computers is sufficiently timely is independent ofwhether external links to other networks exist.Answer (D) is incorrect because whether software maintenance on the personal computers becomes more costly isindependent of whether external links to other networks exist.

[1140] Gleim #: 10.58 -- Source: Publisher

Attacks on computer networks may take many forms. Which of the following uses the computers of innocent partiesinfected with Trojan horse programs?

A distributed denial-of-service attack.A.A man-in-the-middle attack.B.A brute-force attack.C.A password-cracking attack.D.

Answer (A) is correct. A denial-of-service (DOS) attack is an attempt to overload a system (e.g., a network or Webserver) with false messages so that it cannot function (a system crash). A distributed DOS attack comes from multiplesources, for example, the machines of innocent parties infected by Trojan horses. When activated, these programssend messages to the target and leave the connection open. A DOS may establish as many network connections aspossible to exclude other users, overload primary memory, or corrupt file systems.Answer (B) is incorrect because a man-in-the-middle attack takes advantage of network packet sniffing and routingand transport protocols to access packets flowing through a network.Answer (C) is incorrect because a brute-force attack uses password cracking software to try large numbers of letterand number combinations to access a network.Answer (D) is incorrect because password-cracking software is used to access a network by using a large number ofletter and number combinations.

[1141] Gleim #: 10.59 -- Source: Publisher

Spoofing is one type of online activity used to launch malicious attacks. Spoofing is

Trying large numbers of letter and number combinations to access a network.A.Eavesdropping on information sent by a user to the host computer of a website.B.Accessing packets flowing through a network.C.Identity misrepresentation in cyberspace.D.

Answer (A) is incorrect because a brute-force attack uses password cracking software to try large numbers of letterand number combinations to access a network.Answer (B) is incorrect because sniffing is use of software to eavesdrop on information sent by a user to the hostcomputer of a website.Answer (C) is incorrect because a man-in-the-middle attack takes advantage of network packet sniffing and routingand transport protocols to access packets flowing through a network.Answer (D) is correct. Passwords, user account numbers, and other information may be stolen using techniques suchas Trojan horses, IP spoofing, and packet sniffers. Spoofing is identity misrepresentation in cyberspace, for example,by using a false website to obtain information about visitors.

[1142] Gleim #: 10.60 -- Source: Publisher

An organization’s computer system should have an intrusion detection system (IDS) if it has external connections. An IDS

Must monitor every call on the system as it occurs.A.May examine only packets with certain signatures.B.Uses only knowledge-based detection.C.Uses only behavior-based detection.D.

Gleim's CIA Test Prep: Part III: Business Analysis and Information Technology(1165 questions)

Copyright 2004 Gleim Publications, Inc. Page 446Printed for g j

Page 447: Part three cia_with_ answers

Answer (A) is incorrect because a host IDS provides maximum protection only when the software is installed on eachcomputer. It may operate in the following ways:  The aggressive response is to monitor every call on the operating systemand application as it occurs. A less effective method of preventing attacks is analysis of access log files. A host IDS mayalso identify questionable processes and verify the security of system files.Answer (B) is correct. A network IDS works by using sensors to examine packets traveling on the network. Each sensormonitors only the segment of the network to which it is attached. A packet is examined if it matches a signature. Stringsignatures (certain strings of text) are potential signs of attack. Port signatures alert the IDS that a point subject to frequentintrusion attempts may be under attack. A header signature is a suspicious combination in a packet header.Answer (C) is incorrect because an IDS is not limited to knowledge-based detection. Knowledge-based detection is basedon information about the system’s weaknesses and searches for intrusions that take advantage of them.Answer (D) is incorrect because an IDS is not limited to behavior-based detection. Behavior-based detection presumes thatan attack will cause an observable anomaly. Actual and normal system behavior (a model of expected operations) arecompared. A discrepancy results in an alert.

[1143] Gleim #: 10.61 -- Source: CPA AUD R99-8

Which of the following is usually a benefit of using electronic funds transfer (EFT) for international cash transactions?

Improvement of the audit trail for cash receipts and disbursements.A.Creation of self-monitoring access controls.B.Reduction of the frequency of data entry errors.C.Off-site storage of source documents for cash transactions.D.

Answer (A) is incorrect because the audit trail is typically less apparent in an electronic environment than in a manualenvironment.Answer (B) is incorrect because a key control is management’s establishment and monitoring of access controls.Answer (C) is correct. The processing and transmission of electronic transactions, such as EFTs, virtually eliminateshuman interaction. This process not only helps eliminate errors but also allows for the rapid detection and recoveryfrom errors when they do occur.Answer (D) is incorrect because source documents are often eliminated in EFT transactions.

[1144] Gleim #: 10.62 -- Source: CIA 596 III-57

Which of the following is likely to be a benefit of electronic data interchange (EDI)?

Increased transmission speed of actual documents.A.Improved business relationships with trading partners.B.Decreased liability related to protection of proprietary business data.C.Decreased requirements for backup and contingency planning.D.

Answer (A) is incorrect because EDI transmits document data, not the actual document.Answer (B) is correct. Electronic data interchange is the electronic transfer of documents between businesses. EDIwas developed to enhance just-in-time (JIT) inventory management. Advantages include speed, reduction of clericalerrors, and elimination of repetitive clerical tasks and their costs. Improved business relationships result because ofthe mutual benefits conferred by EDI. Accordingly, some organizations require EDI.Answer (C) is incorrect because liability for protection of a trading partner's proprietary business data is a major riskthat must be addressed by the control structure.Answer (D) is incorrect because backup and contingency planning requirements are not diminished by use of EDItechnology. Materials purchased from non-EDI vendors are delivered to the receiving dock and recorded manually onreceiving reports. Copies of these reports are given to the purchasing and accounts payable departments. The internalaudit department is scheduled to complete a full audit of the purchasing and accounts payable cycle before the end ofthe year. However, there are severe time pressures because other matters delayed the start of the audit.

[1145] Gleim #: 10.63 -- Source: CIA 1192 III-30

Which of the following is an accepted example of electronic data interchange?

Request for an airline reservation by a travel agent.A.Withdrawal of cash from an automated teller by a bank's customer.B.Transfer of summary data from a local area network to a centralized mainframe.C.Placement of order entry transactions from a customer to its supplier.D.

Gleim's CIA Test Prep: Part III: Business Analysis and Information Technology(1165 questions)

Copyright 2004 Gleim Publications, Inc. Page 447Printed for g j

Page 448: Part three cia_with_ answers

Answer (A) is incorrect because a request for an airline reservation requires an online, real-time reservations system.Answer (B) is incorrect because withdrawal of cash from an automated teller is accomplished via online transactions tocopies of master files.Answer (C) is incorrect because the transfer of summary data to headquarters may be accomplished with point-to-pointcommunications, known as distributed computing.Answer (D) is correct. EDI is the communication of electronic documents directly from a computer in one entity to acomputer in another entity. Placement of order entry transactions from a customer to its supplier is an accepted use of EDIbetween trading partners.

[1146] Gleim #: 10.64 -- Source: CIA 596 I-55

An internal auditor was performing an operational audit of the purchasing and accounts payable system. The auditobjective was to identify changes in processes that would improve efficiency and effectiveness. Which of the followingstatements support the auditor's recommendation that electronic data interchange (EDI) be implemented within a company?

There is a small number of transactions.I.There is a time-sensitive just-in-time purchase environment.II.There is a large volume of custom purchases.III.There are multiple transactions with the same vendor.IV.

I only.A.II and IV only.B.I and III.C.II, III, and IV.D.

Answer (A) is incorrect because a large volume of repetitive purchases from the same vendors suggests that EDIshould be implemented.Answer (B) is correct. EDI is advantageous in a JIT environment because it provides the capacity for instantaneousordering. Moreover, a JIT environment already is characterized by the close vendor-purchaser cooperation requiredby an EDI system. Another reason for implementing an EDI system is that the purchaser has a large volume oftransactions with the same vendor(s). Otherwise, EDI may not be cost efficient.Answer (C) is incorrect because a large volume of repetitive purchases from the same vendors suggests that EDIshould be implemented.Answer (D) is incorrect because a large volume of custom purchases is inappropriate for EDI. The absence ofstandardization imposes unacceptable costs.

[1147] Gleim #: 10.65 -- Source: CIA 596 I-47

The auditor determined that the risks associated with the EDI purchases were less than the risks associated with thepurchases made through the traditional system. Which one of the following factors best supports this prioritization of risks?

There are three vendors connected through EDI.A.About half of the materials are purchased through EDI.B.The internal auditors were involved with systems development and testing of the EDI software.C.The external auditor did not examine EDI purchase controls during the annual financial audit.D.

Answer (A) is incorrect because the number of vendors does not indicate the size of the purchases.Answer (B) is incorrect because the amount of purchases is equally divided between the EDI and non-EDI systemsand does not provide a basis for prioritizing risks.Answer (C) is correct. Sound controls mitigate the risks associated with EDI. The question states that the internalauditing department's prior involvement consisted of assessing and testing the EDI system. This review found nosignificant problems. Accordingly, the risk of the EDI system is decreased.Answer (D) is incorrect because failure to examine EDI purchase controls increases risk.

[1148] Gleim #: 10.66 -- Source: CIA 596 III-59

In a review of an EDI application using a third-party service provider, the auditor should

Ensure encryption keys meet ISO standardsI.Determine whether an independent review of the service provider’s operation has been conductedII.Verify that only public-switched data networks are used by the service providerIII.Verify that the service provider’s contracts include necessary clauses, such as the right to auditIV.

Gleim's CIA Test Prep: Part III: Business Analysis and Information Technology(1165 questions)

Copyright 2004 Gleim Publications, Inc. Page 448Printed for g j

Page 449: Part three cia_with_ answers

I and II.A.I and IV.B.II and III.C.II and IV.D.

Answer (A) is incorrect because using a third-party service provider does not require encryption.Answer (B) is incorrect because using a third-party service provider does not require encryption.Answer (C) is incorrect because use of public-switched data networks is not a requirement of EDI.Answer (D) is correct. An auditor should review trading partner agreements and contracts with third-party serviceproviders. These documents should contain necessary clauses and appropriately limit liabilities. Moreover, legal counselshould have reviewed the agreements or contracts. An auditor should also determine whether the third-party serviceprovider’s operations and controls have been independently reviewed (e.g., by public accountants).

[1149] Gleim #: 10.67 -- Source: CIA 593 III-31

Which of the following represents the greatest exposure to the integrity of electronic funds transfer data transmitted from aremote terminal?

Poor physical access controls over the data center.A.Network viruses.B.Poor system documentation.C.Leased telephone circuits.D.

Answer (A) is incorrect because poor physical access controls represent a secondary exposure for compromise ofremote data communications lines.Answer (B) is incorrect because network viruses represent a secondary exposure for compromise of remote datacommunications lines.Answer (C) is incorrect because poor system documentation represent a secondary exposure for compromise ofremote data communications lines.Answer (D) is correct. Leased telephone circuits represent a direct exposure to the risk of breached data integrity.They use public lines that can be easily identified and tapped.

[1150] Gleim #: 10.68 -- Source: CIA 1196 III-63

Which of the following is a risk that is higher when an electronic funds transfer (EFT) system is used?

Improper change control procedures.A.Unauthorized access and activity.B.Insufficient online edit checks.C.Inadequate backups and disaster recovery procedures.D.

Answer (A) is incorrect because improper change control procedures is a risk common to all information technologyenvironments. This risk is not higher than for other systems.Answer (B) is correct. Unauthorized access to money transfer activities or data is an inherent and unique risk of EFTsystems. An unauthorized person may attempt to read, alter, or delete information in data files or to enter authorizedfund transfers. Hence, in the financial services industry, protection of confidential customer transactions is especiallyimportant. Moreover, unauthorized transfers subject a financial institution to a direct risk of serious loss.Answer (C) is incorrect because insufficient online edit checks is a risk common to all information technologyenvironments. This risk is not higher than for other systems.Answer (D) is incorrect because inadequate backups and disaster recovery procedures is a risk common to allinformation technology environments. This risk is not higher than for other systems.

[1151] Gleim #: 10.69 -- Source: CIA 1193 III-47

Regardless of whether a company develops, buys, leases, or pays for the use of the software for EDI transmissions, internalaudit should be responsible for evaluating whether the software

Was developed in a controlled environment.A.Is backed up adequately to permit recovery.B.Was acquired with adequate review by legal counsel.C.Meets business objectives.D.

Gleim's CIA Test Prep: Part III: Business Analysis and Information Technology(1165 questions)

Copyright 2004 Gleim Publications, Inc. Page 449Printed for g j

Page 450: Part three cia_with_ answers

Answer (A) is incorrect because determining whether the software was developed in a controlled environment is feasibleonly if the company develops it.Answer (B) is incorrect because, if the company developed and maintained its own software, internal audit would beresponsible for evaluating that the software is backed up adequately to permit recovery in the event of a system failure.Answer (C) is incorrect because, if the company purchased, leased, or paid for the use of the software, internal audit wouldbe responsible for evaluating that the software was acquired with legal counsel review of contract terms.Answer (D) is correct. An EDI application should meet business objectives and satisfy user and control requirements. Theinternal auditors should consider the organization's important EDI applications because they represent significant riskexposures and control problems. This role is within the scope of work of the internal auditors, who are charged withexamining and evaluating internal control and the quality of performance in carrying out assigned responsibilities.

[1152] Gleim #: 10.70 -- Source: CIA 597 III-52

A company using EDI made it a practice to track the functional acknowledgments from trading partners and to issuewarning messages if acknowledgments did not occur within a reasonable length of time. What risk was the companyattempting to address by this practice?

Transactions that have not originated from a legitimate trading partner may be inserted into the EDI network.A.Transmission of EDI transactions to trading partners may sometimes fail.B.There may be disagreement between the parties as to whether the EDI transactions form a legal contract.C.EDI data may not be accurately and completely processed by the EDI software.D.

Answer (A) is incorrect because unauthorized access to the EDI system should be prevented by procedures thatensure the effective use of passwords, and data integrity and privacy should be maintained through the use ofencryption and authentication measures.Answer (B) is correct. Tracking of customers’ functional acknowledgments, when required, will help to ensuresuccessful transmission of EDI transactions. Some possible controls include the provision of end-to-endacknowledgments, particularly when multiple, interconnected networks are involved, and maintenance of a tickler fileof outstanding functional acknowledgments, with issuance of warnings for those that are overdue.Answer (C) is incorrect because contractual issues should be resolved by the company and its trading partners beforeEDI is implemented.Answer (D) is incorrect because the risk that EDI data may not be completely and accurately processed is minimizedby system-based controls, not by acknowledgments from trading partners.

[1153] Gleim #: 10.71 -- Source: CIA 591 III-50

An audit of the electronic data interchange (EDI) area of a banking group revealed the facts listed below. Which oneindicates the need for improved internal control?

Employees may only access the computer system via an ID and an encrypted password.A.The system employs message sequencing as a way to monitor data transmissions.B.Certain types of transactions may only be made at specific terminals.C.Branch office employees may access the mainframe with a single call via modem.D.

Answer (A) is incorrect because employee access to the computer system via an ID and an encrypted password isconsidered acceptable. Encrypted passwords further decrease the likelihood of unauthorized access.Answer (B) is incorrect because message sequencing detects unauthorized access by numbering each message andincrementing each message by one more than the last one sent. This procedure will detect a gap or duplicate.Answer (C) is incorrect because allowing certain types of transactions (such as payroll transactions) to be made onlyat specific terminals minimizes the likelihood of unauthorized access.Answer (D) is correct. The system should employ automatic dial-back to prevent intrusion by unauthorized parties.This procedure accepts an incoming modem call, disconnects, and automatically dials back a prearranged number toestablish a permanent connection for data transfer or inquiry.

[1154] Gleim #: 10.72 -- Source: CIA 596 I-46

Before authorizing payment of an EDI invoice, the computer automatically compares the invoice with the purchase orderand receiving report data. When the system was being developed, the auditor reviewed the payment authorization programand made recommendations. Which one of the following was most likely recommended by the auditor for the situation inwhich the quantity invoiced is greater than the quantity received?

Gleim's CIA Test Prep: Part III: Business Analysis and Information Technology(1165 questions)

Copyright 2004 Gleim Publications, Inc. Page 450Printed for g j

Page 451: Part three cia_with_ answers

Prepare an exception report.A.Pay the amount billed and adjust the inventory for the difference.B.Return the invoice to the vendor.C.Authorize payment of the full invoice, but maintain an open purchase order record for the missing goods.D.

Answer (A) is correct. An exception report (error listing) should be issued so that company personnel can investigate thediscrepancy, determine its cause, and take appropriate corrective action.Answer (B) is incorrect because the company should not pay for goods not received.Answer (C) is incorrect because the company should first determine the cause of the discrepancy.Answer (D) is incorrect because the company should not pay for goods not received.

[1155] Gleim #: 10.73 -- Source: CIA 593 III-59

Consider the following computer applications:

(a) At a catalog sales firm, as phone orders are entered into their computer, both inventory andcredit are immediately checked.

(b) A manufacturer's computer sends the coming week's production schedule and parts orders to asupplier's computer.

Which statement below is true for these applications?

Both applications are examples of electronic data interchange (EDI).A.Both applications are examples of online real-time processing (OLRT).B.The first application is an example of EDI and the second an example of OLRT.C.The first application is an example of OLRT and the second an example of EDI.D.

Answer (A) is incorrect because catalog phone orders do not involve EDI. They are not computer to computer.Answer (B) is incorrect because the ordering of parts by computer is not OLRT. Processing does not occur.Answer (C) is incorrect because the catalog phone orders are OLRT, and the ordering of parts by computer is EDI.Answer (D) is correct. The catalog sales firm is using an online processing system known as an online, real-timesystem (OLRT). OLRT systems handle transactions as they are entered, processing the input and providing outputsoon enough to affect a current decision making process, e.g., inventory and credit status of phone call orders. Thesending of upcoming production schedules and parts orders by and manufacturer's computer to a supplier's computeris an example of electronic data interchange (EDI). EDI is the communication of electronic documents directly fromone entity's computer to another's computer.

[1156] Gleim #: 10.74 -- Source: CIA 1193 III-51

If implementing EDI with suppliers permitted more frequent orders and more frequent communication about them, acompany could be more effective by using EDI to

Reduce costs by reducing raw materials inventory.A.Ensure that it always maintained a 25-day buffer stock.B.Track materials through production to completed orders.C.Schedule production to reduce the number of setups required.D.

Answer (A) is correct. A well-managed company seeks to minimize the sum of the costs of holding and orderinginventory. An EDI system reduces the costs of ordering, thereby permitting the company to order more frequently.More frequent ordering reduces inventory and the associated holding costs.Answer (B) is incorrect because the company would not need to ensure that it maintained a 25-day buffer stock if itcan order more frequently.Answer (C) is incorrect because tracking materials through production is not an EDI, which is the intercompanyexchange of business information.Answer (D) is incorrect because scheduling production is not an application of EDI.

Gleim's CIA Test Prep: Part III: Business Analysis and Information Technology(1165 questions)

Copyright 2004 Gleim Publications, Inc. Page 451Printed for g j

Page 452: Part three cia_with_ answers

[1157] Gleim #: 10.75 -- Source: CIA 1196 I-11Two major retail companies, both publicly traded and operating in the same geographic area, have recently merged. Thecompanies are approximately the same size and have audit departments. Company A has little EDI experience. Company Bhas invested heavily in information technology and has EDI connections with its major vendors. Which of the following isthe least important risk factor when considering the ability to integrate the two companies’ computer systems?

The number of programmers and systems analysts employed by each company.A.The extent of EDI connections with vendors.B.The compatibility of existing operating systems and database structures.C.The size of company databases and the number of database servers used.D.

Answer (A) is correct. The number of systems personnel employed may reflect differences in operating philosophy(outsourcing vs. in-house development of applications). However, the number of personnel in each company is a lessserious concern than the compatibility of hardware and software.Answer (B) is incorrect because Company A has little EDI experience. Hence, the greater the number of vendors thatmust be connected with Company A, the greater the risk exposure.Answer (C) is incorrect because the difficulty and expense of conversion will be increased if the computer systemshave significant compatibility problems.Answer (D) is incorrect because the greater the complexity of the systems to be integrated, the greater the riskexposure.

[1158] Gleim #: 10.76 -- Source: CIA 1193 III-49

After implementing EDI with suppliers, a company discovered a dramatic increase in the prices it paid the single supplierof some special materials for its primary product line. After consulting with the supplier, the company determined that thesupplier had assumed the risk of not having inventory and raised its prices accordingly since the company was the onlybuyer for the special materials. The best approach for managing inventory in this situation is for the company to

Give the supplier more information about expected use of the materials.A.Demand that the supplier reduce the prices of the materials.B.Find another supplier to replace the one charging higher prices.C.Change its product line so the special materials are no longer needed.D.

Answer (A) is correct. If the company gives the supplier more information about use of the materials, the suppliermay be able to plan its production more effectively. It could then reduce its inventory of the materials and itsinventory costs, thus permitting it to charge a lower price.Answer (B) is incorrect because the company could demand that the supplier reduce the price of the materials, but thesupplier could then decline to supply them.Answer (C) is incorrect because other suppliers may also charge a high price.Answer (D) is incorrect because if the special materials are needed in the primary product line, it is unlikely that thecompany would discontinue it before investigating alternatives, e.g., working with the supplier to help the suppliermanage its inventory.

[1159] Gleim #: 10.77 -- Source: CIA 597 III-51

Electronic data interchange (EDI) offers significant benefits to organizations, but it is not without certain major obstacles.Successful EDI implementation begins with which of the following?

Mapping the work processes and flows that support the organization’s goals.A.Purchasing new hardware for the EDI system.B.Selecting reliable vendors for translation and communication software.C.Standardizing transaction formats and data.D.

Gleim's CIA Test Prep: Part III: Business Analysis and Information Technology(1165 questions)

Copyright 2004 Gleim Publications, Inc. Page 452Printed for g j

Page 453: Part three cia_with_ answers

Answer (A) is correct. Marked benefits arise when EDI is tied to strategic efforts that alter, not mirror, previous practices.Applying EDI to an inefficient process results in continuing to do things the wrong way, only faster. Hence, the initial phaseof EDI implementation includes understanding the organization’s mission and an analysis of its activities as part of anintegrated solution to the organization’s needs.Answer (B) is incorrect because the prerequisite for EDI success is an understanding of the mission of the business and theprocesses and flows that support its goals, followed by cooperation with external partners. Purchasing new hardware is asubsequent step.Answer (C) is incorrect because, before applying EDI technology to the business, EDI must be viewed as part of an overallintegrated solution to organizational requirements.Answer (D) is incorrect because EDI is not a solution by itself. Instead of considering how to transmit and receivetransactions, a company must first analyze the entire process.

[1160] Gleim #: 10.78 -- Source: CIA 1193 III-46

The best approach for minimizing the likelihood of software incompatibilities leading to unintelligible messages is for acompany and its customers to

Acquire their software from the same software vendor.A.Agree to synchronize their updating of EDI-related software.B.Agree to use the same software in the same ways indefinitely.C.Each write their own version of the EDI-related software.D.

Answer (A) is incorrect because the company and its customers may obtain their EDI-related software from the samevendor but still have software incompatibility problems if they do not synchronize their installation of updatedversions.Answer (B) is correct. EDI entails the exchange of common business data converted into standard message formats.Thus, two crucial requirements are that the participants agree on transaction formats and that translation software bedeveloped to convert messages into a form understandable by other companies. Thus, if one company changes itssoftware, its trading partners must also do so.Answer (C) is incorrect because, as business requirements change, it may not be possible to use the same software inthe same ways indefinitely.Answer (D) is incorrect because, even if the company and its customers each write their own versions,synchronization problems will arise from updates.

[1161] Gleim #: 10.79 -- Source: CIA 1193 III-45

Before sending or receiving EDI messages, a company should

Execute a trading partner agreement with each of its customers and suppliers.A.Reduce inventory levels in anticipation of receiving shipments.B.Demand that all its suppliers implement EDI capabilities.C.Evaluate the effectiveness of its use of EDI transmissions.D.

Answer (A) is correct. Before sending or receiving EDI messages, a company should execute a trading partneragreement with its customers and suppliers. All parties should understand their responsibilities, the messages eachwill initiate, how they will interpret messages, the means of authenticating and verifying the completeness andaccuracy of messages, the moment when the contract between the parties is effective, the required level of security,etc.Answer (B) is incorrect because the company may intend to reduce inventory levels, but that intention is unrelated tothe timing of its first EDI messages.Answer (C) is incorrect because the company may want to demand or encourage all its customers and suppliers toimplement EDI capabilities, but that request is independent of sending and receiving messages.Answer (D) is incorrect because it is not possible to evaluate the effectiveness of EDI transmissions until after theyoccur.

[1162] Gleim #: 10.80 -- Source: CIA 1193 IV-25

The company uses a planning system that focuses first on the amount and timing of finished goods demanded and thendetermines the derived demand for raw materials, components, and subassemblies at each of the prior stages of production.This system is referred to as

Economic order quantity.A.Materials requirements planning.B.Linear programming.C.Just-in-time purchasing.D.

Gleim's CIA Test Prep: Part III: Business Analysis and Information Technology(1165 questions)

Copyright 2004 Gleim Publications, Inc. Page 453Printed for g j

Page 454: Part three cia_with_ answers

Answer (A) is incorrect because the EOQ model focuses on the trade-off between carrying and ordering costs.Answer (B) is correct. Materials requirements planning (MRP) is a system that translates a production schedule intorequirements for each component needed to meet the schedule. It is usually implemented in the form of a computer-basedinformation system designed to plan and control raw materials used in production. It assumes that forecasted demand isreasonably accurate and that suppliers can deliver based upon this accurate schedule. MRP is a centralized push-throughsystem; output based on forecasted demand is pushed through to the next department or to inventory.Answer (C) is incorrect because linear programming is a decision model concerned with allocating scarce resources tomaximize profit or minimize costs.Answer (D) is incorrect because JIT is a decentralized demand-pull system. It is driven by actual demand.

[1163] Gleim #: 10.81 -- Source: Publisher

Enterprise resource planning (ERP) software packages, such as SAP and Oracle, are all-inclusive systems that attempt toprovide entity-wide information. ERP systems provide advantages to the auditor because they

Have proven difficult for some firms to install.A.Typically require firms to reduce the division of duties and responsibilities found in traditional systems.B.Typically have built-in transaction logs and ability to produce a variety of diagnostic reports.C.Have been installed by smaller firms so, to date, few auditors have encountered them.D.

Answer (A) is incorrect because the difficulty of installing ERP systems is a disadvantage to the auditor.Answer (B) is incorrect because ERP systems often require the client to depart from the traditional functionaldivision of duties such as accounting, finance, and marketing. The result is increased audit risk.Answer (C) is correct. ERP systems have a variety of controls and report generation functions that allow the auditorto abstract and monitor data collected and processed. Some ERP systems have built-in audit functions.Answer (D) is incorrect because ERP systems are very costly and therefore have been implemented only by largeorganizations. However, the trend is for more and more organizations to install these systems.

[1164] Gleim #: 10.82 -- Source: CIA 596 III-73

Use of unlicensed software in an organization

Increases the risk of introducing viruses into the organizationI.Is not a serious exposure if only low-cost software is involvedII.Can be detected by software checking routines that run from a network serverIII.

I only.A.I and II only.B.I, II, and III.C.I and III only.D.

Answer (A) is incorrect because use of unlicensed software increases the risk of viral infection, and its use can bedetected by software checking routines. Moreover, the cost of the software is not relevant. Any software may containa virus.Answer (B) is incorrect because use of unlicensed software increases the risk of viral infection, and its use can bedetected by software checking routines. Moreover, the cost of the software is not relevant. Any software may containa virus.Answer (C) is incorrect because use of unlicensed software increases the risk of viral infection, and its use can bedetected by software checking routines. Moreover, the cost of the software is not relevant. Any software may containa virus.Answer (D) is correct. Antivirus measures should include strict adherence to software acquisition policies.Unlicensed software is less likely to have come from reputable vendors and to have been carefully tested. Specialsoftware is available to test software in use to determine whether it has authorized.

[1165] Gleim #: 10.83 -- Source: CIA 593 II-20

Which of the following is the best program for the protection of a company’s vital information resources from computerviruses?

Stringent corporate hiring policies for staff working with computerized functions.A.Existence of a software program for virus prevention.B.Prudent management policies and procedures instituted in conjunction with technological safeguards.C.Physical protection devices in use for hardware, software, and library facilities.D.

Gleim's CIA Test Prep: Part III: Business Analysis and Information Technology(1165 questions)

Copyright 2004 Gleim Publications, Inc. Page 454Printed for g j

Page 455: Part three cia_with_ answers

Answer (A) is incorrect because hiring policies can provide assurance of qualified personnel for operation of the system, butthey cannot prevent introduction of viruses from bulletin boards or from outside sources.Answer (B) is incorrect because software programs can identify and neutralize known viruses but may not recognize andproperly neutralize new strains of a computer virus.Answer (C) is correct. Acceptably safe computing can be achieved by carefully crafted policies and procedures used inconjunction with antivirus and access control software.Answer (D) is incorrect because physical protection devices can reduce access but cannot prevent introduction of viruses byerrant employees or from outside sources.

Gleim's CIA Test Prep: Part III: Business Analysis and Information Technology(1165 questions)

Copyright 2004 Gleim Publications, Inc. Page 455Printed for g j